Top-Rated Free Essay
Preview

Economic Test Bank

Satisfactory Essays
392876 Words
Grammar
Grammar
Plagiarism
Plagiarism
Writing
Writing
Score
Score
Economic Test Bank
Test Bank 1 to accompany

Economics
Sixteenth Edition

Campbell R. McConnell University of Nebraska Stanley L. Brue Pacific Lutheran University

Prepared by Stanley L. Brue Pacific Lutheran University

Test Bank 1 to accompany ECONOMICS Campbell R. McConnell and Stanley L. Brue Published by McGraw-Hill, an imprint of The McGraw-Hill Companies, Inc., 1221 Avenue of the Americas, New York, NY 10020. Copyright  2005 by The McGraw-Hill Companies, Inc. All rights reserved. The contents, or parts thereof, may be reproduced in print form solely for classroom use with ECONOMICS provided such reproductions bear copyright notice, but may not be reproduced in any other form or for any other purpose without the prior written consent of The McGraw-Hill Companies, Inc., including, but not limited to, in any network or other electronic storage or transmission, or broadcast for distance learning.

1 2 3 4 5 6 7 8 9 0 Text printer code/Cover printer code 0 9 8 7 6 5 4 ISBN 0-07-288479-7

www.mhhe.com

Table of Contents

1 2 3 3W 4 5 6 7 8 9 10 11 12 13 14 15 16 17 18 19 20 21 22 23 24 25 26 27 28 29 30 31 32 33 34 35 36 37 38 39W 40W

The Nature and Method of Economics ................................................................ 1 The Economizing Problem................................................................................. 31 Individual Markets: Demand and Supply........................................................... 69 Applications and Extensions of Supply and Demand Analysis ....................... 109 The Market System .......................................................................................... 125 The U.S. Economy: Private and Public Sectors ............................................... 141 The United States in the Global Economy ....................................................... 175 Measuring Domestic Output and National Income.......................................... 197 Introduction to Economic Growth and Instability............................................ 229 Basic Macroeconomic Relationships ............................................................... 253 The Aggregate Expenditures Model ................................................................ 289 Aggregate Demand and Aggregate Supply...................................................... 335 Fiscal Policy ..................................................................................................... 369 Money and Banking ......................................................................................... 403 How Banks and Thrifts Create Money............................................................. 433 Monetary Policy ............................................................................................... 455 Extending the Analysis of Aggregate Supply .................................................. 489 Economic Growth ............................................................................................ 515 Deficits, Surpluses, and the Public Debt .......................................................... 531 Disputes over Macro Theory and Policy.......................................................... 547 Elasticity of Demand and Supply..................................................................... 575 Consumer Behavior and Utility Maximization ................................................ 611 The Costs of Production................................................................................... 647 Pure Competition.............................................................................................. 687 Pure Monopoly................................................................................................. 731 Monopolistic Competition and Oligopoly........................................................ 773 Technology, R&D, and Efficiency................................................................... 819 The Demand for Resources .............................................................................. 843 Wage Determination ........................................................................................ 875 Rent, Interest, and Profit .................................................................................. 909 Government and Market Failure ...................................................................... 929 Public Choice Theory and Taxation................................................................. 953 Antitrust Policy and Regulation ....................................................................... 979 Agriculture: Economics and Policy................................................................ 1001 Income Inequality and Poverty ...................................................................... 1021 Labor Market Institutions and Issues: Unionism, Discrimination, and Immigration ................................................................... 1047 The Economics of Health Care ...................................................................... 1077 International Trade ......................................................................................... 1097 Exchange Rates, the Balance of Payments, and Trade Deficits..................... 1127 The Economics of Developing Countries (Internet-Only Chapter) ............... 1157 Transition Economies: Russia and China (Internet-Only Chapter) ............... 1177

Preface Test Bank I contains more than 6,300 multiple-choice and true-false questions. We have created approximately 670 new question for this edition; the remaining 5,630 have been checked and refined to ensure compatibility with the 16th Edition. Test Bank I continues to be accurate, functional, and user-friendly. 1. New questions: All new question are so identified. This will provide long-time users of Economics with a fresh set of questions. Grouping by topic: As in previous editions, all questions are grouped by topic and sequenced in the same order as the text material. A table at the beginning of each Test Bank chapter lists the topics and corresponding question numbers. Each topic has been numbered for easy reference. This sequencing should reduce the time required to make up well-balanced exams. Consider This and Last Word questions: Test Bank I contains and identifies two or three questions on each Consider This and Last Word vignette. Coding: Each item contains a code that identifies the particular type of question. For example, A=application of concept; D=definition; G=graphical analysis, and so forth. These codes are identified following this Preface, while the coding itself is found above each question. Former users of Test Bank I have found these codes useful in preparing exams and in identifying the types of questions missed by students. Page references: Each question includes a 16th Edition page reference. To aid instructors who are using the paperback split volumes, we list three separate page numbers: E indicates the Economics page number; MA indicates the Macroeconomics split volume page number; and MI indicates the Microeconomics split volume page number.

2.

3.

4.

5.

All Test Bank I questions are also available in a Brownstone test generating system for PCs and Macintosh computers. Contact your McGraw-Hill/Irwin sales representative for more information. We wish to thank Robert Jensen for assistance and Becky Szura for her expertise in creating the Brownstone Diploma version. Stanley L Brue Campbell R. McConnell

Conversion Chart
There are three versions of the 16th edition of McConnell/Brue Economics: Macroeconomics, Microeconomics, and Economics. Test Bank I follows the Economics chapter order. Use the following chart to see how Test Bank I/Economics chapters correspond with the other two books:

Economics/TB I Chapter 1-6 7-19 20-36 37-38 3W 39W 40W

Macroeconomics Chapter 1-6 7-19 *** 20-21 3W 22W 23W

Microeconomics Chapter 1-6 *** 7-23 24-25 3W *** 26W

Bold type indicates chapters common to all three books. *** indicates chapters that do not appear in a particular split version of the text. W indicates chapters that are available only at the book’s website, www.mcconnell16.com.

Coding System for Test Bank I
The codes above each question indicate the following question types: A = Application of Concept A question that tests student understanding of a definition or concept. C = Complex Analysis A question that calls for students to apply more than one concept in order to reach the correct answer. D = Definition A straightforward question that tests recognition of a definition. E = Equation A question that calls for solving one or more equations. Many of these are designated as advanced analysis questions. F = Fact A problem that tests ability to recall a fact or some data. G = Graphical A question that requires students to demonstrate working facility with graphs. T = Table A question that requires students to interpret or manipulate tabular material. New = New Question A question that is new to this edition of Test Bank I. E = Economics, 16th Edition MA = Macroeconomics, 16th Edition MI = Microeconomics, 16th Edition

CHAPTER 1

The Nature and Method of Economics

Topic 1. 2. 3. 4. 5. 6. 7. Economics; economic perspective Benefits of studying economics Methodology Economic policy and goals Macroeconomics and microeconomics Positive and normative statements Logical pitfalls Consider This Last Word True-False

Question numbers 1-20 21-23 24-40 41-47 48-56 57-66 67-82 83-84 85-87 88-100

____________________________________________________________

_______________________________________

Appendix 8. Construction and interpretation of graphs

101-155

____________________________________________________________

_______________________________________

Multiple Choice Questions Economics; economic perspective

Type: D Topic: 1 E: 4 MI: 4 MA: 4 Status: New 1. For economists, the word "utility" means: A) versatility and flexibility. B) rationality. C) pleasure and satisfaction. D) purposefulness. Answer: C

Type: D Topic: 1 E: 4 MI: 4 MA: 4 Status: New 2. In economics, the pleasure, happiness, or satisfaction received from a product is called: A) marginal cost. B) rational outcome. C) status fulfillment. D) utility. Answer: D

Type: A Topic: 1 E: 4 MI: 4 MA: 4 Status: New 3. When economists say that people act rationally in their self interest, they mean that individuals: A) look for and pursue opportunities to increase their utility. B) generally disregard the interests of others. C) are mainly creatures of habit. D) are unpredictable. Answer: A

Chapter 1: The Nature and Method of Economics

Type: A Topic: 1 E: 3 MI: 3 MA: 3 4. According to Emerson: "Want is a growing giant whom the coat of Have was never large enough to cover." According to economists, "Want" exceeds "Have" because: A) people are greedy. C) human beings are inherently insecure. B) productive resources are limited. D) people are irrational. Answer: B

Type: A Topic: 1 E: 4 MI: 4 MA: 4 5. According to economists, economic self-interest: A) is a reality that underlies economic behavior. B) has the same meaning as selfishness. Answer: A

C) is more characteristic of men than of women. D) is usually self-defeating.

Type: A Topic: 1 E: 4 MI: 4 MA: 4 6. When entering a building, Sam diverts his path to go through an open door rather than make the physical effort to open the closed door that is directly in his path. This is an example of: A) irrational behavior. C) marginal benefit-marginal cost analysis. B) a lazy person. D) programmed learning. Answer: C

Type: A Topic: 1 E: 4 MI: 4 MA: 4 7. Joe sold gold coins for $1000 that he bought a year ago for $1000. He says, "At least I didn't lose any money on my financial investment." His economist friend points out that in effect he did lose money, because he could have received a 3 percent return on the $1000 if he had bought a bank certificate of deposit instead of the coins. The economist's analysis in this case incorporates the idea of: A) opportunity costs C) imperfect information. B) marginal benefits that exceed marginal costs. D) normative economics. Answer: A

Type: A Topic: 1 E: 4-5 MI: 4-5 MA: 4-5 8. There is too little of a good thing when its marginal: A) benefit exceeds its marginal cost. C) cost equals its marginal benefit. B) cost exceeds its marginal benefit. D) benefit is still positive. Answer: A

Type: D Topic: 1 E: 3 MI: 3 MA: 3 9. Economics may best be defined as the: A) interaction between macro and micro considerations. B) social science concerned with the efficient use of scarce resources to achieve maximum satisfaction of economic wants. C) empirical testing of value judgments through the use of logic. D) use of policy to refute facts and hypotheses. Answer: B

McConnell/Brue: Economics, 16/e

Page 2

Chapter 1: The Nature and Method of Economics

Type: D Topic: 1 E: 3 MI: 3 MA: 3 10. The study of economics is primarily concerned with: A) keeping private businesses from losing money. B) demonstrating that capitalistic economies are superior to socialistic economies. C) choices that are made in seeking to use scarce resources efficiently. D) determining the most equitable distribution of society's output. Answer: C

Type: D Topic: 1 E: 4 MI: 4 MA: 4 11. The economic perspective refers to: A) macroeconomic phenomena, but not microeconomic phenomena. B) microeconomic phenomena, but not macroeconomic phenomena. C) the making of rational decisions in a context of marginal costs and marginal benefits. D) unlimited resources in a context of limited economic wants. Answer: C

Type: D Topic: 1 E: 4 MI: 4 MA: 4 12. The economic perspective entails: A) irrational behavior by individuals and institutions. B) a comparison of marginal benefits and marginal costs in decision making. C) short-term but not long-term thinking. D) rejection of the scientific method. Answer: B

Type: D Topic: 1 E: 4 MI: 4 MA: 4 13. Rational behavior suggests that: A) everyone will make identical choices. B) resource availability exceeds economic wants. C) individuals will make different choices because their preferences and circumstances differ. D) an individual's economic goals cannot involve tradeoffs. Answer: C

Type: D Topic: 1 E: 4 MI: 4 MA: 4 14. Economics involves marginal analysis because: A) most decisions involve changes from the present situation. B) marginal benefits always exceed marginal costs. C) marginal costs always exceed marginal benefits. D) much economic behavior is irrational. Answer: A

Type: A Topic: 1 E: 5 MI: 5 MA: 5 15. You should decide to go to a movie: A) if the marginal cost of the movie exceeds its marginal benefit. B) if the marginal benefit of the movie exceeds its marginal cost. C) if your income will allow you to buy a ticket. D) because movies are enjoyable. Answer: B

McConnell/Brue: Economics, 16/e

Page 3

Chapter 1: The Nature and Method of Economics

Type: D Topic: 1 E: 5 MI: 5 MA: 5 16. Marginal costs exist because: A) the decision to produce more of some product means the sacrifice of other products. B) wants are scarce relative to resources. C) households and businesses make rational decisions. D) most decisions do not involve sacrifices or tradeoffs. Answer: A

Type: D Topic: 1 E: 3 MI: 3 MA: 3 17. The assertion that "There is no free lunch" means that: A) there are always tradeoffs between economic goals. B) all production involves the use of scarce resources and thus the sacrifice of alternative goods. C) marginal analysis is not used in economic reasoning. D) choices need not be made if behavior is rational. Answer: B

Type: A Topic: 1 E: 4 MI: 4 MA: 4 18. Consumers spend their incomes to get the maximum benefit or satisfaction from the goods and services they purchase. This is a reflection of: A) resource scarcity and the necessity of choice. B) rational behavior. C) marginal costs that exceed marginal benefits. D) the tradeoff problem that exists between competing goals. Answer: B

Type: A Topic: 1 E: 5 MI: 5 MA: 5 19. There can be too much of a good thing. This statement suggests that: A) rational choice cannot be applied to many economic decisions. B) a good may be produced to the point where its marginal cost exceeds its marginal benefit. C) certain goods and services such as education and health care are inherently desirable and should be produced regardless of costs and benefits. D) a good may be produced to the point where its marginal benefit exceeds its marginal cost. Answer: B

Type: A Topic: 1 E: 5 MI: 5 MA: 5 20. Even though local newspapers are very inexpensive, people rarely buy more than one of them each day. This fact: A) is an example of irrational behavior. B) implies that reading should be taught through phonics rather than the whole language method. C) contradicts the economic perspective. D) implies that, for most people, the marginal benefit of reading a second newspaper is less than the marginal cost. Answer: D

McConnell/Brue: Economics, 16/e

Page 4

Chapter 1: The Nature and Method of Economics

Benefits of studying economics

Type: A Topic: 2 E: 5 MI: 5 MA: 5 21. Studying economics: A) helps one become a better-informed citizen and voter. B) is detrimental to good citizenship because economics emphasizes individualism. C) is a waste of time since we all participate in the economy whether we understand it or not. D) is important because economics is the science of earning money. Answer: A

Type: A Topic: 2 E: 5 MI: 5 MA: 5 22. Learning economics: A) is detrimental to good citizenship because economics emphasizes individualism. B) is helpful to employers, but not to workers and consumers. C) is important because economics is the science of earning money. D) helps students improve analytical skills that are in great demand in the workplace. Answer: D

Type: A Topic: 2 E: 6 MI: 6 MA: 6 23. The study of economics: A) is similar to management, marketing, and finance since it emphasizes how to make money. B) is helpful to businesses, but not particularly helpful in making personal buying decisions. C) is mainly an academic, not a vocational subject. D) looks at the economy from the viewpoint of one's own pocketbook, not from the standpoint of society's interest. Answer: C

Methodology

Type: D Topic: 3 E: 6 MI: 6 MA: 6 24. An economic hypothesis: A) has the same meaning as an economic principal or economic law. B) is usually a normative statement. C) is a possible explanation of cause and effect. D) is a stronger generalization than an economic law. Answer: C

Type: A Topic: 3 E: 6 MI: 6 MA: 6 25. Which of the following terms implies the least degree of confidence in an economic generalization? A) a hypothesis B) a theory C) a principle D) a law Answer: A

Type: A Topic: 3 E: 7 MI: 7 MA: 7 26. Which of the following terms implies the greatest degree of confidence in an economic generalization? A) a hypothesis B) a theory C) a principle D) an anomaly Answer: C

McConnell/Brue: Economics, 16/e

Page 5

Chapter 1: The Nature and Method of Economics

Type: D Topic: 3 E: 7 MI: 7 MA: 7 27. A well-tested economic theory is often called: A) a hypothesis. B) a prototype. C) a principle. D) an anomaly. Answer: C

Type: C Topic: 3 E: 7 MI: 7 MA: 7 28. Economists: A) use both the economic perspective and the scientific method. B) use the economic perspective but not the scientific method. C) make positive economic statements, but not normative economic statements. D) reject theorizing as being impractical. Answer: A

Type: A Topic: 3 E: 6 MI: 6 MA: 6 29. The scientific method is: A) not applicable to economics, because economics deals with human beings. B) also known as the economic perspective. C) analysis that moves from broad generalizations called laws to theories and then to hypotheses. D) used by economists and other social scientists, as well as by physical scientists and life scientists. Answer: D

Type: D Topic: 3 E: 7 MI: 7 MA: 7 30. The process by which economists test hypotheses against facts to develop theories, principles, and models is called: A) the economic perspective. B) the scientific method. C) policy economics. D) microeconomcis. Answer: B

Type: D Topic: 3 E: 6 MI: 6 MA: 6 31. Economic theories: A) are useless because they are not based on laboratory experimentation. B) that are true for individual economic units are never true for the economy as a whole. C) are generalizations based on a careful observation of facts. D) are abstractions and therefore of no application to real situations. Answer: C

Type: A Topic: 3 E: 6 MI: 6 MA: 6 32. Theoretical economics: A) is also known as policy economics. B) is the process of deriving principles of economics. C) is highly impractical since it does not deal with the real world. D) rejects the scientific method as being inappropriate for the social sciences. Answer: B

McConnell/Brue: Economics, 16/e

Page 6

Chapter 1: The Nature and Method of Economics

Type: D Topic: 3 E: 7 MI: 7 MA: 7 33. Which of the following is a correct statement? A) Economic concepts or laws that are valid during depression are necessarily valid during prosperity. B) Though not quantitatively exact, economic laws are useful because they allow us to predict and therefore control or adjust to events. C) Economics is as scientific as are physics and chemistry because economic laws are as quantitatively precise as the laws of physics or chemistry. D) Because economics is concerned with questions of "ought," it is a branch of applied ethics and not scientific. Answer: B

Type: D Topic: 3 E: 7 MI: 7 MA: 7 34. In constructing models, economists: A) make simplifying assumptions. B) include all available information. Answer: A

C) must use mathematical equations. D) attempt to duplicate the real world.

Type: D Topic: 3 E: 7 MI: 7 MA: 7 35. Economic models: A) are of limited use because they cannot be tested empirically. B) are limited to variables that are directly related to one another. C) emphasize basic economic relationships by abstracting from the complexities of the real world. D) are unrealistic and therefore of no practical consequence. Answer: C

Type: D Topic: 3 E: 7 MI: 7 MA: 7 36. The term "ceteris paribus" means: A) that if event A precedes event B, A has caused B. B) that economics deals with facts, not values. C) other things equal. D) prosperity inevitably follows recession. Answer: C

Type: D Topic: 3 E: 7 MI: 7 MA: 7 37. The basic purpose of the other-things-equal assumption is to: A) allow one to reason about the relationship between variables X and Y without the intrusion of variable Z. B) allow one to focus upon micro variables by ignoring macro variables. C) allow one to focus upon macro variables by ignoring micro variables. D) determine whether X causes Y or vice versa. Answer: A

McConnell/Brue: Economics, 16/e

Page 7

Chapter 1: The Nature and Method of Economics

Type: A Topic: 3 E: 7 MI: 7 MA: 7 38. Suppose an economist says that "Other things equal, the lower the price of bananas, the greater the amount of bananas purchased." This statement indicates that: A) the quantity of bananas purchased determines the price of bananas. B) all factors other than the price of bananas (for example, consumer tastes and incomes) are assumed to be constant. C) economists can conduct controlled laboratory experiments. D) one cannot generalize about the relationship between the price of bananas and the quantity purchased. Answer: B

Type: D Topic: 3 E: 6 MI: 6 MA: 6 39. An hypothesis is: A) a fundamental truth that all economists accept. B) a tentative, untested statement of possible cause and effect. C) the same as a normative statement. D) also known as a principle or law. Answer: B

Type: D Topic: 3 E: 8 MI: 8 MA: 8 40. The term "other things equal" means that: A) the associated statement is normative. B) many variables affect the variable under consideration. C) a number of relevant variables are assumed to be constant. D) when variable X increases so does related variable Y. Answer: C

Economic policy and goals

Type: F Topic: 4 E: 8 MI: 8 MA: 8 Status: New 41. Which of the following statements is true? A) The process of deriving economic theories and principles is known as policy economics. B) Full employment, price-level stability, and economic growth are widely accepted macroeconomics goals in the United States. C) Normative economics deals with "what is," whereas positive economics deals with "what ought to be." D) There can be too little of a good thing, but never too much of a good thing. Answer: B

Type: A Topic: 4 E: 8 MI: 8 MA: 8 42. In formulating economic policy it is important to: A) use only positive economics, not normative economics. B) consider the benefits and costs of the various policy options for reaching desired goals. C) avoid outcomes for which marginal benefits exceed marginal costs. D) use only normative economics, not positive economics. Answer: B

McConnell/Brue: Economics, 16/e

Page 8

Chapter 1: The Nature and Method of Economics

Type: D Topic: 4 E: 9 MI: 9 MA: 9 43. If there is a tradeoff between goals A and B: A) greater fulfillment of A means lesser fulfillment of B. B) goals A and B are positively correlated. C) causation exists between the two goals. D) greater fulfillment of A means greater fulfillment of B. Answer: A

Type: A Topic: 4 E: 9 MI: 9 MA: 9 44. The achieving of complete economic freedom is most likely to conflict with the goal of: A) price-level stability. C) an equitable distribution of income. B) economic growth. D) economic efficiency. Answer: C

Type: A Topic: 4 E: 9 MI: 9 MA: 9 45. If a reduction in the unemployment rate is always accompanied by a reduction in income inequality, then: A) full employment and greater income equality are conflicting goals. B) full employment and greater income equality are compatible goals. C) reduced income inequality is a more important goal than is achieving full employment. D) achieving full employment is a more important goal than reducing income inequality. Answer: B

Type: A Topic: 4 E: 10 MI: 10 MA: 10 46. Most of the disagreement among economists involves: A) the use of statistics. B) the gathering of facts. C) generalizing upon facts. Answer: D

D) policy economics.

Type: A Topic: 4 E: 9 MI: 9 MA: 9 47. With respect to the basic economic goals of society, for example, economic growth, full employment, economic efficiency, price level stability, economic freedom, distributional equity, and economic security, it can be said that: A) all are accepted and assigned similar priorities in every industrialized society. B) all are precisely measurable. C) some goals are complementary and others are conflicting. D) all are conflicting goals. Answer: C

Macroeconomics and microeconomics

Type: D Topic: 5 E: 9 MI: 9 MA: 9 48. Macroeconomics approaches the study of economics from the viewpoint of: A) the entire economy. B) governmental units. C) the operation of specific product and resource markets. D) individual firms. Answer: A

McConnell/Brue: Economics, 16/e

Page 9

Chapter 1: The Nature and Method of Economics

Type: A Topic: 5 E: 9 MI: 9 MA: 9 49. Which of the following is associated with macroeconomics? A) an examination of the incomes of Harvard Business School graduates B) an empirical investigation of the general price level and unemployment rates since 1990 C) a study of the trend of pecan prices since the Second World War D) a case study of pricing and production in the textbook industry Answer: B

Type: A Topic: 5 E: 9 MI: 9 MA: 9 50. The problems of aggregate inflation and unemployment are: A) major topics of macroeconomics. C) major topics of microeconomics. B) not relevant to the U.S. economy. D) peculiar to command economies. Answer: A

Type: A Topic: 5 E: 9 MI: 9 MA: 9 51. Which of the following statements pertains to macroeconomics? A) Because the minimum wage was raised, Mrs. Olsen decided to enter the labor force. B) A decline in the price of soybeans caused farmer Wanek to plant more land in wheat. C) The national productivity rate grew by 2.7 percent last year. D) The Pumpkin Center State Bank increased its interest rate on consumer loans by 1 percentage point. Answer: C

Type: D Topic: 5 E: 9 MI: 9 MA: 9 52. Macroeconomics can best be described as the: A) analysis of how a consumer tries to spend income. B) study of the large aggregates of the economy or the economy as a whole. C) analysis of how firms attempt to maximize their profits. D) study of how supply and demand determine prices in individual markets. Answer: B

Type: D Topic: 5 E: 9 MI: 9 MA: 9 53. Microeconomics is concerned with: A) the aggregate or total levels of income, employment, and output. B) a detailed examination of specific economic units that make up the economic system. C) positive economics, but not normative economics. D) the establishing of an overall view of the operation of the economic system. Answer: B

Type: D Topic: 5 E: 9 MI: 9 MA: 9 54. Microeconomics: A) is the basis for the "after this, therefore because of this" fallacy. B) is not concerned with details, but only with the overall big picture of the economy. C) is concerned with individual economic units and specific markets. D) describes the aggregate flows of output and income. Answer: C

McConnell/Brue: Economics, 16/e

Page 10

Chapter 1: The Nature and Method of Economics

Type: A Topic: 5 E: 9 MI: 9 MA: 9 55. Which of the following is a macroeconomic statement? A) The gross profits of all U.S. businesses were $182 billion last year. B) The price of beef declined by 3 percent last year. C) General Motors' profits increased last year. D) The productivity of steelworkers increased by 1 percent last year. Answer: A

Type: A Topic: 5 E: 9 MI: 9 MA: 9 56. Which of the following is a microeconomic statement? A) The real domestic output increased by 2.5 percent last year. B) Unemployment was 6.8 percent of the labor force last year. C) The price of personal computers declined last year. D) The general price level increased by 4 percent last year. Answer: C

Positive and normative statements

Type: D Topic: 6 E: 10 MI: 10 MA: 10 57. A normative statement is one that: A) is based on the law of averages. B) applies only to microeconomics. Answer: D

C) applies only to macroeconomics. D) is based on value judgments.

Type: A Topic: 6 E: 10 MI: 10 MA: 10 58. Which of the following is a normative statement? A) The temperature is high today. B) The humidity is high today. Answer: C

C) It is too hot to play tennis today. D) It will cool off later this evening.

Type: D Topic: 6 E: 10 MI: 10 MA: 10 59. A positive statement is one which is: A) derived by induction. B) derived by deduction. Answer: D

C) subjective and is based on a value judgment. D) objective and is based on facts.

Type: A Topic: 6 E: 10 MI: 10 MA: 10 60. Which of the following is a positive statement? A) The humidity is too high today. B) It is too hot to jog today. Answer: C

C) The temperature is 92 degrees today. D) I enjoy summer evenings when it cools off.

Type: D Topic: 6 E: 10 MI: 10 MA: 10 61. Normative statements are concerned with: A) facts and theories. B) what ought to be. Answer: B

C) what is. D) rational choice involving costs and benefits.

McConnell/Brue: Economics, 16/e

Page 11

Chapter 1: The Nature and Method of Economics

Type: D Topic: 6 E: 10 MI: 10 MA: 10 62. A positive statement is concerned with: A) some goal that is desirable to society. B) what should be. Answer: C

C) what is. D) the formulation of economic policy.

Type: A Topic: 6 E: 10 MI: 10 MA: 10 63. Ideally, value judgments are involved at the: A) levels of facts, theory, and policy. B) levels of facts and theory only. Answer: D

C) level of facts only. D) level of policy only.

Type: A Topic: 6 E: 10 MI: 10 MA: 10 64. Most of the disagreement among economists involves: A) facts. B) principles. C) positive statements. D) normative statements. Answer: D

Type: A Topic: 6 E: 10 MI: 10 MA: 10 65. "Economics is concerned with using scarce productive resources efficiently in attempting to satisfy society's economic wants." This statement is: A) positive, but incorrect. C) normative, but incorrect. B) positive and correct. D) normative and correct. Answer: B

Type: A Topic: 6 E: 10 MI: 10 MA: 10 66. Ben says that "An increase in the tax on beer will raise its price." Holly argues that "Taxes should be increased on beer because college students drink too much." We can conclude that: A) Ben's statement is normative, but Holly's is positive. B) Holly's statement is normative, but Ben's is positive. C) Both statements are normative. D) Both statements are positive. Answer: B

Logical pitfalls

Type: D Topic: 7 E: 11 MI: 11 MA: 11 67. The fallacy of composition states that: A) because economic systems are composed of so many diverse economic units, economic laws are necessarily inexact. B) the anticipation of a particular event can affect the composition of that event when it occurs. C) what is true for the individual must necessarily be true for the group. D) because event A precedes event B, A is necessarily the cause of B. Answer: C

McConnell/Brue: Economics, 16/e

Page 12

Chapter 1: The Nature and Method of Economics

Type: D Topic: 7 E: 11 MI: 11 MA: 11 68. The "after this, because of this" fallacy states that: A) because event A precedes event B, A is necessarily the cause of B. B) the very attempt to accomplish a certain objective may create conditions that prohibit the achievement of that goal. C) events may drastically alter plans; one's intentions and actual accomplishments may differ considerably. D) generalizations that are accurate at the level of microeconomics may be inaccurate at the level of macroeconomics. Answer: A

Type: A Topic: 7 E: 11 MI: 11 MA: 11 69. The safest way for an individual to leave a burning theater is to run for the nearest exit; it is therefore also the best means of escape for a large audience. This assertion illustrates the: A) "after this, therefore because of this" fallacy. C) fallacy of composition. B) correlation fallacy. D) fallacy of limited decisions. Answer: C

Type: D Topic: 7 E: 11 MI: 11 MA: 11 70. Which of the following has to do with the idea that generalizations that apply to individuals are also always valid for a group? A) the law of large numbers C) the fallacy of composition B) the law of averages D) the post hoc, ergo propter hoc fallacy Answer: C

Type: D Topic: 7 E: 11 MI: 11 MA: 11 71. The fallacy of composition states that: A) generalizations relevant to microeconomics never apply to macroeconomics. B) expectations give rise to self-fulfilling prophesies. C) generalizations pertaining to individuals always apply to the group. D) quantifiable economic goals are always incompatible with one another. Answer: C

Type: D Topic: 7 E: 11 MI: 11 MA: 11 72. The "after this, because of this" fallacy states that: A) positive statements are always followed by normative judgments. B) positive statements can never be proven true or false. C) if one acts on one's expectations, those expectations will always be fulfilled. D) cause and effect can be determined merely by observing the sequence of events. Answer: D

Type: A Topic: 7 E: 11 MI: 11 MA: 11 73. If you leave a football game at the end of the third quarter, you will avoid traffic and get home more quickly. Therefore, everyone should leave the game early. This assertion illustrates the: A) moral hazard problem. C) fallacy of limited decisions. B) adverse selection problem. D) fallacy of composition. Answer: D

McConnell/Brue: Economics, 16/e

Page 13

Chapter 1: The Nature and Method of Economics

Type: D Topic: 7 E: 11 MI: 11 MA: 11 74. Which of the following has to do with the problem of distinguishing cause and effect in economic reasoning? A) the law of large numbers C) the post hoc, ergo propter hoc fallacy B) the law of averages D) the fallacy of composition Answer: C

Type: D Topic: 7 E: 11 MI: 11 MA: 11 75. Which of the following best illustrates the post hoc, ergo propter hoc fallacy? A) Because it was 90 degrees today, I worked up a sweat playing tennis. B) I took the day off work to go to the beach and that's why it rained. C) Because it rained at the football game, my new sweater got wet. D) Because I have studied diligently this semester, my grade average has improved. Answer: B

Type: D Topic: 7 E: 11 MI: 11 MA: 11 76. The fallacy of composition is essentially the error of: A) omitting relevant variables in constructing a model. B) reasoning from the general to the particular. C) confusing cause and effect in economic relationships. D) generalizing from the particular to the general. Answer: D

Type: D Topic: 7 E: 11 MI: 11 MA: 11 77. The post hoc fallacy and the correlation problem both relate to: A) the calculation of marginal costs and marginal benefits of any economic activity. B) the issue of determining causation. C) the frequent inability of households and businesses to behave rationally. D) the tradeoff problem associated with competing goals. Answer: B

Type: A Topic: 7 E: 11 MI: 11 MA: 11 78. If variables X and Y are positively correlated, this means that: A) X is the cause of Y . B) Y is the cause of X. C) causation necessarily exists, but we don't know whether X or Y is the cause. D) causation may or may not exist between X and Y . Answer: D

Type: A Topic: 7 E: 11 MI: 11 MA: 11 Status: New 79. "The government deregulated the electricity industry in California and a shortage of electricity soon occurred. It is clear that the deregulation caused the shortage." This statement needs careful analysis because it may reflect the: A) the fallacy of composition. C) use of loaded terminology. B) post hoc, ergo propter hoc fallacy. D) the law of averages. Answer: B

McConnell/Brue: Economics, 16/e

Page 14

Chapter 1: The Nature and Method of Economics

Type: A Topic: 7 E: 10 MI: 10 MA: 10 Status: New 80. A caller to a radio talk show states that multinational corporations are "greedy exploiters of the poor." This is an example of: A) loaded terminology. B) the "after this, therefore because of this fallacy." C) the fallacy of composition. D) the economic perspective. Answer: A

Type: A Topic: 7 E: 10 MI: 10 MA: 10 Status: New 81. A caller to a radio talk show states that protesters against globalization are a collection of "anarchist punks, naïve college students, and trade union radicals." This is an example of: A) the fallacy of composition. C) loaded terminology. B) the economic perspective. D) marginal analysis. Answer: C

Type: F Topic: 7 E: 11 MI: 11 MA: 11 Status: New 82. A study found that the incidence of skin cancer increases along with the amount of time people work under fluorescent light, leading some people to conclude that fluorescent lighting is a cause of skin cancer. But further analysis found that people who work in offices, where fluorescent light is common, suffer more sunburn on their vacations than other workers. The sunburns, not the fluorescent light, were the cause of the higher incidence of skin cancer. The original conclusion illustrates: A) the fallacy of composition. B) confusion of correlation and causation. C) identifying marginal costs and marginal benefits. D) biases and loaded terminology. Answer: B

Consider This Questions

Type: F E: 4 MI: 4 MA: 4 Status: New 83. (Consider This) Free products offered by firms: A) may or may not be free to society, but are never free to individuals. B) may or may not be free to individuals, but are never free to society. C) are produced and distributed at no cost to society. D) usually are items nobody wants. Answer: B

Type: F E: 4 MI: 4 MA: 4 Status: New 84. (Consider This) The assertion by economists that "there is no free lunch" is: A) contradicted by the presence of free goods offered by firms. B) applies to goods that have prices, not to goods given away free by firms. C) remains true even for goods given away free by firms. D) applies to agricultural goods, but not to manufactured goods. Answer: C

McConnell/Brue: Economics, 16/e

Page 15

Chapter 1: The Nature and Method of Economics

Last Word Questions

Type: A E: 12 MI: 12 MA: 12 85. (Last Word) The economic perspective used in customer decision making at fast-food restaurants is reflected in: A) customers selecting the shortest line. B) decisions for which marginal costs exceed marginal benefits. C) all customer lines tending to be of different length. D) irrational purchasing of high-fat-content food. Answer: A

Type: A E: 12 MI: 12 MA: 12 86. (Last Word) At fast-food restaurants: A) consumers enjoy complete and accurate information. B) decisions are usually made by trial and error. C) decisions entail comparisons of marginal costs and marginal benefits. D) benefits always exceed costs. Answer: C

Type: A E: 12 MI: 12 MA: 12 87. (Last Word) Consumers might leave a fast-food restaurant without being served because: A) they are misinformed about the marginal cost and marginal benefits of the food being served. B) they conclude that the marginal cost (monetary plus time costs) exceeds the marginal benefit. C) the environment is not conducive to a rational choice. D) the lines waiting for service are not of equal length. Answer: B

True/False Questions

Type: D E: 7 MI: 7 MA: 7 88. An economic model is an ideal or utopian type of economy that society should strive to obtain through economic policy. Answer: False

Type: D E: 7 MI: 7 MA: 7 89. Because economic generalizations are abstract, they are impractical and useless. Answer: False

Type: D E: 7 MI: 7 MA: 7 90. If economic theories are solidly based on relevant facts, then appropriate economic policy becomes obvious and undebatable. Answer: False

Type: A E: 10 MI: 10 MA: 10 91. Normative statements are expressions of facts. Answer: False

McConnell/Brue: Economics, 16/e

Page 16

Chapter 1: The Nature and Method of Economics

Type: D E: 10 MI: 10 MA: 10 92. Positive statements are expressions of value judgments. Answer: False

Type: D E: 3 MI: 3 MA: 3 93. The primary objective of economics is to help businesspersons operate their firms profitably. Answer: False

Type: D E: 9 MI: 9 MA: 9 94. The basic goals for the U.S. economy are always complementary since the achievement of any one goal simultaneously furthers the attainment of other basic goals. Answer: False

Type: D E: 9 MI: 9 MA: 9 95. Macroeconomics explains the behavior of individual households and business firms; microeconomics is concerned with the behavior of aggregates or the economy as a whole. Answer: False

Type: D E: 4 MI: 4 MA: 4 96. Rational behavior implies that everyone will make identical choices. Answer: False

Type: D E: 4 MI: 4 MA: 4 97. Marginal analysis means that decision-makers compare the extra benefits with the extra costs of a specific choice. Answer: True

Type: D E: 4 MI: 4 MA: 4 98. Certain inherently desirable products such as education and health care should be produced so long as resources are available. Answer: False

Type: A E: 4 MI: 4 MA: 4 99. Rational individuals may make different choices because their preferences and circumstances differ. Answer: True

Type: D E: 4 MI: 4 MA: 4 100. Choices entail marginal costs because resources are scarce. Answer: True

McConnell/Brue: Economics, 16/e

Page 17

Chapter 1: The Nature and Method of Economics

Appendix Multiple Choice Questions Construction and interpretation of graphs

Type: D Topic: 8 E: 16 MI: 16 MA: 16 101. If we say that two variables are directly related, this means that: A) the relationship between the two is purely random. B) an increase in one variable is associated with a decrease in the other variable. C) an increase in one variable is associated with an increase in the other variable. D) the two graph as a downsloping line. Answer: C

Type: D Topic: 8 E: 16 MI: 16 MA: 16 102. If we say that two variables are inversely related, this means that: A) the two graph as an upsloping line. B) an increase in one variable is associated with a decrease in the other. C) an increase in one variable is associated with an increase in the other. D) the resulting relationship can be portrayed by a straight line parallel to the horizontal axis. Answer: B

Type: D Topic: 8 E: 16 MI: 16 MA: 16 103. Economists: A) always put the independent variable on the horizontal axis and the dependent variable on the vertical axis. B) always put the dependent variable on the horizontal axis and the independent variable on the vertical axis. C) are somewhat arbitrary in assigning independent and dependent variables to the horizontal and vertical axes. D) measure the slope of a line differently than do mathematicians. Answer: C

Type: D Topic: 8 E: 16 MI: 16 MA: 16 104. Which of the following statements is correct? A) The value of the independent variable is determined by the value of the dependent variable. B) The value of the dependent variable is determined by the value of the independent variable. C) The dependent variable designates the "cause" and the independent variable the "effect." D) Dependent variables graph as upsloping lines; independent variables graph as downsloping lines. Answer: B

McConnell/Brue: Economics, 16/e

Page 18

Chapter 1: The Nature and Method of Economics

Use the following to answer questions 105-108:

Type: G Topic: 8 E: 16 MI: 16 MA: 16 105. Refer to the above diagram. Which line(s) show(s) a positive relationship between x and y? A) A only B) both A and D C) A, B, and D D) both C and E Answer: C

Type: G Topic: 8 E: 16 MI: 16 MA: 16 106. Refer to the above diagram. Which line(s) show(s) a negative relationship between x and y? A) A only B) both A and D C) A, B, and D D) both C and E Answer: D

Type: G Topic: 8 E: 18 MI: 18 MA: 18 107. Refer to the above diagram. Which line(s) show(s) a positive vertical intercept? A) A and D only B) B and C only C) A, D, and E D) A, D, and B Answer: C

Type: G Topic: 8 E: 18 MI: 18 MA: 18 108. Refer to the above diagram. Which line(s) show(s) a negative vertical intercept? A) C only B) both C and E C) B, C, and E D) both B and C Answer: D

Type: G Topic: 8 E: 16 MI: 16 MA: 16 109. If two variables are inversely related, then as the value of one variable: A) increases, the value of the other may either increase or decrease. B) decreases, the value of the other decreases. C) increases, the value of the other decreases. D) increases, the value of the other increases. Answer: C

Type: G Topic: 8 E: 16 MI: 16 MA: 16 110. If a positive relationship exists between x and y: A) an increase in x will cause y to decrease. B) a decrease in x will cause y to increase. Answer: C

C) the relationship will graph as an upsloping line. D) the vertical intercept must be positive.

McConnell/Brue: Economics, 16/e

Page 19

Chapter 1: The Nature and Method of Economics

Use the following to answer questions 111-113:

Type: D Topic: 8 E: 16 MI: 16 MA: 16 111. Answer on the basis of the relationships shown in the above four figures. The amount of Y is directly related to the amount of X in: A) both 1 and 3. B) both 1 and 2. C) 2 only. D) l only. Answer: D

Type: A Topic: 8 E: 16 MI: 16 MA: 16 112. Answer on the basis of the relationships shown in the above four figures. The amount of Y is inversely related to the amount of X in: A) 2 only. B) both 1 and 3. C) 3 only. D) 1 only. Answer: C

Type: G Topic: 8 E: 16 MI: 16 MA: 16 113. Answer on the basis of the relationships shown in the above four figures. The amount of Y is unrelated to the amount of X in: A) both 2 and 4. B) 3 only. C) 2 only. D) 1. Answer: C

Type: G Topic: 8 E: 16 MI: 16 MA: 16 114. If price (P) and quantity (Q) are directly related, this means that: A) a change in Q will alter P, but a change in P will not alter Q. B) if P increases, Q will decrease. C) if P increases, Q will also increase. D) an increase in P will cause Q to change, but the direction in which Q changes cannot be predicted. Answer: C

McConnell/Brue: Economics, 16/e

Page 20

Chapter 1: The Nature and Method of Economics

Use the following to answer questions 115-117: Answer the next question(s) on the basis of the following information. Assume that if the interest rate that businesses must pay to borrow funds were 20 percent, it would be unprofitable for businesses to invest in new machinery and equipment so that investment would be zero. But if the interest rate were 16 percent, businesses will find it profitable to invest $10 billion. If the interest rate were 12 percent, $20 billion would be invested. Assume that total investment continues to increase by $10 billion for each successive 4 percentage point decline in the interest rate.

Type: G Topic: 8 E: 16 MI: 16 MA: 16 115. Refer to the above information. Which of the following is an accurate verbal statement of the described relationship? A) There is no regular or dependable relationship between business investment and the interest rate. B) The amount of business investment is unaffected by changes in the interest rate. C) Investment spending by businesses varies inversely with the interest rate. D) Investment spending by businesses varies directly with the interest rate. Answer: C

Type: A Topic: 8 E: 16 MI: 16 MA: 16 116. Refer to the above information. Using i and I to indicate the interest rate and investment (in billions of dollars) respectively, which of the following is the correct tabular presentation of the described relationship?
(a) i 20 16 12 8 4 0 I $50 40 30 20 10 0 i 24 20 16 12 8 4 (b) I $10 20 30 40 50 60 i 20 16 12 8 4 0 (c) I $ 0 10 20 30 40 50 i 20 16 12 8 4 0 (d) I $10 20 30 40 50 60

Answer: C

Type: D Topic: 8 E: 16 MI: 16 MA: 16 117. Refer to the above information. Which of the following correctly expresses the indicated relationship as an equation? A) i = 20 - 4I. B) i = 20 - .4I. C) i = 24 - .4I. D) i = 20 - 10I. Answer: B

McConnell/Brue: Economics, 16/e

Page 21

Chapter 1: The Nature and Method of Economics

Type: T Topic: 8 E: 16 MI: 16 MA: 16 118.

Assume that if the interest rate that businesses must pay to borrow funds were 20 percent, it would be unprofitable for businesses to invest in new machinery and equipment so that investment would be zero. But if the interest rate were 16 percent, businesses will find it profitable to invest $10 billion. If the interest rate were 12 percent, $20 billion would be invested. Assume that total investment continues to increase by $10 billion for each successive 4 percentage point decline in the interest rate. Refer to the above graph. Which of the following is the correct graphical presentation of the indicated relationship? A) line 4 B) line 3 C) line 2 D) line 1 Answer: D

Use the following to answer questions 119-121: Answer the next question(s) on the basis of the following data:
After-tax income $1000 2000 3000 4000 5000 Consumption $ 900 1800 2700 3600 4500

Type: G Topic: 8 E: 16 MI: 16 MA: 16 119. The above data suggest that: A) consumption varies inversely with after-tax income. B) consumption varies directly with after-tax income. C) consumption and after-tax income are unrelated. D) a tax increase will increase consumption. Answer: B

McConnell/Brue: Economics, 16/e

Page 22

Chapter 1: The Nature and Method of Economics

Type: G Topic: 8 E: 16 MI: 16 MA: 16 120. The above data indicate that: A) consumers spend 80 percent of their after-tax incomes. B) consumers spend 90 percent of their after-tax incomes. C) a tax reduction will reduce consumption. D) the relationship between consumption and after-tax income is random. Answer: B

Type: T Topic: 8 E: 17 MI: 17 MA: 17 121. The above data suggest that: A) a policy of tax reduction will increase consumption. B) a policy of tax increases will increase consumption. C) tax changes will have no impact on consumption. D) after-tax income should be lowered to increase consumption. Answer: A

Type: T Topic: 8 E: 17 MI: 17 MA: 17 122. The slope of a straight line can be determined by: A) comparing the absolute horizontal change to the absolute vertical change between two points on the line. B) comparing the absolute vertical change to the absolute horizontal change between two points on the line. C) taking the reciprocal of the vertical intercept. D) comparing the percentage vertical change to the percentage horizontal change between two points on the line. Answer: B

Use the following to answer questions 123-126:

Type: C Topic: 8 E: 16 MI: 16 MA: 16 123. Refer to the above diagram. The variables X and Y are: A) inversely related. B) directly related. C) unrelated. Answer: B

D) negatively related.

Type: A Topic: 8 E: 18 MI: 18 MA: 18 124. Refer to the above diagram. The vertical intercept: A) is 40. B) is 50. C) is 60. D) cannot be determined from the information given. Answer: B

McConnell/Brue: Economics, 16/e

Page 23

Chapter 1: The Nature and Method of Economics

Type: G Topic: 8 E: 17 MI: 17 MA: 17 125. Refer to the above diagram. The slope of the line: A) is - 1/4. B) is + 1/4. C) is .40. D) cannot be determined from the information given. Answer: B

Type: G Topic: 8 E: 17 MI: 17 MA: 17 126. Refer to the above diagram. The equation that shows the relationship between Y and X is: A) Y = 50 + 1/4 X . B) X = 1/4 Y . C) Y = .4X . D) Y = 1/4 X - 50. Answer: A

Use the following to answer questions 127-131:

Type: E Topic: 8 E: 16 MI: 16 MA: 16 127. Refer to the above graph. Which of the following statements is correct? A) Quantity demanded and quantity supplied are independent of price. B) Price and quantity demanded are directly related. C) Price and quantity supplied are directly related. D) Price and quantity supplied are inversely related. Answer: C

Type: E Topic: 8 E: 16 MI: 16 MA: 16 128. Refer to the above graph. Which of the following schedules correctly reflects "demand"?

(a) P $12 10 8 6 4 2 Qd 0 0 10 20 30 40 P $14 12 10 8 6 4

(b) Qd 0 0 20 40 60 80 P $14 12 10 8 6 4

(c) Qd 60 50 40 30 20 10 P $12 10 8 6 4 2

(d) Qd 0 10 20 30 40 50

Answer: A

McConnell/Brue: Economics, 16/e

Page 24

Chapter 1: The Nature and Method of Economics

Type: G Topic: 8 E: 16 MI: 16 MA: 16 129. Refer to the above graph. Which of the following schedules correctly reflects "supply"?
(a) P Qs $12 50 10 30 8 10 6 0 4 0 2 0 Answer: C (b) P $14 12 10 8 6 4 Qs 50 40 30 20 10 0 P $12 10 8 6 4 2 (c) Qs 50 40 30 20 10 0 (d) P $12 10 8 6 4 2

Qs 0 0 10 20 30 40

Type: C Topic: 8 E: 18 MI: 18 MA: 18 130. Refer to the above graph. Using Qd for quantity demanded and P for price, which of the following equations correctly states the demand for this product? A) P = Qd/10. B) P = 50 - P/2. C) P = 10 - .2Qd. D) P = 10 - 2Qd. Answer: C

Type: C Topic: 8 E: 18 MI: 18 MA: 18 131. Refer to the above graph. Using Qs for quantity supplied and P for price, which of the following equations correctly states the supply of this product? A) P = 4 + .2Qs. B) P = 60/Qs. C) P = 10Qs - 2P. D) P = 2 + .2Qs. Answer: D

Type: E Topic: 8 E: 16 MI: 16 MA: 16 132. Assume a household would consume $100 worth of goods and services per week if its weekly income were zero and would spend an additional $80 per week for each $100 of additional income. Letting C represent consumption and Y represent income, the equation that summarizes this relationship is: A) C = 80 + 100Y . B) C = 100 + .8Y . C) C = 100 + 80Y . D) C = 80 + .1Y . Answer: B

McConnell/Brue: Economics, 16/e

Page 25

Chapter 1: The Nature and Method of Economics

Use the following to answer questions 133-139: Answer the next question(s) on the basis of the following five data sets wherein it is assumed that the variable shown on the left is the independent variable and the one on the right is the dependent variable. Assume in graphing these data that the independent variable is shown on the horizontal axis and the dependent variable on the vertical axis.

(1) J 0 40 80 120 160 200

(2) K 10 20 30 40 50 60 L 0 30 60 90 120 150 M -15 -5 5 15 25 35

(3) N 100 80 60 40 20 0

(4) P 40 50 60 70 80 90 R 0 20 40 60 80 100 T -15 -25 -35 -45 -55 -65 U 0 5 10 15 20 25

(5) V 0 10 20 30 40 50

Type: E Topic: 8 E: 18 MI: 18 MA: 18 133. Refer to the above data sets. The variables are directly related in: A) all five data sets. C) in data sets 1, 2, and 3 only. B) in none of the data sets. D) in data sets 1, 2, and 5 only. Answer: D

Type: E Topic: 8 E: 18 MI: 18 MA: 18 134. Refer to the above data sets. The vertical intercept is positive for: A) all five data sets. B) data sets 1 and 3 only. C) data sets 1, 3, and 5 only. Answer: B

D) data set 2 only.

Type: T Topic: 8 E: 18 MI: 18 MA: 18 135. Refer to the above data sets. The vertical intercept is negative for: A) none of the data sets. C) data sets 2 and 4 only. B) data sets 1 and 3 only. D) data sets 1 and 5 only. Answer: C

Type: T Topic: 8 E: 18 MI: 18 MA: 18 136. Refer to the above data sets. The equation for data set 3 is: A) P = 90 - .5N . B) P = 90 + .5N . C) P = .5N . D) P = 40 + .5N . Answer: A

Type: T Topic: 8 E: 18 MI: 18 MA: 18 137. Refer to the above data sets. For which data set(s) is the vertical intercept zero? A) data set 4 B) data set 5 C) data sets 2 and 3 D) data set 1 Answer: B

Type: E Topic: 8 E: 18 MI: 18 MA: 18 138. Refer to the above data sets. The equation for data set 5 is: A) V = .5Y . B) U = -.5V . C) U = V . D) V = 2U . Answer: D

McConnell/Brue: Economics, 16/e

Page 26

Chapter 1: The Nature and Method of Economics

Type: E Topic: 8 E: 17 MI: 17 MA: 17 139. Refer to the above data sets. Which of the data sets would graph as an upsloping line? A) 1 only B) 1, 2, and 3 only C) 4 and 5 only D) 1, 2, and 5 only Answer: D

Type: E Topic: 8 E: 17 MI: 17 MA: 17 140. If the equation y = 5 + 6x was graphed, the: A) slope would be -5. B) slope would be +5. Answer: C

C) slope would be +.6. D) vertical intercept would be +.6.

Type: C Topic: 8 E: 17 MI: 17 MA: 17 141. If the equation y = 15 - 4x was plotted, the: A) vertical intercept would be -4. B) vertical intercept would be +4. Answer: D

C) vertical intercept would be +9. D) slope would be -4.

Type: E Topic: 8 E: 17 MI: 17 MA: 17 142. If the equation y = -10 + 2.5x was plotted: A) the vertical intercept would be -10. B) the slope would be -7.5. Answer: A

C) it would graph as a downsloping line. D) the slope would be -10.

Type: E Topic: 8 E: 18 MI: 18 MA: 18 143.

The movement from line A to line A' represents a change in: A) the slope only. C) both the slope and the intercept. B) the intercept only. D) neither the slope nor the intercept. Answer: B

McConnell/Brue: Economics, 16/e

Page 27

Chapter 1: The Nature and Method of Economics

Use the following to answer questions 144-146:

Type: E Topic: 8 E: 16 MI: 16 MA: 16 144. In the above diagram variables x and y are: A) both dependent variables. B) directly related. Answer: C

C) inversely related.

D) unrelated.

Type: G Topic: 8 E: 18 MI: 18 MA: 18 145. In the above diagram the vertical intercept and slope are: A) 4 and -11/3 respectively. C) 3 and + 3/4 respectively. 1/ respectively. B) 3 and -1 3 D) 4 and + 3/4 respectively. Answer: A

Type: G Topic: 8 E: 18 MI: 18 MA: 18 146. In the above diagram the equation for this line is: A) y = 4 - 11/3 x. B) y = 3 + 3/4 x. C) y = 4 - 3/4 x. Answer: A

D) y = 4 + 11/3 x.

Type: G Topic: 8 E: 17 MI: 17 MA: 17 147. If we are considering the relationship between two variables and release one of the other-things-equal assumptions, we would expect: A) the relationship to change from direct to inverse. B) the line representing that relationship on a graph to change locations. C) the data points to have a tighter fit to the line representing the relationship. D) the relationship to change from inverse to direct. Answer: B

Type: G Topic: 8 E: 18 MI: 18 MA: 18 148. The amount of pizzas that consumers want to buy per week is reflected in the equation P = 15 - .02Qd, where Qd is the amount of pizzas purchased per week and P is the price of pizzas. On the basis of this information we can say that: A) if pizzas were free, people would consume 800 per week. B) more pizzas will be purchased at a high price than at a low price. C) if the price of pizzas is $6, then 150 will be purchased. D) 50 fewer pizzas will be purchased per week for every $1 increase in price. Answer: D

McConnell/Brue: Economics, 16/e

Page 28

Chapter 1: The Nature and Method of Economics

Use the following to answer questions 149-151:

Type: A Topic: 8 E: 18 MI: 18 MA: 18 149. Refer to the above diagram. The slope of curve ZZ at point A is: A) +2. B) +21/2. C) -21/2. D) +4. Answer: B

Type: E Topic: 8 E: 18 MI: 18 MA: 18 150. Refer to the above diagram. The slope of curve ZZ at point B is: A) infinity. B) zero. C) +1. D) -1. Answer: B

Type: G Topic: 8 E: 18 MI: 18 MA: 18 151. Refer to the above diagram. The slope of curve ZZ at point C is: A) -4. B) -2. C) -22/5. D) +3. Answer: C

Type: G Topic: 8 E: 18 MI: 18 MA: 18 152. The slope of a line parallel to the vertical axis is: A) zero. B) one. C) infinite. D) one-half. Answer: C

Type: G Topic: 8 E: 18 MI: 18 MA: 18 153. The slope of a line parallel to the horizontal axis is: A) zero. B) one. C) infinite. D) one-half. Answer: A

Type: D Topic: 8 E: 18 MI: 18 MA: 18 154. The measured slope of a line: A) is independent of how the two variables are denominated. B) will be affected by how the two variables are denominated. C) necessarily diminishes as one moves rightward on the line. D) necessarily increases as one moves rightward on the line. Answer: B

McConnell/Brue: Economics, 16/e

Page 29

Chapter 1: The Nature and Method of Economics

Type: D Topic: 8 E: 18 MI: 18 MA: 18 155. Slopes of lines are especially important in economics because: A) they measure marginal changes. B) they always tell us something about profits. C) positive slopes are always preferred to negative slopes. D) they always relate to resource and output scarcity. Answer: A

McConnell/Brue: Economics, 16/e

Page 30

CHAPTER 2

The Economizing Problem

Topic 1. 2. 3. 4. 5. 6. 7. 8. 9. 10. Economizing problem Economic resources Efficiency; full employment and full production Production possibilities analysis Opportunity costs Allocative efficiency Economic growth and decline Applications Economic systems Circular flow model Consider This Last Word True-False

Question numbers 1-12 13-23 24-27 28-95 96-105 106-117 118-132 133-140 141-149 150-165 166-167 168-170 171-184

____________________________________________________________

_______________________________________

____________________________________________________________

_______________________________________

Multiple Choice Questions Economizing problem

Type: D Topic: 1 E: 22 MI: 22 MA: 22 1. The economizing problem is one of deciding how to make the best use of: A) virtually unlimited resources to satisfy virtually unlimited wants. B) limited resources to satisfy virtually unlimited wants. C) unlimited resources to satisfy limited wants. D) limited resources to satisfy limited wants. Answer: B

Type: D Topic: 1 E: 24 MI: 24 MA: 24 2. The concept of economic efficiency is primarily concerned with: A) the limited wants-unlimited resources dilemma. B) considerations of equity in the distribution of wealth. C) obtaining the maximum output from available resources. D) the conservation of irreplaceable natural resources. Answer: C

Chapter 2: The Economizing Problem

Type: D Topic: 1 E: 23 MI: 22 MA: 22 3. When the economist says that economic wants are insatiable, this means that: A) economic resources are valuable only because they can be used to produce consumer goods. B) economic resources--land, labor, capital, and entrepreneurial ability--are scarce. C) these wants are virtually unlimited and therefore incapable of complete satisfaction. D) the structure of consumer demand varies from time to time and from country to country. Answer: C

Type: D Topic: 1 E: 22 MI: 23 MA: 23 4. The fundamental problem of economics is: A) to establish a democratic political framework for the provision of social goods and services. B) the establishment of prices that accurately reflect the relative scarcities of products and resources. C) the scarcity of productive resources relative to economic wants. D) to achieve a more equitable distribution of money income in order to mitigate poverty. Answer: C

Type: D Topic: 1 E: 22 MI: 22 MA: 22 5. The science of economics stems from the fact that: A) the production possibilities curve is bowed inward to the origin. B) resources are scarce relative to people's demand for goods and services. C) individuals and institutions behave only in their self-interest. D) historically the production possibilities curve has been shifting toward the origin. Answer: B

Type: D Topic: 1 E: 23 MI: 23 MA: 23 6. The study of economics exists because: A) government interferes with the efficient allocation of scarce resources. B) resources are scarce in relation to economic wants. C) the market system is an obstacle to the efficient use of plentiful resources to satisfy constrained wants. D) resources are overly abundant as compared to wants; thus, an allocation problem exists. Answer: B

Type: D Topic: 1 E: 23 MI: 23 MA: 23 7. The scarcity problem: A) persists only because countries have failed to achieve continuous full employment. B) persists because economic wants exceed available productive resources. C) has been solved in all industrialized nations. D) has been eliminated in affluent societies such as the United States and Canada. Answer: B

Type: D Topic: 1 E: 24 MI: 24 MA: 24 8. Because of their scarcity, the efficient use of resources is: A) an important issue in all economies. B) an important issue only in centrally planned economies. C) an important issue only in market economies. D) not an important issue. Answer: A

McConnell/Brue: Economics, 16/e

Page 32

Chapter 2: The Economizing Problem

Type: A Topic: 1 E: 24 MI: 24 MA: 24 9. An increase in efficiency suggests that an economy: A) has moved from a point outside of, to a point on, its production possibilities curve. B) has decided to produce more consumer goods and fewer capital goods. C) has moved from a point on, to a point inside, its production possibilities curve. D) is able to get more output from a given amount of inputs. Answer: D

Type: D Topic: 1 E: 24 MI: 24 MA: 24 10. Economics can best be described as the study of: A) how to profitably invest one's income in stocks and bonds. B) how to use scarce productive resources efficiently. C) how government policies affect businesses and labor. D) managing business enterprises for profit. Answer: B

Type: D Topic: 1 E: 23 MI: 23 MA: 23 11. As used in economics, the idea of scarce resources means that: A) mineral deposits are only available in finite amounts. B) resources are not so plentiful that all economic wants can be fulfilled. C) some resources are free while others have price tags on them. D) the quantities available of some resources exceed the demand for them. Answer: B

Type: D Topic: 1 E: 24 MI: 24 MA: 24 12. Economics is primarily the study of: A) why resources are scarce. B) how advertising and sales promotion shape consumer wants. C) how to make profitable financial investments. D) how to use scarce resources efficiently. Answer: D

Economic resources

Type: D Topic: 2 E: 23 MI: 23 MA: 23 Status: New 13. Which of the following is a land resource? A) a computer programmer C) silicon (sand) used to make computer chips B) a computer D) a piece of software used by a firm Answer: C

Type: D Topic: 2 E: 23 MI: 23 MA: 23 Status: New 14. Which of the following is a labor resource? A) a computer programmer C) silicon (sand) used to make computer chips B) a computer D) a piece of software used by a firm Answer: A

McConnell/Brue: Economics, 16/e

Page 33

Chapter 2: The Economizing Problem

Type: D Topic: 2 E: 23 MI: 23 MA: 23 Status: New 15. Which of the following is a capital resource? A) a computer programmer B) a corporate bond issued by a computer manufacturer C) silicone (sand) used to make computer chips D) a piece of software used by a firm Answer: D

Type: D Topic: 2 E: 23 MI: 23 MA: 23 Status: New 16. The four factors of production are: A) land, labor, capital, and money B) land, labor, capital, and entrepreneurial ability C) labor, capital, technology, and entrepreneurial ability D) labor, capital, entrepreneurial ability, and money Answer: B

Type: D Topic: 2 E: 29 MI: 29 MA: 29 17. Which of the following is a land resource? A) a farmer B) an oil drilling rig C) a machine for detecting earthquakes. D) natural gas Answer: D

Type: D Topic: 2 E: 23 MI: 23 MA: 23 18. Which of the following lists includes only capital resources (and therefore no labor or land resources)? A) an ice arena; a professional hockey player; hockey uniforms. B) the owner of a new startup firm; a chemistry lab; a researcher. C) a hydroelectric dam; water behind the dam; power lines. D) autos owned by a car rental firm; computers at the car rental agency; the vans that shuffle rental customers to and from the airport. Answer: D

Type: A Topic: 2 E: 23 MI: 23 MA: 23 19. Money is not an economic resource because: A) money, as such, is not productive. B) idle money balances do not earn interest income. C) the terms of trade can be determined in nonmonetary terms. D) money is not a free gift of nature. Answer: A

Type: D Topic: 2 E: 24 MI: 24 MA: 24 20. The money payments made to owners of land, labor, capital, and entrepreneurial ability are: A) interest, wages, rent, and profits respectively. B) rent, wages, dividends, and interest respectively. C) rent, profits, wages, and interest respectively. D) rent, wages, interest, and profits respectively. Answer: D

McConnell/Brue: Economics, 16/e

Page 34

Chapter 2: The Economizing Problem

Type: D Topic: 2 E: 24 MI: 24 MA: 24 21. Economic resources are also called: A) free gifts of nature. B) consumption goods. Answer: D

C) units of money capital.

D) factors of production.

Type: D Topic: 2 E: 23 MI: 23 MA: 23 22. Which of the following is real capital? A) a pair of stockings B) a construction crane Answer: B

C) a savings account

D) a share of IBM stock

Type: D Topic: 2 E: 23 MI: 23 MA: 23 23. The main function of the entrepreneur is to: A) make routine pricing decisions. B) innovate. Answer: B

C) purchase capital. D) create market demand.

Efficiency; full employment and full production

Type: D Topic: 3 E: 24 MI: 24 MA: 24 24. Assuming an economy has fixed quantities of resources, that economy: A) is more efficient, the larger the amount of goods and services it produces. B) is able to satisfy all consumer wants. C) will produce the same output whether or not resources are used efficiently. D) is able to produce the same amount of output regardless of the production technologies it chooses. Answer: A

Type: D Topic: 3 E: 24 MI: 24 MA: 24 25. Productive efficiency refers to: A) the use of the least-cost method of production. B) the production of the product-mix most wanted by society. C) the full employment of all available resources. D) production at some point inside of the production possibilities curve. Answer: A

Type: A Topic: 3 E: 24 MI: 24 MA: 24 26. If an economy produces its most wanted goods but uses outdated production methods, it is: A) achieving productive efficiency, but not allocative efficiency. B) not achieving productive efficiency. C) achieving both productive and allocative efficiency. D) engaged in roundabout production. Answer: B

Type: D Topic: 3 E: 24 MI: 24 MA: 24 27. To realize full production a society must achieve: A) income inequality. B) productive efficiency only. C) both allocative and productive efficiency. D) any output lying inside of its production possibilities curve. Answer: C

McConnell/Brue: Economics, 16/e

Page 35

Chapter 2: The Economizing Problem

Production possibilities analysis

Type: A Topic: 4 E: 26 MI: 26 MA: 26 28. The production possibilities curve illustrates the basic principle that: A) the production of more of any one good will in time require smaller and smaller sacrifices of other goods. B) an economy will automatically obtain full employment of its resources. C) if all the resources of an economy are in use, more of one good can be produced only if less of another good is produced. D) an economy's capacity to produce increases in proportion to its population size. Answer: C

Type: A Topic: 4 E: 29 MI: 29 MA: 29 29. Which of the following will not produce an outward shift of the production possibilities curve? A) an upgrading of the quality of a nation's human resources B) the reduction of unemployment C) an increase in the quantity of a society's labor force D) the improvement of a society's technological knowledge Answer: B

Type: A Topic: 4 E: 29 MI: 29 MA: 29 30. Unemployment and/or productive inefficiencies: A) cause the production possibilities curve to shift outward. B) can exist at any point on a production possibilities curve. C) are both illustrated by a point outside the production possibilities curve. D) are both illustrated by a point inside the production possibilities curve. Answer: D

Type: A Topic: 4 E: 27 MI: 27 MA: 27 31. If the production possibilities curve is a straight line: A) the two products will sell at the same market prices. B) economic resources are perfectly shiftable between the production of the two products. C) the two products are equally important to consumers. D) equal quantities of the two products will be produced at each possible point on the curve. Answer: B

Type: A Topic: 4 E: 26 MI: 26 MA: 26 32. A production possibilities curve illustrates: A) scarcity. B) market prices. C) consumer preferences. D) the distribution of income. Answer: A

McConnell/Brue: Economics, 16/e

Page 36

Chapter 2: The Economizing Problem

Type: A Topic: 4 E: 26 MI: 26 MA: 26 33. A production possibilities curve shows: A) that resources are unlimited. B) that people prefer one of the goods more than the other. C) the maximum amounts of two goods that can be produced assuming the full and efficient use of available resources. D) combinations of capital and labor necessary to produce specific levels of output. Answer: C

Type: A Topic: 4 E: 27 MI: 27 MA: 27 34. A nation's production possibilities curve is bowed out from the origin because: A) resources are not equally efficient in producing every good. B) the originator of the idea drew it this way and modern economists follow this convention. C) resources are scarce. D) wants are virtually unlimited. Answer: A

Use the following to answer questions 35-39: Answer the next question(s) on the basis of the data given in the following production possibilities table:
Production possibilities (alternatives) A B C D E F 5 1 4 3 2 0 0 14 5 9 12 15

Capital goods Consumer goods

Type: T Topic: 4 E: 27 MI: 27 MA: 27 35. Refer to the above table. If the economy is producing at production alternative C, the opportunity cost of the tenth unit of consumer goods will be: A) 4 units of capital goods. C) 3 units of capital goods. B) 2 units of capital goods. D) 1/3 of a unit of capital goods. Answer: D

Type: C Topic: 4 E: 27 MI: 27 MA: 27 36. Refer to the above table. As compared to production alternative D, the choice of alternative C would: A) tend to generate a more rapid growth rate. C) entail unemployment. B) be unattainable. D) tend to generate a slower growth rate. Answer: A

Type: C Topic: 4 E: 28 MI: 28 MA: 28 37. Refer to the above table. A total output of 3 units of capital goods and 4 units of consumer goods: A) is irrelevant because the economy is capable of producing a larger total output. B) will result in the maximum rate of growth available to this economy. C) would involve an inefficient use of the economy's scarce resources. D) is unobtainable in this economy. Answer: C

McConnell/Brue: Economics, 16/e

Page 37

Chapter 2: The Economizing Problem

Type: C Topic: 4 E: 29 MI: 29 MA: 29 38. Refer to the above table. For this economy to produce a total output of 3 units of capital goods and 13 units of consumer goods it must: A) achieve economic growth. B) use its resources more efficiently than the data in the table now indicate. C) allocate its available resources most efficiently among alternative uses. D) achieve the full employment of available resources. Answer: A

Type: A Topic: 4 E: 27 MI: 27 MA: 27 39. Refer to the above table. For these data the law of increasing opportunity costs is reflected in the fact that: A) the amount of consumer goods that must be sacrificed to get more capital goods diminishes beyond a point. B) larger and larger amounts of capital goods must be sacrificed to get additional units of consumer goods. C) the production possibilities data would graph as a straight downsloping line. D) the economy's resources are presumed to be scarce. Answer: B

Type: A Topic: 4 E: 27 MI: 27 MA: 27 40. When an economy is operating with maximum efficiency, the production of more of commodity A will mean the production of less of commodity B because: A) of the law of increasing opportunity costs. B) economic wants are insatiable. C) resources are limited. D) resources are specialized and only imperfectly shiftable. Answer: C

Type: C Topic: 4 E: 30 MI: 30 MA: 30 41. Assume that a change in government policy results in greater production of both consumer goods and investment goods. We can conclude that: A) the economy was suffering from unemployment and/or the inefficient use of resources before the policy change. B) the economy's production possibilities curve has been shifted to the left as a result of the policy decision. C) this economy's production possibilities curve is convex (bowed inward) to the origin. D) the law of increasing opportunity costs does not apply in this society. Answer: A

Type: D Topic: 4 E: 26 MI: 26 MA: 26 42. The production possibilities curve: A) shows all of those levels of production that are consistent with a stable price level. B) indicates that any combination of goods lying outside the curve is economically inefficient. C) is a frontier between all combinations of two goods that can be produced and those combinations that cannot be produced. D) shows all of those combinations of two goods that are most preferred by society. Answer: C

McConnell/Brue: Economics, 16/e

Page 38

Chapter 2: The Economizing Problem

Type: C Topic: 4 E: 27 MI: 27 MA: 27 43. Assume an economy is operating at some point on its production possibilities curve, which shows civilian and military goods. If the output of military goods is increased, the output of civilian goods: A) will remain unchanged. C) must be decreased. B) may be either increased or decreased. D) must also be increased. Answer: C

Type: A Topic: 4 E: 26 MI: 26 MA: 26 44. Any point inside the production possibilities curve indicates: A) the realization of allocative efficiency. B) that resources are imperfectly shiftable among alternative uses. C) the presence of inflationary pressures. D) that more output could be produced with available resources. Answer: D

Use the following to answer questions 45-46:

Type: G Topic: 4 E: 30 MI: 30 MA: 30 45. Refer to the above diagram. Other things equal, this economy will achieve the most rapid rate of growth if: A) the ratio of capital to consumer goods is minimized. B) it chooses point C. C) it chooses point B. D) it chooses point A. Answer: D

Type: G Topic: 4 E: 29 MI: 29 MA: 29 46. Refer to the above diagram. This economy will experience unemployment if it produces at point: A) A. B) B. C) C. D) D. Answer: D

Type: D Topic: 4 E: 29 MI: 29 MA: 29 47. In drawing the production possibilities curve we assume that: A) technology is fixed. C) economic resources are unlimited. B) unemployment exists. D) wants are limited. Answer: A

McConnell/Brue: Economics, 16/e

Page 39

Chapter 2: The Economizing Problem

Type: D Topic: 4 E: 29 MI: 29 MA: 29 48. Which of the following is assumed in constructing a typical production possibilities curve? A) the economy is using its resources inefficiently. B) resources are perfectly shiftable among alternative uses. C) production technology is fixed. D) the economy is engaging in international trade. Answer: C

Type: A Topic: 4 E: 27 MI: 27 MA: 27 49. If the production possibilities curve were a straight downsloping line, this would suggest that: A) resources are perfectly shiftable between the production of these two goods. B) it is possible to produce more of both products. C) both products are equally capable of satisfying consumer wants. D) the two products have identical prices. Answer: A

Type: D Topic: 4 E: 27 MI: 27 MA: 27 50. The typical production possibilities curve is: A) an upsloping line that is concave to the origin. B) a downsloping line that is convex to the origin. Answer: C

C) a downsloping line that is concave to the origin. D) a straight upsloping line.

Type: C Topic: 4 E: 29 MI: 29 MA: 29 51. Deltonia produces both consumer and capital goods. Other things equal, if Deltonia reduces the percentage of its output devoted to capital goods, then: A) its rate of growth will decline. B) its production possibilities curve will shift to the left. C) it must also reduce the percentage of its output devoted to consumer goods. D) its rate of growth will increase. Answer: A

Type: A Topic: 4 E: 27 MI: 27 MA: 27 52. The slope of the typical production possibilities curve: A) is positive. B) increases as one moves southeast along the curve. C) is constant as one moves down the curve. D) decreases as one moves southeast along the curve. Answer: B

Type: A Topic: 4 E: 29 MI: 29 MA: 29 53. Assume an economy is incurring unemployment and failing to realize least-cost production. The effect of resolving these problems will be to: A) move the level of actual output to the economy's production possibilities curve. B) create a less equal distribution of income. C) shift its production possibilities curve to the left. D) shift its production possibilities curve to the right. Answer: A

McConnell/Brue: Economics, 16/e

Page 40

Chapter 2: The Economizing Problem

Use the following to answer questions 54-58:

Civilian goods Military goods

Duckistan Production possibilities A B C D E 20 18 14 8 0 0 1 2 3 4 Herbania Production possiblities A B C D E 14 40 36 26 0 3 0 1 2 4

Civilian goods Military goods

Type: T Topic: 4 E: 30 MI: 30 MA: 30 Status: New 54. Refer to the above tables. Suppose that the amount and quality of resources are the same in both countries. We can conclude that: A) Duckistan is technologically better than Herbania at producing military goods. B) Herbania is technologically better than Herbania at producing both military goods and civilian goods. C) the total opportunity cost of producing 4 units of military goods is the same in both countries. D) Herbania is technologically superior to Duckistan in producing civilian goods. Answer: D

Type: T Topic: 4 E: 31 MI: 31 MA: 31 Status: New 55. Refer to the above tables. Suppose that technology and the quality of resources are the same in both countries. We can conclude that: A) Duckistan has more resources than Herbania. B) Herbania has more resources than Herbania. C) Duckistan has greater opportunity costs than Herbania. D) Prices are twice as high in Herbania as in Duckistan. Answer: B

Type: T Topic: 4 E: 27 MI: 27 MA: 27 Status: New 56. Refer to the above tables. Opportunity costs are: A) increasing in Duckistan but constant in Herbania. B) constant in both Duckistan and Herbania. C) larger in Duckistan than in Herbania. D) smaller in Duckistan than Herbania. Answer: D

Type: T Topic: 4 E: 30 MI: 30 MA: 30 Status: New 57. Refer to the above tables. Opportunity costs are: A) constant in both Duckistan and Herbania. B) larger in Duckistan than in Herbania. C) increasing in both Duckistan and Herbania. D) increasing in Duckistan and constant in Herbania. Answer: C

McConnell/Brue: Economics, 16/e

Page 41

Chapter 2: The Economizing Problem

Type: T Topic: 4 E: 30 MI: 30 MA: 30 Status: New 58. Refer to the above tables. Suppose that Duckistan and Herbania are each producing 14 units of civilian goods and 2 unit of military goods. Then: A) Duckistan is fully and efficiently using its resources but Herbania is not. B) both Duckistan and Herbania are fully and efficiently using their resources. C) Herbania is fully and efficiently using its resources by Duckistan is not. D) neither Duckistan nor Herbania are fully and efficiently using their resource. Answer: A

Type: G Topic: 4 E: 27 MI: 27 MA: 27 59.

In the figure above are two linear production possibilities curves for countries Alpha and Beta. We can conclude that: A) different value systems make it impossible to compare opportunity costs in the two countries. B) the opportunity cost of shelter is greater in Beta than it is in Alpha. C) the opportunity cost of food is greater in Alpha than it is in Beta. D) the opportunity cost of shelter is greater in Alpha than it is in Beta. Answer: D

Type: D Topic: 4 E: 23 MI: 23 MA: 23 60. The process of producing and accumulating capital goods is called: A) money capital. B) depreciation. C) investment. D) consumption. Answer: C

Type: D Topic: 4 E: 26 MI: 26 MA: 26 61. Which of the following is not correct? A typical production possibilities curve: A) indicates how much of two products a society can produce. B) reveals how much each additional unit of one product will cost in terms of the other product. C) specifies how much of each product society should produce. D) indicates that to produce more of one product society must forgo larger and larger amounts of the other product. Answer: C

McConnell/Brue: Economics, 16/e

Page 42

Chapter 2: The Economizing Problem

Type: A Topic: 4 E: 31 MI: 31 MA: 31 62. A country can achieve some combination of goods outside its production possibilities curve by: A) idling some of its resources. B) specializing and engaging in international trade. C) buying the debt (bonds and stocks) of foreign nations. D) producing more capital goods and fewer consumer goods. Answer: B

Type: A Topic: 4 E: 29 MI: 29 MA: 29 63. A point inside a production possibilities curve best illustrates: A) unemployment. C) the use of best-available technology. B) the efficient use of resources. D) unlimited wants. Answer: A

Use the following to answer questions 64-68:

Type: G Topic: 4 E: 27 MI: 27 MA: 27 64. Refer to the above diagram. This production possibilities curve is constructed so that: A) resources are presumed to be perfectly shiftable between bread and tractors. B) the opportunity cost of bread diminishes as more bread is produced. C) the opportunity cost of tractors diminishes as more bread is produced. D) the opportunity cost of both bread and tractors increases as more of each is produced. Answer: D

Type: G Topic: 4 E: 10, 31 MI: 10, 31 MA: 10, 31 65. Refer to the above diagram. Which of the following is a normative statement? A) Point C is superior to point B because it is important to enhance the future of society. B) If society is initially at point C, it must sacrifice 6 units of bread to obtain one more unit of tractors. C) If society produces 2 units of tractors and 12 units of bread, it is not using its available resources with maximum efficiency. D) Other things equal, the combination of outputs represented by point D will result in more rapid economic growth than will the combination represented by point C. Answer: A

McConnell/Brue: Economics, 16/e

Page 43

Chapter 2: The Economizing Problem

Type: G Topic: 4 E: 10, 26 MI: 10, 26 MA: 10, 26 66. Refer to the above diagram. Which of the following is a positive statement? A) A point inside the production possibilities curve is superior to a point on the curve because the former requires less work effort. B) Because any society should stress economic growth as its major goal, point D is superior to point C. C) Point B is preferable to point C because the ultimate goal of economic activity is to maximize consumption. D) Given its resources and technology, this society is incapable of simultaneously producing 3 units of tractors and 15 units of bread. Answer: D

Type: G Topic: 4 E: 27 MI: 27 MA: 27 67. Refer to the above diagram. Starting at point A, the opportunity cost of producing each successive unit of tractors is: A) a constant 2 units of bread. C) 8, 6, 4, and 2 units of bread. B) 2, 4, 6, and 8 units of bread. D) the reciprocal of the output of tractors. Answer: B

Type: G Topic: 4 E: 27 MI: 27 MA: 27 68. Refer to the above diagram. Starting at point E, the production of successive units of bread will cost: A) a constant 8 units of tractors. C) 1/8, 1/6, 1/4, and 1/2 units of tractors. B) a constant 6 units of tractors. D) 1/2, 1/4, 1/6, and 1/8 units of tractors. Answer: C

Use the following to answer questions 69-70:

War goods ( “guns”)

b a c

d

Civilian goods (“butter”)
Type: G Topic: 4 E: 29, 32 MI: 29, 32 MA: 29, 32 69. Refer to the above production possibilities curve. At the onset of the Second World War the United States had large amounts of idle human and property resources. Its economic adjustment from peacetime to wartime can best be described by the movement from point: A) c to point b. B) b to point c. C) a to point b. D) c to point d. Answer: C

McConnell/Brue: Economics, 16/e

Page 44

Chapter 2: The Economizing Problem

Type: G Topic: 4 E: 30 MI: 30 MA: 30 70. Refer to the above production possibilities curve. At the onset of the Second World War the Soviet Union was already at full employment. Its economic adjustment from peacetime to wartime can best be described by the movement from point: A) c to point b. B) b to point c. C) a to point b. D) c to point d. Answer: A

Type: D Topic: 4 E: 26 MI: 26 MA: 26 71. The production possibilities curve shows: A) the various combinations of two goods that can be produced when society uses its scarce resources efficiently. B) the minimum outputs of two goods that will sustain a society. C) the various combinations of two goods that can be produced when some resources are unemployed. D) the ideal, but unattainable, combinations of two goods that would maximize consumer satisfactions. Answer: A

Type: D Topic: 4 E: 26 MI: 26 MA: 26 72. The negative slope of the production possibilities curve is a graphical way of indicating that: A) any economy "can have its cake and eat it too." B) to produce more of one product we must do with less of another. C) the principle of increasing opportunity costs applies to only parts of the economy. D) consumers buy more when prices are low than when prices are high. Answer: B

Type: A Topic: 4 E: 26 MI: 26 MA: 26 73. If an economy is operating on its production possibilities curve for consumer goods and capital goods, this means that: A) it is impossible to produce more consumer goods. B) resources cannot be reallocated between the two goods. C) it is impossible to produce more capital goods. D) more consumer goods can only be produced at the cost of fewer capital goods. Answer: D

Type: D Topic: 4 E: 25 MI: 25 MA: 25 74. In drawing a production possibilities curve we hold constant: A) the money supply. C) both technology and resource supplies. B) the consumer price index. D) resource supplies only. Answer: C

Type: D Topic: 4 E: 25 MI: 25 MA: 25 75. The construction of a production possibilities curve assumes: A) the quantities of all resources are unlimited. B) technology is fixed. C) full employment, but not full production, is being realized. D) there is no inflation in the economy. Answer: B

McConnell/Brue: Economics, 16/e

Page 45

Chapter 2: The Economizing Problem

Type: D Topic: 4 E: 27 MI: 27 MA: 27 76. A typical concave (to the origin) production possibilities curve implies: A) that economic resources are unlimited. B) that society must choose among various attainable combinations of goods. C) decreasing opportunity costs. D) that society is using a market system to allocate resources. Answer: B

Type: D Topic: 4 E: 26 MI: 26 MA: 26 77. The production possibilities curve tells us: A) the specific combination of two products that is most desired by society. B) that costs do not change as society varies its output. C) costs are irrelevant in a society that has fixed resources. D) the combinations of two goods that can be produced with society's available resources. Answer: D

Type: D Topic: 4 E: 27 MI: 27 MA: 27 78. The production possibilities curve has: A) a positive slope that increases as we move along it from left to right. B) a negative slope that increases as we move along it from left to right. C) a negative slope that decreases as we move along it from left to right. D) a negative slope that is constant as we move along it from left to right. Answer: B

Type: A Topic: 4 E: 26 MI: 26 MA: 26 79. Which one of the following statements is correct? A) Relative scarcity is no longer a central idea in economics because we are in an age of abundance. B) Most production possibilities curves are convex to the origin. C) The production possibilities curve shows society's preferences for consumer goods relative to capital goods. D) The central concept underlying the production possibilities curve is that of limited resources. Answer: D

McConnell/Brue: Economics, 16/e

Page 46

Chapter 2: The Economizing Problem

Use the following to answer questions 80-84: Answer the next question(s) on the basis of the following production possibilities tables for two countries, North Cantina and South Cantina:
North Cantina Production possibilities B C D E 4 3 2 1 10 18 24 28

Capital goods Consumer goods

A 5 0

F 0 30

Capital goods Consumer goods

A 5 0

South Cantina Production possibilities B C D E 2 1 4 3 21 25 8 15

F 0 27

Type: T Topic: 4 E: 27 MI: 27 MA: 27 80. Refer to the above tables. If South Cantina is producing at production alternative D, the opportunity cost of the third unit of capital goods will be: A) 3 units of consumer goods. C) 5 units of consumer goods. B) 4 units of consumer goods. D) 6 units of consumer goods. Answer: D

Type: T Topic: 4 E: 27 MI: 27 MA: 27 81. Refer to the above tables. If North Cantina is producing at production alternative B, the opportunity cost of the eleventh unit of consumer goods will be: A) 10 units of capital goods. C) 8 units of capital goods. B) 1/4 of a unit of capital goods. D) 1/8 of a unit of capital goods. Answer: D

Type: T Topic: 4 E: 27 MI: 27 MA: 27 82. Refer to the above tables. Suppose that North Cantina is producing 2 units of capital goods and 17 units of consumer goods while South Cantina is producing 2 units of capital goods and 21 units of consumer goods. We can conclude that: A) North Cantina is fully and efficiently using its resources, but South Cantina is not. B) South Cantina is fully and efficiently using its resources, but North Cantina is not. C) neither South Cantina nor North Cantina are fully and efficiently using their resources. D) both South Cantina and North Cantina are fully and efficiently using their resources. Answer: B

Type: C Topic: 4 E: 27 MI: 27 MA: 27 83. Refer to the above tables. Suppose that resources in North Cantina and South Cantina are identical in quantity and quality. We can conclude that: A) South Cantina has better technology than North Cantina in producing both capital and consumer goods. B) North Cantina has better technology than South Cantina in producing both capital and consumer goods. C) North Cantina is growing more rapidly than South Cantina. D) North Cantina has better technology than South Cantina in producing consumer goods, but not capital goods. Answer: D

McConnell/Brue: Economics, 16/e

Page 47

Chapter 2: The Economizing Problem

Type: T Topic: 4 E: 27 MI: 27 MA: 27 84. Refer to the above tables. The opportunity cost of the fifth unit of capital goods: A) is higher in North Cantina than in South Cantina. B) is the same in North Cantina and South Cantina. C) is lower in North Cantina than in South Cantina. D) cannot be determined from the information provided. Answer: A

Type: A Topic: 4 E: 26 MI: 26 MA: 26 85. If an economy is operating inside its production possibilities curve for consumer goods and capital goods, it: A) can only produce more consumer goods by producing fewer capital goods. B) can only produce more capital goods by producing fewer consumer goods. C) can produce more of both consumer goods and capital goods by using its resources more efficiently. D) must improve its technology to produce more output. Answer: C

Use the following to answer questions 86-92:

Type: G Topic: 4 E: 26 MI: 26 MA: 26 86. Refer to the above diagram. Points A, B, C, D, and E show: A) that the opportunity cost of bicycles increases, while that of computers is constant. B) combinations of bicycles and computers that society can produce by using its resources efficiently. C) that the opportunity cost of computers increases, while that of bicycles is constant. D) that society's demand for computers is greater than its demand for bicycles. Answer: B

Type: G Topic: 4 E: 27 MI: 27 MA: 27 87. Refer to the above diagram. This production possibilities curve is: A) convex to the origin because opportunity costs are constant. B) linear because opportunity costs are constant. C) concave to the origin because of increasing opportunity costs. D) convex to the origin because of increasing opportunity costs. Answer: C

McConnell/Brue: Economics, 16/e

Page 48

Chapter 2: The Economizing Problem

Type: G Topic: 4 E: 27 MI: 27 MA: 27 88. Refer to the above diagram. If society is currently producing 9 units of bicycles and 4 units of computers and it now decides to increase computer output to 6, the cost: A) will be 4 units of bicycles. B) will be 2 units of bicycles. C) will be zero because unemployed resources are available. D) of doing so cannot be determined from the information given. Answer: A

Type: G Topic: 4 E: 26 MI: 26 MA: 26 89. Refer to the above diagram. The combination of computers and bicycles shown by point G is: A) attainable, but too costly. B) unattainable, given currently available resources and technology. C) attainable, but involves unemployment. D) irrelevant because it is inconsistent with consumer preferences. Answer: B

Type: G Topic: 4 E: 27 MI: 27 MA: 27 90. Refer to the above diagram. If society is currently producing the combination of bicycles and computers shown by point D, the production of 2 more units of bicycles: A) cannot be achieved because resources are fully employed. B) will cost 1 unit of computers. C) will cost 2 units of computers. D) will cause some resources to become unemployed. Answer: B

Type: G Topic: 4 E: 27 MI: 27 MA: 27 91. Refer to the above diagram. The combination of computers and bicycles shown by point F: A) is unattainable, given currently available resources and technology. B) is attainable, but entails economic inefficiency. C) is irrelevant because it is inconsistent with consumer preferences. D) suggests that opportunity costs are constant. Answer: B

Type: G Topic: 4 E: 27 MI: 27 MA: 27 92. Refer to the above diagram. The movement down the production possibilities curve from point A to point E suggests that the production of: A) computers, but not bicycles, is subject to increasing opportunity costs. B) bicycles, but not computers, is subject to increasing opportunity costs. C) both bicycles and computers are subject to constant opportunity costs. D) both bicycles and computers are subject to increasing opportunity costs. Answer: D

McConnell/Brue: Economics, 16/e

Page 49

Chapter 2: The Economizing Problem

Use the following to answer questions 93-95:

Capital goods

A

B

D

C

0

Consumer goods

Type: G Topic: 4 E: 27 MI: 27 MA: 27 93. Refer to the above diagram. As it relates to production possibilities analysis, the law of increasing opportunity cost is reflected in curve: A) A B) B C) C D) D Answer: C

Type: G Topic: 4 E: 27 MI: 27 MA: 27 94. Refer to the above diagram. Curve B is a: A) production possibilities curve indicating constant opportunity costs. B) production possibilities curve indicating increasing opportunity costs. C) demand curve indicating that the quantity of consumer goods demanded increases as the price of capital falls. D) technology frontier curve. Answer: A

Type: G Topic: 4 E: 27 MI: 27 MA: 27 95. Refer to the above diagram. Curve C A) reflects increasing opportunity costs because the slope of the curve becomes less steep as one moves down along the curve. B) is a less desirable production possibilities curve for an economy than curve B. C) is a more desirable production possibilities curve for an economy than curve A. D) has a steeper slope throughout than curve B. Answer: C

McConnell/Brue: Economics, 16/e

Page 50

Chapter 2: The Economizing Problem

Opportunity costs

Type: A Topic: 5 E: 27 MI: 27 MA: 27 96. In deciding whether to study for an economics quiz or go to a movie, one is confronted by the idea(s) of: A) scarcity and opportunity costs. C) complementary economic goals. B) money and real capital. D) full production. Answer: A

Type: D Topic: 5 E: 27 MI: 27 MA: 27 97. Which one of the following expressions best states the idea of opportunity cost? A) "A penny saved is a penny earned." C) "There is no such thing as a free lunch." B) "He who hesitates is lost." D) "All that glitters is not gold." Answer: C

Type: A Topic: 5 E: 27 MI: 27 MA: 27 98. The fact that the slope of the production possibilities curve becomes steeper as we move down along the curve indicates that: A) the principle of increasing opportunity costs is relevant. B) society's resources are limited. C) the opportunity cost of producing each product is constant. D) resources are perfectly shiftable between alternative uses. Answer: A

Type: C Topic: 5 E: 27 MI: 27 MA: 27 99. The idea of opportunity cost: A) applies to consumers, but not to businesses. B) applies to businesses, but not to consumers. C) is relevant to economies of all ideological persuasions. D) would disappear if we were able to eliminate poverty. Answer: C

Type: A Topic: 5 E: 27 MI: 27 MA: 27 100. Which of the following is an economic explanation for why most college-aged movie stars do not attend college. A) they are too dumb to get into college B) they would find college life boring C) the opportunity cost in terms of reduced income is too great D) they cannot afford the room, board, and tuition fees most colleges charge. Answer: C

McConnell/Brue: Economics, 16/e

Page 51

Chapter 2: The Economizing Problem

Type: D Topic: 5 E: 27 MI: 27 MA: 27 101. The law of increasing opportunity costs states that: A) if society wants to produce more of a particular good, it must sacrifice larger and larger amounts of other goods to do so. B) the sum of the costs of producing a particular good cannot rise above the current market price of that good. C) if the sum of the costs of producing a particular good rises by a specified percent, the price of that good must rise by a greater relative amount. D) if the prices of all the resources used to produce goods increase, the cost of producing any particular good will increase at the same rate. Answer: A

Type: D Topic: 5 E: 27 MI: 27 MA: 27 102. The concept of opportunity cost: A) is irrelevant in socialistic economies because of central planning. B) suggests that the use of resources in any particular line of production means that alternative outputs must be forgone. C) is irrelevant if the production possibilities curve is shifting to the right. D) suggests that insatiable wants can be fulfilled. Answer: B

Type: A Topic: 5 E: 27 MI: 27 MA: 27 103. The law of increasing opportunity costs exists because: A) resources are not equally efficient in producing various goods. B) the value of the dollar has diminished historically because of persistent inflation. C) wage rates invariably rise as the economy approaches full employment. D) consumers tend to value any good more highly when they have little of it. Answer: A

Type: A Topic: 5 E: 27 MI: 27 MA: 27 104. The law of increasing opportunity costs is reflected in a production possibilities curve that is: A) an upsloping straight line. C) concave to the origin. B) a downsloping straight line. D) convex to the origin. Answer: C

Type: D Topic: 5 E: 27 MI: 27 MA: 27 105. Opportunity cost is best defined as: A) the monetary price of any productive resource. B) the amount of labor that must be used to produce one unit of any product. C) the ratio of the prices of imported goods to the prices of exported goods. D) the amount of one product that must be given up to produce one more unit of another product. Answer: D

McConnell/Brue: Economics, 16/e

Page 52

Chapter 2: The Economizing Problem

Allocative efficiency

Type: D Topic: 6 E: 27 MI: 27 MA: 27 106. Allocative efficiency is concerned with: A) producing the combination of goods most desired by society. B) achieving the full employment of all available resources. C) producing every good with the least-cost combination of inputs. D) reducing the concavity of the production possibilities curve. Answer: A

Type: D Topic: 6 E: 27 MI: 27 MA: 27 107. Allocative efficiency involves determining: A) which output-mix will result in the most rapid rate of economic growth. B) which production possibilities curve reflects the lowest opportunity costs. C) the point on the production possibilities curve that will maximize society's satisfaction. D) the optimal rate of technological progress. Answer: C

Type: D Topic: 6 E: 27 MI: 27 MA: 27 108. The point on the production possibilities curve that produces allocative efficiency can be found by: A) estimating the marginal costs of both products in real or physical terms. B) comparing marginal benefits and marginal costs in monetary terms. C) determining where least-cost production occurs. D) calculating where economic growth will be greatest. Answer: B

Type: D Topic: 6 E: 27 MI: 27 MA: 27 109. Allocative efficiency refers to: A) the use of the least-cost method of production. B) the production of the product-mix most wanted by society. C) the full employment of all available resources. D) production at some point inside of the production possibilities curve. Answer: B

Type: D Topic: 6 E: 27 MI: 27 MA: 27 110. The optimal or allocatively efficient point on a production possibilities curve is achieved where: A) the smallest physical amounts of inputs are used to produce each good. B) each good is produced at a level where marginal benefits equal marginal costs. C) large amounts of capital goods are produced relative to consumer goods. D) large amounts of consumer goods are produced relative to capital goods. Answer: B

Type: D Topic: 6 E: 28 MI: 28 MA: 28 111. The marginal benefit curve is: A) upsloping because of increasing marginal opportunity costs. B) upsloping because successive units of a specific product yield less and less extra utility. C) downsloping because of increasing marginal opportunity costs. D) downsloping because successive units of a specific product yield less and less extra utility. Answer: D

McConnell/Brue: Economics, 16/e

Page 53

Chapter 2: The Economizing Problem

Type: D Topic: 6 E: 28 MI: 28 MA: 28 112. The marginal cost curve is: A) upsloping because of increasing marginal opportunity costs. B) upsloping because successive units of a specific product yield less and less extra utility. C) downsloping because of increasing marginal opportunity costs. D) downsloping because successive units of a specific product yield less and less extra utility. Answer: A

Type: A Topic: 6 E: 28 MI: 28 MA: 28 113. The output of compact disc players should be: A) reduced if marginal benefits exceed marginal costs. B) reduced if marginal costs exceed marginal benefits. C) increased if marginal costs exceed marginal benefits. D) reduced to zero if their unit costs exceed the unit costs of alternative products. Answer: B

Type: D Topic: 6 E: 28 MI: 28 MA: 28 114. If the output of product X is such that marginal benefit equals marginal cost: A) the correct amount of resources is being allocated to X's production. B) the value of producing X exceeds the value of producing alternative products with the available resources. C) there can be a net gain to society by allocating either more or less resources to producing X. D) resources are overallocated to the production of X. Answer: A

Use the following to answer questions 115-117:

Marginal benefit and marginal cost (dollars)

Marginal Cost

Marginal Benefit Q1 Q2 Q3 Quantity of shoes

0

Type: A Topic: 6 E: 28 MI: 28 MA: 28 115. Refer to the above diagram for athletic shoes. The optimal output of shoes is: A) Q1. B) Q2. C) Q3. D) greater than Q3. Answer: B

McConnell/Brue: Economics, 16/e

Page 54

Chapter 2: The Economizing Problem

Type: A Topic: 6 E: 28 MI: 28 MA: 28 116. Refer to the above diagram for athletic shoes. If the current output of shoes is Q1, then: A) society would consider additional units of shoes to be more valuable than alternative products. B) society would consider additional units of shoes to be less valuable than alternative products. C) society would experience a net loss by producing more shoes. D) resources are being allocated efficiently to the production of shoes. Answer: A

Type: A Topic: 6 E: 28 MI: 28 MA: 28 117. Refer to the above diagram for athletic shoes. If the current output of shoes is Q3, then: A) resources are being allocated efficiently to the production of shoes. B) society would consider additional units of shoes to be more valuable than alternative products. C) society would consider additional units of shoes to be less valuable than alternative products. D) society would experience a net gain by producing more shoes. Answer: C

Economic growth and decline

Use the following to answer questions 118-120:

E
Capital goods

C A G

0

D H B Consumer Goods

F

Type: G Topic: 7 E: 30 MI: 30 MA: 30 118. Technological advance in producing both capital goods and consumer goods is shown by the shift of the production possibilities curve from AB to: A) CD. B) EB. C) AF. D) GH. Answer: A

Type: G Topic: 7 E: 30 MI: 30 MA: 30 119. Technological advance that is useful in producing capital goods but not in producing consumer goods is shown by the shift of the production possibilities curve from AB to: A) CD. B) BE. C) AF. D) GH. Answer: B

McConnell/Brue: Economics, 16/e

Page 55

Chapter 2: The Economizing Problem

Type: G Topic: 7 E: 30 MI: 30 MA: 30 120. Technological advance that is useful in producing consumer goods but not in producing capital goods is shown by the shift of the production possibilities curve from AB to: A) CD. B) EB. C) AF. D) GH. Answer: C

Type: D Topic: 7 E: 25 MI: 25 MA: 25 121. The basic difference between consumer goods and capital goods is that A) consumer goods are produced in the private sector and capital goods are produced in the public sector. B) an economy that commits a relatively large proportion of its resources to capital goods must accept a lower growth rate. C) the production of capital goods is not subject to the law of increasing opportunity costs. D) consumer goods satisfy wants directly while capital goods satisfy wants indirectly. Answer: D

Type: A Topic: 7 E: 32 MI: 32 MA: 32 122. Which of the following would be most likely to shift the production possibilities curve to the right? A) a sudden and substantial expansion of consumer wants B) an improvement in the literacy level and general level of education C) a decline in the size of the population and labor force D) shifting resources from the production of capital goods to the production of consumer goods. Answer: B

Type: D Topic: 7 E: 32 MI: 32 MA: 32 123. Which of the following will shift the production possibilities curve to the right? A) an increase in the unemployment rate from 6 to 8 percent B) a decline in the efficiency with which the present labor force is allocated C) a decrease in the unemployment rate from 8 to 6 percent D) a technological advance that allows farmers to produce more output from given inputs Answer: D

Type: C Topic: 7 E: 32 MI: 32 MA: 32 124. Other things equal, which of the following would shift an economy's production possibilities curve to the left? A) the discovery of a low-cost means of generating and storing solar energy B) the entrance of more women into the labor force C) a law requiring mandatory retirement from the labor force at age 55 D) an increase in the proportion of total output that consists of capital or investment goods Answer: C

McConnell/Brue: Economics, 16/e

Page 56

Chapter 2: The Economizing Problem

Use the following to answer questions 125-129:

Type: G Topic: 7 E: 27 MI: 27 MA: 27 125. Refer to the above diagram. The concave shape of each production possibilities curve indicates that: A) resources are perfectly substitutable. B) wants are virtually unlimited. C) prices are constant. D) resources are not equally suited for alternative uses. Answer: D

Type: G Topic: 7 E: 27 MI: 27 MA: 27 126. Refer to the above diagram. The concept of opportunity cost is best represented by the: A) shift of the production possibilities curve from PP1 to PP2. B) move from B on PP1 to E on PP2. C) move from B on PP1 to C on PP1. D) move from D inside PP1 to B on PP1. Answer: C

Type: C Topic: 7 E: 31 MI: 31 MA: 31 127. Refer to the above diagram. Which of the following positions relative to PP1 would be the most likely to result in a future production possibilities curve of PP3, rather than PP2 ? A) A. B) B. C) C. D) D. Answer: A

Type: G Topic: 7 E: 30 MI: 30 MA: 30 128. Refer to the above diagram. An improvement in technology will: A) shift the production possibilities curve from PP1 to PP2. B) shift the production possibilities curve from PP2 to PP1. C) move the economy from A to C along PP1. D) move the economy from A, B, or C on PP1 to D. Answer: A

McConnell/Brue: Economics, 16/e

Page 57

Chapter 2: The Economizing Problem

Type: G Topic: 7 E: 30 MI: 30 MA: 30 129. Refer to the above diagram. Which one of the following would shift the production possibilities curve from PP1 to PP2? A) immigration of skilled workers into the economy B) worsening of the AIDS epidemic C) an increase in consumer prices D) a reduction in hourly wages Answer: A

Type: D Topic: 7 E: 31 MI: 31 MA: 31 130. Which of the following statements, if any, is correct for a nation that is producing only consumption and capital goods? A) Other things equal, the more consumer goods a nation produces, the greater will be its future growth rate. B) Other things equal, the more capital goods a nation produces, the greater will be its future growth rate. C) There is no general relationship between the current division of output between consumer and capital goods and the future growth rate. D) None of the above statements is correct. Answer: B

Type: A Topic: 7 E: 32 MI: 32 MA: 32 131. All of the following could immediately or eventually lead to an inward shift of a nation's production possibilities curve, except: A) emigration of skilled workers from the nation. B) a decline in the birth rate. C) an increase in the average skill level of all occupational groups. D) depletion and reduced availability of major energy resources. Answer: C

Type: A Topic: 7 E: 32 MI: 32 MA: 32 132. A nation's production possibilities curve might shift to the left (inward) as a result of: A) technological advance. B) increases in the size of the labor force. C) the depletion of its soil fertility due to overplanting and overgrazing. D) investing in more capital goods. Answer: C

Applications

Type: A Topic: 8 E: 32 MI: 32 MA: 32 133. Which of the following might shift a nation's production possibilities curve inward? A) improved technology. B) devastation by war. C) improved health care. D) a business downturn in which unemployment temporarily rises. Answer: B

McConnell/Brue: Economics, 16/e

Page 58

Chapter 2: The Economizing Problem

Type: A Topic: 8 E: 32 MI: 32 MA: 32 134. Suppose that a university decides to spend $1 million to upgrade personal computers and scientific equipment for faculty rather than spend $1 million to expand parking for students. This example illustrates: A) distorted priorities. C) increasing opportunity costs. B) opportunity costs. D) productive efficiency. Answer: B

Type: A Topic: 8 E: 32 MI: 32 MA: 32 135. Which of the following most closely relates to the idea of opportunity costs? A) tradeoffs B) economic growth C) technological change D) capitalism Answer: A

Type: A Topic: 8 E: 32 MI: 32 MA: 32 136. Which of the following will enable a nation to obtain a combination of consumer goods and capital goods outside its production possibilities curve? A) full employment C) full production B) international specialization and trade D) productive efficiency. Answer: B

Type: C Topic: 8 E: 32 MI: 32 MA: 32 137. Suppose that Zualia, which has full employment and full production, can obtain 1 unit of capital goods by sacrificing 2 units of consumer goods domestically, but can obtain 1 unit of capital goods from another country by trading 1 unit of consumer goods for it. This reality illustrates: A) a rightward (outward) shift of the production possibilities curve. B) increasing opportunity costs. C) achieving points beyond the production possibilities curve through international specialization and trade. D) productive efficiency. Answer: C

Type: A Topic: 8 E: 31 MI: 31 MA: 31 138. Through specialization and international trade a nation: A) can attain some combination of goods lying outside its production possibilities curve. B) can move from a high consumption-low investment to a high investment-low consumption point on its production possibilities curve. C) will attain some combination of goods lying within its production possibilities curve. D) will cause its production possibilities curve to shift leftward. Answer: A

Type: A Topic: 8 E: 32 MI: 32 MA: 32 139. Some agricultural sub-Saharan nations of Africa have overfarmed and overgrazed their land to the extent that significant portions of it have turned into desert. This suggests that: A) the concavity of the production possibilities curves of such nations has increased. B) the production possibilities curves of such nations have shifted inward. C) the production possibilities curves of such nations have shifted outward. D) these nations are operating at some point outside of their production possibilities curves. Answer: B

McConnell/Brue: Economics, 16/e

Page 59

Chapter 2: The Economizing Problem

Type: A Topic: 8 E: 32 MI: 32 MA: 32 140. If all discrimination in the United States were eliminated, the economy would: A) have a less concave production possibilities curve. B) produce at some point closer to its production possibilities curve. C) be able to produce at some point outside of its production possibilities curve. D) produce more consumer goods and fewer investment goods. Answer: B

Economic systems

Type: F Topic: 9 E: 33 MI: 33 MA: 33 141. The two general types of economic systems that exist today are: A) market systems and capitalism. B) socialism and central planning. C) market systems and command systems. D) laissez faire systems and pure command systems. Answer: C

Type: D Topic: 9 E: 33 MI: 33 MA: 33 142. Which of the following is a distinguishing feature of a command system? A) private ownership of all capital. C) heavy reliance on markets. B) central planning. D) wide-spread dispersion of economic power. Answer: B

Type: D Topic: 9 E: 33 MI: 33 MA: 33 143. Which of the following is a distinguishing feature of a market system? A) public ownership of all capital. C) wide-spread private ownership of capital. B) central planning. D) a circular flow of goods, resources, and money. Answer: C

Type: A Topic: 9 E: 33 MI: 33 MA: 33 144. Examples of command economies are: A) the United States and Japan. B) Sweden and Norway. Answer: D

C) Mexico and Brazil. D) Cuba and North Korea.

Type: A Topic: 9 E: 33 MI: 33 MA: 33 145. Of the following countries, the one that best exhibits the characteristics of a market economy is: A) Canada. B) Cuba. C) North Korea. D) China. Answer: A

Type: D Topic: 9 E: 33 MI: 33 MA: 33 146. The French term "laissez-faire" means: A) "there is no free lunch." B) "let it be." C) "circular flow." D) "public ownership." Answer: B

McConnell/Brue: Economics, 16/e

Page 60

Chapter 2: The Economizing Problem

Type: D Topic: 9 E: 33 MI: 33 MA: 33 147. An economic system: A) requires a grouping of private markets linked to one another. B) is a particular set of institutional arrangements and a coordinating mechanism used to respond to the economizing problem. C) requires some sort of centralized authority (such as government) to coordinate economic activity. D) is a plan or scheme that allows a firm to make money at some other firm's expense. Answer: B

Type: D Topic: 9 E: 33 MI: 33 MA: 33 148. The term laissez faire suggests that: A) land and other natural resources should be privately owned, but capital should be publicly owned. B) land and other natural resources should be publicly owned, but capital equipment should be privately owned. C) government should not interfere with the operation of the economy. D) government action is necessary if the economy is to achieve full employment and full production. Answer: C

Type: D Topic: 9 E: 33 MI: 33 MA: 33 149. Economic scarcity: A) is peculiar to the United States economy. B) applies to all economies. Answer: B

C) is peculiar to command systems. D) is peculiar to market systems.

Circular flow model

Type: D Topic: 10 E: 34 MI: 34 MA: 34 150. The simple circular flow model shows that: A) households are on the buying side of both product and resource markets. B) businesses are on the selling side of both product and resource markets. C) households are on the selling side of the resource market and on the buying side of the product market. D) businesses are on the buying side of the product market and on the selling side of the resource market. Answer: C

Type: D Topic: 10 E: 34 MI: 34 MA: 34 151. The two basic markets shown by the simple circular flow model are: A) capital goods and consumer goods. C) product and resource. B) free and controlled. D) household and business. Answer: C

Type: D Topic: 10 E: 36 MI: 36 MA: 36 152. In the resource market: A) businesses borrow financial capital from households. B) businesses sell services to households. C) households sell resources to businesses. D) firms sell raw materials to households. Answer: C

McConnell/Brue: Economics, 16/e

Page 61

Chapter 2: The Economizing Problem

Type: A Topic: 10 E: 34 MI: 34 MA: 34 153. Which of the following is a limitation of the simple circular flow model? A) product markets are ignored. B) resource markets are ignored. C) The determination of product and resource prices is not explained. D) households are included, but not businesses. Answer: C

Type: D Topic: 10 E: 36 MI: 36 MA: 36 154. In the simple circular flow model: A) households are buyers of resources. B) businesses are sellers of final products. C) households are sellers of final products. D) there are real flows of goods, services, and resources, but not money flows. Answer: B

Use the following to answer questions 155-158:

Type: A Topic: 10 E: 34 MI: 34 MA: 34 155. Refer to the above diagram. Flow (1) represents: A) wage, rent, interest, and profit income. B) land, labor, capital, and entrepreneurial ability. Answer: A

C) goods and services. D) consumer expenditures.

Type: A Topic: 10 E: 34 MI: 34 MA: 34 156. Refer to the above diagram. Flow (2) represents: A) wage, rent, interest, and profit income. B) land, labor, capital, and entrepreneurial ability. Answer: B

C) goods and services. D) consumer expenditures.

Type: A Topic: 10 E: 34 MI: 34 MA: 34 157. Refer to the above diagram. Flow (3) represents: A) wage, rent, interest, and profit income. B) land, labor, capital, and entrepreneurial ability. Answer: C

C) goods and services. D) consumer expenditures.

McConnell/Brue: Economics, 16/e

Page 62

Chapter 2: The Economizing Problem

Type: A Topic: 10 E: 34 MI: 34 MA: 34 158. Refer to the above diagram. Flow (4) represents: A) wage, rent, interest, and profit income. B) land, labor, capital, and entrepreneurial ability. Answer: D

C) goods and services. D) consumer expenditures.

Type: D Topic: 10 E: 36 MI: 36 MA: 36 159. In terms of the circular flow diagram, households make expenditures in the _____ market and receive income through the _____ market. A) product; financial B) resource; product C) product; resource D) capital; product Answer: C

Type: D Topic: 10 E: 36 MI: 36 MA: 36 160. In terms of the circular flow diagram, businesses obtain revenue through the _____ market and make expenditures in the _____ market. A) product; financial B) resource; product C) product; resource D) capital; product Answer: C

Type: D Topic: 10 E: 36 MI: 36 MA: 36 161. Households and businesses are: A) both buyers in the resource market. B) both sellers in the product market. C) sellers in the resource and product markets respectively. D) sellers in the product and resource markets respectively. Answer: C

Type: D Topic: 10 E: 36 MI: 36 MA: 36 162. In the circular flow model: A) households sell resources to firms. B) households receive income through the product market. C) households spend income in the resource market. D) businesses neither buy nor sell resources. Answer: A

McConnell/Brue: Economics, 16/e

Page 63

Chapter 2: The Economizing Problem

Use the following to answer questions 163-165:

Type: A Topic: 10 E: 34 MI: 34 MA: 34 163. Refer to the above diagram. Arrows (1) and (2) represent: A) goods and resources respectively. C) output and money incomes respectively. B) money incomes and output respectively. D) resources and goods respectively. Answer: D

Type: A Topic: 10 E: 34 MI: 34 MA: 34 164. Refer to the above diagram. Arrows (3) and (4) represent: A) goods and services respectively. B) incomes and consumer expenditures respectively. C) resources and goods respectively. D) consumer expenditures and income respectively. Answer: B

Type: A Topic: 10 E: 36 MI: 36 MA: 36 165. Refer to the above diagram. Arrows (1) and (3) are associated with: A) the money market. B) the resource market. C) the product market. Answer: B

D) international trade.

Consider This Questions

Type: D E: 32 MI: 32 MA: 32 Status: New 166. (Consider This) A direct cost of going to college is: A) tuition, while an indirect cost (opportunity cost) is books and other supplies. B) forgone income while in college, while an indirect cost (opportunity cost) is tuition. C) tuition, while an indirect cost (opportunity cost) is forgone income while in college. D) books and supplies, while an indirect cost (opportunity cost) is food and housing. Answer: C

McConnell/Brue: Economics, 16/e

Page 64

Chapter 2: The Economizing Problem

Type: A E: 32 MI: 32 MA: 32 Status: New 167. (Consider This) An exception to the advice "go to college, stay in college, and earn a degree" occurs when: A) tuition expenses are high and rising. B) the opportunity cost of attending college is extraordinarily high. C) the price of textbooks is high and rising. D) the economy is growing rapidly and jobs are plentiful. Answer: B

Last Word Questions

Use the following to answer questions 168-170:

C A x Civilian goods y 0

Defense goods

B

D

Type: G E: 35 MI: 35 MA: 35 Status: New 168. (Last Word) Refer to the above diagram. The direct economic impact of the destruction and loss of lives caused by the terrorist attacks of September 11, 2001 is illustrated by the: A) shift of the production possibilities curve from CD to AB. B) shift of the production possibilities curve from AB to CD. C) move from x to y on production possibilities curve AB. D) move from y to x on production possibilities curve AB. Answer: A

Type: G E: 35 MI: 35 MA: 35 Status: New 169. (Last Word) Refer to the above diagram. The U.S. response to the events of September 11, 2001, is illustrated by the: A) shift of the production possibilities curve from CD to AB. B) shift of the production possibilities curve from AB to CD. C) move from x to y on production possibilities curve AB. D) move from y to x on production possibilities curve AB. Answer: C

McConnell/Brue: Economics, 16/e

Page 65

Chapter 2: The Economizing Problem

Type: G E: 35 MI: 35 MA: 35 Status: New 170. (Last Word) Refer to the above diagram. Suppose that point y represents the optimal combination of civilian goods and defense goods. We can conclude that at y the marginal benefit of defense goods: A) exceeds the marginal cost of defense goods. C) is zero. B) equals the marginal cost of defense goods. D) is negative. Answer: B

True/False Questions

Type: D E: 24 MI: 24 MA: 24 171. Full production means that all available workers have jobs. Answer: False

Type: D E: 26 MI: 26 MA: 26 172. The production possibilities curve shows various combinations of two products that an economy can produce when achieving full employment and productive efficiency. Answer: True

Type: D E: 23 MI: 23 MA: 23 173. The entrepreneur's sole function is to combine other resources (land, labor, and capital) in the production of some good or service. Answer: False

Type: D E: 23 MI: 23 MA: 23 174. Products and services are scarce because resources are scarce. Answer: True

Type: D E: 26 MI: 26 MA: 26 175. An economy cannot produce at a point outside of its production possibilities curve because human economic wants are insatiable. Answer: False

Type: D E: 23 MI: 23 MA: 23 176. The process by which capital goods are accumulated is known as investment. Answer: True

Type: D E: 30 MI: 30 MA: 30 177. The present choice of position on the production possibilities curve will not influence the future location of the curve. Answer: False

Type: A E: 27 MI: 27 MA: 27 178. Although sleeping in on a work day or school day has an opportunity cost, sleeping late on the weekend does not. Answer: False

McConnell/Brue: Economics, 16/e

Page 66

Chapter 2: The Economizing Problem

Use the following to answer questions 179-182:

Type: A Topic: 5 E: 26 MI: 26 MA: 26 179. Refer to the above diagram. Given production possibilities curve (a), the combination of civilian and war goods indicated by point X is unattainable to this economy. Answer: False

Type: G E: 26 MI: 26 MA: 26 180. Refer to the above diagram. Given production possibilities curve (a), point Y indicates that society is failing to use available resources efficiently. Answer: False

Type: G E: 30 MI: 30 MA: 30 181. Refer to the above diagram. The movement from curve (a) to curve (b) implies an increase in the quantity and/or quality of society's productive resources. Answer: True

Type: G E: 30 MI: 30 MA: 30 182. Refer to the above diagram. The movement from curve (a) to curve (c) indicates an improvement in civilian goods technology but not in war goods technology. Answer: False

Type: G E: 29 MI: 29 MA: 29 183. An economy will always operate at some point on its production possibilities curve. Answer: False

Type: G E: 33 MI: 33 MA: 33 184. The increasing wealth of the United States has reduced the relevance of economics. Answer: False

McConnell/Brue: Economics, 16/e

Page 67

CHAPTER 3

Individual Markets: Demand and Supply

Topic 1. Demand and demand curve 2. Determinants of demand 3. Change in demand versus change in quantity demanded 4. Supply and supply curve 5. Determinants of supply 6. Equilibrium; rationing function 7. Changes in equilibrium price and quantity 8. Government-set prices Consider This Last Word True-False Multiple Choice Questions Demand and demand curve

Question numbers 1-18 19-55 56-69 70-77 78-92 93-126 127-174 175-194 195-196 197-200 201-215

____________________________________________________________

_______________________________________

____________________________________________________________

_______________________________________

Type: D Topic: 1 E: 39 MI: 39 MA: 39 1. A market: A) reflects upsloping demand and downsloping supply curves. B) entails the exchange of goods, but not services. C) is an institution that brings together buyers and sellers. D) always requires face-to-face contact between buyer and seller. Answer: C

Type: D Topic: 1 E: 39 MI: 39 MA: 39 2. Markets explained on the basis of supply and demand: A) assume many buyers and many sellers of a standardized product. B) assume market power so that buyers and sellers bargain with one another. C) do not exist in the real-world economy. D) are approximated by markets in which a single seller determines price. Answer: A

Type: D Topic: 1 E: 40 MI: 40 MA: 40 3. The law of demand states that: A) price and quantity demanded are inversely related. B) the larger the number of buyers in a market, the lower will be product price. C) price and quantity demanded are directly related. D) consumers will buy more of a product at high prices than at low prices. Answer: A

Chapter 3: Individual Markets: Demand and Supply

Type: D Topic: 1 E: 41 MI: 41 MA: 41 4. Graphically, the market demand curve is: A) steeper than any individual demand curve that is part of it. B) greater than the sum of the individual demand curves. C) the horizontal sum of individual demand curves. D) the vertical sum of individual demand curves. Answer: C

Type: D Topic: 1 E: 41 MI: 41 MA: 41 5. The demand curve shows the relationship between: A) money income and quantity demanded. B) price and production costs. Answer: C

C) price and quantity demanded. D) consumer tastes and the quantity demanded.

Type: D Topic: 1 E: 40 MI: 40 MA: 40 6. Economists use the term demand to refer to: A) a particular price-quantity combination on a stable demand curve. B) the total amount spent on a particular commodity over a stipulated time period. C) an upsloping line on a graph that relates consumer purchases and product price. D) a schedule of various combinations of market prices and amounts demanded. Answer: D

Type: A Topic: 1 E: 40 MI: 40 MA: 40 7. The relationship between quantity supplied and price is _____ and the relationship between quantity demanded and price is _____. A) direct, inverse B) inverse, direct C) inverse, inverse D) direct, direct Answer: A

Type: D Topic: 1 E: 40 MI: 40 MA: 40 8. When the price of a product increases, a consumer is able to buy less of it with a given money income. This describes: A) the cost effect. B) the inflationary effect. C) the income effect. D) the substitution effect. Answer: C

Type: D Topic: 1 E: 41 MI: 41 MA: 41 9. A demand curve: A) shows the relationship between price and quantity supplied. B) indicates the quantity demanded at each price in a series of prices. C) graphs as an upsloping line. D) shows the relationship between income and spending. Answer: B

Type: D Topic: 1 E: 41 MI: 41 MA: 41 10. In presenting the idea of a demand curve economists presume that the most important variable in determining the quantity demanded is: A) the price of the product itself. C) the prices of related goods. B) consumer income. D) consumer tastes. Answer: A

McConnell/Brue: Economics, 16/e

Page 70

Chapter 3: Individual Markets: Demand and Supply

Type: D Topic: 1 E: 41 MI: 41 MA: 41 11. An increase in the price of a product will reduce the amount of it purchased because: A) supply curves are upsloping. B) the higher price means that real incomes have risen. C) consumers will substitute other products for the one whose price has risen. D) consumers substitute relatively high-priced for relatively low-priced products. Answer: C

Type: A Topic: 1 E: 40-41 MI: 40-41 MA: 40-41 12. The income and substitution effects account for: A) the upward sloping supply curve. C) movements along a given supply curve. B) the downward sloping demand curve. D) the "other things equal" assumption. Answer: B

Type: D Topic: 1 E: 41 MI: 41 MA: 41 13. When the price of a product rises, consumers shift their purchases to other products whose prices are now relatively lower. This statement describes: A) an inferior good. C) the substitution effect. B) the rationing function of prices. D) the income effect. Answer: C

Type: D Topic: 1 E: 40 MI: 40 MA: 40 14. When the price of a product falls, the purchasing power of our money income rises and thus permits consumers to purchase more of the product. This statement describes: A) an inferior good. C) the substitution effect. B) the rationing function of prices. D) the income effect. Answer: D

Type: G Topic: 1 E: 58 MI: 58 MA: 58 15.

(Advanced analysis) The equation for the demand curve in the above diagram: A) is P = 70 - Q. B) is P = 35 - 2Q. C) is P = 35 - .5Q. D) cannot be determined from the information given. Answer: C

McConnell/Brue: Economics, 16/e

Page 71

Chapter 3: Individual Markets: Demand and Supply

Type: D Topic: 1 E: 41 MI: 41 MA: 41 16. When product prices change, consumers are inclined to purchase larger amounts of the now cheaper products and less of the now more expensive products. This describes: A) the cost effect. B) the price effect. C) the income effect. D) the substitution effect. Answer: D

Type: D Topic: 1 E: 40, 45 MI: 40, 45 MA: 40, 45 17. The construction of demand and supply curves assumes that the primary variable influencing decisions to produce and purchase goods is: A) price. B) expectations. C) preferences. D) incomes. Answer: A

Type: A Topic: 1 E: 40 MI: 40 MA: 40 18. One reason that the quantity of a good demanded increases when its price falls is that the: A) price decline shifts the supply curve to the left. B) lower price shifts the demand curve to the left. C) lower price shifts the demand curve to the right. D) lower price increases the real incomes of buyers, enabling them to buy more. Answer: D

Determinants of demand

Type: C Topic: 2 E: 44 MI: 44 MA: 44 19. A recent study found that an increase in the Federal tax on beer (and thus an increase in the price of beer) would reduce the demand for marijuana. We can conclude that: A) beer and marijuana are substitute goods. C) beer is an inferior good. B) beer and marijuana are complementary goods. D) marijuana is an inferior good. Answer: B

Type: A Topic: 2 E: 43 MI: 43 MA: 43 20. In the past few years, the demand for donuts has greatly increased. This increase in demand might best be explained by: A) an increase in consumer income. C) a change in consumer expectations. B) an increase in the price of a substitute good. D) a change in buyer tastes. Answer: D

Type: A Topic: 2 E: 44-45 MI: 44-45 MA: 44-45 21. Which of the following will not cause the demand for product K to change? A) a change in the price of close-substitute product J B) an increase in consumer incomes C) a change in the price of K D) a change in consumer tastes Answer: C

McConnell/Brue: Economics, 16/e

Page 72

Chapter 3: Individual Markets: Demand and Supply

Type: A Topic: 2 E: 44 MI: 44 MA: 44 22. Which of the following would not shift the demand curve for beef? A) a widely publicized study that indicates beef increases one's cholesterol B) a reduction in the price of cattle feed C) an effective advertising campaign by pork producers D) a change in the incomes of beef consumers Answer: B

Type: A Topic: 2 E: 44 MI: 44 MA: 44 23. In 2003 the price of oil increased, which in turn caused the price of natural gas to rise. This can best be explained by saying that oil and natural gas are: A) complementary goods and the higher price for oil increased the demand for natural gas. B) substitute goods and the higher price for oil increased the demand for natural gas. C) complementary goods and the higher price for oil decreased the supply of natural gas. D) substitute goods and the higher price for oil decreased the supply of natural gas. Answer: B

Type: A Topic: 2 E: 43 MI: 43 MA: 43 24. An economist for a bicycle company predicts that, other things equal, a rise in consumer incomes will increase the demand for bicycles. This prediction is based on the assumption that: A) there are many goods that are substitutes for bicycles. B) there are many goods that are complementary to bicycles. C) there are few goods that are substitutes for bicycles. D) bicycles are normal goods. Answer: D

Type: C Topic: 2 E: 44 MI: 44 MA: 44 25. A rightward shift in the demand curve for product C might be caused by: A) an increase in income if C is an inferior good. B) a decrease in income if C is a normal good. C) a decrease in the price of a product that is a close substitute for C. D) a decrease in the price of a product that is complementary to C. Answer: D

Type: D Topic: 2 E: 44 MI: 44 MA: 44 26. If two goods are complements: A) they are consumed independently. B) an increase in the price of one will increase the demand for the other. C) a decrease in the price of one will increase the demand for the other. D) they are necessarily inferior goods. Answer: C

Type: A Topic: 2 E: 44 MI: 44 MA: 44 27. DVD players and DVDs are: A) complementary goods. B) substitute goods. Answer: A

C) independent goods.

D) inferior goods.

McConnell/Brue: Economics, 16/e

Page 73

Chapter 3: Individual Markets: Demand and Supply

Type: A Topic: 2 E: 44 MI: 44 MA: 44 28. If the demand curve for product B shifts to the right as the price of product A declines, then: A) both A and B are inferior goods. C) A is an inferior good and B is a superior good. B) A is a superior good and B is an inferior good. D) A and B are complementary goods. Answer: D

Type: A Topic: 2 E: 44 MI: 44 MA: 44 29. If the price of product L increases, the demand curve for close-substitute product J will: A) shift downward toward the horizontal axis. C) shift to the right. B) shift to the left. D) remain unchanged. Answer: C

Type: A Topic: 2 E: 44 MI: 44 MA: 44 30. If the price of K declines, the demand curve for the complementary product J will: A) shift to the left. B) decrease. C) shift to the right. D) remain unchanged. Answer: C

Type: A Topic: 2 E: 43 MI: 43 MA: 43 31. Which of the following is most likely to be an inferior good? A) fur coats B) ocean cruises C) used clothing D) steak Answer: C

Type: A Topic: 2 E: 44 MI: 44 MA: 44 32. Which of the following statements is correct? A) An increase in the price of C will decrease the demand for complementary product D. B) A decrease in income will decrease the demand for an inferior good. C) An increase in income will reduce the demand for a normal good. D) A decline in the price of X will increase the demand for substitute product Y. Answer: A

Type: A Topic: 2 E: 43 MI: 43 MA: 43 33. A shift to the right in the demand curve for product A can be most reasonably explained by saying that: A) consumer incomes have declined and they now want to buy less of A at each possible price. B) the price of A has increased and, as a result, consumers want to purchase less of it. C) consumer preferences have changed in favor of A so that they now want to buy more at each possible price. D) the price of A has declined and, as a result, consumers want to purchase more of it. Answer: C

Type: A Topic: 2 E: 44 MI: 44 MA: 44 34. If L and M are complementary goods, an increase in the price of L will result in: A) an increase in the sales of L. C) a decrease in the sales of M. B) no change in either the price or sales of M. D) an increase in the sales of M. Answer: C

McConnell/Brue: Economics, 16/e

Page 74

Chapter 3: Individual Markets: Demand and Supply

Type: A Topic: 2 E: 43 MI: 43 MA: 43 35. Which of the following will cause the demand curve for product A to shift to the left? A) population growth that causes an expansion in the number of persons consuming A B) an increase in money income if A is a normal good C) a decrease in the price of complementary product C D) an increase in money income if A is an inferior good Answer: D

Type: A Topic: 2 E: 43 MI: 43 MA: 43 36. If X is a normal good, a rise in money income will shift the: A) supply curve for X to the left. C) demand curve for X to the left. B) supply curve for X to the right. D) demand curve for X to the right. Answer: D

Type: A Topic: 2 E: 43 MI: 43 MA: 43 37. If Z is an inferior good, a decrease in money income will shift the: A) supply curve for Z to the left. C) demand curve for Z to the left. B) supply curve for Z to the right. D) demand curve for Z to the right. Answer: D

Type: C Topic: 2 E: 43 MI: 43 MA: 43 38. In 1994 Ford sold 500,000 Escorts at an average price of $7,200 per car; in 1995, 600,000 Escorts were sold at an average price of $7,500 per car. These statements: A) suggest that the demand for Escorts decreased between 1994 and 1995. B) imply that Escorts are an inferior good. C) suggest that the demand for Escorts increased between 1994 and 1995. D) constitute an exception to the law of demand in that they suggest an upsloping demand curve. Answer: C

Type: A Topic: 2 E: 44 MI: 44 MA: 44 39. Other things equal, which of the following might shift the demand curve for gasoline to the left? A) the discovery of vast new oil reserves in Montana B) the development of a low-cost electric automobile C) an increase in the price of train and air transportation D) a large decline in the price of automobiles Answer: B

Type: A Topic: 2 E: 43 MI: 43 MA: 43 40. An increase in consumer incomes will: A) increase the demand for an inferior good. B) increase the supply of an inferior good. Answer: C

C) increase the demand for a normal good. D) decrease the supply of a normal good.

Type: A Topic: 2 E: 44 MI: 44 MA: 44 41. Tennis rackets and ballpoint pens are: A) substitute goods. B) complementary goods. Answer: D

C) inferior goods.

D) independent goods.

McConnell/Brue: Economics, 16/e

Page 75

Chapter 3: Individual Markets: Demand and Supply

Type: D Topic: 2 E: 43 MI: 43 MA: 43 42. The demand for most products varies directly with changes in consumer incomes. Such products are known as: A) complementary goods. B) competitive goods. C) inferior goods. D) normal goods. Answer: D

Type: A Topic: 2 E: 43 MI: 43 MA: 43 43. Assume the demand curve for product X shifts to the right. This might be caused by: A) a decline in income if X is an inferior good. B) a decline in the price of Z if X and Z are substitute goods. C) a change in consumer tastes that is unfavorable to X. D) an increase in the price of Y if X and Y are complementary goods. Answer: A

Type: A Topic: 2 E: 44 MI: 44 MA: 44 44. Cameras and film are: A) substitute goods. B) complementary goods. Answer: B

C) independent goods.

D) inferior goods.

Type: A Topic: 2 E: 44 MI: 44 MA: 44 45. A decrease in the price of cameras will: A) cause the demand curve for film to become vertical. B) shift the demand curve for film to the right. C) shift the demand curve for film to the left. D) not affect the demand for film. Answer: B

Type: D Topic: 2 E: 43 MI: 43 MA: 43 46. A normal good is one: A) whose amount demanded will increase as its price decreases. B) whose amount demanded will increase as its price increases. C) whose demand curve will shift leftward as incomes rise. D) the consumption of which varies directly with incomes. Answer: D

Type: A Topic: 2 E: 43 MI: 43 MA: 43 47. If the demand for a normal good (for example, steak) shifts to the left, the most likely reason is that: A) consumer incomes have fallen. C) the price of steak has risen. B) cattle production has declined. D) the price of cattle feed has gone up. Answer: A

Type: A Topic: 2 E: 43 MI: 43 MA: 43 48. If consumer incomes increase, the demand for product X: A) will necessarily remain unchanged. C) will necessarily shift to the right. B) may shift either to the right or left. D) will necessarily shift to the left. Answer: B

McConnell/Brue: Economics, 16/e

Page 76

Chapter 3: Individual Markets: Demand and Supply

Type: A Topic: 2 E: 44 MI: 44 MA: 44 49. If products A and B are complements and the price of B decreases the: A) demand curves for both A and B will shift to the left. B) amount of B purchased will increase, but the demand curve for A will not shift. C) demand for A will increase and the amount of B demanded will increase. D) demand for A will decline and the demand for B will increase. Answer: C

Type: A Topic: 2 E: 44 MI: 44 MA: 44 50. If products C and D are close substitutes, an increase in the price of C will: A) tend to cause the price of D to fall. B) shift the demand curve of C to the left and the demand curve of D to the right. C) shift the demand curve of D to the right. D) shift the demand curves of both products to the right. Answer: C

Type: D Topic: 2 E: 40 MI: 40 MA: 40 51. In constructing a stable demand curve for product X: A) consumer preferences are allowed to vary. B) the prices of other goods are assumed constant. C) money incomes are allowed to vary. D) the supply curve of product X is assumed to be fixed. Answer: B

Type: D Topic: 2 E: 43-44 MI: 43-44 MA: 43-44 52. The demand curve for a product might shift as the result of a change in: A) consumer tastes. B) consumer incomes. C) the prices of related goods. Answer: D

D) all of the above.

Type: D Topic: 2 E: 43 MI: 43 MA: 43 53. An inferior good is: A) one whose demand curve will shift rightward as incomes rise. B) one whose price and quantity demanded vary directly. C) one which has not been approved by the Federal Food and Drug Administration. D) not accurately defined by any of the above statements. Answer: D

Type: C Topic: 2 E: 44 MI: 44 MA: 44 54. Suppose an excise tax is imposed on product X. We would expect this tax to: A) increase the demand for complementary good Y and decrease the demand for substitute product Z. B) decrease the demand for complementary good Y and increase the demand for substitute product Z. C) increase the demands for both complementary good Y and substitute product Z. D) decrease the demands for both complementary good Y and substitute product Z. Answer: B

McConnell/Brue: Economics, 16/e

Page 77

Chapter 3: Individual Markets: Demand and Supply

Type: A Topic: 2 E: 44 MI: 44 MA: 44 55. An increase in the price of product A will: A) reduce the demand for resources used in the production of A. B) increase the demand for complementary product C. C) increase the demand for substitute product B. D) reduce the demand for substitute product B. Answer: C

Change in demand versus change in quantity demanded

Use the following to answer questions 56-57:

Price

x y D1 0 Quantity D2

Type: G Topic: 3 E: 42 MI: 42 MA: 42 56. A decrease in demand is depicted by a: A) move from point x to point y. B) shift from D1 to D2. Answer: C

C) shift from D2 to D1. D) move from point y to point x.

Type: G Topic: 3 E: 45 MI: 45 MA: 45 57. A decrease in quantity demanded (as distinct from a decrease in demand) is depicted by a: A) move from point x to point y. C) shift from D2 to D1. B) shift from D1 to D2. D) move from point y to point x. Answer: D

Type: D Topic: 3 E: 42-43 MI: 42-43 MA: 42-43 58. When an economist says that the demand for a product has increased, this means that: A) consumers are now willing to purchase more of this product at each possible price. B) the product has become particularly scarce for some reason. C) product price has fallen and as a consequence consumers are buying a larger quantity of the product. D) the demand curve has shifted to the left. Answer: A

McConnell/Brue: Economics, 16/e

Page 78

Chapter 3: Individual Markets: Demand and Supply

Type: A Topic: 3 E: 44-45 MI: 44-45 MA: 44-45 59. "In the corn market, demand often exceeds supply and supply sometimes exceeds demand." "The price of corn rises and falls in response to changes in supply and demand." In which of these two statements are the terms demand and supply being used correctly? A) in neither statement B) in the second statement C) in the first statement D) in both statements Answer: B

Type: D Topic: 3 E: 44-45 MI: 44-45 MA: 44-45 60. By an increase in demand we mean that : A) product price has fallen so consumers move down to a new point on the demand curve. B) the quantity demanded at each price in a set of prices is greater. C) the quantity demanded at each price in a set of prices is smaller. D) a leftward shif of the demand curve has occurred. Answer: B

Type: C Topic: 3 E: 45 MI: 45 MA: 45 61. The quantity demanded of a product increases as its price declines because the: A) lower price shifts the demand curve rightward. C) lower price results in an increase in supply. B) lower price shifts the demand curve leftward. D) demand curve is downsloping. Answer: D

Type: D Topic: 3 E: 45 MI: 45 MA: 45 62. The term quantity demanded: A) refers to the entire series of prices and quantities that comprise the demand schedule. B) refers to a situation in which the income and substitution effects do not apply. C) refers to the amount of a product that will be purchased at some specific price. D) means the same thing as demand. Answer: C

Type: A Topic: 3 E: 44-45 MI: 44-45 MA: 44-45 63. If consumers are willing to pay a higher price than previously for each level of output, we can say that that following has occurred: A) a decrease in demand. C) a decrease in supply. B) an increase in demand. D) an increase in supply. Answer: B

Type: A Topic: 3 E: 44 MI: 44 MA: 44 64. A decrease in the demand for recreational fishing boats might be caused by an increase in the: A) income of sports fishers. C) size and number of fish available. B) price of outboard motors. D) price of sailing boats. Answer: B

Type: D Topic: 3 E: 42 MI: 42 MA: 42 65. An increase in demand means that: A) given supply, the price of the product will decline. B) the demand curve has shifted to the right. C) price has declined and consumers therefore want to purchase more of the product. D) the demand curve has shifted to the left. Answer: B

McConnell/Brue: Economics, 16/e

Page 79

Chapter 3: Individual Markets: Demand and Supply

Type: A Topic: 3 E: 45 MI: 45 MA: 45 66. Assume that the demand schedule for product C is downsloping. If the price of C falls from $2.00 to $1.75: A) a smaller quantity of C will be demanded. C) the demand for C will increase. B) a larger quantity of C will be demanded. D) the demand for C will decrease. Answer: B

Type: A Topic: 3 E: 45 MI: 45 MA: 45 67. An increase in the quantity demanded means that: A) given supply, the price of the product can be expected to decline. B) price has declined and consumers therefore want to purchase more of the product. C) the demand curve has shifted to the right. D) the demand curve has shifted to the left. Answer: B

Type: A Topic: 3 E: 45 MI: 45 MA: 45 68. An increase in product price will cause: A) quantity demanded to decrease. B) quantity supplied to decrease. Answer: A

C) quantity demanded to increase. D) the supply curve to shift to the right.

Type: A Topic: 3 E: 45 MI: 45 MA: 45 69. In moving along a stable demand curve which of the following is not held constant? A) the price of the product for which the demand curve is relevant B) price expectations C) consumer incomes D) the prices of complementary goods Answer: A

Supply and supply curve Use the following to answer questions 70-71:

x Price

S1

S2

y

0

Quantity

Type: G Topic: 4 E: 46 MI: 46 MA: 46 70. A decrease in supply is depicted by a: A) move from point x to point y. B) shift from S1 to S2. Answer: C

C) shift from S2 to S1. D) move from point y to point x.

McConnell/Brue: Economics, 16/e

Page 80

Chapter 3: Individual Markets: Demand and Supply

Type: G Topic: 4 E: 47 MI: 47 MA: 47 71. An increase in quantity supplied (as distinct from an increase in supply) is depicted by a: A) move from point y to point x. C) shift from S2 to S1. B) shift from S1 to S2. D) move from point x to point y. Answer: A

Type: D Topic: 4 E: 45 MI: 45 MA: 45 72. The law of supply indicates that: A) producers will offer more of a product at high prices than they will at low prices. B) the product supply curve is downsloping. C) consumers will purchase less of a good at high prices than they will at low prices. D) producers will offer more of a product at low prices than they will at high prices. Answer: A

Type: D Topic: 4 E: 45 MI: 45 MA: 45 73. The law of supply: A) reflects the amounts that producers will want to offer at each price in a series of prices. B) is reflected in a downsloping supply curve. C) shows that the relationship between producer revenue and quantity supplied is negative. D) reflects the income and substitution effects of a price change. Answer: A

Type: D Topic: 4 E: 46 MI: 46 MA: 46 74. The upward slope of the supply curve reflects the: A) principle of specialization in production. B) law of supply. C) fact that price and quantity supplied are inversely related. D) law of diminishing marginal utility. Answer: B

Type: G Topic: 4 E: 58 MI: 58 MA: 58 75.

(Advanced

analysis) The equation for the supply curve in the above diagram: A) is P = 5 + 1/3Q. B) is P = 5 + 2Q. C) is P = 5 + 3Q. D) is P = 5 - 3Q. Answer: A

McConnell/Brue: Economics, 16/e

Page 81

Chapter 3: Individual Markets: Demand and Supply

Type: D Topic: 4 E: 45 MI: 45 MA: 45 76. The supply curve shows the relationship between: A) price and quantity supplied. B) production costs and the amount demanded. C) total business revenues and quantity supplied. D) physical inputs of resources and the resulting units of output. Answer: A

Type: A Topic: 4 E: 45 MI: 45 MA: 45 77. A firm's supply curve is upsloping because: A) the expansion of production necessitates the use of qualitatively inferior inputs. B) mass production economies are associated with larger levels of output. C) consumers envision a positive relationship between price and quality. D) beyond some point the production costs of additional units of output will rise. Answer: D

Determinants of supply

Type: A Topic: 5 E: 47 MI: 47 MA: 47 78. A leftward shift of a product supply curve might be caused by: A) an improvement in the relevant technique of production. B) a decline in the prices of needed inputs. C) an increase in consumer incomes. D) some firms leaving an industry. Answer: D

Type: D Topic: 5 E: 47 MI: 47 MA: 47 79. The location of the product supply curve depends on: A) production technology. C) the tastes of buyers. B) the number of buyers in the market. D) the location of the demand curve. Answer: A

Type: A Topic: 5 E: 47 MI: 47 MA: 47 80. An improvement in production technology will: A) increase equilibrium price. B) shift the supply curve to the left. Answer: C

C) shift the supply curve to the right. D) shift the demand curve to the left.

Type: A Topic: 5 E: 47 MI: 47 MA: 47 81. Because of unseasonably cold weather, the supply of oranges has substantially decreased. This statement indicates that: A) the demand for oranges will necessarily rise. B) the equilibrium quantity of oranges will rise. C) the amount of oranges that will be available at various prices has declined. D) the price of oranges will fall. Answer: C

McConnell/Brue: Economics, 16/e

Page 82

Chapter 3: Individual Markets: Demand and Supply

Type: A Topic: 5 E: 47 MI: 47 MA: 47 82. If producers must obtain higher prices than previously to produce various levels of output, the following has occurred: A) a decrease in demand. C) a decrease in supply. B) an increase in demand. D) an increase in supply. Answer: C

Type: A Topic: 5 E: 47-48 MI: 47-48 MA: 47-48 83. In moving along a stable supply curve which of the following is not held constant? A) the number of firms producing this good B) expectations about the future price of the product C) techniques used in producing this product D) the price of the product for which the supply curve is relevant Answer: D

Type: D Topic: 5 E: 46 MI: 46 MA: 46 84. The location of the supply curve of a product depends on: A) the technology used to produce it. C) the number of sellers in the market. B) the prices of resources used in its production. D) all of the above. Answer: D

Type: C Topic: 5 E: 47 MI: 47 MA: 47 85. Assume product A is an input in the production of product B. In turn product B is a complement to product C. We can expect a decrease in the price of A to: A) increase the supply of B and increase the demand for C. B) decrease the supply of B and increase the demand for C. C) decrease the supply of B and decrease the demand for C. D) increase the supply of B and decrease the demand for C. Answer: A

Type: C Topic: 5 E: 47 MI: 47 MA: 47 86. Assume a drought in the Great Plains reduces the supply of wheat. Noting that wheat is a basic ingredient in the production of bread and that potatoes are a consumer substitute for bread, we would expect the price of wheat to: A) rise, the supply of bread to increase, and the demand for potatoes to increase. B) rise, the supply of bread to decrease, and the demand for potatoes to increase. C) rise, the supply of bread to decrease, and the demand for potatoes to decrease. D) fall, the supply of bread to increase, and the demand for potatoes to increase. Answer: B

Type: A Topic: 5 E: 47 MI: 47 MA: 47 87. Suppose product X is an input in the production of product Y. Product Y in turn is a substitute for product Z. An increase in the price of X can be expected to: A) decrease the demand for Z. C) have no effect on the demand for Z. B) increase the demand for Z. D) decrease the supply of Z. Answer: B

McConnell/Brue: Economics, 16/e

Page 83

Chapter 3: Individual Markets: Demand and Supply

Type: A Topic: 5 E: 46-47 MI: 46-47 MA: 46-47 Status: New 88. Other things equal, if the price of a key resource used to produce product X falls, the: A) product supply curve of X will shift to the right. B) product demand curve of X will shift to the right. C) product supply curve of X will shift to the left. D) product demand curve of X will shift to the right. Answer: A

Type: A Topic: 5 E: 47 MI: 47 MA: 47 Status: New 89. When the price of oil declines significantly, the price of gasoline also declines. The latter occurs because of a(n): A) increase in the demand for gasoline. C) increase in the supply of gasoline. B) decrease in the demand for gasoline. D) decrease in the supply of gasoline. Answer: C

Type: A Topic: 5 E: 47 MI: 47 MA: 47 Status: New 90. An increase in the excise tax on cigarettes raises the price of cigarettes by shifting the: A) demand curve for cigarettes rightward. C) supply curve for cigarettes rightward. B) demand curve for cigarettes leftward. D) supply curve for cigarettes leftward. Answer: D

Type: A Topic: 5 E: 47 MI: 47 MA: 47 Status: New 91. A government subsidy to the producers of a product: A) reduces product supply. C) reduces product demand. B) increases product supply. D) increases product demand. Answer: B

Type: C Topic: 5 E: 47 MI: 47 MA: 47 Status: New 92. Suppose that at prices of $5, $4, $3, $2, and $1 for product Z, the corresponding quantities supplied are 3, 4, 5, 6, and 7 units, respectively. Which of the following would increase the quantities supplied of Z to, say, 6, 8, 10, 12, and 14 units at these prices? A) improved technology for producing Z B) an increase in the prices of the resources used to make Z C) an increase in the excise tax on product Z D) increases in the incomes of the buyers of Z Answer: A

McConnell/Brue: Economics, 16/e

Page 84

Chapter 3: Individual Markets: Demand and Supply

Equilibrium; rationing function

Use the following to answer questions 93-98:
(1) Qd 50 60 80 90 100 (2) Qd 40 50 60 70 80 (3) Price $10 9 8 7 6 (4) Qs 70 60 50 40 30 (5) Qs 80 70 60 50 40

Type: T Topic: 6 E: 48-49 MI: 48-49 MA: 48-49 93. Refer to the above table. If demand is represented by columns (3) and (2) and supply is represented by columns (3) and (5), equilibrium price and quantity will be: A) $10 and 60 units. B) $9 and 50 units. C) $8 and 60 units. D) $7 and 50 units. Answer: C

Type: T Topic: 6 E: 48-49 MI: 48-49 MA: 48-49 94. Refer to the above table. If demand is represented by columns (3) and (1) and supply is represented by columns (3) and (4), equilibrium price and quantity will be: A) $10 and 60 units. B) $9 and 60 units. C) $8 and 80 units. D) $7 and 30 units. Answer: B

Type: T Topic: 6 E: 45 MI: 45 MA: 45 95. Refer to the above table. In relation to column (3), a change from column (2) to column (1) would indicate a(n): A) increase in demand. B) decrease in demand. C) increase in supply. D) decrease in supply. Answer: A

Type: T Topic: 6 E: 47 MI: 47 MA: 47 96. Refer to the above table. In relation to column (3), a change from column (5) to column (4) would indicate a(n): A) increase in demand. B) decrease in demand. C) increase in supply. D) decrease in supply. Answer: D

Type: T Topic: 6 E: 48 MI: 48 MA: 48 97. Refer to the above table. Suppose that demand is represented by columns (3) and (2) and supply is represented by columns (3) and (5). If the price were artificially set at $9, a: A) the market would clear. B) a surplus of 20 units would occur. C) a shortage of 20 units would occur. D) demand would change from columns (3) and (2) to columns (3 and (1). Answer: B

McConnell/Brue: Economics, 16/e

Page 85

Chapter 3: Individual Markets: Demand and Supply

Type: T Topic: 6 E: 48 MI: 48 MA: 48 98. Refer to the above table. Suppose that demand is represented by columns (3) and (2) and supply is represented by columns (3) and (5). If the price were artificially set at $6, a: A) the market would clear. B) a surplus of 40 units would occur. C) a shortage of 40 units would occur. D) demand would change from columns (3) and (2) to columns (3 and (1). Answer: C

Use the following to answer questions 99-101: Answer the next question(s) on the basis of the given supply and demand data for wheat:
Bushels demanded per month 45 50 56 61 67 Price per bushel $5 4 3 2 1 Bushels supplied per month 77 73 68 61 57

Type: T Topic: 6 E: 48-49 MI: 48-49 MA: 48-49 99. Refer to the above data. Equilibrium price will be: A) $4. B) $3. C) $2. D) $1. Answer: C

Type: T Topic: 6 E: 48 MI: 48 MA: 48 100. Refer to the above data. If the price in this market was $4: A) the market would clear; quantity demanded would equal quantity supplied. B) buyers would want to purchase more wheat than is currently being supplied. C) farmers would not be able to sell all their wheat. D) there would be a shortage of wheat. Answer: C

Type: T Topic: 6 E: 48 MI: 48 MA: 48 101. Refer to the above data. If price was initially $4 and free to fluctuate, we would expect: A) quantity supplied to continue to exceed quantity demanded. B) the quantity of wheat supplied to decline as a result of the subsequent price change. C) the quantity of wheat demanded to fall as a result of the subsequent price change. D) the price of wheat to rise. Answer: B

McConnell/Brue: Economics, 16/e

Page 86

Chapter 3: Individual Markets: Demand and Supply

Use the following to answer questions 102-104:

Type: G Topic: 6 E: 48-49 MI: 48-49 MA: 48-49 102. Refer to the above diagram. The equilibrium price and quantity in this market will be: A) $1.00 and 200. B) $1.60 and 130. C) $.50 and 130. D) $1.60 and 290. Answer: A

Type: G Topic: 6 E: 48 MI: 48 MA: 48 103. Refer to the above diagram. A surplus of 160 units would be encountered if price was: A) $1.10, that is, $1.60 minus $.50. B) $1.60. C) $1.00. D) $.50. Answer: B

Type: G Topic: 6 E: 48 MI: 48 MA: 48 104. Refer to the above diagram. A shortage of 160 units would be encountered if price was: A) $1.10, that is, $1.60 minus $.50. B) $1.60. C) $1.00. D) $.50. Answer: D

Type: G Topic: 6 E: 48 MI: 48 MA: 48 105. If a product is in surplus supply, its price: A) is below the equilibrium level. B) is above the equilibrium level. Answer: B

C) will rise in the near future. D) is in equilibrium.

Type: D Topic: 6 E: 48 MI: 48 MA: 48 106. A market is in equilibrium: A) provided there is no surplus of the product. B) at all prices above that shown by the intersection of the supply and demand curves. C) if the amount producers want to sell is equal to the amount consumers want to buy. D) whenever the demand curve is downsloping and the supply curve is upsloping. Answer: C

McConnell/Brue: Economics, 16/e

Page 87

Chapter 3: Individual Markets: Demand and Supply

Type: C Topic: 6 E: 49 MI: 49 MA: 49 107. If the demand and supply curves for product X are stable, a government-mandated increase in the price of X will: A) increase the supply of X and decrease the demand for X. B) increase the demand for X and decrease the supply of X. C) increase the quantity supplied and decrease the quantity demanded of X. D) decrease the quantity supplied of X and increase the quantity demanded of X. Answer: C

Type: A Topic: 6 E: 48-49 MI: 48-49 MA: 48-49 108. At the equilibrium price: A) quantity supplied may exceed quantity demanded or vice versa. B) there are no pressures on price to either rise or fall. C) there are forces that cause price to rise. D) there are forces that cause price to fall. Answer: B

Use the following to answer questions 109-112:

Type: G Topic: 6 E: 48-49 MI: 48-49 MA: 48-49 109. Refer to the above diagram. A price of $60 in this market will result in: A) equilibrium. B) a shortage of 50 units. C) a surplus of 50 units. Answer: D

D) a surplus of 100 units.

Type: G Topic: 6 E: 48-49 MI: 48-49 MA: 48-49 110. Refer to the above diagram. A price of $20 in this market will result in: A) a shortage of 50 units. C) a surplus of 100 units. B) a surplus of 50 units. D) a shortage of 100 units. Answer: D

Type: G Topic: 6 E: 40 MI: 40 MA: 40 111. Refer to the above diagram. The highest price that buyers will be willing and able to pay for 100 units of this product is: A) $30. B) $60. C) $40. D) $20. Answer: B

McConnell/Brue: Economics, 16/e

Page 88

Chapter 3: Individual Markets: Demand and Supply

Type: G Topic: 6 E: 48-49 MI: 48-49 MA: 48-49 112. Refer to the above diagram. If this is a competitive market, price and quantity will move toward: A) $60 and 100 respectively. C) $40 and 150 respectively. B) $60 and 200 respectively. D) $20 and 150 respectively. Answer: C

Type: A Topic: 6 E: 49 MI: 49 MA: 49 113. At the point where the demand and supply curves for a product intersect: A) the selling price and the buying price need not be equal. B) the market may, or may not, be in equilibrium. C) either a shortage or a surplus of the product might exist, depending on the degree of competition. D) the quantity that consumers want to purchase and the amount producers choose to sell are the same. Answer: D

Type: D Topic: 6 E: 50 MI: 50 MA: 50 114. The rationing function of prices refers to the: A) tendency of supply and demand to shift in opposite directions. B) fact that ration coupons are needed to alleviate wartime shortages of goods. C) capacity of a competitive market to equate the quantity demanded and the quantity supplied. D) ability of the market system to generate an equitable distribution of income. Answer: C

Type: A Topic: 6 E: 50 MI: 50 MA: 50 115. A competitive market will: A) achieve an equilibrium price. B) produce shortages. Answer: A

C) produce surpluses.

D) create disorder.

Type: A Topic: 6 E: 48 MI: 48 MA: 48 116. If there is a shortage of product X: A) fewer resources will be allocated to the production of this good. B) the price of the product will rise. C) the price of the product will decline. D) the supply curve will shift to the left and the demand curve to the right, eliminating the shortage. Answer: B

Type: A Topic: 6 E: 49 MI: 49 MA: 49 117. At the point where the demand and supply curves intersect: A) the buying and selling decisions of consumers and producers are inconsistent with one another. B) the market is in disequilibrium. C) there is neither a surplus nor a shortage of the product. D) quantity demanded exceeds quantity supplied. Answer: C

McConnell/Brue: Economics, 16/e

Page 89

Chapter 3: Individual Markets: Demand and Supply

Type: A Topic: 6 E: 49 MI: 49 MA: 49 118. At the current price there is a shortage of a product. We would expect price to: A) increase, quantity demanded to increase, and quantity supplied to decrease. B) increase, quantity demanded to decrease, and quantity supplied to increase. C) increase, quantity demanded to increase, and quantity supplied to increase. D) decrease, quantity demanded to increase, and quantity supplied to decrease. Answer: B

Type: D Topic: 6 E: 50 MI: 50 MA: 50 119. In a competitive market the equilibrium price and quantity occur where: A) the downsloping demand curve intersects the upsloping supply curve. B) the upsloping demand curve intersects the downsloping supply curve. C) consumers and suppliers bargain to a mutually acceptable price. D) quantity demanded exceeds quantity supplied or vice versa. Answer: A

Type: A Topic: 6 E: 49 MI: 49 MA: 49 120. A surplus of a product will arise when price is: A) above equilibrium with the result that quantity demanded exceeds quantity supplied. B) above equilibrium with the result that quantity supplied exceeds quantity demanded. C) below equilibrium with the result that quantity demanded exceeds quantity supplied. D) below equilibrium with the result that quantity supplied exceeds quantity demanded. Answer: B

Type: A Topic: 6 E: 49 MI: 49 MA: 49 121. If price is above the equilibrium level, competition among sellers to reduce the resulting: A) surplus will increase quantity demanded and decrease quantity supplied. B) shortage will decrease quantity demanded and increase quantity supplied. C) surplus will decrease quantity demanded and increase quantity supplied. D) shortage will increase quantity demanded and decrease quantity supplied. Answer: A

Type: D Topic: 6 E: 50 MI: 50 MA: 50 122. If we say that a price is too high to clear the market, we mean that: A) quantity demanded exceeds quantity supplied. C) quantity supplied exceeds quantity demanded. B) the equilibrium price is above the current price. D) the price of the good is likely to rise. Answer: C

Type: A Topic: 6 E: 49 MI: 49 MA: 49 123. Assume in a competitive market that price is initially above the equilibrium level. We can predict that price will: A) decrease, quantity demanded will decrease, and quantity supplied will increase. B) decrease and quantity demanded and quantity supplied will both decrease. C) decrease, quantity demanded will increase, and quantity supplied will decrease. D) increase, quantity demanded will decrease, and quantity supplied will increase. Answer: C

McConnell/Brue: Economics, 16/e

Page 90

Chapter 3: Individual Markets: Demand and Supply

Type: A Topic: 6 E: 49 MI: 49 MA: 49 124. Assume in a competitive market that price is initially below the equilibrium level. We can predict that price will: A) decrease, quantity demanded will decrease, and quantity supplied will increase. B) decrease and quantity demanded and quantity supplied will both decrease. C) decrease, quantity demanded will increase, and quantity supplied will decrease. D) increase, quantity demanded will decrease, and quantity supplied will increase. Answer: D

Type: D Topic: 6 E: 49 MI: 49 MA: 49 125. A product market is in equilibrium: A) when there is no surplus of the product. B) when there is no shortage of the product. C) when consumers want to buy more of the product than producers offer for sale. D) where the demand and supply curves intersect. Answer: D

Type: D Topic: 6 E: 48 MI: 48 MA: 48 126. There will be a surplus of a product when: A) price is below the equilibrium level. B) the supply curve is downward sloping and the demand curve is upward sloping. C) the demand and supply curves fail to intersect. D) consumers want to buy less than producers offer for sale. Answer: D

Changes in equilibrium price and quantity

127.

Type: C Topic: 7 E: 47 MI: 47 MA: 47 Other things equal, an excise tax on a product will: A) increase its supply. B) increase its price. C) increase the quantity sold. Answer: B

D) increase its demand.

Type: A Topic: 7 E: 50-52 MI: 50-52 MA: 50-52 128. Assuming conventional supply and demand curves, changes in the determinants of supply and demand will: A) in all likelihood alter both equilibrium price and quantity. B) alter equilibrium quantity, but not equilibrium price. C) alter equilibrium price, but not equilibrium quantity. D) have no effect on equilibrium price or quantity. Answer: A

Type: A Topic: 7 E: 50 MI: 50 MA: 50 129. Which of the following will cause a decrease in market equilibrium price and an increase in equilibrium quantity? A) an increase in supply B) an increase in demand C) a decrease in supply D) a decrease in demand Answer: A

McConnell/Brue: Economics, 16/e

Page 91

Chapter 3: Individual Markets: Demand and Supply

Type: A Topic: 7 E: 50-51 MI: 50-51 MA: 50-51 130. Suppose in each of four successive years producers sell more of their product and at lower prices. This could be explained: A) by small annual increases in supply accompanied by large annual increases in demand. B) in terms of a stable supply curve and increasing demand. C) in terms of a stable demand curve and increasing supply. D) as an exception to the law of supply. Answer: C

Type: C Topic: 7 E: 47 MI: 47 MA: 47 131. Other things equal, the provision of a per unit subsidy for a product will: A) increase its supply. C) decrease the quantity sold. B) increase its price. D) decrease its demand. Answer: A

Type: C Topic: 7 E: 51 MI: 51 MA: 51 132. Which of the following statements is correct? A) If demand increases and supply decreases, equilibrium price will fall. B) If supply increases and demand decreases, equilibrium price will fall. C) If demand decreases and supply increases, equilibrium price will rise. D) If supply declines and demand remains constant, equilibrium price will fall. Answer: B

Type: C Topic: 7 E: 50-52 MI: 50-52 MA: 50-52 133. In which of the following instances will the effect on equilibrium price be dependent on the magnitude of the shifts in supply and demand? A) demand rises and supply rises C) demand rises and supply falls B) supply falls and demand remains constant D) supply rises and demand falls Answer: A

McConnell/Brue: Economics, 16/e

Page 92

Chapter 3: Individual Markets: Demand and Supply

Use the following to answer questions 134-138:

Type: G Topic: 7 E: 48-49 MI: 48-49 MA: 48-49 134. Refer to the above diagram, which shows demand and supply conditions in the competitive market for product X. If the initial demand and supply curves are D0 and S0, equilibrium price and quantity will be: A) 0F and 0C respectively. C) 0F and 0A respectively. B) 0G and 0B respectively. D) 0E and 0B respectively. Answer: A

Type: G Topic: 7 E: 52 MI: 52 MA: 52 135. Refer to the above diagram, which shows demand and supply conditions in the competitive market for product X. Given D0, if the supply curve moved from S0 to S1 , then: A) supply has increased and equilibrium quantity has decreased. B) supply has decreased and equilibrium quantity has decreased. C) there has been an increase in the quantity supplied. D) supply has increased and price has risen to 0G. Answer: B

Type: G Topic: 7 E: 48 MI: 48 MA: 48 136. Refer to the above diagram, which shows demand and supply conditions in the competitive market for product X. If supply is S1 and demand D0, then A) at any price above 0G a shortage would occur. B) 0F represents a price that would result in a surplus of AC. C) a surplus of GH would occur. D) 0F represents a price that would result in a shortage of AC. Answer: D

Type: G Topic: 7 E: 43-44 MI: 43-44 MA: 43-44 137. Refer to the above diagram, which shows demand and supply conditions in the competitive market for product X. A shift in the demand curve from D0 to D1 might be caused by a(n): A) decrease in income if X is an inferior good. B) increase in the price of complementary good Y. C) increase in money incomes if X is a normal good. D) increase in the price of substitute product Y. Answer: B

McConnell/Brue: Economics, 16/e

Page 93

Chapter 3: Individual Markets: Demand and Supply

Type: G Topic: 7 E: 46-47 MI: 46-47 MA: 46-47 138. Refer to the above diagram, which shows demand and supply conditions in the competitive market for product X. Other things equal, a shift of the supply curve from S0 to S1 might be caused by a(n): A) increase in the wage rates paid to laborers employed in the production of X. B) government subsidy per unit of output paid to firms producing X. C) decline in the price of the basic raw material used in producing X. D) increase in the number of firms producing X. Answer: A

Type: C Topic: 7 E: 52 MI: 52 MA: 52 139. If the supply and demand curves for a product both decrease, then equilibrium: A) quantity must fall and equilibrium price must rise. B) price must fall, but equilibrium quantity may either rise, fall, or remain unchanged. C) quantity must decline, but equilibrium price may either rise, fall, or remain unchanged. D) quantity and equilibrium price must both decline. Answer: C

Type: C Topic: 7 E: 51-52 MI: 51-52 MA: 51-52 140. If the supply of a product decreases and the demand for that product simultaneously increases, then equilibrium: A) price must rise, but equilibrium quantity may either rise, fall, or remain unchanged. B) price must rise and equilibrium quantity must fall. C) price and equilibrium quantity must both increase. D) price and equilibrium quantity must both decline. Answer: A

Type: C Topic: 7 E: 50 MI: 50 MA: 50 141. Assuming competitive markets with typical supply and demand curves, which of the following statements is correct? A) An increase in supply with a decrease in demand will result in an increase in price. B) An increase in supply with no change in demand will result in an increase in price. C) An increase in supply with no change in demand will result in a decline in sales. D) An increase in demand with no change in supply will result in an increase in sales. Answer: D

McConnell/Brue: Economics, 16/e

Page 94

Chapter 3: Individual Markets: Demand and Supply

Use the following to answer questions 142-145:

Type: G Topic: 7 E: 50-51 MI: 50-51 MA: 50-51 142. Refer to the above diagram, in which S1 and D1 represent the original supply and demand curves and S2 and D2 the new curves. In this market: A) supply has decreased and equilibrium price has increased. B) demand has increased and equilibrium price has decreased. C) demand has decreased and equilibrium price has decreased. D) demand has increased and equilibrium price has increased. Answer: B

Type: G Topic: 7 E: 50-51 MI: 50-51 MA: 50-51 143. Refer to the above diagram, in which S1 and D1 represent the original supply and demand curves and S2 and D2 the new curves. In this market: A) the equilibrium position has shifted from M to K. B) an increase in demand has been more than offset by an increase in supply. C) the new equilibrium price and quantity are both greater than originally. D) point M shows the new equilibrium position. Answer: B

Type: C Topic: 7 E: 47 MI: 47 MA: 47 144. Refer to the above diagram, in which S1 and D1 represent the original supply and demand curves and S2 and D2 the new curves. In this market the indicated shift in supply may have been caused by: A) an increase in the wages paid to workers producing this good. B) the development of more efficient machinery for producing this commodity. C) this product becoming less fashionable. D) an increase in consumer incomes. Answer: B

McConnell/Brue: Economics, 16/e

Page 95

Chapter 3: Individual Markets: Demand and Supply

Type: C Topic: 7 E: 43 MI: 43 MA: 43 145. Refer to the above diagram, in which S1 and D1 represent the original supply and demand curves and S2 and D2 the new curves. In this market the indicated shift in demand may have been caused by: A) a decline in the number of buyers in the market. B) a decline in the price of a substitute good. C) an increase in incomes if the product is a normal good. D) an increase in incomes if the product is an inferior good. Answer: C

Type: C Topic: 7 E: 47 MI: 47 MA: 47 146. An unusually large crop of coffee beans might: A) increase the supply of coffee. B) increase the price of coffee. Answer: A

C) decrease the quantity of coffee consumed. D) increase the price of tea.

Use the following to answer questions 147-156: In the following question(s) you are asked to determine, other things equal, the effects of a given change in a determinant of demand or supply for product X upon (1) the demand (D) for, or supply (S) of, X, (2) the equilibrium price (P) of X and (3) the equilibrium quantity (Q) of X.

Type: C Topic: 7 E: 50 MI: 50 MA: 50 147. Refer to the above. An increase in income, if X is a normal good, will: A) increase D, increase P, and increase Q. C) increase S, increase P, and increase Q. B) increase D, increase P, and decrease Q. D) decrease D, increase P, and increase Q. Answer: A

Type: C Topic: 7 E: 50 MI: 50 MA: 50 148. Refer to the above. An increase in the price of a product that is a close substitute for X will: A) decrease D, increase P, and decrease Q. C) increase D, increase P, and increase Q. B) increase D, increase P, and decrease Q. D) increase D, decrease P, and increase Q. Answer: C

Type: C Topic: 7 E: 50 MI: 50 MA: 50 149. Refer to the above. A decrease in the number of consumers of product X will: A) decrease S, decrease P, and decrease Q. C) decrease D, decrease P, and decrease Q. B) increase D, increase P, and increase Q. D) decrease D, decrease P, and increase Q. Answer: C

Type: C Topic: 7 E: 50 MI: 50 MA: 50 150. Refer to the above. An increase in the tastes and preferences for X will: A) increase S, decrease P, and increase Q. C) increase D, increase P, and increase Q. B) decrease S, decrease P, and decrease Q. D) decrease D, decrease P, and decrease Q. Answer: C

McConnell/Brue: Economics, 16/e

Page 96

Chapter 3: Individual Markets: Demand and Supply

Type: C Topic: 7 E: 50 MI: 50 MA: 50 151. Refer to the above. An increase in the prices of resources used to produce X will: A) increase S, increase P, and increase Q. C) decrease S, decrease P, and decrease Q. B) increase D, increase P, and increase Q. D) decrease S, increase P, and decrease Q. Answer: D

Type: C Topic: 7 E: 50 MI: 50 MA: 50 152. Refer to the above. An improvement in the technology used to produce X will: A) decrease S, increase P, and decrease Q. C) increase S, decrease P, and increase Q. B) decrease S, increase P, and increase Q. D) decrease D, decrease P, and decrease Q. Answer: C

Type: C Topic: 7 E: 50 MI: 50 MA: 50 153. Refer to the above. A reduction in the number of firms producing X will: A) increase D, increase P, and increase Q. C) decrease S, increase P, and decrease Q. B) increase S, decrease P, and increase Q. D) decrease S, decrease P, and increase Q. Answer: C

Type: C Topic: 7 E: 50 MI: 50 MA: 50 154. Refer to the above. An increase in the price of a product that is a complement to X will: A) decrease S, decrease P, and decrease Q. C) decrease D, decrease P, and decrease Q. B) increase D, increase P, and increase Q. D) increase D, increase P, and decrease Q. Answer: C

Type: C Topic: 7 E: 50 MI: 50 MA: 50 155. Refer to the above. If X is an inferior good, a decrease in income will: A) decrease D, decrease P, and decrease Q. C) increase S, decrease P, and increase Q. B) decrease D, decrease P, and increase Q. D) increase D, increase P, and increase Q. Answer: D

Type: C Topic: 7 E: 50 MI: 50 MA: 50 156. Refer to the above. Consumer expectations that the price of X will rise sharply in the future will: A) increase S, increase P, and increase Q. C) decrease S, increase P, and increase Q. B) increase D, increase P, and increase Q. D) increase D, decrease P, and increase Q. Answer: B

Type: C Topic: 7 E: 50 MI: 50 MA: 50 157. Data from the registrar's office at Gigantic State University indicate that over the past twenty years tuition and enrollment have both increased. From this information we can conclude that: A) higher education is an exception to the law of demand. B) the supply of education provided by GSU has also increased over the twenty-year period. C) school-age population, incomes, and preferences for education have changed over the twenty-year period. D) GSU's supply curve of education is downsloping. Answer: C

McConnell/Brue: Economics, 16/e

Page 97

Chapter 3: Individual Markets: Demand and Supply

Type: C Topic: 7 E: 51 MI: 51 MA: 51 158. One can say with certainty that equilibrium price will decline when supply: A) and demand both decrease. C) decreases and demand increases. B) increases and demand decreases. D) and demand both increase. Answer: B

Type: C Topic: 7 E: 52 MI: 52 MA: 52 159. One can say with certainty that equilibrium quantity will increase when supply: A) and demand both decrease. C) decreases and demand increases. B) increases and demand decreases. D) and demand both increase. Answer: D

Use the following to answer questions 160-165:

Type: G Topic: 7 E: 50-51 MI: 50-51 MA: 50-51 160. Which of the above diagrams illustrate(s) the effect of an increase in automobile worker wages on the market for automobiles? A) A only B) B only C) C only D) D only Answer: D

Type: G Topic: 7 E: 50-51 MI: 50-51 MA: 50-51 161. Which of the above diagrams illustrate(s) the effect of a decline in the price of personal computers on the market for software? A) A only B) A and D C) B only D) D only Answer: A

Type: G Topic: 7 E: 50-51 MI: 50-51 MA: 50-51 162. Which of the above diagrams illustrate(s) the effect of an increase in the price of Budweiser beer on the market for Coors beer? A) A and C B) A only C) B only D) C only Answer: B

McConnell/Brue: Economics, 16/e

Page 98

Chapter 3: Individual Markets: Demand and Supply

Type: G Topic: 7 E: 50-51 MI: 50-51 MA: 50-51 163. Which of the above diagrams illustrate(s) the effect of a decrease in incomes on the market for secondhand clothing? A) A and C B) A only C) B only D) C only Answer: B

Type: G Topic: 7 E: 50-51 MI: 50-51 MA: 50-51 164. Which of the above diagrams illustrate(s) the effect of a governmental subsidy on the market for AIDS research? A) A only B) B only C) C only D) D only Answer: C

Type: G Topic: 7 E: 50-51 MI: 50-51 MA: 50-51 165. Which of the above diagrams illustrate(s) the effect of a decline in the price of irrigation equipment on the market for corn? A) B only B) C only C) B and C D) D only Answer: B

Type: C Topic: 7 E: 50-51 MI: 50-51 MA: 50-51 166. With a downsloping demand curve and an upsloping supply curve for a product, an increase in consumer income will: A) increase equilibrium price and quantity if the product is a normal good. B) decrease equilibrium price and quantity if the product is a normal good. C) have no effect on equilibrium price and quantity. D) reduce the quantity demanded, but not shift the demand curve. Answer: A

Type: C Topic: 7 E: 50-51 MI: 50-51 MA: 50-51 167. With a downsloping demand curve and an upsloping supply curve for a product, a decrease in resource prices will: A) increase equilibrium price and quantity. B) decrease equilibrium price and quantity. C) decrease equilibrium price and increase equilibrium quantity. D) increase equilibrium price and decrease equilibrium quantity. Answer: C

Type: C Topic: 7 E: 50-51 MI: 50-51 MA: 50-51 168. With a downsloping demand curve and an upsloping supply curve for a product, placing an excise tax on this product will: A) increase equilibrium price and quantity. B) decrease equilibrium price and quantity. C) decrease equilibrium price and increase equilibrium quantity. D) increase equilibrium price and decrease equilibrium quantity. Answer: D

McConnell/Brue: Economics, 16/e

Page 99

Chapter 3: Individual Markets: Demand and Supply

Type: C Topic: 7 E: 50-51 MI: 50-51 MA: 50-51 169. Given a downsloping demand curve and an upsloping supply curve for a product, an increase in the price of a substitute good will: A) increase equilibrium price and quantity. B) decrease equilibrium price and quantity. C) increase equilibrium price and decrease equilibrium quantity. D) decrease equilibrium price and increase equilibrium quantity. Answer: A

Use the following to answer questions 170-174: (Advanced analysis) Answer the next question(s) on the basis of the following information. The demand for commodity X is represented by the equation P = 10 - 0.2Q and supply by the equation P = 2 + 0.2Q.

Type: E Topic: 7 E: 58 MI: 58 MA: 58 170. Refer to the above information. The equilibrium quantity is: A) 10. B) 20. C) 15. D) 30. Answer: B

Type: E Topic: 7 E: 58 MI: 58 MA: 58 171. Refer to the above information. The equilibrium price for X is: A) $2. B) $4. C) $6. D) $7. Answer: C

Type: E Topic: 7 E: 58 MI: 58 MA: 58 172. Refer to the above information. If demand changed from P = 10 - .2Q to P = 7 - .3Q, we can conclude that: A) demand has increased. C) supply will increase. B) demand has declined. D) supply will decrease. Answer: B

Type: E Topic: 7 E: 58 MI: 58 MA: 58 173. Refer to the above information. After the change in demand, the new equilibrium quantity is: A) 10. B) 20. C) 15. D) 30. Answer: A

Type: E Topic: 7 E: 58 MI: 58 MA: 58 174. Refer to the above information. After the change in demand, the new equilibrium price is: A) $2. B) $4. C) $6. D) $7. Answer: B

McConnell/Brue: Economics, 16/e

Page 100

Chapter 3: Individual Markets: Demand and Supply

Government-set prices

Use the following to answer questions 175-178:

Quantity Demanded 52 62 72 82 92

Price $50 45 40 35 30

Quantity Supplied 73 62 51 42 33

Type: T Topic: 8 E: 52-53 MI: 52-53 MA: 52-53 Status: New 175. In the above market, economists would call a government-set minimum price of $50 a: A) price ceiling. B) price floor. C) equilibrium price. D) fair price. Answer: B

Type: T Topic: 8 E: 52-53 MI: 52-53 MA: 52-53 Status: New 176. In the above market, economists would call a government-set minimum price of $40 a: A) price ceiling. B) price floor. C) equilibrium price. D) fair price. Answer: A

Type: T Topic: 8 E: 54 MI: 54 MA: 54 Status: New 177. If government set a minimum price of $50 in the above market, a: A) shortage of 21 units would occur. C) surplus of 21 units would occur. B) shortage of 125 units would occur. D) surplus of 125 units would occur. Answer: C

Type: T Topic: 8 E: 53 MI: 53 MA: 53 Status: New 178. If government set a maximum price of $45 in the above market: A) a shortage of 21 units would arise. C) a surplus of 40 units would arise. B) a surplus of 21 units would arise. D) neither a shortage nor a surplus would arise. Answer: D

McConnell/Brue: Economics, 16/e

Page 101

Chapter 3: Individual Markets: Demand and Supply

Use the following to answer questions 179-183:

S

C
B

Price

A D 0 E Quantity

Type: G Topic: 8 E: 54 MI: 54 MA: 54 179. Refer to the above diagram. A government-set price floor is best illustrated by: A) price A. B) quantity E. C) price C. D) price B. Answer: C

Type: G Topic: 8 E: 53 MI: 53 MA: 53 180. Refer to the above diagram. A government-set price ceiling is best illustrated by: A) price A. B) quantity E. C) price C. D) price B. Answer: A

Type: G Topic: 8 E: 53-54 MI: 53-54 MA: 53-54 181. Refer to the above diagram. Rent controls are best illustrated by: A) price A. B) quantity E. C) price C. D) price B. Answer: A

Type: G Topic: 8 E: 54 MI: 54 MA: 54 182. Refer to the above diagram. A government price support program to aid farmers is best illustrated by: A) quantity E. B) price C. C) price A. D) price B. Answer: B

Type: G Topic: 8 E: 53 MI: 53 MA: 53 183. Refer to the above diagram. A government-set maximum permissible interest rate is best illustrated by: A) price B. B) quantity E. C) price C. D) price A. Answer: D

McConnell/Brue: Economics, 16/e

Page 102

Chapter 3: Individual Markets: Demand and Supply

Type: A Topic: 8 E: 53 MI: 53 MA: 53 184. Price floors and ceiling prices: A) both cause shortages. B) both cause surpluses. C) cause the supply and demand curves to shift until equilibrium is established. D) interfere with the rationing function of prices. Answer: D

Type: D Topic: 8 E: 54 MI: 54 MA: 54 185. A price floor means that: A) inflation is severe in this particular market. B) sellers are artificially restricting supply to raise price. C) government is imposing a legal price that is below the equilibrium price. D) government is imposing a legal price that is above the equilibrium price. Answer: D

Type: A Topic: 8 E: 53 MI: 53 MA: 53 186. An effective ceiling price will: A) induce new firms to enter the industry. B) result in a product surplus. Answer: C

C) result in a product shortage. D) clear the market.

Type: A Topic: 8 E: 54 MI: 54 MA: 54 187. An effective price floor will: A) force some firms in this industry to go out of business. B) result in a product surplus. C) result in a product shortage. D) clear the market. Answer: B

Type: C Topic: 8 E: 53 MI: 53 MA: 53 188. Other things equal, the shortage associated with a price ceiling will be greater the: A) smaller the elasticity of both demand and supply. B) greater the elasticity of both demand and supply. C) greater the elasticity of supply and the smaller the elasticity of demand. D) greater the elasticity of demand and the smaller the elasticity of supply. Answer: B

Type: A Topic: 8 E: 53 MI: 53 MA: 53 189. Black markets are associated with: A) price floors and the resulting product surpluses. B) price floors and the resulting product shortages. C) ceiling prices and the resulting product shortages. D) ceiling prices and the resulting product surpluses. Answer: C

McConnell/Brue: Economics, 16/e

Page 103

Chapter 3: Individual Markets: Demand and Supply

Type: A Topic: 8 E: 53 MI: 53 MA: 53 190. Price ceilings and price floors: A) cause surpluses and shortages respectively. B) make the rationing function of free markets more efficient. C) interfere with the rationing function of prices. D) shift demand and supply curves and therefore have no effect on the rationing function of prices. Answer: C

Type: D Topic: 8 E: 53 MI: 53 MA: 53 191. A price ceiling means that: A) there is currently a surplus of the relevant product. B) government is imposing a legal price that is below the equilibrium price. C) government wants to stop a deflationary spiral. D) government is imposing a legal price that is above the equilibrium price. Answer: B

Type: A Topic: 8 E: 53 MI: 53 MA: 53 192. If an effective ceiling price is placed on hamburgers then: A) the quantity demanded will exceed the quantity supplied. B) a black market for hamburger may evolve. C) that consumers may want government to ration hamburger. D) all of the above are likely outcomes. Answer: D

Type: A Topic: 8 E: 53 MI: 53 MA: 53 193. If a legal ceiling price is set above the equilibrium price: A) a shortage of the product will occur. B) a surplus of the product will occur. C) a black market will evolve. D) neither the equilibrium price nor equilibrium quantity will be affected. Answer: D

Type: A Topic: 8 E: 54 MI: 54 MA: 54 194. An effective price floor on wheat will: A) force otherwise profitable farmers out of business. B) result in a shortage of wheat. C) result in a surplus of wheat. D) clear the market for wheat. Answer: C

Consider This Questions

Type: F E: 50 MI: 50 MA: 50 Status: New 195. (Consider This) Alfred Marshall's scissors analogy emphasizes that: A) there is no free lunch. B) supply curves slope upward and to the right. C) changes in demand differ from changes in quantity demanded. D) supply and demand are equally important in determining equilibrium price and quantity. Answer: D

McConnell/Brue: Economics, 16/e

Page 104

Chapter 3: Individual Markets: Demand and Supply

Type: A E: 50 MI: 50 MA: 50 Status: New 196. (Consider This) The idea that equilibrium price and quantity are jointly determined by supply and demand is best illustrated through an analogy of: A) the wind and tide. B) a chess match. C) a scissors. D) two sides of a coin. Answer: C

Last Word Questions

Type: D E: 55 MI: 55 MA: 55 197. (Last Word) Ticket scalping refers to: A) the surplus of tickets that occurs when price is set below equilibrium. B) the shortage of tickets that occurs when price is set above equilibrium. C) pricing tickets so high that an athletic or artistic event will not be sold out. D) reselling a ticket at a price above its original purchase price. Answer: D

Type: A E: 55 MI: 55 MA: 55 198. (Last Word) Ticket scalping: A) imposes economic losses on both buyers and sellers. B) creates economic gains for both buyers and sellers. C) imposes losses on buyers, but creates gains for sellers. D) imposes losses on sellers, but creates gains for buyers. Answer: B

Type: A E: 55 MI: 55 MA: 55 199. (Last Word) Ticket scalping implies that: A) event sponsors have established ticket prices at above-equilibrium levels. B) an event is not likely to be sold out. C) event sponsors have established ticket prices at below-equilibrium levels. D) the demand for tickets has fallen between the time tickets were originally sold and the event takes place. Answer: C

Type: A E: 55 MI: 55 MA: 55 200. (Last Word) Ticket scalping is likely to: A) produce a less interested audience. B) reduce the well-being of ticket sellers. Answer: D

C) reduce the well-being of ticket buyers. D) produce a more interested audience.

True/False Questions

Type: A E: 48 MI: 48 MA: 48 201. Surpluses drive market prices up; shortages drive them down. Answer: False

McConnell/Brue: Economics, 16/e

Page 105

Chapter 3: Individual Markets: Demand and Supply

Type: C E: 51-52 MI: 51-52 MA: 51-52 202. If demand increases and supply simultaneously decreases, equilibrium price will rise. Answer: True

Type: D E: 50 MI: 50 MA: 50 203. The rationing function of prices refers to the fact that government must distribute any surplus goods that may be left in a competitive market. Answer: False

Type: A E: 47 MI: 47 MA: 47 204. An increase in quantity supplied might be caused by an increase in production costs. Answer: False

Type: D E: 45 MI: 45 MA: 45 205. Supply refers to the amount of a product that a producer will offer in the market at some particular price. Answer: False

Type: C E: 52 MI: 52 MA: 52 206. An increase in demand accompanied by an increase in supply will increase the equilibrium quantity but the effect on equilibrium price will be indeterminate. Answer: True

Type: A E: 47 MI: 47 MA: 47 207. A government subsidy per unit of output increases supply. Answer: True

Type: D E: 43 MI: 43 MA: 43 208. Consumers buy more of normal goods as their incomes rise. Answer: True

Type: D E: 44 MI: 44 MA: 44 209. Toothpaste and toothbrushes are substitute goods. Answer: False

Type: A E: 47 MI: 47 MA: 47 210. A government tax per unit of output reduces supply. Answer: True

Type: C E: 51 MI: 51 MA: 51 211. If market demand increases and market supply decreases, the change in equilibrium price is unpredictable without first knowing the exact magnitudes of the demand and supply changes. Answer: False

McConnell/Brue: Economics, 16/e

Page 106

Chapter 3: Individual Markets: Demand and Supply

Type: C E: 50 MI: 50 MA: 50 212. A decrease in supply of X increases the equilibrium price of X, which reduces the demand for X and automatically returns the price of X to its initial level. Answer: False

Type: A E: 49 MI: 49 MA: 49 213. In a competitive market, every consumer willing to pay the market price can buy a product and every producer willing to sell the product at that price can sell it. Answer: True

Type: A E: 54 MI: 54 MA: 54 214. A price floor in a competitive market will result in persistent shortages of a product. Answer: False

Type: A E: 53 MI: 53 MA: 53 215. A ceiling price in a competitive market will result in persistent surpluses of a product. Answer: False

McConnell/Brue: Economics, 16/e

Page 107

Bonus Web Chapter 3W

Applications and Extensions of Supply and Demand Analysis
(Found at the book's website, www.mcconnell16.com)

Topic 1. 2. 3. 4. Changes in supply and demand Preset prices Nonprice goods Consider and producer surplus Consider This Last Word True-False

Question numbers 1-14 15-27 28-32 33-52 53-54 55-57 58-68

____________________________________________________________

_______________________________________

____________________________________________________________

_______________________________________

Multiple Choice Questions Changes in supply and demand

Type: A Topic: 1 E: 2 MI: 2 MA: 2 Status: New 1. An increase in demand is shown as a: A) movement from a higher point to a lower point on an existing demand curve. B) movement from a lower point to a higher point on an existing demand curve. C) rightward shift of a demand curve. D) leftward shift of a demand curve. Answer: C

Type: A Topic: 1 E: 2 MI: 2 MA: 2 Status: New 2. A decrease in demand is shown as a: A) movement from a higher point to a lower point on an existing demand curve. B) movement from a lower point to a higher point on an existing demand curve. C) rightward shift of a demand curve. D) leftward shift of a demand curve. Answer: D

Chapter 3W: Applications and Extensions of Supply and Demand Analysis

Type: A Topic: 1 E: 2 MI: 2 MA: 2 Status: New 3. An increase in supply is shown as a: A) rightward shift of a supply curve. B) leftward shift of a supply curve. C) movement from a higher point to a lower point on an existing supply curve. D) movement from a lower point to a higher point on an existing supply curve. Answer: A

Type: A Topic: 1 E: 2 MI: 2 MA: 2 Status: New 4. A decrease in supply is shown as a: A) movement from a higher point to a lower point on an existing supply curve. B) rightward shift of a supply curve. C) movement from a lower point to a higher point on an existing supply curve. D) leftward shift of a supply curve. Answer: D

Type: A Topic: 1 E: 2 MI: 2 MA: 2 Status: New 5. An increase in demand (with no change in supply) will: A) increase equilibrium price and reduce equilibrium quantity. B) decrease equilibrium price and increase equilibrium quantity. C) increase both equilibrium price and equilibrium quantity. D) decrease both equilibrium price and equilibrium quantity. Answer: C

Type: A Topic: 1 E: 2 MI: 2 MA: 2 Status: New 6. A decrease in demand (with no change in supply) will: A) increase equilibrium price and reduce equilibrium quantity. B) decrease equilibrium price and increase equilibrium quantity. C) increase both equilibrium price and equilibrium quantity. D) decrease both equilibrium price and equilibrium quantity. Answer: D

Type: A Topic: 1 E: 2 MI: 2 MA: 2 Status: New 7. An increase in supply (with no change in demand) will: A) increase equilibrium price and reduce equilibrium quantity. B) decrease equilibrium price and increase equilibrium quantity. C) increase both equilibrium price and equilibrium quantity. D) decrease both equilibrium price and equilibrium quantity. Answer: B

Type: A Topic: 1 E: 2 MI: 2 MA: 2 Status: New 8. A decrease in supply (with no change in demand) will: A) increase equilibrium price and reduce equilibrium quantity. B) decrease equilibrium price and increase equilibrium quantity. C) increase both equilibrium price and equilibrium quantity. D) decrease both equilibrium price and equilibrium quantity. Answer: A

McConnell/Brue: Economics, 16/e

Page 110

Chapter 3W: Applications and Extensions of Supply and Demand Analysis

Type: C Topic: 1 E: 3 MI: 3 MA: 3 Status: New 9. An increase in demand that exceeds an increase in supply will: A) increase both equilibrium price and equilibrium quantity. B) increase equilibrium price and reduce equilibrium quantity. C) decrease equilibrium price and increase equilibrium quantity. D) decrease both equilibrium price and quantity. Answer: A

Type: C Topic: 1 E: 2 MI: 2 MA: 2 Status: New 10. An increase in supply that exceeds an increase in demand will: A) increase both equilibrium price and equilibrium quantity. B) increase equilibrium price and reduce equilibrium quantity. C) decrease equilibrium price and increase equilibrium quantity. D) decrease both equilibrium price and quantity. Answer: C

Type: C Topic: 1 E: 4 MI: 4 MA: 4 Status: New 11. Equal increases in demand and supply will: A) increase both equilibrium price and equilibrium quantity. B) leave equilibrium price unchanged and increase equilibrium quantity. C) decrease equilibrium price and increase equilibrium quantity. D) decrease both equilibrium price and quantity. Answer: B

Use the following to answer questions 12-14:

S1 S2

Price

D1

D2

0

Quantity

Type: G Topic: 1 E: 3 MI: 3 MA: 3 Status: New 12. Refer to the above diagram. If demand changes from D1 to D2 at the same time supply changes from S2 to S1, equilibrium price will: A) rise and equilibrium quantity will fall. C) rise, as will equilibrium quantity. B) fall and equilibrium quantity will rise. D) fall, as will equilibrium quantity. Answer: C

McConnell/Brue: Economics, 16/e

Page 111

Chapter 3W: Applications and Extensions of Supply and Demand Analysis

Type: G Topic: 1 E: 3 MI: 3 MA: 3 Status: New 13. Refer to the above diagram. If demand changes from D2 to D1 at the same time supply changes from S1 to S2, equilibrium price will: A) rise and equilibrium quantity will fall. C) rise, as will equilibrium quantity. B) fall and equilibrium quantity will rise. D) fall, as will equilibrium quantity. Answer: D

Type: G Topic: 1 E: 3 MI: 3 MA: 3 Status: New 14. Refer to the above diagram. Which of the following will increase equilibrium price and reduce equilibrium quantity? A) a shift of supply from S1 to S2, with demand remaining at D1. B) a shift of supply from S2 to S1, with demand remaining at D2. C) simultaneous shifts of demand from D1 to D2 and supply from S1 to S2. D) simultaneous shifts of demand from D2 to D1 and supply from S2 to S1. Answer: B

Preset prices

Type: A Topic: 2 E: 4 MI: 4 MA: 4 Status: New 15. If the preset price of a product is below the equilibrium price: A) the demand curve will shift to the right. B) the supply curve will shift to the left. C) a surplus will occur and a secondary market will emerge. D) a shortage will occur and a secondary market will emerge. Answer: D

Type: A Topic: 2 E: 5 MI: 5 MA: 5 Status: New 16. If the preset price of a product is above the equilibrium price: A) the demand curve will shift to the right. B) the supply curve will shift to the left. C) a surplus of unsold goods will occur. D) a shortage will occur and a secondary market will emerge. Answer: C

Type: A Topic: 2 E: 4-5 MI: 4-5 MA: 4-5 Status: New 17. If the preset price of a product equals the equilibrium price: A) neither a surplus nor shortage will occur. B) a surplus will occur and a secondary market will emerge. C) the supply curve will shift to the left. D) a shortage will occur and a secondary market will emerge. Answer: A

McConnell/Brue: Economics, 16/e

Page 112

Chapter 3W: Applications and Extensions of Supply and Demand Analysis

Type: A Topic: 2 E: 4 MI: 4 MA: 4 Status: New 18. A shortage of a product will occur and a secondary market will emerge if the: A) product supply curve suddenly shifts to the right. B) preset price is below the equilibrium price. C) product demand curve suddenly shifts to the left. D) preset price is above the equilibrium price. Answer: B

Use the following to answer questions 19-21:

S1

P3

Price

P2 D3 P1 D1 D2

0

Q1

Quantity
Type: G Topic: 2 E: 5 MI: 5 MA: 5 Status: New 19. Refer to the above diagram. Suppose that a seller offers Q1 units of tickets to some event, regardless of price, as shown by supply curve S1. If the seller presets the price at P2 and demand turns out to be D1: A) a shortage will occur and a secondary market will emerge. B) the event will not be a sellout. C) the event will be a sellout. D) the seller will necessarily experience a loss. Answer: B

Type: G Topic: 2 E: 4 MI: 4 MA: 4 Status: New 20. Refer to the above diagram. Suppose that a seller offers Q1 units of tickets to some event, regardless of price, as shown by supply curve S1. If the seller presets the price at P2 and demand turns out to be D3: A) a shortage will occur and a secondary market will emerge. B) the event will not be a sellout. C) the price of tickets in the secondary market will be below the preset price. D) the seller will necessarily experience a loss. Answer: A

McConnell/Brue: Economics, 16/e

Page 113

Chapter 3W: Applications and Extensions of Supply and Demand Analysis

Type: G Topic: 2 E: 4-5 MI: 4-5 MA: 4 Status: New 21. Refer to the above diagram. Suppose that a seller offers Q1 units of tickets to some event, regardless of price, as shown by supply curve S1. If the seller presets the price at P2 and demand turns out to be D2: A) a shortage will occur and a secondary market will emerge. B) the event will not be a sellout. C) the price of tickets in the secondary market will be below the preset price. D) the event will be a sellout. Answer: D

Use the following to answer questions 22-25:
Quantity demanded (thousands) 40 50 60 70 80 Price $80 60 40 20 10 Quantity supplied (thousands) 60 60 60 60 60

Type: G Topic: 2 E: 4-5 MI: 4-5 MA: 4-5 Status: New 22. Suppose that the market above is for tickets to a specific football game. We can conclude that the: A) preset ticket price will necessarily be $40. B) stadium capacity is 60,000. C) stadium capacity can be adjusted from 40,000 to 80,000. D) preset ticket price will necessarily be $80. Answer: B

Type: G Topic: 2 E: 5 MI: 5 MA: 5 Status: New 23. Suppose that the market above is for tickets to a specific football game. If the preset ticket price is $60: A) ticket scalping will occur. C) a surplus of 20,000 tickets will occur. B) a shortage of 10,000 tickets will occur. D) 10,000 seats will remain unfilled. Answer: D

Type: G Topic: 2 E: 4 MI: 4 MA: 4 Status: New 24. Suppose that the market above is for tickets to a specific football game. If the preset ticket price is $20: A) a shortage of 20,000 tickets will occur. C) a surplus of 10,000 tickets will occur. B) ticket scalping will occur. D) 20,000 seats will remain unfilled. Answer: B

Type: G Topic: 2 E: 4-5 MI: 4-5 MA: 4-5 Status: New 25. Suppose that the market above is for tickets to a specific football game. If the preset ticket price is $40: A) a shortage of 10,000 tickets will occur. C) all the tickets will be sold. B) 10,000 seats will remain unfilled. D) a surplus of 20,000 tickets will occur. Answer: C

McConnell/Brue: Economics, 16/e

Page 114

Chapter 3W: Applications and Extensions of Supply and Demand Analysis

Type: A Topic: 2 E: 4 MI: 4 MA: 4 Status: New 26. Where there is a secondary market in which the price of an item exceeds the original preset price: A) the preset price was above the equilibrium price. B) quantity supplied exceeded quantity demanded in the original market. C) the preset price was below the equilibrium price. D) the supply of the item must have increased after the price was first set. Answer: C

Type: A Topic: 2 E: 5 MI: 5 MA: 5 Status: New 27. Where there is a surplus of unsold goods: A) the preset price was above the equilibrium price. B) quantity demanded exceeded quantity supplied in the original market. C) the preset price was below the equilibrium price. D) the demand for the item must have increased after the price was first set. Answer: A

Nonprice goods

Type: D Topic: 3 E: 6 MI: 6 MA: 6 Status: New 28. A nonpriced good: A) is always in short supply relative to demand. B) is always in abundant supply relative to demand. C) is owned in common by the public and not priced in markets. D) has a perfectly horizontal supply curve. Answer: C

Type: A Topic: 3 E: 6 MI: 6 MA: 6 Status: New 29. An example of a nonpriced good is a: A) computer monitor. B) wild mushroom. C) can of soda. Answer: B

D) pair of scissors.

McConnell/Brue: Economics, 16/e

Page 115

Chapter 3W: Applications and Extensions of Supply and Demand Analysis

Use the following to answer questions 30-31:

S1 D2

Price

D1

0

a

Quantity
Type: G Topic: 3 E: 6 MI: 6 MA: 6 Status: New 30. Suppose the diagram above represents the hypothetical demand and supply curves for beach clams, a nonpriced good owned in common. If demand is D1, a: A) shortage of ba will occur and clams will be overharvested. B) surplus of ba will occur and there will be no need for licenses, season restrictions, and limits. C) shortage of cb will occur and clams will be overharvested. D) surplus of cb will occur and there will no need for licenses, season restrictions, and limits. Answer: B

b

c

Type: G Topic: 3 E: 7 MI: 7 MA: 7 Status: New 31. Suppose the diagram above represents the hypothetical demand and supply curves for beach clams, a nonpriced good owned in common. If demand is D2, a: A) shortage of ba will occur and clams will be overharvested. B) surplus of ba will occur and there will be no need for licenses, season restrictions, and limits. C) shortage of cb will occur and clams will be overharvested. D) surplus of cb will occur and there will no need for licenses, season restrictions, and limits. Answer: C

Type: F Topic: 3 E: 7 MI: 7 MA: 7 Status: New 32. Faced with continuous rising demand, many goods or resources owned in common (such as fish in oceans): A) increase in price and expand in total availability. B) decline in price and expand in total availability. C) are overconsumed and eventually exhausted, unless protected through restrictions. D) are underconsumed and eventually neglected, unless promoted by government. Answer: C

McConnell/Brue: Economics, 16/e

Page 116

Chapter 3W: Applications and Extensions of Supply and Demand Analysis

Consumer and producer surplus

Type: D Topic: 4 E: 7 MI: 7 MA: 7 Status: New 33. Consumer surplus: A) is the difference between the maximum prices consumers are willing to pay for a product and the lower equilibrium price. B) the difference between the maximum prices consumers are willing to pay for a product and the minimum prices producers are willing to accept. C) the difference between the minimum prices producers are willing to accept for a product and the higher equilibrium price. D) rises as equilibrium price rises. Answer: A

Type: D Topic: 4 E: 8 MI: 8 MA: 8 Status: New 34. Producer surplus: A) is the difference between the maximum prices consumers are willing to pay for a product and the lower equilibrium price. B) rises as equilibrium price falls. C) is the difference between the minimum prices producers are willing to accept for a product and the higher equilibrium price. D) is the difference between the maximum prices consumers are willing to pay for a product and the minimum prices producers are willing to accept. Answer: C

Type: C Topic: 4 E: 8 MI: 8 MA: 8 Status: New 35. Jennifer buys a piece of costume jewelry for $33 for which she was willing to pay $42. The minimum acceptable price to the seller, Nathan, was $39. Jennifer experiences: A) a consumer surplus of $75 and Nathan experiences a producer surplus of $3. B) a producer surplus of $9 and Nathan experiences a consumer surplus of $3. C) a consumer surplus of $9 and Nathan experiences a producer surplus of $3. D) a producer surplus of $75 and Nathan experiences a producer surplus of $81. Answer: C

Type: C Topic: 4 E: 8 MI: 8 MA: 8 Status: New 36. Amanda buys a ruby for $330 for which she was willing to pay $340. The minimum acceptable price to the seller, Tony, was $140. Amanda experiences: A) a consumer surplus of $10 and Tony experiences a producer surplus of $190. B) a producer surplus of $200 and Tony experiences a consumer surplus of $10. C) a consumer surplus of $770 and Tony experiences a producer surplus of $480. D) a producer surplus of $10 and Tony experiences a consumer surplus of $190. Answer: A

Type: F Topic: 4 E: 8 MI: 8 MA: 8 Status: New 37. Graphically, consumer surplus is measured as the triangle: A) under the demand curve and below the actual price. B) under the demand curve and above the actual price. C) above the supply curve and above the actual price. D) above the supply curve and below the actual price. Answer: B

McConnell/Brue: Economics, 16/e

Page 117

Chapter 3W: Applications and Extensions of Supply and Demand Analysis

Type: F Topic: 4 E: 9 MI: 9 MA: 9 Status: New 38. Graphically, producer surplus is measured as the triangle: A) under the demand curve and below the actual price. B) under the demand curve and above the actual price. C) above the supply curve and above the actual price. D) above the supply curve and below the actual price. Answer: D

Use the following to answer questions 39-47:

S

a Price b P1 c d e f

D 0 Q1 Q2 Quantity Q3

Type: G Topic: 4 E: 8 MI: 8 MA: 8 Status: New 39. Refer to the above diagram. Assuming equilibrium price P1, consumer surplus is represented by areas: A) a + b. B) a + b + c + d. C) c + d. D) a + c. Answer: A

Type: G Topic: 4 E: 9 MI: 9 MA: 9 Status: New 40. Refer to the above diagram. Assuming equilibrium price P1, producer surplus is represented by areas: A) a + b. B) a + b + c + d. C) c + d. D) a + c. Answer: C

Type: G Topic: 4 E: 10 MI: 10 MA: 10 Status: New 41. Refer to the above diagram. The area that identifies the maximum sum of consumer surplus and producer surplus is: A) a + b + c + d + e + f. B) c + d + f. C) a + b + e. D) a + b + c + d. Answer: D

McConnell/Brue: Economics, 16/e

Page 118

Chapter 3W: Applications and Extensions of Supply and Demand Analysis

Type: G Topic: 4 E: 10 MI: 10 MA: 10 Status: New 42. Refer to the above diagram. At Q1: A) efficiency is achieved. C) consumer surplus is maximized. B) an efficiency loss of b + d occurs. D) an efficiency loss of e + d occurs. Answer: B

Type: G Topic: 4 E: 10 MI: 10 MA: 10 Status: New 43. Refer to the above diagram. At Q2: A) efficiency is achieved. C) an efficiency loss of a + b + c + d occurs. B) an efficiency loss of a + c occurs. D) an efficiency loss of e + f occurs. Answer: A

Type: G Topic: 4 E: 10 MI: 10 MA: 10 Status: New 44. Refer to the above diagram. At Q3: A) efficiency is achieved. C) an efficiency loss of a + b + c + d occurs. B) an efficiency loss of e + f occurs. D) an efficiency loss of a + c occurs. Answer: B

Type: G Topic: 4 E: 10 MI: 10 MA: 10 Status: New 45. Refer to the above diagram. At Q1: A) maximum willingness to pay exceeds minimum acceptable price. B) the sum of consumer and producer surplus is maximized. C) minimum acceptable price exceeds maximum willingness to pay. D) an efficiency loss of a + b occurs. Answer: A

Type: G Topic: 4 E: 10 MI: 10 MA: 10 Status: New 46. Refer to the above diagram. At Q3: A) maximum willingness to pay exceeds minimum acceptable price. B) the sum of consumer and producer surplus is maximized. C) minimum acceptable price exceeds maximum willingness to pay. D) an efficiency loss of a + b occurs. Answer: C

Type: G Topic: 4 E: 10 MI: 10 MA: 10 Status: New 47. Refer to the above diagram. At Q2: A) maximum willingness to pay exceeds minimum acceptable price. B) the sum of consumer and producer surplus is maximized. C) minimum acceptable price exceeds maximum willingness to pay. D) an efficiency loss of b + d occurs. Answer: B

McConnell/Brue: Economics, 16/e

Page 119

Chapter 3W: Applications and Extensions of Supply and Demand Analysis

Type: F Topic: 4 E: 10 MI: 10 MA: 10 Status: New 48. Allocative efficiency occurs only at that output where: A) marginal benefit exceeds marginal cost the by the greatest amount. B) consumer surplus exceeds producer surplus by the greatest amount. C) the combined amounts of consumer surplus and producer surplus are maximized. D) the areas of consumer and producer surplus are equal. Answer: C

Type: F Topic: 4 E: 10 MI: 10 MA: 10 Status: New 49. At the output level defining allocative efficiency: A) the areas of consumer and producer surplus necessarily are equal. B) marginal benefit exceeds marginal cost the by the greatest amount. C) consumer surplus exceeds producer surplus by the greatest amount. D) the maximum willingness to pay for the last unit of output equals the minimum acceptable price of that unit of output. Answer: D

Type: F Topic: 4 E: 10 MI: 10 MA: 10 Status: New 50. At the output where the combined amounts of consumer and producer surplus are largest: A) the areas of consumer and producer surplus necessarily are equal. B) the maximum willingness to pay for the last unit of output equals the minimum acceptable price of that unit of output. C) consumer surplus exceeds producer surplus by the greatest amount. D) marginal benefit exceeds marginal cost by the greatest amount. Answer: B

Type: D Topic: 4 E: 10 MI: 10 MA: 10 Status: New 51. An efficiency loss: A) is measured as the combined loss of consumer surplus and producer surplus. B) results from producing a unit of output for which the maximum willingness to pay exceeds the minimum acceptable price. C) can result from underproduction, but not from overproduction. D) can result from overproduction, but not from underproduction. Answer: A

Type: A Topic: 4 E: 10 MI: 10 MA: 10 Status: New 52. An efficiency loss declines in size when a unit of output is produced for which: A) marginal cost exceeds marginal benefit. B) maximum willingness to pay exceeds minimum acceptable price. C) consumer surplus exceeds producer surplus. D) producer surplus exceeds consumer surplus. Answer: B

McConnell/Brue: Economics, 16/e

Page 120

Chapter 3W: Applications and Extensions of Supply and Demand Analysis

Consider This Questions

Type: F E: 5 MI: 5 MA: 5 Status: New 53. (Consider This) Why, in the past, did some rock superstars price their concert tickets below equilibrium levels? A) To reward loyal fans who could not afford the higher prices B) Because they systematically underestimated the equilibrium prices C) To make sure their concerts were completely sold out D) To create long ticket lines, scalping, and the attendant press publicity, which helped boost sales of CDs Answer: D

Type: F E: 5 MI: 5 MA: 5 Status: New 54. (Consider This) In recent years, many rock stars have abandoned their previous practice of pricing concert tickets below their equilibrium levels. The main reason for the change in pricing strategy is that: A) illegal downloading of CDs has reduced the value of the free publicity associated with long ticket lines and scalping activity. B) computer software has made it easier to estimate equilibrium prices in advance. C) concert goers have considerably higher incomes than in the past. D) rock stars tend to be greedier than in the old days. Answer: A

Last Word Questions

Type: D E: 11 MI: 11 MA: 11 Status: New 55. (Last Word) A prescription drug that has the same chemical properties and product quality as a previously patented drug is called a: A) synergistic drug. B) knock-off drug. C) generic drug. D) quasi-legal drug. Answer: C

Type: A E: 11 MI: 11 MA: 11 Status: New 56. (Last Word) When a patent expires on a brand name drug, and generic drugs emerge to compete with it: A) consumer surplus increases. C) allocative inefficiency worsens. B) efficiency losses rise. D) consumer surplus declines. Answer: A

Type: A E: 11 MI: 11 MA: 11 Status: New 57. (Last Word) Generic drugs: A) increase the price of drugs to consumers and increase consumer surplus. B) reduce the price of a drugs to consumers and reduce consumer surplus. C) increase the price of drugs to consumers and reduce consumer surplus. D) reduce the price of a drug to consumer and increase consumer surplus. Answer: D

McConnell/Brue: Economics, 16/e

Page 121

Chapter 3W: Applications and Extensions of Supply and Demand Analysis

True/False Questions

Type: A E: 2 MI: 2 MA: 2 Status: New 58. An increase in demand with no change in supply will increase equilibrium price and reduce equilibrium quantity. Answer: False

Type: A E: 2 MI: 2 MA: 2 Status: New 59. A decrease in supply with no change in demand will increase equilibrium price and reduce equilibrium quantity. Answer: True

Type: A E: 4 MI: 4 MA: 4 Status: New 60. Equal decreases in demand and supply will leave equilibrium price unchanged and reduce equilibrium quantity. Answer: True

Type: A E: 4 MI: 4 MA: 4 Status: New 61. If a preset price is below the equilibrium price, a surplus of unsold goods will occur. Answer: False

Type: A E: 4 MI: 4 MA: 4 Status: New 62. If a preset price is below the equilibrium price, a shortage and secondary market will occur. Answer: True

Type: F E: 6 MI: 6 MA: 6 Status: New 63. Wild game (elk, deer, antelope, bears) on public lands are examples of nonpriced goods. Answer: True

Type: F E: 6 MI: 6 MA: 6 Status: New 64. Faced with rising demand, some nonpriced goods eventually get consumed beyond their sustainable levels. Answer: True

Type: A E: 8 MI: 8 MA: 8 Status: New 65. Along a demand curve, product price and consumer surplus are inversely related. Answer: True

Type: A E: 9 MI: 9 MA: 9 Status: New 66. Along a supply curve, product price and producer surplus are inversely related. Answer: False

McConnell/Brue: Economics, 16/e

Page 122

Chapter 3W: Applications and Extensions of Supply and Demand Analysis

Type: A E: 10 MI: 10 MA: 10 Status: New 67. Allocative efficiency occurs where (for the last unit) maximum willingness to pay exceeds minimum acceptable price by the greatest amount. Answer: False

Type: A E: 10 MI: 10 MA: 10 Status: New 68. Allocative efficiency occurs where the collective sum of consumer and producer surplus is at a maximum. Answer: True

McConnell/Brue: Economics, 16/e

Page 123

CHAPTER 4

The Market System

Topic 1. Characteristics of the market system; capital accumulation 2. Specialization 3. Exchange; use of money 4. Market system; profits and costs 5. Least-cost combination of resources 6. Consumer sovereignty; invisible hand Consider This Last Word True-False

Question numbers

____________________________________________________________

_______________________________________

1-14 15-18 19-24 25-49 50-57 58-72 73-74 75-76 77-87

____________________________________________________________

_______________________________________

Multiple Choice Questions Characteristics of the market system; capital accumulation

Type: D Topic: 1 E: 60-62 MI: 60-62 MA: 60-62 1. Which of the following is not a characteristic of the market system? A) private property C) government ownership of the major industries B) freedom of enterprise D) competition in product and resource markets Answer: C

Type: D Topic: 1 E: 60 MI: 60 MA: 60 2. Which of the following is a fundamental characteristic of the market system? A) property rights C) unselfish behavior B) central planning by government D) government-set wages and prices Answer: A

Type: A Topic: 1 E: 61 MI: 61 MA: 61 3. Copyrights and trademarks are examples of: A) capital goods. B) human capital. C) property rights. D) public goods. Answer: C

Chapter 4: The Market System

Type: A Topic: 1 E: 61 MI: 61 MA: 61 4. The pursuit of self-interest: A) is highly detrimental to the market system. B) means the same as "selfishness." C) is reflected in the behavior of firms, but not in the behavior of consumers. D) gives direction to the market system. Answer: D

Type: A Topic: 1 E: 62 MI: 62 MA: 62 5. The regulatory mechanism of the market system is: A) self-interest. B) private property. C) competition. D) specialization. Answer: C

Type: D Topic: 1 E: 62 MI: 62 MA: 62 6. Broadly defined, competition involves: A) private property and freedom of expression. B) independently acting buyers and sellers and freedom to enter or leave markets. C) increasing opportunity costs and diminishing marginal utility. D) capital goods and division of labor. Answer: B

Type: D Topic: 1 E: 62 MI: 62 MA: 62 7. Competition means that: A) sellers can manipulate market price by causing product scarcities. B) there are independently-acting buyers and sellers in each market. C) a product can be purchased at a number of different prices. D) there is more than one seller in a market. Answer: B

Type: A Topic: 1 E: 61 MI: 61 MA: 61 8. Well-defined property rights: A) discourage investment and growth. B) discourage hard work. C) impede exchange. D) encourage owners to maintain or improve their property. Answer: D

Type: A Topic: 1 E: 62 MI: 62 MA: 62 9. To increase the amount of capital goods in a full employment-full production economy, society must: A) decrease consumption. C) inflate the general price level. B) increase consumption. D) deflate the general price level. Answer: A

McConnell/Brue: Economics, 16/e

Page 126

Chapter 4: The Market System

Type: A Topic: 1 E: 63 MI: 63 MA: 63 10. Roundabout production usually leads to: A) the use of fewer capital goods. B) smaller markets. C) greater production. D) simultaneous increases in consumer goods production. Answer: C

Type: A Topic: 1 E: 62 MI: 62 MA: 62 11. Which of the following is an example of a capital good? A) a Federal government bond. C) a bag of potato chips B) a share of General Motors stock D) a Boeing 777 airplane Answer: D

Type: A Topic: 1 E: 62 MI: 62 MA: 62 12. If an economy is operating on its production possibilities curve, an increase in the production of capital goods: A) necessarily involves an increase in the division of labor. B) is in conflict with the concept of consumer sovereignty. C) necessitates production of fewer consumer goods. D) will impair future productive efficiency. Answer: C

Type: A Topic: 1 E: 62-63 MI: 62-63 MA: 62-63 13. The basic reason for the production of capital goods is to: A) promote the concentration of economic power in the business sector. B) facilitate exchange where a coincidence of wants does not exist. C) better synchronize the operation of resource and product markets. D) enhance future productive efficiency. Answer: D

Type: D Topic: 1 E: 63 MI: 63 MA: 63 14. Roundabout production refers to the: A) use of capital goods in the production process. B) double exchange of money for goods and goods for money. C) use of money as a medium of exchange. D) fact that barter impedes specialization. Answer: A

Specialization

Type: D Topic: 2 E: 63 MI: 63 MA: 63 15. The division of labor means that: A) labor markets are geographically segmented. B) unskilled workers outnumber skilled workers. Answer: C

C) workers specialize in various production tasks. D) each worker performs a large number of tasks.

McConnell/Brue: Economics, 16/e

Page 127

Chapter 4: The Market System

Type: D Topic: 2 E: 63 MI: 63 MA: 63 16. Specialization in production is important primarily because it: A) results in greater total output. B) allows society to avoid the coincidence-of-wants problem. C) allows society to trade by barter. D) allows society to have fewer capital goods. Answer: A

Type: D Topic: 2 E: 63 MI: 63 MA: 63 17. Specialization--the division of labor--enhances productivity and efficiency by: A) allowing workers to take advantage of existing differences in their abilities and skills. B) avoiding the time loss involved in shifting from one production task to another. C) allowing workers to develop skills by working on one, or a limited number, of tasks. D) all of the above means. Answer: D

Type: D Topic: 2 E: 63 MI: 63 MA: 63 18. Specialization in production is economically beneficial primarily because it: A) allows everyone to have a job which they like. B) permits the production of a larger output with fixed amounts of resources. C) facilitates trade by bartering. D) guarantees full employment. Answer: B

Exchange; use of money

Use the following to answer questions 19-21:

Type: A Topic: 3 E: 63 MI: 63 MA: 63 19. On the basis of the above information it can be said that: A) no coincidence of wants exists between any two states. B) a coincidence of wants exists between Michigan and Washington. C) a coincidence of wants exists between Texas and Washington. D) a coincidence of wants exists between Michigan and Texas. Answer: A

McConnell/Brue: Economics, 16/e

Page 128

Chapter 4: The Market System

Type: A Topic: 3 E: 63 MI: 63 MA: 63 20. On the basis of the above information and assuming trade occurs between the three states we can expect: A) Washington to exchange apples with Texas and receive money in return. B) Washington to exchange apples with Michigan and receive money in return. C) Texas to exchange lettuce with Michigan and receive autos in return. D) Texas to trade lettuce directly for Washington apples. Answer: A

Type: A Topic: 3 E: 64 MI: 64 MA: 64 21. Given the above information and assuming trade occurs between the three states we can expect: A) that there is no means by which Michigan can obtain lettuce while specializing in the production of autos. B) that money will not be needed to accomplish the desired exchanges. C) money to flow counterclockwise from Michigan to Texas to Washington. D) money to flow clockwise from Michigan to Washington to Texas. Answer: C

Type: D Topic: 3 E: 63 MI: 63 MA: 63 22. Barter: A) is the major means of exchange in centrally planned economies. B) accounts for over 30 percent of the dollar volume of all exchange in the U.S. economy. C) entails the exchange of goods for goods. D) is used to circumvent the problem of a lack of coincidence of wants among potential buyers and sellers. Answer: C

Type: D Topic: 3 E: 63 MI: 63 MA: 63 23. The coincidence-of-wants problem associated with barter refers to the fact that: A) for exchange to occur each seller must have a product that some buyer wants. B) money must be used as a medium of exchange or trade will never occur. C) specialization is restricted by the size or scope of a market. D) buyers in resource markets and sellers in product markets can never engage in exchange. Answer: A

Type: A Topic: 3 E: 63 MI: 63 MA: 63 24. The use of money contributes to economic efficiency because: A) governmental direction of the production and distribution of output can be avoided by using money. B) roundabout production could not occur without the availability of money. C) it is necessary for the creation of capital goods. D) it promotes specialization by overcoming the problems with barter. Answer: D

McConnell/Brue: Economics, 16/e

Page 129

Chapter 4: The Market System

Market system; profits and costs

Type: D Topic: 4 E: 65 MI: 65 MA: 65 25. Which of the following is one of the Four Fundamental Questions? A) Which products will be in scarce supply and which in excess supply? B) Who should appoint the head of the central bank? C) How much should the society save? D) What goods and services will be produced? Answer: D

Type: D Topic: 4 E: 65 MI: 65 MA: 65 26. Normal profit is: A) a cost because any excess of total receipts over total costs will go to the businessperson. B) a cost because they represent payments made for the resources which the businessperson owns and supplies in his or her own enterprise. C) not a cost because a firm can avoid this payment by temporarily closing down. D) not a cost of production because it need not be realized for a firm to retain entrepreneurial ability. Answer: B

Type: D Topic: 4 E: 65 MI: 65 MA: 65 27. Economic profit is: A) a cost because it is really a part of wage costs. B) a cost because it accrues to the entrepreneur. C) not a cost because it cannot be calculated. D) not an economic cost because it need not be realized for a business to acquire and retain entrepreneurial ability. Answer: D

Type: D Topic: 4 E: 65 MI: 65 MA: 65 28. A firm's economic profit is: A) usually lower than its normal profit. B) profit over and above that which it needs to compensate for the time and other resources the owner supplies to the business. C) a cost of production. D) a signal to the firm that it is producing too much output. Answer: B

Type: A Topic: 4 E: 65 MI: 65 MA: 65 29. Which of the following is not an economic cost? A) wages B) rents C) economic profits D) normal profits Answer: C

Type: A Topic: 4 E: 65 MI: 65 MA: 65 30. If competitive industry Z is making substantial economic profit, output will: A) fall, product price will fall, and economic profit will disappear. B) fall, product price will rise, and economic profit will disappear. C) expand, product price will fall, and economic profit will disappear. D) expand, product price will fall, and economic profit will rise. Answer: C

McConnell/Brue: Economics, 16/e

Page 130

Chapter 4: The Market System

Type: A Topic: 4 E: 65-66 MI: 65-66 MA: 65-66 31. When a competitive industry is in equilibrium: A) economic profit will be zero. B) product demand and derived demand are equal. C) the economizing problem will have been solved for that industry. D) normal profit will not be realized. Answer: A

Type: A Topic: 4 E: 65 MI: 65 MA: 65 32. From society's point of view the economic function of profits and losses is to: A) promote the equal distribution of real assets and wealth. B) achieve full employment and price level stability. C) contribute to a more equal distribution of income. D) reallocate resources from less desired to more desired uses. Answer: D

Type: A Topic: 4 E: 66 MI: 66 MA: 66 33. In a market economy a significant change in the demand for product X will: A) alter the profits or losses received by certain firms. B) cause a reallocation of scarce resources. C) cause some industries to expand and others to contract. D) do all of the above. Answer: D

Type: A Topic: 4 E: 65 MI: 65 MA: 65 34. Economic profits in an industry suggests the industry: A) can earn more profits by increasing product price. B) should be larger to better satisfy consumer demand. C) has excess production capacity. D) is the size that consumers want it to be. Answer: B

Type: A Topic: 4 E: 65 MI: 65 MA: 65 35. Economic profits and losses: A) are both considered by economists to be a part of production costs. B) are essential to the reallocation of resources from less desired goods to more desired goods. C) have no influence on the composition of the domestic output. D) equalize the distribution of income in the long run. Answer: B

Type: A Topic: 4 E: 68 MI: 68 MA: 68 36. If an increase occurs in the demand for product X, all of the following will occur except: A) an increase in the profits of industry X. B) an increase in the demand for resources employed by industry X. C) an increase in the output of industry X. D) a decrease in the prices of resources employed in industry X. Answer: D

McConnell/Brue: Economics, 16/e

Page 131

Chapter 4: The Market System

Type: A Topic: 4 E: 66 MI: 66 MA: 66 37. An increase in demand for strawberries is most likely to: A) increase the demand for strawberry pickers. C) reduce the supply of strawberries. B) reduce the supply of strawberry pickers. D) reduce the demand for strawberry pickers. Answer: A

Type: A Topic: 4 E: 66 MI: 66 MA: 66 38. If competitive industry Y is incurring substantial losses, output will: A) expand, product price will rise, and losses will disappear. B) contract, product price will fall, and losses will increase. C) contract, product price will rise, and losses will disappear. D) expand, product price will fall, and losses will disappear. Answer: C

Type: D Topic: 4 E: 65 MI: 65 MA: 65 39. The economic function of profits and losses is to: A) bring about a more equal distribution of income. B) signal that resources should be reallocated. C) eliminate small firms and reduce competition. D) tell government which industries need to be subsidized. Answer: B

Type: A Topic: 4 E: 65-66 MI: 65-66 MA: 65-66 40. In a competitive economy prices: A) influence consumers in their purchases of goods and services. B) influence businesses in their purchases of economic resources. C) influence workers in making occupational choices. D) do all of the above. Answer: D

Type: A Topic: 4 E: 65 MI: 65 MA: 65 41. Suppose a firm's total economic cost in producing 1,000 aluminum baseball bats is $10,000. These bats are then sold by the firm for $12,000. Thus: A) the firm is necessarily using the least-cost production technique because it is realizing an economic profit. B) the firm's normal profit is $2000. C) the firm's economic profit is $2000. D) there is no economic reason for the aluminum bat industry to expand or contract. Answer: C

Type: A Topic: 4 E: 65 MI: 65 MA: 65 42. If a competitive industry is neither expanding nor contracting, we would expect: A) normal profits to be zero. B) economic profits to be zero. C) consumer demand and derived demand to be equal. D) external costs or benefits to be large. Answer: B

McConnell/Brue: Economics, 16/e

Page 132

Chapter 4: The Market System

Type: A Topic: 4 E: 65 MI: 65 MA: 65 43. Suppose industry A is realizing substantial economic profit. Which of the following best describes the adjustment process that would bring about a new equilibrium? A) Firms will leave the industry, output will fall, and product price will rise. B) Firms will enter the industry, output will rise, and product price will rise. C) Firms will leave the industry, output will rise, and product price will fall. D) Firms will enter the industry, output will rise, and product price will fall. Answer: D

Type: D Topic: 4 E: 65 MI: 65 MA: 65 44. An industry is in long-run equilibrium when: A) normal profits are zero. B) total revenue exceeds total economic costs. Answer: D

C) total economic costs exceed total revenue. D) economic profits are zero.

Type: D Topic: 4 E: 66 MI: 66 MA: 66 45. When economists say that the demand for a resource is a derived demand, they mean that: A) producers substitute low-priced for high-priced resources. B) the demand for resources depends on the demand for the product that those resources produce. C) government demand complements private demand for most goods and services. D) resource demand curves are often upsloping. Answer: B

Type: D Topic: 4 E: 66 MI: 66 MA: 66 46. Which of the following best reflects the idea of derived demand? A) Product demand is determined by the demand for resources. B) The demand for automobiles will decline if the price of gasoline goes up. C) Economic resources are demanded because there is a demand for the goods they produce. D) The dollar votes of consumers determine the composition of output. Answer: C

Type: A Topic: 4 E: 65 MI: 65 MA: 65 47. The competitive market system: A) encourages innovation because government provides tax breaks and subsidies to those who develop new products or new productive techniques. B) discourages innovation because it is difficult to acquire additional capital in the form of new machinery and equipment. C) discourages innovation because firms want to get all the profits possible from existing machinery and equipment. D) encourages innovation because successful innovators are rewarded with economic profits. Answer: D

Type: A Topic: 4 E: 67 MI: 67 MA: 67 48. An increase in consumer demand for product X increases the demand for resources used in producing X. This indicates that: A) consumer sovereignty is not functioning in this economy. B) the idea of derived demand is relevant. C) production must be taking place at a loss. D) firms are failing to use the least-cost production technique. Answer: B

McConnell/Brue: Economics, 16/e

Page 133

Chapter 4: The Market System

Type: A Topic: 4 E: 67-68 MI: 67-68 MA: 67-68 49. In a market economy the distribution of income will be determined primarily by: A) consumer needs and preferences. B) the quantities and prices of the resources that households supply. C) government regulations that provide a minimum income for all. D) a social consensus as to what distribution of income is most equitable. Answer: B

Least-cost combination of resources

Type: D Topic: 5 E: 67 MI: 67 MA: 67 50. The most efficient combination of resources in producing any output is the combination that: A) comes closest to using the same quantities of land, labor, capital, and entrepreneurial ability. B) can be obtained for the smallest money outlay. C) uses the smallest total quantity of all resources. D) conserves most on the use of labor. Answer: B

Use the following to answer questions 51-54: Answer the next question(s) using the following data which show all available techniques for producing 20 units of a particular commodity:
Resource Land Labor Capital Entrepreneurial ability Resource prices $4 3 3 2 Possible production techniques #1 #2 #3 #4 4 4 2 2 2 1 1 4 2 1 5 3 1 4 3 1 #5 4 3 2 1

Type: T Topic: 5 E: 67 MI: 67 MA: 67 51. Refer to the above data. In view of the indicated resource prices, the economically most efficient production technique(s) is (are) technique(s): A) #1. B) #2 and #4. C) #3. D) #1 and #3. Answer: B

Type: T Topic: 5 E: 67 MI: 67 MA: 67 52. Refer to the above data. Assuming that the firm is motivated by self-interest and that the 20 units which can be produced with each technique can be sold for $2 per unit, the firm will: A) realize an economic profit of $10. B) realize an economic profit of $4. C) only make a normal profit. D) close down rather than incur a loss by producing. Answer: A

McConnell/Brue: Economics, 16/e

Page 134

Chapter 4: The Market System

Type: T Topic: 5 E: 67 MI: 67 MA: 67 53. Refer to the above data. Which of the following statements concerning this industry is correct? A) Firms in this industry will find that firms in other industries are able to outbid them for resources. B) The industry will contract as firms are forced out of business. C) The industry will expand as new firms enter. D) The industry is in equilibrium in that there is no reason for it to expand or contract. Answer: C

Type: T Topic: 5 E: 67 MI: 67 MA: 67 54. Refer to the above data. If a new production technique is developed that enables a firm to produce 20 units of output with 3 units of land, 3 of labor, 1 of capital, and 2 of entrepreneurial ability, this technique would: A) not be adopted because, although it reduces production costs, it does not increase profit. B) be adopted because it would lower production costs and increase economic profit. C) not be adopted because it entails higher production costs than other available techniques. D) be adopted, even though economic profits would be reduced slightly. Answer: B

Use the following to answer questions 55-56: Answer the next question(s) on the basis of the following information: Suppose 30 units of product A can be produced by employing just labor and capital in the four ways shown below. Assume the prices of labor and capital are $2 and $3 respectively.
I 4 2 Production techniques: II III IV 3 2 5 3 5 1

Labor Capital

Type: T Topic: 5 E: 67 MI: 67 MA: 67 55. Refer to the above information. Which technique is economically most efficient in producing A? A) I B) II C) III D) IV Answer: D

Type: T Topic: 5 E: 67 MI: 67 MA: 67 56. Refer to the above information. If the price of product A is $.50, the firm will realize: A) an economic profit of $4. C) an economic profit of $6. B) an economic profit of $2. D) a loss of $3. Answer: B

Type: A Topic: 5 E: 67 MI: 67 MA: 67 57. In a competitive market economy firms will select the least-cost production technique because: A) such choices will result in the full employment of available resources. B) to do so will maximize the firms' profits. C) this will prevent new firms from entering the industry. D) "dollar voting" by consumers mandates such a choice. Answer: B

McConnell/Brue: Economics, 16/e

Page 135

Chapter 4: The Market System

Consumer sovereignty; invisible hand

Type: A Topic: 6 E: 65 MI: 65 MA: 65 58. The market system's answer to the fundamental question "What will be produced?" is essentially: A) "Goods and services that are profitable." B) "Low cost goods and services." C) "Goods and service that can be produced using round-about production." D) "Goods and services that possess lasting value." Answer: A

Type: A Topic: 6 E: 67 MI: 67 MA: 67 59. The market system's answer to the fundamental question "How will the goods and services be produced?" is essentially: A) "With as much machinery as possible." C) "By exploiting labor." B) "Using the latest technology." D) "At least-cost production." Answer: D

Type: A Topic: 6 E: 67-68 MI: 67-68 MA: 67-68 60. The market system's answer to the fundamental question "Who will get the goods and services?" is essentially: A) "Those willing and able to pay for them." C) "Those who most need them." B) "Those who physically produced them." D) "Those who get utility from them." Answer: A

Type: A Topic: 6 E: 68-69 MI: 68-69 MA: 68-69 61. The market system's answer to the fundamental question "How will the system accommodate change?" is essentially: A) "Through government leadership and direction." B) "Through the guiding function of prices and the incentive function of profits." C) "Through training and retraining programs." D) "Through random trial and error." Answer: B

Type: A Topic: 6 E: 69 MI: 69 MA: 69 62. The advent of DVDs threatens to eventually demolish the market for videocassettes. This is an example of: A) creative destruction. B) derived demand. C) capital accumulation. D) the difference between normal and economic profits. Answer: A

Type: D Topic: 6 E: 66 MI: 66 MA: 66 63. Consumer sovereignty refers to the: A) fact that resource prices are higher than product prices in capitalistic economies. B) idea that the pursuit of self-interest is in the public interest. C) idea that the decisions of producers and resource suppliers with respect to the kinds and amounts of goods produced must be appropriate to consumer demands. D) fact that a Federal agency exists to protect consumers from harmful and defective products. Answer: C

McConnell/Brue: Economics, 16/e

Page 136

Chapter 4: The Market System

Type: D Topic: 6 E: 66 MI: 66 MA: 66 64. The dollar votes of consumers ultimately determine the composition of output and the allocation of resources in a market economy. This statement best describes the concept of: A) derived demand. B) consumer sovereignty. C) the invisible hand. D) market failure. Answer: B

Type: C Topic: 6 E: 68 MI: 68 MA: 68 65. Assume the demand for product Y increases and the market system responds by producing more Y. This illustrates: A) that the concept of derived demand is inapplicable. B) that consumer sovereignty is inoperative in this economy. C) the scarcity function of prices. D) the directing or guiding function of prices. Answer: D

Type: D Topic: 6 E: 68 MI: 68 MA: 68 66. Which of the following best describes the guiding function of competitive prices? A) Profitable industries contract and unprofitable industries expand. B) The market system will always generate economic profits for firms that use the least costly production technology. C) The market system can negotiate reallocations of resources that are appropriate to changes in consumer tastes, technology, and resource supplies. D) When prices are in equilibrium, product shortages or surpluses cannot occur. Answer: C

Type: D Topic: 6 E: 69 MI: 69 MA: 69 67. Which of the following best describes the invisible-hand concept? A) The desires of resource suppliers and producers to further their own self-interest will automatically further the public interest. B) The nonsubstitutability of resources creates a conflict between private and public interests and calls for government intervention. C) The market system is the best system for overcoming the scarce resources-unlimited wants problem. D) Central direction by the government will improve resource allocation in a capitalistic economy. Answer: A

Type: D Topic: 6 E: 69 MI: 69 MA: 69 68. The invisible hand refers to the: A) fact that the U.S. tax system redistributes income from rich to poor. B) notion that, under competition, decisions motivated by self-interest promote the social interest. C) tendency of monopolistic sellers to raise prices above competitive levels. D) fact that government controls the functioning of the market system. Answer: B

Type: A Topic: 6 E: 69 MI: 69 MA: 69 69. The invisible-hand concept suggests that: A) market failures imply the need for a national economic plan. B) big businesses are inherently more efficient than small businesses. C) the competitiveness of a capitalistic market economy invariably diminishes over time. D) assuming competition, private and public interests will coincide. Answer: D

McConnell/Brue: Economics, 16/e

Page 137

Chapter 4: The Market System

Type: A Topic: 6 E: 69 MI: 69 MA: 69 70. The invisible-hand concept suggests that: A) changes in product demands are only randomly reflected in changes in the demands for resources. B) profit maximization is inconsistent with an efficient allocation of resources. C) government action is necessary to correct for market failures. D) when firms maximize their profits, society's output will also be maximized. Answer: D

Type: A Topic: 6 E: 69 MI: 69 MA: 69 71. Two major virtues of the market system are that it: A) allocates resources efficiently and allows economic freedom. B) results in an equitable personal distribution of income and always maintains full employment. C) results in price level stability and a fair personal distribution of income. D) eliminates discrimination and minimizes environmental pollution. Answer: A

Type: A Topic: 6 E: 69 MI: 69 MA: 69 72. The market system: A) produces considerable inefficiency in the use of scarce resources. B) effectively harnesses the incentives of workers and entrepreneurs. C) is inconsistent with freedom of choice in the long run. D) has slowly lost ground to emerging command systems. Answer: B

Consider This Questions

Type: A E: 66 MI: 66 MA: 66 Status: New 73. (Consider This) In 1975 McDonald's introduced its Egg McMuffin breakfast sandwich, which remains popular and profitable today. This longevity illustrates the idea of: A) opportunity cost. B) upsloping supply. C) consumer sovereignty. D) specialization. Answer: C

Type: A E: 66 MI: 66 MA: 66 Status: New 74. (Consider This) In 1996 McDonald's introduced its Arch Deluxe hamburger, which failed to catch on with the public and was subsequently dropped from the menu. This failure illustrates the idea of: A) consumer sovereignty. B) technological change. C) downsloping demand. D) specialization. Answer: A

Last Word Questions Type: A E: 70 MI: 70 MA: 70 75. (Last Word) According to economist Donald Boudreaux, the world's tens of billions of individual resources get arranged productively: A) because government has become highly effective at central planning. B) because private property encourages people to consider the alternative uses of their resources and select those that provide the most rewards. C) because people tend to be creative and orderly. D) through random trial and error. Answer: B

McConnell/Brue: Economics, 16/e

Page 138

Chapter 4: The Market System

Type: A E: 70 MI: 70 MA: 70 76. (Last Word) According to economist Donald Boudreaux: A) private property eliminates the possibility that resource arrangements will be random. B) the market system threatens to do irreparable harm to the world's ecosystem. C) arranging resources under the market system is much like shuffling a deck of cards. D) the market system works wondrously for advanced industrial nations but not for developing nations. Answer: A

True/False Questions

Type: A E: 62 MI: 62 MA: 62 77. Market economies use capital goods because they improve productive efficiency. Answer: True

Type: A E: 63 MI: 63 MA: 63 78. Differences in opportunity costs are the basis for specialized production and trade. Answer: True

Type: D E: 63 MI: 63 MA: 63 79. Money functions as a medium of exchange in permitting the ready comparison of the relative worth of heterogeneous products. Answer: False

Type: A E: 66 MI: 66 MA: 66 80. Consumer sovereignty means that legislation now protects the rights of consumers to dispose of their incomes as they see fit. Answer: False

Type: D E: 68 MI: 68 MA: 68 81. Programs by state governments to keep milk prices higher than market-determined prices to protect family dairy farms from bankruptcy promote the efficient allocation of resources. Answer: False

Type: A E: 63 MI: 63 MA: 63 82. Specialization may expand total output even though the individuals involved may have identical abilities. Answer: True

Type: A E: 66 MI: 66 MA: 66 83. The wants of consumers are expressed on the demand side of the product market. Answer: True

Type: A E: 65 MI: 65 MA: 65 84. Costs can be defined as total payments made to workers, land owners, and capital suppliers less normal profits. Answer: False

McConnell/Brue: Economics, 16/e

Page 139

Chapter 4: The Market System

Type: D E: 65 MI: 65 MA: 65 85. If firms in any industry fail to earn normal profits, firms will immediately or eventually leave the industry. Answer: True

Type: A E: 68 MI: 68 MA: 68 86. The guiding function of prices indicates that, at equilibrium prices, neither product surpluses nor shortages can occur. Answer: False

Type: A E: 69 MI: 69 MA: 69 87. The invisible hand refers to the many indirect controls that the Federal government imposes in a the market system. Answer: False

McConnell/Brue: Economics, 16/e

Page 140

CHAPTER 5

The U.S. Economy: Private and Public Sectors

Topic 1. Personal and functional distribution of income 2. Legal forms of business 3. Economic functions of government 4. Spillover costs and benefits 5. Public goods 6. Stabilization 7. Government in circular flow 8. Purchases, transfers, and government size 9. Federal finance and progressive taxes 10. State and local finance Consider This Last Word True-False

Question numbers 1-18 19-54 55-59 60-86 87-115 116-122 123-133 134-142 143-170 171-174 175-176 177-180 1181-204

____________________________________________________________

_______________________________________

____________________________________________________________

_______________________________________

Multiple Choice Questions Personal and functional distribution of income

Type: D Topic: 1 E: 73 MI: 73 MA: 73 1. The personal distribution of income refers to the: A) division of income between personal taxes, consumption expenditures, and saving. B) division of income on the basis of industry sources, for example, agriculture, transportation, and mining. C) distribution of income to basic resource classes, that is, wages, rents, interest, and profits. D) way income is distributed among specific households or spending units. Answer: D

Type: D Topic: 1 E: 73 MI: 73 MA: 73 2. The functional distribution of income refers to the: A) division of income between personal taxes, consumption expenditures, and saving. B) division of income on the basis of industry sources, for example, agriculture, transportation, mining, etc. C) distribution of income to basic resource classes, that is, wages, rents, interest, and profits. D) way income is distributed among specific households or spending units. Answer: C

Chapter 5: The U.S. Economy: Private and Public Sectors

Type: D Topic: 1 E: 73 MI: 73 MA: 73 3. Income data that show the percentage of total income received by each fifth of all households describe the: A) functional distribution of income. C) personal distribution of income. B) horizontal distribution of income. D) vertical distribution of income. Answer: C

Type: D Topic: 1 E: 73 MI: 73 MA: 73 4. Income data that show how total income is distributed as wages, rents, interest, and profits describe the: A) functional distribution of income. C) personal distribution of income. B) horizontal distribution of income. D) vertical distribution of income. Answer: A

Type: F Topic: 1 E: 74 MI: 74 MA: 74 5. The personal distribution of income in the United States is such that the richest fifth receives about _____ percent of personal income. A) 30 B) 40 C) 50 D) 60 Answer: C

Type: F Topic: 1 E: 74 MI: 74 MA: 74 6. The largest functional share of the national income consists of: A) wages and salaries. B) interest and rental income. C) proprietors' income, that is, the income of unincorporated businesses. D) corporate profits. Answer: A

Use the following to answer questions 7-9:

Type: A Topic: 1 E: 74 MI: 74 MA: 74 7. In the above diagrams for a hypothetical economy, Figure 1 shows the: A) personal distribution of income. C) microeconomic distribution of income. B) functional distribution of income. D) international distribution of income. Answer: B

McConnell/Brue: Economics, 16/e

Page 142

Chapter 5: The U.S. Economy: Private and Public Sectors

Type: A Topic: 1 E: 74 MI: 74 MA: 74 8. In the above diagrams for a hypothetical economy, Figure 2 shows the: A) personal distribution of income. C) microeconomic distribution of income. B) functional distribution of income. D) rates of poverty in the United States. Answer: A

Type: A Topic: 1 E: 74 MI: 74 MA: 74 9. In the above diagrams for a hypothetical economy, from Figure 2 we can conclude that: A) income is quite equally distributed. B) wages are the main source of income. C) the distribution of income has become more unequal over time. D) the top one-fifth of all income receivers get about eight times as much income as the lowest one-fifth. Answer: D

Type: D Topic: 1 E: 73 MI: 73 MA: 73 10. The distribution of income among the owners of land, labor, capital, and entrepreneurial ability is known as: A) income differentials. C) the functional distribution of income. B) the personal distribution of income. D) the concentration ratio. Answer: C

Type: D Topic: 1 E: 73 MI: 73 MA: 73 11. The distribution of income among individual households is known as: A) income differentials. C) the functional distribution of income. B) the personal distribution of income. D) the concentration ratio. Answer: B

Type: F Topic: 1 E: 75 MI: 75 MA: 75 12. Listed in descending order of relative size, households divide their total incomes among: A) consumption expenditures, saving, and taxes. C) consumption expenditures, taxes, and saving. B) saving, consumption expenditures, and taxes. D) taxes, consumption expenditures, and saving. Answer: C

Type: C Topic: 1 E: 74 MI: 74 MA: 74 13. Other things equal, a rapid rise in profits relative to other types of income would affect: A) the functional distribution of income only. B) the personal distribution of income only. C) both the functional and personal distributions of income. D) the relative size of the public sector. Answer: C

Type: F Topic: 1 E: 74 MI: 74 MA: 74 14. Since 1940 personal taxes have: A) risen absolutely, but declined as a percentage of personal income. B) risen both absolutely and as a percentage of personal income. C) fallen absolutely, but risen as a percentage of personal income. D) fallen both absolutely and as a percentage of personal income. Answer: B

McConnell/Brue: Economics, 16/e

Page 143

Chapter 5: The U.S. Economy: Private and Public Sectors

Type: F Topic: 1 E: 75 MI: 75 MA: 75 15. Listed in descending order of relative size, total consumption spending is comprised of: A) nondurable goods, durable goods, and services. C) services, durable goods, and nondurable goods. B) services, nondurable goods, and durable goods. D) durable goods, nondurable goods, and services. Answer: B

Type: A Topic: 1 E: 75 MI: 75 MA: 75 16. If the aggregate income of households is $300 billion, consumption is $210 billion, and personal taxes are $60 billion, then personal saving: A) is $70 billion. B) is $30 billion. C) is $40 billion. D) cannot be determined from the information given. Answer: B

Type: D Topic: 1 E: 74 MI: 74 MA: 74 17. Economists define saving as: A) that part of after-tax income which is not consumed. B) total income less taxes. C) bank accounts. D) purchases of stocks and bonds. Answer: A

Type: F Topic: 1 E: 75 MI: 75 MA: 75 18. Households in the aggregate use the largest share of their total income to: A) pay taxes. B) consume. C) save. D) buy capital goods. Answer: B

Legal forms of business

Type: D Topic: 2 E: 75 MI: 75 MA: 75 19. In economics, a physical establishment such as a factory, farm, mine, store, or warehouse that performs one or more functions in fabricating and distributing goods is called a(n): A) industry. B) plant. C) conglomerate. D) shop. Answer: B

Type: D Topic: 2 E: 76 MI: 76 MA: 76 20. In economics, a group of firms that produce identical or similar products is called a(n): A) industry. B) plant. C) conglomerate. D) firm. Answer: A

Type: D Topic: 2 E: 75 MI: 75 MA: 75 21. In economics, a business establishment that owns one or more plants is called a(n): A) industry. B) shop. C) conglomerate. D) firm. Answer: D

McConnell/Brue: Economics, 16/e

Page 144

Chapter 5: The U.S. Economy: Private and Public Sectors

Type: D Topic: 2 E: 76 MI: 76 MA: 76 22. An industry is best defined as a group of firms that: A) compete for labor. B) produce identical or similar products. Answer: B

C) use identical production techniques. D) are located in the same city or geographic area.

Type: D Topic: 2 E: 76 MI: 76 MA: 76 23. A firm that makes computer chips at several locations across the country best illustrates a: A) vertically integrated firm. B) multinational corporation. C) multiplant firm. D) conglomerate. Answer: C

Type: D Topic: 2 E: 76 MI: 76 MA: 76 24. A firm that produces a single product but owns plants in many different stages of the production processfor example, a steel producer that owns iron ore mines and rolling mills-best illustrates a: A) vertically integrated firm. B) multinational corporation. C) virtual corporation. D) conglomerate. Answer: A

Type: D Topic: 2 E: 76 MI: 76 MA: 76 25. A firm comprised of plants or units operating in different industries, say, beer and theme parks, best illustrates a: A) vertically integrated firm. B) multinational corporation. C) multiplant firm. D) conglomerate. Answer: D

Type: D Topic: 2 E: 76 MI: 76 MA: 76 26. A group of plants that is owned and operated by a single firm and that consists of oil fields, refineries, and gasoline stations best illustrates a: A) trust. B) holding company. C) vertically integrated firm. D) multinational corporation. Answer: C

Type: F Topic: 2 E: 76 MI: 76 MA: 76 27. The division of U.S. businesses into the categories of proprietorships, partnerships, and corporations is based on: A) generally accepted accounting principles. B) legal considerations. C) the judgment of the American Economic Association. D) an executive order of the President. Answer: B

Type: F Topic: 2 E: 76 MI: 76 MA: 76 28. Which of the following is numerically the dominant type of business in the United States? A) corporations B) proprietorships C) partnerships D) cooperatives Answer: B

Type: F Topic: 2 E: 76 MI: 76 MA: 76 29. Which form of business enterprise accounts for the largest proportion of total output? A) corporations B) proprietorships C) partnerships D) cooperatives Answer: A

McConnell/Brue: Economics, 16/e

Page 145

Chapter 5: The U.S. Economy: Private and Public Sectors

Type: D Topic: 2 E: 76 MI: 76 MA: 76 30. The three basic legal forms of business enterprise are: A) monopolists, competitors, and enterprises. B) proprietorships, partnerships, and corporations. C) vertical, horizontal, and conglomerate corporations. D) conglomerates, multinationals, and partnerships. Answer: B

Type: F Topic: 2 E: 76 MI: 76 MA: 76 31. Listed in descending order of total numbers, the business population is comprised of: A) sole proprietorships, corporations, and partnerships. B) sole proprietorships, partnerships, and corporations. C) partnerships, corporations, and sole proprietorships. D) corporations, partnerships, and sole proprietorships. Answer: A

Type: D Topic: 2 E: 77 MI: 77 MA: 77 32. The advantages of the corporate form of business include: A) the ability to raise financial capital by selling stocks and bonds. B) the fact that owners are subject to unlimited liability. C) the elimination of the principal-agent problem. D) single taxation of corporate earnings. Answer: A

Type: F Topic: 2 E: 77 MI: 77 MA: 77 33. If a corporation goes bankrupt, its stockholders will lose: A) only the value of their stock. B) the value of their stock plus any other business assets they may own. C) the value of their stock plus any other personal assets they may own. D) the value of their stock plus any other business and personal assets they may own. Answer: A

Type: F Topic: 2 E: 77 MI: 77 MA: 77 34. A major disadvantage of corporations is that: A) they cannot issue bonds. B) dividends are taxed both as corporate income and as income to stockholders. C) stockholders are subject to unlimited liability. D) their charters last for only 20 years, at which time the corporation must reorganize. Answer: B

Type: A Topic: 2 E: 77 MI: 77 MA: 77 35. The most crucial determinant of the legal form of an enterprise will usually be the: A) ability of the firm to sell bonds to the public. B) amount of financial capital required by the line of production. C) amount of unemployment in the particular industry. D) state in which it is located. Answer: B

McConnell/Brue: Economics, 16/e

Page 146

Chapter 5: The U.S. Economy: Private and Public Sectors

Type: A Topic: 2 E: 77 MI: 77 MA: 77 36. The corporate form of business enterprise has promoted the development of large-scale business operations because: A) the corporate form of enterprise is in a highly advantageous position to protect itself from the effects of long-run inflation. B) corporations have been able to marshal large amounts of financial capital through the sale of stocks and bonds. C) corporations have been more successful than unincorporated businesses in keeping labor unions out of their plants. D) this legal form of enterprise has been subject to fewer government controls than have proprietorships and partnerships. Answer: B

Type: F Topic: 2 E: 77 MI: 77 MA: 77 37. An owner's liability for the debts of a business is: A) limited in a corporation to the assets of preferred stockholders. B) limited in a corporation to the assets of bondholders. C) limited to the owner's investment in a single proprietorship. D) unlimited in a partnership. Answer: D

Type: A Topic: 2 E: 77 MI: 77 MA: 77 38. Suppose you own $50,000 of personal property, $5,000 of stock in General Statics Corporation, a $10,000 savings account, and $20,000 of government bonds. If General Statics goes bankrupt, the most you could lose is: A) $50,000. B) $5,000. C) $35,000. D) $85,000. Answer: B

Type: F Topic: 2 E: 77 MI: 77 MA: 77 39. Limited liability applies to: A) partnerships. B) proprietorships. C) all corporations. D) financial corporations but not to manufacturing corporations. Answer: C

Type: F Topic: 2 E: 77 MI: 77 MA: 77 40. The owners of a firm face unlimited liability for the firm's debts in: A) a corporation. C) a proprietorship, but not in a partnership. B) a partnership, but not in a proprietorship. D) both a proprietorship and a partnership. Answer: D

Type: A Topic: 2 E: 78 MI: 78 MA: 78 41. The separation of ownership and control in a corporation means that: A) hired managers play a larger role in determining company policy than do a corporation's legal owners. B) the ownership of corporations is becoming increasingly concentrated in the hands of a few common stockholders. C) a firm's board of directors has no power over hired managers. D) stockholders have lost their voting privileges. Answer: A

McConnell/Brue: Economics, 16/e

Page 147

Chapter 5: The U.S. Economy: Private and Public Sectors

Type: D Topic: 2 E: 77 MI: 77 MA: 77 42. Limited liability means that: A) creditors have no legal claim on the personal assets of a proprietor. B) corporations cannot be sued. C) creditors have no legal claim on the personal assets of a corporate stockholder. D) corporations have a legal life independent of their owners and managers. Answer: C

Type: D Topic: 2 E: 77-78 MI: 77-78 MA: 77-78 43. Stocks are: A) promises to repay a loan. C) issued by sole proprietorships. B) also known as bonds. D) shares of ownership of a corporation. Answer: D

Type: D Topic: 2 E: 77 MI: 7 MA: 77 44. Corporate bonds are: A) promises by a corporation to repay a loan. B) also known as stocks. Answer: A

C) illegal in the United States. D) shares of ownership of a corporation.

Type: D Topic: 2 E: 78 MI: 78 MA: 78 45. As it relates to corporations, the principal-agent problem is that: A) the goals of the corporate managers (the principals) may not match the goals of the corporate owners (the agents). B) the goals of the corporate managers (the agents) may not match the goals of the corporate owners (the principals). C) the Federal government (the agent) taxes both corporate profits and the dividends paid to stockholders (the principals). D) It is costly for the corporate owners (the principals) to obtain a corporate charter from government (the agent). Answer: B

Type: A Topic: 2 E: 78 MI: 78 MA: 78 46. In corporations, owners are __________________ and managers are ________________. A) agents; principals. B) stockholders; bondholders. C) agents; employees. D) principals; agents. Answer: D

Type: A Topic: 2 E: 78 MI: 78 MA: 78 47. As it relates to owners and managers, the principal-agent problem results from the: A) separation of corporate ownership and control. B) double taxation of corporate profit. C) limited liability of corporate owners. D) differing interests of corporate stockholders and bondholders. Answer: A

McConnell/Brue: Economics, 16/e

Page 148

Chapter 5: The U.S. Economy: Private and Public Sectors

Type: A Topic: 2 E: 78 MI: 78 MA: 78 48. An S corporation is a: A) form of business organization designed to protect firms from lawsuits by disgruntled stockholders. B) super-size corporation characterized by 50,000 or more employees. C) form of business organization (reserved for relatively small firms) that avoids double taxation of corporate profit and provides limited liability for the owners. D) firm that accepts savings deposits. Answer: C

Type: A Topic: 2 E: 77-78 MI: 77-78 MA: 77-78 49. Double taxation of a firm's income occurs in a: A) corporation. B) limited-liability company. C) partnership. D) sole proprietorship. Answer: A

Type: D Topic: 2 E: 77-78 MI: 77-78 MA: 77-78 50. Double taxation means that: A) payroll taxes that finance social security are levied on both workers and employers. B) taxes levied on wholesale products also apply at the retail level. C) the part of corporate earnings paid as dividends is taxed as corporate profits and also as income to stockholders. D) corporate tax rates are twice as high as those on the incomes of incorporated businesses. Answer: C

Type: F Topic: 2 E: 76 MI: 76 MA: 76 51. Most output in the United States is produced by: A) cooperatives. B) partnerships. C) sole proprietorships. Answer: D

D) corporations.

Type: F Topic: 2 E: 76 MI: 76 MA: 76 52. Corporations comprise about _____ percent of all businesses, but produce nearly _____ percent of domestic output. A) 40; 60 B) 5; 70 C) 20; 90 D) 80; 20 Answer: C

Type: D Topic: 2 E: 78 MI: 78 MA: 78 53. Limited liability companies: A) limit the liability of owners whose income is subject to the corporate income tax. B) are the same as partnerships. C) have a limited life and their net incomes are taxed as personal income. D) are the same as corporations. Answer: C

Type: A Topic: 2 E: 78 MI: 78 MA: 78 54. The XYZ Company is subject to limited liability; its income is treated as personal income for tax purposes; it has a limited life of 30 years. XYZ is a: A) corporation. B) partnership. C) limited liability company. D) cooperative. Answer: C

McConnell/Brue: Economics, 16/e

Page 149

Chapter 5: The U.S. Economy: Private and Public Sectors

Economic functions of government

Type: A Topic: 3 E: 81 MI: 81 MA: 81 55. Which of the following is a shortcoming of the market system? A) It leads to firms that are too large to achieve productive efficiency. B) Certain goods will not be produced because there is no way of excluding nonpaying ("free-rider") individuals from the associated benefits. C) The resulting distribution of personal incomes might be too equal to maintain incentives. D) It is controlled by a handful of multinational corporations. Answer: B

Type: D Topic: 3 E: 79 MI: 79 MA: 79 56. The economic policies and programs of government affect all of the following except: A) the distribution of income. B) the allocation of resources. C) the composition of output. D) the underlying motives of consumers, workers, and firms. Answer: D

Type: A Topic: 3 E: 79 MI: 79 MA: 79 57. The Pure Food and Drug Act is an illustration of: A) governmental provision of public goods. B) the redistributional function of government. C) governmental provision of a suitable legal framework for the market system. D) governmental action designed to enhance competition. Answer: C

Type: A Topic: 3 E: 79 MI: 79 MA: 79 58. The U.S. Food Stamp program, which provides coupons that allow low-income individuals to buy food, is an illustration of: A) public provision of a suitable legal framework for the market system. B) the redistributional function of government. C) a government action designed to enhance competition. D) the stabilization function of government. Answer: B

Type: A Topic: 3 E: 80 MI: 80 MA: 80 59. Government may lessen income inequality by: A) providing transfer payments to the poor. B) directly modifying market prices as, for example, by establishing a legal minimum wage. C) using the tax system to tax the wealthy relatively more heavily than the poor. D) doing all of the above. Answer: D

McConnell/Brue: Economics, 16/e

Page 150

Chapter 5: The U.S. Economy: Private and Public Sectors

Spillover costs and benefits

Type: A Topic: 4 E: 81 MI: 81 MA: 81 60. In a competitive market: A) demand will not always reflect all spillover benefits. B) demand will always reflect all spillover benefits. C) supply will always reflect all spillover costs. D) supply will always reflect all spillover benefits. Answer: A

Type: A Topic: 4 E: 81 MI: 81 MA: 81 61. In a competitive market: A) spillover benefits will always exceed spillover costs. B) resources will be misallocated if government does not properly adjust demand and supply for large spillover costs and benefits. C) resources will be allocated efficiently only if spillover benefits equal spillover costs. D) an efficient allocation of resources is realized even where there are large spillover costs and benefits. Answer: B

Type: A Topic: 4 E: 80 MI: 80 MA: 80 62. Spillover costs arise: A) when firms pay more than the opportunity cost of resources. B) when the demand curve for a product is located too far to the left. C) when firms "use" resources without being compelled to pay for their full costs. D) only in capitalistic societies. Answer: C

Type: D Topic: 4 E: 81 MI: 81 MA: 81 63. Spillover benefits refer to: A) benefits that accrue to parties other than the producer and buyer of a good. B) the benefits that resource suppliers obtain from the production and sale of a good. C) the benefit that a consumer receives from buying a good. D) the combined benefits that buyer and seller receive from a voluntary market transaction. Answer: A

Type: A Topic: 4 E: 80 MI: 80 MA: 80 64. A pure market economy overallocates resources to the production of goods that: A) involve spillover costs (external costs). C) are public goods. B) involve spillover benefits (external benefits). D) are inexpensive to produce. Answer: A

Type: D Topic: 4 E: 80 MI: 80 MA: 80 Status: New 65. A spillover cost or spillover benefit is also known as a(n): A) marginal benefit. B) principal-agent problem. C) transfer payment. D) externality. Answer: D

McConnell/Brue: Economics, 16/e

Page 151

Chapter 5: The U.S. Economy: Private and Public Sectors

Type: A Topic: 4 E: 80 MI: 80 MA: 80 66. When spillovers cause substantial positive benefits for third parties, a competitive market: A) underallocates resources to the production of the good. B) overallocates resources to the production of the good. C) is allocatively efficient. D) compensates people for the value of the benefits that these third parties receive. Answer: A

Type: A Topic: 4 E: 80 MI: 80 MA: 80 67. If a market is competitive but externalities are present, the resulting equilibrium output: A) will also be the most efficient output. B) will always be less than the most efficient output. C) will always be greater than the most efficient output. D) may be either larger or smaller than the most efficient output. Answer: D

Type: D Topic: 4 E: 80-81 MI: 80-81 MA: 80-81 68. Spillovers or externalities: A) relate to costs only. C) relate to both costs and benefits. B) relate to benefits only. D) have been legislated out of existence. Answer: C

Type: D Topic: 4 E: 80 MI: 80 MA: 80 69. When the production or consumption of a good involves an externality: A) resources are necessarily overallocated to the product. B) resources are necessarily underallocated to the product. C) someone not involved in buying or selling the good is affected. D) the market will efficiently allocate resources to its production. Answer: C

Type: A Topic: 4 E: 81 MI: 81 MA: 81 70. If a good's production creates substantial spillover benefits and no spillover costs, then too: A) much of the good will be produced unless firms are subsidized. B) much of the good will be produced unless firms are taxed. C) little of the good will be produced unless firms are subsidized. D) little of the good will be produced unless firms are taxed. Answer: C

Type: A Topic: 4 E: 81 MI: 81 MA: 81 71. If the production of a good or service creates sizable spillover benefits, government might correct for the: A) underallocation of resources to its production by imposing an excise tax. B) overallocation of resources to its production by imposing an excise tax. C) underallocation of resources to its production by granting a subsidy. D) overallocation of resources to its production by granting a subsidy. Answer: C

McConnell/Brue: Economics, 16/e

Page 152

Chapter 5: The U.S. Economy: Private and Public Sectors

Type: A Topic: 4 E: 82 MI: 82 MA: 82 72. For which of the following goods or services would a government subsidy be most likely to improve the allocation of resources? A) wheat B) newspaper publishing C) cancer research D) toys Answer: C

Type: A Topic: 4 E: 81 MI: 81 MA: 81 73. If there are important spillover benefits associated with the consumption of a product: A) government should enact legislation to prohibit the production of the commodity. B) special excise taxes should be levied on producers of the product. C) the market demand curve understates the relative importance of the product and resources are therefore underallocated to its production. D) the market supply curve for the product lies too far to the right to provide an efficient allocation of resources. Answer: C

Type: A Topic: 4 E: 80 MI: 80 MA: 80 74. Suppose a product creates substantial spillover costs. If government adopts a policy that forces producers to pay these costs, the: A) output of the product will decrease. B) initial misallocation of resources will be intensified. C) output of the product will increase. D) price of the product will decrease. Answer: A

Type: A Topic: 4 E: 80 MI: 80 MA: 80 75. If spillover costs are not internalized, the: A) equilibrium output will exceed the most efficient output. B) most efficient output will exceed the equilibrium output. C) product must be a public good. D) distribution of income will necessarily become more equal. Answer: A

Type: A Topic: 4 E: 81 MI: 81 MA: 81 76. A productive activity that creates substantial spillover benefits should be: A) encouraged by subsidization. C) exempt from governmental intervention. B) discouraged by special taxes or legislation. D) prohibited. Answer: A

Type: A Topic: 4 E: 81 MI: 81 MA: 81 77. Suppose the production of a certain good creates substantial spillover benefits. If government adopts a policy that adjusts demand to take these benefits into account, then: A) firms in this industry will go out of business. C) the output of the product will decrease. B) the output of the product will increase. D) the price of the product will decrease. Answer: B

McConnell/Brue: Economics, 16/e

Page 153

Chapter 5: The U.S. Economy: Private and Public Sectors

Type: A Topic: 4 E: 80 MI: 80 MA: 80 78. The Federal government requires automobile manufacturers to install pollution control equipment. This is an illustration of the: A) intrusion problem. C) internalization of external benefits. B) production of public goods. D) internalization of external costs. Answer: D

Type: A Topic: 4 E: 80 MI: 80 MA: 80 79. Pollution: A) should be corrected by the subsidization of offending firms. B) is not an economic problem because it is external to the market system. C) is an example of private production costs. D) is an example of a spillover or external cost. Answer: D

Type: A Topic: 4 E: 82 MI: 82 MA: 82 80. Which of the following is most likely to be accompanied by external or spillover benefits? A) the construction of a nuclear power plant C) eating dinner at an expensive French restaurant B) studying in the library D) being immunized for measles Answer: D

Type: D Topic: 4 E: 80 MI: 80 MA: 80 81. A market externality refers to: A) economic costs and benefits of market activities that go to those who are not directly involved in the market transaction. B) the impact of legal and institutional forces on market behavior. C) any unanticipated change in market price or output. D) any noneconomic force, for example, political disruption of the flow of Middle East oil, which has market effects. Answer: A

Type: A Topic: 4 E: 80 MI: 80 MA: 80 82. Spillovers or externalities weaken the efficiency of the market system because they: A) are a major source of inflation. B) mean that certain essential goods and services do not get produced at all. C) are a major source of employment. D) cause certain goods to be overproduced or underproduced. Answer: D

Type: A Topic: 4 E: 81 MI: 81 MA: 81 83. A subsidy: A) should be provided when there are external costs. B) should be provided when there are external benefits. C) should be provided only when a public good is being produced. D) is appropriate when firms are guilty of pollution. Answer: B

McConnell/Brue: Economics, 16/e

Page 154

Chapter 5: The U.S. Economy: Private and Public Sectors

Type: A Topic: 4 E: 80 MI: 80 MA: 80 84. If an industry causes spillovers so that resources are overallocated to the industry: A) a principal-agent problem must be present. C) spillover costs are involved. B) the industry is producing a public good. D) spillover benefits are involved. Answer: C

Type: A Topic: 4 E: 80 MI: 80 MA: 80 85. If the production and consumption of a good cause both spillover benefits and spillover costs: A) the equilibrium output will be the optimal allocation of resources to its production. B) resources will necessarily be underallocated to its production. C) resources will necessarily be overallocated to its production. D) resources may be either underallocated or overallocated to its production. Answer: D

Type: C Topic: 4 E: 81 MI: 81 MA: 81 86. Education levels and crime rates are inversely related. This suggests that education: A) is not subject to the exclusion principle. C) creates a free-rider problem. B) entails external benefits. D) should be produced in the private sector. Answer: B

Public goods

Type: D Topic: 5 E: 81 MI: 81 MA: 81 Status: New 87. The main characteristics of a public good are: A) nonrivalry and nonexcludability. C) nonrivalry and large spillover costs. B) nonexcludability and rising costs of production. D) production at constant cost and rising demand. Answer: A

Type: D Topic: 5 E: 81 MI: 81 MA: 81 Status: New 88. As it relates to a public good, nonrivalry means that: A) the public sector is able to provide the good profitably. B) there is no need or demand for the good. C) either the public sector or the public sector can produce the good, but not both. D) one person's benefit from the good does not reduce the benefit available to others. Answer: D

Type: D Topic: 5 E: 81 MI: 81 MA: 81 Status: New 89. As it relates to a public good, nonexcludability means that: A) free riders cannot be barred from receiving the benefits. B) there is no need or demand for the good. C) either the public sector or the public sector can produce the good, but not both. D) one person's benefit from the good does not reduce the benefit available to others. Answer: A

Type: D Topic: 5 E: 81 MI: 81 MA: 81 Status: New 90. Rivalry and excludability are the main characteristics of: A) capital goods. B) private goods. C) public goods. D) consumption goods. Answer: B

McConnell/Brue: Economics, 16/e

Page 155

Chapter 5: The U.S. Economy: Private and Public Sectors

Type: D Topic: 5 E: 81 MI: 81 MA: 81 Status: New 91. Nonrivalry and nonexcludability are the main characteristics of: A) capital goods. B) private goods. C) public goods. D) consumption goods. Answer: C

Type: D Topic: 5 E: 81 MI: 81 MA: 81 Status: New 92. Unlike a private good, a public good: A) produces no spillover benefits or spillover costs. B) has no opportunity costs. C) has benefits that are available to all, regardless of payment. D) is characterized by rivalry and excludability. Answer: C

Type: A Topic: 5 E: 81 MI: 81 MA: 81 Status: New 93. Which of the following is an example of a public good? A) a fireworks display. B) a hotdog C) a barbeque grill D) a personal computer Answer: A

Type: A Topic: 5 E: 81 MI: 81 MA: 81 Status: New 94. An example of a public good is: A) a movie theater. B) a freight train. C) the war on terrorism. D) Disneyland. Answer: C

Type: A Topic: 5 E: 81 MI: 81 MA: 81 Status: New 95. Which of the following is a public good? A) chewing gum B) bread C) a professional baseball game D) street lights in a city Answer: D

Type: A Topic: 5 E: 81 MI: 81 MA: 81 Status: New 96. A public good: A) can be produced profitably by private firms. B) is available to all and cannot be denied to anyone. C) is characterized by rivalry and excludability. D) produces no spillover costs or spillover benefits. Answer: B

Type: D Topic: 5 E: 81 MI: 81 MA: 81 Status: New 97. The market system does not produce public goods because: A) there is no need or demand for such goods. B) private firms cannot stop consumers who are unwilling to pay for such goods from benefiting from them. C) public enterprises can produce such goods at lower cost than can private enterprises. D) their production seriously distorts the distribution of income. Answer: B

McConnell/Brue: Economics, 16/e

Page 156

Chapter 5: The U.S. Economy: Private and Public Sectors

Type: D Topic: 5 E: 81 MI: 81 MA: 81 Status: New 98. Nonexcludability is the idea that: A) government actions cannot remedy market failure. B) the presence of external costs and benefits produces a misallocation of resources. C) individuals cannot receive benefits from a good without paying for it. D) individuals who are unable or unwilling to pay for a good cannot be excluded from the benefits provided by that product. Answer: D

Type: D Topic: 5 E: 81 MI: 81 MA: 81 Status: New 99. Excludability: A) is a characteristics of private goods, but not of public goods. B) applies only where spillover benefits exceed spillover costs. C) suggest there are certain economic functions from which government must be excluded. D) is a fundamental characteristic of public goods. Answer: A

Use the following to answer questions 100-101:
Rivalry Excludability Nonexcludability A C Nonrivalry B D

Type: A Topic: 5 E: 81 MI: 81 MA: 81 Status: New 100. Refer to the above matrix. A private good is characterized by combination: A) A. B) B. C) C. D) D. Answer: A

Type: A Topic: 5 E: 81 MI: 81 MA: 81 Status: New 101. Refer to the above matrix. A public good is characterized by combination: A) A. B) B. C) C. D) D. Answer: D

Type: A Topic: 5 E: 81 MI: 81 MA: 81 Status: New 102. Government rather than the private sector must provide economically desirable public goods because: A) private production of those goods would entail unacceptably high external costs. B) the availability of such goods yields no benefits to individuals. C) the benefits yielded by such goods are available to everyone and cannot be withheld from those who refuse to pay for them. D) their provision is necessary to achieve full employment and price-level stability. Answer: C

McConnell/Brue: Economics, 16/e

Page 157

Chapter 5: The U.S. Economy: Private and Public Sectors

Type: A Topic: 5 E: 81 MI: 81 MA: 81 Status: New 103. The free-rider problem is associated with: A) all market goods. B) the structure of airline fares. C) producing goods that have characteristics of public goods. D) the overallocation of the resources to transportation facilities. Answer: C

Type: A Topic: 5 E: 81 MI: 81 MA: 81 Status: New 104. The free-rider problem is that: A) free public transportation is overcrowded. B) people will not voluntarily pay for something that they can obtain without paying. C) government supplies goods at no charge to people who can afford to pay for them. D) public goods often create large spillover costs. Answer: B

Type: A Topic: 5 E: 81 MI: 81 MA: 81 Status: New 105. Government rather than private firms must provide economically desirable public goods because: A) high marginal costs preclude their production in the private sector. B) public goods have characteristics that make it difficult or impossible for private firms to produce them profitably. C) public goods have marginal costs that exceed marginal benefits. D) the law of increasing opportunity costs applies only to private goods. Answer: B

Type: D Topic: 5 E: 81, 82 MI: 81, 82 MA: 81, 82 Status: New 106. A quasi-public good is: A) a public good that is produced profitably by private firms, without government subsidy. B) one characterized by nonrivalry and nonexcludability. C) one characterized by rivalry but not excludability. D) a good for which exclusion could take place but that has such large spillover benefits that government provides it to prevent an underallocation of resources. Answer: D

Type: A Topic: 5 E: 82 MI: 82 MA: 82 Status: New 107. An example of a quasi-public good is: A) public higher education. B) an automobile. C) a DVD player. D) national defense. Answer: A

Type: A Topic: 5 E: 81, 82 MI: 81, 82 MA: 81, 82 Status: New 108. Which list provides, in order, examples of a private good, a public good, and a quasi-public good? A) national defense, a national park, an automobile B) an automobile, national defense, a national park C) a national park, an automobile, national defense D) national defense, an automobile, a national park. Answer: B

McConnell/Brue: Economics, 16/e

Page 158

Chapter 5: The U.S. Economy: Private and Public Sectors

Type: A Topic: 5 E: 81, 82 MI: 81, 82 MA: 81, 82 Status: New 109. Which list provides, in order, examples of a private good, a public good, and a quasi-public good? A) a weather warning system, a DVD player, higher education. B) higher education, a DVD player, a weather warning system. C) a weather warning system, higher education, a DVD player. D) a DVD player, a weather warning system, higher education. Answer: D

Type: A Topic: 5 E: 81 MI: 81 MA: 81 110. The public decision to build the interstate highway system is an example of: A) the allocative function of government. B) the redistributional function of government. C) public provision of a suitable legal framework for the market system. D) the stabilization function of government. Answer: A

Type: A Topic: 5 E: 82 MI: 82 MA: 82 111. In a full-employment economy government can reallocate resources from private to public employment by: A) reducing taxes and increasing government expenditures. B) reducing taxes and reducing government expenditures. C) increasing taxes and reducing government expenditures. D) increasing taxes and increasing government expenditures. Answer: D

Use the following to answer questions 112-115:

Type: G Topic: 5 E: 82 MI: 82 MA: 82 112. Refer to the above diagram. The movement from point b to point c suggests that more: A) private goods are being produced at the expense of fewer public goods. B) public goods are being produced at the expense of fewer private goods. C) public goods are being produced by employing currently idle resources. D) private goods are being produced by employing currently idle resources. Answer: B

McConnell/Brue: Economics, 16/e

Page 159

Chapter 5: The U.S. Economy: Private and Public Sectors

Type: G Topic: 5 E: 82 MI: 82 MA: 82 113. Refer to the above diagram. The movement from point a to point c suggests that more: A) private goods are being produced by employing currently idle resources. B) public goods are being produced by employing currently idle resources. C) public goods are being produced at the expense of fewer private goods. D) private goods are being produced at the expense of fewer public goods. Answer: B

Type: G Topic: 5 E: 82 MI: 82 MA: 82 114. Refer to the above diagram. The movement from point b to point c can be achieved by: A) increasing taxes and increasing government spending. B) increasing taxes and decreasing government spending. C) decreasing taxes and increasing government spending. D) decreasing taxes and decreasing government spending. Answer: A

Type: G Topic: 5 E: 82 MI: 82 MA: 82 115. Refer to the above diagram. The movement from point a to point c can best be achieved by: A) increasing taxes and increasing government spending. B) increasing taxes and decreasing government spending. C) decreasing taxes and increasing government spending. D) decreasing taxes and decreasing government spending. Answer: C

Stabilization

Type: D Topic: 6 E: 82 MI: 82 MA: 82 116. The stabilization function of government involves government's efforts to: A) alter the output of specific goods when external costs or benefits are present. B) reduce the after-tax incomes of the rich and increase the after-tax incomes of the poor. C) deal with the problems of substantial unemployment and rapid inflation. D) provide the socially desired output of public goods. Answer: C

Type: A Topic: 6 E: 82 MI: 82 MA: 82 117. In performing its stabilization function it may be appropriate for government to: A) increase both government spending and taxes when the economy is experiencing rapid inflation. B) reduce government spending and increase taxes when the economy experiences substantial unemployment. C) increase government spending and reduce taxes when the economy experiences rapid inflation. D) increase government spending and reduce taxes when the economy experiences substantial unemployment. Answer: D

McConnell/Brue: Economics, 16/e

Page 160

Chapter 5: The U.S. Economy: Private and Public Sectors

Type: A Topic: 6 E: 82, 83 MI: 82, 83 MA: 82, 83 118. Which of the following would not be appropriate if government were attempting to restrain a dramatic rise in the general price level? A) an increase in tax rates C) an increase in transfer payments to households B) a decrease in subsidies to businesses D) a decrease in government spending Answer: C

Type: A Topic: 6 E: 82 MI: 82 MA: 82 119. Which of the following would not be appropriate if government were trying to reduce high unemployment? A) an increase in tax rates C) an increase in transfer payments to households B) an increase in subsidies to businesses D) an increase in government spending Answer: A

Type: A Topic: 6 E: 83 MI: 83 MA: 83 Status: New 120. In performing its stabilization function, it may be appropriate for the nation's central bank (the Federal Reserve in the United States) to: A) raise taxes to reduce inflation. B) increase government spending to reduce unemployment. C) increase subsidies to businesses to reduce unemployment. D) lower interest rates to stimulate private spending and reduce unemployment. Answer: D

Type: A Topic: 6 E: 83 MI: 83 MA: 83 Status: New 121. In performing its stabilization function, it may be appropriate for the nation's central bank (the Federal Reserve in the United States) to take actions to: A) increase taxes to reduce inflation. B) increase interest rates to reduce inflation. C) reduce government spending to reduce inflation. D) increase subsidies to businesses to reduce unemployment. Answer: B

Type: F Topic: 6 E: 83 MI: 83 MA: 83 Status: New 122. Government's economic role is complicated by the fact that: A) public goods entail no opportunity costs. B) the public sector cannot base decisions on marginal costs and marginal benefits. C) economic decisions are made in a political context. D) the marginal utility of public goods rises rather than falls. Answer: C

Government in circular flow

Type: A Topic: 7 E: 83 MI: 83 MA: 83 Status: New 123. The addition of government to the circular-flow model illustrates that government: A) purchases resources in the resource market. C) purchases goods in the product market. B) provides services to businesses and households. D) does all of the above. Answer: D

McConnell/Brue: Economics, 16/e

Page 161

Chapter 5: The U.S. Economy: Private and Public Sectors

Use the following to answer questions 124-133:

Type: G Topic: 7 E: 84 MI: 84 MA: 84 124. Refer to the above diagram, in which solid arrows reflect real flows; broken arrows are monetary flows. Flow (1) might represent: A) corporate income tax payments. B) government provision of highways for truck transportation. C) business property tax payments. D) transfer payments to low-income families. Answer: B

Type: G Topic: 7 E: 87 MI: 87 MA: 87 125. Refer to the above diagram, in which solid arrows reflect real flows; broken arrows are monetary flows. Flow (2) might represent: A) the provision of national defense by government. B) a government subsidy to farmers. C) corporate income tax payments. D) welfare payments to low-income families. Answer: C

Type: G Topic: 7 E: 83-84 MI: 83-84 MA: 83-84 126. Refer to the above diagram, in which solid arrows reflect real flows; broken arrows are monetary flows. Flow (3) might represent: A) government salaries paid to school teachers. C) a state university's purchase of computers. B) property tax payments. D) social security payments to retirees. Answer: A

Type: G Topic: 7 E: 83-84 MI: 83-84 MA: 83-84 127. Refer to the above diagram, in which solid arrows reflect real flows; broken arrows are monetary flows. Flow (4) might represent: A) the services of NASA astrophysicists. C) personal income taxes. B) the purchase of Stealth bombers. D) investment spending by private corporations. Answer: A

McConnell/Brue: Economics, 16/e

Page 162

Chapter 5: The U.S. Economy: Private and Public Sectors

Type: G Topic: 7 E: 83-84 MI: 83-84 MA: 83-84 128. Refer to the above diagram, in which solid arrows reflect real flows; broken arrows are monetary flows. Flow (5) might represent: A) personal income tax revenues. B) the provision of public schools by local governments. C) the purchase of laptop computers by the state of Iowa. D) transfer payments to unemployed workers. Answer: B

Type: G Topic: 7 E: 86-87 MI: 86-87 MA: 86-87 129. Refer to the above diagram, in which solid arrows reflect real flows; broken arrows are monetary flows. Flow (6) might represent: A) the payment of payroll taxes by households. B) corporate income tax payments. C) the purchase of basketballs by the Ogallala school district. D) the purchase of armored personnel vehicles by government. Answer: A

Type: G Topic: 7 E: 83-84 MI: 83-84 MA: 83-84 130. Refer to the above diagram, in which solid arrows reflect real flows; broken arrows are monetary flows. Flow (7) might represent: A) a transfer payment to disabled persons. B) wage payments to public school teachers. C) subsidies to corporations to stimulate exports. D) the U. S. Bureau of Engraving and Printing's expenditures for paper. Answer: D

Type: G Topic: 7 E: 83-84 MI: 83-84 MA: 83-84 131. Refer to the above diagram, in which solid arrows reflect real flows; broken arrows are monetary flows. Flow (8) might represent: A) personal income taxes. C) the services of firefighters. B) automobile purchases by the state of Maine. D) subsidies to farmers. Answer: B

Type: G Topic: 7 E: 82-83 MI: 82-83 MA: 82-83 132. Refer to the above diagram, in which solid arrows reflect real flows; broken arrows are monetary flows. If the economy were in a serious recession, it would be most appropriate for government to: A) increase flows (3) and (7) and reduce flows (2) and (6). B) decrease flows (3) and (7) and increase flows (2) and (6). C) increase flows (2) and (3) and reduce flows (6) and (7). D) increase all of the monetary flows. Answer: A

McConnell/Brue: Economics, 16/e

Page 163

Chapter 5: The U.S. Economy: Private and Public Sectors

Type: G Topic: 7 E: 82-83 MI: 82-83 MA: 82-83 133. Refer to the above diagram, in which solid arrows reflect real flows; broken arrows are monetary flows. If the economy were experiencing inflation due to excess aggregate spending, it would be most appropriate for government to: A) increase flows (3) and (7) and reduce flows (2) and (6). B) decrease flows (3) and (7) and increase flows (2) and (6). C) increase flows (2) and (3) and reduce flows (6) and (7). D) increase all of the monetary flows. Answer: B

Purchases, transfers, and government size

Type: F Topic: 8 E: 84 MI: 84 MA: 84 134. In 2003, "Tax-Freedom Day" (the day average workers have earned enough to pay their tax bills) was: A) February 1. B) April 5. C) April 18. D) July 7. Answer: C

Type: F Topic: 8 E: 84-85 MI: 84-85 MA: 84-85 135. Transfer payments are about ____ percent of U. S. domestic output. A) 50 percent B) 5 percent C) 12 percent D) 22 percent Answer: C

Type: F Topic: 8 E: 84 MI: 84 MA: 84 136. Total governmental purchases--Federal, state, and local combined--account for about what percentage of domestic output? A) 30 percent B) 18 percent C) 10 percent D) 5 percent Answer: B

Type: F Topic: 8 E: 84 MI: 84 MA: 84 137. The total amount of U. S. tax revenue needed to finance the public sector: A) has been a declining percentage of the domestic output in this century. B) equals about 40 percent of domestic output. C) equals about 15 percent of domestic output. D) is slightly larger today than in 1960. Answer: D

Type: F Topic: 8 E: 85 MI: 85 MA: 85 138. As a proportion of domestic outputs, taxes in the United States: A) are lower than in most other industrially advanced countries. B) are higher than in most other industrially advanced countries. C) are approximately the same as in most other industrially advanced countries. D) doubled in the 1990s. Answer: A

McConnell/Brue: Economics, 16/e

Page 164

Chapter 5: The U.S. Economy: Private and Public Sectors

Type: A Topic: 8 E: 84 MI: 84 MA: 84 139. Which of the following is an exhaustive governmental outlay? A) a Federal $5,000 subsidy check to an Illinois farmer B) a Temporary Assistance to Needy Families payment made by the state of New York C) a NASA payment to Boeing Corporation for space hardware D) a Federal old age insurance payment to a retired coal miner Answer: C

Type: A Topic: 8 E: 84 MI: 84 MA: 84 140. Government purchases and transfer payments: A) differ because the latter absorb resources while the former do not. B) differ because the former absorb resources while the latter do not. C) are alike because both are more inflationary than private spending. D) are alike because both absorb resources. Answer: B

Type: A Topic: 8 E: 84 MI: 84 MA: 84 141. Government transfer payments: A) have been virtually eliminated by Federal revenue sharing. B) have virtually no effect on the distribution of income. C) make the distribution of income less equal. D) make the distribution of income less unequal. Answer: D

Type: A Topic: 8 E: 84 MI: 84 MA: 84 142. Transfer payments differ from government purchases in that the: A) former make a contribution to the domestic output, while the latter do not. B) former are associated with state and local governments, while the latter are associated with the Federal government. C) latter are not directly resource absorbing, while the former are. D) former are not directly resource absorbing, while the latter are. Answer: D

Federal finance and progressive taxes

Type: F Topic: 9 E: 85 MI: 85 MA: 85 143. The major source of tax revenue for the Federal government is: A) personal income taxes. C) corporate income taxes. B) property taxes. D) sales and excise taxes. Answer: A

Type: F Topic: 9 E: 86 MI: 86 MA: 86 144. Approximately what percentage of the Federal government's tax revenues are currently in the form of personal income taxes? A) 80 percent B) 15 percent C) 33 percent D) 46 percent Answer: D

McConnell/Brue: Economics, 16/e

Page 165

Chapter 5: The U.S. Economy: Private and Public Sectors

Type: F Topic: 9 E: 86 MI: 86 MA: 86 145. The three most important sources of Federal tax revenue in order of descending importance are: A) sales, payroll, and personal income taxes. B) personal income, corporate income, and sales taxes. C) personal income, corporate income, and payroll taxes. D) personal income, payroll, and corporate income taxes. Answer: D

Type: D Topic: 9 E: 85 MI: 85 MA: 85 146. In determining one's personal income tax, taxable income is: A) total income less deductions and exemptions. C) all income other than wages and salaries. B) earned income less property income. D) wage and salary income only. Answer: A

Type: F Topic: 9 E: 85-86 MI: 85-86 MA: 85-86 147. The largest category of Federal spending is for: A) agriculture and rural development. C) pensions and income security. B) science, space, and technology. D) highway construction. Answer: C

Type: A Topic: 9 E: 86 MI: 86 MA: 86 148. The tax rates embodied in the Federal personal income tax are such that: A) a rising absolute amount, but a declining proportion, of income is paid in taxes. B) the marginal and average tax rates are equal, making the tax progressive. C) the average tax rate rises more rapidly than does the marginal tax rate. D) the marginal tax rate is higher than the average tax rate, causing the average tax rate to rise. Answer: D

Type: F Topic: 9 E: 86 MI: 86 MA: 86 Status: New 149. The maximum Federal marginal tax rate on taxable personal income is currently (2003): A) 50 percent. B) 35 percent. C) 45 percent. D) 28 percent. Answer: B

Type: F Topic: 9 E: 86 MI: 86 MA: 86 150. Which of the following is not an important source of revenue for the Federal government? A) corporate income taxes B) property taxes C) payroll taxes D) personal income taxes Answer: B

Type: D Topic: 9 E: 85-86 MI: 85-86 MA: 85-86 151. A progressive tax is such that: A) tax rates are higher the greater one's income. B) the same tax rate applies to all income receivers, so that the rich pay absolutely more taxes than the poor. C) entrepreneurial income is exempt from taxation. D) the revenues it yields are spent on transfer payments. Answer: A

McConnell/Brue: Economics, 16/e

Page 166

Chapter 5: The U.S. Economy: Private and Public Sectors

Type: D Topic: 9 E: 85-86 MI: 85-86 MA: 85-86 152. An income tax is progressive if the: A) absolute amount paid as taxes varies directly with income. B) percentage of income paid as taxes is the same regardless of the size of income. C) percentage of income paid as taxes increases as income increases. D) tax rate varies inversely with income. Answer: C

Type: D Topic: 9 E: 86 MI: 86 MA: 86 153. The average tax rate is: A) equal to the marginal tax rate if the tax is progressive. B) the total tax rate minus the marginal tax rate. C) the ratio of total taxes paid to total taxable income. D) the tax rate that applies to incremental dollars of income. Answer: C

Type: A Topic: 9 E: 86 MI: 86 MA: 86 154. If you would have to pay $5000 in taxes on a $25,000 taxable income and $7000 on a $30,000 taxable income, then the marginal tax rate on the additional $5000 of income is: A) 40 percent and the average tax rate is about 23 percent at the $30,000 income level. B) 50 percent and the average tax rate is 40 percent at the $30,000 income level. C) 40 percent and the average tax rate is 25 percent at the $25,000 income level. D) 30 percent but average tax rates cannot be determined from the information given. Answer: A

Type: D Topic: 9 E: 86 MI: 86 MA: 86 155. The marginal tax rate is: A) the difference between the total tax rate and the average tax rate. B) the percentage of total income paid as taxes. C) change in taxes / change in taxable income D) total taxes / total taxable income Answer: C

Type: D Topic: 9 E: 86 MI: 86 MA: 86 156. The average tax rate is: A) change in taxes / change in taxable income B) total taxes / total taxable income C) the sum of the marginal tax rate and the rate of transfer payments. D) the tax on incremental income less the tax on total income. Answer: B

Type: A Topic: 9 E: 86 MI: 86 MA: 86 157. For a progressive tax the: A) average tax rate exceeds the marginal tax rate as income rises. B) marginal tax rate declines as income increases. C) average and marginal tax rates are equal. D) marginal tax rate exceeds the average tax rate as income rises. Answer: D

McConnell/Brue: Economics, 16/e

Page 167

Chapter 5: The U.S. Economy: Private and Public Sectors

Type: A Topic: 9 E: 86 MI: 86 MA: 86 158. Assume that in year 1 you pay an average tax rate of 20 percent on a taxable income of $20,000. In year 2, you pay an average tax rate of 25 percent on a taxable income of $30,000. Assuming no change in tax rates, the marginal tax rate on your additional $10,000 of income is: A) 5 percent. B) 12 percent. C) 35 percent. D) 42 percent. Answer: C

Type: D Topic: 9 E: 86 MI: 86 MA: 86 159. The marginal tax rate is: A) less than the average tax rate when a tax is progressive. B) calculated by dividing total taxes paid by one's total taxable income. C) the percentage of one's total income that is paid in taxes. D) the percentage of an increment of income that is paid in taxes. Answer: D

Type: F Topic: 9 E: 86 MI: 86 MA: 86 Status: New 160. Currently (2003) the marginal tax rates of the Federal personal income tax: A) are less than corresponding average tax rates. C) fall from 20 to 10 percent. B) rise from 5 to 50 percent. D) rise from 10 to 35 percent. Answer: D

Type: A Topic: 9 E: 86 MI: 86 MA: 86 161. Assume that in year 1 your average tax rate is 20 percent on a taxable income of $20,000. If the marginal tax rate on the next $10,000 of taxable income is 30 percent, what will be the average tax rate if your taxable income rises to $30,000? A) 7 percent B) 30 percent C) 16 percent D) about 23 percent Answer: D

Use the following to answer questions 162-163: The following data represent a personal income tax schedule. Answer the next question(s) on the basis of this information.

Taxable income $ 0 2000 4000 6000 8000 10,000

Total tax $ 0 200 600 1200 2000 3000

Type: T Topic: 9 E: 86 MI: 86 MA: 86 162. Refer to the above table. If your taxable income is $8,000, your average tax rate is: A) 25 percent and the marginal rate on additional income is also 25 percent. B) 25 percent and the marginal rate on additional income is 50 percent. C) 25 percent and the marginal rate on additional income cannot be determined from the information given. D) 20 percent and the marginal rate on additional income is 30 percent. Answer: B

McConnell/Brue: Economics, 16/e

Page 168

Chapter 5: The U.S. Economy: Private and Public Sectors

Type: T Topic: 9 E: 85-86 MI: 85-86 MA: 85-86 163. Refer to the above table. This tax is such that the after-tax distribution of income will be: A) more equal than the before-tax distribution. B) less equal than the before-tax distribution. C) distributed in precisely the same way as the before-tax distribution. D) less than the before-tax distribution by the same percentage at each income level. Answer: A

Use the following to answer questions 164-166: Answer the next question(s) on the basis of the following data:
Taxable income $1000 2000 3000 4000 5000 6000 Total tax $ 0 100 300 600 1000 1500

Type: T Topic: 9 E: 85-86 MI: 85-86 MA: 85-86 164. The tax represented above is: A) optimal. B) proportional. C) regressive. D) progressive. Answer: D

Type: T Topic: 9 E: 86 MI: 86 MA: 86 165. Refer to the above data. If your taxable income is $4000, your average tax rate will be: A) 20 percent. B) 15 percent. C) 10 percent. D) 5 percent. Answer: B

Type: T Topic: 9 E: 86 MI: 86 MA: 86 166. Refer to the above data. If your taxable income increases from $4000 to $5000, you will encounter a marginal tax rate of: A) 40 percent. B) 25 percent. C) 15 percent. D) 10 percent. Answer: A

Type: A Topic: 9 E: 86 MI: 86 MA: 86 167. If the marginal tax rate is greater than the average tax rate, then: A) the average tax rate must be rising. B) the average tax rate must be falling. C) the average tax rate may be either rising or falling. D) the tax is regressive. Answer: A

Type: F Topic: 9 E: 85-86 MI: 85-86 MA: 85-86 168. The most important source of Federal tax revenue is: A) sales taxes. B) personal income taxes. C) corporate income taxes. Answer: B

D) payroll taxes.

McConnell/Brue: Economics, 16/e

Page 169

Chapter 5: The U.S. Economy: Private and Public Sectors

Type: D Topic: 9 E: 85 MI: 85 MA: 85 169. Taxable income is: A) total income less deductions and exemptions. B) the same as gross income. Answer: A

C) only income to which marginal tax rates apply. D) the sum of all wage and property income.

Type: F Topic: 9 E: 87 MI: 87 MA: 87 170. The basic tax rate on taxable corporate income is: A) 15 percent. B) 22 percent. C) 35 percent. Answer: C

D) 52 percent.

State and local finance

Type: F Topic: 10 E: 87 MI: 87 MA: 87 171. With respect to state finance: A) death and gift taxes are the major source of revenue and most expenditures are for hospitals and health services. B) the corporate income tax is the major source of revenue and natural resource development the major type of expenditure. C) property taxes are the basic source of revenue and education is the major type of expenditure. D) sales and excise taxes are the major source of revenue and education is the major type of expenditure. Answer: D

Type: D Topic: 10 E: 87 MI: 87 MA: 87 172. The essential difference between sales and excise taxes is that: A) sales taxes apply to a wide range of products, while excises apply only to a select group of products. B) excise taxes apply to a wide range of products, while sales taxes apply only to a select list of products. C) sales taxes are consumption taxes, while excises are not. D) excise taxes are consumption taxes, while sales taxes are not. Answer: A

Type: F Topic: 10 E: 87 MI: 87 MA: 87 173. With respect to local finance: A) death and gift taxes are the major source of revenue and most expenditures are for hospitals and health services. B) the corporate income tax is the major source of revenue and natural resource development the major type of expenditure. C) property taxes are the basic source of revenue and education is the major type of expenditure. D) sales and excise taxes are the major source of revenue and highway construction and maintenance is the major type of expenditure. Answer: C

Type: F Topic: 10 E: 87 MI: 87 MA: 87 174. Government lotteries are: A) used by a large number of states to supplement their tax revenues. B) illegal in the United States, but are a common source of revenue in other countries. C) used by local governments, but not by state governments. D) a form of progressive taxation. Answer: A

McConnell/Brue: Economics, 16/e

Page 170

Chapter 5: The U.S. Economy: Private and Public Sectors

Consider This Questions

Type: A E: 82 MI: 82 MA: 82 Status: New 175. (Consider This) Street entertainers generally have a difficult time making large incomes from their performances because they: A) are not sufficiently talented. B) provide public goods and therefore face free-rider problems. C) fail to attract audiences. D) tend to lack marketing skills. Answer: B

Type: A E: 82 MI: 82 MA: 82 Status: New 176. (Consider This) Which of the following groups face a severe free-rider problem in providing their services? A) store clerks B) tax accountants C) college professors D) street entertainers Answer: D

Last Word Questions

Type: F E: 88 MI: 88 MA: 88 177. (Last Word) Corporate common stockholders: A) are guaranteed an annual fixed payment for each share of stock they own. B) may or may not receive dividends, depending on the prosperity of the corporation. C) are guaranteed a fixed payment for their shares when they mature. D) are entitled to dividends before any payments can be made to bondholders. Answer: B

Type: D E: 88 MI: 88 MA: 88 178. (Last Word) "Bondholders have a 'legally prior claim' on a corporation's earnings." This means that: A) corporate bonds are riskless. B) bondholders have the power to change the interest rate they receive each year. C) interest must be paid on bonds before any dividends can be paid to stockholders. D) the market value of bonds cannot fluctuate from year to year. Answer: C

Type: A E: 88 MI: 88 MA: 88 179. (Last Word) Which of the following is correct? A) The market price of bonds and interest rates change in the same direction. B) The market price of bonds and interest rates are unrelated. C) Changes in interest rates affect stock prices, but not bond prices. D) The market price of bonds and interest rates are inversely related. Answer: D

McConnell/Brue: Economics, 16/e

Page 171

Chapter 5: The U.S. Economy: Private and Public Sectors

Type: A E: 88 MI: 88 MA: 88 180. (Last Word) Bonds entail risks because: A) the issuing corporation may encounter economic misfortune or go bankrupt. B) interest rates may rise. C) inflation may occur. D) of all of the above reasons. Answer: D

True/False Questions

Type: F E: 77 MI: 77 MA: 77 181. Only the bondholders of a corporation have the right to vote for a corporation's directors. Answer: False

Type: F E: 77 MI: 77 MA: 77 182. Limited liability means all members of a partnership are liable for the debts incurred by one another. Answer: False

Type: D E: 76 MI: 76 MA: 76 183. A vertically integrated firm is a group of plants each operating at different stages of production. Answer: True

Type: D E: 75 MI: 75 MA: 75 184. Households spend a larger proportion of their incomes for services than for either nondurable goods or durable goods. Answer: True

Type: F E: 74 MI: 74 MA: 74 185. Wages and salaries are the largest source of household income. Answer: True

Type: F E: 74-75 MI: 74-75 MA: 74-75 186. If total income is $200 billion, personal taxes are $40 billion, and consumption is $140 billion, then personal saving must be $60 billion. Answer: False

Type: A E: 80 MI: 80 MA: 80 187. When there are spillover costs involved in the production of a good, fewer resources are allocated to its production than if all costs were internalized. Answer: False

Type: A E: 80 MI: 80 MA: 80 188. If a good's production creates substantial spillover costs, then too little of it will be produced unless firms are subsidized. Answer: False

McConnell/Brue: Economics, 16/e

Page 172

Chapter 5: The U.S. Economy: Private and Public Sectors

Type: A E: 82-83 MI: 82-83 MA: 82-83 189. Increasing taxes and reducing government expenditures might be helpful in constraining inflation. Answer: True

Type: A E: 80 MI: 80 MA: 80 190. If a market is competitive, the resulting equilibrium output will always be the socially efficient output. Answer: False

Type: A E: 81 MI: 81 MA: 81 191. The free-rider problem refers to the possibility that someone may benefit from a good without paying for it. Answer: True

Type: D E: 81 MI: 81 MA: 81 192. The misallocation of resources resulting from large spillover benefits can best be corrected by levying a specific tax on the production of that good or service. Answer: False

Type: D E: 81 MI: 81 MA: 81 Status: New 193. Public goods are characterized by nonrivalry and nonexcludability. Answer: True

Type: D E: 81 MI: 81 MA: 81 Status: New 194. As it relates to public goods, nonexcludability means that nonpayers cannot be barred from obtaining the benefits. Answer: True

Type: A E: 81 MI: 81 MA: 81 Status: New 195. An example of a public good is cable television. Answer: False

Type: F E: 84 MI: 84 MA: 84 Status: New 196. "Tax Freedom Day" in the United occurs in mid-July. Answer: False

Type: A E: 79-80 MI: 79-80 MA: 79-80 197. Government assumes some responsibility for providing a minimum standard of living for all citizens to compensate for the increase in income inequality caused by government tax revenues and expenditures. Answer: False

Type: A E: 82 MI: 82 MA: 82 198. If there is substantial unemployment in the economy, it would be appropriate for government to cut tax rates. Answer: True

McConnell/Brue: Economics, 16/e

Page 173

Chapter 5: The U.S. Economy: Private and Public Sectors

Type: A E: 87 MI: 87 MA: 87 199. The largest source of local government's revenue is the sales tax. Answer: False

Type: F E: 87 MI: 87 MA: 87 200. The basic source of state government's revenue is the property tax. Answer: False

Type: F E: 87 MI: 87 MA: 87 201. The major expenditure of local governments is for education. Answer: True

Type: F E: 85 MI: 85 MA: 85 202. A tax is progressive if the average tax rate rises as income increases. Answer: True

Type: D E: 84 MI: 84 MA: 84 203. Unemployment compensation payments are an example of a transfer payment. Answer: True

Type: A E: 85-86 MI: 85-86 MA: 85-86 204. If you pay a $2000 tax on $10,000 of taxable income and a $4000 tax on a taxable income of $16,000, the tax is progressive. Answer: True

Type: A E: 86 MI: 86 MA: 86 205. The marginal tax rate is the tax rate that applies to additional income. Answer: True

Type: D E: 86 MI: 86 MA: 86 206. In descending order the three most important sources of Federal tax revenue are the personal income tax, payroll taxes, and corporate income taxes. Answer: True

Type: F E: 86 MI: 86 MA: 86 207. About two-thirds of all Federal spending is for national defense. Answer: False

Type: F E: 86-87 MI: 86-87 MA: 86-87 208. A progressive tax takes relatively more from the rich than it does from the poor. Answer: True

McConnell/Brue: Economics, 16/e

Page 174

CHAPTER 6

The United States in the Global Economy

Topic Question numbers __________________________________________________________ International linkages 1-7 Trade: volume, facilitating factors, participants 8-25 Specialization and comparative advantage 26-50 Foreign exchange market 51-68 Trade barriers 69-74 Multilateral agreements: free trade zones; implications of the global economy 75-102 Consider This 103-106 Last Word 107-108 True-False 109-124 __________________________________________________________ Multiple Choice Questions International linkages 1. 2. 3. 4. 5. 6.

Type: D Topic: 1 E: 91 MI: 91 MA: 91 1. The physical export of motorcycles from the United States to Mexico best illustrates a: A) trade flow. B) resource flow. C) financial flow. D) technology flow. Answer: A

Type: D Topic: 1 E: 91 MI: 91 MA: 91 2. The physical import of DVD players to the United States from Japan best illustrates a: A) resource flow. B) financial flow. C) trade flow. D) technology flow. Answer: C

Type: D Topic: 1 E: 92 MI: 92 MA: 92 3. The spending by Americans while traveling in Europe best illustrates a: A) trade flow. B) labor flow. C) financial flow. D) technology flow. Answer: C

Type: D Topic: 1 E: 91 MI: 91 MA: 91 4. The emigration of software designers from India to the United States best illustrates a(n): A) trade flow. B) resource flow. C) financial flow. D) information flow. Answer: B

Chapter 6: The United States in the Global Economy

Type: D Topic: 1 E: 91 MI: 91 MA: 91 5. The purchase by an American firm of the right to produce a prescription drug patented in Germany best illustrates a: A) trade flow. B) capital flow. C) goods and services flow. D) technology flow. Answer: D

Type: D Topic: 1 E: 91 MI: 91 MA: 91 6. The business-to-business (B2B) retrieval of prices of foreign resources via the Internet best illustrates a(n): A) trade flow. B) capital and labor flow. C) financial flow. D) information flow. Answer: D

Type: D Topic: 1 E: 91 MI: 91 MA: 91 7. The building of a production plant in China by an American firm best illustrates a(n): A) trade flow. B) resource flow. C) financial flow. D) information flow. Answer: B

Trade: volume, facilitating factors, participants

Type: F Topic: 2 E: 92 MI: 92 MA: 92 8. American exports and imports of goods and services each are about what percentage of GDP? A) 4-6 percent B) 25-28 percent C) 11-16 percent D) 30-32 percent Answer: C

Type: F Topic: 2 E: 92 MI: 92 MA: 92 9. Which of the following statements is correct? A) The United States' exports and imports are smaller absolutely, but larger as a percentage of GDP, than other nations'. B) A number of other nations have exports and imports that are absolutely larger than those of the United States. C) The United States' exports and imports are absolutely larger than any other nation's, but the exports and imports of many other nations are a larger percentage of their GDPs. D) The United States' exports and imports are larger absolutely and as a percentage of GDP than any other nation's. Answer: C

Type: F Topic: 2 E: 94 MI: 94 MA: 94 10. More than half of United States international trade is with: A) other industrially advanced capitalist countries. C) developing countries. B) the OPEC countries. D) Russia and China. Answer: A

Type: F Topic: 2 E: 94 MI: 94 MA: 94 11. The United States' most important trading partner in terms of dollar volume is: A) Mexico. B) Canada. C) Germany. D) Japan. Answer: B

McConnell/Brue: Economics, 16/e

Page 176

Chapter 6: The United States in the Global Economy

Type: F Topic: 2 E: 92 MI: 92 MA: 92 12. In terms of absolute volumes of imports and exports, the world's leading trading nation is: A) France. B) Japan. C) the United States. D) South Korea. Answer: C

Type: F Topic: 2 E: 92 MI: 92 MA: 92 13. Which of the following statements is correct? A) United States exports and imports have been decreasing as a percentage of U.S. GDP but the U.S. share of total world trade has been increasing. B) United States exports and imports have been decreasing as a percentage of U.S. GDP and the U.S. share of total world trade has been declining. C) United States exports and imports have been expanding as a percentage of U.S. GDP and the U.S. share of total world trade has been increasing. D) United States exports and imports have been expanding as a percentage of U.S. GDP but the U.S. share of total world trade has been declining. Answer: D

Type: F Topic: 2 E: 92 MI: 92 MA: 92 14. Since 1960, United States exports and imports have: A) grown absolutely, but remained a constant proportion of GDP. B) grown absolutely, but declined as a proportion of GDP. C) grown both absolutely and as a percentage of GDP. D) declined both absolutely and as a percentage of GDP. Answer: C

Type: F Topic: 2 E: 93 MI: 93 MA: 93 15. In recent years the United States has: A) exported more goods and services than it has imported. B) imported more goods and services than it has exported. C) realized an approximate balance in its imports and exports. D) experienced a falling absolute dollar amount of imports and a rising absolute dollar amount of exports. Answer: B

Type: F Topic: 2 E: 94 MI: 94 MA: 94 16. More than half of the U.S. international trade is with: A) the nations of Eastern Europe. B) the developing countries of Africa, Asia, and Latin America. C) other industrialized nations, for example, Canada, Japan, and the countries of Western Europe. D) China. Answer: C

Type: F Topic: 2 E: 92 MI: 92 MA: 92 17. Which of the following is a true statement? A) The United States has the world's largest ratio of exports to GDP. B) The United States is almost entirely dependent on other countries in obtaining items such as silk, nickel, tin, and coffee. C) U.S. exports to China greatly exceed U.S. imports from China. D) Since 1947 the United States has accounted for a rising percentage of total world trade. Answer: B

McConnell/Brue: Economics, 16/e

Page 177

Chapter 6: The United States in the Global Economy

Type: F Topic: 2 E: 92 MI: 92 MA: 92 18. As a percentage of GDP (total output), U.S. exports are: A) about 20 percent. B) lower than in some other industrial countries, including Germany and Canada. C) less today than they were in 1975. D) the highest in the world. Answer: B

Type: F Topic: 2 E: 95 MI: 95 MA: 95 19. The average U.S. tariff rate on imported goods is about: A) 5 percent. B) 12 percent. C) 25 percent. D) 50 percent. Answer: A

Type: F Topic: 2 E: 93 MI: 93 MA: 93 20. The major goods exports of the United States (in dollar volume) are: A) chemicals, semiconductors, consumer durables, and computers. B) petroleum, automobiles, clothing, and household appliances. C) iron and steel, clothing, beef, and sugar. D) aircraft, glassware, television sets, and furniture. Answer: A

Type: F Topic: 2 E: 93 MI: 93 MA: 93 21. The major goods imports of the United States (in dollar volume) are: A) chemicals, consumer durables, aircraft, and grain. B) automobiles, petroleum, computers, and household appliances. C) iron and steel, clothing, electronic equipment, and sugar. D) aircraft, paper products, television sets, and furniture. Answer: B

Type: F Topic: 2 E: 95 MI: 95 MA: 95 22. Which of the following has not been a facilitating factor in world trade? A) dramatic improvements in communications technology B) general declines in tariffs C) import quotas D) improvements in transportation technology. Answer: C

Type: F Topic: 2 E: 95 MI: 95 MA: 95 23. In terms of absolute dollar volume, the world's leading export nations are: A) the United States, Germany, and Japan. C) Japan, China, and Great Britain. B) the United States, Japan, and Canada. D) Japan, the United States, and France. Answer: A

Type: T Topic: 2 E: 95 MI: 95 MA: 95 Status: New 24. Which of the following countries has recently emerged as one of the world's top trading nations in terms of total trade volume? A) Chile B) India C) Ireland D) China Answer: D

McConnell/Brue: Economics, 16/e

Page 178

Chapter 6: The United States in the Global Economy

Type: D Topic: 2 E: 95 MI: 95 MA: 95 25. Multinational corporations: A) mainly are headquartered in Switzerland. B) are so named because of their heavy export volume. C) are illegal under the U.S. antitrust laws. D) are so named because of their sizable foreign production and distribution assets. Answer: D

Specialization and comparative advantage

Type: T Topic: 3 E: 97 MI: 97 MA: 97 Status: New 26. Which of the following concepts provides the basic rationale for international trade? A) increasing opportunity costs. C) comparative advantage. B) consumer sovereignty. D) the law of supply. Answer: C

Use the following to answer questions 27-35:
Alpha’s production possibilities: A 60 0 B 45 15 C 30 30 D 15 45 E 0 60

Steel (tons) Wheat (tons)

Omega’s production possibilities : A 20 0 B 15 15 C 10 30 D 5 45 E 0 60

Steel (tons) Wheat (tons)

Type: T Topic: 3 E: 97 MI: 97 MA: 97 Status: New 27. The above data would graph as: A) a straight line for Alpha, but as a concave curve for Omega. B) a concave curve for Alpha, but as a straight line for Omega. C) concave curves for both Alpha and Omega. D) straight lines for both Alpha and Omega. Answer: D

Type: T Topic: 3 E: 97 MI: 97 MA: 97 Status: New 28. Refer to the above data. Alpha is a(n): A) increasing cost economy, whereas Omega is a constant cost economy. B) constant cost economy, whereas Omega is an increasing cost economy. C) increasing cost economy, as is Omega. D) constant cost economy, as is Omega. Answer: D

McConnell/Brue: Economics, 16/e

Page 179

Chapter 6: The United States in the Global Economy

Type: T Topic: 3 E: 97 MI: 97 MA: 97 Status: New 29. Refer to the above data. The domestic opportunity cost of producing 1 ton of steel in Alpha is: A) ½ ton of wheat. B) 1 ton of wheat. C) 15 tons of wheat. D) 30 tons of wheat. Answer: B

Type: T Topic: 3 E: 97 MI: 97 MA: 97 Status: New 30. Refer to the above data. The domestic opportunity cost of producing 1 ton of steel in Omega is: A) ½ ton of wheat. B) 2 tons of wheat. C) 3 tons of wheat. D) 5 tons of wheat. Answer: C

Type: T Topic: 3 E: 97 MI: 97 MA: 97 Status: New 31. Refer to the above data. Alpha has a comparative advantage in producing: A) neither steel nor wheat. B) both steel and wheat. C) steel. D) wheat. Answer: C

Type: T Topic: 3 E: 98 MI: 98 MA: 98 Status: New 32. Refer to the above data. On the basis of the above information: A) Alpha should export both steel and wheat to Omega. B) Omega should export both steel and wheat to Alpha. C) Omega should export steel to Alpha and Alpha should export wheat to Omega. D) Alpha should export steel to Omega and Omega should export wheat to Alpha. Answer: C

Type: T Topic: 3 E: 98 MI: 98 MA: 98 Status: New 33. Refer to the above data. After specialization, Alpha will produce: A) 60 tons of steel and Omega will produce 45 tons of wheat. B) 20 tons of steel and Omega will produce 60 tons of wheat. C) 60 tons of steel and Omega will produce 60 tons of wheat. D) 30 tons of steel and Omega will produce 30 tons of wheat. Answer: C

Type: T Topic: 3 E: 98 MI: 98 MA: 98 Status: New 34. Refer to the above data. If Alpha and Omega each were producing at alternatives B before trade, the gain from specialization and trade would be: A) 30 tons of wheat. C) 30 tons of steel and 30 tons of wheat. B) 15 tons of steel. D) 60 tons of wheat and 60 tons of steel. Answer: A

Type: T Topic: 3 E: 98 MI: 98 MA: 98 Status: New 35. Refer to the above data. If Alpha was producing at alternative B and Omega was at alternative C before trade, the gain from specialization and trade would be: A) 30 tons of wheat. C) 5 tons of steel and 15 tons of wheat. B) 5 tons of steel. D) 15 tons of steel and 5 tons of wheat. Answer: C

McConnell/Brue: Economics, 16/e

Page 180

Chapter 6: The United States in the Global Economy

Type: D Topic: 3 E: 97 MI: 97 MA: 97 36. According to the concept of comparative advantage, a good should be produced in that nation where: A) its domestic opportunity cost is greatest. C) its domestic opportunity cost is least. B) money is used as a medium of exchange. D) the terms of trade are maximized. Answer: C

Type: D Topic: 3 E: 98 MI: 98 MA: 98 37. The terms of trade: A) show the ratio at which nations will exchange two goods. B) show how the gains from trade can be equally shared. C) show the value of one nation's currency in terms of another nation's currency. D) compare the volume of a nation's exports and imports. Answer: A

Use the following to answer questions 38-43: Answer the next question(s) on the basis of the following production possibilities data for Landia and Scandia:
Landia production possibilities: A 8 0 B 6 10 C 4 20 D 2 30 E 0 40

Fish Chips

Scandia production possibilities: A 16 0 B 12 12 C 8 24 D 4 36 E 0 48

Fish Chips

Type: T Topic: 3 E: 97 MI: 97 MA: 97 38. Refer to the above data. The domestic opportunity cost of 1 fish in Landia is: A) 10 chips. B) 2 chips. C) 4 chips. D) 5 chips. Answer: D

Type: T Topic: 3 E: 97 MI: 97 MA: 97 39. Refer to the above data. The domestic opportunity cost of 1 fish in Scandia is: A) 12 chips. B) 4 chips. C) 3 chips. D) 1 chip. Answer: C

Type: C Topic: 3 E: 97 MI: 97 MA: 97 40. On the basis of the production possibilities data shown: A) Landia has a comparative advantage in chips while Scandia has a comparative advantage in fish. B) Landia has a comparative advantage in fish while Scandia has a comparative advantage in chips. C) both Landia and Scandia have a comparative advantage in fish. D) both Landia and Scandia have a comparative advantage in chips. Answer: A

McConnell/Brue: Economics, 16/e

Page 181

Chapter 6: The United States in the Global Economy

Type: C Topic: 3 E: 98 MI: 98 MA: 98 41. Refer to the above data. If Landia and Scandia fully specialize based on comparative advantage, their aggregate output will be: A) 48 chips and 8 fish. B) 40 chips and 16 fish. C) 36 chips and 10 fish. D) 42 chips and 12 fish. Answer: B

Type: C Topic: 3 E: 98 MI: 98 MA: 98 42. Refer to the above data. Assume that before specialization and trade Landia was producing combination C and Scandia was producing combination B. If these two nations now specialize completely based on with comparative advantage, the total gains from specialization and trade would be: A) 8 fish and 2 chips. B) 10 fish and 4 chips. C) 0 fish and 8 chips. D) 4 fish and 6 chips. Answer: C

Type: C Topic: 3 E: 98 MI: 98 MA: 98 43. Refer to the above data. Which of the following would be feasible terms of trade between Landia and Scandia? A) 1 fish for 4 chips B) 1 fish for 6 chips C) 1 fish for 7 chips D) 2 fish for 4 chips Answer: A

Type: A Topic: 3 E: 96 MI: 96 MA: 96 44. The primary benefits of international trade include: A) the more efficient use of world resources and higher living standards. B) greater stability of domestic output, employment, and the price level. C) diminished dependence on foreign supplies of goods and materials. D) greater economic security for our domestic producers. Answer: A

Use the following to answer questions 45-49: Answer the next question(s) on the basis of the following production possibilities tables for countries Alpha and Beta:
Alpha Production Possibilities Product A B C D X 3 2 1 0 Y 0 4 8 12 Beta Product X Y Production Possibilities A B C D 6 4 2 0 0 4 8 12

Type: T Topic: 3 E: 97 MI: 97 MA: 97 45. Refer to the above tables. The domestic opportunity cost of one unit of X in Alpha is: A) 2 units of Y. B) 4 units of Y. C) 1 unit of Y. D) 3 units of Y. Answer: B

Type: T Topic: 3 E: 97 MI: 97 MA: 97 46. Refer to the above tables. The domestic opportunity cost of one unit of X in Beta is: A) 2 units of Y. B) 4 units of Y. C) 1 unit of Y. D) 3 units of Y. Answer: A

McConnell/Brue: Economics, 16/e

Page 182

Chapter 6: The United States in the Global Economy

Type: T Topic: 3 E: 98 MI: 98 MA: 98 47. Refer to the above tables. According to the concept of comparative advantage: A) Alpha should specialize in X; Beta in Y. C) Alpha should produce some X and some Y. B) Beta should produce some X and some Y. D) Beta should specialize in X; Alpha in Y. Answer: D

Type: T Topic: 3 E: 98 MI: 98 MA: 98 48. Refer to the above tables. Assume that before specialization both nations chose to produce alternative B. The gains from specialization and trade would be: A) 2 units of X and 2 units of Y. B) 4 units of X. C) 4 units of Y. D) 6 units of X and 3 units of Y. Answer: C

Type: T Topic: 3 E: 98 MI: 98 MA: 98 49. Refer to the above tables. Which one of the following terms of trade would be acceptable to both countries? A) 1 unit of X for 3 units of Y C) 1 unit of X for 12 units of Y B) 1 unit of X for 5 units of Y D) 1 unit of X for 1 unit of Y Answer: A

Type: G Topic: 3 E: 99 MI: 99 MA: 99 50.

Refer to the above domestic production possibilities curve for Abernorelco. The gain to Abernorelco from specialization and international trade is represented by a move from: A) A to B. B) C to A. C) C to D. D) B to E. Answer: D

Foreign exchange market

Type: A Topic: 4 E: 99 MI: 99 MA: 99 51. Exchange rates are particularly important because: A) they present a challenge to financial speculators. B) they link the price levels of various nations to one another. C) they represent exceptions to the laws of demand and supply. D) equilibrium is never achieved in such markets. Answer: B

McConnell/Brue: Economics, 16/e

Page 183

Chapter 6: The United States in the Global Economy

Type: A Topic: 4 E: 100-101 MI: 100-101 MA: 100-101 52. If the equilibrium exchange rate changes so that it takes more dollars to buy a British pound, then: A) the dollar has appreciated in value. C) the British will buy fewer U.S. goods. B) Americans will import more British goods. D) the dollar has depreciated in value. Answer: D

Type: A Topic: 4 E: 100-101 MI: 100-101 MA: 100-101 53. If incomes rise rapidly in the United States and U.S. preferences for foreign goods strengthen, we would expect: A) the dollar to appreciate in value. C) the dollar price of foreign monies to decrease. B) the dollar to depreciate in value. D) U.S. exports to increase. Answer: B

Type: A Topic: 4 E: 101 MI: 101 MA: 101 54. If the exchange rate changes from $1 = 2 euros to $1 = 3 euros: A) the dollar has appreciated in value. B) the dollar has depreciated in value. C) the dollar has neither appreciated nor depreciated, but the euro has appreciated in value. D) U.S. exports to Europe will increase. Answer: A

Type: A Topic: 4 E: 100 MI: 100 MA: 100 55. Mexican imports of U.S. goods: A) create a supply of pesos. B) create a supply of dollars. C) reduce the demand for dollars. D) have no effect on the peso-dollar exchange rate. Answer: A

Type: A Topic: 4 E: 101 MI: 101 MA: 101 56. A change in the dollar price of yen from $1 = 100 yen to $1 = 50 yen will: A) make U.S. goods more expensive to the Japanese. B) make Japanese goods less expensive to Americans C) increase U.S. exports and depress Japanese exports. D) increase Japanese exports and depress U.S. exports. Answer: C

Type: A Topic: 4 E: 100-101 MI: 100-101 MA: 100-101 57. Depreciation of the dollar will: A) increase the prices of U.S. imports, but decrease the prices of U.S. exports. B) decrease the prices of U.S. imports, but increase the prices of U.S. exports. C) increase the prices of both U.S. imports and exports. D) decrease the prices of both U.S. imports and exports. Answer: A

McConnell/Brue: Economics, 16/e

Page 184

Chapter 6: The United States in the Global Economy

Type: A Topic: 4 E: 101 MI: 101 MA: 101 58. Appreciation of the Mexican peso will: A) make Mexico's exports and imports both more expensive. B) make Mexico's exports more expensive and its imports less expensive. C) make Mexico's exports less expensive and its imports more expensive. D) increase Mexican exports. Answer: B

Type: A Topic: 4 E: 100-101 MI: 100-101 MA: 100-101 59. All else equal, depreciation of the Mexican peso relative to the U.S. dollar would make a trip by: A) an American to Mexico more expensive. B) a Mexican to the United States less expensive. C) an American to Mexico less expensive. D) an Australian to the United States more expensive. Answer: C

Type: A Topic: 4 E: 101 MI: 101 MA: 101 60. If the Japanese yen appreciates relative to the Swedish krona, then the krona: A) will be more expensive to the Japanese. B) may either appreciate or depreciate relative to the yen. C) will appreciate relative to the yen. D) will depreciate relative to the yen. Answer: D

Type: A Topic: 4 E: 100 MI: 100 MA: 100 61. Other things equal, Canadian imports of U.S. goods: A) create a supply of Canadian dollars in the foreign exchange market. B) create a supply of U.S. dollars in the foreign exchange market. C) reduce the demand for U.S. dollars. D) have no effect on the U.S. dollar price of Canadian dollars. Answer: A

Type: A Topic: 4 E: 100-101 MI: 100-101 MA: 100-101 62. If yesterday $1 would buy 800 South Korean won, but today $1 will only buy 790 won; the: A) dollar has appreciated in value. B) dollar has depreciated in value. C) demand for dollars in the foreign exchange market has increased relative to the supply of won. D) won price of dollars has gone up. Answer: B

McConnell/Brue: Economics, 16/e

Page 185

Chapter 6: The United States in the Global Economy

Use the following to answer questions 63-65: Answer the next question(s) on the basis of the following table which indicates the dollar price of luta, the currency used in the hypothetical economy of Luteland:

Quantity of luta demanded (billions) 200 400 600 800

Dollar price of luta $10 8 6 2

Quantity of luta supplied (billions) 650 400 200 150

Type: T Topic: 4 E: 100 MI: 100 MA: 100 63. Refer to the above table. The equilibrium dollar price of luta is: A) $10. B) $8. C) $6. D) $2. Answer: B

Type: T Topic: 4 E: 100 MI: 100 MA: 100 64. Refer to the above table. The exchange rate in this market is: A) 8 luta for one dollar. C) 6 luta for one dollar. B) .60 luta for one dollar. D) .125 luta for one dollar. Answer: D

Type: T Topic: 4 E: 100-101 MI: 100-101 MA: 100-101 65. Refer to the above table. Suppose that the United States imports more products from Luteland than before. All else equal, the dollar price of luta will: A) rise and the dollar will depreciate. C) rise and the dollar will appreciate. B) fall and the dollar will depreciate. D) fall and the dollar will appreciate. Answer: A

Type: A Topic: 4 E: 99 MI: 99 MA: 99 66. All else equal, U.S. imports from Germany create a: A) demand for German marks. C) demand for American dollars. B) supply of German marks. D) surplus of German marks. Answer: A

Type: A Topic: 4 E: 99 MI: 99 MA: 99 67. All else equal, U.S. exports to Germany create a: A) demand for German marks. B) supply of German marks. Answer: B

C) supply of American dollars. D) shortage of German marks.

Type: A Topic: 4 E: 99 MI: 99 MA: 99 68. If the dollar price of one yen is $.04, a Japanese good priced at 560 yen would cost an American: A) $22.40. B) $2240. C) $14,000. D) $2.40. Answer: A

McConnell/Brue: Economics, 16/e

Page 186

Chapter 6: The United States in the Global Economy

Trade barriers

Type: D Topic: 5 E: 101 MI: 101 MA: 101 69. Protective tariffs are: A) maximum limits on the quantity or total value of specific products imported to a nation. B) excise taxes or duties placed on imported products. C) licensing requirements, unreasonable quality standards, and the like designed to impede imports. D) government payments to domestic producers to reduce the world prices of exported goods. Answer: B

Type: D Topic: 5 E: 102 MI: 102 MA: 102 70. Import quotas are: A) maximum limits on the quantity or total value of specific products imported to a nation. B) excise taxes or duties placed on imported products. C) licensing requirements, unreasonable quality standards, and the like designed to impede imports. D) government payments to domestic producers to reduce the world prices of exported goods. Answer: A

Type: D Topic: 5 E: 102 MI: 102 MA: 102 71. Export subsidies are: A) maximum limits on the quantity or total value of specific products imported to a nation. B) excise taxes or duties placed on imported products. C) licensing requirements, unreasonable quality standards, and the like designed to impede imports. D) government payments to domestic producers to reduce the world prices of exported goods. Answer: D

Type: D Topic: 5 E: 102 MI: 102 MA: 102 72. Nontariff barriers are: A) maximum limits on the quantity or total value of specific products imported to a nation. B) excise taxes or duties placed on imported products. C) licensing requirements, unreasonable quality standards, and the like designed to impede imports. D) government payments to domestic producers to reduce the world prices of exported goods. Answer: C

Type: D Topic: 5 E: 102 MI: 102 MA: 102 73. A nation's true gain from international trade is: A) increased employment in export industries. B) an overall increase in output obtained through specialization and exchange. C) added technological knowledge. D) the tariff revenue that goes to the national treasury. Answer: B

McConnell/Brue: Economics, 16/e

Page 187

Chapter 6: The United States in the Global Economy

Type: F Topic: 5 E: 103 MI: 103 MA: 103 74. The Smoot-Hawley Act: A) bound the world's nations to a gradual process of tariff reduction. B) established very high tariffs on goods imported to the United States. C) exempted American exporters from the Sherman Antitrust Act. D) established the reciprocal trade agreements program. Answer: B

Multilateral agreements: free trade zones; implications of the global economy

Type: F Topic: 6 E: 103-104 MI: 103-104 MA: 103-104 75. The Reciprocal Trade Agreements Act: A) exempted American exporters from the Sherman Antitrust Act. B) provided technological assistance to developing countries. C) brought about considerable reductions in American trade barriers. D) eliminated American subsidies to agricultural exports. Answer: C

Type: D Topic: 6 E: 104 MI: 104 MA: 104 76. The "most-favored-nation" clause of reciprocal trade agreements: A) outlaw tariffs on products for which an exporting nation has a comparative advantage. B) single out a particular nation for exemption from an import quota. C) means that any tariff reductions the United States negotiates with a specific nation will automatically apply to many other nations. D) confers special trade privileges to nations in which the United States has military bases. Answer: C

Type: D Topic: 6 E: 104 MI: 104 MA: 104 77. The General Agreement on Tariffs and Trade (GATT) is based on the principle of: A) establishing a single international currency. B) tariff reductions through multilateral negotiations. C) converting tariffs to import quotas. D) establishing common environmental and labor standards for all member nations. Answer: B

Type: F Topic: 6 E: 104 MI: 104 MA: 104 Status: New 78. The latest, on-going international round of trade negotiations is called the: A) Bangkok Round. B) Doha Round. C) Seattle Round. D) Stockholm Round. Answer: B

Type: F Topic: 6 E: 104 MI: 104 MA: 104 79. The Uruguay Round of GATT negotiations completed in late 1993: A) established a free trade zone between the United States and Mexico. B) made the Russian ruble convertible into other currencies. C) created the European Union (EU). D) created international protections for intellectual property such as patents, copyrights, and trademarks. Answer: D

McConnell/Brue: Economics, 16/e

Page 188

Chapter 6: The United States in the Global Economy

Type: F Topic: 6 E: 104 MI: 104 MA: 104 80. An important outcome of the Uruguay Round of GATT negotiations was: A) a worldwide reduction of agricultural export subsidies. B) establishment of the European Union. C) the elimination of all tariffs and quotas worldwide. D) establishment of the World Bank. Answer: A

Type: F Topic: 6 E: 104 MI: 104 MA: 104 81. The World Trade Organization (WTO): A) sets tariffs to balance international trade among nations. B) is the successor to GATT. C) is better known as the European Union. D) sets exchange rates to balance international trade among nations. Answer: B

Type: F Topic: 6 E: 104 MI: 104 MA: 104 82. The World Trade Organization (WTO) A) sets tariffs to balance international trade among nations. B) is the successor to NAFTA. C) hears and rules on trade disputes between nations. D) sets exchange rates to balance international trade among nations. Answer: C

Type: A Topic: 6 E: 104 MI: 104 MA: 104 Status: New 83. American critics of the WTO argue that free international trade and investment will: A) reduce U.S. imports. B) reduce employment in developing nations. C) undermine environmental and labor protections in the United States. D) increase immigration from low-income to high-income nations. Answer: C

Type: A Topic: 6 E: 104 MI: 104 MA: 104 Status: New 84. Proponents of the WTO argue that free international trade and investment will: A) reduce economic growth rates in the industrial economies. B) reduce employment in developing nations. C) eliminate world poverty. D) increase living standards of all trading nations. Answer: D

Type: D Topic: 6 E: 105 MI: 105 MA: 105 85. A trade bloc is: A) a tariff or quota that impedes imports. B) a group of nations that allows free trade among member nations but restrict imports from nonmember nations via tariffs and quotas. C) an area of a nation where manufacturers can import product components without paying tariffs. D) a group of nations that advertise their common export goods abroad. Answer: B

McConnell/Brue: Economics, 16/e

Page 189

Chapter 6: The United States in the Global Economy

Type: F Topic: 6 E: 104 MI: 104 MA: 104 86. Which of the following nations is not a member of the European Union? A) Switzerland B) France C) Germany D) Italy Answer: A

Type: A Topic: 6 E: 105 MI: 105 MA: 105 87. The primary economic advantage of the European Union (EU) to its members is that: A) the tax structures of each participating nation have been made nearly identical. B) each nation is free to formulate its own antitrust and agricultural policies. C) participating nations use a common currency. D) the reduction of trade barriers permits producers to achieve mass-production economies. Answer: D

Type: A Topic: 6 E: 105 MI: 105 MA: 105 88. The main problem posed by trade blocs for nonmember nations is that: A) member nations may achieve growth rates that exceed those of nonmember nations. B) nonmembers must exchange their currencies for foreign monies before they can engage in export or import transactions. C) nonmembers face tariffs that member nations do not. D) member nations refuse to participate in tariff negotiations sponsored by GATT. Answer: C

Type: F Topic: 6 E: 105 MI: 105 MA: 105 89. The European Union (EU) comprises a group of European nations that have: A) abolished tariffs among one another and established a system of common tariffs with respect to nonmember nations. B) fully integrated their economies by establishing a central bank, a common currency, and a coordinated set of governmental budgetary policies. C) agreed to trade only among one another. D) eliminated all tariffs and trade barriers with nonmember nations. Answer: A

Type: F Topic: 6 E: 105 MI: 105 MA: 105 Status: New 90. The countries comprising NAFTA are: A) Canada, the United States, and Puerto Rico. C) the United States, United Kingdom, and France. B) the United States, Mexico, and Chile. D) Canada, Mexico, and the United States. Answer: D

Type: F Topic: 6 E: 104-105 MI: 104-105 MA: 104-105 Status: New 91. Which of the following sets of countries were among those admitted to the European Union in 2004? A) Spain, Portugal, and Turkey C) Poland, Hungary, and the Czech Republic B) Estonia, Bulgaria, and Switzerland D) Belgium, the Netherlands, and Luxembourg Answer: C

McConnell/Brue: Economics, 16/e

Page 190

Chapter 6: The United States in the Global Economy

Type: F Topic: 6 E: 105 MI: 105 MA: 105 92. The North American Free Trade Agreement (NAFTA): A) resulted from GATT negotiations at the Uruguay Round. B) established a free trade zone encompassing Canada, Mexico, and the United States. C) is also known as the Reciprocal Trade Act. D) permits the former republics of the Soviet Union to export goods duty free to North America. Answer: B

Type: F Topic: 6 E: 105 MI: 105 MA: 105 93. Which of the following has been an outcome of the North American Free Trade Agreement (NAFTA)? A) a lower standard of living in Canada, Mexico, and the United States B) lower wages in the United States and Canada C) increased trade among Canada, Mexico, and the United States D) lower wages and reduce employment in Mexico Answer: C

Type: F Topic: 6 E: 105 MI: 105 MA: 105 Status: New 94. A number of European nations have agreed to use the___________ as a common currency: A) mark. B) pound. C) euro. D) continental. Answer: C

Type: F Topic: 6 E: 105 MI: 105 MA: 105 Status: New 95. Which of these groups of nations are all members of the Euro zone? A) Great Britain, France, and Switzerland C) Denmark, Sweden, and Norway B) France, Germany, and Italy D) Russia, Poland, and Hungary Answer: B

Type: A Topic: 6 E: 105 MI: 105 MA: 105 Status: New 96. Which of the following nations is not a member of the Euro zone? A) Italy B) Spain C) Germany D) Great Britain Answer: D

Type: A Topic: 6 E: 105 MI: 105 MA: 105 97. The nations of the Euro zone have: A) abandoned their national currencies and switched to a common currency. B) abandoned their national currencies and switched to American dollars. C) formed a single country called the Union of European Nations (UEN). D) recently admitted 10 new members. Answer: A

Type: A Topic: 6 E: 105 MI: 105 MA: 105 98. The introduction and use of the euro is expected to: A) increase the rate of inflation slightly in the Euro zone nations. B) increase poverty substantially in Euro zone nations. C) increase international trade among the Euro zone nations. D) increase income inequality within the Euro zone nations. Answer: C

McConnell/Brue: Economics, 16/e

Page 191

Chapter 6: The United States in the Global Economy

Type: D Topic: 6 E: 105 MI: 105 MA: 105 Status: New 99. NAFTA refers to the: A) National Association of Free Trade Agencies. B) National Alliance for Foreign Trade and Assistance. C) North American Free Trade Agreement. D) Northern Alliance For Tariff Adjustment. Answer: C

Type: F Topic: 6 E: 105 MI: 105 MA: 105 100. Critics of the North American Free Trade Agreement (NAFTA) falsely feared that it would: A) increase the flow of illegal Mexican immigrants to the United States. B) cause the Eurpoean Union and Japan to raise trade barriers against U.S. goods. C) cause a massive loss of U.S. jobs to Mexico. D) increase foreign ownership of assets in the United States. Answer: C

Type: A Topic: 6 E: 106 MI: 106 MA: 106 101. Global competition: A) forces domestic producers to become more efficient and to improve product quality. B) drives up prices worldwide. C) reduces employment worldwide. D) creates higher flows of international migration than without trade. Answer: A

Type: A Topic: 6 E: 105 MI: 105 MA: 105 Status: New 102. Since the signing of the North American Free Trade Agreement (NAFTA) in 1993: A) total employment in the United States has slightly declined. B) total employment in the United States has greatly increased. C) Mexican exports to the United States have declined. D) productivity growth in Canada, Mexico, and the United States has stagnated. Answer: B

Consider This Questions

Type: F E: 100 MI: 100 MA: 100 Status: New 103. (Consider This) The relationship between dollars and tickets for rides provides insights on: A) opportunity cost. B) the law of demand. C) exchange rates. D) comparative advantage. Answer: C

Type: A E: 100 MI: 100 MA: 100 Status: New 104. (Consider This) If 10 fair tickets are needed to ride the roller coaster and each fair ticket costs $.50, the dollar-ticket exchange rate is: A) $1 = 2 tickets and the dollar cost of the ride is $5. B) $1 = 1/2 ticket and the dollar cost of the ride is $5. C) 1 ticket = $2 and the dollar cost of the ride is $20. D) $1 = 2 tickets and the dollar cost of the ride is $10. Answer: A

McConnell/Brue: Economics, 16/e

Page 192

Chapter 6: The United States in the Global Economy

Type: A E: 100 MI: 100 MA: 100 Status: New 105. (Consider This) If the price of tickets for fair rides increases from $.50 to $1 and the same number of tickets are required for each ride as before: A) the dollar has appreciated relative to fair tickets. B) the dollar price of the fair rides as declined. C) the dollar has depreciated relative to fair tickets. D) dollar ticket exchange rate has changed from $1 equals ½ ticket to $1 equals 1 ticket. Answer: C

Type: A E: 100 MI: 100 MA: 100 Status: New 106. (Consider This) If the price of tickets for fair rides decreases from $.50 to $.25 and the same number of tickets are required for each ride as before: A) fair tickets have appreciated relative to the dollar. B) the dollar price of the fair rides has declined. C) the dollar price of fair rides has increased. D) the dollar price of fair rides has not changed. Answer: B

Last Word Questions

Type: A E: 106 MI: 106 MA: 106 Status: New 107. (Last Word) In the "Petition of the Candlemakers" the French workers sought protection from which foreign rival? A) the sun B) the moon C) the Americans D) the Japanese Answer: A

Type: A E: 106 MI: 106 MA: 106 Status: New 108. (Last Word) The main point of the "Petition of the Candlemakers" is that: A) a nation must export in order to import. B) trading nations face rising, not constant, opportunity costs. C) rationales for trade barriers are often absurd. D) indusrial policies are doomed to fail. Answer: C

True/False Questions

Type: A E: 91 MI: 91 MA: 91 109. Immigration of workers is an example of a goods and services flow: Answer: False

Type: F E: 94 MI: 94 MA: 94 110. The United States exports a higher U.S. dollar volume of goods to Canada than to any other nation. Answer: True

McConnell/Brue: Economics, 16/e

Page 193

Chapter 6: The United States in the Global Economy

Type: F E: 95 MI: 95 MA: 95 111. The United States, Japan, and the western European nations are the major international traders in terms of overall volume. Answer: True

Type: F E: 96 MI: 96 MA: 96 112. Specialized production and international trade increase a nation's productivity and increase the availability of goods and services. Answer: True

Type: A Topic: 3 E: 97 MI: 97 MA: 97 113. A nation has a comparative advantage in some product when it can produce that good at a lower domestic opportunity cost than can a potential trading partner. Answer: True

Type: D E: 97 MI: 97 MA: 97 114. Terms of trade of 1X=5Y will be acceptable to two countries that have domestic opportunity costs of 1X=4Y and 1X=1Y, respectively. Answer: False

Type: A E: 99 MI: 99 MA: 99 115. Mexican importers are suppliers of pesos in the foreign exchange market. Answer: True

Type: A E: 101 MI: 101 MA: 101 116. When the dollar price of yen rises, the dollar appreciates in value relative to the yen. Answer: False

Type: A E: 102 MI: 102 MA: 102 117. Import quotas are taxes or duties on imported products. Answer: False

Type: D E: 102-103 MI: 102-103 MA: 102-103 118. Barriers to free trade impair efficiency in the international allocation of resources. Answer: True

Type: A E: 104 MI: 104 MA: 104 119. The most-favored-nation clause in reciprocal trade agreements means that any tariff reductions the United States negotiates with a specific nation will automatically apply to many other nations. Answer: True

Type: D E: 104 MI: 104 MA: 104 120. The WTO is comprised of 25 European nations. Answer: False

McConnell/Brue: Economics, 16/e

Page 194

Chapter 6: The United States in the Global Economy

Type: F E: 104 MI: 104 MA: 104 Status: New 121. The current, on-going round of trade negotiations is called the Doha Round. Answer: True

Type: F E: 104-105 MI: 104-105 MA: 104-105 122. The European Union (EU) is a free trade zone comprising all the nations of eastern and western Europe. Answer: False

Type: A E: 105 MI: 105 MA: 105 123. France, Germany, and Italy are all members of the Euro zone. Answer: True

Type: D E: 105 MI: 105 MA: 105 124. NAFTA is an international accord that will eliminate all tariffs and quotas worldwide by the year 2025. Answer: False

McConnell/Brue: Economics, 16/e

Page 195

CHAPTER 7

Measuring Domestic Output, National Income, and the Price Level

Topic 1. 2. 3. 4. 5. 6. 7. 8. GDP concept C, I, G, and Xn components Investment and the capital stock GDP accounting: expenditure approach GDP accounting: value added; income approach Other social accounts Real versus nominal GDP; price indexes GDP and social welfare Consider This Last Word True-False

Question numbers 1-15 16-43 44-57 58-77 78-90 91-105 106-153 154-161 162-165 166-168 169-184

____________________________________________________________

_______________________________________

____________________________________________________________

_______________________________________

Multiple Choice Questions GDP concepts

Type: D Topic: 1 E: 112 MA: 112 1. A nation's gross domestic product (GDP): A) is the dollar value of the total output produced within the borders of the nation. B) is the dollar value of the total output produced by its citizens, regardless of where they are living. C) can be found by summing C + In + S + Xn. D) is always some amount less than its C + Ig + G + Xn. Answer: A

Type: D Topic: 1 E: 117 MA: 117 2. A nation's gross domestic product (GDP): A) can be found by summing C + Ig + G + Xn. B) is the dollar value of the total output produced by its citizens, regardless of where they are living. C) can be found by summing C + S + G + Xn. D) is always some amount less than its NDP. Answer: A

Chapter 7: Measuring Domestic Output, National Income, and the Price Level

Type: D Topic: 1 E: 112 MA: 112 3. The GDP is the: A) monetary value of all final goods and services produced within a nation in a particular year. B) national income minus all nonincome charges against output. C) monetary value of all economic resources used in producing a year's output. D) monetary value of all goods and services, final and intermediate, produced in a specific year. Answer: A

Type: A Topic: 1 E: 113 MA: 113 4. Suppose Smith pays $100 to Jones. A) We can say with certainty that the GDP has increased by $100. B) We can say with certainty that the GDP has increased, but we cannot determine the amount. C) We can say with certainty that the nominal GDP has increased, but we can't say whether real GDP has increased or decreased. D) We need more information to determine whether GDP has changed. Answer: D

Type: A Topic: 1 E: 112 MA: 112 5. Suppose the total market value of all final goods and services produced in a particular country in 2004 is $500 billion and the total market value of final goods and services sold is $450 billion. We can conclude that: A) GDP in 2004 is $450 billion. C) GDP in 2004 is $500 billion. B) NDP in 2004 is $450 billion. D) inventories in 2004 fell by $50 billion. Answer: C

Type: A Topic: 1 E: 113 MA: 113 6. National income accountants can avoid multiple counting by: A) including transfers in their calculations. C) only counting final goods. B) counting both intermediate and final goods. D) only counting intermediate goods. Answer: C

Type: D Topic: 1 E: 113 MA: 113 7. Gross domestic product (GDP) measures and reports output: A) as an index number. B) in percentage terms. C) in dollar amounts. D) in quantities of physical units (for example, pounds, gallons, and bushels). Answer: C

Type: D Topic: 1 E: 112 MA: 112 8. GDP is: A) the monetary value of all goods and services (final, intermediate, and non-market) produced in a given year. B) total resource income less taxes, saving, and spending on exports. C) the economic value of all economic resources used in the production of a year's output. D) the market value of all final goods and services produced within a nation in a specific year. Answer: D

McConnell/Brue: Economics, 16/e

Page 198

Chapter 7: Measuring Domestic Output, National Income, and the Price Level

Type: A Topic: 1 E: 113 MA: 113 9. By summing the dollar value of all market transactions in the economy we would: A) be determining the market value of all resources used in the production process. B) obtain a sum substantially larger than the GDP. C) be determining value added for the economy. D) be measuring GDP. Answer: B

Type: D Topic: 1 E: 113 MA: 113 10. Final goods and services refer to: A) goods and services that are unsold and therefore added to inventories. B) goods and services whose value has been adjusted for changes in the price level. C) goods and services purchased by ultimate users, rather than for resale or further processing. D) the excess of U.S. exports over U.S. imports. Answer: C

Type: A Topic: 1 E: 113 MA: 113 11. If intermediate goods and services were included in GDP: A) the GDP would then have to be deflated for changes in the price level. B) nominal GDP would exceed real GDP. C) the GDP would be overstated. D) the GDP would be understated. Answer: C

Type: A Topic: 1 E: 113 MA: 113 12. Which of the following is a final good or service? A) diesel fuel bought for a delivery truck B) fertilizer purchased by a farm supplier C) a haircut D) Chevrolet windows purchased by a General Motors assembly plant Answer: C

Type: A Topic: 1 E: 113 MA: 113 13. Which of the following is an intermediate good? A) the purchase of gasoline for a ski trip to Colorado B) the purchase of a pizza by a college student. C) the purchase of baseball uniforms by a professional baseball team. D) the purchase of jogging shoes by a professor Answer: C

Type: A Topic: 1 E: 113 MA: 113 14. Tom Atoe grows tomatoes for home consumption. This activity is: A) excluded from GDP in order to avoid double counting. B) excluded from GDP because an intermediate good is involved. C) productive but is excluded from GDP because no market transaction occurs. D) included in GDP because it reflects production. Answer: C

McConnell/Brue: Economics, 16/e

Page 199

Chapter 7: Measuring Domestic Output, National Income, and the Price Level

Type: D Topic: 1 E: 113 MA: 113 15. GDP includes: A) neither intermediate nor final goods. B) both intermediate and final goods. Answer: D

C) intermediate, but not final, goods. D) final, but not intermediate, goods.

C, I, G, and Xn components

Type: A Topic: 2 E: 114 MA: 114 16. GDP can be calculated by summing: A) consumption, investment, government purchases, exports, and imports. B) investment, government purchases, consumption, and net exports. C) consumption, investment, wages, and rents. D) consumption, investment, government purchases, and imports. Answer: B

Type: D Topic: 2 E: 115 MA: 115 17. In national income accounting, consumption expenditures include purchases of: A) both new and used consumer goods. B) automobiles for personal use, but not houses. C) consumer durable and nondurable goods, but not services. D) consumer nondurable goods and services, but not consumer durable goods. Answer: B

Type: D Topic: 2 E: 115 MA: 115 18. In national income accounting, consumption expenditures include: A) purchases of both new and used consumer goods. B) consumer durable goods and consumer nondurable goods, but not services. C) consumer durable goods, consumer nondurable goods, and services. D) changes in business inventories. Answer: C

Type: D Topic: 2 E: 117 MA: 117 19. Net exports are: A) that portion of consumption and investment goods sent to other countries. B) exports plus imports. C) exports less imports. D) imports less exports. Answer: C

Type: D Topic: 2 E: 115-116 MA: 115-116 20. Gross investment refers to: A) private investment minus public investment. B) net investment plus replacement investment. C) net investment after it has been "inflated" for changes in the price level. D) net investment plus net exports. Answer: B

McConnell/Brue: Economics, 16/e

Page 200

Chapter 7: Measuring Domestic Output, National Income, and the Price Level

Type: A Topic: 2 E: 117 MA: 117 21. Net exports are negative when: A) a nation's imports exceed its exports. B) the economy's stock of capital goods is declining. C) depreciation exceeds domestic investment. D) a nation's exports exceed its imports. Answer: A

Type: A Topic: 2 E: 116 MA: 116 22. Which of the following is not economic investment? A) the purchase of a drill press by the Ajax Manufacturing Company B) the purchase of 100 shares of AT&T by a retired business executive C) construction of a suburban housing project D) the piling up of inventories on a grocer's shelf Answer: B

Type: A Topic: 2 E: 115 MA: 115 23. Which of the following do national income accountants consider to be investment? A) the purchase of an automobile for private, nonbusiness use B) the purchase of a new house C) the purchase of corporate bonds D) the purchase of gold coins Answer: B

Type: D Topic: 2 E: 117 MA: 117 24. GDP is equal to: A) C + Ig + G + Xn. B) C + Ig + G - Xn. Answer: A

C) C + In + G + Xn.

D) C + In + G - Xn.

Type: D Topic: 2 E: 115 MA: 115 25. National income accountants define investment to include: A) any increase in business inventories. B) the addition of cash to a savings account. C) the purchase of common or preferred stock. D) the purchase of any durable good, for example, an automobile or a refrigerator. Answer: A

Type: D Topic: 2 E: 115 MA: 115 26. As defined in national income accounting, investment includes: A) business expenditures on machinery and equipment. B) all consumption. C) imports, but not exports. D) all nonfood items. Answer: A

McConnell/Brue: Economics, 16/e

Page 201

Chapter 7: Measuring Domestic Output, National Income, and the Price Level

Type: A Topic: 2 E: 115 MA: 115 27. Suppose that inventories were $40 billion in 2003 and $50 billion in 2004. In 2004, accountants would: A) add $10 billion to other elements of investment in calculating total investment. B) subtract $10 billion from other elements of investments in calculating total investment. C) add $45 billion (= $90/2) to other elements of investment in calculating total investment. D) subtract $45 billion (= $90/2) from other elements of investment in calculating total investment. Answer: A

Type: A Topic: 2 E: 115 MA: 115 28. Suppose that inventories were $80 billion in 2003 and $70 billion in 2004. In 2004, accountants would: A) add $10 billion to other elements of investment in calculating total investment. B) subtract $10 billion from other elements of investments in calculating total investment. C) add $45 billion (= $90/2) to other elements of investment in calculating total investment. D) subtract $45 billion (= $90/2) from other elements of investment in calculating total investment. Answer: B

Type: C Topic: 2 E: 115-116 MA: 115-116 29. Suppose that GDP was $200 billion in year 1 and that all other components of expenditures remained the same in year 2 except that business inventories increased by $10 billion. GDP in year 2 is: A) $180 billion. B) $190 billion. C) $200 billion. D) $210 billion. Answer: D

Type: C Topic: 2 E: 115 MA: 115 30. Suppose that GDP was $200 billion in year 1 and that all other components of expenditures remained the same in year 2 except that business inventories fell by $10 billion. GDP in year 2 is: A) $180 billion. B) $190 billion. C) $200 billion. D) $210 billion. Answer: B

Type: A Topic: 2 E: 115 MA: 115 31. If the economy adds to its inventory of goods during some year: A) gross investment will exceed net investment by the amount of the inventory increase. B) this amount should be ignored in calculating that year's GDP. C) this amount should be subtracted in calculating that year's GDP. D) this amount should be included in calculating that year's GDP. Answer: D

Type: F Topic: 2 E: 117 MA: 117 32. The smallest component of aggregate spending in the United States is: A) net exports. B) government purchases. C) investment. D) consumption. Answer: A

Type: A Topic: 2 E: 114 MA: 114 33. In calculating GDP, governmental transfer payments, such as social security or unemployment compensation, are: A) not counted. C) counted as government spending. B) counted as investment spending. D) counted as consumption spending. Answer: A

McConnell/Brue: Economics, 16/e

Page 202

Chapter 7: Measuring Domestic Output, National Income, and the Price Level

Type: F Topic: 2 E: 117 MA: 117 34. The largest component of total expenditures in the United States is: A) net exports. B) government purchases. C) consumption. D) gross investment. Answer: C

Type: D Topic: 2 E: 116 MA: 116 35. Government purchases include government spending on: A) government consumption goods and public capital goods. B) government consumption goods only. C) public capital goods only. D) government consumption goods, public capital goods, and transfer payments. Answer: A

Type: D Topic: 2 E: 116 MA: 116 36. In national income accounting, government purchases include: A) purchases by Federal, state, and local governments . B) purchases by the Federal government only. C) government transfer payments. D) purchases of goods for consumption, but not public capital goods. Answer: A

Type: A Topic: 2 E: 114 MA: 114 37. Transfer payments are: A) excluded when calculating GDP because they only reflect inflation. B) excluded when calculating GDP because they do not reflect current production. C) included when calculating GDP because they are a category of investment spending. D) included when calculating GDP because they increase the spending of recipients. Answer: B

Type: A Topic: 2 E: 117 MA: 117 38. The value of U.S. imports is: A) added to exports when calculating GDP because imports reflect spending by Americans. B) subtracted from exports when calculating GDP because imports do not constitute spending by Americans. C) subtracted from exports when calculating GDP because imports do not constitute production in the United States. D) added when calculating GDP because imports do not constitute production in the United States. Answer: C

Type: A Topic: 2 E: 117 MA: 117 39. In the treatment of U.S. exports and imports, national income accountants: A) subtract exports, but add imports, in calculating GDP. B) subtract both exports and imports in calculating GDP. C) add both exports and imports in calculating GDP. D) add exports, but subtract imports, in calculating GDP. Answer: D

McConnell/Brue: Economics, 16/e

Page 203

Chapter 7: Measuring Domestic Output, National Income, and the Price Level

Type: A Topic: 2 E: 115-116 MA: 115-116 40. In calculating the GDP national income accountants: A) treat inventory changes as an adjustment to personal consumption expenditures. B) ignore inventories because they do not represent final goods. C) subtract increases in inventories or add decreases in inventories. D) add increases in inventories or subtract decreases in inventories. Answer: D

Type: A Topic: 2 E: 115 MA: 115 41. The ZZZ Corporation issued $25 million in new common stock in 2004. It used $18 million of the proceeds to replace obsolete equipment in its factory and $7 million to repay bank loans. As a result, investment: A) of $7 million has occurred. C) of $18 million has occurred. B) of $25 million has occurred. D) has not occurred. Answer: C

Type: A Topic: 2 E: 115 MA: 115 42. In 2003 Trailblazer Bicycle Company produced a mountain bike that was delivered to a retail outlet in November of 2003. The bicycle was sold to E.Z. Ryder in March of 2004. This bicycle is counted as: A) consumption in 2003 and as disinvestment in 2004. B) disinvestment in 2003 and as consumption in 2004. C) disinvestment in 2003 and as investment in 2004. D) investment in 2003 and as disinvestment in 2004. Answer: D

Type: D Topic: 2 E: 116 MA: 116 43. In national income accounting, G stands for: A) government purchases. B) gross investment. C) government transfer payments. D) gross saving. Answer: A

Investment and the capital stock

Type: A Topic: 3 E: 120 MA: 120 44. GDP differs from NDP in that: A) GDP is based on gross exports, while NDP is based on net exports. B) GDP includes, but NDP excludes, indirect business taxes. C) net investment is used in calculating GDP and gross investment is used in calculating NDP. D) gross investment is used in calculating GDP and net investment is used in calculating NDP. Answer: D

Type: A Topic: 3 E: 120 MA: 120 45. If depreciation exceeds gross investment: A) the economy's stock of capital may be either growing or shrinking. B) the economy's stock of capital is shrinking. C) the economy's stock of capital is growing. D) net investment is zero. Answer: B

McConnell/Brue: Economics, 16/e

Page 204

Chapter 7: Measuring Domestic Output, National Income, and the Price Level

Type: D Topic: 3 E: 120 MA: 120 46. The concept of net domestic investment refers to: A) the amount of machinery and equipment used up in producing the GDP in a specific year. B) the difference between the market value and book value of outstanding capital stock. C) gross domestic investment less net exports. D) total investment less the amount of investment goods used up in producing the year's output. Answer: D

Type: A Topic: 3 E: 120 MA: 120 47. If depreciation (consumption of fixed capital) exceeds domestic investment, we can conclude that: A) nominal GDP is rising but real GDP is declining. B) net investment is negative. C) the economy is importing more than it exports. D) the economy's production capacity is expanding. Answer: B

Type: D Topic: 3 E: 120 MA: 120 48. When an economy's production capacity is expanding: A) nominal GDP, but not necessarily real GDP, is rising. B) net exports is always a positive amount. C) DI exceeds PI. D) domestic investment exceeds depreciation. Answer: D

Use the following to answer questions 49-50: Economy A: gross investment equals depreciation Economy B: depreciation exceeds gross investment Economy C: gross investment exceeds depreciation

Type: A Topic: 3 E: 120 MA: 120 49. Refer to the above information. Positive net investment is occurring in: A) economy A only. B) economy B only. C) economy C only. D) economies A and B only. Answer: C

Type: A Topic: 3 E: 120 MA: 120 50. Other things equal, the above information suggests that the production capacity in economy: A) B is growing more rapidly than either A or C. C) A is growing less rapidly than economy B. B) A is growing more rapidly than either B or C. D) C is growing more rapidly than economy B. Answer: D

Type: A Topic: 3 E: 120 MA: 120 51. In 1933 net private domestic investment was a minus $6.0 billion. This means that: A) gross private domestic investment exceeded depreciation by $6.0 billion. B) the economy was expanding in that year. C) the production of 1933's GDP used up more capital goods than were produced in that year. D) the economy produced no capital goods at all in 1933. Answer: C

McConnell/Brue: Economics, 16/e

Page 205

Chapter 7: Measuring Domestic Output, National Income, and the Price Level

Type: A Topic: 3 E: 120 MA: 120 52. An economy is enlarging its stock of capital goods: A) when net investment exceeds gross investment. B) when gross investment exceeds replacement investment. C) whenever gross investment is positive. D) when replacement investment exceeds gross investment. Answer: B

Type: D Topic: 3 E: 120 MA: 120 53. A nation's stock of capital goods will decline when: A) gross investment exceeds net investment. B) net investment is positive, but less than gross investment. C) depreciation exceeds gross investment. D) gross investment exceeds depreciation. Answer: C

Type: D Topic: 3 E: 120 MA: 120 54. In an economy experiencing a declining production capacity: A) the nation's stock of capital goods is expanding. C) depreciation exceeds gross investment. B) net exports are necessarily zero. D) NDP exceeds GDP. Answer: C

Type: A Topic: 3 E: 120 MA: 120 55. If in some year gross investment was $120 billion and net investment was $65 billion, then in that year the country's capital stock: A) may have either increased or decreased. C) increased by $55 billion. B) increased by $65 billion. D) decreased by $55 billion. Answer: B

Type: D Topic: 3 E: 123-124 MA: 123-124 56. Consumption of fixed capital (depreciation) can be determined by: A) adding indirect business taxes to NDP. C) subtracting net investment from GDP. B) subtracting NDP from GDP. D) adding net investment to gross investment. Answer: B

Type: C Topic: 3 E: 113 MA: 113 57. GDP excludes: A) the market value of unpaid work in the home. B) the production of services. Answer: A

C) the production of nondurable goods. D) positive changes in inventories.

McConnell/Brue: Economics, 16/e

Page 206

Chapter 7: Measuring Domestic Output, National Income, and the Price Level

GDP accounting: expenditure approach

Use the following to answer questions 58-59: Answer the next question(s) on the basis of the following data. All figures are in billions of dollars:
Government purchases Consumption Gross investment Consumption of fixed capital Exports Imports $15 90 20 5 8 12

Type: T Topic: 4 E: 117 MA: 117 Status: New 58. Refer to the above data. GDP is: A) $116. B) $121. C) $125. D) $150. Answer: B

Type: T Topic: 4 E: 120 MA: 120 Status: New 59. Refer to the above data. NDP is: A) $116. B) $121. C) $125. D) $150. Answer: A

Use the following to answer questions 60-64: Answer the next question(s) on the basis of the following data. All figures are in billions of dollars.

Personal taxes Social Security contributions Indirect business taxes Corporate income taxes Transfer payments U.S. exports Undistributed corporate profits Government purchases Gross private domestic investment U.S. imports Personal consumption expenditures Consumption of fixed capital Net foreign factor income earned in the U.S.

$ 40 15 20 40 22 24 35 90 75 22 250 25 10

Type: T Topic: 4 E: 115-118 MA: 115-118 60. Refer to the above data. GDP is: A) $390. B) $417. C) $422. D) $492. E) $512. Answer: B

Type: T Topic: 4 E: 120 MA: 120 61. Refer to the above data. NDP is: A) $370. B) $402. C) $392. D) $467. Answer: C

McConnell/Brue: Economics, 16/e

Page 207

Chapter 7: Measuring Domestic Output, National Income, and the Price Level

Type: T Topic: 4 E: 120 MA: 120 62. Refer to the above data. NI is: A) $364. B) $372. C) $447. D) $362. Answer: D

Type: T Topic: 4 E: 120 MA: 120 63. Refer to the above data. PI is: A) $294. B) $346. C) $408. D) $437. Answer: A

Type: T Topic: 4 E: 121 MA: 121 64. Refer to the above data. DI is: A) $284. B) $329. C) $254. D) $402. Answer: C

Use the following to answer questions 65-68: Answer the next question(s) on the basis of the following data. All figures are in billions of dollars.
Gross investment National income Net exports Personal income Personal consumption expenditures Saving Government purchases Net domestic product $ 18 100 2 85 70 5 20 105

Type: T Topic: 4 E: 115-118 MA: 115-118 65. The gross domestic product for the above economy is: A) $100. B) $95. C) $110. D) $107. Answer: C

Type: T Topic: 4 E: 119 MA: 119 66. Refer to the above data. Consumption of fixed capital is: A) $5. B) $10. C) $20. D) $30. Answer: A

Type: T Topic: 4 E: 121 MA: 121 67. Refer to the above data. Disposable income is: A) $83. B) $73. C) $75. D) $77. Answer: C

Type: T Topic: 4 E: 119 MA: 119 68. Refer to the above data. From this information we can conclude that the sum of indirect business taxes and net foreign factor income is. A) $5 billion. B) zero. C) $1 billion. D) $15 billion. Answer: A

McConnell/Brue: Economics, 16/e

Page 208

Chapter 7: Measuring Domestic Output, National Income, and the Price Level

Use the following to answer questions 69-72: Answer the next question(s) on the basis of the following national income data for the economy. All figures are in billions.

Personal consumption expenditures Government purchases Gross private domestic investment Net exports Net foreign factor income earned in the U.S. Consumption of fixed capital Indirect business taxes Compensation of employees Rents Interest Proprietors' income Corporate income taxes Dividends Undistributed corporate profits

$400 128 88 7 0 43 50 369 12 15 52 36 24 22

Type: T Topic: 4 E: 117 MA: 117 69. The gross domestic product for the above economy is: A) $584. B) $592. C) $609. D) $623. Answer: D

Type: T Topic: 4 E: 120 MA: 120 70. Refer to the above data. Net domestic product is: A) $520. B) $580. C) $623. D) $573. Answer: B

Type: T Topic: 4 E: 120 MA: 120 71. Refer to the above data. The national income is: A) $561. B) $573. C) $580. D) $530. Answer: D

Type: T Topic: 4 E: 121 MA: 121 72. Refer to the above data. Disposable income: A) cannot be determined from the data given. Answer: A

B) is $484.

C) is $416.

D) is $502.

McConnell/Brue: Economics, 16/e

Page 209

Chapter 7: Measuring Domestic Output, National Income, and the Price Level

Use the following to answer questions 73-77: Answer the next question(s) on the basis of the following data. All figures are in billions of dollars.
Gross private domestic investment Exports of the U.S. Disposable income Personal saving Government purchases Net foreign factor income earned in the U.S. Consumption of fixed capital Dividends Imports of the U.S. Indirect business taxes Personal taxes Social Security contributions $ 46 9 190 10 84 10 52 13 12 22 38 23

Type: T Topic: 4 E: 117 MA: 117 73. Refer to the above data. The gross domestic product is: A) $326. B) $282. C) $307. D) $300. Answer: C

Type: T Topic: 4 E: 120 MA: 120 74. Refer to the above data. The net domestic product is: A) $233. B) $255. C) $230. D) $348. Answer: B

Type: T Topic: 4 E: 120 MA: 120 75. Refer to the above data. The national income is: A) $223. B) $249. C) $208. D) $346. Answer: A

Type: T Topic: 4 E: 120 MA: 120 76. Refer to the above data. Personal income is: A) $184. B) $221. C) $149. D) $208. Answer: D

Type: T Topic: 4 E: 116 MA: 116 77. The economy characterized by the above data is: A) experiencing inflation because disposable income exceeds personal income. B) experiencing declining production capacity because net investment is negative. C) in a depression because personal income exceeds disposable income. D) experiencing expanding production capacity because net private domestic investment is positive. Answer: B

McConnell/Brue: Economics, 16/e

Page 210

Chapter 7: Measuring Domestic Output, National Income, and the Price Level

GDP accounting: value added; income approach

Use the following to answer questions 78-80: Answer the next question(s) on the basis of the following data. All figures are in billions of dollars.

Proprietors’ income Compensation of employees Consumption of fixed capital Gross investment Rents Interest Exports Imports Corporate profits Indirect business taxes Net foreign factor income

$ 20 300 15 80 10 20 30 50 25 5 0

Type: T Topic: 5 E: 120 MA: 120 Status: New 78. Refer to the above data. National income is: A) $395. B) $380. C) $375. D) $360. Answer: C

Type: T Topic: 5 E: 117 MA: 117 Status: New 79. Refer to the above data. Gross domestic product is: A) $395. B) $380. C) $375. D) $360. Answer: A

Type: T Topic: 5 E: 117 MA: 117 Status: New 80. Refer to the above data. Net domestic product is: A) $395. B) $380. C) $375. D) $360. Answer: B

Type: D Topic: 5 E: 113 MA: 113 81. Value added refers to: A) any increase in GDP that has been adjusted for adverse environmental effects. B) the excess of gross investment over net investment. C) the difference between the value of a firm's output and the value of the inputs it has purchased from others. D) the portion of any increase in GDP that is caused by inflation as opposed to an increase in real output. Answer: C

Type: A Topic: 5 E: 113 MA: 113 82. Assume a manufacturer of stereo speakers purchases $40 worth of components for each speaker. The completed speaker sells for $70. The value added by the manufacturer for each speaker is: A) $110. B) $30. C) $40. D) $70. Answer: B

McConnell/Brue: Economics, 16/e

Page 211

Chapter 7: Measuring Domestic Output, National Income, and the Price Level

Type: A Topic: 5 E: 113 MA: 113 83. Setup Corporation buys $100,000 of sand, rock, and cement to produce redi-mix concrete. It sells 10,000 cubic yards of concrete at $30 a cubic yard. The value added by Setup Corporation is: A) $300,000. B) $100,000. C) $200,000. D) zero dollars. Answer: C

Type: D Topic: 5 E: 113 MA: 113 84. By summing the values added at each stage in the production of some good we obtain: A) the price of that good. B) the total income generated by that good's production. C) the total cost (including profits) of that product. D) all of the above. Answer: D

Type: D Topic: 5 E: 113 MA: 113 85. Value added can be determined by: A) summing the profits of all enterprises in the economy. B) subtracting the purchase of intermediate products from the value of the sales of final products. C) calculating the year-to-year changes in real GDP. D) deflating nominal GDP. Answer: B

Use the following to answer questions 86-90: Answer the next question(s) on the basis of the following national income data. All figures are in billions of dollars.
Personal taxes Net private domestic investment Net exports National income U.S. exports Gross private domestic investment Disposable income Indirect business taxes Undistributed corporate profits Proprietors' income Net foreign factor income earned in the U.S. $ 23 33 6 278 20 56 220 32 15 45 0

Type: T Topic: 5 E: 116 MA: 116 86. Refer to the above data. Consumption of fixed capital (private sector) is: A) $23. B) $14. C) $32. D) $26. Answer: A

Type: T Topic: 5 E: 117 MA: 117 87. Refer to the above data. U.S. imports are: A) $26. B) $16. C) $24. D) $14. Answer: D

McConnell/Brue: Economics, 16/e

Page 212

Chapter 7: Measuring Domestic Output, National Income, and the Price Level

Type: T Topic: 5 E: 115 MA: 115 88. Refer to the above data. Personal consumption expenditures: A) cannot be calculated. B) are $231. C) are $225. D) are $205. Answer: A

Type: T Topic: 5 E: 118-119 MA: 118-119 89. Refer to the above data. The gross domestic product is: A) $328. B) $402. C) $382. D) $333. Answer: D

Type: T Topic: 5 E: 120 MA: 120 90. Refer to the above data. Personal income is: A) $229. B) $253. C) $274. D) $243. Answer: D

Other social accounts

Type: A Topic: 6 E: 119 MA: 119 91. If net foreign factor income earned in the U.S. is zero, the sum of national income, indirect business taxes, and the consumption of fixed capital equals: A) disposable income. B) personal income. C) net domestic product. D) gross domestic product. Answer: D

Type: A Topic: 6 E: 120 MA: 120 92. NDP is: A) NI plus net foreign factor income earned in the U.S. plus indirect business taxes. B) NI plus corporate income taxes. C) GDP deflated for increases in the price level. D) GDP minus indirect business taxes. Answer: A

Type: D Topic: 6 E: 120 MA: 120 93. Which of the following best defines national income? A) income received by households less personal taxes B) the before-tax income received by households C) all incomes earned by U.S. resource suppliers for their current contributions to production D) the market value of the annual output net of consumption of fixed capital Answer: C

Type: D Topic: 6 E: 120 MA: 120 94. The total amount of income earned by U.S. resource suppliers in a year is measured by: A) gross domestic product. B) national income. C) personal income. D) disposable income. Answer: B

McConnell/Brue: Economics, 16/e

Page 213

Chapter 7: Measuring Domestic Output, National Income, and the Price Level

Type: D Topic: 6 E: 121 MA: 121 95. The largest component of national income is: A) compensation of employees. B) rents. C) interest. Answer: A

D) corporate profits.

Type: D Topic: 6 E: 120 MA: 120 96. The total income earned in any year by U.S. resource suppliers is measured by: A) DI. B) NI. C) PI. D) GDP. Answer: B

Type: D Topic: 6 E: 120 MA: 120 97. National income measures: A) nominal GDP after it has been inflated or deflated for changes in the value of the dollar. B) the after-tax income of resource suppliers. C) the market value or cost of the resources used in the production of the national output. D) the amount of wage, rent, interest, and profits income actually received by households. Answer: C

Type: A Topic: 6 E: 120 MA: 120 98. Personal income is most likely to exceed national income: A) when gross and net investment are equal. C) when gross investment exceeds net investment. B) during a period of recession or depression. D) during a period of extended inflation. Answer: B

Type: A Topic: 6 E: 120 MA: 120 99. If personal income exceeds national income in a particular year, we can conclude that: A) transfer payments exceeded the sum of Social Security contributions, corporate income taxes, and indirect business taxes. B) the sum of Social Security contributions, corporate income taxes, and undistributed corporate profits exceeded transfer payments. C) consumption of fixed capital and indirect business taxes exceeded personal taxes. D) transfer payments exceeded the sum of Social Security contributions, corporate income taxes, and undistributed corporate profits. Answer: D

Type: D Topic: 6 E: 121 MA: 121 100. Which of the following best defines disposable income? A) income received by households less personal taxes B) the before-tax income received by households C) all income earned by resource suppliers for their current contributions to production D) the market value of the annual output net of consumption of fixed capital Answer: A

Type: F Topic: 6 E: 121 MA: 121 101. Which of the following is the largest dollar amount in the United States? A) disposable income B) personal income C) gross domestic product Answer: C

D) national income

McConnell/Brue: Economics, 16/e

Page 214

Chapter 7: Measuring Domestic Output, National Income, and the Price Level

Type: F Topic: 6 E: 121 MA: 121 102. Which of the following is the smallest dollar amount in the United States? A) disposable income B) personal income C) gross domestic product Answer: A

D) national income

Type: D Topic: 6 E: 120 MA: 120 103. Transfer payments are included in: A) NI. B) PI. C) GDP. D) NDP. Answer: B

Type: D Topic: 6 E: 121 MA: 121 104. The amount of after-tax income received by households is measured by: A) discretionary income. B) national income. C) disposable income. Answer: C

D) personal income.

Type: A Topic: 6 E: 121 MA: 121 105. In a typical year which of the following measures of aggregate output and income is likely to be the smallest? A) gross domestic product B) national income C) disposable income D) personal income Answer: C

Real versus nominal GDP; price indexes

Type: D Topic: 7 E: 123 MA: 123 106. Nominal GDP is: A) the sum of all monetary transactions that occur in the economy in a year. B) the sum of all monetary transactions involving final goods and services that occur in the economy in a year. C) the amount of production that occurs when the economy is operating at full employment. D) money GDP adjusted for inflation. Answer: B

Type: D Topic: 7 E: 123 MA: 123 107. Real GDP refers to: A) the value of the domestic output after adjustments have been made for environmental pollution and changes in the distribution of income. B) GDP data that embody changes in the price level, but not changes in physical output. C) GDP data that reflect changes in both physical output and the price level. D) GDP data that have been adjusted for changes in the price level. Answer: D

Type: D Topic: 7 E: 123 MA: 123 108. Real GDP measures: A) current output at current prices. B) current output at base year prices. Answer: B

C) base year output at current prices. D) base year output at current exchange rates.

McConnell/Brue: Economics, 16/e

Page 215

Chapter 7: Measuring Domestic Output, National Income, and the Price Level

Type: D Topic: 7 E: 123 MA: 123 109. Nominal GDP is adjusted for price changes through the use of: A) the Consumer Price Index (CPI). C) the GDP price index. B) the Producer Price Index (PPI). D) exchange rates. Answer: C

Type: A Topic: 7 E: 123 MA: 123 Status: New 110. In the second quarter (3-month period) of 2001, U.S. nominal GDP increased but U.S. real GDP declined. We can conclude that: A) nominal income declined by more than personal income. B) the price level rose by more than nominal GDP. C) real wages declined by more than real GDP. D) the price level fell by more than real GDP. Answer: B

Type: D Topic: 7 E: 123 MA: 123 111. A price index is: A) a comparison of the price of a market basket from a fixed point of reference. B) a comparison of real GDP in one period relative to another. C) the cost of a market basket of goods and services in a base period divided by the cost of the same market basket in another period. D) a ratio of real GDP to nominal GDP. Answer: A

Type: D Topic: 7 E: 123 MA: 123 112. The GDP price index: A) includes fewer goods and services than the consumer price index. B) is identical to the consumer price index. C) is another term for the producer price index. D) includes all goods comprising the nation's domestic output. Answer: D

Type: A Topic: 7 E: 123 MA: 123 113. If real GDP falls from one period to another, we can conclude that: A) deflation occurred. C) nominal GDP fell. B) inflation occurred. D) none of the above necessarily occurred. Answer: D

McConnell/Brue: Economics, 16/e

Page 216

Chapter 7: Measuring Domestic Output, National Income, and the Price Level

Use the following to answer questions 114-117: Use the following table for a hypothetical single-product economy.
Units of Output 10 12 15 20 Price of bagel per unit $10 20 30 40 Price index (year 1 = 100) 100 200 300 400

Year 1 2 3 4

Type: T Topic: 7 E: 123 MA: 123 114. Refer to the above data. Nominal GDP in year 3 is: A) $100. B) $450. C) $225. D) $150. Answer: B

Type: T Topic: 7 E: 123 MA: 123 115. Refer to the above data. Real GDP in year 3 is: A) $100. B) $450. C) $225. D) $150. Answer: D

Type: T Topic: 7 E: 123 MA: 123 116. Refer to the above data. Nominal GDP in year 4 is: A) $320. B) $450. C) $225. D) $800. Answer: D

Type: T Topic: 7 E: 123 MA: 123 117. Refer to the above data. Real GDP in year 4 is: A) $320. B) $450. C) $200. D) $800. Answer: C

Type: A Topic: 7 E: 123 MA: 123 118. If real GDP in a particular year is $80 billion and nominal GDP is $240 billion, the GDP price index for that year is: A) 100. B) 200. C) 240. D) 300. Answer: D

Type: A Topic: 7 E: 124 MA: 124 119. Suppose a nation's 2003 nominal GDP was $972 billion and the general price index was 90. To make the 2003 GDP comparable with the base year GDP, the 2003 GDP must be: A) adjusted downward to $678 billion. C) inflated to $1080 billion. B) deflated to $896 billion. D) deflated to $1080 billion. Answer: C

McConnell/Brue: Economics, 16/e

Page 217

Chapter 7: Measuring Domestic Output, National Income, and the Price Level

Type: C Topic: 7 E: 124 MA: 124 120. Suppose nominal GDP in 2002 was $100 billion and in 2003 it was $260 billion. The general price index in 2002 was 100 and in 2003 it was 180. Between 2002 and 2003 the real GDP rose by: A) 160 percent. B) 44 percent. C) 37 percent. D) 12 percent. Answer: B

Type: F Topic: 7 E: 124 MA: 124 121. Historically, real GDP has increased less rapidly than nominal GDP because: A) price indices have not reflected improvements in product quality. B) the general price level has increased. C) technological progress has resulted in more efficient production. D) the general price level has declined. Answer: B

Type: A Topic: 7 E: 124 MA: 124 122. Suppose nominal GDP was $360 billion in 1990 and $450 billion in 2000. The appropriate price index (1985 = 100) was 120 in 1990 and 125 in 2000. Between 1990 and 2000 real GDP: A) increased by $60 billion. C) increased by $100 billion. B) decreased by $32 billion. D) increased by $117 billion. Answer: A

Use the following to answer questions 123-127: Assume an economy that is producing only one product and that year 3 is the base year. Output and price data for a five-year period are as follows. Answer the next question(s) on the basis of these data.

Year 1 2 3 4 5

Units of output 3 4 6 7 8

Price per unit $3 4 5 7 8

Type: T Topic: 7 E: 123-124 MA: 123-124 123. Refer to the above data. If year 3 is chosen as the base year, the price index for year 1 is: A) 140. B) 40. C) 167. D) 60. Answer: D

Type: T Topic: 7 E: 123 MA: 123 124. Refer to the above data. The nominal GDP for year 4 is: A) $49. B) $55. C) $40. D) $35. Answer: A

Type: T Topic: 7 E: 123-124 MA: 123-124 125. Refer to the above data. Real GDP for year 5 is: A) $160. B) $49. C) $40. D) $64. Answer: C

McConnell/Brue: Economics, 16/e

Page 218

Chapter 7: Measuring Domestic Output, National Income, and the Price Level

Type: T Topic: 7 E: 125 MA: 125 126. Refer to the above data. In determining real GDP, the nominal GDP for: A) each year must be multiplied by the relevant price index. B) years 1 and 2 must be inflated. C) years 4 and 5 must be inflated. D) years 1 and 2 must be deflated. Answer: B

Type: T Topic: 7 E: 125 MA: 125 127. Refer to the above data. For the years shown the growth of: A) real GDP has exceeded the growth of nominal GDP. B) nominal GDP accurately reflects changes in real output. C) nominal GDP overstates increases in real output. D) nominal GDP understates increases in real output. Answer: C

Type: A Topic: 7 E: 125 MA: 125 128. A price index can rise from one year to the next even though: A) some individual prices in the economy fall. C) real GDP falls. B) nominal GDP falls. D) all of the above occur. Answer: D

Type: D Topic: 7 E: 123 MA: 123 129. Real GDP and nominal GDP differ because the real GDP: A) is adjusted for changes in the volume of intermediate transactions. B) includes the economic effects of international trade. C) has been adjusted for changes in the price level. D) excludes depreciation charges. Answer: C

Type: A Topic: 7 E: 124 MA: 124 130. Nominal GDP was $130 and $150 in years 1 and 2 respectively. Real GDP was $100 and $110 in years 1 and 2 respectively. On the basis of this information we can conclude that: A) the price level increased between years 1 and 2. B) more intermediate goods were produced in year 1 than in year 2. C) the increase in nominal GDP between years 1 and 2 understated the increase in production which occurred. D) the price level declined between years 1 and 2. Answer: A

Type: A Topic: 7 E: 124 MA: 124 131. If nominal GDP rises: A) real GDP may either rise or fall. B) we can be certain that the price level has risen. Answer: A

C) real GDP must fall. D) real GDP must also rise.

McConnell/Brue: Economics, 16/e

Page 219

Chapter 7: Measuring Domestic Output, National Income, and the Price Level

Type: D Topic: 7 E: 123 MA: 123 132. Real GDP is: A) the nominal value of all goods and services produced in the economy. B) the nominal value of all goods and services produced in the domestic economy corrected for inflation or deflation. C) that aggregate output that is produced when the economy is operating at full employment. D) always greater than nominal GDP. Answer: B

Type: A Topic: 7 E: 125 MA: 125 133. In comparing GDP data over a period of years, a difference between nominal and real GDP may arise because: A) of changes in trade deficits and surpluses. B) the length of the workweek has declined historically. C) the price level may change over time. D) depreciation may be greater or smaller than gross investment. Answer: C

Use the following to answer questions 134-136: Answer the next question(s) on the basis of the following information: Only three goods are produced in an economy in the following amounts: A = 10, B = 30, C = 5. The current year per unit prices of these three goods are A = $2, B = $3, and C = $1.

Type: A Topic: 7 E: 123 MA: 123 134. Refer to the above information. Nominal GDP in the current year is: A) $110. B) $115. C) $45. D) $90 Answer: B

Type: A Topic: 7 E: 124-125 MA: 124-125 135. (Advanced analysis) Refer to the above information. If the per unit prices of the three goods each were $1 in a base year used to construct a GDP price index, then the GDP price index in the current year is: A) 205.5. B) 255.5. C) 39.3. D) 100. Answer: B

Type: A Topic: 7 E: 124-125 MA: 124-125 136. (Advanced analysis) Refer to the above information. If the per unit prices of the three goods each were $1 in a base year used to construct a GDP price index, then real GDP in the current year is: A) $110. B) $115. C) $45. D) $160. Answer: C

McConnell/Brue: Economics, 16/e

Page 220

Chapter 7: Measuring Domestic Output, National Income, and the Price Level

Use the following to answer questions 137-141: Assume an economy that is producing only one product. Output and price data for a three-year period are as follows. Answer the next question(s) on the basis of these data.
Year 1 2 3 Units of output 20 25 30 Price per unit $ 4 4 6

Type: T Topic: 7 E: 123 MA: 123 137. Refer to the above data. If year 2 is chosen as the base year, the price index for year one is: A) 80. B) 100. C) 120. D) 20. Answer: B

Type: T Topic: 7 E: 123 MA: 123 138. Refer to the above data. The nominal GDP for year 3 is: A) 125 percent higher than the nominal GDP for year 1. B) 50 percent higher than the nominal GDP for year 1. C) $120. D) $30. Answer: A

Type: T Topic: 7 E: 123 MA: 123 139. Refer to the above data. If year 2 is chosen as the base year, real GDP for year 1 is: A) $25. B) $100. C) $20. D) $80. Answer: C

Type: T Topic: 7 E: 123-124 MA: 123-124 140. Refer to the above data. If year 2 is chosen for the base year, in year 3 nominal GDP and real GDP, respectively, are: A) $180 and $30 B) $30 and $5 C) $180 and $120 D) $120 and $100 Answer: C

Type: T Topic: 7 E: 123-124 MA: 123-124 141. Refer to the above data. If year 2 is chosen as the base year, in years 1 and 3 the price index values, respectively, are: A) 4 and 6. B) 6 and 4. C) 120 and 100. D) 100 and 150. Answer: D

McConnell/Brue: Economics, 16/e

Page 221

Chapter 7: Measuring Domestic Output, National Income, and the Price Level

Use the following to answer questions 142-144: Answer the next question(s) on the basis of the following information:
Year 1 2 3 4 5 Nominal GDP $ 550 560 576 586 604 Price index 140 135 120 117 108

Type: T Topic: 7 E: 125 MA: 125 142. The economy above has experienced a: A) a declining nominal GDP. B) a rising price level. Answer: D

C) a declining real GDP.

D) deflation.

Type: T Topic: 7 E: 125 MA: 125 143. In the economy above: A) the price level is rising faster than nominal GDP. B) nominal and real GDP are growing at the same rate. C) the growth of nominal GDP understates the growth of real GDP. D) the growth of nominal GDP overstates the growth of real GDP. Answer: C

Type: T Topic: 7 E: 124 MA: 124 144. In the economy above, real GDP for year 3 is: A) $512. B) $428. C) $480. D) $691. Answer: C

Type: A Topic: 7 E: 125 MA: 125 145. Assume that in 2002 the nominal GDP was $350 billion and in 2003 it was $375 billion. On the basis of this information we: A) cannot make a meaningful comparison of the economy's performance in 2002 relative to 2003. B) can conclude that the economy was achieving real economic growth. C) can conclude that real GDP was higher in 2002 than in 2003. D) can conclude that real GDP was lower in 2002 than in 2003. Answer: A

Type: A Topic: 7 E: 124 MA: 124 146. If nominal GDP in some year is $280 and real GDP is $160. The GDP price index for that year is: A) 175. B) 57. C) 160. D) 280. Answer: A

Type: C Topic: 7 E: 124 MA: 124 147. If real disposable income fell during a particular year, we can conclude that: A) personal taxes increased. C) transfer payments declined. B) inflation occurred. D) none of the above necessarily occurred. Answer: D

McConnell/Brue: Economics, 16/e

Page 222

Chapter 7: Measuring Domestic Output, National Income, and the Price Level

Use the following to answer questions 148-149:

Nominal and real GDP ($)

Nominal GDP Real GDP

0

1996

Time

Type: G Topic: 7 E: 124 MA: 124 148. Refer to the above diagram. The base year used in determining the price indices for this economy: A) cannot be determined from the information given. B) is some year before 1996. C) is more recent than 1996. D) is 1996. Answer: D

Type: G Topic: 7 E: 125 MA: 125 149. Refer to the above diagram. Which of the following statements is correct? A) The price index is greater than 100 for every year shown on the graph. B) Nominal GDP must be deflated in each year prior to 1996 to determine real GDP. C) Real GDP has grown in this economy, but nominal GDP has not. D) Nominal GDP must be deflated in each year since 1996 to determine real GDP. Answer: D

Type: A Topic: 7 E: 125 MA: 125 150. In an economy experiencing persistent deflation: A) potential GDP will necessarily exceed actual GDP. B) changes in nominal GDP may either overstate or understate changes in real GDP. C) changes in nominal GDP understate changes in real GDP. D) changes in nominal GDP overstate changes in real GDP. Answer: C

Type: A Topic: 7 E: 124-125 MA: 124-125 151. If real GDP rises and the GDP price index has increased: A) the percentage increase in nominal GDP must have been less than the percentage increase in the price level. B) nominal GDP may have either increased or decreased. C) nominal GDP must have increased. D) nominal GDP must have fallen. Answer: C

McConnell/Brue: Economics, 16/e

Page 223

Chapter 7: Measuring Domestic Output, National Income, and the Price Level

Type: F Topic: 7 E: 123 MA: 123 152. In determining real GDP economists adjust the nominal GDP by using the: A) national productivity index. C) GDP price index. B) wholesale (producers') price index. D) consumer price index. Answer: C

Type: A Topic: 7 E: 124-125 MA: 124-125 153. The fact that nominal GDP has risen faster than real GDP: A) suggests that the base year of the GDP price index has been shifted. B) tells us nothing about what has happened to the price level. C) suggests that the general price level has fallen. D) suggests that the general price level has risen. Answer: D

GDP and social welfare

Type: F Topic: 8 E: 125 MA: 125 Status: New 154. Which of the following activities is excluded from GDP, causing GDP to understate a nation's well-being? A) the services of used-car dealers B) the child-care services provided by stay-at-home parents C) the construction of new houses D) government expenditures on military equipment Answer: B

Type: F Topic: 8 E: 126 MA: 126 Status: New 155. Which of the following activities is excluded from GDP, causing GDP to understate a nation's well-being? A) the services of health care workers B) the services of military personnel C) the construction of new buildings D) goods and services produced in the underground economy. Answer: D

Type: F Topic: 8 E: 126 MA: 126 Status: New 156. A large underground economy results in an: A) understated GDP. B) overstated GDP. Answer: A

C) understated GDP price index. D) overstated GDP price index.

Type: D Topic: 8 E: 125 MA: 125 157. The GDP tends to: A) overstate economic welfare because it does not include certain nonmarket activities such as the productive work of housewives. B) understate economic welfare because it includes expenditures undertaken to offset or correct pollution. C) understate economic welfare because it does not take into account increases in leisure. D) overstate economic welfare because it does not reflect improvements in product quality. Answer: C

McConnell/Brue: Economics, 16/e

Page 224

Chapter 7: Measuring Domestic Output, National Income, and the Price Level

Type: D Topic: 8 E: 125-126 MA: 125-126 158. The growth of GDP may understate changes in the economy's economic well-being over time if the: A) distribution of income becomes increasingly unequal. B) quality of products and services improves. C) environment deteriorates because of pollution. D) amount of leisure decreases. Answer: B

Type: D Topic: 8 E: 125-126 MA: 125-126 159. GDP data are criticized as being inaccurate measures of economic welfare because: A) they do not take into account changes in the amount of leisure. B) they do not take into account all changes in product quality. C) they do not take into account the adverse effects of economic activity on the environment. D) of all of the above considerations. Answer: D

Type: A Topic: 8 E: 126 MA: 126 160. Environmental pollution is accounted for in: A) GDP. B) PI. C) DI. D) none of the above. Answer: D

Type: A Topic: 8 E: 126 MA: 126 161. Assume that the size of the underground economy increases both absolutely and relatively over time. As a result: A) real GDP will rise more rapidly than nominal GDP. B) GDP will tend to increasingly understate the level of output through time. C) GDP will tend to increasingly overstate the level of output through time. D) the accuracy of GDP will be unaffected through time. Answer: B

Consider This Questions

Type: F E: 117 MA: 117 Status: New 162. (Consider This) In terms of a reservoir analogy, the: A) outflow below the dam is gross investment. B) inflow from the river is the stock of capital. C) level of water in the reservoir is the stock of capital. D) level of water in the reservoir is net investment. Answer: C

Type: F E: 117 MA: 117 Status: New 163. (Consider This) In terms of a reservoir analogy, the: A) outflow below the dam is the stock of capital. C) level of water in the reservoir is depreciation. B) inflow from the river is gross investment. D) level of water in the reservoir is net investment. Answer: B

McConnell/Brue: Economics, 16/e

Page 225

Chapter 7: Measuring Domestic Output, National Income, and the Price Level

Type: A E: 117 MA: 117 Status: New 164. (Consider This) Gross investment is a: A) flow, as is depreciation. B) flow, as is capital. Answer: A

C) stock, whereas depreciation is a flow. D) stock, whereas capital is a flow.

Type: A E: 117 MA: 117 Status: New 165. (Consider This) Capital is a: A) flow, whereas gross investment and depreciation are stocks. B) flow, as are gross investment and depreciation. C) stock, as are gross investment and depreciation. D) stock , whereas gross investment and depreciation are flows. Answer: D

Last Word Questions

Type: F E: 127 MA: 127 166. (Last Word) The U.S. government agency responsible for compiling the national income accounts is the: A) Census Bureau. B) Bureau of Labor Statistics (BLS). C) Commerce Department's Bureau of Economic Analysis (BEA). D) Government Accounting Office (GAO). Answer: C

Type: F E: 127 MA: 127 167. (Last Word) Which of the following is a source of data for the consumption component of the U.S. GDP? A) the Census Bureau's Retail Trade Survey B) the Census Bureau's Survey of Government Finance C) the Conference Board's Index of Leading Indicators D) the Bureau of Labor Statistics Consumer Price Index Answer: A

Type: F E: 127 MA: 127 168. (Last Word) Which of the following is a source of data for the investment component of U.S. GDP? A) the Census Bureau's Retail Trade Survey B) the Census Bureau's Housing Starts Survey and Housing Sales Survey. C) the Conference Board's Survey of Consumer Sentiment D) the Bureau of Labor Statistics Consumer Price Index Answer: B

True/False Questions

Type: D E: 121 MA: 121 169. Disposable income measures the before-tax income received by resource suppliers. Answer: False

McConnell/Brue: Economics, 16/e

Page 226

Chapter 7: Measuring Domestic Output, National Income, and the Price Level

Type: D E: 120 MA: 120 170. NDP can be determined by adding indirect business taxes to GDP. Answer: False

Type: D E: 115-116 MA: 115-116 171. In determining GDP by the expenditures method it is appropriate to use net investment rather than gross investment as a measure of investment spending. Answer: False

Type: A E: 114 MA: 114 172. The expenditures and income approaches to GDP yield identical results because goods that are not sold in one year will be sold in some following year. Answer: False

Type: D E: 120 MA: 120 173. Welfare payments to low-income families are included in national income. Answer: False

Type: C E: 121-122 MA: 121-122 174. Within the circular flow model, the level of total resource income and total spending on output will be approximately equal: Answer: True

Type: D E: 116 MA: 116 175. Interest on the public debt is included as a part of government purchases in determining GDP by the expenditures method. Answer: False

Type: A E: 115 MA: 115 176. The simplest way to calculate GDP is to sum the total sales of all business firms. Answer: False

Type: A E: 115-116 MA: 115-116 177. The purchase of Wal-Mart stock is a part of gross investment, but not of net investment. Answer: False

Type: A E: 120 MA: 120 178. Personal income usually exceeds disposable income. Answer: False

Type: A E: 116 MA: 116 179. Gross private domestic investment exceeds depreciation in an economy that experiences expanding production capacity. Answer: True

McConnell/Brue: Economics, 16/e

Page 227

Chapter 7: Measuring Domestic Output, National Income, and the Price Level

Type: D E: 123-124 MA: 123-124 180. A price index is 100 times the ratio of real GDP to nominal GDP. Answer: False

Type: A E: 124 MA: 124 181. If nominal GDP is 150 and the GDP price index is 200, real GDP is 75. Answer: True

Type: A E: 115-116 MA: 115-116 182. All expenditures on new construction are included as investment in calculating GDP. Answer: True

Type: D E: 117 MA: 117 183. Exports are subtracted from imports in calculating U.S. GDP because exports are not available for domestic consumption. Answer: False

Type: A E: 124 MA: 124 184. If real GDP is 50 and nominal GDP is 100, the GDP price index is 200. Answer: True

McConnell/Brue: Economics, 16/e

Page 228

CHAPTER 8

Introduction to Economic Growth and Instability

Topic 1. 2. 3. 4. 5. 6. 7. 8. Economic growth Business cycle Unemployment GDP gap; Okun's law Inflation: measurement, rule of 70 Demand-pull and cost-push inflation Nominal versus real income Inflation effects Consider This Last Word True-False

Question numbers 1-22 23-35 36-78 79-95 96-105 106-112 113-120 121-133 134-135 136-139 140-154

____________________________________________________________

_______________________________________

____________________________________________________________

_______________________________________

Multiple Choice Questions Economic growth

Type: D Topic: 1 E: 131 MA: 131 1. Economic growth is best defined as an increase in: A) either real GDP or real GDP per capita. B) nominal GDP. Answer: A

C) total consumption expenditures. D) wealth in the economy.

Type: A Topic: 1 E: 131-132 MA: 131-132 2. Real GDP per capita is found by: A) adding real GDP and population. B) subtracting population from real GDP. Answer: C

C) dividing real GDP by population. D) dividing population by real GDP.

Type: A Topic: 1 E: 132 MA: 132 Status: New 3. Real GDP per capita: A) cannot grow more rapidly than real GDP. B) cannot grow more slowly than real GDP. C) necessarily grows more rapidly than real GDP. D) can grow either more slowly or more rapidly than real GDP. Answer: D

Chapter 8: Introduction to Economic Growth and Instability

Type: F Topic: 1 E: 132 MA: 132 Status: New 4. Which of the following best measures improvements in the standard of living of a nation? A) growth of nominal GDP C) growth of real GDP per capita B) growth of real GDP D) growth of national income Answer: C

Type: A Topic: 1 E: 132 MA: 132 Status: New 5. If a nation's real GDP increases from 100 billion to 106 billion and its population jumps from 200 million to 212 million, it real GDP per capita will: A) remain constant. B) fall by 6 percent. C) rise by 6 percent. D) fall by 12 percent. Answer: A

Type: A Topic: 1 E: 132 MA: 132 Status: New 6. For a nation's real GDP per capita to rise during a year: A) consumption spending must increase. B) real GDP must increase more rapidly than population. C) population must increase more rapidly than real GDP. D) investment spending must increase. Answer: B

Type: A Topic: 1 E: 132 MA: 132 7. Growth is advantageous to a nation because it: A) promotes faster population growth. B) lessens the burden of scarcity. Answer: B

C) eliminates the economizing problem. D) slows the growth of wants.

Type: A Topic: 1 E: 132 MA: 132 8. For comparing changes in potential military strength and political preeminence, the most meaningful measure of economic growth would be: A) changes in total nominal output. C) changes in per capita output. B) changes in total real output. D) changes in per family output. Answer: B

Use the following to answer questions 9-12:
Alta (real GDP) $ 2,000 2,100 2,200 Zorn (real GDP) $ 150,000 152,000 154,000 Alta (population) 200 202 210 Zorn (population) 500 505 508

Year 1 2 3

Type: T Topic: 1 E: 132 MA: 132 9. Refer to the above table. Between years 1 and 2, real GDP grew by __________ percent in Alta: A) 3 percent. B) 4 percent. C) 5 percent. D) 10 percent. Answer: C

McConnell/Brue: Economics, 16/e

Page 230

Chapter 8: Introduction to Economic Growth and Instability

Type: T Topic: 1 E: 132 MA: 132 10. Refer to the above table. Between years 1 and 2, real GDP per capita grew by __________ percent in Alta: A) 3 B) 4 C) 5 D) 10 Answer: B

Type: T Topic: 1 E: 132 MA: 132 11. Refer to the above table. Between years 2 and 3: A) Alta's real GDP grew more rapidly than Zorn's real GDP. B) real GDP fell in Zorn. C) population growth reduced Alta's real GDP growth to zero. D) population fell in Alta's. Answer: A

Type: T Topic: 1 E: 132 MA: 132 12. Refer to the above table. Per capita GDP was about: A) $105 in year 3 in Alta. C) $200 in year 1 in Zorn. B) $303 in year 3 in Zorn. D) $5 in year 2 in Alta. Answer: B

Type: D Topic: 1 E: 132 MA: 132 Status: New 13. Given the annual rate of economic growth, the "rule of 70" allows one to: A) determine the accompanying rate of inflation. B) calculate the size of the GDP gap. C) calculate the number of years required for real GDP to double. D) determine the growth rate of per capita GDP. Answer: C

Type: A Topic: 1 E: 132 MA: 132 Status: New 14. The number of years required for real GDP to double can be found by: A) dividing the annual growth rate by .07. C) dividing 70 by the annual growth rate. B) multiplying the annual growth rate by 70. D) adding 14 to annual growth rate. Answer: C

Type: A Topic: 1 E: 132 MA: 132 Status: New 15. At an annual growth rate of 4 percent, real GDP will double in about: A) 17 ½ years. B) 20 years. C) 13 ½ years. D) 15 years. Answer: A

Type: A Topic: 1 E: 132 MA: 132 Status: New 16. At an annual growth rate of 7 percent, real GDP will double in about: A) 11 ½ years. B) 10 years. C) 13 ½ years. D) 9 years. Answer: B

Type: A Topic: 1 E: 132 MA: 132 17. If a nation's real GDP is growing by 5 percent per year, its real GDP will double in approximately: A) 22 years. B) 20 years. C) 14 years. D) 8 years. Answer: C

McConnell/Brue: Economics, 16/e

Page 231

Chapter 8: Introduction to Economic Growth and Instability

Type: A Topic: 1 E: 132 MA: 132 18. If the economy's real GDP doubles in 18 years, we can: A) not say anything about the average annual rate of growth. B) conclude that its average annual rate of growth is about 5.5 percent. C) conclude that its average annual rate of growth is about 2 percent. D) conclude that its average annual rate of growth is about 4 percent. Answer: D

Type: F Topic: 1 E: 132 MA: 132 19. About ________ of U.S. economic growth comes from improved productivity (as opposed to added inputs). A) one-fourth. B) one-third. C) one-half. D) two-thirds. Answer: D

Type: F Topic: 1 E: 133 MA: 133 20. Between 1950 and 2002, U.S. real GDP grew at an average annual rate of about: A) 1.3 percent. B) 3.4 percent. C) 5.1 percent. D) 8.6 percent. Answer: B

Type: F Topic: 1 E: 133 MA: 133 21. Between 1950 and 2002, U.S. real GDP per capita grew at an average annual rate of about: A) 5.5 percent. B) 4.2 percent. C) 3.5 percent. D) 2.1 percent. Answer: D

Type: F Topic: 1 E: 133 MA: 133 22. The United States experienced a: A) higher economic growth rate than Japan between 1950 and 2002, but a lower rate between 1992 and 2002. B) lower economic growth rate than Japan between 1950 and 2002, but a higher rate between 1992 and 2002. C) higher rate of growth of real GDP than of real GDP per capita between 1950 and 2002. D) higher rate of growth of real GDP than of real GDP per capita between 1990 and 2002. Answer: B

Business cycle

Type: D Topic: 2 E: 134 MA: 134 23. Recurring upswings and downswings in an economy's real GDP over time are called: A) recessions. B) business cycles. C) output yo-yos. D) total product oscillations. Answer: B

Type: F Topic: 2 E: 134 MA: 134 24. In the United States, business cycles have occurred against a backdrop of a long-run trend of: A) declining unemployment. C) rising real GDP. B) stagnant productivity growth. D) rising inflation. Answer: C

McConnell/Brue: Economics, 16/e

Page 232

Chapter 8: Introduction to Economic Growth and Instability

Type: F Topic: 2 E: 135 MA: 135 25. The immediate determinant of the volume of output and employment is the: A) composition of consumer spending. B) ratio of public goods to private goods production. C) level of total spending. D) size of the labor force. Answer: C

Type: D Topic: 2 E: 135 MA: 135 26. As it relates to economic growth, the term long-run trend refers to: A) the long-run increase in the relative importance of durable goods in the U.S. economy. B) the long-term expansion or contraction of business activity that occurs over 50 or 100 years. C) fluctuations in business activity that average 40 months in duration. D) fluctuations in business activity that occur around Christmas, Easter, and so forth. Answer: B

Type: A Topic: 2 E: 135 MA: 135 27. In which of the following industries or sectors of the economy is output likely to be most strongly affected by the business cycle? A) military goods B) capital goods C) textile products D) agricultural commodities Answer: B

Type: A Topic: 2 E: 135 MA: 135 Status: New 28. The industries or sectors of the economy in which output is likely to be most strongly affected by the business cycle are: A) military goods and capital goods. C) clothing and education. B) services and nondurable consumer goods. D) capital goods and durable consumer goods. Answer: D

Type: A Topic: 2 E: 135 MA: 135 29. During a severe recession, we would expect output to fall the most in: A) the health-care industry. B) the clothing industry. C) agriculture. Answer: D

D) the construction industry.

Type: D Topic: 2 E: 134 MA: 134 30. The phase of the business cycle in which real GDP declines is called: A) the peak. B) a recovery. C) a recession. D) the trough. Answer: C

Type: D Topic: 2 E: 134 MA: 134 31. The phase of the business cycle in which real GDP is at a minimum is called: A) the peak. B) a recession. C) the trough. D) the pits. Answer: C

McConnell/Brue: Economics, 16/e

Page 233

Chapter 8: Introduction to Economic Growth and Instability

Type: F Topic: 2 E: 133-134 MA: 133-134 32. Market economies have been characterized by: A) occasional instability of employment and price levels. B) uninterrupted economic growth. C) persistent full employment. D) declining populations. Answer: A

Type: A Topic: 2 E: 135 MA: 135 33. The production of durable goods varies more than the production of nondurable goods because: A) durables purchases are nonpostponable. B) durables purchases are postponable. C) the producers of nondurables have monopoly power. D) producers of durables are highly competitive. Answer: B

Type: D Topic: 2 E: 134 MA: 134 34. A recession is a period in which: A) cost-push inflation is present. B) nominal domestic output falls. Answer: D

C) demand-pull inflation is present. D) real domestic output falls.

Type: F Topic: 2 E: 134 MA: 134 Status: New 35. The main cause of the 2001 recession in the United States was a large decline in: A) investment spending. B) government spending. C) net exports. D) interest rates. Answer: A

Unemployment

Use the following to answer questions 36-38: Answer the next question(s) on the basis of the following information about the hypothetical economy of Bralone. All figures are in millions.

Not in the labor force Unemployed Total population Employed Discouraged workers

45 7 145 95 3

Type: A Topic: 3 E: 136 MA: 136 Status: New 36. Refer to the above information. The labor force in Bralone is: A) 95 million. B) 102 million. C) 105 million. D) 145 million. Answer: B

McConnell/Brue: Economics, 16/e

Page 234

Chapter 8: Introduction to Economic Growth and Instability

Type: A Topic: 3 E: 136 MA: 136 Status: New 37. Refer to the above information. The unemployment rate in Bralone is: A) 2.5 percent. B) 3.2 percent. C) 5.0 percent. D) 6.9 percent. Answer: D

Type: C Topic: 3 E: 136 MA: 136 Status: New 38. Refer to the above information. If the natural rate of unemployment in Bralone is 5 percent, then: A) structural unemployment is 3 percent. C) cyclical unemployment is 2 percent. B) frictional unemployment is 2 percent. D) hidden unemployment is 5 percent. Answer: C

Type: F Topic: 3 E: 137-138 MA: 137-138 39. The United States' economy is considered to be at full employment when: A) 90 percent of the total population is employed. B) 90 percent of the labor force is employed. C) about 4-5 percent of the labor force is unemployed. D) 100 percent of the labor force is employed. Answer: C

Type: F Topic: 3 E: 138, 140 MA: 138, 140 40. In the United States, the rate of unemployment is highest for: A) white teenagers. B) black teenagers. C) married women. D) unmarried women. Answer: B

Type: A Topic: 3 E: 136-137 MA: 136-137 41. Kimberly voluntarily quit her job as an insurance agent to return to school full-time to earn an MBA degree. With degree in hand she is now searching for a position in management. Kimberly presently is: A) cyclically unemployed. C) frictionally unemployed. B) structurally unemployed. D) not a member of the labor force. Answer: C

Type: D Topic: 3 E: 136 MA: 136 42. To be officially unemployed a person must: A) be in the labor force. B) be 21 years of age or older. Answer: A

C) have just lost a job. D) be waiting to be called back from a layoff.

Type: D Topic: 3 E: 137 MA: 137 43. The natural rate of unemployment is: A) higher than the full-employment rate of unemployment. B) lower than the full-employment rate of unemployment. C) that rate of unemployment occurring when the economy is at its potential output. D) found by dividing total unemployment by the size of the labor force. Answer: C

McConnell/Brue: Economics, 16/e

Page 235

Chapter 8: Introduction to Economic Growth and Instability

Type: D Topic: 3 E: 136 MA: 136 44. The labor force includes: A) employed workers and persons who are officially unemployed. B) employed workers, but excludes persons who are officially unemployed. C) full-time workers, but excludes part-time workers. D) permanent employees, but excludes temporary employees. Answer: A

Type: F Topic: 3 E: 140 MA: 140 45. The unemployment rate of: A) women greatly exceeds that of men. B) whites is roughly equal to that of blacks. C) white-collar workers exceeds that of blue-collar workers. D) teenagers is much higher than that of adults. Answer: D

Type: A Topic: 3 E: 136 MA: 136 46. Anne Kasperson works in her own home as a full-time caretaker and homemaker. Officially, she is: A) unemployed. B) employed. C) not in the labor force. D) in the labor force. Answer: C

Type: A Topic: 3 E: 137-138 MA: 137-138 47. If the unemployment rate is 9 percent and the natural rate of unemployment is 5 percent, then the: A) frictional unemployment rate is 5 percent. B) cyclical unemployment rate and the frictional unemployment rate together are 5 percent. C) cyclical unemployment rate is 4 percent. D) natural rate of unemployment will eventually increase. Answer: C

Type: A Topic: 3 E: 136 MA: 136 48. Official unemployment statistics: A) understate unemployment because individuals receiving unemployment compensation are counted as employed. B) understate unemployment because discouraged workers are not counted as unemployed. C) include cyclical and structural unemployment, but not frictional unemployment. D) overstate unemployment because workers who are involuntarily working part time are counted as being employed. Answer: B

Type: A Topic: 3 E: 136 MA: 136 49. The presence of discouraged workers: A) increases the size of the labor force, but does not affect the unemployment rate. B) reduces the size of the labor force, but does not affect the unemployment rate. C) may cause the official unemployment rate to understate the amount of unemployment. D) may cause the official unemployment rate to overstate the amount of unemployment. Answer: C

McConnell/Brue: Economics, 16/e

Page 236

Chapter 8: Introduction to Economic Growth and Instability

Type: A Topic: 3 E: 136 MA: 136 50. Part-time workers are counted as: A) unemployed and therefore the official unemployment rate may overstate the level of unemployment. B) unemployed and therefore the official unemployment rate may understate the level of unemployment. C) fully employed and therefore the official unemployment rate may overstate the level of unemployment. D) fully employed and therefore the official unemployment rate may understate the level of unemployment. Answer: D

Type: F Topic: 3 E: 138 MA: 138 51. The natural rate of unemployment: A) is fixed over time. B) is found by adding the cyclical and structural unemployment rates. C) may change from one decade to another. D) cannot be changed through public policy. Answer: C

Type: A Topic: 3 E: 136 MA: 136 52. Assuming the total population is 100 million, the civilian labor force is 50 million, and 47 million workers are unemployed, the unemployment rate is: A) 3 percent. B) 6 percent. C) 7 percent. D) 9 percent. Answer: B

Type: D Topic: 3 E: 137 MA: 137 53. The natural rate of unemployment is the: A) unemployment rate experienced at the depth of a depression. B) full-employment unemployment rate. C) unemployment rate experienced by the least-skilled workers in the economy. D) unemployment rate experienced by the most-skilled workers in the economy. Answer: B

Type: A Topic: 3 E: 137 MA: 137 54. Assume that Hernandez is temporarily unemployed because he has voluntarily quit his job with company A and will begin a better job next week with company B. Hernandez will be considered as: A) cyclically unemployed. B) frictionally unemployed. C) secularly unemployed. D) employed. Answer: B

Type: F Topic: 3 E: 140 MA: 140 55. During periods of full employment the: A) burden of unemployment is quite evenly distributed among males and females, blacks and whites, and young and old workers. B) unemployment rate for teenagers is below the rate for the labor force as a whole. C) unemployment rate for women is considerably lower than that for men. D) unemployment rate for blacks is about twice the rate for whites. Answer: D

McConnell/Brue: Economics, 16/e

Page 237

Chapter 8: Introduction to Economic Growth and Instability

Type: D Topic: 3 E: 136 MA: 136 56. The unemployment rate is the: A) ratio of unemployed to employed workers. B) number of employed workers minus the number of workers who are not in the labor force. C) percentage of the labor force that is out of work. D) percentage of the total population that is out of work. Answer: C

Type: F Topic: 3 E: 138 MA: 138 57. The full-employment unemployment rate means an unemployment rate of about: A) 15 percent. B) 10 percent. C) 4-5 percent. D) 2 percent. Answer: C

Type: D Topic: 3 E: 136 MA: 136 58. The official unemployment rate: A) involves people over 16 years of age who are not currently seeking employment. B) is the ratio of unemployed to employed workers. C) is the percentage of the labor force that is unemployed. D) is the percentage of the total population that is not working. Answer: C

Type: A Topic: 3 E: 136 MA: 136 59. Suppose there are 5 million unemployed workers seeking jobs. After a period of time, 1 million of them become discouraged over their job prospects and cease to look for work. As a result of this, the official unemployment rate would: A) increase in the short run but eventually decline. B) increase. C) decline. D) be unchanged. Answer: C

Type: A Topic: 3 E: 137 MA: 137 60. Eckstein has lost her job in a Massachusetts textile plant because of import competition. She intends to take a short course in electronics and move to California where she anticipates that a new job will be available. We can say that Eckstein is faced with: A) secular unemployment. C) structural unemployment. B) cyclical unemployment. D) frictional unemployment. Answer: C

Type: D Topic: 3 E: 137 MA: 137 61. Cyclical unemployment results from: A) a deficiency of aggregate spending. B) the decreasing relative importance of goods and the increasing relative importance of services in the U.S. economy. C) the everyday dynamics of a free labor market. D) technological change. Answer: A

McConnell/Brue: Economics, 16/e

Page 238

Chapter 8: Introduction to Economic Growth and Instability

Type: F Topic: 3 E: 140 MA: 140 62. Which of the following is correct? A) The unemployment rates of men and women workers are roughly the same. B) Unemployment rates for black and white workers are approximately the same. C) Teenagers experience approximately the same unemployment rates as do adults. D) Laborers are less vulnerable to unemployment than are professional workers. Answer: A

Type: A Topic: 3 E: 136-137 MA: 136-137 63. A college graduate using the summer following graduation to search for a job would best be classified as: A) not officially a member of the labor force. C) a part of cyclical unemployment. B) a part of structural unemployment. D) a part of frictional unemployment. Answer: D

Type: D Topic: 3 E: 137 MA: 137 64. Unemployment involving a mismatch of the skills of unemployed workers and the skills required for available jobs is called: A) frictional unemployment. C) cyclical unemployment. B) structural unemployment. D) compositional unemployment. Answer: B

Type: D Topic: 3 E: 137 MA: 137 65. Structural unemployment: A) is also known as frictional unemployment. B) is the main component of cyclical unemployment. C) is said to occur when people are waiting to be called back to previous jobs. D) may involve a locational mismatch between unemployed workers and job openings. Answer: D

Type: A Topic: 3 E: 136 MA: 136 66. Dr. Homer Simpson, an economics professor, decided to take a year off from teaching to run a commercial fishing boat in Alaska. That year, Professor Simpson would be officially counted as: A) structurally unemployed. B) frictionally unemployed. C) not in the labor force. D) employed. Answer: D

Type: D Topic: 3 E: 138 MA: 138 67. When the U.S. economy has achieved full employment, the unemployment rate is between: A) 5 and 6 percent. B) 4 and 5 percent. C) 3 and 4 percent. D) 2 and 3 percent. Answer: B

Type: D Topic: 3 E: 137 MA: 137 68. Which of the following constitute the unemployment occurring at the natural rate of unemployment? A) frictional and cyclical unemployment B) structural and frictional unemployment C) cyclical and structural unemployment D) frictional, structural, and cyclical unemployment. Answer: B

McConnell/Brue: Economics, 16/e

Page 239

Chapter 8: Introduction to Economic Growth and Instability

Type: A Topic: 3 E: 137 MA: 137 69. Wait unemployment and search unemployment are both types of: A) cyclical unemployment. C) frictional unemployment. B) hidden unemployment. D) structural unemployment. Answer: C

Type: A Topic: 3 E: 137 MA: 137 70. The type of unemployment associated with recessions is called: A) frictional unemployment. C) cyclical unemployment. B) structural unemployment. D) seasonal unemployment. Answer: C

Type: A Topic: 3 E: 136 MA: 136 71. Suppose there are 10 million part-time workers and 90 million full-time workers in an economy. Five million of the part-time workers switch to full-time work. As a result: A) the official unemployment rate will fall. B) the official unemployment rate will rise. C) the official unemployment rate will remain unchanged. D) the size of the labor force will increase. Answer: C

Type: D Topic: 3 E: 137 MA: 137 72. Cyclical unemployment is also called: A) wait unemployment. B) search unemployment. Answer: D

C) seasonal unemployment. D) deficient-demand unemployment.

Type: D Topic: 3 E: 137 MA: 137 73. At the economy's natural rate of unemployment: A) the economy achieves its potential output. B) there is only a relatively small amount of cyclical unemployment. C) only frictional unemployment exists. D) only structural unemployment exists. Answer: A

Type: A Topic: 3 E: 137 MA: 137 74. Search unemployment and wait unemployment are types of: A) frictional unemployment. C) deficient-demand unemployment. B) structural unemployment. D) cyclical unemployment. Answer: A

Type: F Topic: 3 E: 137 MA: 137 75. In the depth of the Great Depression, the unemployment rate in the United States was about: A) 15 percent. B) 33 percent. C) 25 percent. D) 40 percent. Answer: C

McConnell/Brue: Economics, 16/e

Page 240

Chapter 8: Introduction to Economic Growth and Instability

Use the following to answer questions 76-78: Answer the next question(s) on the basis of the following information about a hypothetical economy: Full-time employed = 80 Part-time employed = 25 Unemployed = 15 Discouraged workers = 5 Members of underground economy = 6 Consumer Price Index = 110

Type: A Topic: 3 E: 136 MA: 136 76. Refer to the above information. The unemployment rate is: A) 18.8 percent. B) 12.5 percent. C) 16.7 percent. D) 25 percent. Answer: B

Type: C Topic: 3 E: 137 MA: 137 77. Refer to the above information. If the members of the underground economy are presently counted as part of the unemployed when in fact they are employed, the official unemployment rate is overstated by: A) 0 percentage points. B) 2 percentage points. C) 5 percentage points. D) 6 percentage points. Answer: C

Type: A Topic: 3 E: 141 MA: 141 78. Refer to the above information. The rate of inflation: A) is 110 percent. B) is 10 percent. C) is 0 percent. Answer: D

D) cannot be determined from the data.

GDP gap; Okun's law

Type: D Topic: 4 E: 138 MA: 138 Status: New 79. The GDP gap measures the difference between: A) NDP and GDP. C) actual GDP and potential GDP. B) NI and PI. D) nominal GDP and real GDP. Answer: C

Type: A Topic: 4 E: 138 MA: 138 80. A large negative GDP gap implies: A) an excess of imports over exports. B) a low rate of unemployment. Answer: C

C) a high rate of unemployment. D) a sharply rising price level.

Type: A Topic: 4 E: 138 MA: 138 81. The aggregate cost of unemployment can be measured by the: A) amount by which actual GDP exceeds potential GDP. B) amount by which potential GDP exceeds actual GDP. C) excess of real GDP over nominal GDP. D) excess of nominal GDP over real GDP. Answer: B

McConnell/Brue: Economics, 16/e

Page 241

Chapter 8: Introduction to Economic Growth and Instability

Type: A Topic: 4 E: 138 MA: 138 82. If the U.S. unemployment rate is 9 percent, we can infer that: A) the economy is in the expansion phase of the business cycle. B) potential GDP is in excess of actual GDP. C) actual GDP is in excess of potential GDP. D) actual GDP is equal to potential GDP. Answer: B

Type: A Topic: 4 E: 138 MA: 138 Status: New 83. If actual GDP is $500 billion and there is a negative GDP gap of $10 billion, potential GDP is: A) $510 billion. B) $490 billion. C) $10 billion. D) $990 billion. Answer: A

Type: A Topic: 4 E: 138 MA: 138 Status: New 84. If actual GDP is $340 billion and there is a positive GDP gap of $20 billion, potential GDP is: A) $360 billion. B) $660 billion. C) $320 billion. D) $20 billion. Answer: C

Type: A Topic: 4 E: 138 MA: 138 Status: New 85. If potential GDP is $330 billion and there is a positive GDP gap of $30 billion, real GDP is: A) $300 billion. B) $30 billion. C) $360 billion. D) $630 billion. Answer: C

Type: A Topic: 4 E: 138 MA: 138 Status: New 86. If potential GDP is $400 billion and there is a negative GDP gap of $15 billion, real GDP is: A) $415 billion. B) $385 billion. C) $15 billion. D) $785 billion. Answer: B

Type: A Topic: 4 E: 138 MA: 138 87. Assume the natural rate of unemployment in the U.S. economy is 5 percent and the actual rate of unemployment is 9 percent. According to Okun's law, the negative GDP gap as a percent of potential GDP is: A) 4 percent. B) 8 percent. C) 10 percent. D) 2 percent. Answer: B

Type: D Topic: 4 E: 138 MA: 138 88. The relationship between the size of the negative GDP gap and the unemployment rate is: A) direct. B) inverse. C) undefined. D) direct during recession, but inverse during expansion. Answer: A

Type: A Topic: 4 E: 138 MA: 138 89. If actual GDP is less than potential GDP: A) potential GDP will fall. B) the price level will rise. C) investment spending will fall. D) the actual unemployment rate will be higher than the natural unemployment rate. Answer: D

McConnell/Brue: Economics, 16/e

Page 242

Chapter 8: Introduction to Economic Growth and Instability

Type: D Topic: 4 E: 138 MA: 138 90. Full-employment output is also called: A) zero-unemployment output. B) equilibrium output. Answer: C

C) potential output.

D) zero-savings output.

Use the following to answer questions 91-93: Answer the next question(s) on the basis of the following information for a specific year in a hypothetical economy for which Okun's law is applicable: Potential Real GDP = $200 billion Natural rate of unemployment = 6 percent Actual rate of unemployment = 12 percent

Type: A Topic: 4 E: 138 MA: 138 91. The size of the negative GDP gap as a percent of potential GDP for the above economy is: A) 9 percent. B) 12 percent. C) 15 percent. D) 18 percent. Answer: B

Type: C Topic: 4 E: 138 MA: 138 92. The amount of output being forgone by the above economy is: A) $12 billion. B) $15 billion. C) $18 billion. D) $24 billion. Answer: D

Type: A Topic: 4 E: 138 MA: 138 93. If the unemployment rate in the above economy declined to 6 percent, we could conclude that: A) only structural unemployment remained. B) the economy's production possibilities curve shifted outward. C) the economy had moved from a point inside its production possibilities curve to a point on or near the curve. D) nominal GDP would rise but real GDP would fall. Answer: C

Type: D Topic: 4 E: 138 MA: 138 94. Okun's law: A) measures the tradeoff between the rate of inflation and the rate of unemployment. B) indicates the number of years it will take for a constant rate of inflation to double the price level. C) quantifies the relationship between nominal and real incomes. D) shows the relationship between the unemployment rate and the size of the negative GDP gap. Answer: D

Type: D Topic: 4 E: 138 MA: 138 95. For every 1 percentage point that the actual unemployment rate exceeds the natural rate, a 2 percentage point negative GDP gap occurs. This is a statement of: A) Taylor's rule. B) Okun's law. C) Say's law. D) the Coase theorem. Answer: B

McConnell/Brue: Economics, 16/e

Page 243

Chapter 8: Introduction to Economic Growth and Instability

Inflation: measurement, rule of 70

Type: D Topic: 5 E: 141 MA: 141 96. Inflation means that: A) all prices are rising, but at different rates. B) all prices are rising and at the same rate. C) prices in the aggregate are rising, although some particular prices may be falling. D) real incomes are rising. Answer: C

Type: A Topic: 5 E: 141 MA: 141 97. If the consumer price index falls from 120 to 116 in a particular year, the economy has experienced: A) inflation of 4 percent. C) deflation of 3.33 percent. B) inflation of 3.33 percent. D) deflation of 4 percent. Answer: C

Type: A Topic: 5 E: 141 MA: 141 98. The consumer price index was 177.1 in 2001 and 179.9 in 2002. Therefore, the rate of inflation in 2002 was about: A) 6.7 percent. B) 3.4 percent. C) 1.6 percent. D) 4.1 percent. Answer: C

Type: A Topic: 5 E: 141 MA: 141 99. The rate of inflation can be found by subtracting: A) the real income from the nominal income. B) last year's price index from this year's price index. C) this year's price index from last year's price index and dividing the difference by this year's price index. D) last year's price index from this year's price index and dividing the difference by last year's price index. Answer: D

Type: A Topic: 5 E: 141 MA: 141 100. If the Consumer Price index rises from 300 to 333 in a particular year, the rate of inflation in that year is: A) 11 percent. B) 33 percent. C) 91 percent. D) 10 percent. Answer: A

Type: D Topic: 5 E: 142 MA: 142 101. As applied to the price level, the "rule of 70" indicates that the number of years required for the price to double can be found by: A) dividing "70" into the annual rate of inflation. B) dividing the annual rate of inflation into "70." C) subtracting the annual change in nominal incomes from "70." D) multiplying the annual rate of inflation by "70." Answer: B

McConnell/Brue: Economics, 16/e

Page 244

Chapter 8: Introduction to Economic Growth and Instability

Type: C Topic: 5 E: 142 MA: 142 102. Between 1980 and 2000 the price level approximately doubled. The average annual rate of inflation over this 20-year period was about: A) 5.5 percent. B) 4.7 percent. C) 3.5 percent. D) 2.8 percent. Answer: C

Type: D Topic: 5 E: 141-142 MA: 141-142 103. Given the annual rate of inflation, the "rule of 70" allows one to: A) determine whether the inflation is demand-pull or cost-push. B) calculate the accompanying rate of unemployment. C) determine when the value of a real asset will approach zero. D) calculate the number of years required for the price level to double. Answer: D

Type: C Topic: 5 E: 141-142 MA: 141-142 104. If Ernie's annual real income rises by 8 percent each year, his annual real income will double in about: A) 8-9 years. B) 10-11 years. C) 5-6 years. D) 19-20 years. Answer: A

Type: A Topic: 5 E: 141-142 MA: 141-142 105. If the rate of inflation is 12 percent per year, the price level will double in about: A) 4 years. B) 6 years. C) 10 years. D) 12 years. Answer: B

Demand-pull and cost-push inflation

Type: D Topic: 6 E: 142 MA: 142 106. Demand-pull inflation: A) occurs when prices of resources rise, pushing up costs and the price level. B) occurs when total spending exceeds the economy's ability to provide output at the existing price level. C) occurs only when the economy has reached its absolute production capacity. D) is also called cost-push inflation. Answer: B

Type: A Topic: 6 E: 142 MA: 142 107. Demand-pull inflation: A) occurs when total spending in the economy is excessive. B) is measured differently than cost-push inflation. C) can be present even during an economic depression. D) is also called "hyperinflation." Answer: A

Type: D Topic: 6 E: 142 MA: 142 108. "Too much money chasing too few goods" best describes: A) the GDP gap. B) demand-pull inflation. C) the inflation premium. Answer: B

D) cost-push inflation.

McConnell/Brue: Economics, 16/e

Page 245

Chapter 8: Introduction to Economic Growth and Instability

Type: A Topic: 6 E: 143 MA: 143 109. Unlike demand-pull inflation, cost-push inflation: A) is self-limiting. B) drives up the price level. Answer: A

C) increases nominal income. D) increases real income.

Type: D Topic: 6 E: 143 MA: 143 110. Inflation initiated by increases in wages or other resource prices is labeled: A) demand-pull inflation. B) demand-push inflation. C) cost-push inflation. Answer: C

D) cost-pull inflation.

Type: A Topic: 6 E: 143 MA: 143 111. Cost-push inflation: A) is caused by excessive total spending. B) shifts the nation's production possibilities curve leftward. C) moves the economy inward from its production possibilities curve. D) is a mixed blessing because it has positive effects on real output and employment. Answer: C

Type: A Topic: 6 E: 143 MA: 143 112. Cost-push inflation may be caused by: A) a decline in per unit production costs. B) a decrease in wage rates. Answer: C

C) a negative supply shock. D) an increase in resource availability.

Nominal versus real income

Type: F Topic: 7 E: 144 MA: 144 Status: New 113. Real income is found by: A) dividing nominal income by 70. B) multiplying nominal income by 1.03. C) dividing the price index (in hundredths) by nominal income. D) dividing nominal income by the price index (in hundredths). Answer: D

Type: D Topic: 7 E: 144 MA: 144 114. Which of the following formulas is correct? Percentage change in: A) price level approximates percentage change in real income minus percentage change in nominal income. B) real income approximates percentage change in nominal income minus percentage change in price level. C) nominal income approximates percentage change in price level minus percentage change in real income. D) real income approximates percentage change in price level minus percentage change in nominal income. Answer: B

McConnell/Brue: Economics, 16/e

Page 246

Chapter 8: Introduction to Economic Growth and Instability

Type: A Topic: 7 E: 144 MA: 144 115. Real income can be determined by: A) dividing the price level by nominal income. B) inflating nominal income for inflation. C) dividing the annual rate of inflation into the number "70." D) deflating nominal income for inflation. Answer: D

Type: A Topic: 7 E: 144 MA: 144 116. Suppose that a person's nominal income rises from $10,000 to $12,000 and the consumer price index rises from 100 to 105. The person's real income will: A) fall by about 20 percent. C) rise by about 15 percent. B) fall by about 2 percent. D) rise by about 25 percent. Answer: C

Type: A Topic: 7 E: 144 MA: 144 117. Recently a labor union argued that the standard of living of its members was falling. A critic of the union argued that this could not possibly be true because the union had been acquiring increases in the nominal incomes of its members through collective bargaining. Is the critic correct? A) Yes, because when you have a large nominal income your standard of living automatically increases. B) No, because real income may fall if price increases are more proportionately than the increase in nominal income. C) No, because real income may fall if price increases are less proportionately than the increases in nominal income. D) Yes, because real income may fall if price increases are less proportionately than the increases in nominal income. Answer: B

Type: A Topic: 7 E: 144 MA: 144 118. Suppose that a person's nominal income rises by 5 percent and the price level rises from 125 to 130. The person's real income will: A) fall by about 1 percent. C) rise by about 4 percent. B) remain constant. D) rise by about 1 percent. Answer: D

Type: A Topic: 7 E: 144 MA: 144 119. In 2002 Ortega's nominal income rose by 4.6 percent and the price level rose by 1.6 percent. We can conclude that Ortega's real income: A) may have either increased or decreased. C) rose by 3 percent. B) rose by 6.2 percent. D) fell by 13 percent. Answer: C

Type: A Topic: 7 E: 144 MA: 144 120. Which of the following is correct? A) Real and nominal incomes always move in the same direction. B) Inflation increases the purchasing power of the dollar and necessarily reduces one's nominal income. C) Inflation reduces the purchasing power of the dollar and necessarily reduces one's real income. D) Inflation reduces the purchasing power of the dollar, but does not necessarily reduce one's real income. Answer: D

McConnell/Brue: Economics, 16/e

Page 247

Chapter 8: Introduction to Economic Growth and Instability

Inflation effects

Type: F Topic: 8 E: 143 MA: 143 Status: New 121. Cost-push inflation: A) reduces real output. C) reduces the unemployment rate. B) increases real output. D) raises the natural rate of unemployment. Answer: A

Type: F Topic: 8 E: 143 MA: 143 Status: New 122. Mild demand-pull inflation: A) raises the natural rate of unemployment. C) has an uncertain effect on real output. B) usually leads to hyperinflation.. D) decreases the natural rate of unemployment. Answer: C

Type: A Topic: 8 E: 145 MA: 145 123. Cost-of-living adjustment clauses (COLAs): A) invalidate the "rule of 70." B) apply only to demand-pull inflation. C) increase the gap between nominal and real income. D) tie wage increases to changes in the price level. Answer: D

Type: D Topic: 8 E: 148 MA: 148 124. During a period of hyperinflation: A) creditors gain because their loans are repaid with dollars of higher value. B) people tend to hold goods rather than money. C) income is redistributed away from borrowers. D) the real value of the national currency rises. Answer: B

Type: A Topic: 8 E: 144 MA: 144 125. Inflation is undesirable because it: A) arbitrarily redistributes real income and wealth. C) usually is accompanied by declining real GDP. B) invariably leads to hyperinflation. D) reduces everyone's standard of living. Answer: A

Type: A Topic: 8 E: 146 MA: 146 126. Who is least likely to be hurt by unanticipated inflation? A) a disabled laborer who is living off accumulated savings B) an owner of a small business C) a secretary D) a pensioned steelworker Answer: B

McConnell/Brue: Economics, 16/e

Page 248

Chapter 8: Introduction to Economic Growth and Instability

Type: A Topic: 8 E: 144 MA: 144 127. Which of the following statements is correct? Unanticipated inflation: A) arbitrarily "taxes" fixed-income groups. C) increases the purchasing power of the dollar. B) increases the real value of savings. D) benefits creditors at the expense of debtors. Answer: A

Type: A Topic: 8 E: 146 MA: 146 128. A lender need not be penalized by inflation if the: A) long-term rate of inflation is less than the short-term rate of inflation. B) short-term rate of inflation is less than the long-term rate of inflation. C) lender correctly anticipates inflation and increases the nominal interest rate accordingly. D) inflation is unanticipated by both borrower and lender. Answer: C

Type: A Topic: 8 E: 146 MA: 146 129. Unanticipated inflation: A) reduces the real burden of the public debt to the Federal government. B) hurts borrowers and helps lenders. C) hurts people whose sole source of income is from Social Security benefits. D) helps savers. Answer: A

Type: A Topic: 8 E: 146 MA: 146 130. Inflation affects: A) both the level and the distribution of income. B) neither the level nor the distribution of income. Answer: A

C) the distribution, but not the level, of income. D) the level, but not the distribution, of income.

Type: A Topic: 8 E: 146 MA: 146 131. If the nominal interest rate is 5 percent and the real interest rate is 2 percent, then the inflation premium is: A) 8 percent. B) 5 percent. C) 3 percent. D) 2 percent. Answer: C

Type: A Topic: 8 E: 146 MA: 146 132. If both the real interest rate and the nominal interest rate are 3 percent, then the: A) inflation premium is zero. C) nominal GDP must exceed real GDP. B) real GDP must exceed the nominal GDP. D) inflation premium also is 3 percent. Answer: A

Type: A Topic: 8 E: 146 MA: 146 133. Suppose the nominal annual interest rate on a two year loan is 8 percent and lenders expect inflation to be 5 percent in each of the two years. The annual real rate of interest is: A) 6 percent. B) 8 percent. C) 2 percent. D) 3 percent. Answer: D

McConnell/Brue: Economics, 16/e

Page 249

Chapter 8: Introduction to Economic Growth and Instability

Consider This Questions

Type: A E: 143 MA: 143 Status: New 134. (Consider This) The feudal practice of clipping coins illustrates the idea of: A) taxation through inflation. C) the derived demand for resources. B) good money driving out bad money. D) cost-push inflation. Answer: A

Type: A E: 143 MA: 143 Status: New 135. (Consider This) The main point of the Consider This box on clipping coins is that: A) decreases in the money supply cause deflation. B) decreases in tax rates often increase tax revenues. C) inflation imposes a "hidden tax" on those who hold money. D) demand creates its own supply. Answer: C

Last Word Questions

Type: F E: 149 MA: 149 136. (Last Word) Which of the following statements is false? A) Stock prices are one clue as to the future direction of the economy. B) Research has shown that changes in stock prices have a relatively modest impact on the economy. C) Black Monday (1987) was followed immediately by a severe recession. D) A rise in stock prices increases consumer wealth and therefore slightly boosts consumption spending. Answer: C

Type: F E: 149 MA: 149 137. (Last Word) Declines in stock prices measured by the Dow Jones average: A) are a major cause of recessions. B) usually reduce saving and increase consumption spending. C) usually increase investment and reduce net exports. D) sometimes precede recessions; sometimes do not. Answer: D

Type: F E: 149 MA: 149 138. (Last Word) Changes in stock market prices: A) do not greatly impact the macroeconomy and used alone are not reliable predictors of the future health of the economy. B) greatly impact the macroeconomy but used alone are not reliable predictors of the future health of the economy. C) greatly impact the macroeconomy and used alone are reliable predictors of the future health of the economy. D) do not greatly impact the macroeconomy but used alone are reliable predictors of the future health of the economy. Answer: A

McConnell/Brue: Economics, 16/e

Page 250

Chapter 8: Introduction to Economic Growth and Instability

Type: F E: 149 MA: 149 139. (Last Word) A burst stock market bubble might adversely affect the economy by: A) causing rapid inflation. B) greatly reducing net exports. C) causing a severe negative wealth effect and engendering pessimism about the economy's future. D) raising interest rates. Answer: C

True/False Questions

Type: A E: 131 MA: 131 140. If real GDP is $300 billion in year 1 and $312 billion in year 2, the growth rate between the two years is 4 percent. Answer: True

Type: F E: 132 MA: 132 141. Between 1992 and 2000, real GDP per capita increased more rapidly than real GDP in the United States. Answer: False

Type: F E: 132 MA: 132 142. About two-thirds of U.S. economic growth derives from more inputs and one-third derives from greater productivity of inputs. Answer: False

Type: A E: 135 MA: 135 143. The production of durable goods is more stable than the production of nondurables over the business cycle. Answer: False

Type: F E: 146 MA: 146 144. Unanticipated inflation benefits debtors at the expense of creditors. Answer: True

Type: A E: 132 MA: 132 145. If the price level doubles in a 23-year period, we can conclude that the average annual rate of inflation over that period was about 3 percent. Answer: True

Type: A E: 134 MA: 134 146. The business cycle is so named because upswings and downswings in business activity are equal in terms of duration and intensity. Answer: False

Type: D E: 136 MA: 136 147. People who work part time, but desire to work full time, are considered to be officially unemployed. Answer: False

McConnell/Brue: Economics, 16/e

Page 251

Chapter 8: Introduction to Economic Growth and Instability

Type: A E: 138 MA: 138 148. The natural rate of unemployment in the United States is about 4 to 5 percent. Answer: True

Type: F E: 132 MA: 132 149. An annual rate of inflation of 7 percent will double the price level in about 15 years. Answer: False

Type: A E: 145 MA: 145 150. Unanticipated inflation benefits some groups in the economy. Answer: True

Type: A E: 146 MA: 146 151. If the nominal interest rate is 8 percent and the real interest rate is 5 percent, then the inflation premium is 13 percent. Answer: False

Type: A E: 134 MA: 134 152. During the past ten years the U.S. economy has experience three recessions. Answer: False

Type: F E: 142 MA: 142 153. During the past ten years the annual rate of inflation in the United States has averaged less than 1 percent. Answer: False

Type: F E: 140-141 MA: 140-141 154. During recent years the U.S. unemployment rate has been substantially higher than the rate in most of the other major industrial nations. Answer: False

McConnell/Brue: Economics, 16/e

Page 252

CHAPTER 9

Basic Macroeconomic Relationships

Topic 1. 2. 3. 4 5. 6. Consumption function/APC/MPC Saving function/APS/MPS Shifts in consumption and saving functions Graphs/tables: mixed consumption and saving Investment demand Multiplier effect Consider This Last Word True-False

Question numbers 1-39 40-53 54-69 70-106 107-145 146-181 182-183 184-185 186-200

____________________________________________________________

_______________________________________

____________________________________________________________

_______________________________________

Multiple Choice Questions Consumption function/APC/MPC

Type: A Topic: 1 E: 152 MA: 152 1. The most important determinant of consumer spending is: A) the level of household debt. C) the stock of wealth. B) consumer expectations. D) the level of income. Answer: D

Type: D Topic: 1 E: 152 MA: 152 2. The most important determinant of consumption and saving is the: A) level of bank credit. B) level of income. C) interest rate. D) price level. Answer: B

Type: A Topic: 1 E: 156 MA: 156 3. If Smith's disposable income increases from $1,200 to $1,700 and her level of saving increases from minus $100 to a plus $100, her marginal propensity to: A) save is three-fifths. C) consume is three-fifths. B) consume is one-half. D) consume is one-sixth. Answer: C

Type: A Topic: 1 E: 156 MA: 156 4. With an MPS of .4, the MPC will be: A) 1.0 minus .4. B) .4 minus 1.0. C) the reciprocal of the MPS. Answer: A

D) .4.

Chapter 9: Basic Macroeconomic Relationships

Type: D Topic: 1 E: 156 MA: 156 5. The MPC can be defined as that fraction of a: A) change in income that is not spent. B) change in income that is spent. Answer: B

C) given total income that is not consumed. D) given total income that is consumed.

Type: A Topic: 1 E: 154 MA: 154 6. The 45-degree line on a graph relating consumption and income shows: A) all points where the MPC is constant. B) all points at which saving and income are equal. C) all the points at which consumption and income are equal. D) the amounts households will plan to save at each possible level of income. Answer: C

Type: A Topic: 1 E: 154 MA: 154 7. As disposable income goes up the: A) APC falls. B) APS falls. Answer: A

C) volume of consumption declines absolutely. D) volume of investment diminishes.

Type: D Topic: 1 E: 153 MA: 153 8. The consumption schedule shows: A) that the MPC increases in proportion to GDP. B) that households consume more when interest rates are low. C) that consumption depends primarily on the level of business investment. D) the amounts households plan or intend to consume at various possible levels of aggregate income. Answer: D

Type: D Topic: 1 E: 153 MA: 153 9. The consumption schedule relates: A) consumption to the level of disposable income. B) saving to the level of disposable income. Answer: A

C) disposable income to domestic income. D) consumption to saving.

Type: A Topic: 1 E: 153 MA: 153 10. A decline in disposable income: A) increases consumption by moving upward along a specific consumption schedule. B) decreases consumption because it shifts the consumption schedule downward. C) decreases consumption by moving downward along a specific consumption schedule. D) increases consumption because it shifts the consumption schedule upward. Answer: C

Type: D Topic: 1 E: 154 MA: 154 11. The APC is calculated as: A) change in consumption / change in income B) consumption / income Answer: B

C) change in income / change in consumption D) income / consumption

McConnell/Brue: Economics, 16/e

Page 254

Chapter 9: Basic Macroeconomic Relationships

Type: A Topic: 1 E: 153 MA: 153 12. The consumption schedule shows: A) a direct relationship between aggregate consumption and accumulated wealth. B) a direct relationship between aggregate consumption and aggregate income. C) an inverse relationship between aggregate consumption and accumulated financial wealth. D) an inverse relationship between aggregate consumption and aggregate income. Answer: B

Type: D Topic: 1 E: 153 MA: 153 13. The APC can be defined as the fraction of a: A) change in income that is not spent. B) change in income that is spent. C) specific level of total income that is not consumed. D) specific level of total income that is consumed. Answer: D

Type: G Topic: 1 E: 154-155 MA: 154-155 14.

The consumption schedule in the above diagram indicates that: A) consumers will maximize their satisfaction where the consumption schedule and 45° line intersect. B) up to a point consumption exceeds income, but then falls below income. C) the MPC falls as income increases. D) households consume as much as they earn. Answer: B

Type: A Topic: 1 E: 154 MA: 154 15. The consumption schedule is drawn on the assumption that as income increases consumption will: A) be unaffected. B) increase absolutely, but remain constant as a percentage of income. C) increase absolutely, but decline as a percentage of income. D) increase both absolutely and as a percentage of income. Answer: C

Type: A Topic: 1 E: 154 MA: 154 16. Which of the following is correct? A) APC + APS = 1. B) APC + MPS = 1. Answer: A

C) APS + MPC = 1.

D) APS + MPS = 1.

McConnell/Brue: Economics, 16/e

Page 255

Chapter 9: Basic Macroeconomic Relationships

Type: A Topic: 1 E: 154-156 MA: 161 17. The consumption schedule is such that: A) both the APC and the MPC increase as income rises. B) the APC is constant and the MPC declines as income rises. C) the MPC is constant and the APC declines as income rises. D) the MPC and APC must be equal at all levels of income. Answer: C

Type: A Topic: 1 E: 154 MA: 154 18. For all levels of income to the left of the intersection of the 45-degree line and the consumption schedule, the APC is: A) greater than 100 percent. B) less than the APS. C) equal to the MPC. D) equal to 100 percent. Answer: A

Type: A Topic: 1 E: 156 MA: 156 19. The consumption and saving schedules reveal that the: A) MPC is greater than zero, but less than one. B) MPC and APC are equal at the point where the consumption schedule intersects the 45-degree line. C) APS is positive at all income levels. D) MPC is equal to or greater than one at all income levels. Answer: A

Type: A Topic: 1 E: 156 MA: 156 20. The size of the MPC is assumed to be: A) less than zero. B) greater than one. Answer: C

C) greater than zero, but less than one.

D) two or more.

Type: A Topic: 1 E: 153-154 MA: 153-154 21. As disposable income increases, consumption: A) and saving both increase. B) and saving both decrease. Answer: A

C) decreases and saving increases. D) increases and saving decreases.

Type: D Topic: 1 E: 154 MA: 154 22. The average propensity to consume indicates the: A) amount by which income exceeds consumption. B) relationship between a change in saving and the consequent change in consumption. C) percentage of total income that will be consumed. D) percentage of a change in income that will be consumed. Answer: C

Type: A Topic: 1 E: 153 MA: 153 23. The relationship between consumption and disposable income is such that: A) an inverse and stable relationship exists between consumption and income. B) a direct, but very volatile, relationship exists between consumption and income. C) a direct and relatively stable relationship exists between consumption and income. D) the two are always equal. Answer: C

McConnell/Brue: Economics, 16/e

Page 256

Chapter 9: Basic Macroeconomic Relationships

Type: A Topic: 1 E: 156 MA: 156 24. If the MPC is .8 and disposable income is $200, then A) consumption and saving cannot be determined from the information given. B) saving will be $20. C) personal consumption expenditures will be $80. D) saving will be $40. Answer: A

Type: A Topic: 1 E: 156 MA: 156 25. The MPC for an economy is: A) the slope of the consumption schedule or line. B) the slope of the savings schedule or line. C) 1 divided by the slope of the consumption schedule or line. D) 1 divided by the slope of the savings schedule or line. Answer: A

Type: F Topic: 1 E: 158 MA: 158 26. In contrast to investment, consumption is: A) relatively stable. B) relatively unstable. Answer: A

C) measurable.

D) unmeasurable.

Use the following to answer questions 27-28: (Advanced analysis) Answer the next question(s) on the basis of the following consumption schedule: C = 20 + .9Y , where C is consumption and Y is disposable income.

Type: E Topic: 1 E: 156 MA: 156 27. Refer to the above data. The MPC is: A) .45. B) .20. C) .50. D) .90. Answer: D

Type: E Topic: 1 E: 156 MA: 156 28. Refer to the above data. At an $800 level of disposable income, the level of saving is: A) $180. B) $740. C) $60. D) $18. Answer: C

Type: A Topic: 1 E: 156 MA: 156 29. Which one of the following will cause a movement down along an economy's consumption schedule? A) an increase in stock prices C) an increase in consumer indebtedness B) a decrease in stock prices D) a decrease in disposable income Answer: D

McConnell/Brue: Economics, 16/e

Page 257

Chapter 9: Basic Macroeconomic Relationships

Type: G Topic: 1 E: 156 MA: 156 30.

The above diagram shows consumption schedules for economies A and B. We can say that the: A) MPC is greater in B than in A. B) APC at any given income level is greater in B than in A. C) MPS is smaller in B than in A. D) MPC is greater in A than in B. Answer: D

Type: A Topic: 1 E: 154 MA: 154 31. At the point where the consumption schedule intersects the 45-degree line: A) the MPC is 1.00. C) saving is equal to consumption. B) the APC is 1.00. D) the economy is in equilibrium. Answer: B

Type: C Topic: 1 E: 156 MA: 156 32. Holly's break-even level of income is $10,000 and her MPC is 0.75. If her actual disposable income is $16,000, her level of: A) consumption spending will be $14,500. C) consumption spending will be $13,000. B) consumption spending will be $15,500. D) saving will be $2,500. Answer: A

Type: A Topic: 1 E: 156 MA: 156 33. If Ben's MPC is .80, this means that he will: A) spend eight-tenths of any increase in his disposable income. B) spend eight-tenths of any level of disposable income. C) break even when his disposable income is $8,000. D) save two-tenths of any level of disposable income. Answer: A

Type: A Topic: 1 E: 154 MA: 154 34. Suppose a family's consumption exceeds its disposable income. This means that its: A) MPC is greater than 1. B) MPS is negative. C) APC is greater than 1. D) APS is positive. Answer: C

McConnell/Brue: Economics, 16/e

Page 258

Chapter 9: Basic Macroeconomic Relationships

Type: E Topic: 1 E: 154 MA: 154 35. (Advanced analysis) If the equation for the consumption schedule is C = 20 + 0.8Y , where C is consumption and Y is disposable income, then the average propensity to consume is 1 when disposable income is: A) $80. B) $100. C) $120. D) $160. Answer: B

Type: E Topic: 1 E: 156 MA: 156 36. (Advanced analysis) The equation C = 35 + .75Y , where C is consumption and Y is disposable income, shows that: A) households will consume three-fourths of whatever level of disposable income they receive. B) households will consume $35 if their disposable income is zero and will consume three-fourths of any increase in disposable income they receive. C) there is an inverse relationship between disposable income and consumption. D) households will save $35 if their disposable income is zero and will consume three-fourths of any increase in disposable income they receive. Answer: B

Type: E Topic: 1 E: 156 MA: 156 37. (Advanced analysis) If the equation C = 20 + .6Y , where C is consumption and Y is disposable income, were graphed: A) the vertical intercept would be +.6 and the slope would be +20. B) it would reveal an inverse relationship between consumption and disposable income. C) the vertical intercept would be negative, but consumption would increase as disposable income rises. D) the vertical intercept would be +20 and the slope would be +.6. Answer: D

Type: A Topic: 1 E: 154 MA: 154 38. One can determine the amount of any level of total income that is consumed by: A) multiplying total income by the slope of the consumption schedule. B) multiplying total income by the APC. C) subtracting the MPS from total income. D) multiplying total income by the MPC. Answer: B

Type: C Topic: 1 E: 154, 156 MA: 154, 156 39. Which of the following is correct? A) MPC + MPS = APC + APS B) APC + MPS = APS + MPC Answer: A

C) APC + MPC = APS + MPS D) APC - APS = MPC - MPS

Saving function/APS/MPS Type: A Topic: 2 E: 154 MA: 154 40. The consumption and saving schedules reveal that: A) consumption rises, but saving declines, as disposable income rises. B) saving varies inversely with the profitability of investment. C) saving varies directly with the level of disposable income. D) saving is inversely related to the rate of interest. Answer: C

McConnell/Brue: Economics, 16/e

Page 259

Chapter 9: Basic Macroeconomic Relationships

Type: D Topic: 2 E: 154 MA: 154 41. Dissaving means: A) the same thing as disinvesting. B) that households are spending more than their current incomes. C) that saving and investment are equal. D) that disposable income is less than zero. Answer: B

Type: D Topic: 2 E: 154 MA: 154 42. Dissaving occurs where: A) income exceeds consumption. B) saving exceeds consumption. Answer: C

C) consumption exceeds income. D) saving exceeds income.

Type: A Topic: 2 E: 156 MA: 156 43. Which of the following relations is not correct? A) 1 - MPC = MPS B) APS + APC = 1 C) MPS = MPC + 1 Answer: C

D) MPC + MPS = 1

Type: A Topic: 2 E: 154 MA: 154 44. The saving schedule is drawn on the assumption that as income increases: A) saving will decline absolutely and as a percentage of income. B) saving will increase absolutely, but remain constant as a percentage of income. C) saving will increase absolutely, but decline as a percentage of income. D) saving will increase absolutely and as a percentage of income. Answer: D

Type: A Topic: 2 E: 154 MA: 154 45. At the point where the consumption schedule intersects the 45-degree line: A) the MPC equals 1. B) the APC is zero. C) saving equals income. D) saving is zero. Answer: D

Type: A Topic: 2 E: 154 MA: 154 46. The saving schedule is such that as aggregate income increases by a certain amount saving: A) increases by the same amount as the increase in income. B) does not change. C) increases, but by a smaller amount. D) increases by an even larger amount. Answer: C

Type: A Topic: 2 E: 156 MA: 156 47. If the consumption schedule is linear, then the: A) saving schedule will also be linear. B) MPS will decline as income rises. Answer: A

C) MPC will decline as income rises. D) APC will be constant at all levels of income.

McConnell/Brue: Economics, 16/e

Page 260

Chapter 9: Basic Macroeconomic Relationships

Type: A Topic: 2 E: 153 MA: 153 48. Given the consumption schedule, it is possible to graph the relevant saving schedule by: A) subtracting the MPC from 1 at each level of income. B) subtracting investment from consumption at each level of GDP. C) plotting the horizontal differences between the consumption schedule and the 45-degree line. D) plotting the vertical differences between the consumption schedule and the 45-degree line. Answer: D

Type: A Topic: 2 E: 154 MA: 154 49. As aggregate income decreases, the APC: A) and APS will both increase. B) will decrease, but the APS will increase. Answer: C

C) will increase, but the APS will decrease. D) and APS will both decrease.

Type: A Topic: 2 E: 156 MA: 156 50. If the marginal propensity to consume is .9, then the marginal propensity to save must be: A) 1. B) .1. C) 1.1. D) .9. Answer: B

Type: A Topic: 2 E: 156 MA: 156 51. The greater is the marginal propensity to consume, the: A) smaller is the marginal propensity to save. C) lower is the average propensity to consume. B) higher is the interest rate. D) lower is the price level. Answer: A

Type: A Topic: 2 E: 156 MA: 156 52. If the saving schedule is a straight line, the: A) MPS must be constant. B) APS must be constant. Answer: A

C) APC must be constant. D) MPC must be rising.

Type: A Topic: 2 E: 154 MA: 154 53. Which one of the following will cause a movement up along an economy's saving schedule? A) an increase in household debt outstanding C) an increase in stock prices B) an increase in disposable income D) an increase in interest rates Answer: B

Shifts in consumption and saving functions

Type: D Topic: 3 E: 156-157 MA: 156-157 54. In the late 1990s the U. S. stock market boomed, causing U.S. consumption to rise. Economists refer to this outcome as the: A) Keynes effect. B) interest-rate effect. C) wealth effect. D) multiplier effect. Answer: C

McConnell/Brue: Economics, 16/e

Page 261

Chapter 9: Basic Macroeconomic Relationships

Type: A Topic: 3 E: 157 MA: 157 55. The wealth effect is shown graphically as a: A) shift of the consumption schedule. B) movement along an existing consumption schedule. C) shift of the investment schedule. D) movement along an existing investment schedule. Answer: A

Use the following to answer questions 56-59:

C2
Consumption

C1 b a

0

Income

Type: G Topic: 3 E: 157 MA: 157 56. Refer to the above graph. A movement from b to a along C1 might be caused by a: A) recession. B) wealth effect of an increase in stock market prices. C) decrease in income tax rates. D) increase in saving. Answer: A

Type: G Topic: 3 E: 157 MA: 157 57. Refer to the above graph. A shift of the consumption schedule from C1 to C2 might be caused by a: A) recession. B) wealth effect of an increase in stock market prices. C) increase in income tax rates. D) increase in saving. Answer: B

Type: G Topic: 3 E: 157 MA: 157 58. Refer to the above graph. A movement from a to b along C1 might be caused by a: A) recession. B) wealth effect of an increase in stock market prices. C) increase in income tax rates. D) increase in real GDP. Answer: D

McConnell/Brue: Economics, 16/e

Page 262

Chapter 9: Basic Macroeconomic Relationships

Type: G Topic: 3 E: 157 MA: 157 59. Refer to the above graph. A shift of the consumption schedule from C2 to C1 might be caused by a: A) increase in real GDP. B) reverse wealth effect, caused by a decrease in stock market prices. C) decrease in income tax rates. D) decrease in saving. Answer: B

Type: C Topic: 3 E: 157 MA: 157 60. An upward shift of the saving schedule suggests: A) nothing with respect to changes in the APC and APS. B) that the APC and APS have both decreased at each GDP level. C) that the APC and APS have both increased at each GDP level. D) that the APC has decreased and the APS has increased at each GDP level. Answer: D

Type: A Topic: 3 E: 157 MA: 157 61. Which of the following will not tend to shift the consumption schedule upward? A) a currently small stock of durable goods in the possession of consumers B) the expectation of a future decline in the consumer price index C) a currently low level of household debt. D) the expectation of future shortages of essential consumer goods. Answer: B

Type: A Topic: 3 E: 157 MA: 157 62. If the consumption schedule shifts upward and the shift was not caused by a tax change, the saving schedule: A) will not shift. C) will shift downward. B) may shift either upward or downward. D) will also shift upward. Answer: C

Type: A Topic: 3 E: 156 MA: 156 63. Which of the following will not cause the consumption schedule to shift? A) a sharp increase in the amount of wealth held by households B) a change in consumer incomes C) the expectation of a recession D) a growing expectation that consumer durables will be in short supply Answer: B

Type: A Topic: 3 E: 157 MA: 157 64. An increase in personal taxes will shift: A) both the consumption and saving schedules downward. B) both the consumption and saving schedules upward. C) the consumption schedule upward and the saving schedule downward. D) the consumption schedule downward and the saving schedule upward. Answer: A

McConnell/Brue: Economics, 16/e

Page 263

Chapter 9: Basic Macroeconomic Relationships

Type: A Topic: 3 E: 157 MA: 157 65. If for some reason households become increasingly thrifty, we could show this by: A) a downshift of the saving schedule. C) an upshift of the saving schedule. B) an upshift of the consumption schedule. D) an increase in the equilibrium GDP. Answer: C

Type: G Topic: 3 E: 156 MA: 156 66.

Suppose the economy's saving schedule shifts from S1 to S 2 as shown in the above diagram. We can say that its: A) MPC has increased. B) MPS has increased. C) APS has increased at all levels of disposable income. D) APS has decreased at all levels of disposable income. Answer: B

Type: C Topic: 3 E: 154 MA: 154 67. If a consumption schedule shifts upward, this necessarily means that the: A) MPC has increased. B) MPS has decreased. C) APC is now higher at each level of disposable income. D) APC is now lower at each level of disposable income. Answer: C

Type: A Topic: 3 E: 158 MA: 158 68. Assume the economy's consumption and saving schedules simultaneously shift downward. This must be the result of: A) an increase in disposable income. C) an increase in personal taxes. B) an increase in household wealth. D) the expectation of a recession. Answer: C

McConnell/Brue: Economics, 16/e

Page 264

Chapter 9: Basic Macroeconomic Relationships

Type: G Topic: 3 E: 154 MA: 154 69.

Suppose an economy's consumption schedule shifts from C1 to C2 as shown in the above diagram. We can say that its: A) MPC has increased but its APC at each income level is unchanged. B) APC at each income level is increased but its MPC is unchanged. C) MPC and APC at each income level have both increased. D) MPC and APC at each income level have both decreased. Answer: C

Graphs/tables: mixed consumption and saving

Use the following to answer questions 70-72:
Disposable income $ 200 225 250 275 300

Consumption $ 205 225 245 265 285

Type: T Topic: 4 E: 156 MA: 156 70. Refer to the above data. The marginal propensity to consume is: A) .25. B) .75. C) .20. D) .80. Answer: D

Type: T Topic: 4 E: 154 MA: 154 71. Refer to the above data. At the $200 level of disposable income: A) the marginal propensity to save is 2½ percent. C) the average propensity to save is .20. B) dissaving is $5. D) the average propensity to consume is .80. Answer: B

Type: T Topic: 4 E: 156 MA: 156 72. Refer to the above data. If disposable income was $325, we would expect consumption to be: A) $315. B) $305. C) $20. D) $290. Answer: B

McConnell/Brue: Economics, 16/e

Page 265

Chapter 9: Basic Macroeconomic Relationships

Use the following to answer questions 73-78:

Type: G Topic: 4 E: 154 MA: 154 73. Refer to the above diagram. The average propensity to consume is 1 at point: A) F. B) A. C) D. D) B. Answer: B

Type: G Topic: 4 E: 156 MA: 156 74. Refer to the above diagram. The marginal propensity to consume is equal to: A) AE/0E. B) CF/CD. C) CB/AB. D) CD/CF. Answer: C

Type: G Topic: 4 E: 154-155 MA: 154-155 75. Refer to the above diagram. At income level F the volume of saving is: A) BD. B) AB. C) CF-BF. D) CD. Answer: D

Type: G Topic: 4 E: 154 MA: 154 76. Refer to the above diagram. Consumption will be equal to income at: A) an income of E. B) an income of F. C) point C. D) point D. Answer: A

Type: G Topic: 4 E: 154-155 MA: 154-155 77. Refer to the above diagram. The economy is dissaving: A) in the amount CD. C) at income level H. B) at all income levels greater than E. D) at income level E. Answer: C

Type: G Topic: 4 E: 156 MA: 156 78. Refer to the above diagram. The marginal propensity to save is: A) CD/EF. B) CB/CF. C) CB/AF. D) EF/CB. Answer: A

McConnell/Brue: Economics, 16/e

Page 266

Chapter 9: Basic Macroeconomic Relationships

Use the following to answer questions 79-80:
120 100 80 60
40

Consumption (billions)

C

20

45 20 40 60 80 100 120 Income (billions)

0

Type: A Topic: 4 E: 153 MA: 153 79. The above figure suggests that: A) consumption would be $60 billion even if income were zero. B) saving is zero at the $120 billion income level. C) as income increases, consumption decreases as a percentage of income. D) as income increases, consumption decreases absolutely. Answer: C

Type: A Topic: 4 E: 154-155 MA: 154-155 80. Refer to the above figure. If the relevant saving schedule were constructed: A) saving would be minus $20 billion at the zero level of income. B) aggregate saving would be $60 at the $60 billion level of income. C) its slope would be 1/2. D) it would slope downward and to the right Answer: A

Use the following to answer questions 81-83: Answer the next question(s) on the basis of the following data for a hypothetical economy.
Disposable income $ 0 50 100 150 200 Saving $-10 0 10 20 30

Type: T Topic: 4 E: 156 MA: 156 81. Refer to the above data. The marginal propensity to consume is: A) .80. B) .75. C) .20. D) .25. Answer: A

McConnell/Brue: Economics, 16/e

Page 267

Chapter 9: Basic Macroeconomic Relationships

Type: T Topic: 4 E: 154 MA: 154 82. Refer to the above data. At the $100 level of income, the average propensity to save is: A) .10. B) .20. C) .25. D) .90. Answer: A

Type: T Topic: 4 E: 156 MA: 156 83. Refer to the above data. If plotted on a graph, the slope of the saving schedule would be: A) .80. B) .10. C) .20. D) .15. Answer: C

Use the following to answer questions 84-88:

Type: G Topic: 4 E: 156 MA: 156 84. Refer to the above diagram. The marginal propensity to save is equal to: A) CD/0D. B) 0B/0A. C) 0D/0D. D) CD/BD. Answer: D

Type: G Topic: 4 E: 154 MA: 154 85. Refer to the above diagram. At disposable income level D, the average propensity to save is equal to: A) CD/BD. B) CD/D. C) D/CD. D) A/B. Answer: B

Type: G Topic: 4 E: 154-155 MA: 154-155 86. Refer to the above diagram. At disposable income level D, consumption is: A) equal to CD. B) equal to D minus CD. C) equal to CD/D. D) equal to CD plus BD. Answer: B

Type: G Topic: 4 E: 154-155 MA: 154-155 87. Refer to the above diagram. Consumption equals disposable income when: A) disposable income is B. B) disposable income is D. C) CD equals A. Answer: A

D) B equals CD.

McConnell/Brue: Economics, 16/e

Page 268

Chapter 9: Basic Macroeconomic Relationships

Type: A Topic: 4 E: 156-157 MA: 156-157 88. The saving schedule shown in the above diagram would shift downward if, all else equal: A) the average propensity to save increased at each income level. B) the marginal propensity to save rose at each income level. C) consumer wealth rose rapidly because of a significant increase in stock market prices. D) the real interest rate fell. Answer: C Use the following to answer questions 89-96: Answer the next question(s) on the basis of the following consumption schedules. DI signifies disposable income and C represents consumption expenditures. All figures are in billions of dollars.

(1) DI $ 0 10 20 30 40 50 C $ 4 11 18 25 32 39 DI $ 0 80 160 240 320 400

(2) C $ 65 125 185 245 305 365

(3) DI C $ 0 $ 2 20 20 40 38 60 56 80 74 100 92

Type: T Topic: 4 E: 156 MA: 156 89. Refer to the above data. The marginal propensity to consume in economy (1) is: A) .5. B) .3. C) .8. D) .7. Answer: D

Type: T Topic: 4 E: 156 MA: 156 90. Refer to the above data. The marginal propensity to consume: A) is highest in economy (1). C) is highest in economy (3). B) is highest in economy (2). D) cannot be calculated from the data given. Answer: C

Type: T Topic: 4 E: 156 MA: 156 91. Refer to the above data. The marginal propensity to save: A) is highest in economy (1). C) is highest in economy (3). B) is highest in economy (2). D) cannot be determined from the data given. Answer: A

Type: T Topic: 4 E: 154 MA: 154 92. Refer to the above data. At an income level of $40 billion, the average propensity to consume: A) is highest in economy (1). C) is highest in economy (3). B) is highest in economy (2). D) cannot be determined from the data given. Answer: B

Type: T Topic: 4 E: 154 MA: 154 93. Refer to the above data. At an income level of $400 billion, the average propensity to save in economy (2) is: A) .9125. B) .0725. C) .0875. D) .9305. Answer: C

McConnell/Brue: Economics, 16/e

Page 269

Chapter 9: Basic Macroeconomic Relationships

Type: T Topic: 4 E: 156 MA: 156 94. (Advanced analysis) Refer to the above data. When plotted on a graph, the vertical intercept of the consumption schedule in economy (3) is _____ and the slope is _____. A) minus $2; .9. B) $2; .18. C) $100; .5. D) $2; .9. Answer: D

Type: T Topic: 4 E: 158 MA: 158 95. Refer to the above data. Suppose that consumption decreased by $2 billion at each level of DI in each of the three countries. We can conclude that the: A) marginal propensity to consume will remain unchanged in each of the three countries. B) marginal propensity to consume will decline in each of the three countries. C) average propensity to save will fall at each level of DI in each of the three countries. D) marginal propensity to save will rise in each of the three countries. Answer: A

Type: T Topic: 4 E: 157 MA: 157 96. Refer to the above data. A $2 billion increase in consumption at each level of DI could be caused by: A) a decrease in consumer wealth. C) an increase in taxation. B) new expectations of higher future income. D) an increase in saving. Answer: B

Use the following to answer questions 97-100:

Type: G Topic: 4 E: 154 MA: 154 97. Refer to the above diagram. The break-even level of disposable income: A) is zero. B) is minus $10. C) is $100. D) cannot be determined from the information given. Answer: C

Type: G Topic: 4 E: 156 MA: 156 98. Refer to the above diagram. The marginal propensity to consume is: A) .2. B) .8. C) .4. D) .3. Answer: B

Type: G Topic: 4 E: 156 MA: 156 99. (Advanced analysis) The equation for the above saving schedule is: A) Yd = -20 + .8S. B) Yd = 20 + .2S. C) S = -20 + .2Yd. D) S = 20 + .8Yd. Answer: C

McConnell/Brue: Economics, 16/e

Page 270

Chapter 9: Basic Macroeconomic Relationships

Type: G Topic: 4 E: 154-155 MA: 154-155 100. Refer to the above diagram. The average propensity to consume: A) is greater than 1 at all levels of disposable income above $100. B) is greater than 1 at all levels of disposable income below $100. C) is equal to the average propensity to save. D) cannot be determined from the information given. Answer: B

Use the following to answer questions 101-104:
$270 240 210

Consumption (C)

45 line

180 150 120 90 60

C

30
0

45 $30 60 90 120 150 180 210 240 270 300 Income (Y)

Type: G Topic: 4 E: 154 MA: 154 101. Refer to the above diagram. The break-even level of income is: A) zero. B) $150. C) $60. D) $120. Answer: B

Type: G Topic: 4 E: 154 MA: 154 102. Refer to the above diagram. The average propensity to consume is: A) greater than 1 at all levels of income above $150. B) greater than 1 at all levels of income below $150. C) zero. D) .6. Answer: B

Type: G Topic: 4 E: 156 MA: 156 103. Refer to the above diagram. The marginal propensity to consume is: A) .4. B) .6. C) .5. D) .8. Answer: B

Type: G Topic: 4 E: 156 MA: 156 104. (Advanced analysis) Refer to the above diagram. The equation for the consumption schedule is: A) C = .6Y . B) Y = 60 + .6C. C) C = 60 + .6Y . D) C = 60 + .4Y . Answer: C

McConnell/Brue: Economics, 16/e

Page 271

Chapter 9: Basic Macroeconomic Relationships

Use the following to answer questions 105-106: (Advanced analysis) Answer the next question(s) on the basis of the following data:

Disposable income (Yd) $ 0 100 200 300 400

Consumption (C) $ 40 100 160 220 280

Type: T Topic: 4 E: 156 MA: 156 105. Which of the following equations correctly represents the above data? A) Yd = 40 + .6C B) C = 60 + .4Yd C) C = 40 + .6Yd D) C = .6Yd Answer: C

Type: T Topic: 4 E: 156 MA: 156 106. Which of the following equations represents the saving schedule implicit in the above data? A) S = C - Yd B) S = 40 + .4Yd C) S = 40 + .6Yd D) S = -40 + .4Yd Answer: D

Investment demand

Type: F Topic: 5 E: 160 MA: 160 Status: New 107. The investment demand curve portrays an inverse (negative) relationship between: A) investment and real GDP. C) the nominal interest rate and investment. B) the real interest rate and investment. D) the price level and investment. Answer: B

Type: F Topic: 5 E: 160 MA: 160 Status: New 108. The investment demand slopes downward and to the right because lower real interest rates: A) expand consumer borrowing, making investments more profitable. B) boost expected rates of returns on investment. C) enable more investment projects to be undertaken profitably. D) create tax incentives to invest. Answer: C

Type: A Topic: 5 E: 159 MA: 159 Status: New 109. Other things equal, a decrease in the real interest rate will: A) shift the investment demand curve to the right. B) shift the investment demand curve to the left. C) move the economy upward along its existing investment demand curve. D) move the economy downward along its existing investment demand curve. Answer: D

McConnell/Brue: Economics, 16/e

Page 272

Chapter 9: Basic Macroeconomic Relationships

Type: A Topic: 5 E: 159 MA: 159 110. Suppose that a new machine tool having a useful life of only one year costs $80,000. Suppose, also, that the net additional revenue resulting from buying this tool is expected to be $96,000. The expected rate of return on this tool is: A) 80 percent. B) 8 percent. C) 2 percent. D) 20 percent. Answer: D

Type: A Topic: 5 E: 159 MA: 159 111. Assume a machine which has a useful life of only one year costs $2,000. Assume, also, that net of such operating costs as power, taxes, and so forth, the additional revenue from the output of this machine is expected to be $2,300. The expected rate of return on this machine is: A) 7.5 percent. B) 10 percent. C) 15 percent. D) 20 percent. Answer: C

Type: A Topic: 5 E: 159 MA: 159 112. If the firm in the previous question finds it can borrow funds at an interest rate of 10 percent the firm should: A) not purchase the machine because the expected rate of return exceeds the interest rate. B) not purchase the machine because the interest rate exceeds the expected rate of return. C) purchase the machine because the expected rate of return exceeds the interest rate. D) purchase the machine because the interest rate exceeds the expected rate of return. Answer: C

Type: D Topic: 5 E: 159-160 MA: 159-160 113. The relationship between the real interest rate and investment is shown by the: A) investment demand schedule. C) saving schedule. B) consumption of fixed capital schedule. D) aggregate supply curve. Answer: A

Type: A Topic: 5 E: 159-160 MA: 159-160 114. Given the expected rate of return on all possible investment opportunities in the economy: A) an increase in the real rate of interest will reduce the level of investment. B) a decrease in the real rate of interest will reduce the level of investment. C) a change in the real interest rate will have no impact on the level of investment. D) an increase in the real interest rate will increase the level of investment. Answer: A

Type: A Topic: 5 E: 159-160 MA: 159-160 115. A decline in the real interest rate will: A) increase the amount of investment spending. B) shift the investment schedule downward. Answer: A

C) shift the investment demand curve to the right. D) shift the investment demand curve to the left.

McConnell/Brue: Economics, 16/e

Page 273

Chapter 9: Basic Macroeconomic Relationships

Type: A Topic: 5 E: 159-160 MA: 159-160 116. The immediate determinants of investment spending are the: A) expected rate of return on capital goods and the real interest rate. B) level of saving and the real interest rate. C) marginal propensity to consume and the real interest rate. D) interest rate and the expected price level. Answer: A

Type: A Topic: 5 E: 160 MA: 160 117. The investment demand curve suggests: A) that changes in the real interest rate will not affect the amount invested. B) there is an inverse relationship between the real rate of interest and the level of investment spending. C) that an increase in business taxes will tend to stimulate investment spending. D) there is a direct relationship between the real rate of interest and the level of investment spending. Answer: B

Type: T Topic: 5 E: 160 MA: 160 118. Assume there are no prospective investment projects (I) that will yield an expected rate of return (r) of 25 percent or more, but that there are $5 billion of investment opportunities with an expected rate of return between 20 and 25 percent, an additional $5 billion between 15 and 20 percent, and so on. The investmentdemand curve for this economy is:

(a) r I 25% $10 20 15 15 20 10 25 5 30 0 35 Answer: B

(b) r I 25% $ 0 20 5 15 10 10 15 5 20 0 25

(c) r I 20% $10 15 20 10 30 5 40 0 50

(d) r 25% 20 15 10 5 0 I $ 5 10 15 20 25 30

Type: T Topic: 5 E: 160 MA: 160 119. In view of your answer to the previous question, if the real interest rate is 15 percent in this economy, the aggregate amount of investment will be: A) $25. B) $20. C) $15. D) $10. Answer: D

Type: C Topic: 5 E: 162 MA: 162 120. If business taxes are reduced and the real interest rate increases: A) consumption and saving will necessarily increase. B) the level of investment spending might either increase or decrease. C) the level of investment spending will necessarily increase. D) the level of investment spending will necessarily decrease. Answer: B

McConnell/Brue: Economics, 16/e

Page 274

Chapter 9: Basic Macroeconomic Relationships

Type: A Topic: 5 E: 162 MA: 162 121. Other things equal, a 10 percent decrease in corporate income taxes will: A) decrease the market price of real capital goods. B) have no effect on the location of the investment-demand curve. C) shift the investment-demand curve to the right. D) shift the investment-demand curve to the left. Answer: C

Type: A Topic: 5 E: 162 MA: 162 122. The investment demand curve will shift to the right as the result of: A) the availability of excess production capacity. B) an increase in business taxes. C) businesses becoming more optimistic about future business conditions. D) an increase in the real interest rate. Answer: C

Type: A Topic: 5 E: 159-160 MA: 159-160 123. Other things equal, the real interest rate and the level of investment are: A) related only when saving equals planned investment. B) unrelated. C) inversely related. D) directly related. Answer: C

Use the following to answer questions 124-125: Answer the next question(s) on the basis of the following table:
Expected rate of return 12% 10 8 6 4 2 Amount of investment with this rate of return or higher (billions) $10 20 30 40 50 60

Type: T Topic: 5 E: 160 MA: 160 124. The above table reflects a(n): A) interest rate schedule. B) demand-for-money schedule. Answer: C

C) investment demand schedule. D) profit schedule.

Type: T Topic: 5 E: 160 MA: 160 125. The above schedule indicates that if the real interest rate is 8 percent, then: A) we cannot tell what volume of investment will be profitable. B) $30 billion will be both saved and invested. C) $30 billion of investment will be undertaken. D) $60 billion of investment will be undertaken. Answer: C

McConnell/Brue: Economics, 16/e

Page 275

Chapter 9: Basic Macroeconomic Relationships

Type: C Topic: 5 E: 162 MA: 162 126. Other things equal, if the real interest rate falls and business taxes rise: A) investment will rise until it is equal to saving. B) we will be uncertain as to the resulting change in investment. C) we can be certain that investment will rise. D) we can be certain that investment will fall. Answer: B

Type: A Topic: 5 E: 162 MA: 162 127. The investment demand curve will shift to the right as a result of: A) an increase in the excess production capacity available in industry. B) an increase in business taxes. C) technological progress. D) an increase in the acquisition and maintenance cost of capital goods. Answer: C

Type: A Topic: 5 E: 162 MA: 162 128. The investment demand curve will shift to the left as a result of: A) an increase in the excess production capacity available in industry. B) a decrease in business taxes. C) increased business optimism with respect to future economic conditions. D) a decrease in labor costs. Answer: A

Type: A Topic: 5 E: 159 MA: 159 129. If the real interest rate in the economy is i and the expected rate of return from additional investment is r, then more investment will be forthcoming when: A) r falls. B) i is greater than r. C) r is greater than i. D) i rises. Answer: C

Type: A Topic: 5 E: 162 MA: 162 130. A rightward shift of the investment demand curve might be caused by: A) an increase in the price level. B) a decline in the real interest rate. C) an increase in the expected rate of return on investment. D) an increase in business taxes. Answer: C

Type: A Topic: 5 E: 159 MA: 159 131. The real interest rate is: A) the percentage increase in money that the lender receives on a loan. B) the percentage increase in purchasing power that the lender receives on a loan. C) also called the after-tax interest rate. D) usually higher than the nominal interest rate. Answer: B

McConnell/Brue: Economics, 16/e

Page 276

Chapter 9: Basic Macroeconomic Relationships

Type: A Topic: 5 E: 160 MA: 160 132. When we draw an investment demand curve we hold constant all of the following except: A) the expected rate of return on the investment. C) the interest rate. B) business taxes. D) the present stock of capital goods. Answer: C

Type: A Topic: 5 E: 159 MA: 159 133. If the nominal interest rate is 18 percent and the real interest rate is 6 percent, the inflation rate is: A) 18 percent. B) 24 percent. C) 12 percent. D) 6 percent. Answer: C

Type: A Topic: 5 E: 159-160 MA: 159-160 134. If the inflation rate is 10 percent and the real interest rate is 12 percent, the nominal interest rate is: A) 2 percent. B) zero percent. C) 10 percent. D) 22 percent. Answer: D

Type: A Topic: 5 E: 160 MA: 160 135. A high rate of inflation is likely to cause a: A) high nominal interest rate. B) low nominal interest rate. Answer: A

C) low rate of growth of nominal GDP. D) decrease in nominal wages.

Type: A Topic: 5 E: 160 MA: 160 136. If the real interest rate in the economy is i and the expected rate of return on additional investment is r, then other things equal: A) more investment will be forthcoming when i exceeds r. B) less investment will be forthcoming when r rises. C) r will fall as more investment is undertaken. D) r will exceed i at all possible levels of investment. Answer: C

Type: A Topic: 5 E: 159 MA: 159 137. If the real interest rate in the economy is i and the expected rate of return on additional investment is r, then other things equal: A) investment will take place until i and r are equal. B) investment will take place until r exceeds i by the greatest amount. C) r will rise as more investment is undertaken. D) i will fall as more investment is undertaken. Answer: A

McConnell/Brue: Economics, 16/e

Page 277

Chapter 9: Basic Macroeconomic Relationships

Type: G Topic: 5 E: 160 MA: 160 138. Assume that for the entire business sector of a private closed economy there is $0 worth of investment projects that will yield an expected rate of return of 25 percent or more. But there are $15 worth of investments that will yield an expected rate of return of 20-25 percent; another $15 with an expected rate of return of 15-20 percent; and similarly an additional $15 of investment projects in each successive rate of return range down to and including the 0-5 percent range.

Which of the lines on the above diagram represents these data? A) A B) B C) C D) D Answer: B

Use the following to answer questions 139-141: Answer the next question(s) on the basis of the following information for a private closed economy. Assume that for the entire business sector of the economy there is $0 worth of investment projects that will yield an expected rate of return of 25 percent or more. But there are $15 worth of investments that will yield an expected rate of return of 20-25 percent; another $15 with an expected rate of return of 15-20 percent; and similarly an additional $15 of investment projects in each successive rate of return range down to and including the 0-5 percent range.

Type: G Topic: 5 E: 159 MA: 159 139. Refer to the above information. If the real interest rate is 15 percent, what amount of investment will be undertaken? A) $15 B) $30 C) $45 D) $60 Answer: B

Type: G Topic: 5 E: 159 MA: 159 140. Refer to the above information. If the real interest rate is 5 percent, what amount of investment will be undertaken? A) $15 B) $30 C) $45 D) $60 Answer: D

McConnell/Brue: Economics, 16/e

Page 278

Chapter 9: Basic Macroeconomic Relationships

Type: G Topic: 5 E: 160 MA: 160 141. Refer to the above information. The expected rate of return curve: A) shows a direct relationship between the interest rate and investment. B) is also the investment demand curve. C) is indeterminant. D) implies a direct (positive) relationship between the interest rate and the level of GDP. Answer: B

Use the following to answer questions 142-144:

Real interest rate

ID2 ID1
ID3

0

Investment

Type: G Topic: 5 E: 162 MA: 162 Status: New 142. Which of the following would shift the investment demand curve from ID1 to ID2? A) a lower interest rate C) a higher interest rate B) lower expected rates of return on investment D) higher expected rates of return on investment Answer: D

Type: G Topic: 5 E: 162 MA: 162 Status: New 143. Which of the following would shift the investment demand curve from ID1 to ID3? A) a lower interest rate C) a higher interest rate B) lower expected rates of return on investment D) higher expected rates of return on investment Answer: B

Type: G Topic: 5 E: 160 MA: 160 Status: New 144. Which of the following would increase investment, while leaving an existing investment demand curve, say, ID2, in place? A) a lower interest rate C) lower expected returns on investment B) a higher interest rate D) higher expected returns on investment Answer: A

McConnell/Brue: Economics, 16/e

Page 279

Chapter 9: Basic Macroeconomic Relationships

Type: F Topic: 5 E: 162-163 MA: 162-163 Status: New 145. In annual percentage terms, investment spending in the United States is: A) less variable than real GDP. C) less variable than the price level. B) less variable than consumption spending. D) more variable than real GDP. Answer: D

Multiplier effect

Type: A Topic: 6 E: 164 MA: 164 146. The multiplier effect means that: A) consumption is typically several times as large as saving. B) a change in consumption can cause a larger increase in investment. C) an increase in investment can cause GDP to change by a larger amount. D) a decline in the MPC can cause GDP to rise by several times that amount. Answer: C

Type: E Topic: 6 E: 166 MA: 166 147. The multiplier is: A) 1/MPC. B) 1/(1 + MPC). C) 1/MPS. Answer: C

D) 1/(1 - MPS).

Type: A Topic: 6 E: 164 MA: 164 148. The multiplier is useful in determining the: A) full-employment unemployment rate. B) level of business inventories. C) rate of inflation. D) change in GDP resulting from a change in spending. Answer: D

Type: D Topic: 6 E: 164 MA: 164 149. The multiplier is defined as: A) 1 - MPS. B) change in GDP × initial change in spending. Answer: C

C) change in GDP/initial change in spending. D) change in GDP - initial change in spending.

McConnell/Brue: Economics, 16/e

Page 280

Chapter 9: Basic Macroeconomic Relationships

Use the following to answer questions 150-151:

Type: G Topic: 6 E: 156 MA: 156 150. The above figure shows the saving schedules for economies 1, 2, 3, and 4. Which economy has the highest marginal propensity to consume? A) 1 B) 2 C) 3 D) 4 Answer: D

Type: G Topic: 6 E: 166 MA: 166 151. The above figure shows the saving schedules for economies 1, 2, 3, and 4. Which economy has the largest multiplier? A) 1 B) 2 C) 3 D) 4 Answer: D

Type: E Topic: 6 E: 166 MA: 166 152. If 100 percent of any change in income is spent, the multiplier will be: A) equal to the MPC. B) 1. C) zero. D) infinitely large. Answer: D

Type: E Topic: 6 E: 166 MA: 166 153. The multiplier can be calculated as: A) 1/(MPS + MPC) B) MPC/MPS Answer: C

C) 1/(1 - MPC)

D) 1 - MPC = MPS

Type: D Topic: 6 E: 166 MA: 166 154. The multiplier: A) occurs only in response to a change in the level of investment spending. B) can be found by taking the reciprocal of the MPS. C) occurs only when intended investment increases as GDP increases. D) is measured by the slope of the saving schedule. Answer: B

McConnell/Brue: Economics, 16/e

Page 281

Chapter 9: Basic Macroeconomic Relationships

Type: A Topic: 6 E: 166 MA: 166 155. The size of the multiplier is equal to the: A) slope of the consumption schedule. B) reciprocal of the slope of the consumption schedule. C) slope of the saving schedule. D) reciprocal of the slope of the saving schedule. Answer: D

Type: C Topic: 6 E: 166 MA: 166 156. If the MPS is only half as large as the MPC, the multiplier is: A) 2. B) 3. C) 4. D) 5. Answer: B

Type: A Topic: 6 E: 166 MA: 166 157. If the MPC is .70 and gross investment increases by $3 billion, the equilibrium GDP will: A) increase by $10 billion. C) decrease by $4.29 billion. B) increase by $2.10 billion. D) increase by $4.29 billion. Answer: A

Type: A Topic: 6 E: 166 MA: 166 158. The numerical value of the multiplier will be smaller the: A) larger the average propensity to consume. C) larger the slope of the consumption schedule. B) larger the slope of the saving schedule. D) smaller the slope of the saving schedule. Answer: B

Type: A Topic: 6 E: 165 MA: 165 159. The practical significance of the multiplier is that it: A) equates the real interest rate and the expected rate of return on investment. B) magnifies initial changes in spending into larger changes in GDP. C) keeps inflation within tolerable limits. D) helps to stabilize the economy. Answer: B

Type: F Topic: 6 E: 166 MA: 166 160. The multiplier: A) varies directly with the slope of the investment demand schedule. B) is unrelated to the slope of the saving schedule. C) will be greater, the smaller is the slope of the saving schedule. D) will be greater, the steeper is the slope of the saving schedule. Answer: C

Type: A Topic: 6 E: 166 MA: 166 161. The increase in income that results from an increase in investment spending would be greater the: A) smaller the MPS. B) smaller the APC. C) larger the MPS. D) smaller the MPC. Answer: A

McConnell/Brue: Economics, 16/e

Page 282

Chapter 9: Basic Macroeconomic Relationships

Type: A Topic: 6 E: 164 MA: 164 162. The multiplier effect: A) reduces the MPC. B) magnifies changes in spending into larger changes in output and income. C) promotes stability of the general price level. D) lessens upswings and downswings in business activity. Answer: B

Type: E Topic: 6 E: 166 MA: 166 163. If the MPC is .6, the multiplier will be: A) 4.0. B) 6.0. C) 2.5. D) 1.67. Answer: C

Type: C Topic: 6 E: 166 MA: 166 164. Assume the MPC is 2/3. If investment spending increases by $2 billion, the level of GDP will increase by: A) $3 billion. B) $2/3 billion. C) $6 billion. D) $2 billion. Answer: C

Type: E Topic: 6 E: 166 MA: 166 165. The multiplier is: A) 1/APS. B) 1/APC. C) 1/MPC. Answer: D

D) 1/MPS.

Type: A Topic: 6 E: 164 MA: 164 Status: New 166. The multiplier applies to: A) investment but not to net exports or government spending. B) investment, net exports, and government spending. C) increases in spending but not to decreases in spending. D) spending by the private sector but not by the public sector. Answer: B

Type: A Topic: 6 E: 164 MA: 164 167. The multiplier effect indicates that: A) a decline in the interest rate will cause a proportionately larger increase in investment. B) a change in spending will change aggregate income by a larger amount. C) a change in spending will increase aggregate income by the same amount. D) an increase in total income will generate a larger change in aggregate expenditures. Answer: B

McConnell/Brue: Economics, 16/e

Page 283

Chapter 9: Basic Macroeconomic Relationships

Use the following to answer questions 168-173: Answer the next question(s) on the basis of the following table that illustrates the multiplier process.
Change in income Assumed increase in investment Second round All other rounds Totals $20 $_____ $_____ $_____ Change in consumption $_____ $12.80 $36.00 $_____ Change in saving $4 $_____ $_____ $20

Type: T Topic: 6 E: 156 MA: 156 168. Refer to the above table. The marginal propensity to consume is: A) .5. B) .75. C) .8. D) .9. Answer: C

Type: T Topic: 6 E: 156 MA: 156 169. Refer to the above table. The marginal propensity to save is: A) .5. B) .25. C) .2. D) .1. Answer: C

Type: T Topic: 6 E: 156 MA: 156 170. Refer to the above table. The change in income in round two will be: A) $4. B) $16. C) $20. D) $24. Answer: B

Type: T Topic: 6 E: 164 MA: 164 171. Refer to the above table. The total change in income resulting from the initial change in investment will be: A) $100. B) $20. C) $80. D) $200. Answer: A

Type: T Topic: 6 E: 165 MA: 165 172. Refer to the above table. The total change in consumption resulting from the initial change in investment will be: A) $100. B) $96. C) $180. D) $80. Answer: D

Type: T Topic: 6 E: 166 MA: 166 173. Refer to the above table. The multiplier in this economy is: A) 2. B) 4. C) 5. D) 10. Answer: C

McConnell/Brue: Economics, 16/e

Page 284

Chapter 9: Basic Macroeconomic Relationships

Type: C Topic: 6 E: 164 MA: 164 174. If a $200 billion increase in investment spending creates $200 billion of new income in the first round of the multiplier process and $160 billion in the second round, the multiplier in the economy is: A) 4. B) 5. C) 3.33. D) 2.5. Answer: B

Type: C Topic: 6 E: 164 MA: 164 175. If a $50 billion decrease in investment spending causes income to decline by $50 billion in the first round of the multiplier process and by $25 in the second round, the multiplier in the economy is: A) 2. B) 3.33. C) 5. D) 10. Answer: A

Type: C Topic: 6 E: 165 MA: 165 176. If a $100 billion decrease in investment spending causes income to decline by $100 billion in the first round of the multiplier process and by $75 billion in the second round, income will eventually decline by: A) $200 billion. B) $300 billion. C) $400 billion. D) $500 billion. Answer: C

Type: C Topic: 6 E: 165 MA: 165 177. If a $500 billion increase in investment spending increases income by $500 billion in the first round of the multiplier process and by $450 in the second round, income will eventually increase by: A) $2500 billion. B) $3000 billion. C) $4000 billion. D) $5000 billion. Answer: D

Type: C Topic: 6 E: 166 MA: 166 178. If the marginal propensity to save is 0.2 in an economy, a $20 billion rise in investment spending will increase: A) GDP by $120 billion. C) saving by $25 billion. B) GDP by $20 billion. D) consumption by $80 billion. Answer: D

Type: A Topic: 6 E: 166 MA: 166 179. A $1 billion increase in investment will cause a: A) (1/MPS) billion increase in GDP. B) (MPS) billion increase in GDP. Answer: A

C) (1 - MPC) billion increase in GDP. D) (MPC - MPS) billion increase in GDP.

Type: F Topic: 6 E: 166-167 MA: 166-167 180. The Council of Economic Advisers has estimated that the actual multiplier for the U.S. economy is approximately: A) 4. B) 3.5. C) 3. D) 2. Answer: D

McConnell/Brue: Economics, 16/e

Page 285

Chapter 9: Basic Macroeconomic Relationships

Type: F Topic: 6 E: 166-167 MA: 166-167 Status: New 181. The actual multiplier effect in the U.S. economy is less than the multiplier effect in the text examples because: A) the real-world MPS is larger than the MPS in the examples. B) in addition to saving, households use some of any increase in income to buy imported goods and to pay higher taxes. C) the gap between the nominal interest rate and the real interest rate widens as the economy expands or contracts. D) the MPC in the United States is greater than 1. Answer: B

Consider This Questions

Type: F E: 159 MA: 159 Status: New 182. (Consider This) U.S. consumption increased between March 2000 and July 2002 even though stock values declined by $3.7 trillion. One of the reasons was that: A) lower interest rates allowed many households to reduce their monthly loan payments and increase their consumption spending. B) deflation occurred, which increased purchasing power. C) economic growth accelerated relative to the prior two years. D) the unemployment rate dramatically declined. Answer: A

Type: F E: 159 MA: 159 Status: New 183. (Consider This) Part of the wealth effect of a $3.7 trillion decline in stock values between March 2000 and July 2002 was offset by rising: A) tax rates. B) interest rates. C) house values. D) expectations of future income. Answer: C

Last Word Questions

Type: A E: 167 MA: 167 184. (Last Word) Art Buchwald's article "Squaring the Economic Circle" is a humorous description of: A) a negative GDP gap. C) the marginal propensity to save. B) a positive GDP gap. D) the multiplier. Answer: D

Type: A E: 167 MA: 167 185. (Last Word) Art Buchwald's article "Squaring the Economic Circle" humorously describes how: A) a person's decision not to buy an automobile eventually reduces many people's incomes, including that of the person making the original decision. B) a price increase on a single product eventually leads to rapid inflation. C) an increase in imports eventually leads to a greater increase in exports. D) a government tax rate increase eventually results in the government collecting less tax revenue than before the tax rate hike. Answer: A

McConnell/Brue: Economics, 16/e

Page 286

Chapter 9: Basic Macroeconomic Relationships

True/False Questions

Type: A E: 154 MA: 154 186. If DI is $275 billion and the APC is 0.8, we can conclude that saving is $55 billion. Answer: True

Type: A E: 156 MA: 156 187. If the MPC is constant at various levels of income, then the APC must also be constant at all of those income levels. Answer: False

Type: A E: 154 MA: 154 188. The average propensity to consume is defined as income divided by consumption. Answer: False

Type: D E: 156 MA: 156 189. 1 - MPC = MPS. Answer: True

Type: A E: 159 MA: 159 190. A decline in the real interest rate will shift the investment demand curve to the right. Answer: False

Type: A E: 156 MA: 156 191. If the Brown family's marginal propensity to consume is 0.70, then it will necessarily consume seven-tenths of its total income. Answer: False

Type: A E: 156 MA: 156 192. 1 + MPS = MPC. Answer: False

Type: A E: 156 MA: 156 193. The slope of the consumption schedule is measured by the MPC. Answer: True

194.

Type: A E: 159 MA: 159 A specific investment will be undertaken if the expected rate of return, r, exceeds the interest rate, i. Answer: True

Type: A E: 163-164 MA: 163-164 195. Investment is highly stable; it rarely changes. Answer: False

McConnell/Brue: Economics, 16/e

Page 287

Chapter 9: Basic Macroeconomic Relationships

Type: A E: 156 MA: 156 196. The greater the MPC, the greater the multiplier. Answer: True

Type: A E: 166 MA: 166 197. If the MPS is 1, the multiplier will be 1. Answer: True

Type: A E: 166 MA: 166 198. The multiplier is equal to the reciprocal of the MPC. Answer: False

Type: F E: 164 MA: 164 Status: New 199. The multiplier shows the relationship between changes in a component of spending, say, investment, and the consequent changes in real income and output. Answer: True

Type: F E: 167 MA: 167 Status: New 200. The estimate for the value of the real-world multiplier is 2. Answer: True

McConnell/Brue: Economics, 16/e

Page 288

CHAPTER 10

The Aggregate Expenditures Model

Topic 1. 2. 3. 4. 5. 6. Investment schedule Equilibrium GDP in private closed economy Open economy Mixed economy Recessionary and inflationary gaps Applications and critiques Last Word True-False

Question numbers 1-9 10-96 97-125 126-199 200-225 226-233 234-236 237-255

____________________________________________________________

_______________________________________

____________________________________________________________

_______________________________________

Multiple Choice Questions Investment schedule

Type: D Topic: 1 E: 173 MA: 173 1. The relationship between investment and GDP is shown by the: A) consumption of fixed capital schedule. C) investment schedule. B) saving schedule. D) consumption schedule. Answer: C

Type: D Topic: 1 E: 174 MA: 174 2. In the aggregate expenditures model, it is assumed that investment: A) automatically changes in response to changes in real GDP. B) changes by less in percentage terms than changes in real GDP. C) does not respond to changes in interest rates. D) does not change when real GDP changes. Answer: D

Type: C Topic: 1 E: 173 MA: 173 3. All else equal, a large decline in the real interest rate will shift the: A) investment demand curve leftward. C) investment schedule upward. B) investment demand curve rightward. D) investment schedule downward. Answer: C

Chapter 10: The Aggregate Expenditures Model

Use the following to answer questions 4-9:

Type: G Topic: 1 E: 173 MA: 173 4. Refer to the above diagrams. Curve A: A) is an investment schedule and curve B is a consumption of fixed capital schedule. B) is an investment demand curve and curve B is an investment schedule. C) and B are totally unrelated. D) shifts to the left when curve B shifts upward. Answer: B

Type: G Topic: 1 E: 173 MA: 173 5. Refer to the above diagrams. Other things equal, Curve B will shift upward when: A) the level of GDP increases. C) curve A shifts to the left. B) the interest rate increases. D) curve A shifts to the right. Answer: D

Type: G Topic: 1 E: 173 MA: 173 6. Refer to the above diagrams. Other things equal, an interest rate decrease will: A) shift curve A to the right and shift curve B upward. B) shift curve A to the left and shift curve B downward. C) leave curve A in place but shift curve B downward. D) leave curve A in place but shift curve A upward. Answer: D

Type: G Topic: 1 E: 173 MA: 173 7. Refer to the above diagrams. Other things equal, an interest rate increase will: A) shift curve A to the right and shift curve B upward. B) shift curve A to the left and shift curve B downward. C) leave curve A in place but shift curve B downward. D) leave curve A in place but shfit curve A upward. Answer: C

McConnell/Brue: Economics, 16/e

Page 290

Chapter 10: The Aggregate Expenditures Model

Type: G Topic: 1 E: 173 MA: 173 8. Refer to the above diagram. Other things equal, an interest rate reduction coupled with a rightward shift in curve A will: A) shift curve B upward. C) have no effeect on curve B. B) shift curve B downward. D) reduce GDP. Answer: A

Type: G Topic: 1 E: 173 MA: 173 9. Refer to the above diagram. The location of curve B depends on the: A) level of real GDP. B) location of curve A only. C) interest rate only. D) interest rate together with the location of curve A. Answer: D

Equilibrium GDP in private closed economy

Type: D Topic: 2 E: 174 MA: 174 10. The level of aggregate expenditures in the private closed economy is determined by the: A) expenditures of consumers and businesses. B) intersection of the saving schedule and the 45-degree line. C) equality of the MPC and MPS. D) intersection of the saving and consumption schedules. Answer: A

Use the following to answer questions 11-13: Answer the next question(s) on the basis of the following data for a private closed economy.
Possible levels of domestic output and income (GDP = DI) $320 330 340 350 360 370 380 Consumption $320 327 334 341 348 355 362

Type: T Topic: 2 E: 155-156 MA: 155-156 11. Refer to the above data. The MPS is: A) 7/10. B) 3/10. C) 2/5. D) 3/5. Answer: B

Type: T Topic: 2 E: 154-155 MA: 154-155 12. Refer to the above data. At the $370 billion level of DI the APS is approximately: A) 4 percent. B) 7 percent. C) 1 percent. D) 16 percent. Answer: A

McConnell/Brue: Economics, 16/e

Page 291

Chapter 10: The Aggregate Expenditures Model

Type: T Topic: 2 E: 174 MA: 174 13. Refer to the above data. If gross investment is $12 billion, the equilibrium level of GDP will be: A) $380. B) $370. C) $360. D) $350. Answer: C

Use the following to answer questions 14-17:

C + Ig
C + Ig

C $300
$200

$100 0

$100 $200 $300 GDP

Type: G Topic: 2 E: 175-176 MA: 175-176 14. Refer to the above diagram for a private closed economy. The equilibrium level of GDP is: A) $400. B) $300. C) $200. D) $100. Answer: B

Type: G Topic: 2 E: 175-176 MA: 175-176 15. Refer to the above diagram for a private closed economy. At the equilibrium level of GDP, investment and saving are both: A) $50. B) $100. C) $20. D) $40. Answer: A

Type: G Topic: 2 E: 175-176 MA: 175-176 16. Refer to the above diagram for a private closed economy. The $400 level of GDP is: A) that output at which saving is zero. B) too high because consumption exceeds investment. C) unstable because aggregate expenditures exceed GDP. D) unstable because aggregate expenditures are less than GDP. Answer: D

Type: G Topic: 2 E: 175-176 MA: 175-176 17. Refer to the above diagram for a private closed economy. Unplanned changes in inventories will be zero: A) only at the $300 level of GDP. C) at all levels of GDP. B) only at the $200 level of GDP. D) only at the $400 level of GDP. Answer: A

McConnell/Brue: Economics, 16/e

Page 292

Chapter 10: The Aggregate Expenditures Model

Use the following to answer questions 18-21:

Type: G Topic: 2 E: 154-155 MA: 154-155 18. Refer to the above diagram that applies to a private closed economy. The APC is equal to 1 at income level: A) J. B) M. C) H. D) G. Answer: D

Type: G Topic: 2 E: 175-176 MA: 175-176 19. Refer to the above diagram that applies to a private closed economy. If aggregate expenditures are C + Ig2, the amount of saving at income level J is: A) LK. B) KN. C) KD. D) JD. Answer: B

Type: G Topic: 2 E: 155-156 MA: 155-156 20. Refer to the above diagram that applies to a private closed economy. The slope of the consumption schedule in this figure reveals that the: A) MPS rises as income rises. C) APC is constant. B) MPC is constant. D) APC increases as income increases. Answer: B

Type: G Topic: 2 E: 175-176 MA: 175-176 21. Refer to the above diagram that applies to a private closed economy. If gross investment is Ig1, the equilibrium GDP and the level of consumption will be: A) H and HB respectively. C) J and JK respectively B) J and JI respectively. D) H and HF respectively. Answer: D

Type: A Topic: 2 E: 175 MA: 175 22. Other things equal, the slope of the aggregate expenditures schedule will increase as a result of: A) a decline in the size of the inflationary gap. C) an increase in the MPS. B) an increase in the MPC. D) a decline in the general price level. Answer: B

McConnell/Brue: Economics, 16/e

Page 293

Chapter 10: The Aggregate Expenditures Model

Type: A Topic: 2 E: 174 MA: 174 23. The equilibrium level of GDP in a private closed economy is where: A) MPC = APC. B) unemployment is about 3 percent of the labor force. C) consumption equals saving. D) aggregate expenditures equal GDP. Answer: D

Type: A Topic: 2 E: 176-177 MA: 176-177 24. In a private closed economy, when aggregate expenditures equal GDP: A) consumption equals investment. B) consumption equals aggregate expenditures. C) planned investment equals saving. D) disposable income equals consumption minus saving. Answer: C

Type: A Topic: 2 E: 174-175 MA: 174-175 25. In a private closed economy, when aggregate expenditures exceed GDP: A) GDP will decline. C) saving will decline. B) business inventories will rise. D) business inventories will fall. Answer: D

Type: A Topic: 2 E: 177 MA: 177 26. If an unintended increase in business inventories occurs at some level of GDP, then GDP: A) entails a rate of aggregate expenditures in excess of the rate of aggregate production. B) may be either above or below the equilibrium output. C) is too low for equilibrium. D) is too high for equilibrium. Answer: D

Type: A Topic: 2 E: 177 MA: 177 27. The equilibrium level of GDP is associated with: A) an excess of planned investment over saving. B) no unintended changes in inventories. Answer: B

C) an unintended decrease in business inventories. D) an unintended increase in business inventories.

McConnell/Brue: Economics, 16/e

Page 294

Chapter 10: The Aggregate Expenditures Model

Type: G Topic: 2 E: 175 MA: 175 28.
AE4 Consumption and investment AE3 AE2

AE1

45 0 Real GDP

Which aggregate expenditure schedule AE in the above diagram for a private closed economy implies the largest MPC, assuming investment is the same at each level of income? A) AE4 B) AE3 C) AE2 D) AE1 Answer: A

Type: A Topic: 2 E: 177 MA: 177 29. If at some level of GDP the economy is experiencing an unintended decrease in inventories: A) the aggregate level of saving will decline. C) the business sector will lay off workers. B) the price level will fall. D) domestic output will increase. Answer: D

Type: D Topic: 2 E: 174 MA: 174 30. The equilibrium GDP is the level of domestic output: A) where consumption equals saving. C) which is sustainable. B) where actual investment equals consumption. D) where full employment exists. Answer: C

Type: A Topic: 2 E: 177 MA: 177 31. If an unintended increase in business inventories occurs: A) we can expect aggregate production to be unaffected. B) we can expect businesses to increase the level of production. C) we can expect businesses to lower the level of production. D) aggregate expenditures must exceed the domestic output. Answer: C

32.

Type: A Topic: 2 E: 177 MA: 177 Assume that in a private closed economy consumption is $240 billion and investment is $50 billion, both at the $280 billion level of domestic output. Thus: A) saving is $10 billion. B) unplanned decreases in inventories of $10 billion will occur. C) the MPC is .80. D) unplanned increases in inventories of $10 billion will occur. Answer: B

McConnell/Brue: Economics, 16/e

Page 295

Chapter 10: The Aggregate Expenditures Model

Type: A Topic: 2 E: 177 MA: 177 33. A private closed economy will expand when: A) actual GDP is less than potential GDP. B) unplanned decreases in inventories occur. Answer: B

C) aggregate expenditures are less than GDP. D) unplanned increases in inventories occur.

Type: A Topic: 2 E: 177 MA: 177 34. If aggregate expenditures exceed GDP in a private closed economy: A) leakages will exceed injections. C) unplanned investment in inventories will occur. B) planned investment will exceed saving. D) saving will exceed planned investment. Answer: B

Type: A Topic: 2 E: 177 MA: 177 35. For a private closed economy, an unintended decline in inventories suggests that: A) aggregate expenditures are less than the business sector expected them to be. B) aggregate expenditures exceed GDP. C) actual investment exceeds saving. D) planned investment is greater than consumption. Answer: B

Use the following to answer questions 36-39:

$200 180
Private spending (billions)

C+Ig

160 140
120 100 80 60 40 20

C

45 $20 40 60 80 100 120 140 160 180 200 GDP (billions)

0

Type: G Topic: 2 E: 175-176 MA: 175-176 36. Refer to the above diagram for a private closed economy. The equilibrium GDP is: A) $60 billion. C) between $60 and $180 billion. B) $180 billion. D) $60 billion at all levels of GDP. Answer: B

McConnell/Brue: Economics, 16/e

Page 296

Chapter 10: The Aggregate Expenditures Model

Type: G Topic: 2 E: 175-176 MA: 175-176 37. Refer to the above diagram for a private closed economy. In this economy investment: A) decreases as GDP increases. C) is $40 billion at all levels of GDP. B) increases as GDP increases. D) is $60 billion at all levels of GDP. Answer: C

Type: G Topic: 2 E: 175 MA: 175 38. Refer to the above diagram for a private closed economy. In this economy aggregate expenditures: A) do not change as GDP increases. C) increase by $2 for every $4 increase in GDP. B) increase by $2 for every $5 increase in GDP. D) increase by $2 for every $3 increase in GDP. Answer: D

Type: G Topic: 2 E: 177 MA: 177 39. Refer to the above diagram for a private closed economy. Aggregate saving in this economy will be zero when: A) C + Ig cuts the 45-degree line. C) GDP is $60 billion. B) GDP is $180 billion. D) GDP is also zero. Answer: C

Type: D Topic: 2 E: 174 MA: 174 40. For a private closed economy aggregate expenditures consist of: A) C + Ig. B) C - Ig. C) C + S. D) C - S. Answer: A

Use the following to answer questions 41-43: (Advanced analysis) Answer the next question(s) on the basis of the following consumption and investment data for a private closed economy. Figures are in billions of dollars.

C = 60 + .6Y I = I0 = 30

Type: E Topic: 2 E: 175, 191 MA: 175, 191 41. Refer to the above data. The equilibrium level of income (Y ) is: A) 360. B) 225. C) 200. D) 135. Answer: B

Type: E Topic: 2 E: 175, 191 MA: 175, 191 42. Refer to the above data. In equilibrium the level of consumption spending will be: A) 170. B) 270. C) 160. D) 195. Answer: D

Type: E Topic: 2 E: 175, 191 MA: 175, 191 43. Refer to the above data. In equilibrium the level of saving will be: A) 30. B) 26. C) 25. D) 60. Answer: A

McConnell/Brue: Economics, 16/e

Page 297

Chapter 10: The Aggregate Expenditures Model

Use the following to answer questions 44-46: (Advanced analysis) Answer the next question(s) on the basis of the following data. The letters Y, C, S, and I are used to represent GDP, consumption, saving, and investment respectively.
GDP(Y) $ 0 100 200 300 400 500 Consumption(C) $ 60 120 180 240 300 360 Investment(I) $ 30 40 50 60 70 80

Type: E Topic: 2 E: 175 MA: 175 44. The equation representing the consumption schedule for the above economy is: A) C = Y - .6S. B) Y = C + S. C) C = 60 + .4Y . D) C = 60 + .6Y . Answer: D

Type: E Topic: 2 E: 175 MA: 175 45. The equation representing the investment schedule for the above economy is: A) I = .3Y . B) I = 80 -.3Y . C) I = 30 + .1Y . D) I = I0 = 30. Answer: C

Type: E Topic: 2 E: 175 MA: 175 46. Refer to the above data. Equilibrium Y (= GDP) is: A) $100. B) $200. C) $300. D) $400. Answer: C

Type: A Topic: 2 E: 175 MA: 175 47. When investment remains the same at each level of GDP in a private closed economy, the slope of the aggregate expenditures schedule: A) exceeds the MPC. B) is less than the MPC. C) equals the MPS. D) equals the MPC. Answer: D

Type: C Topic: 2 E: 154 MA: 154 48. Actual investment is $62 billion at an equilibrium output level of $620 billion in a private closed economy. The average propensity to save at this level of output is: A) .10. B) 10. C) .62. D) .84. Answer: A

McConnell/Brue: Economics, 16/e

Page 298

Chapter 10: The Aggregate Expenditures Model

Use the following to answer questions 49-54:

Type: G Topic: 2 E: 175 MA: 175 49. Refer to the above diagram for a private closed economy. The MPC and MPS are: A) .6 and .4 respectively. B) .7 and .3 respectively. C) both .5. D) both .7. Answer: C

Type: G Topic: 2 E: 174-175 MA: 174-175 50. Refer to the above diagram for a private closed economy. Gross investment: A) is positively related to the level of GDP. B) is negatively related to the level of GDP. C) is independent of the level of GDP. D) must be subtracted from consumption to determine aggregate expenditures. Answer: C

Type: G Topic: 2 E: 175-176 MA: 175-176 51. Refer to the above diagram for a private closed economy. At the $200 level of GDP: A) consumption is $200 and planned investment is $50 so that aggregate expenditures are $250. B) consumption is $200 and planned investment is $100 so that aggregate expenditures are $300. C) consumption is $250 and actual investment is $50 so that aggregate expenditures are $300. D) aggregate expenditures fall short of GDP with the result that GDP will decline. Answer: A

Type: G Topic: 2 E: 175-176 MA: 175-176 52. Refer to the above diagram for a private closed economy. At the $400 level of GDP: A) aggregate expenditures exceed GDP with the result that GDP will rise. B) consumption is $350 and planned investment is zero so that aggregate expenditures are $350. C) consumption is $300 and planned investment is $50 so that aggregate expenditures are $350. D) consumption is $300 and actual investment is $100 so that aggregate expenditures are $400. Answer: C

McConnell/Brue: Economics, 16/e

Page 299

Chapter 10: The Aggregate Expenditures Model

Type: G Topic: 2 E: 175-176 MA: 175-176 53. Refer to the above diagram for a private closed economy. At the $300 level of GDP: A) aggregate expenditures and GDP are equal. B) consumption is $200 and planned investment is $50. C) saving exceeds planned investment. D) consumption plus saving is $400. Answer: A

Type: G Topic: 2 E: 154 MA: 154 54. Refer to the above diagram for a private closed economy. At the equilibrium level of GDP the APC and APS: A) are 5/6 and 1/6 respectively. B) are equal to the MPC and MPS respectively. C) are 4/5 and 1/5 respectively. D) cannot be determined from the information given. Answer: A

Type: A Topic: 2 E: 177 MA: 177 55. If unintended increases in business inventories occur, we can expect: A) a decline in GDP and rising unemployment. C) an increase in consumption. B) inflation. D) an offsetting increase in planned investment. Answer: A

Use the following to answer questions 56-58: (Advanced analysis) Answer the next question(s) on the basis of the following information for a private closed economy, where Ig is gross investment, S is saving, and Y is gross domestic product (GDP). _ Ig = Ig = 80 S = -80 + .4Y

Type: E Topic: 2 E: 175-176 MA: 175-176 56. Refer to the above information. The equilibrium GDP will be: A) $160. B) $400. C) $360. D) $480. Answer: B

Type: E Topic: 2 E: 175-176 MA: 175-176 57. Refer to the above information. In equilibrium consumption will be: A) $400. B) $280. C) $320. D) $360. Answer: C

Type: E Topic: 2 E: 175-176 MA: 175-176 58. Refer to the above information. In equilibrium saving will be: A) $40. B) $120. C) $60. D) $80. Answer: D

McConnell/Brue: Economics, 16/e

Page 300

Chapter 10: The Aggregate Expenditures Model

Use the following to answer questions 59-61: (Advanced analysis) Answer the next question(s) on the basis of the following information for a private closed economy.

S = -20 + .4Y Ig = 25 - 3i where S is saving, Ig is gross investment, i is the real interest rate, and Y is GDP.

Type: E Topic: 2 E: 175-176 MA: 175-176 59. Refer to the above information. If the real interest rate is 5 percent, investment will be: A) $10 and the equilibrium GDP will be $75. C) $10 and the equilibrium GDP will be $120. B) $15 and the equilibrium GDP will be $100. D) $15 and the equilibrium GDP will be $180. Answer: A

Type: E Topic: 2 E: 175-176 MA: 175-176 60. Refer to the above information. In equilibrium the level of saving will be: A) $10. B) $15. C) $20. D) $30. Answer: A

Type: E Topic: 2 E: 175-176 MA: 175-176 61. Refer to the above information. In equilibrium the level of consumption will be: A) $80. B) $95. C) $65. D) $70. Answer: C

Type: A Topic: 2 E: 177 MA: 177 62. In a private closed economy _____ investment is equal to saving at all levels of GDP and equilibrium occurs only at that level of GDP where _____ investment is equal to saving. A) planned; actual B) actual; planned C) gross; net D) net; gross Answer: B

Type: E Topic: 2 E: 175-176 MA: 175-176 63. (Advanced analysis) If S = - 60 + .25Y and Ig = Ig = 60, where S is saving, Ig is gross investment, and Y is gross domestic product (GDP), then the equilibrium level of GDP is: A) $200. B) $320. C) $360. D) $480. Answer: D

Type: A Topic: 2 E: 175-176 MA: 175-176 64. In the aggregate expenditures model, equilibrium GDP in a private closed economy is indicated by: A) the equality of saving and planned investment. B) the intersection of aggregate expenditures and the 45-degree line. C) the absence of unplanned changes in inventories. D) all of the above. Answer: D

McConnell/Brue: Economics, 16/e

Page 301

Chapter 10: The Aggregate Expenditures Model

Type: C Topic: 2 E: 178 MA: 178 65. In the aggregate expenditures model, technological progress will shift the investment schedule: A) downward and increase aggregate expenditures. B) downward and decrease aggregate expenditures. C) upward and increase aggregate expenditures. D) upward and decrease aggregate expenditures. Answer: C

Type: A Topic: 2 E: 175-176 MA: 175-176 66. In the aggregate expenditures model, the level of GDP moves toward an equilibrium because: A) the investment schedule is steeper than the saving schedule. B) the 45-degree line is steeper than the aggregate expenditures schedule. C) the saving schedule is steeper than the consumption schedule. D) wages and prices are flexible downward. Answer: B

Type: D Topic: 2 E: 175 MA: 175 67. At equilibrium real GDP in a private closed economy: A) the MPC must equal the APC. B) the slope of the aggregate expenditures schedule equals the MPS. C) aggregate expenditures and real GDP are equal. D) planned saving and consumption are equal. Answer: C

Use the following to answer questions 68-69: (Advanced analysis) Answer the next question(s) on the basis of the following information for a private closed economy where C is consumption, Y is the gross domestic product, Ig is gross investment, and i is the interest rate:

C = 40 + .8Y Ig = 60 - 2i _ i = i = 10

Type: E Topic: 2 E: 174-176 MA: 174-176 68. Refer to the above information. Given that the interest rate is 10 (percent), the amount that businesses will want to invest will be: A) $58. B) $60. C) $40. D) $20. Answer: C

Type: E Topic: 2 E: 174-176 MA: 174-176 69. Refer to the above information. The equilibrium level of GDP in this economy is: A) $240. B) $300. C) $360. D) $400. Answer: D

McConnell/Brue: Economics, 16/e

Page 302

Chapter 10: The Aggregate Expenditures Model

Type: A Topic: 2 E: 177 MA: 177 70. What will be the effect of an excess of planned investment over saving in a private closed economy with unemployed resources? A) a decline in the rate of interest B) an unintended accumulation of inventories by businesses C) a rise in the real GDP D) the Federal budget will automatically move toward a deficit Answer: C

Type: A Topic: 2 E: 177 MA: 177 71. Which of the following statements is correct for a private closed economy? A) Saving equals planned investment only at the equilibrium level of GDP. B) All levels of GDP where planned investment exceeds saving will be too high for equilibrium. C) Planned and actual investment are identical at all possible levels of GDP. D) Saving equals actual investment only at the equilibrium level of GDP. Answer: A

Type: A Topic: 2 E: 177 MA: 177 72. At the $180 billion equilibrium level of income, saving is $38 billion in a private closed economy. Planned investment must be: A) $138 billion. B) $126 billion. C) $38 billion. D) $180 billion. Answer: C

Type: A Topic: 2 E: 177 MA: 177 73. In which of the following situations for a private closed economy will the level of GDP expand? A) when planned investment exceeds saving C) when saving exceeds consumption B) when planned investment exceeds consumption D) when consumption exceeds investment Answer: A

Type: A Topic: 2 E: 176-177 MA: 176-177 74. Planned investment plus unintended increases in inventories equals: A) actual investment. C) consumption minus saving. B) consumption of fixed capital. D) unintended saving. Answer: A

Type: A Topic: 2 E: 177 MA: 177 75. Saving is always equal to: A) planned investment less unintended increases in inventories. B) actual investment. C) planned investment. D) unintended changes in inventories. Answer: B

McConnell/Brue: Economics, 16/e

Page 303

Chapter 10: The Aggregate Expenditures Model

Type: D Topic: 2 E: 177 MA: 177 76. Actual investment is: A) gross investment less replacement investment. B) the ratio of planned investment to unintended increases in inventories. C) unintended increases in inventories less planned investment. D) planned investment plus unintended increases in inventories. Answer: D

Type: A Topic: 2 E: 177 MA: 177 77. Actual investment equals saving: A) at all levels of GDP. B) at all below-equilibrium levels of GDP. Answer: A

C) at all above-equilibrium levels of GDP. D) only at the equilibrium GDP.

Type: A Topic: 2 E: 177 MA: 177 78. Planned investment equals saving: A) at all levels of GDP. B) at all below-equilibrium levels of GDP. Answer: D

C) at all above-equilibrium levels of GDP. D) only at the equilibrium GDP.

Type: A Topic: 2 E: 174, 176-177 MA: 174, 176-177 79. That the economy has achieved aggregate equilibrium is indicated by: A) an equality of saving and planned investment. B) an equality of aggregate expenditures and GDP. C) the absence of unplanned changes in inventories. D) all of the above. Answer: D

Type: A Topic: 2 E: 177 MA: 177 80. Unintended changes in inventories: A) cause the economy to move away from the equilibrium GDP. B) are treated as components of consumption. C) bring actual investment and saving into equality only at the equilibrium level of GDP. D) bring actual investment and saving into equality at all levels of GDP. Answer: D

Type: D Topic: 2 E: 176-177 MA: 176-177 81. Investment and saving are, respectively: A) income and wealth. B) stocks and flows. Answer: C

C) injections and leakages. D) leakages and injections.

Type: C Topic: 2 E: 175-176 MA: 175-176 82. In a private closed economy (a) the marginal propensity to save is 0.25, (b) consumption equals income at $120 billion, and (c) the level of investment is $40 billion. What is the equilibrium level of income? A) $280 billion B) $320 billion C) $262 billion D) $198 billion Answer: A

McConnell/Brue: Economics, 16/e

Page 304

Chapter 10: The Aggregate Expenditures Model

Type: C Topic: 2 E: 175-176 MA: 175-176 83. If the marginal propensity to consume is 0.9 in a private closed economy, a $20 billion decline in investment spending will decrease: A) GDP by $20 billion. C) saving by $20. B) GDP by $100 billion. D) consumption by $200 billion. Answer: C

Type: C Topic: 2 E: 175-176 MA: 175-176 84. Suppose that the level of GDP increased by $100 billion in a private closed economy where the marginal propensity to consume is 0.5. Aggregate expenditures must have increased by: A) $100 billion. B) $50 billion. C) $500 billion. D) $5 billion. Answer: B

Type: E Topic: 2 E: 178 MA: 178 85. (Advanced analysis) Assume the consumption schedule for a private closed economy is C = 40 + 0.75Y , where C is consumption and Y is gross domestic product. The multiplier for this economy is: A) 3. B) 4. C) 5. D) 10. Answer: B

Type: E Topic: 2 E: 178 MA: 178 86. (Advanced analysis) Assume the saving schedule for a private closed economy is S = -20 + 0.2Y , where S is saving and Y is gross domestic product. The multiplier for this economy is: A) 3. B) 4. C) 5. D) 10. Answer: C

Use the following to answer questions 87-90: Answer the next question(s) below on the basis of the following information for a private closed economy:
Gross domestic product $100 200 300 400 500 Consumption $120 180 240 300 360 Expected rate of return 25% 20 15 10 5 Amount of investment $ 0 20 40 60 80

Type: T Topic: 2 E: 172-174 MA: 172-174 87. Refer to the above information. If the real interest rate is 20 percent, the equilibrium GDP will be: A) $100. B) $200. C) $300. D) $400. Answer: B

Type: T Topic: 2 E: 172-174 MA: 172-174 88. Refer to the above information. If the real interest rate is 10 percent, the equilibrium GDP will be: A) $100. B) $200. C) $300. D) $400. Answer: C

McConnell/Brue: Economics, 16/e

Page 305

Chapter 10: The Aggregate Expenditures Model

Type: T Topic: 2 E: 172-174 MA: 172-174 89. Refer to the above information. Your answers to the two previous questions suggest that: A) the interest rate and the equilibrium GDP are directly related. B) the interest rate and the equilibrium GDP are inversely related. C) the interest rate and the equilibrium GDP are unrelated. D) as the interest rate falls, investment also falls. Answer: B

Type: T Topic: 2 E: 178 MA: 178 90. Refer to the above information. The multiplier for this economy is: A) 2. B) 2.5. C) 3. D) 4. Answer: B

Use the following to answer questions 91-93: Answer the next question(s) on the basis of the following information for a private closed economy:

Expected rate of return 15% 12 9 6 3 0

Investment 0 40 80 120 160 200

GDP $100 200 300 400 500 600

Consumption $100 160 220 280 340 400

Type: T Topic: 2 E: 172-174 MA: 172-174 91. Refer to the above information. If the real interest rate is 9 percent, the equilibrium GDP will be: A) $600. B) $500. C) $400. D) $300. Answer: D

Type: T Topic: 2 E: 172-174 MA: 172-174 92. Refer to the above information. In this economy a 3 percentage point decrease in the interest rate will: A) increase equilibrium GDP by $200. C) increase equilibrium GDP by $100. B) increase equilibrium GDP by $50. D) decrease equilibrium GDP by $50. Answer: C

Type: T Topic: 2 E: 178 MA: 178 93. Refer to the above information. The multiplier in this economy is: A) 4. B) 5. C) 2.5. D) 3.5. Answer: C

McConnell/Brue: Economics, 16/e

Page 306

Chapter 10: The Aggregate Expenditures Model

Use the following to answer questions 94-96:

Type: G Topic: 2 E: 175-176 MA: 175-176 94. Refer to the above diagram for a private closed economy. The marginal propensity to consume is: A) GF/GB. B) DA/GB. C) FE/DE. D) FB/0B. Answer: C

Type: G Topic: 2 E: 178 MA: 178 95. Refer to the above diagram for a private closed economy. The upshift of the aggregate expenditures schedule from (C + Ig)1 to (C + Ig) 2 reflects: A) an increase in investment expenditures. C) an increase in the MPC. B) a decrease in consumption expenditures. D) an increase in the APS. Answer: A

Type: G Topic: 2 E: 178 MA: 178 96. Refer to the above diagram for a private closed economy. The multiplier is: A) GF/DE. B) GF/GB. C) FE/GF. D) AB/GF. Answer: D

Open economy

Type: A Topic: 3 E: 179 MA: 179 97. Inports have the same effect on the current size of GDP as: A) exports. B) investment. C) consumption. D) saving. Answer: D

Type: A Topic: 3 E: 179 MA: 179 98. Exports have the same effect on the current size of GDP as: A) imports. B) investment. C) taxes. D) saving. Answer: B

McConnell/Brue: Economics, 16/e

Page 307

Chapter 10: The Aggregate Expenditures Model

Type: A Topic: 3 E: 179 MA: 179 99. At the equilibrium GDP for an open economy: A) net exports may be either positive or negative. B) imports will always exceed exports. Answer: A

C) exports will always exceed imports. D) exports and imports will be equal.

Type: A Topic: 3 E: 180 MA: 180 100. Other things equal, if a change in the tastes of American consumers causes them to purchase more foreign goods at each level of U.S. GDP: A) unemployment will decrease domestically. C) inflation will occur domestically. B) U.S. GDP will fall. D) U.S. real GDP will rise. Answer: B

Use the following to answer questions 101-102: Complete the following table and answer the next question(s) on the basis of the resulting data. All figures are in billions of dollars.

Domestic output (GDP = PI) $ 200 250 300 350 400 450 500

Aggregate expenditures, closed economy $ 230 270 310 350 390 430 470

Exports $ 30 30 30 30 30 30 30

Imports $ 20 20 20 20 20 20 20

Net exports $____ ____ ____ ____ ____ ____ ____

Aggregate expenditures, open economy $____ ____ ____ ____ ____ ____ ____

Type: T Topic: 3 E: 179 MA: 179 101. If the above economy was closed to international trade, the equilibrium GDP and the multiplier would be: A) $300 and 5. B) $350 and 4. C) $400 and 4. D) $350 and 5. Answer: D

Type: T Topic: 3 E: 179-180 MA: 179-180 102. Refer to the above table. For the open economy the equilibrium GDP and the multiplier are: A) $300 and 2.5. B) $450 and 5. C) $400 and 4. D) $400 and 5. Answer: D

Type: A Topic: 3 E: 180 MA: 180 103. If net exports decline from zero to some negative amount, the aggregate expenditures schedule would: A) shift upward. B) shift downward. C) not move (net exports do not affect aggregate expenditures). D) become steeper. Answer: B

McConnell/Brue: Economics, 16/e

Page 308

Chapter 10: The Aggregate Expenditures Model

Type: A Topic: 3 E: 179 MA: 179 104. If net exports are positive: A) the equilibrium GDP must be greater than the full-employment GDP. B) imports must exceed exports. C) aggregate expenditures are greater at each level of GDP than when net exports are zero or negative. D) some other component of aggregate expenditures must be negative. Answer: C

Type: C Topic: 3 E: 179-180 MA: 179-180 105. An upward shift of the aggregate expenditures schedule might be caused by: A) a decrease in exports, with no change in imports. B) a decrease in imports, with no change in exports. C) an increase in exports, with an equal decrease in investment spending. D) an increase in imports, with no change in exports. Answer: B

Type: A Topic: 3 E: 179 MA: 179 106. Other things equal, an increase in an economy's exports will: A) lower the marginal propensity to import. B) have no effect on domestic GDP because imports will change by an offsetting amount. C) decrease its domestic aggregate expenditures and therefore decrease its equilibrium GDP. D) increase its domestic aggregate expenditures and therefore increase its equilibrium GDP. Answer: D

Type: A Topic: 3 E: 181 MA: 181 107. If the dollar appreciates relative to foreign currencies, we would expect: A) the multiplier to decrease. C) a country's net exports to rise. B) a country's exports and imports to both fall. D) a country's net exports to fall. Answer: D

Type: A Topic: 3 E: 181 MA: 181 108. If a nation imposes tariffs and quotas on foreign products, the immediate effect will be to: A) reduce the rate of domestic inflation. C) increase domestic output and employment. B) increase efficiency in the world economy. D) reduce domestic output and employment. Answer: C

Type: A Topic: 3 E: 179 MA: 179 109. If the multiplier in an economy is 5, a $20 billion increase in net exports will: A) increase GDP by $100 billion. C) decrease GDP by $100 billion. B) reduce GDP by $20 billion. D) increase GDP by $20 billion. Answer: A

McConnell/Brue: Economics, 16/e

Page 309

Chapter 10: The Aggregate Expenditures Model

Use the following to answer questions 110-113: (Advanced analysis) Answer the next question(s) on the basis of the following information for a private open economy:

C = 40 + .8Y _ Ig = Ig = 40 _ X = X = 20 _ M = M = 30

Type: E Topic: 3 E: 180 MA: 180 110. The equilibrium GDP (=Y ) in the above economy is: A) $200. B) $245. C) $320. D) $350. Answer: D

Type: E Topic: 3 E: 180 MA: 180 111. Refer to the above information. In equilibrium, saving is: A) $20. B) $30. C) $40. D) $50. Answer: B

Type: E Topic: 3 E: 180 MA: 180 112. Refer to the above information. This nation is incurring: A) a trade surplus. B) balance in its international trade. Answer: C

C) a trade deficit.

D) unemployment.

Type: A Topic: 3 E: 181 MA: 181 113. Refer to the above information. International trade in this case: A) has an expansionary effect on GDP. C) has no effect on GDP. B) has a contractionary effect on GDP. D) is causing inflation in this economy. Answer: B

Type: A Topic: 3 E: 180 MA: 180 114. If the equilibrium level of GDP in a private open economy is $1000 billion and consumption is $700 billion at that level of GDP, then: A) saving must be $300 billion. C) S + C must equal $300 billion. B) net exports must be $300 billion. D) Ig+ Xn must equal $300 billion. Answer: D

Type: D Topic: 3 E: 181 MA: 181 115. An exchange rate: A) is the ratio of the dollar volume of a nation's exports to the dollar volume of its imports. B) measures the interest rate ratios of any two nations. C) is the amount that one nation must export to obtain $1 worth of imports. D) is the price at that the currencies of any two nations exchange for one another. Answer: D

McConnell/Brue: Economics, 16/e

Page 310

Chapter 10: The Aggregate Expenditures Model

Type: A Topic: 3 E: 181 MA: 181 116. If the United States wants to increase its net exports, it might take steps to: A) increase its GDP. C) decrease the dollar price of foreign currencies. B) reduce existing tariffs and import quotas. D) increase the dollar price of foreign currencies. Answer: D

Type: A Topic: 3 E: 181 MA: 181 117. Other things equal, serious recession in the economies of U.S. trading partners will: A) have no perceptible impact on the U.S. economy. B) cause inflation in the U.S. economy. C) depress real output and employment in the U.S. economy. D) stimulate real output and employment in the U.S. economy. Answer: C

Use the following to answer questions 118-123:

Type: G Topic: 3 E: 179-180 MA: 179-180 118. Refer to the above diagram. If (C + Ig) are the private expenditures in the closed economy and Xn2 are the net exports in the open economy: A) exports are negative. C) net exports are negative. B) net exports are positive. D) exports are positive. Answer: B

Type: G Topic: 3 E: 180 MA: 180 119. Refer to the above diagram. If net exports are Xn2, the GDP in the open economy will exceed GDP in the closed economy by: A) AB. B) AD. C) FG. D) BD. Answer: D

Type: G Topic: 3 E: 180 MA: 180 120. Refer to the above diagram. The multiplier in this economy is: A) 0E/0A. B) BD/FG. C) FG/BD. D) BD/AD. Answer: B

McConnell/Brue: Economics, 16/e

Page 311

Chapter 10: The Aggregate Expenditures Model

Type: G Topic: 3 E: 179-180 MA: 179-180 121. Refer to the above diagram. If aggregate expenditures in this economy are (C + Ig + Xn2), then the equilibrium levels of GDP and aggregate expenditures respectively will be: A) 0A and 0E. B) 0B and 0F. C) 0A and AH . D) 0D and DJ . Answer: D

Type: G Topic: 3 E: 181 MA: 181 122. Refer to the above diagram. The change in aggregate expenditures as shown from (C + Ig + Xn2) to (C + Ig + Xn 1) might be caused by: A) an appreciation of this nation's currency relative to the currencies of its trading partners. B) a depreciation of this nation's currency relative to the currencies of its trading partners. C) a decrease in this nation's price level relative to price levels abroad. D) a rightward shift in this nation's 45-degree line. Answer: A

Type: G Topic: 3 E: 181 MA: 181 123. Refer to the above diagram. The change in aggregate expenditures as shown from (C + Ig + Xn1) to (C + Ig + Xn 2) will produce: A) a decrease in real GDP. B) an inflationary gap if 0D is this nation's full-employment level of GDP. C) an increase in real GDP if 0B is this nation's full-employment level of GDP. D) an inflationary gap if 0B is this nation's full-employment level of GDP. Answer: D

Use the following to answer questions 124-125: (Advanced analysis) Answer the next question(s) on the basis of the following information for a private open economy. The letters Y , C, Ig, X , and M stand for GDP, consumption, gross investment, exports, and imports respectively. Figures are in billions of dollars.

C = 26 + .75Y Ig = 60 X = 24 M = 10

Type: E Topic: 3 E: 179-181 MA: 179-181 124. The equilibrium GDP for the above open economy is: A) $390. B) $375. C) $320. D) $400. Answer: D

Type: G Topic: 3 E: 179-181 MA: 179-181 125. The multiplier for the above economy is: A) 4.60. B) 3.33. C) 5.00. D) 4.00. Answer: D

McConnell/Brue: Economics, 16/e

Page 312

Chapter 10: The Aggregate Expenditures Model

Mixed economy

Type: D Topic: 4 E: 182 MA: 182 126. In a mixed open economy the equilibrium GDP exists where: A) Ca + Ig + Xn intersects the 45-degree line. C) Ca + Ig + Xn + G = GDP. B) Ca + Ig = Sa + T + X . D) Ca + Ig + Xn = Sa + T . Answer: C

Type: D Topic: 4 E: 185 MA: 185 127. In a mixed open economy the equilibrium GDP is determined at that point where: A) Sa + M + T = Ig + X + G. B) the 45-degree line and the saving schedule intersect. C) Sa + X + G = Ig + T . D) Sa + Ig + X = G + T . Answer: A

Type: C Topic: 4 E: 185 MA: 185 128. Suppose that a mixed open economy is producing at its equilibrium income and that net exports are zero. If at the equilibrium income the public sector's budget shows a surplus: A) Ca + Ig + Xn + G must exceed GDP. C) a recessionary gap must exist. B) planned investment must exceed saving. D) saving must exceed planned investment. Answer: B

129.

Type: C Topic: 4 E: 182 MA: 182 Other things equal, if $100 billion of government purchases (G) is added to private spending (C + Ig + Xn), GDP will: A) increase by $100 billion. C) increase by more than $100 billion. B) increase by less than $100 billion. D) fall by $100 billion Answer: C

Type: C Topic: 4 E: 182 MA: 182 130. Suppose the economy's multiplier is 2. Other things equal, a $25 billion decrease in government expenditures on national defense will cause equilibrium GDP to: A) decrease by $50 billion. B) decrease by $150 billion. C) remain unchanged since spending on military goods is unproductive and usually wasteful. D) decrease by $25 billion. Answer: A

Type: A Topic: 4 E: 183 MA: 183 131. Assume the MPC is .8. If government were to impose $50 billion of new taxes on household income, consumption spending would decrease by: A) $100 billion. B) $90 billion. C) $40 billion D) $50 billion. Answer: C

McConnell/Brue: Economics, 16/e

Page 313

Chapter 10: The Aggregate Expenditures Model

Use the following to answer questions 132-134:

Type: G Topic: 4 E: 183 MA: 183 132. Refer to the above diagram. The level of government spending: A) is equal to tax collections at each level of GDP. C) varies inversely with the level of GDP. B) is the same at all levels of GDP. D) varies directly with the level of GDP. Answer: B

Type: G Topic: 4 E: 183 MA: 183 133. Refer to the above diagram. The sizes of the multipliers associated with changes in investment and government spending in this economy are: A) 2.5 and 1.5 respectively. B) 3 and 2 respectively. C) both 2.5. D) 2 and 3 respectively. Answer: C

Type: G Topic: 4 E: 183 MA: 183 134. Refer to the above diagram. The impact of the public sector on the equilibrium GDP: A) is expansionary. B) is contractionary. C) is neutral. D) cannot be determined from the information given. Answer: A

Type: A Topic: 4 E: 182 MA: 182 135. Other things equal, the multiplier effect associated with a change in government spending is: A) the same as that associated with a change in taxes. B) equal to that associated with a change in investment or consumption. C) less than that associated with a change in investment. D) greater than that associated with a change in investment. Answer: B

Type: A Topic: 4 E: 185 MA: 185 136. In which of the following situations for a mixed open economy will the level of GDP expand? A) when Ig + X + G exceeds Sa + M + T C) when GDP exceeds Ca + Ig + G + Xn B) when Sa + T + M exceeds Ig + G + X D) when Ig + M + T exceeds Ca + X + S Answer: A

McConnell/Brue: Economics, 16/e

Page 314

Chapter 10: The Aggregate Expenditures Model

Type: A Topic: 4 E: 183 MA: 183 137. If a lump-sum income tax of $25 billion is levied and the MPS is 0.20, the: A) saving schedule will shift upward by $5 billion. B) consumption schedule will shift downward by $25 billion. C) consumption schedule will shift downward by $20 billion. D) consumption schedule will shift upward by $25 billion. Answer: C

Type: A Topic: 4 E: 183 MA: 183 138. Which of the following statements is incorrect? A) Given the economy's MPS, a $15 billion reduction in government spending will reduce the equilibrium GDP by more than would a $15 billion increase in taxes. B) Other things unchanged, a tax reduction of $10 billion will increase the equilibrium GDP by $25 billion when the MPS is 0.4. C) If the MPC is 0.8 and GDP has declined by $40 billion, this was caused by a decline in aggregate expenditures of $8 billion. D) A government surplus is anti-inflationary; a government deficit is expansionary. Answer: B

Type: C Topic: 4 E: 184-185 MA: 184-185 139. Suppose the economy is operating at its full-employment-noninflationary GDP and the MPC is 0.75. The Federal government now finds that it must increase spending on military goods by $21 billion in response to a deterioration in the international political situation. To sustain full-employment-noninflationary GDP government must: A) reduce taxes by $28 billion. C) increase taxes by $21 billion. B) reduce transfer payments by $21 billion. D) increase taxes by $28 billion. Answer: D

Type: A Topic: 4 E: 184-185 MA: 184-185 140. A $1 increase in government spending on goods and services will have a greater impact on the equilibrium GDP than will a $1 decline in taxes because: A) government spending is more employment-intensive than is either consumption or investment spending. B) government spending increases the money supply and a tax reduction does not. C) a portion of a tax cut will be saved. D) taxes vary directly with income. Answer: C

Type: A Topic: 4 E: 183 MA: 183 141. The multiplier associated with a change in government purchases is: A) always equal to 1. B) smaller than that associated with an equal change in taxes. C) the same as that associated with a change in investment. D) less than that associated with a change in investment. Answer: C

McConnell/Brue: Economics, 16/e

Page 315

Chapter 10: The Aggregate Expenditures Model

Type: A Topic: 4 E: 185 MA: 185 142. In a mixed open economy, if aggregate expenditures exceed GDP: A) Ig + X + G = Ca. C) Ig > S . B) Ca + Ig + Xn + G < domestic output. D) Ig + X + G > Sa + M + T . Answer: D

Type: A Topic: 4 E: 182 MA: 182 143. Ignoring international trade, in a mixed economy aggregate expenditures are comprised of: A) Ca + S + G. B) Ca + Ig + G. C) Ca + S + Ig. D) Ca + T + Ig. Answer: B

Type: A Topic: 4 E: 184-185 MA: 184-185 144. An increase in taxes of a specific amount will have a smaller impact on the equilibrium GDP than will a decline in government spending of the same amount because: A) the MPC is smaller in the private sector than it is in the public sector. B) declines in government spending always tend to stimulate private investment. C) disposable income will fall by some amount smaller than the tax increase. D) some of the tax increase will be paid out of income that would otherwise have been saved. Answer: D

Type: A Topic: 4 E: 184-185 MA: 184-185 145. If APC = .6 and MPC = .7, the immediate impact of an increase in personal taxes of $20 will be to: A) have no effect on consumption. C) decrease consumption by $12. B) decrease consumption by $14. D) increase consumption by $14. Answer: B

Use the following to answer questions 146-149: The following schedule contains data for a private closed economy. All figures are in billions. Use these data in answering the next question(s).
GDP $140 180 220 260 300 C $150 180 210 240 270

Type: T Topic: 4 E: 173-174 MA: 173-174 146. Refer to the above data. If gross investment is $10 at all levels of GDP, the equilibrium GDP will be: A) $300. B) $260. C) $220. D) $180. Answer: C

McConnell/Brue: Economics, 16/e

Page 316

Chapter 10: The Aggregate Expenditures Model

Type: T Topic: 4 E: 183 MA: 183 147. Refer to the above data. If a lump-sum tax of $20 is imposed, the consumption schedule will become:

(a) GDP $120 160 200 240 280 Answer: C C $150 180 210 240 270 GDP $140 180 220 260 300

(b) C $155 185 215 245 275 GDP $140 180 220 260 300

(c) C $135 165 195 225 255 GDP $140 180 220 260 300

(d) C $130 160 190 220 250

Type: T Topic: 4 E: 183 MA: 183 148. Refer to the above data. If gross investment remains at $10 at all levels of GDP, the after-tax equilibrium level of GDP will be: A) $220. B) $190. C) $180. D) $160. Answer: D

Type: T Topic: 4 E: 184-185 MA: 184-185 149. Refer to the above data. Given the levels of investment and taxes already specified, the addition of governmental expenditures of $10 at each level of GDP will result in an equilibrium GDP of: A) $235. B) $220. C) $200. D) $180. Answer: C

Type: A Topic: 4 E: 183 MA: 183 150. Which of the following is a correct statement of the impacts of a lump-sum tax? A) Disposable income will increase by the amount of the tax and consumption at each level of GDP will decline by the amount of the tax multiplied by the MPC. B) Disposable income will decline by the amount of the tax and consumption at each level of GDP will decline by the amount of the tax multiplied by the multiplier. C) Disposable income will decline by the amount of the tax and consumption at each level of GDP will also decline by the amount of the tax. D) Disposable income will decline by the amount of the tax and consumption at each level of GDP will decline by the amount of the tax multiplied by the MPC. Answer: D

Type: A Topic: 4 E: 185 MA: 185 151. When the public sector is added to the aggregate expenditures model: A) the equilibrium condition becomes G + S = T + Ig + X . B) the equilibrium condition becomes G + T = S + Ig + X . C) the equilibrium condition becomes Ca + Ig + Xn + G + T = GDP. D) we add a new leakage in the form of taxes and a new injection in the form of government spending. Answer: D

Type: D Topic: 4 E: 182 MA: 182 152. The level of aggregate expenditures in a mixed open economy is comprised of: A) Ca + Ig + Xn B) Ca + Ig + G + T + Xn . C) Ca + Ig + Xn + G. D) Ca + G. Answer: C

McConnell/Brue: Economics, 16/e

Page 317

Chapter 10: The Aggregate Expenditures Model

Type: A Topic: 4 E: 183 MA: 183 153. If the MPC is 2/3, the initial impact of an increase of $12 billion in lump-sum taxes will be to cause: A) a rightward shift in the investment demand schedule. B) an $8 billion downshift in the consumption schedule. C) a $4 billion upshift in the consumption schedule. D) a $12 billion downshift in the consumption schedule. Answer: B

Type: A Topic: 4 E: 185 MA: 185 154. In a mixed closed economy: A) government purchases and saving are injections, while investment and taxes are leakages. B) taxes and government purchases are leakages, while investment and saving are injections. C) taxes and savings are leakages, while investment and government purchases are injections. D) taxes and investment are injections, while saving and government purchases are leakages. Answer: C

Type: A Topic: 4 E: 184-185 MA: 184-185 155. An increase in taxes will have a greater effect on the equilibrium GDP: A) if the tax revenues are redistributed through transfer payments. B) the larger the MPS. C) the smaller the MPC. D) the larger the MPC. Answer: D

Type: A Topic: 4 E: 183 MA: 183 156. A lump-sum tax causes the after-tax consumption schedule: A) and the before-tax consumption schedule to coincide. B) to be steeper than the before-tax consumption schedule. C) to be flatter than the before-tax consumption schedule. D) to be parallel to the before-tax consumption schedule. Answer: D

Use the following to answer questions 157-160: (Advanced analysis) Answer the next question(s) on the basis of the following information for a mixed open economy. The letters Y , Ca, Ig, Xn, G , and T stand for GDP, consumption, gross investment, net exports, government purchases, and net taxes respectively. Figures are in billions of dollars.

Ca = 25 + .75(Y - T ) Ig = Ig0 = 50 Xn = Xn0 = 10 G = G0 = 70 T = T0 = 30

Type: A Topic: 4 E: 185 MA: 185 157. Refer to the above information. The equilibrium level of GDP for this economy is: A) $600. B) $530. C) $415. D) $400. Answer: B

McConnell/Brue: Economics, 16/e

Page 318

Chapter 10: The Aggregate Expenditures Model

Type: A Topic: 4 E: 185 MA: 185 158. Refer to the above information. The multiplier for this economy is: A) 4. B) 3. C) 2. D) 2.33. Answer: A

Type: A Topic: 4 E: 185 MA: 185 159. Refer to the above information. If government desired to raise the equilibrium GDP to $650, it could: A) raise G by $45 and reduce T by $10. C) raise G by $30 or reduce T by $40. B) raise G by $40 and reduce T by $30. D) raise both G and T by $40. Answer: C

Type: A Topic: 4 E: 185 MA: 185 160. Refer to the above information. If the economy's tax schedule was T = 0.2Y rather than T = T0 = 30, the equilibrium GDP would be: A) $387.5. B) $518.5. C) $316. D) $412. Answer: A

Type: A Topic: 4 E: 184-185 MA: 184-185 161. Which of the following would increase GDP by the greatest amount? A) a $20 billion reduction in taxes B) $20 billion increases in both government spending and taxes C) $20 billion decreases in both government spending and taxes D) a $20 billion increase in government spending Answer: D

Type: A Topic: 4 E: 184-185 MA: 184-185 162. Which of the following is correct? A) Government expenditures and taxes both increase GDP. B) Government expenditures and taxes both decrease GDP. C) Government expenditures increase, but taxes decrease, GDP. D) Government expenditures decrease, but taxes increase, GDP. Answer: C

Type: C Topic: 4 E: 184-185 MA: 184-185 163. Which of the following would reduce GDP by the greatest amount? A) a $20 billion increase in taxes B) $20 billion increases in both government spending and taxes C) $20 billion decreases in both government spending and taxes D) a $20 billion decrease in government spending Answer: D

Type: A Topic: 4 E: 182 MA: 182 164. What do investment and government expenditures have in common? A) both represent injections to the circular flow C) neither is subject to the multiplier effect B) both represent leakages from the circular flow D) both represent a decline in indebtedness Answer: A

McConnell/Brue: Economics, 16/e

Page 319

Chapter 10: The Aggregate Expenditures Model

Type: A Topic: 4 E: 185 MA: 185 165. Taxes represent: A) a leakage of purchasing power, like saving. B) an injection of purchasing power, like investment. C) an injection of purchasing power, like government spending. D) a leakage of purchasing power, like government spending. Answer: A

Type: A Topic: 4 E: 182 MA: 182 166. In moving from a private closed economy to a mixed closed economy in the aggregate expenditures model, government spending must be: A) added to saving. B) added to consumption and gross investment. C) subtracted from consumption and gross investment. D) added to gross investment and saving. Answer: B

Use the following to answer questions 167-171: The following information is for a closed economy:

GDP $100 200 300 400 500 600 700

C $100 160 220 280 340 400 460

S $ 0 40 80 120 160 200 240

Ig $80 80 80 80 80 80 80

Type: T Topic: 4 E: 182 MA: 182 167. Refer to the above information. If both government spending and taxes are zero, the equilibrium level of GDP is: A) $200. B) $300. C) $400. D) $500. Answer: B

Type: T Topic: 4 E: 183 MA: 183 168. Refer to the above information. If government now spends $80 billion at each level of GDP and taxes remain at zero, the equilibrium GDP: A) will rise to $700. B) will rise to $600. C) will rise to $500. D) may either rise or fall. Answer: C

Type: T Topic: 4 E: 183 MA: 183 169. Refer to the above information. The introduction of $80 billion of government spending has: A) lowered the multiplier from 2.5 to 2.0. C) increased the multiplier from 2.0 to 2.5. B) increased the multiplier from 2.5 to 3.0. D) had no effect on the size of the multiplier. Answer: D

McConnell/Brue: Economics, 16/e

Page 320

Chapter 10: The Aggregate Expenditures Model

Type: T Topic: 4 E: 183 MA: 183 170. Refer to the above information. If in addition to spending $80 billion at each level of GDP, government imposes a lump-sum tax of $100: A) equilibrium GDP will now be $350. C) equilibrium GDP will now be $300. B) equilibrium GDP will now be $400. D) the equilibrium GDP cannot be determined. Answer: A

Type: T Topic: 4 E: 183 MA: 183 171. Refer to the above information. The addition of a $100 billion lump-sum tax: A) reduces the MPC and increases the multiplier. B) increases the MPC and decreases the multiplier. C) increases both the MPC and the multiplier. D) has no effect on either the MPC or the multiplier. Answer: D

Type: A Topic: 4 E: 185 MA: 185 172. ln moving from a private closed to a mixed closed economy in the aggregate expenditures model, taxes: A) must be added to gross investment. B) must be added to saving. C) must be added to consumption and gross investment. D) have no impact upon the equilibrium GDP. Answer: B

Type: C Topic: 4 E: 185 MA: 185 173. Suppose government finds it can increase the equilibrium real GDP $45 billion by increasing government purchases by $18 billion. On the basis of this information we can say that the: A) MPS in this economy is .4. B) MPC in this economy is .4. C) balanced-budget multiplier does not apply in this economy. D) multiplier is 3. Answer: A

Use the following to answer questions 174-176: Answer the next question(s) on the basis of the following table:
GDP $ 500 510 520 530 540 550 560 570 Before taxes C S $ 480 $ 20 486 24 492 28 498 32 504 36 510 40 516 44 522 48 After taxes Ca Sa $ 474 $ 16 480 20 486 24 492 28 498 32 504 36 510 40 516 44

Type: T Topic: 4 E: 183-184 MA: 183-184 174. The tax in the above economy is a: A) 10 percent proportional tax. B) lump-sum tax of $20. Answer: C

C) lump-sum tax of $10. D) progressive tax.

McConnell/Brue: Economics, 16/e

Page 321

Chapter 10: The Aggregate Expenditures Model

Type: T Topic: 4 E: 183 MA: 183 175. The MPC and MPS in the above economy: A) are .4 and .6 respectively. B) are .6 and .4 respectively. C) are .8 and .2 respectively. D) cannot be determined from the information given. Answer: B

Type: T Topic: 4 E: 183-184 MA: 183-184 176. Refer to the above table. If an additional lump-sum tax of $20 were imposed, we would expect: A) equilibrium GDP to fall by $30. C) equilibrium GDP to fall by $50. B) equilibrium GDP to fall by $20. D) equilibrium GDP to rise by $24. Answer: A

Type: A Topic: 4 E: 183 MA: 183 177. Suppose the multiplier is 4 and lump-sum taxes are increased by $16 in a closed economy. We can predict that: A) GDP will increase by $64. B) GDP will decrease by $64. C) the aggregate expenditures schedule will shift downward by $12. D) inflation will occur. Answer: C

Type: A Topic: 4 E: 185 MA: 185 178. In a mixed open economy, which of the following all affect the equilibrium GDP in the same direction? A) Ca, Ig, Sa, and M B) Sa, T, and M C) Ig, T, and Ca D) Sa, Ig, and X Answer: B

Type: A Topic: 4 E: 184 MA: 184 179. In the aggregate expenditures model, a reduction in taxes may: A) increase saving. B) decrease real GDP. C) increase unemployment. Answer: A

D) reduce consumption.

Type: A Topic: 4 E: 182 MA: 182 180. In the aggregate expenditures model, an increase in government spending may: A) decrease real GDP. B) increase output and employment. C) shift the aggregate expenditures schedule downward. D) reduce the size of the inflationary gap. Answer: B

Type: C Topic: 4 E: 182 MA: 182 181. If a $20 billion increase in government expenditures increases equilibrium GDP by $50 billion then: A) the multiplier is 2. C) inflation is occurring. B) the MPC for this economy is .6. D) the MPS for this economy is .6. Answer: B

McConnell/Brue: Economics, 16/e

Page 322

Chapter 10: The Aggregate Expenditures Model

Type: C Topic: 4 E: 183 MA: 183 182. If a $10 billion decrease in lump-sum taxes increases equilibrium GDP by $40 billion then: A) the multiplier is 4. C) the MPC for this economy is .6. B) the MPC for this economy is .8. D) the multiplier is 3. Answer: B

Type: A Topic: 4 E: 183 MA: 183 183. A lump-sum tax means that: A) the tax only applies to one time period. B) the same amount of tax revenue is collected at each level of GDP. C) tax revenues vary directly with GDP. D) tax revenues vary inversely with GDP. Answer: B

Type: C Topic: 4 E: 183 MA: 183 184. In an aggregate expenditures diagram, a lump-sum tax (T ) will: A) not affect the C + Ig + Xn line. B) shift the C + Ig + Xn line upward by an amount equal to T . C) shift the C + Ig + Xn line downward by an amount equal to T . D) shift the C + Ig + Xn line downward by an amount equal to T × MPC. Answer: D

Type: A Topic: 4 E: 183 MA: 183 185. If government increases lump-sum taxes by $20 billion and the economy's MPC is .6, then the: A) consumption schedule will shift upward by $12 billion. B) consumption schedule will shift downward by $12 billion. C) equilibrium GDP will increase by $40 billion. D) equilibrium GDP will decrease by $40 billion. Answer: B

Type: A Topic: 4 E: 183 MA: 183 186. The effect of imposing a lump-sum tax is to: A) reduce the absolute levels of consumption and saving at each level of GDP and to reduce the size of the multiplier. B) reduce the absolute levels of consumption and saving at each level of GDP, but to not change the size of the multiplier. C) reduce the absolute levels of consumption and saving at each level of GDP and to increase the size of the multiplier. D) increase the absolute levels of consumption and saving at each level of GDP and to increase the size of the multiplier. Answer: B

Type: C Topic: 4 E: 185 MA: 185 187. Suppose that unintended increases in inventories are occurring in a mixed closed economy. We can surmise that: A) Ig + T > Sa + G. B) T + G > Sa + Ig. C) T + Sa > Ig + G. D) T + Sa < Ig + G. Answer: C

McConnell/Brue: Economics, 16/e

Page 323

Chapter 10: The Aggregate Expenditures Model

Type: C Topic: 4 E: 183 MA: 183 188. If a lump-sum tax of $40 billion is imposed and the MPC is 0.6, the saving schedule will shift: A) downward by $24 billion. C) downward by $16 billion. B) upward by $24 billion. D) upward by $16 billion. Answer: C

Type: C Topic: 4 E: 183 MA: 183 189. If the MPC in an economy is .75, a $1 billion increase in taxes will ultimately reduce consumption by: A) $1 billion. B) $.75 billion. C) $3 billion. D) $4 billion. Answer: C

Type: C Topic: 4 E: 182 MA: 182 190. If the MPC in an economy is .9, a $1 billion increase in government spending will ultimately increase consumption by: A) $1 billion. B) $.9 billion. C) $10 billion. D) $9 billion. Answer: D

Type: A Topic: 4 E: 183 MA: 183 191. If the marginal propensity to save in a closed economy is 0.25 and a lump-sum tax is imposed, the slope of the economy's aggregate expenditures schedule will be: A) .25. B) less than the slope before the tax. C) greater than the slope before the tax. D) .75. Answer: D

Type: A Topic: 4 E: 182 MA: 182 192. If the marginal propensity to consume in an economy is 0.8, net exports are zero, and government spending is $33 billion at each level of real GDP, the slope of the economy's aggregate expenditures schedule will be: A) .8. B) .2. C) 5. D) .125. Answer: A

Type: A Topic: 4 E: 184-185 MA: 184-185 193. If MPC = .5, a simultaneous increase in both taxes and government spending of $20 will: A) decrease GDP by $20. C) increase GDP by $20. B) decrease GDP by $40. D) increase GDP by $40. Answer: C

Type: A Topic: 4 E: 182 MA: 182 194. If government increases its purchases by $15 billion and the MPC is 2/3, then we would expect the equilibrium GDP to: A) increase by $30 billion. C) decrease by $35 billion. B) increase by $45 billion. D) increase by $50 billion. Answer: B

McConnell/Brue: Economics, 16/e

Page 324

Chapter 10: The Aggregate Expenditures Model

Type: A Topic: 4 E: 182 MA: 182 195. If government increases its tax revenues by $15 billion and the MPC is 2/3, then we can expect the equilibrium GDP to: A) decrease by $30 billion. C) decrease by $35 billion. B) decrease by $45 billion. D) decrease by $55 billion. Answer: A

Type: A Topic: 4 E: 184-185 MA: 184-185 196. It is true that: A) equal increases in government spending and taxes do not change the equilibrium GDP. B) equal increases in government spending and taxes reduce the equilibrium GDP. C) equal increases in government spending and taxes increase the equilibrium GDP. D) taxes have a stronger effect upon equilibrium GDP than do government purchases. Answer: C

Type: A Topic: 4 E: 184-185 MA: 184-185 197. In an aggregate expenditures diagram equal increases in government spending and in lump-sum taxes will: A) shift the aggregate expenditures line downward. C) not affect the aggregate expenditures line. B) shift the aggregate expenditures line upward. D) reduce the equilibrium GDP. Answer: B

Type: A Topic: 4 E: 185 MA: 185 198. Equal increases in government purchases and taxes will: A) increase the equilibrium GDP and the size of that increase varies directly with the size of the MPC. B) increase the equilibrium GDP and the size of that increase is independent of the size of the MPC. C) increase the equilibrium GDP and the size of that increase varies inversely with the size of the MPC. D) decrease the equilibrium GDP and the size of that decrease is independent of the size of the MPC. Answer: B

Type: C Topic: 4 E: 185 MA: 185 199. Assume in a private closed economy that the equilibrium level of income is $380 and the MPS is 0.25. Now suppose government collects taxes of $50 and spends the entire amount. As a result: A) the equilibrium level of real income and the price level will both remain unchanged. B) nominal wage rates will fall. C) the equilibrium level of income will rise to $420. D) the equilibrium level of income will rise to $430. Answer: D

Recessionary and inflationary gaps

Type: D Topic: 5 E: 187 MA: 187 200. An inflationary gap is the amount by which: A) equilibrium GDP falls short of the full-employment GDP. B) aggregate expenditures exceed any given level of GDP. C) saving exceeds investment at the full-employment GDP. D) aggregate expenditures exceed the full-employment level of GDP. Answer: D

McConnell/Brue: Economics, 16/e

Page 325

Chapter 10: The Aggregate Expenditures Model

Type: D Topic: 5 E: 185 MA: 185 201. A recessionary gap is: A) the amount by which the full-employment GDP exceeds the level of aggregate expenditures. B) the amount by which equilibrium GDP falls short of the full-employment GDP. C) the amount by which investment exceeds saving at the full-employment GDP. D) the amount by which aggregate expenditures exceed the full-employment level of GDP. Answer: A

Use the following to answer questions 202-212:

Real GDP 0 10 40 70 100 130 160

Consumption (after taxes) $ -20 0 20 40 60 80 100

Gross investment $10 10 10 10 10 10 10

Net exports $ +5 +5 +5 +5 +5 +5 +5

Government purchases $15 15 15 15 15 15 15

Type: G Topic: 5 E: 184 MA: 184 202. Refer to the above table. The economy shown is a: A) private economy. B) private open economy. C) mixed closed economy. D) mixed open economy. Answer: D

Type: G Topic: 5 E: 183 MA: 183 203. Refer to the above table. The after-tax MPC in the economy shown is: A) .5. B) .67. C) .75. D) .8. Answer: B

Type: G Topic: 5 E: 183 MA: 183 204. Refer to the above table. The after-tax MPS shown is: A) .1. B) .2. C) .33. D) .4. Answer: C

Type: G Topic: 5 E: 183 MA: 183 205. Refer to the above table. The multiplier is: A) 5. B) 4. C) 3. D) 2. Answer: C

Type: G Topic: 5 E: 182-183 MA: 182-183 206. Refer to the above table. Equilibrium GDP is: A) $40. B) $70. C) $100. D) $130. Answer: B

McConnell/Brue: Economics, 16/e

Page 326

Chapter 10: The Aggregate Expenditures Model

Type: G Topic: 5 E: 187 MA: 187 207. Refer to the above table. If the full-employment real GDP is $100 the: A) inflationary gap is $30. C) recessionary gap is $20. B) inflationary gap is $10. D) recessionary gap is $10. Answer: D

Type: G Topic: 5 E: 187 MA: 187 208. Refer to the above table. If the full-employment real GDP is $40 the: A) inflationary gap is $20. C) recessionary gap is $30. B) inflationary gap is $10. D) recessionary gap is $10. Answer: B

Type: G Topic: 5 E: 185-187 MA: 185-187 209. Refer to the above table. If the full-employment real GDP is $70 the: A) inflationary gap is $30. C) recessionary and inflationary gaps are both $0. B) inflationary gap is $10. D) recessionary gap is $10. Answer: C

Type: G Topic: 5 E: 184 MA: 184 210. Refer to the above table. Exports might be ____ and imports ____. A) $10; $5. B) $10; $0. C) $0; $5. D) $5; 10. Answer: A

Type: G Topic: 5 E: 179 MA: 179 211. Refer to the above table. An increase in net exports of $10 would: A) increase real GDP by $10. C) decrease real GDP by $10. B) increase real GDP by $30. D) decrease real GDP by $30. Answer: B

Type: G Topic: 5 E: 182 MA: 182 212. Refer to the above table. A decrease in government purchases of $5 would: A) increase real GDP by $5. C) decrease real GDP by $5. B) increase real GDP by $10. D) decrease real GDP by $15. Answer: D

McConnell/Brue: Economics, 16/e

Page 327

Chapter 10: The Aggregate Expenditures Model

Use the following to answer questions 213-216:

Type: G Topic: 5 E: 185 MA: 185 213. Refer to the above diagram. If the full-employment level of GDP is B and aggregate expenditures are at AE3, the: A) inflationary gap is BC. C) recessionary gap is ed. B) recessionary gap is BC. D) inflationary gap is ed. Answer: C

Type: G Topic: 5 E: 187 MA: 187 214. Refer to the above diagram. If the full-employment level of GDP is B and aggregate expenditures are at AE1, the: A) inflationary gap is BC. C) inflationary gap is zero. B) recessionary gap is BC. D) inflationary gap is ei. Answer: D

Type: G Topic: 5 E: 185-187 MA: 185-187 215. Refer to the above diagram. If the full-employment level of GDP is B and aggregate expenditures are at AE2, the: A) inflationary gap is ed. C) inflationary gap is eg. B) recessionary gap is BC. D) economy is in equilibrium, at full employment. Answer: D

Type: G Topic: 5 E: 185 MA: 185 216. Refer to the above diagram. The value of the multiplier for this economy is: A) BC/hg. B) BC/AB. C) ed/di. D) df/BC. Answer: A

McConnell/Brue: Economics, 16/e

Page 328

Chapter 10: The Aggregate Expenditures Model

Type: A Topic: 5 E: 185 MA: 185 217. A recessionary gap exists if: A) planned investment exceeds saving at the full-employment GDP. B) the aggregate expenditures schedule lies below the 45-degree line at the full-employment GDP. C) the aggregate expenditures schedule intersects the 45-degree line at any level of GDP. D) the aggregate expenditures schedule lies above the 45-degree line at the full-employment GDP. Answer: B

Type: C Topic: 5 E: 185 MA: 185 218. Assume the current equilibrium level of income is $200 billion as compared to the full-employment income level of $240 billion. If the MPC is 0.625, what change in aggregate expenditures is needed to achieve full employment? A) a decrease of $12 billion C) an increase of $10 billion B) an increase of $25 billion D) an increase of $15 billion Answer: D

Type: A Topic: 5 E: 185 MA: 185 219. Cyclical unemployment in the United States is essentially the consequence of: A) procyclical fiscal policies. C) rapid technological progress. B) a deficient level of aggregate expenditures. D) the geographic immobility of the labor force. Answer: B

Type: A Topic: 5 E: 185 MA: 185 220. If the MPS is .25 and the economy has a recessionary gap of $5 billion, then equilibrium GDP is: A) $5 billion below the full-employment GDP. C) $20 billion below the full-employment GDP. B) $5 billion above the full-employment GDP. D) $20 billion above the full-employment GDP. Answer: C

Type: A Topic: 5 E: 185 MA: 185 221. Which of the following statements concerning the equilibrium level of GDP is incorrect? A) there will be no tendency for businesses to alter the aggregate rate of production B) full employment will necessarily be realized C) no unintended changes in inventories will occur D) leakages equal injections Answer: B

Type: A Topic: 5 E: 185 MA: 185 222. If the economy is in equilibrium at $400 billion of GDP and the full-employment GDP is $500 billion: A) real and nominal GDP will both increase. B) GDP will remain at $400 billion unless aggregate expenditures change. C) real GDP will increase, but nominal GDP will decrease. D) the price level will increase. Answer: B

McConnell/Brue: Economics, 16/e

Page 329

Chapter 10: The Aggregate Expenditures Model

Type: A Topic: 5 E: 185 MA: 185 223. If an increase in aggregate expenditures results in no increase in real GDP we can surmise that the: A) economy is in a deep recession. B) MPC equals 1. C) economy is already operating at full employment. D) price level has fallen. Answer: C

Type: A Topic: 5 E: 185 MA: 185 224. When the level of domestic output is $500 billion, the level of aggregate expenditures: A) may be greater than, less than, or equal to $500 billion. B) must be greater than $500 billion, because investment will occur. C) must be less than $500 billion, because saving will occur. D) must also be $500 billion. Answer: A

Type: C Topic: 5 E: 185 MA: 185 225. If the MPC is .50, all taxes are lump-sum taxes, and the equilibrium GDP is $40 billion below the fullemployment GDP, then the size of the recessionary gap is: A) $40 billion. B) $20 billion. C) $60 billion. D) $80 billion. Answer: B

Applications and critiques

Type: F Topic: 6 E: 187 MA: 187 Status: New 226. The recessionary gap associated with the recession of 2001 resulted from: A) the government's attempt to control hyperinflation. B) a major increase in personal and corporate taxes. C) a rapid decline in investment spending. D) a rapid increase in imports resulting from large tariff reductions. Answer: C

Type: F Topic: 6 E: 186 MA: 186 Status: New 227. Which of the following was not a contributing cause of the decline in investment and thus the recessionary gap occurring during the U.S. recession of 2001? A) overcapacity in major industries B) pessimism relating to the stock market crash C) the collapse of numerous Internet-related start-up firms D) low interest rates Answer: D

Type: A Topic: 6 E: 186-187 MA: 186-187 Status: New 228. The U.S. recession of 2001 provides a good example of: A) demand-pull inflation. C) a recessionary gap. B) cost-push inflation. D) the repercussions of hyperinflation. Answer: C

McConnell/Brue: Economics, 16/e

Page 330

Chapter 10: The Aggregate Expenditures Model

Type: F Topic: 6 E: 187 MA: 187 Status: New 229. During the late 1980s, the U.S. economy experienced: A) cost-push inflation. B) an inflationary gap. C) a recessionary gap. Answer: B

D) cyclical unemployment.

Type: F Topic: 6 E: 187 MA: 187 Status: New 230. The inflationary gap in the United States in the late 1980s was caused by: A) a rapid decline of net exports. C) rising aggregate expenditures. B) a major tax increase. D) cost-push inflationary forces. Answer: C

Type: F Topic: 6 E: 187 MA: 187 Status: New 231. Viewed through the aggregate expenditures model, the U.S. recession of 2001 resulted mainly from: A) a fall in the average propensity to save. C) reduced government spending. B) insufficient aggregate expenditures. D) increased taxes. Answer: B

Type: A Topic: 6 E: 187, 189 MA: 187, 189 232. Which one of the following is not a shortcoming of the aggregate expenditures model? A) failure to show price level changes B) failure to allow for "self-correction" of the economy C) failure to account for cost-push inflation D) failure to account for cyclical unemployment Answer: D

Type: A Topic: 6 E: 189 MA: 189 233. A shortcoming of the aggregate expenditures model is that it does not: A) account for cost-push inflation. C) explain how cyclical unemployment can arise. B) explain how demand-pull inflation can arise. D) detail the components of aggregate spending. Answer: A

Last Word Questions

Type: F E: 188 MA: 188 234. (Last Word) Say's law and classical macroeconomics were disputed by: A) Adam Smith. B) Jeremy Bentham. C) John Stuart Mill. D) John Maynard Keynes. Answer: D

Type: F E: 188 MA: 188 235. (Last Word) Classical macroeconomics was dealt severe blows by: A) the Great Depression and Keynes's macroeconomic theory. B) the Second World War and the writings of Milton Friedman. C) Adam Smith and his idea of the invisible hand. D) the strong recovery after the Second World War and Alvin Hansen's stagnation thesis. Answer: A

McConnell/Brue: Economics, 16/e

Page 331

Chapter 10: The Aggregate Expenditures Model

Type: F E: 188 MA: 188 236. (Last Word) In The General Theory of Employment, Interest, and Money: A) Adam Smith stated his idea of the invisible hand. B) Thorstein Veblen poked fun at the leisure class. C) John Maynard Keynes attacked the classical economist's contention that recession or depression will automatically cure itself. D) J. B. Say developed "Say's law." Answer: C

True/False Questions

Type: G E: 173 MA: 173 Status: New 237. Graphically, the height of the investment schedule depends on the real interest rate, together with the location of the investment demand curve. Answer: True

Type: F E: 174 MA: 174 Status: New 238. In the aggregate expenditure model presented in the textbook, investment is assumed to rise with increases in real GDP and fall with decreases in real GDP. Answer: False

Type: D E: 173-174 MA: 173-174 Status: New 239. In the private closed economy, equilibrium GDP occurs where C + Ig = GDP. Answer: True

Type: F E: 176-177 MA: 176-177 Status: New 240. When C + Ig = GDP in a private closed economy, S = I and there are no unplanned changes in inventories. Answer: True

Type: A E: 175 MA: 175 Status: New 241. If C + Ig exceeds GDP in a private closed economy, GDP will decline. Answer: False

Type: A E: 176-177 MA: 176-177 Status: New 242. If the MPC is .8 in a private closed economy, a $30 billion increase in planned investment will increase equilibrium real GDP by $120 billion. Answer: False

Type: A E: 177 MA: 177 Status: New 243. Actual investment consists of planned investment plus unplanned changes in inventories (plus or minus.) Answer: True

McConnell/Brue: Economics, 16/e

Page 332

Chapter 10: The Aggregate Expenditures Model

Type: A E: 177 MA: 177 Status: New 244. A $20 billion decrease in investment in a private closed economy that has an MPS of .5 will reduce saving by $10 billion once the multiplier process has ended. Answer: False

Type: G E: 179 MA: 179 245. Exports are added to, and imports are subtracted from, aggregate expenditures in moving from a closed to an open economy. Answer: True

Type: G E: 185 MA: 185 246. For an open mixed economy the equilibrium level of GDP is determined where Sa + Ig + X = T + G. Answer: False

Type: A E: 184-185 MA: 184-185 247. Equal increases in government expenditures and tax collections will leave the equilibrium GDP unchanged. Answer: False

Type: A E: 184-185 MA: 184-185 248. A $10 billion decrease in taxes will increase the equilibrium GDP by more than would a $10 billion increase in government expenditures. Answer: False

Type: A E: 183 MA: 183 249. A lump-sum tax causes the after-tax consumption schedule to be flatter than the before-tax consumption schedule. Answer: False

Type: A E: 182 MA: 182 250. If government decreases its purchases by $20 billion and the MPC is 0.8, equilibrium GDP will decrease by $100 billion. Answer: True

Type: A E: 183 MA: 183 251. If the MPC is .9, a $20 billion increase in a lump-sum tax will reduce GDP by $200 billion. Answer: False

Type: A E: 185 MA: 185 252. A recessionary gap in a mixed open economy can be measured as the extent to which aggregate expenditures (Ca + Ig + Xn + G) fall short of real GDP at the full-employment level of real GDP. Answer: True

Type: D E: 185 MA: 185 253. The recessionary gap is the amount by which the equilibrium GDP and the full-employment GDP differ. Answer: False

McConnell/Brue: Economics, 16/e

Page 333

Chapter 10: The Aggregate Expenditures Model

Type: F E: 183 MA: 183 254. The aggregate expenditures schedule in the mixed open economy has a negative slope. Answer: False

Type: A E: 187, 189 MA: 187, 189 255. The aggregate expenditures model does not allow for cost-push inflation. Answer: True

McConnell/Brue: Economics, 16/e

Page 334

CHAPTER 11

Aggregate Demand and Aggregate Supply

Topic 1. 2. 3. 4. 5. Aggregate demand Long-run aggregate supply Aggregate supply (short run) Equilibrium; changes in equilibrium Downward price and wage inflexibility Consider This Last Word True-False

Question numbers 1-22 23-27 28-63 64-125 126-134 135-136 137-138 139-155

____________________________________________________________

_______________________________________

Appendix 6. AD in relation to the AE model Multiple Choice Questions Aggregate demand

156-166

____________________________________________________________

_______________________________________

Type: D Topic: 1 E: 193 MA: 193 1. The aggregate demand curve: A) is upsloping because a higher price level is necessary to make production profitable as production costs rise. B) is downsloping because production costs decline as real output increases. C) shows the amount of expenditures required to induce the production of each possible level of real output. D) shows the amount of real output that will be purchased at each possible price level. Answer: D

Type: A Topic: 1 E: 194 MA: 194 2. The aggregate demand curve is: A) vertical if full employment exists. B) horizontal when there is considerable unemployment in the economy. C) downsloping because of the interest-rate, real-balances, and foreign purchases effects. D) downsloping because production costs decrease as real output rises. Answer: C

Chapter 11: Aggregate Demand and Aggregate Supply

Type: A Topic: 1 E: 194 MA: 194 3. The interest-rate effect suggests that: A) a decrease in the supply of money will increase interest rates and reduce interest-sensitive consumption and investment spending. B) an increase in the price level will increase the demand for money, reduce interest rates, and decrease consumption and investment spending. C) an increase in the price level will increase the demand for money, increase interest rates, and decrease consumption and investment spending. D) an increase in the price level will decrease the demand for money, reduce interest rates, and increase consumption and investment spending. Answer: C

Type: A Topic: 1 E: 194 MA: 194 4. The real-balances effect indicates that: A) an increase in the price level will increase the demand for money, increase interest rates, and reduce consumption and investment spending. B) a lower price level will decrease the real value of many financial assets and therefore reduce spending. C) a higher price level will increase the real value of many financial assets and therefore increase spending. D) a higher price level will decrease the real value of many financial assets and therefore reduce spending. Answer: D

Type: A Topic: 1 E: 195 MA: 195 5. The interest-rate and real-balances effects are important because they help explain: A) rightward and leftward shifts of the aggregate demand curve. B) why fiscal policy cannot be used effectively to curb inflation. C) the shape of the aggregate demand curve. D) the shape of the aggregate supply curve. Answer: C

Type: A Topic: 1 E: 194-195 MA: 194-195 6. The foreign purchases effect suggests that an increase in the U.S. price level relative to other countries will: A) increase the amount of U.S. real output purchased. B) increase U.S. imports and decrease U.S. exports. C) increase both U.S. imports and U.S. exports. D) decrease both U.S. imports and U.S. exports. Answer: B

Type: A Topic: 1 E: 194-195 MA: 194-195 7. The foreign purchases effect suggests that a decrease in the U.S. price level relative to other countries will: A) shift the aggregate demand curve leftward. B) shift the aggregate supply curve leftward. C) decrease U.S. exports and increase U.S. imports. D) increase U.S. exports and decrease U.S. imports. Answer: D

McConnell/Brue: Economics, 16/e

Page 336

Chapter 11: Aggregate Demand and Aggregate Supply

Type: A Topic: 1 E: 195 MA: 195 8. The foreign purchases effect: A) shifts the aggregate demand curve rightward. B) shifts the aggregate demand curve leftward. C) shifts the aggregate supply curve rightward. D) moves the economy along a fixed aggregate demand curve. Answer: D

Type: A Topic: 1 E: 194 MA: 194 9. If the price level increases in the United States relative to foreign countries, then American consumers will purchase more foreign goods and fewer U.S. goods. This statement describes: A) the output effect. C) the real-balances effect. B) the foreign purchases effect. D) the shift-of-spending effect. Answer: B

Type: A Topic: 1 E: 194 MA: 194 10. The real-balances, interest-rate, and foreign purchases effects all help explain: A) why the aggregate demand curve is downsloping. B) why the aggregate supply curve is upsloping. C) shifts in the aggregate demand curve. D) shifts in the aggregate supply curve. Answer: A

Type: A Topic: 1 E: 194 MA: 194 11. Which of the following explains why the aggregate demand schedule is downward sloping: A) the real-balances effect C) the foreign purchases effect B) the interest-rate effect D) all of the above Answer: D

Type: A Topic: 1 E: 194 MA: 194 12. Which of the following is incorrect? A) As the U.S. price level rises, U.S. goods become relatively more expensive so that U.S. exports fall and U.S. imports rise. B) As the price level falls, the demand for money declines, the interest rate declines, and interest-rate sensitive spending increases. C) When the price level increases, real balances increase, businesses and households find themselves wealthier and therefore increase their spending. D) Given aggregate demand, an increase in aggregate supply increases real output and, assuming downward flexible prices, reduces the price level. Answer: C

Type: D Topic: 1 E: 194 MA: 194 13. The factors that affect the amounts that consumers, businesses, government, and foreigners wish to purchase at each price level are the: A) real-balances, interest-rate, and foreign purchases effects. B) determinants of aggregate supply. C) determinants of aggregate demand. D) sole determinants of the equilibrium price level and the equilibrium real output. Answer: C

McConnell/Brue: Economics, 16/e

Page 337

Chapter 11: Aggregate Demand and Aggregate Supply

Type: D Topic: 1 E: 195 MA: 195 14. The determinants of aggregate demand: A) explain why the aggregate demand curve is downsloping. B) explain shifts in the aggregate demand curve. C) demonstrate why real output and the price level are inversely related. D) include input prices and resource productivity. Answer: B

Type: A Topic: 1 E: 195 MA: 195 15. Other things equal, if the national incomes of the major trading partners of the United States were to rise, the U.S.: A) aggregate demand curve would shift to the right. B) aggregate supply curve would shift to the left. C) aggregate supply curve would shift to the right. D) aggregate demand curve would shift to the left. Answer: A

Type: A Topic: 1 E: 194 MA: 194 16. Which one of the following would not shift the aggregate demand curve? A) a change in the price level B) depreciation of the international value of the dollar C) a decline in the interest rate at each possible price level D) an increase in personal income tax rates Answer: A

Type: A Topic: 1 E: 194-195 MA: 194-195 Status: New 17. Other things equal, a decrease in the real interest rate will: A) expand investment and shift the AD curve to the left. B) expand investment and shift the AD curve to the right. C) reduce investment and shift the AD curve to the left. D) reduce investment and shift the AD curve to the right. Answer: B

Type: A Topic: 1 E: 196 MA: 196 Status: New 18. A decline in investment will shift the AD curve to the: A) left by a multiple of the change in investment. B) left by the same amount as the change in investment. C) right by the same amount as the change in investment. D) right by a multiple of the change in investment. Answer: A

Type: A Topic: 1 E: 197 MA: 197 Status: New 19. An increase in net exports will shift the AD curve to the: A) left by a multiple of the change in investment. B) left by the same amount as the change in investment. C) right by the same amount as the change in investment. D) right by a multiple of the change in investment. Answer: D

McConnell/Brue: Economics, 16/e

Page 338

Chapter 11: Aggregate Demand and Aggregate Supply

Type: C Topic: 1 E: 195 MA: 195 Status: New 20. If investment increases by $10 billion and the economy's MPC is .8, the aggregate demand curve will shift: A) leftward by $40 billion at each price level. C) rightward by $50 billion at each price level. B) rightward by $10 billion at each price level. D) leftward by $20 billion at each price level. Answer: C

Type: C Topic: 1 E: 195 MA: 195 Status: New 21. If investment decreases by $20 billion and the economy's MPC is .5, the aggregate demand curve will shift: A) leftward by $40 billion at each price level. C) rightward by $40 billion at each price level. B) rightward by $20 billion at each price level. D) leftward by $20 billion at each price level. Answer: A

Type: C Topic: 1 E: 195 MA: 195 Status: New 22. An economy's aggregate demand curve shifts leftward or rightward by more than changes in initial spending because of the: A) net export effect. B) wealth effect. C) real-balances effect. D) multiplier effect. Answer: D

Long-run aggregate supply

Type: D Topic: 2 E: 198 MA: 198 Status: New 23. The economy's long-run aggregate supply curve: A) slopes upward and to the right. C) is horizontal. B) is vertical. D) slopes downward and to the right. Answer: B

Type: C Topic: 2 E: 198 MA: 198 Status: New 24. The economy's long-run AS curve assumes that wages and other resource prices: A) eventually rise and fall to match upward or downward changes in the price level. B) are flexible upward but inflexible downward. C) rise and fall more rapidly than the price level. D) are relatively inflexible both upward and downward. Answer: A

McConnell/Brue: Economics, 16/e

Page 339

Chapter 11: Aggregate Demand and Aggregate Supply

Use the following to answer questions 25-27:

1

2

Price level

3 4

0

Real domestic output

Type: G Topic: 2 E: 198 MA: 198 Status: New 25. In the above diagram, the economy's long-run aggregate supply curve is shown by line: A) 1. B) 2. C) 3. D) 4. Answer: A

Type: G Topic: 2 E: 198-199 MA: 198-199 Status: New 26. In the above diagram, the economy's relevant aggregate demand and long-run aggregate supply curves are lines: A) 4 and 2. B) 4 and 1. C) 2 and 4. D) 2 and 3. Answer: B

Type: G Topic: 2 E: 198-199 MA: 198-199 Status: New 27. In the above diagram, the economy's short-run AS curve is line ___ and its long-run AS curve is line ___. A) 1; 3. B) 2; 4. C) 3; 4. D) 2; 1. Answer: D

Aggregate supply (short run)

Type: D Topic: 3 E: 198 MA: 198 Status: New 28. The aggregate supply curve: A) is explained by the interest rate, real-balances, and foreign purchases effects. B) gets steeper as the economy moves from the top of the curve to the bottom of the curve. C) shows the various amounts of real output that businesses will produce at each price level. D) is downsloping because real purchasing power increases as the price level falls. Answer: C

Type: D Topic: 3 E: 198 MA: 198 Status: New 29. The graphical relationship between the price level and the amount of real GDP that businesses will offer for sale is known as the: A) aggregate demand curve. C) investment demand curve. B) investment supply curve. D) aggregate supply curve. Answer: D

McConnell/Brue: Economics, 16/e

Page 340

Chapter 11: Aggregate Demand and Aggregate Supply

Type: D Topic: 3 E: 198 MA: 198 Status: New 30. The aggregate supply curve (short-run): A) slopes downward and to the right. C) slopes upward and to the right. B) graphs as a vertical line. D) graphs as a horizontal line. Answer: C

Type: F Topic: 3 E: 198 MA: 198 Status: New 31. The aggregate supply curve (short-run): A) graphs as a horizontal line. B) is steeper above the full-employment output than below it. C) slopes downward and to the right. D) presumes that changes in wages and other resource prices match changes in the price level. Answer: B

Type: A Topic: 3 E: 198 MA: 198 Status: New 32. The aggregate supply curve (short-run) slopes upward and to the right because: A) changes in wages and other resource prices completely offset changes in the price level. B) the price level is flexible upward but inflexible downward. C) supply creates its own demand. D) wages and other resource prices adjust only slowly to changes in the price level. Answer: D

Type: A Topic: 3 E: 198 MA: 198 Status: New 33. The aggregate supply curve (short-run) is upsloping because: A) wages and other resource prices match changes in the price level. B) the price level is flexible upward but inflexible downward. C) per-unit production costs rise as the economy moves toward and beyond its full-employment real output. D) wages and other resource prices are flexible upward but inflexible downward. Answer: C

McConnell/Brue: Economics, 16/e

Page 341

Chapter 11: Aggregate Demand and Aggregate Supply

Use the following to answer questions 34-39:
AS3
AS1 AS2
Price level

0

Real domestic output, GDP

Type: G Topic: 3 E: 200 MA: 200 Status: New 34. In the above diagram, a shift from AS1 to AS3 might be caused by a(n): A) increase in productivity. C) decrease in the prices of domestic resources. B) increase in the prices of imported resources. D) decrease in business taxes. Answer: B

Type: G Topic: 3 E: 200 MA: 200 Status: New 35. In the above diagram, a shift from AS1 to AS2 might be caused by a(n): A) increase in market power of resource sellers. C) decrease in the prices of domestic resources. B) increase in the prices of imported resources. D) increase in business taxes. Answer: C

Type: G Topic: 3 E: 200 MA: 200 Status: New 36. In the above diagram, a shift from AS3 to AS2 might be caused by an increase in: A) business taxes and government regulation. C) the prices of domestic resources. B) the prices of imported resources. D) productivity. Answer: D

Type: G Topic: 3 E: 200 MA: 200 Status: New 37. In the above diagram, a shift from AS2 to AS3 might be caused by a(n): A) decrease in interest rates. B) increase in business taxes and costly government regulation. C) decrease in the prices of domestic resources. D) decrease in the price level. Answer: B

Type: G Topic: 3 E: 200 MA: 200 Status: New 38. In the above diagram, the most favorable shift of the aggregate supply curve for the economy would be from: A) AS1 to AS2. B) AS1 to AS3. C) AS2 to AS3. D) AS3 to AS2. Answer: D

McConnell/Brue: Economics, 16/e

Page 342

Chapter 11: Aggregate Demand and Aggregate Supply

Type: G Topic: 3 E: 200 MA: 200 Status: New 39. In the above diagram, a substantial appreciation of the U.S. dollar with no immediate change in the U.S. price level would result in a: A) movement upward along an existing aggregate supply curve such as AS1. B) movement downward along an existing aggregate supply curve such as AS1. C) rightward shift of the aggregate supply curve, such as from AS1 to AS2. D) leftward shift of the aggregate supply curve, such as from AS1 to AS3. Answer: C

Type: A Topic: 3 E: 200-201 MA: 200-201 Status: New 40. Other things equal, an improvement in productivity will: A) shift the aggregate demand curve to the left. C) shift the aggregate supply curve to the right. B) shift the aggregate supply curve to the left. D) increase the price level. Answer: C

Type: A Topic: 3 E: 200-201 MA: 200-201 41. A rightward shift in the aggregate supply curve is best explained by an increase in: A) business taxes. B) productivity. C) nominal wages. D) the price of imported resources. Answer: B

Use the following to answer questions 42-45: Suppose that real domestic output in an economy is 20 units, the quantity of inputs is 10, and the price of each input is $4. Answer the following question(s) on the basis of this information.

Type: A Topic: 3 E: 201 MA: 201 42. Refer to the above information. The level of productivity is: A) 20. B) 10. C) 5. D) 2. Answer: D

Type: A Topic: 3 E: 201 MA: 201 43. The per unit cost of production in the economy described above is: A) $.50. B) $1. C) $2. D) $5. Answer: C

Type: A Topic: 3 E: 201 MA: 201 44. Refer to the above information. All else being equal, if the price of each input increased from $4 to $6, productivity would: A) fall from 2 to 3. B) fall from .50 to .33. C) rise from 1 to 2. D) remain unchanged. Answer: D

Type: A Topic: 3 E: 199 MA: 199 45. Refer to the above information. Given an increase in input price from $4 to $6, we would expect the aggregate: A) supply curve to shift to the left. C) demand curve to shift to the left. B) supply curve to shift to the right. D) demand curve to shift to the right. Answer: A

McConnell/Brue: Economics, 16/e

Page 343

Chapter 11: Aggregate Demand and Aggregate Supply

Type: A Topic: 3 E: 200 MA: 200 46. Other things equal, if the U.S. dollar were to depreciate, the: A) aggregate demand curve would remain fixed in place. B) aggregate supply curve would shift to the left. C) aggregate supply curve would shift to the right. D) aggregate demand curve would shift to the left. Answer: B

Type: A Topic: 3 E: 200 MA: 200 47. Which one of the following would increase per unit production cost and therefore shift the aggregate supply curve to the left? A) a reduction in business taxes B) production bottlenecks occurring when producers near full plant capacity C) an increase in the price of imported resources D) deregulation of industry Answer: C

Type: D Topic: 3 E: 200 MA: 200 48. Shifts in the aggregate supply curve are caused by changes in: A) consumption spending. B) the quantity of real output demanded. C) the quantity of real output supplied. D) one or more of the determinants of aggregate supply. Answer: D

Use the following to answer questions 49-51: Answer the next question(s) on the basis of the following information: An economy is employing 2 units of capital, 5 units of raw materials, and 8 units of labor to produce its total output of 640 units. Each unit of capital costs $10, each unit of raw materials, $4, and each unit of labor, $3.

Type: A Topic: 3 E: 201 MA: 201 49. Refer to the above information. The per unit cost of production in this economy is: A) $.05. B) $.10. C) $.50. D) $1.00. Answer: B

Type: A Topic: 3 E: 201 MA: 201 50. Refer to the above information. If the per unit price of raw materials rises from $4 to $8 and all else remains constant, the per unit cost of production will rise by about: A) 100 percent. B) 50 percent. C) 40 percent. D) 30 percent. Answer: D

Type: A Topic: 3 E: 201 MA: 201 51. Refer to the above information. As a result of the change indicated in the previous question, the aggregate: A) supply curve would shift to the left. C) demand curve would shift to the left. B) supply curve would shift to the right. D) demand curve would shift to the right. Answer: A

McConnell/Brue: Economics, 16/e

Page 344

Chapter 11: Aggregate Demand and Aggregate Supply

Type: D Topic: 3 E: 199 MA: 199 52. The determinants of aggregate supply: A) are consumption, investment, government, and net export spending. B) explain why real domestic output and the price level are directly related. C) explain the three distinct ranges of the aggregate supply curve. D) include resource prices and resource productivity. Answer: D

Type: A Topic: 3 E: 200 MA: 200 53. Which of the following would not shift the aggregate supply curve? A) an increase in labor productivity C) a decline in business taxes B) a decline in the price of imported oil D) an increase in the price level Answer: D

Type: D Topic: 3 E: 200 MA: 200 54. Productivity measures: A) real output per unit of input. B) per unit production costs. C) the changes in real wealth caused by price level changes. D) the amount of capital goods used per worker. Answer: A

Type: D Topic: 3 E: 201 MA: 201 55. Per unit production cost is: A) real output divided by inputs. B) total input cost divided by units of output. Answer: B

C) units of output divided by total input cost. D) a determinant of aggregate demand.

Type: A Topic: 3 E: 201 MA: 201 56. Suppose that nominal wages fall and productivity rises in a particular economy. Other things equal, the aggregate: A) demand curve will shift leftward. C) supply curve will shift leftward. B) supply curve will shift rightward. D) expenditures curve will shift downward. Answer: B

Type: D Topic: 3 E: 200 MA: 200 57. Monopoly or market power is the ability of a firm to: A) shift its demand curve to the right. C) set its price. B) shift its demand curve to the left. D) achieve economies of scale. Answer: C

McConnell/Brue: Economics, 16/e

Page 345

Chapter 11: Aggregate Demand and Aggregate Supply

Type: C Topic: 3 E: 200 MA: 200 58. Other things equal, appreciation of the dollar: A) increases aggregate demand in the United States and may increase aggregate supply by reducing the prices of imported resources. B) increases aggregate demand in the United States and may decrease aggregate supply by reducing the prices of imported resources. C) decreases aggregate demand in the United States and may increase aggregate supply by reducing the prices of imported resources. D) decreases aggregate demand in the United States and may reduce aggregate supply by increasing the prices of imported resources. Answer: C

Type: C Topic: 3 E: 201 MA: 201 59. Other things equal, a reduction in personal and business taxes can be expected to: A) increase aggregate demand and decrease aggregate supply. B) increase both aggregate demand and aggregate supply. C) decrease both aggregate demand and aggregate supply. D) decrease aggregate demand and increase aggregate supply. Answer: B

Type: A Topic: 3 E: 201 MA: 201 60. Other things equal, an improvement in productivity will: A) increase the equilibrium price level. C) shift the aggregate supply curve to the right. B) shift the aggregate supply curve to the left. D) shift the aggregate demand curve to the left. Answer: C

Use the following to answer questions 61-63: Answer the next question(s) on the basis of the following information about the relationship between input quantities and real domestic output in a hypothetical economy:
Input quantity 100 150 200 Real domestic output 200 300 400

Type: T Topic: 3 E: 201 MA: 201 61. The level of productivity in the above economy is: A) 2. B) .5. C) 4. D) 200. Answer: A

Type: T Topic: 3 E: 201 MA: 201 62. If the price of each input is $5, the per unit cost of production in the above economy is: A) $5. B) $2.75. C) $2.50. D) $.40. Answer: C

McConnell/Brue: Economics, 16/e

Page 346

Chapter 11: Aggregate Demand and Aggregate Supply

Type: T Topic: 3 E: 201 MA: 201 63. Suppose that the price of each input increased from $5 to $8. The per unit cost of production in the above economy would: A) rise by $1.50 and the aggregate supply curve would shift to the right. B) rise by 60 percent and the aggregate supply curve would shift to the left. C) rise by 60 percent and the aggregate demand curve would shift to the left. D) fall by $1.50 and the aggregate demand curve would shift to the right. Answer: B

Equilibrium; changes in equilibrium

Type: D Topic: 4 E: 203 MA: 203 64. The equilibrium price level and level of real output occur where: A) real output is at its highest possible level. B) export equal imports. C) the price level is at its lowest level. D) the aggregate demand and supply curves intersect. Answer: D

Use the following to answer questions 65-68: Answer the next question(s) on the basis of the following aggregate demand and supply schedules for a hypothetical economy:

Amount of real output demanded $200 300 400 500 600

Price level (index value) 300 250 200 150 100

Amount of real output supplied $500 450 400 300 200

Type: T Topic: 4 E: 202 MA: 202 65. Refer to the above data. The equilibrium price level will be: A) 150. B) 200. C) 250. D) 300. Answer: B

Type: T Topic: 4 E: 203 MA: 203 66. Refer to the above data. If the price level is 150 and producers supply $300 of real output: A) a shortage of real output of $200 will occur. B) a shortage of real output of $100 will occur. C) a surplus of real output of $300 will occur. D) neither a shortage nor a surplus of real output will occur. Answer: A

McConnell/Brue: Economics, 16/e

Page 347

Chapter 11: Aggregate Demand and Aggregate Supply

Type: T Topic: 4 E: 202 MA: 202 67. Refer to the above data. If the amount of real output demanded at each price level falls by $200, the equilibrium price level and equilibrium level of real domestic output will fall to: A) 250 and $200, respectively. C) 150 and $300, respectively. B) 200 and $300, respectively. D) 150 adn $200, respectively. Answer: C

Type: T Topic: 4 E: 203 MA: 203 68. Refer to the above data. The change in aggregate demand indicated in the previous question might have been caused by: A) an increase in net exports. C) an increase in consumer wealth. B) a worsening of business expectations. D) a decrease in the personal income tax. Answer: B

Type: A Topic: 4 E: 203 MA: 203 Status: New 69. Graphically, demand-pull inflation is shown as a: A) rightward shift of the AD curve along an upsloping AS curve. B) leftward shift of the AS curve along a downsloping AD curve. C) leftward shift of AS curve along an upsloping AD curve. D) rightward shift of the AD curve along a downsloping AS curve. Answer: A

Type: A Topic: 4 E: 205 MA: 205 70. Graphically, cost-push inflation is shown as a: A) leftward shift of the AD curve. B) rightward shift of the AS curve. Answer: C

C) leftward shift of AS curve. D) rightward shift of the AD curve.

Type: A Topic: 4 E: 203 MA: 203 71. Graphically, the full-employment, low-inflation, rapid-growth economy of the last half of the 1990s is depicted by a: A) rightward shift of the aggregate demand curve along a fixed aggregate supply curve. B) rightward shift of the aggregate supply curve along a fixed aggregate demand curve. C) rightward shift of the aggregate demand curve and a rightward shift of the aggregate supply curve. D) leftward shift of the aggregate demand curve and a leftward shift of the aggregate supply curve. Answer: C

McConnell/Brue: Economics, 16/e

Page 348

Chapter 11: Aggregate Demand and Aggregate Supply

Use the following to answer questions 72-81:
P AS1 P2 P1 AD1 AD2 0 Q2 (A) Q1 Q 0 Q2 (B) Q1 Q 0 Q1 (C) P1 b a AD1 AD1 Q2 AD2 Q P AS2 AS1 P1 a b c P AS1 AS2

Type: G Topic: 4 E: 203-207 MA: 203-207 Status: New 72. Refer to the above diagrams, in which AD1 and AS1 are the "before" curves and AD2 and AS2 are the "after" curves. A recession is depicted by: A) panel (A) only. B) panel (B) only. C) panel (C) only. D) panels (A) and (B). Answer: D

Type: G Topic: 4 E: 203-207 MA: 203-207 73. Refer to the above diagrams, in which AD1 and AS1 are the "before" curves and AD2 and AS2 are the "after" curves. Cost-push inflation is depicted by: A) panel (A) only. B) panel (B) only. C) panel (C) only D) panels (B) and (C). Answer: B

Type: G Topic: 4 E: 203-207 MA: 203-207 74. Refer to the above diagrams, in which AD1 and AS1 are the "before" curves and AD2 and AS2 are the "after" curves. Growth, full-employment and price stability is depicted by: A) panel (A) only. B) panel (B) only. C) panel (C) only D) panels (B) and (C). Answer: C

Type: G Topic: 4 E: 203-207 MA: 203-207 75. Refer to the above diagrams, in which AD1 and AS1 are the "before" curves and AD2 and AS2 are the "after" curves. Other things equal, an increase in investment spending is depicted by: A) panel (A) only. B) panel (B) only. C) panel (C) only D) panels (B) and (C). Answer: C

Type: G Topic: 4 E: 203-207 MA: 203-207 76. Refer to the above diagrams, in which AD1 and AS1 are the "before" curves and AD2 and AS2 are the "after" curves. Other things equal, a decline in productivity is depicted by: A) panel (A) only. B) panel (B) only. C) panel (C) only D) panels (B) and (C). Answer: B

McConnell/Brue: Economics, 16/e

Page 349

Chapter 11: Aggregate Demand and Aggregate Supply

Type: G Topic: 4 E: 203-207 MA: 203-207 77. Refer to the above diagrams, in which AD1 and AS1 are the "before" curves and AD2 and AS2 are the "after" curves. Other things equal, a decline in net exports caused by a change in incomes abroad is depicted by: A) panel (A) only. B) panel (B) only. C) panel (C) only D) panels (B) and (C). Answer: A

Type: G Topic: 4 E: 203-207 MA: 203-207 78. Refer to the above diagrams, in which AD1 and AS1 are the "before" curves and AD2 and AS2 are the "after" curves. Other things equal, a decline in net exports caused by the foreign purchases effect of a price-level increase is depicted by the: A) shift of the AD curve in panel (A). C) shift of the AS curve in panel (B). B) move from point a to point b in panel (B). D) move from point a to point c in panel (C). Answer: B

Type: G Topic: 4 E: 203-207 MA: 203-207 79. Refer to the above diagrams, in which AD1 and AS1 are the "before" curves and AD2 and AS2 are the "after" curves. Other things equal, a decline in investment spending caused by the interest-rate effect of a price-level increase is depicted by the: A) shift of the AD curve in panel (A). C) move from point a to point b in panel (B). B) shift of the AS curve in panel (B). D) move from point a to point c in panel (C). Answer: C

Type: G Topic: 4 E: 203-207 MA: 203-207 80. Refer to the above diagrams, in which AD1 and AS1 are the "before" curves and AD2 and AS2 are the "after" curves. Other things equal, a decrease in resource prices is depicted by: A) panel (A) only. B) panel (B) only. C) panel (C) only D) panels (B) and (C). Answer: C

Type: G Topic: 4 E: 203-207 MA: 203-207 81. Refer to the above diagrams, in which AD1 and AS1 are the "before" curves and AD2 and AS2 are the "after" curves. Other things equal, inflation is absent in: A) panel (A) only. B) panel (B) only. C) panel (C) only D) panels (A) and (C). Answer: D

Type: A Topic: 4 E: 204 MA: 204 Status: New 82. If aggregate demand decreases, and as a result, real output and employment decline but the price level remains unchanged, we can assume that: A) the money supply has declined. B) the price level is inflexible downward and a recession has occurred. C) cost-push inflation has occurred. D) productivity has declined. Answer: B

McConnell/Brue: Economics, 16/e

Page 350

Chapter 11: Aggregate Demand and Aggregate Supply

Type: A Topic: 4 E: 203 MA: 203 Status: New 83. A rightward shift of the AD curve in the very steep upper part of the upsloping AS curve will: A) increase real output by more than the price level. B) increase the price level by more than real output. C) reduce real output by more than the price level. D) reduce the price level by more than real output. Answer: B

Type: A Topic: 4 E: 203 MA: 203 Status: New 84. A rightward shift of the AD curve in the very flat part of the upsloping AS curve will: A) increase real output by more than the price level. B) increase the price level by more than real output. C) reduce real output by more than the price level. D) reduce the price level by more than real output. Answer: A

Type: A Topic: 4 E: 203 MA: 203 Status: New 85. Given a fixed upsloping AS curve, a rightward shift of the AD curve will: A) cause cost push inflation. C) increase the price level but not real output. B) increase real output but not the price level. D) increase both the price level and real output. Answer: D

Type: A Topic: 4 E: 204 MA: 204 Status: New 86. A decrease in aggregate demand will cause a greater decline in real output the: A) less flexible is the economy's price level. B) more flexible is the economy's price level. C) steeper is the economy's AS curve. D) larger is the economy's marginal propensity to save. Answer: A

McConnell/Brue: Economics, 16/e

Page 351

Chapter 11: Aggregate Demand and Aggregate Supply

Use the following to answer questions 87-88:

AS1
AS2

Price level

P1 AD2

AD1 0 Q1 Q2 Q3 Real domestic output

Type: A Topic: 4 E: 206 MA: 206 87. In the above figure AD1 and AS1 represent the original aggregate supply and demand curves and AD2 and AS2 show the new aggregate demand and supply curves. The change in aggregate supply from AS1 to AS2 could be caused by: A) a reduction in the price level. B) the increased availability of entrepreneurial talent. C) an increase in business taxes. D) the real-balances, interest-rate, and foreign purchases effects. Answer: B

Type: G Topic: 4 E: 206 MA: 206 88. In the above figure AD1 and AS1 represent the original aggregate supply and demand curves and AD2 and AS2 show the new aggregate demand and supply curves. The changes in aggregate demand and supply in the above diagram produce: A) a higher price level. B) an expansion of real output and a stable price level. C) an expansion of real output and a higher price level. D) a decline in real output and a stable price level. Answer: B

McConnell/Brue: Economics, 16/e

Page 352

Chapter 11: Aggregate Demand and Aggregate Supply

Use the following to answer questions 89-93:

AS1 AS0 G
Price level

H

F E AD1 AD0

0

A B C Real domestic output

Type: G Topic: 4 E: 209 MA: 209 89. Refer to the above diagram. If the initial aggregate demand and supply curves are AD0 and AS0, the equilibrium price level and level of real domestic output will be: A) F and C, respectively. C) F and A, respectively. B) G and B, respectively. D) E and B, respectively. Answer: A

Type: G Topic: 4 E: 209 MA: 209 90. Refer to the above diagram. If the aggregate supply curve shifted from AS0 to AS1, we could say that: A) aggregate supply has increased, equilibrium output has decreased, and the price level has increased. B) aggregate supply has decreased, equilibrium output has decreased, and the price level has increased. C) an increase in the amount of output supplied has occurred. D) aggregate supply has increased and the price level has risen to G. Answer: B

Type: G Topic: 4 E: 209 MA: 209 91. Refer to the above diagram. If aggregate supply is AS1 and aggregate demand is AD0, then: A) at any price level above G a shortage of real output would occur. B) F represents a price level that would result in a surplus of real output of AC. C) a surplus of real output of GH would occur. D) F represents a price level that would result in a shortage of real output of AC. Answer: D

Type: G Topic: 4 E: 209 MA: 209 92. Refer to the above diagram. A shift of the aggregate demand curve from AD1 to AD0 might be caused by a(n): A) decrease in aggregate supply. C) increase in investment spending. B) decrease in the amount of output supplied. D) decrease in net export spending. Answer: C

McConnell/Brue: Economics, 16/e

Page 353

Chapter 11: Aggregate Demand and Aggregate Supply

Type: G Topic: 4 E: 209 MA: 209 93. Refer to the above diagram. Other things equal, a shift of the aggregate supply curve from AS0 to AS1 might be caused by a(n): A) increase in government regulation. C) increase in productivity. B) increase in aggregate demand. D) decline in nominal wages. Answer: A

Type: C Topic: 4 E: 209 MA: 209 94. If aggregate demand increases and aggregate supply decreases, the price level: A) will decrease, but real output may either increase or decrease. B) will increase, but real output may either increase or decrease. C) and real output will both increase. D) and real output will both decrease. Answer: B

Type: C Topic: 4 E: 200 MA: 200 95. If the dollar price of foreign currencies falls (that is, the dollar appreciates), we would expect: A) aggregate demand to decrease and aggregate supply to increase. B) both aggregate demand and aggregate supply to decrease. C) both aggregate demand and aggregate supply to increase. D) aggregate demand to increase and aggregate supply to decrease. Answer: A

Type: C Topic: 4 E: 196 MA: 196 96. We would expect a decline in personal and corporate income taxes to: A) shift the aggregate demand curve rightward. C) decrease real output. B) decrease consumption and investment spending. D) shift the aggregate supply curve leftward. Answer: A

Type: C Topic: 4 E: 201 MA: 201 97. An increase in input productivity will: A) shift the aggregate supply curve leftward. B) reduce the equilibrium price level, assuming downward flexible prices. C) reduce the equilibrium real output. D) reduce aggregate demand. Answer: B

Type: C Topic: 4 E: 196, 201 MA: 196, 201 98. If personal taxes were decreased and resource productivity increased simultaneously, the equilibrium: A) output would rise. C) price level would necessarily fall. B) output would fall. D) price level would necessarily rise. Answer: A

Type: A Topic: 4 E: 203 MA: 203 99. If the current price level was such that the aggregate quantity demanded exceeded the aggregate quantity supplied, we would expect: A) inflation to occur. C) the aggregate demand curve to shift leftward. B) the aggregate demand curve to shift rightward. D) the aggregate supply curve to shift leftward. Answer: A

McConnell/Brue: Economics, 16/e

Page 354

Chapter 11: Aggregate Demand and Aggregate Supply

Type: C Topic: 4 E: 203 MA: 203 100. In which of the following sets of circumstances can we confidently expect inflation? A) aggregate supply and aggregate demand both increase B) aggregate supply and aggregate demand both decrease C) aggregate supply decreases and aggregate demand increases D) aggregate supply increases and aggregate demand decreases Answer: C

Use the following to answer questions 101-109: Use the following diagrams for the U.S. economy to answer the following questions.

Type: G Topic: 4 E: 201 MA: 201 101. Which of the above diagrams best portrays the effects of an increase in resource productivity? A) A B) B C) C D) D Answer: A

Type: G Topic: 4 E: 200 MA: 200 102. Which of the above diagrams best portrays the effects of a decrease in the availability of key natural resources? A) A B) B C) C D) D Answer: B

Type: G Topic: 4 E: 197 MA: 197 103. Which of the above diagrams best portrays the effects of an increase in foreign spending on U.S. products? A) A B) B C) C D) D Answer: C

McConnell/Brue: Economics, 16/e

Page 355

Chapter 11: Aggregate Demand and Aggregate Supply

Type: G Topic: 4 E: 196 MA: 196 104. Which of the above diagrams best portrays the effects of an increase in consumer spending? A) A B) B C) C D) D Answer: C

Type: G Topic: 4 E: 196 MA: 196 105. Which of the above diagrams best portrays an improvement in expected rates of return on investment? A) A B) B C) C D) D Answer: C

Type: G Topic: 4 E: 197 MA: 197 106. Which of the above diagrams best portrays the effects of declines in the incomes of U.S. trading partners? A) A B) B C) C D) D Answer: D

Type: G Topic: 4 E: 200 MA: 200 107. Which of the above diagrams best portrays the effects of declines in the prices of imported resources? A) A B) B C) C D) D Answer: A

Type: G Topic: 4 E: 197 MA: 197 108. Which of the above diagrams best portrays the effects of a substantial reduction in government spending? A) A B) B C) C D) D Answer: D

Type: G Topic: 4 E: 199-200 MA: 199-200 109. Which of the above diagrams best portrays the effects of a dramatic increase in energy prices? A) A B) B C) C D) D Answer: B

McConnell/Brue: Economics, 16/e

Page 356

Chapter 11: Aggregate Demand and Aggregate Supply

Use the following to answer questions 110-119: Answer the next question(s) on the basis of the following table for a particular country in which C is consumption expenditures, Ig is gross investment expenditures, G is government expenditures, X is exports, and M is imports. All figures are in billions of dollars. Each question is independent of the other questions.
(1) Price level 128 125 122 119 116 (2) C $18 20 22 24 26 (3) Ig $2 4 6 8 10 (4) G $3 3 3 3 3 (5) X $1 2 3 4 5 (6) M $5 4 3 2 1 (7) Real GDP

Type: T Topic: 4 E: 195-197 MA: 195-197 110. Refer to the above table. Which of the following schedules constitutes aggregate demand in this country?
(a) GDP $19 25 31 37 43 (b) GDP $23 27 31 35 39 (c) GDP $20 22 24 26 28 (d) GDP $34 37 40 43 46

P 128 125 122 119 116

P 128 125 122 119 116

P 128 125 122 119 116

P 128 125 122 119 116

Answer: A

Type: T Topic: 4 E: 194 MA: 194 111. Refer to the above table. The interest-rate effect of changes in the price level is shown by columns: A) (1) and (4) of the table. C) (1) and (3) of the table. B) (5) and (6) of the table. D) (2) and (4) of the table. Answer: C

Type: T Topic: 4 E: 194 MA: 194 112. Refer to the above table. The real-balances effect of changes in the price level is: A) shown by columns (1) and (2) of the table. C) shown by columns (1) and (4) of the table. B) shown by columns (1) and (5) of the table. D) not shown by the data in the table. Answer: A

Type: T Topic: 4 E: 202 MA: 202 113. Refer to the above table. If equilibrium real GDP is $31 billion, the equilibrium price level will be: A) 128. B) 125. C) 122. D) 119. E) 116. Answer: C

Type: T Topic: 4 E: 202 MA: 202 Status: New 114. Refer to the above table. If the amounts of GDP supplied at the price levels shown (in descending order) are $45, $43, $40, $37, and $31, the equilibrium level of real GDP will be: A) $37 billion. B) $35 billion. C) $26 billion. D) $43 billion. Answer: A

McConnell/Brue: Economics, 16/e

Page 357

Chapter 11: Aggregate Demand and Aggregate Supply

Type: T Topic: 4 E: 202 MA: 202 Status: New 115. Refer to the above table. If the amounts of GDP supplied at the price levels shown (in descending order) are $27, $25, $22, $18, and $13, the equilibrium price level will be: A) 128. B) 125. C) 122. D) 119. Answer: B

Type: T Topic: 4 E: 197 MA: 197 116. Refer to the above table. If this nation's equilibrium price level is 125, its net exports will be: A) minus $4 billion. B) minus $2 billion. C) zero. D) $2 billion. Answer: B

Type: T Topic: 4 E: 196 MA: 196 117. Refer to the above table. If the equilibrium level of real GDP is $43 billion, its level of consumption will be: A) $20 billion. B) $22 billion. C) $24 billion. D) $26 billion. Answer: D

Type: T Topic: 4 E: 197 MA: 197 118. Refer to the above table. A decline in the international value of the dollar would: A) increase the values in columns (5) and (6) and reduce aggregate demand. B) decrease the values in columns (5) and (6) and increase aggregate demand. C) decrease the values in column (5), increase the values in column (6), and reduce aggregate demand. D) increase the values in column (5), decrease the values in column (6), and increase aggregate demand. Answer: D

Type: T Topic: 4 E: 196 MA: 196 119. Refer to the above table. A decrease in the interest rate would: A) increase the values in column (3) and increase aggregate demand. B) decrease the values in column (3) and increase aggregate demand. C) increase the values in column (2) and decrease aggregate demand. D) decrease the values in column (2) and decrease aggregate demand. Answer: A

McConnell/Brue: Economics, 16/e

Page 358

Chapter 11: Aggregate Demand and Aggregate Supply

Use the following to answer questions 120-124:

AS
Price level

P1

AD2 AD1 0 Q1 Real domestic output

Type: G Topic: 4 E: 202 MA: 202 120. Refer to the above diagram. If equilibrium real output is Q2, then: A) aggregate demand is AD1. C) producers will supply output level Q1. B) the equilibrium price level is P1. D) the equilibrium price level is P2. Answer: D

Type: G Topic: 4 E: 202 MA: 202 121. Refer to the above diagram. If the equilibrium price level is P1, then: A) aggregate demand is AD2. C) the equilibrium output level is Q2. B) the equilibrium output level is Q3. D) producers will supply output level Q1. Answer: D

Type: G Topic: 4 E: 202 MA: 202 122. Refer to the above diagram. At the equilibrium price and quantity: A) aggregate demand exceeds aggregate supply. B) the amount of real output demanded and supplied are equal. C) aggregate demand equals aggregate supply. D) aggregate supply exceeds aggregate demand. Answer: B

Type: G Topic: 4 E: 197 MA: 197 123. Refer to the above diagram. Which of the following would shift the aggregate demand curve from AD2 to AD1? A) a decline in personal income tax rates B) an increase in the international value of the dollar C) an increase in government spending D) an upward revision of expected rates of return on investment projects Answer: B

McConnell/Brue: Economics, 16/e

Page 359

Chapter 11: Aggregate Demand and Aggregate Supply

Type: G Topic: 4 E: 202 MA: 202 124. Refer to the above diagram. Suppose that aggregate demand increased from AD1 to AD2. For the price level to stay constant: A) the aggregate supply curve would have to shift rightward. B) the aggregate supply curve would have to shift leftward. C) real domestic output would have to remain constant. D) the aggregate supply curve would have to be vertical. Answer: A

Type: A Topic: 4 E: 195 MA: 195 125. The size of the multiplier associated with an initial increase in spending will be: A) the same whether or not inflation occurs. C) zero if any increase in the price level occurs. B) diminished if inflation occurs. D) enhanced if inflation occurs. Answer: B

Downward price and wage inflexibility

Type: A Topic: 5 E: 204 MA: 204 126. Which of the following is a true statement? A) firms and resource suppliers generally find it easier to reduce prices than to raise them. B) as the price level increases, interest rates will rise and therefore consumption and investment spending will also rise. C) an initial increase in aggregate demand may cause a further increase in aggregate demand because higher prices mean higher incomes. D) a decline in aggregate demand will primarily affect real output and employment if prices are inflexible downward. Answer: D

Type: D Topic: 5 E: 204 MA: 204 127. Prices and wages tend to be: A) flexible both upward and downward. B) inflexible both upward and downward. Answer: D

C) flexible downward, but inflexible upward. D) flexible upward, but inflexible downward.

Type: D Topic: 5 E: 204 MA: 204 128. Efficiency wages are: A) above-market-wages that bring forth so much added work effort that per-unit production costs are lower than at market wages. B) wage payments necessary to compensate workers for unpleasant or risky work conditions. C) usually less than market wages. D) relevant to macro economics because they explain rightward shifts in aggregate demand. Answer: A

Type: A Topic: 5 E: 204 MA: 204 129. When aggregate demand declines, wage rates may be inflexible downward, at least for a time, because of: A) the foreign purchases effect. B) inflexible product prices. C) wage contracts. D) the wealth effect. Answer: C

McConnell/Brue: Economics, 16/e

Page 360

Chapter 11: Aggregate Demand and Aggregate Supply

Type: A Topic: 5 E: 204 MA: 204 130. When aggregate demand declines, many firms may reduce employment rather than wages because wage reductions may: A) reduce per unit production costs. B) reduce worker morale and work effort, and thus lower productivity. C) increase the firms' cost of raising financial capital. D) reduce the demands for their products. Answer: B

Type: A Topic: 5 E: 204 MA: 204 131. When aggregate demand declines, some firms may reduce employment rather than wages because wage reductions may: A) not be possible due to the minimum wage law. C) reduce the demands for their products. B) increase the cost of raising money capital. D) may set off a price war. Answer: A

Type: A Topic: 5 E: 204 MA: 204 132. When aggregate demand declines, the price level may remain constant, at lease for a time, because: A) firms individually fear that their price cut may set off a price war. B) menu costs rise. C) price cuts tend to increase efficiency wages. D) product markets are highly competitive. Answer: A

Type: D Topic: 5 E: 204 MA: 204 133. Menu costs: A) increase during recession. B) decrease during recession. C) are the costs to firms of changing prices and communicating them to customers. D) are sunk costs and therefore should be disregarded. Answer: C

Type: A Topic: 5 E: 205 MA: 205 134. The fear of unwanted price wars may explain why many firms are reluctant to: A) reduce wages when a decline in aggregate demand occurs. B) reduce prices when a decline in aggregate demand occurs. C) expand production capacity when an increase in aggregate demand occurs. D) provide wage increases when labor productivity rises. Answer: B

Consider This Questions

Type: D E: 205 MA: 205 Status: New 135. (Consider This) The idea that the price level readily moves upward but not downward is called the: A) elevator effect. B) escalator effect. C) ratchet effect. D) stair-step effect. Answer: C

McConnell/Brue: Economics, 16/e

Page 361

Chapter 11: Aggregate Demand and Aggregate Supply

Type: D E: 205 MA: 205 Status: New 136. (Consider This) The ratchet effect is the tendency of: A) the price level to increase but not to decrease. B) nominal GDP to increase more rapidly than real GDP. C) real interest rates to fall more rapidly than nominal interest rates. D) consumption to rise year after year regardless of what happens to disposable income. Answer: A

Last Word Questions

Type: F E: 207 MA: 207 137. (Last Word) In recent years: A) unemployment rates in Europe have been higher than in the United States. B) the natural rate of unemployment in Europe has fallen sharply. C) Europe has had strong aggregate demand and low unemployment rates. D) European nations have greatly reduced their unemployment rates by reducing minimum wages, welfare benefits, and government restrictions against firing workers. Answer: A

Type: A E: 207 MA: 207 138. (Last Word) It is unclear whether: A) high European rates of inflation reflect demand-pull or cost-push forces. B) high European rates of poverty can be reduced by by higher transfer payments. C) high European unemployment rates have resulted from high natural rates of unemployment or insufficient aggregate demand. D) European trade deficits stimulate or retard the European economies. Answer: C

True/False Questions

Type: A E: 194 MA: 194 139. The interest-rate effect is one of the determinants of aggregate demand. Answer: False

Type: D E: 201 MA: 201 140. Other things equal, an increase in productivity will shift the aggregate supply curve rightward. Answer: True

Type: A E: 195 MA: 195 141. An increase in wealth from a substantial increase in stock prices will move the economy along a fixed aggregate demand curve. Answer: False

Type: A E: 195 MA: 195 142. In order to study the macroeconomy we must combine the prices and quantities generated in single-product markets into broad aggregates. Answer: True

McConnell/Brue: Economics, 16/e

Page 362

Chapter 11: Aggregate Demand and Aggregate Supply

Type: A E: 197 MA: 197 143. An increase in imports (independently of a change in the U.S. price level) will increase both U.S. aggregate supply and U.S. aggregate demand. Answer: False

Type: A E: 201 MA: 201 144. An increase in business excise taxes will shift the aggregate supply curve leftward. Answer: True

Type: A E: 200 MA: 200 145. A decrease in per unit production costs will shift the aggregate supply curve leftward. Answer: False

Type: A E: 203 MA: 203 146. Unemployment and inflation can coexist. Answer: True

Type: D E: 199 MA: 199 147. The shape of the aggregate supply curve is determined by what happens to aggregate demand as real output expands. Answer: False

Type: A E: 194 MA: 194 148. The real-balances effect indicates that inflation makes the public feel wealthier and they therefore spend more out of their current incomes. Answer: False

Type: D E: 194 MA: 194 149. In locating a particular aggregate demand curve it is assumed that the money supply is fixed. Answer: True

Type: F E: 198-199 MA: 198-199 Status: New 150. The aggregate supply curve (short-run) becomes steeper as the economy moves rightward and upward along it. Answer: True

Type: A E: 205 MA: 205 Status: New 151. Cost-push inflation is depicted as a rightward shift of the aggregate demand curve along an upsloping aggregate supply curve. Answer: False

Type: F E: 204 MA: 204 Status: New 152. A negative GDP gap can be caused by either a decrease in aggregate demand or a decrease in aggregate supply. Answer: True

McConnell/Brue: Economics, 16/e

Page 363

Chapter 11: Aggregate Demand and Aggregate Supply

Type: D E: 202 MA: 202 Status: New 153. The equilibrium price level and equilibrium level of real GDP occur at the intersection of the aggregate demand curve and the aggregate supply curve. Answer: True

Type: A E: 199 MA: 199 Status: New 154. The greater the upward slope of the AS curve, the larger is the realized multiplier effect of a change in investment spending. Answer: False

Type: A E: 204 MA: 204 Status: New 155. The price level in the United States is more flexible downward than upward. Answer: False

Appendix AD in relation to the AE model

Type: A Topic: 6 E: 211 MA: 211 156. The aggregate expenditures model and the aggregate demand curve can be reconciled because, other things equal, in the aggregate expenditures model: A) changes in the price level have no effect on the equilibrium level of GDP. B) an increase in the price level increases the real value of wealth. C) the level of aggregate expenditures and therefore the level of real GDP vary inversely with the price level. D) the level of aggregate expenditures and therefore the level of real GDP vary directly with the price level. Answer: C

Type: A Topic: 6 E: 212 MA: 212 157. In deriving the aggregate demand curve from the aggregate expenditures model we note that: A) the real-balances effect is irrelevant to both models. B) a change in the price level will have no impact on the aggregate expenditures schedule. C) an increase (decrease) in the price levels shifts the aggregate expenditures schedule upward (downward). D) an increase (decrease) in the price level shifts the aggregate expenditures schedule downward (upward). Answer: D

Type: A Topic: 6 E: 212 MA: 212 158. An increase in aggregate expenditures resulting from a decrease in the price level is equivalent to a: A) rightward shift of the aggregate demand curve. B) leftward shift of the aggregate demand curve. C) movement downward along a fixed aggregate demand curve. D) decrease in aggregate supply. Answer: C

McConnell/Brue: Economics, 16/e

Page 364

Chapter 11: Aggregate Demand and Aggregate Supply

Use the following to answer questions 159-160:

Type: G Topic: 6 E: 212 MA: 212 159. Refer to the above diagrams. A decline in aggregate expenditures from AE2 to AE1 resulting from the realbalances, interest-rate effect, and foreign purchases effects would be depicted as: A) a movement from A to C along aggregate demand curve AD1. B) a movement from C to A along aggregate demand curve AD1. C) a shift of aggregate demand from AD1 to AD2. D) a shift of aggregate demand from AD2 to AD1. Answer: B

Type: G Topic: 6 E: 212 MA: 212 160. Refer to the above diagrams. Assuming a constant price level, an increase in aggregate expenditures from AE1 to AE2 would: A) move the economy from A to C along AD1. C) increase aggregate demand from AD1 to AD2. B) move the economy from C to A along AD1. D) decrease aggregate demand from AD2 to AD1. Answer: C

McConnell/Brue: Economics, 16/e

Page 365

Chapter 11: Aggregate Demand and Aggregate Supply

Type: C Topic: 6 E: 212 MA: 212 161. An increase in net exports will shift the: A) aggregate expenditures curve upward and the aggregate demand curve rightward. B) aggregate expenditures curve upward and the aggregate demand curve leftward. C) aggregate expenditures curve downward and the aggregate demand curve rightward. D) aggregate expenditures curve downward and the aggregate demand curve leftward. Answer: A

Type: A Topic: 6 E: 211-212 MA: 211-212 162. An increase in investment spending caused by higher expected rates of return will: A) shift the aggregate supply curve to the left. B) move the economy up along an existing aggregate demand curve. C) shift the aggregate expenditures curve downward and the aggregate demand curve to the left. D) shift the aggregate expenditures curve upward and the aggregate demand curve to the right. Answer: D

Type: A Topic: 6 E: 212 MA: 212 163. An increase in aggregate expenditures resulting from some factor other than a change in the price level is equivalent to: A) a rightward shift of the aggregate demand curve in the AD-AS model. B) a leftward shift of the aggregate demand curve in the AD-AS model. C) a movement downward along a fixed aggregate demand curve in the AD-AS model. D) a decrease in aggregate supply in the AD-AS model. Answer: A

Type: A Topic: 6 E: 211 MA: 211 164. When deriving the aggregate demand (AD) curve from the aggregate expenditure model, an increase in U.S. product prices would cause an increase in: A) the value of household wealth and lower consumption expendtitures. B) interest rates and lower investment expenditures. C) exports and imports. D) U.S. resource prices and an increase in aggregate supply. Answer: B

Type: C Topic: 6 E: 212 MA: 212 165. (Advanced analysis) Assume that the MPS is .33 in an economy that has an aggregate supply curve with a slope of 1. An increase in investment spending of $10 billion will shift the aggregate demand curve rightward by: A) $30 billion and increase real GDP by $15 billion. B) $30 billion and increase real GDP by $30 billion. C) $10 billion and increase real GDP by $30 billion. D) $10 billion and increase real GDP by $10 billion. Answer: A

McConnell/Brue: Economics, 16/e

Page 366

Chapter 11: Aggregate Demand and Aggregate Supply

Type: C Topic: 6 E: 212 MA: 212 166. (Advanced analysis) Assume that the MPC is .8 in an economy that has an aggregate supply curve with a slope of 1. Also, suppose that the price level is flexible downward. A decrease in investment spending of $10 billion will shift the aggregate demand curve leftward by: A) $50 billion and decrease real GDP by $50 billion. B) $50 billion and decrease real GDP by $25 billion. C) $10 billion and decrease real GDP by $10 billion. D) $10 billion and decrease real GDP by $25 billion. Answer: B

McConnell/Brue: Economics, 16/e

Page 367

CHAPTER 12

Fiscal Policy

Topic 1. 2. 3. 4. 5. 6. 7. Fiscal policy; definitions Discretionary fiscal policy Financing deficits; disposing of surpluses Built-in stabilizers Full-employment budget Problems, criticisms, complications Fiscal policy: AE model (these questions assume coverage of AE model) 8. Fiscal policy: open economy complications Last Word True-False Multiple Choice Questions

Question numbers 1-8 9-49 50-55 56-82 83-117 118-131 132-137 138-144 145-147 148-164

____________________________________________________________

_______________________________________

____________________________________________________________

_______________________________________

Fiscal policy; definitions

Type: F Topic: 1 E: 214 MA: 214 1. In the Employment Act of 1946, the Federal government: A) applied the unemployment compensation program to intrastate workers. B) agreed to subsidize unemployed workers to the extent of 50 percent of their average incomes. C) committed itself to accept some degree of responsibility for the general levels of employment and prices. D) agreed to hire, through public works programs, any employees who cannot find jobs with private industry. Answer: C

Type: F Topic: 1 E: 215 MA: 215 2. Fiscal policy is carried out primarily by: A) the Federal government. B) state and local governments working together. Answer: A

C) state governments alone. D) local governments alone.

Chapter 12: Fiscal Policy

Type: D Topic: 1 E: 215 MA: 215 3. Discretionary fiscal policy refers to: A) any change in government spending or taxes that destabilizes the economy. B) the authority that the President has to change personal income tax rates. C) changes in taxes and government expenditures made by Congress to stabilize the economy. D) the changes in taxes and transfers that occur as GDP changes. Answer: C

Type: A Topic: 1 E: 215 MA: 215 4. Countercyclical discretionary fiscal policy calls for: A) surpluses during recessions and deficits during periods of demand-pull inflation. B) deficits during recessions and surpluses during periods of demand-pull inflation. C) surpluses during both recessions and periods of demand-pull inflation. D) deficits during both recessions and periods of demand-pull inflation. Answer: B

Type: D Topic: 1 E: 214 MA: 214 5. Fiscal policy refers to the: A) manipulation of government spending and taxes to stabilize domestic output, employment, and the price level. B) manipulation of government spending and taxes to achieve greater equality in the distribution of income. C) altering of the interest rate to change aggregate demand. D) fact that equal increases in government spending and taxation will be contractionary. Answer: A

Type: D Topic: 1 E: 215 MA: 215 6. Discretionary fiscal policy is so named because it: A) is undertaken at the option of the nation's central bank. B) occurs automatically as the nation's level of GDP changes. C) involves specific changes in T and G undertaken expressly for stabilization at the option of Congress. D) is invoked secretly by the Council of Economic Advisers. Answer: C

Type: D Topic: 1 E: 215 MA: 215 7. Expansionary fiscal policy is so named because it: A) involves an expansion of the nation's money supply. B) necessarily expands the size of government. C) is aimed at achieving greater price stability. D) is designed to expand real GDP. Answer: D

Type: D Topic: 1 E: 216 MA: 216 8. Contractionary fiscal policy is so named because it: A) involves a contraction of the nation's money supply. B) necessarily reduces the size of government. C) is aimed at reducing aggregate demand and thus achieving price stability. D) is expressly designed to contract real GDP. Answer: C

McConnell/Brue: Economics, 16/e

Page 370

Chapter 12: Fiscal Policy

Discretionary fiscal policy Type: A Topic: 2 E: 215 MA: 215 9. An economist who favors smaller government would recommend: A) tax cuts during recession and reductions in government spending during inflation. B) tax increases during recession and tax cuts during inflation. C) tax cuts during recession and tax increases during inflation. D) increases in government spending during recession and tax increases during inflation. Answer: A

Type: C Topic: 2 E: 216 MA: 216 10. If the MPS in an economy is .1, government could shift the aggregate demand curve rightward by $40 billion by: A) increasing government spending by $4 billion. C) decreasing taxes by $4 billion. B) increasing government spending by $40 billion. D) increasing taxes by $4 billion. Answer: A

Type: C Topic: 2 E: 216 MA: 216 11. If the MPC in an economy is .8, government could shift the aggregate demand curve rightward by $100 billion by: A) increasing government spending by $25 billion. C) decreasing taxes by $25 billion. B) increasing government spending by $80 billion. D) decreasing taxes by $100 billion. Answer: C

Type: A Topic: 2 E: 215 MA: 215 12. An economist who favored expanded government would recommend: A) tax cuts during recession and reductions in government spending during inflation. B) tax increases during recession and tax cuts during inflation. C) tax cuts during recession and tax increases during inflation. D) increases in government spending during recession and tax increases during inflation. Answer: D

Type: C Topic: 2 E: 216 MA: 216 13. If the MPS in an economy is .4, government could shift the aggregate demand curve leftward by $50 billion by: A) reducing government expenditures by $125 billion. B) reducing government expenditures by $20 billion. C) increasing taxes by $50 billion. D) increasing taxes by $250 billion. Answer: B

Type: C Topic: 2 E: 216 MA: 216 14. If the MPC in an economy is .75, government could shift the aggregate demand curve leftward by $60 billion by: A) reducing government expenditures by $12 billion. B) reducing government expenditures by $60 billion. C) increasing taxes by $15 billion. D) increasing taxes by $20 billion. Answer: D

McConnell/Brue: Economics, 16/e

Page 371

Chapter 12: Fiscal Policy

Type: A Topic: 2 E: 215 MA: 215 15. Discretionary fiscal policy will stabilize the economy most when: A) deficits are incurred during recessions and surpluses during inflations. B) the budget is balanced each year. C) deficits are incurred during inflations and surpluses during recessions. D) budget surpluses are continuously incurred. Answer: A

Type: A Topic: 2 E: 216 MA: 216 16. The effect of a government surplus on the equilibrium level of GDP is substantially the same as: A) a decrease in saving. C) an increase in consumption. B) an increase in saving. D) an increase in investment. Answer: B

Type: C Topic: 2 E: 215 MA: 215 17. Assume the economy is at full employment and that investment spending declines dramatically. Under these conditions government fiscal policy should be directed toward: A) an equality of tax receipts and government expenditures. B) an excess of tax receipts over government expenditures. C) an excess of government expenditures over tax receipts. D) a reduction of subsidies and transfer payments and an increase in tax rates. Answer: C

Type: C Topic: 2 E: 215 MA: 215 18. Suppose that the economy is in the midst of a recession. Which of the following policies would be consistent with active fiscal policy? A) a Congressional proposal to incur a Federal surplus to be used for the retirement of public debt B) a reduction in agricultural subsidies and veterans' benefits C) a postponement of a highway construction program D) a reduction in Federal tax rates on personal and corporate income Answer: D

Type: A Topic: 2 E: 217 MA: 217 19. Assume that aggregate demand in the economy is excessive, causing demand-pull inflation. Which of the following would be most in accord with appropriate government fiscal policy? A) an increase in Federal income tax rates B) an increase in the size of income tax exemptions for each dependent C) passage of legislation providing for the construction of 8,000 new school buildings D) an increase in soil conservation subsidies to farmers Answer: A

McConnell/Brue: Economics, 16/e

Page 372

Chapter 12: Fiscal Policy

Type: A Topic: 2 E: 216 MA: 216 20. In a certain year the aggregate amount demanded at the existing price level consists of $100 billion of consumption, $40 billion of investment, $10 billion of net exports, and $20 billion of government purchases. Full-employment GDP is $120 billion. To obtain price level stability under these conditions the government should: A) increase tax rates and reduce government spending. B) discourage personal saving by reducing the interest rate on government bonds. C) increase government expenditures. D) encourage private investment by reducing corporate income taxes. Answer: A

Type: A Topic: 2 E: 216 MA: 216 21. In a certain year the aggregate amount demanded at the existing price level consists of $100 billion of consumption, $40 billion of investment, $10 billion of net exports, and $20 billion of government purchases. Full-employment GDP is $200 billion. To obtain full employment under these conditions the government should: A) encourage personal saving by increasing the interest rate on government bonds. B) decrease government expenditures. C) reduce tax rates and increase government spending. D) discourage private investment by increasing corporate income taxes. Answer: C

Type: A Topic: 2 E: 215 MA: 215 22. An appropriate fiscal policy for a severe recession is: A) a decrease in government spending. C) appreciation of the dollar. B) a decrease in tax rates. D) an increase in interest rates. Answer: B

Type: A Topic: 2 E: 217 MA: 217 23. An appropriate fiscal policy for severe demand-pull inflation is: A) an increase in government spending. C) a reduction in interest rates. B) depreciation of the dollar. D) a tax rate increase. Answer: D

Type: A Topic: 2 E: 215 MA: 215 24. Suppose that in an economy with a MPC of .5 the government wanted to shift the aggregate demand curve rightward by $80 billion at each price level to expand real GDP. It could: A) reduce taxes by $160 billion. C) reduce taxes by $40 billion. B) increase government spending by $80 billion. D) increase government spending by $40 billion. Answer: D

Type: A Topic: 2 E: 215 MA: 215 25. In an aggregate demand-aggregate supply diagram, equal decreases in government spending and taxes will: A) shift the AD curve to the right. C) not affect the AD curve. B) increase the equilibrium GDP. D) shift the AD curve to the left. Answer: D

McConnell/Brue: Economics, 16/e

Page 373

Chapter 12: Fiscal Policy

Type: A Topic: 2 E: 217 MA: 217 26. Suppose that in an economy with a MPC of .8 the government wanted to shift the aggregate demand curve leftward by $40 billion at each price level to remedy demand-pull inflation. It could: A) increase taxes by $10 billion. C) reduce government spending by $5 billion. B) reduce government spending by $40 billion. D) increase taxes by $20 billion. Answer: A

Type: A Topic: 2 E: 215 MA: 215 27. Which of the following represents the most expansionary fiscal policy? A) a $10 billion tax cut C) a $10 billion tax increase B) a $10 billion increase in government spending D) a $10 billion decrease in government spending Answer: B

Type: A Topic: 2 E: 217 MA: 217 28. Which of the following represents the most contractionary fiscal policy? A) a $30 billion tax cut C) a $30 billion tax increase B) a $30 billion increase in government spending D) a $30 billion decrease in government spending Answer: D

Type: A Topic: 2 E: 217 MA: 217 29. A contractionary fiscal policy is shown as a: A) rightward shift in the economy's aggregate demand curve. B) rightward shift in the economy's aggregate supply curve. C) movement along an existing aggregate demand curve. D) leftward shift in the economy's aggregate demand curve. Answer: D

Type: A Topic: 2 E: 215 MA: 215 30. A expansionary fiscal policy is shown as a: A) rightward shift in the economy's aggregate demand curve. B) movement along an existing aggregate demand curve. C) leftward shift in the economy's aggregate supply curve. D) leftward shift in the economy's aggregate demand curve. Answer: A

Type: A Topic: 2 E: 216 MA: 216 31. A tax reduction of a specific amount will be more expansionary, the: A) smaller is the economy's MPC. C) smaller is the economy's multiplier. B) larger is the economy's MPC. D) less the economy's built-in stability. Answer: B

Type: A Topic: 2 E: 216 MA: 216 32. A specific reduction in government spending will dampen demand-pull inflation by a greater amount, the: A) smaller is the economy's MPC. C) smaller is the economy's MPS. B) flatter is the economy's aggregate supply curve. D) less the economy's built-in stability. Answer: C

McConnell/Brue: Economics, 16/e

Page 374

Chapter 12: Fiscal Policy

Use the following to answer questions 33-39:
AD3 AS

Price level

AD1

AD2 AD0 0

Qf Real GDP

Type: G Topic: 2 E: 216-217 MA: 216-217 33. Refer to the above diagram, in which Qf is the full-employment output. A contractionary fiscal policy would be most appropriate if the economy's present aggregate demand curve were at: A) AD0 B) AD1 C) AD2 D) AD3 Answer: D

Type: G Topic: 2 E: 215 MA: 215 34. Refer to the above diagram, in which Qf is the full-employment output. An expansionary fiscal policy would be most appropriate if the economy's present aggregate demand curve were at: A) AD0 B) AD1 C) AD2 D) AD3 Answer: A

Type: G Topic: 2 E: 215 MA: 215 35. Refer to the above diagram, in which Qf is the full-employment output. If the economy's present aggregate demand curve is AD2: A) the most appropriate fiscal policy is an increase of government expenditures or a reduction of taxes. B) the most appropriate fiscal policy is a reduction of government expenditures or an increase of taxes. C) government should undertake neither an expansionary nor a contractionary fiscal policy. D) the economy is achieving its full capacity output. Answer: C

Type: G Topic: 2 E: 215 MA: 215 Status: New 36. Refer to the above diagram, in which Qf is the full-employment output. If the economy's current aggregate demand curve is AD0, it is experiencing: A) a positive GDP gap. B) a negative GDP gap. C) inflation. D) an adverse supply shock. Answer: B

Type: G Topic: 2 E: 216 MA: 216 Status: New 37. Refer to the above diagram, in which Qf is the full-employment output. If the economy's current aggregate demand curve is AD3, it is experiencing: A) a positive GDP gap. B) a negative GDP gap. C) a recession. D) cost-push inflation. Answer: A

McConnell/Brue: Economics, 16/e

Page 375

Chapter 12: Fiscal Policy

Type: G Topic: 2 E: 215 MA: 215 Status: New 38. Refer to the above diagram, in which Qf is the full-employment output. If the economy's current aggregate demand curve is AD0, it would be appropriate for the government to: A) reduce government expenditures and taxes by equal-size amounts. B) reduce government expenditures or increase taxes. C) increase government expenditures or reduce taxes. D) reduce unemployment compensation benefits. Answer: C

Type: G Topic: 2 E: 216-217 MA: 216-217 Status: New 39. Refer to the above diagram, in which Qf is the full-employment output. If the economy's current aggregate demand curve is AD3, it would be appropriate for the government to: A) reduce government expenditures and taxes by equal-size amounts. B) reduce government expenditures or increase taxes. C) increase government expenditures or reduce taxes. D) reduce unemployment compensation benefits. Answer: B

Use the following to answer questions 40-45:

AS
AD1

Price level

AD3

0

Qf Real GDP

Type: G Topic: 2 E: 215-216 MA: 215-216 40. Refer to the above diagram, in which Qf is the full-employment output. If aggregate demand curve AD1 describes the current situation, appropriate fiscal policy would be to: A) increase taxes and reduce government spending to shift the aggregate demand curve rightward to AD2. B) reduce taxes on businesses to shift the aggregate supply curve leftward. C) reduce taxes and increase government spending to shift the aggregate demand curve from AD1 to AD2. D) do nothing since the economy appears to be achieving full-employment real GDP. Answer: C

McConnell/Brue: Economics, 16/e

Page 376

Chapter 12: Fiscal Policy

Type: G Topic: 2 E: 216-217 MA: 216-217 41. Refer to the above diagram, in which Qf is the full-employment output. If aggregate demand curve AD3 describes the current situation, appropriate fiscal policy would be to: A) do nothing since the economy appears to be achieving full-employment real output. B) increase taxes and reduce government spending to shift the aggregate demand curve leftward from AD3 to AD2. C) increase taxes on businesses to shift the aggregate supply curve rightward to reduce the price level. D) increase taxes and reduce government spending to shift the aggregate demand curve from AD3 to AD1. Answer: B

Type: G Topic: 2 E: 215 MA: 215 42. Refer to the above diagram, in which Qf is the full-employment output. If aggregate demand curve AD2 describes the current situation, appropriate fiscal policy would be to: A) do nothing since the economy appears to be achieving full-employment real output. B) increase taxes and reduce government spending to shift the aggregate demand curve rightward from AD2 to AD3. C) increase taxes on businesses to shift the aggregate supply curve rightward to reduce the price level. D) reduce taxes and increase government spending to shift the aggregate demand curve from AD2 to AD1. Answer: A

Type: G Topic: 2 E: 216-217 MA: 216-217 43. Refer to the above diagram, in which Qf is the full-employment output. The shift of the aggregate demand curve from AD3 to AD2 is consistent with: A) an expansionary fiscal policy. C) a contractionary fiscal policy. B) a major recession. D) demand-pull inflation. Answer: C

Type: G Topic: 2 E: 215 MA: 215 44. Refer to the above diagram, in which Qf is the full-employment output. The shift of the aggregate demand curve from AD1 to AD2 is consistent with: A) an expansionary fiscal policy. C) a contractionary fiscal policy. B) a major recession. D) severe demand-pull inflation. Answer: A

Type: G Topic: 2 E: 216-217 MA: 216-217 45. Refer to the above diagram, in which Qf is the full-employment output. The shift in the aggregate demand curve from AD3 to AD2 could result from which of the following fiscal policy actions? A) a tax reduction B) a tax reduction accompanied by an even larger reduction in government spending C) a tax increase accompanied by an even larger increase in government spending D) an increase in government spending Answer: B

McConnell/Brue: Economics, 16/e

Page 377

Chapter 12: Fiscal Policy

Type: C Topic: 2 E: 217 MA: 217 Status: New 46. Suppose the price level is fixed, the MPC is .5, and the GDP gap is a negative $80 billion. To achieve fullemployment output (exactly), government should: A) increase government expenditures by $80 billion. B) reduce government expenditures by $40 billion. C) reduce taxes by $40 billion. D) reduce taxes by $80 billion. Answer: D

Type: C Topic: 2 E: 217 MA: 217 Status: New 47. Suppose the price level is fixed, the MPC is .5, and the GDP gap is a negative $100 billion. To achieve full-employment output (exactly), government should: A) increase government expenditures by $100 billion. B) increase government expenditures by $50 billion. C) reduce taxes by $50 billion. D) reduce taxes by $200 billion. Answer: B

Type: C Topic: 2 E: 217 MA: 217 Status: New 48. Suppose the price level is fixed, the MPC is .8, the GDP gap is a negative $200 billion. To achieve fullemployment output (exactly), government should: A) increase government expenditures by $200 billion. B) reduce taxes by $200 billion. C) increase government expenditures by $40 billion. D) reduce taxes by $160 billion. Answer: C

Type: C Topic: 2 E: 217 MA: 217 Status: New 49. Suppose the price level is fixed, the MPC is .8, and the GDP gap is a negative $320 billion. To achieve full-employment output (exactly), government should: A) increase government expenditures by $320 billion. B) increase government expenditures by $80 billion. C) reduce taxes by $320 billion. D) reduce taxes by $80 billion. Answer: D

Financing deficits; disposing of surpluses

Type: A Topic: 3 E: 217-218 MA: 217-218 50. Which of the following fiscal actions would be the most effective in curbing inflation? A) incurring a budget deficit by borrowing from the public B) incurring a budget surplus which is used to retire debt held by commercial banks C) incurring a budget surplus and impounding that surplus D) incurring a budget surplus which is used to retire debt held by the public Answer: C

McConnell/Brue: Economics, 16/e

Page 378

Chapter 12: Fiscal Policy

Type: A Topic: 3 E: 218 MA: 218 51. If the government increases its spending during recession to assist the economy, the funds for such expenditures must come from some source. Which of the following sources would be the most expansionary? A) additional taxes on personal incomes C) borrowing from the public B) creating new money D) additional taxes on corporate profits Answer: B

Type: A Topic: 3 E: 218 MA: 218 52. The greatest expansionary impact of a budget deficit will occur when the: A) government finances the deficit by obtaining newly printed money. B) government borrows the money from the general public. C) economy is operating in the intermediate range of its aggregate supply curve. D) marginal propensity to save for the economy is high. Answer: A

Type: A Topic: 3 E: 218 MA: 218 53. The greatest anti-inflationary impact of a budget surplus will occur when the Federal government: A) uses the surplus funds to pay off its outstanding debt. B) impounds the surplus funds and lets them stand idle. C) uses the surplus funds to expand transfer payments. D) gives the surplus funds to the states through federal grants. Answer: B

Type: A Topic: 3 E: 218 MA: 218 54. Other things equal, which of the policies will have the most expansionary effect on the economy? A) a balanced budget B) a budget surplus held as an idle money balance C) a budget deficit financed by creating new money D) a budget surplus used for debt retirement Answer: C

Type: A Topic: 3 E: 218 MA: 218 55. Other things equal, which of the policies will have the most contractionary effect on the economy? A) a balanced budget B) a budget surplus held as an idle money balance C) a budget deficit financed by creating new money D) a budget surplus used for debt retirement Answer: B

McConnell/Brue: Economics, 16/e

Page 379

Chapter 12: Fiscal Policy

Built-in stabilizers

Type: A Topic: 4 E: 218 MA: 218 56. Built-in stability means that: A) an annually balanced budget will offset the procyclical tendencies created by state and local finance and thereby stabilize the economy. B) with given tax rates and expenditures policies, a rise in domestic income will reduce a budget deficit or produce a budget surplus while a decline in income will result in a deficit or a lower budget surplus. C) Congress will automatically change the tax structure and expenditure programs to correct upswings and downswings in business activity. D) government expenditures and tax receipts automatically balance over the business cycle, though they may be out of balance in any single year. Answer: B

Type: C Topic: 4 E: 219 MA: 219 57. If Congress adjusted the U.S. tax system so that the MPC was reduced, the A) economy would become more inflation prone. C) stability of the economy would be unaffected. B) economy would become less stable. D) economy would become more stable. Answer: D

Type: A Topic: 4 E: 219 MA: 219 58. A major advantage of the built-in or automatic stabilizers is that they: A) simultaneously stabilize the economy and reduce the absolute size of the public debt. B) automatically produce surpluses during recessions and deficits during inflations. C) require no legislative action by Congress to be made effective. D) guarantee that the Federal budget will be balanced over the course of the business cycle. Answer: C

Type: A Topic: 4 E: 219 MA: 219 59. Which of the following best describes the built-in stabilizers as they function in the United States? A) The size of the balanced-budget multiplier varies inversely with the level of GDP. B) Personal and corporate income tax collections automatically fall and transfers and subsidies automatically rise as GDP rises. C) Personal and corporate income tax collections and transfers and subsidies all automatically vary inversely with the level of GDP. D) Personal and corporate income tax collections automatically rise and transfers and subsidies automatically decline as GDP rises. Answer: D

Type: A Topic: 4 E: 220 MA: 220 60. The effectiveness of the built-in or automatic stabilizers is limited because: A) the stabilizers produce budget surpluses during recessions. B) transfer payments and subsidies increase during inflation and decrease during recessions. C) the offset the stabilizers provide to a change in private spending is less than the change in private spending. D) the stabilizers raise the general price level regardless of the phase of the business cycle. Answer: C

McConnell/Brue: Economics, 16/e

Page 380

Chapter 12: Fiscal Policy

Type: F Topic: 4 E: 220 MA: 220 61. Which of the following statements is correct? A) Built-in stability only partially offsets fluctuations in economic activity. B) Built-in stability works in halting inflation, but it cannot alleviate unemployment. C) Built-in stability can be relied on to eliminate completely any fluctuation in economic activity. D) Built-in stability has eliminated the need for discretionary fiscal policy. Answer: A

Use the following to answer questions 62-67:

Type: G Topic: 4 E: 219 MA: 219 62. Refer to the above diagram in which T is tax revenues and G is government expenditures. All figures are in billions. This diagram portrays the idea of: A) progressive taxation. C) the balanced-budget multiplier. B) built-in stability. D) discretionary fiscal policy. Answer: B

Type: G Topic: 4 E: 219 MA: 219 63. Refer to the above diagram in which T is tax revenues and G is government expenditures. All figures are in billions. The tax system of this economy is such that: A) it is regressive. C) tax revenues equal 50 percent of GDP. B) it is progressive. D) it tends to destabilize the economy. Answer: C

Type: G Topic: 4 E: 219 MA: 219 Status: New 64. Refer to the above diagram in which T is tax revenues and G is government expenditures. All figures are in billions. The equilibrium level of GDP in this economy: A) is $400. B) is greater than $400. C) is less than $400. D) cannot be determined from the information given. Answer: D

Type: G Topic: 4 E: 219 MA: 219 65. Refer to the above diagram in which T is tax revenues and G is government expenditures. All figures are in billions. If GDP is $400: A) there will be a budget deficit. C) the budget will be balanced. B) there will be a budget surplus. D) the macroeconomy will be in equilibrium. Answer: C

McConnell/Brue: Economics, 16/e

Page 381

Chapter 12: Fiscal Policy

Type: G Topic: 4 E: 219 MA: 219 66. Refer to the above diagram in which T is tax revenues and G is government expenditures. All figures are in billions. The budget will entail a deficit: A) at all levels of GDP. C) at any level of GDP below $400. B) at any level of GDP above $400. D) only when GDP is stable. Answer: C

Type: G Topic: 4 E: 219 MA: 219 67. Refer to the above diagram in which T is tax revenues and G is government expenditures. All figures are in billions. In this economy: A) tax revenues and government spending both vary directly with GDP. B) tax revenues vary directly with GDP, but government spending is independent of GDP. C) tax revenues and government spending both vary inversely with GDP. D) government spending varies directly with GDP, but tax revenues are independent of GDP. Answer: B

Use the following to answer questions 68-70:

Type: G Topic: 4 E: 219 MA: 219 68. Refer to the above diagram. Which tax system has the most built-in stability? A) T4 B) T3 C) T2 D) T1 Answer: D

Type: G Topic: 4 E: 219 MA: 219 69. Refer to the above diagram. Which tax system has the least built-in stability? A) T4 B) T3 C) T2 D) T1 Answer: A

Type: G Topic: 4 E: 219 MA: 219 70. Refer to the above diagram. Which tax system will generate the largest cyclical deficits? A) T4 B) T3 C) T2 D) T1 Answer: D

McConnell/Brue: Economics, 16/e

Page 382

Chapter 12: Fiscal Policy

Use the following to answer questions 71-74:

Type: G Topic: 4 E: 219-220 MA: 219-220 71. (Advanced analysis) Refer to the above diagram, in which C1 is the before-tax consumption schedule. The consumption schedule represented by C2 reflects: A) a higher MPC than is embodied in C1. C) a proportional tax system. B) a regressive tax system. D) a progressive tax system. Answer: B

Type: G Topic: 4 E: 219-220 MA: 219-220 72. (Advanced analysis) Refer to the above diagram, in which C1 is the before-tax consumption schedule. The consumption schedule represented by C3 reflects: A) a progressive tax system. C) a regressive tax system. B) a proportional tax system. D) a higher MPC than is embodied in C1. Answer: B

Type: G Topic: 4 E: 219-220 MA: 219-220 73. (Advanced analysis) Refer to the above diagram, in which C1 is the before-tax consumption schedule. The consumption schedule represented by C4 reflects: A) a progressive tax system. C) a regressive tax system. B) a proportional tax system. D) a higher MPC than is embodied in C1. Answer: A

Type: G Topic: 4 E: 219-220 MA: 219-220 74. (Advanced analysis) Refer to the above diagram, in which C1 is the before-tax consumption schedule. Other things being equal, the economy would enjoy the greatest built-in stability with consumption schedule: A) C1. B) C2. C) C3. D) C4. Answer: D

McConnell/Brue: Economics, 16/e

Page 383

Chapter 12: Fiscal Policy

Use the following to answer questions 75-78: Answer the next question(s) on the basis of the following before-tax consumption schedule for a closed economy:
Gross domestic product (GDP) $ 0 100 200 300 400 Consumption (C) $ 40 120 200 280 360

Type: T Topic: 4 E: 216 MA: 216 75. Refer to the above data. If a lump-sum tax (the same tax amount at each level of GDP) of $40 is now imposed in this economy, the consumption schedule will be:
(a) GDP C $ 0 $ 8 100 88 200 168 300 248 400 320 Answer: A (b) GDP C $ 0 $ 0 100 80 200 160 300 240 400 320 (c) GDP C $ 0 $ 10 100 90 200 170 300 250 400 310 (d) GDP C $ 0 $ 0 100 60 200 120 300 180 400 240

Type: T Topic: 4 E: 216 MA: 216 76. Refer to the above data. If a lump-sum tax (the same tax amount at each level of GDP) of $40 is imposed in this economy, the tax system: A) is regressive. C) is progressive. B) is proportional. D) may be either proportional or progressive. Answer: A

Type: T Topic: 4 E: 216 MA: 216 77. Refer to the above data. If a lump-sum tax (the same tax amount at each level of GDP) of $40 is imposed in this economy, the marginal propensity to consume is: A) .8 before taxes and .6 after taxes. C) .6 before taxes and .8 after taxes. B) .8 both before and after taxes. D) .8 before taxes and .4 after taxes. Answer: B

Type: T Topic: 4 E: 216 MA: 216 78. Refer to the above data. If a lump-sum tax (the same tax amount at each level of GDP) of $40 is imposed in this economy, we can conclude that the tax: A) enhances the economy's built-in stability. B) reduces the economy's built-in stability. C) neither increases nor decreases built-in stability. D) increases the MPC and therefore increases the size of the multiplier. Answer: C

McConnell/Brue: Economics, 16/e

Page 384

Chapter 12: Fiscal Policy

Use the following to answer questions 79-82: (Advanced analysis) Answer the next question(s) on the basis of the following before-tax consumption schedule for an economy:
Gross domestic product (GDP) $100 200 300 400 500 Consumption (C) $140 200 260 320 380

Type: T Topic: 4 E: 219 MA: 219 79. Refer to the above data. If a 10 percent proportional tax on income is imposed, the consumption schedule will now be:

(a) GDP $100 200 300 400 500

C $134 188 242 296 350

(b) GDP $100 200 300 400 500

C $144 212 278 344 410

(c) GDP $100 200 300 400 500

C $134 194 254 324 374

(d) GDP $100 200 300 400 500

C $146 218 286 352 412

Answer: A

Type: T Topic: 4 E: 219 MA: 219 80. Refer to the above data. The 10 percent proportional tax on income would cause: A) both consumption and saving to decrease by larger and larger absolute amounts as GDP rises. B) both consumption and saving to decrease by smaller and smaller absolute amounts as GDP rises. C) consumption to decrease by larger amounts and saving to decrease by smaller amounts as GDP rises. D) no change in the amounts consumed and saved at each level of GDP. Answer: A

Type: T Topic: 4 E: 219 MA: 219 81. Refer to the above data. The 10 percent proportional tax on income would: A) reduce the MPC from .6 to .54. C) reduce the MPC from .6 to .5. B) not affect the size of the MPC. D) increase the MPC from .6 to .64. Answer: A

Type: T Topic: 4 E: 219 MA: 219 82. Refer to the above data. A 10 percent proportional tax on income would: A) affect neither the size of the multiplier nor the stability of the economy. B) increase the size of the multiplier and make the economy more stable. C) increase the size of the multiplier and make the economy less stable. D) reduce the size of the multiplier and make the economy more stable. Answer: D

McConnell/Brue: Economics, 16/e

Page 385

Chapter 12: Fiscal Policy

Full-employment budget

Type: D Topic: 5 E: 220 MA: 220 83. The full-employment budget refers to: A) the inflationary impact that the automatic stabilizers have in a full-employment economy. B) that portion of a full-employment GDP that is not consumed in the year it is produced. C) the size of the Federal government's budgetary surplus or deficit when the economy is operating at full employment. D) the number of workers who are underemployed when the level of unemployment is 4 to 5 percent. Answer: C

Type: D Topic: 5 E: 220 MA: 220 84. The full-employment budget tells us: A) that in a full-employment economy the Federal budget should be in balance. B) that tax revenues should vary inversely with GDP. C) what the size of the Federal budget deficit or surplus would be if the economy was at full employment. D) the actual budget deficit or surplus realized in any given year. Answer: C

Type: A Topic: 5 E: 220 MA: 220 85. Which of the following statements is correct? A) The full-employment budget and the actual budget differ because the latter does not take government transfer payments into account. B) The full-employment budget is less likely to show a deficit than is the actual budget. C) The full-employment budget and the actual budget will show the same size deficit or surplus in any given fiscal year. D) The full-employment budget is more likely to show a deficit than is the actual budget. Answer: B

Type: A Topic: 5 E: 220 MA: 220 86. If the economy has a full-employment budget surplus, this means that: A) the public sector is exerting an expansionary impact on the economy. B) tax revenues would exceed government expenditures if full employment were achieved. C) the actual budget is necessarily also in surplus. D) the economy is actually operating at full employment. Answer: B

Type: A Topic: 5 E: 222 MA: 222 87. The actual budget deficit of the Federal government in 1991 was about $269 billion. On the basis of this information it: A) can be concluded that the economy was faced with serious inflation in 1991. B) cannot be determined whether fiscal policy had an expansionary or a contractionary impact in 1991. C) can be concluded that fiscal policy was contractionary in 1991. D) can be concluded that fiscal policy was expansionary in 1991. Answer: B

McConnell/Brue: Economics, 16/e

Page 386

Chapter 12: Fiscal Policy

Type: D Topic: 5 E: 220 MA: 220 88. When current government expenditures equal current tax revenues and the economy is achieving full employment: A) the full-employment budget has neither a deficit nor a surplus. B) the full-employment budget may have either a deficit or a surplus. C) fiscal policy is contractionary. D) nominal GDP and real GDP are equal. Answer: A

Type: A Topic: 5 E: 220 MA: 220 89. When current government expenditures exceed current tax revenues and the economy is achieving full employment: A) the full-employment budget has neither a deficit nor a surplus. B) the full-employment budget has a deficit. C) fiscal policy is contractionary. D) nominal GDP and real GDP are equal. Answer: B

Type: A Topic: 5 E: 220 MA: 220 90. When current tax revenues exceed current government expenditures and the economy is achieving full employment: A) the full-employment budget has neither a deficit nor a surplus. B) the full-employment budget may have either a deficit or a surplus. C) the full-employment budget has a surplus. D) nominal GDP and real GDP are equal. Answer: C

Type: A Topic: 5 E: 216 MA: 216 91. Suppose the government purposely changes the economy's full-employment budget from a deficit of 3 percent of real GDP to a surplus of 1 percent of real GDP. The government is engaging in a(n): A) expansionary fiscal policy. C) neutral fiscal policy. B) contractionary fiscal policy. D) high-interest rate policy. Answer: B

Type: A Topic: 5 E: 215 MA: 215 92. Suppose the government purposely changes the economy's full-employment budget from a deficit of 0 percent of real GDP to a deficit of 3 percent of real GDP. The government is engaging in a(n): A) expansionary fiscal policy. C) neutral fiscal policy. B) contractionary fiscal policy. D) low-interest rate policy. Answer: A

Type: A Topic: 5 E: 216 MA: 216 93. Suppose the government cuts taxes to keep the economy's full-employment budget in balance when the economy is expanding. The government is engaging in a(n): A) contractionary fiscal policy. C) low-interest rate policy. B) expansionary fiscal policy. D) neutral fiscal policy. Answer: D

McConnell/Brue: Economics, 16/e

Page 387

Chapter 12: Fiscal Policy

Use the following to answer questions 94-97:
Actual budget, percent of GDP (- deficits; + surpluses) 0 -3 -5 -2 +2 Full-employment budget, percent of GDP (- deficits; + surpluses) 0 0 -2 -2 +1

Year 1998 1999 2000 2001 2002

Type: T Topic: 5 E: 215-216 MA: 215-216 94. Refer to the above data for a fictional economy. The changes in the budget conditions between 1998 and 1999 best reflect: A) demand-pull inflation. C) a recession. B) an expansionary fiscal policy. D) a contractionary fiscal policy. Answer: C

Type: T Topic: 5 E: 215 MA: 215 95. Refer to the above data for a fictional economy. The changes in the budget conditions between 1999 and 2000 best reflect: A) demand-pull inflation. C) a tax increase. B) an expansionary fiscal policy. D) a contractionary fiscal policy. Answer: B

Type: T Topic: 5 E: 215 MA: 215 96. Refer to the above table for a fictional economy. The changes in the budget conditions between 2000 and 2001 best reflect: A) demand-pull inflation. B) cost-push inflation. C) an expansion of real GDP and an automatic increase in tax revenues. D) a contractionary fiscal policy. Answer: C

Type: T Topic: 5 E: 216 MA: 216 97. Refer to the above table for a fictional economy. The changes in the budget conditions between 2001 and 2002 best reflect a(n): A) recession. B) expansionary fiscal policy. C) tax increase. D) contractionary fiscal policy. Answer: D

McConnell/Brue: Economics, 16/e

Page 388

Chapter 12: Fiscal Policy

Use the following to answer questions 98-104:

Government purchases and tax revenues

T1 d T2

b c a

G

0

A

B

GDP

Type: G Topic: 5 E: 221 MA: 221 98. Refer to the above diagram. Assume that G and T1 are the relevant curves and that the economy is currently at B, which is its full-employment GDP. This economy has a: A) full-employment budget surplus only. B) full-employment budget deficit only. C) full-employment budget surplus and an actual budget surplus. D) full-employment deficit and an actual budget deficit. Answer: C

Type: G Topic: 5 E: 221 MA: 221 99. Refer to the above diagram. Assume that G and T1 are the relevant curves, the economy is currently at A, and the full-employment GDP is B. This economy has a: A) full-employment budget surplus. C) actual budget deficit. B) full-employment budget deficit. D) actual budget surplus. Answer: A

Type: G Topic: 5 E: 221 MA: 221 100. Refer to the above diagram. Assume that G and T1 are the relevant curves, the economy is currently at A, and the full-employment GDP is B. This economy has a: A) full-employment budget deficit. B) actual budget deficit. C) actual budget surplus. D) neither a surplus nor deficit in the actual budget. Answer: D

Type: G Topic: 5 E: 221 MA: 221 101. Refer to the above diagram. Assume that G and T2 are the relevant curves, the economy is currently at A, and the full-employment GDP is B. This economy has a: A) full-employment budget surplus. C) full-employment budget deficit. B) actual budget deficit. D) actual budget surplus. Answer: B

McConnell/Brue: Economics, 16/e

Page 389

Chapter 12: Fiscal Policy

Type: G Topic: 5 E: 221 MA: 221 102. Refer to the above diagram. Assume that G and T1 are the relevant curves, the economy is currently at B, and the full-employment GDP is A. This economy has a: A) full-employment budget surplus. C) full-employment budget deficit. B) actual budget deficit. D) actual budget surplus. Answer: D

Type: G Topic: 5 E: 221 MA: 221 103. Refer to the above diagram. Discretionary fiscal policy designed to slow the economy is illustrated by: A) the shift of curve T1 to T2 . C) a movement from a to c along curve T2. B) the shift of curve T2 to T1. D) a movement from d to b along curve T1. Answer: B

Type: G Topic: 5 E: 221 MA: 221 104. Refer to the above diagram. Discretionary fiscal policy designed to expand GDP is illustrated by: A) the shift of curve T1 to T2. C) a movement from a to c along curve T2. B) the shift of curve T2 to T1. D) a movement from d to b along curve T1. Answer: A

Use the following to answer questions 105-107:

Government purchases (G) and tax revenues (T)

T G

0

H

J

K

L

GDP

Type: G Topic: 5 E: 221 MA: 221 105. If the full-employment GDP for the above economy is at L, the: A) actual budget will have a deficit. C) actual budget will have a surplus. B) full-employment budget will have a deficit. D) full-employment budget will have a surplus. Answer: D

Type: G Topic: 5 E: 221 MA: 221 106. With the expenditures programs and the tax system shown in the above diagram: A) the public budget will be expansionary at all GDP levels above K, and contractionary at all GDP levels below K. B) the public budget will be a destabilizing force at all levels of GDP. C) deficits will occur at income levels below K, and surpluses above K. D) deficits will occur at income levels below H, and surpluses above H. Answer: C

McConnell/Brue: Economics, 16/e

Page 390

Chapter 12: Fiscal Policy

Type: G Topic: 5 E: 221 MA: 221 107. Refer to the above diagram. The degree of built-in stability in the above economy could be increased by: A) reducing government purchases so that the purchases line shifts downward but parallel to its present position. B) changing the tax system so that the tax line is shifted downward but parallel to its present position. C) changing the tax system so that the tax line has a greater slope. D) altering the government expenditures line so that it has a positive slope. Answer: C

Type: A Topic: 5 E: 215 MA: 215 108. An effective expansionary fiscal policy will: A) reduce a cyclical deficit, but necessarily increase the actual deficit. B) reduce a full-employment. C) increase the full-employment deficit but reduce the cyclical deficit. D) always result in a balanced actual budget once full-employment is achieved. Answer: C

Use the following to answer questions 109-114:

Type: G Topic: 5 E: 219 MA: 219 109. Refer to the above diagram where T is tax revenues and G is government expenditures. All figures are in billions of dollars. If the full-employment GDP is $400 billion while the actual GDP is $200 billion, the actual budget deficit is: A) $200 billion. B) $20 billion. C) $40 billion. D) $60 billion. Answer: C

Type: G Topic: 5 E: 219 MA: 219 110. Refer to the above diagram where T is tax revenues and G is government expenditures. All figures are in billions of dollars. If the full-employment GDP is $400 billion while the actual GDP is $200 billion, the full-employment budget deficit is: A) $40 billion. B) zero. C) $60 billion. D) $20 billion. Answer: D

McConnell/Brue: Economics, 16/e

Page 391

Chapter 12: Fiscal Policy

Type: G Topic: 5 E: 219 MA: 219 111. Refer to the above diagram where T is tax revenues and G is government expenditures. All figures are in billions of dollars. If the full-employment GDP is $400 billion while the actual GDP is $200 billion, the cyclical deficit is: A) $40 billion. B) $20 billion. C) zero. D) $60 billion. Answer: B

Type: G Topic: 5 E: 219 MA: 219 112. Refer to the above diagram where T is tax revenues and G is government expenditures. All figures are in billions of dollars. If the full-employment GDP is $400 billion while the actual GDP is $200 billion, the: A) actual budget deficit exceeds the full-employment budget deficit. B) actual budget deficit is less than the full-employment budget deficit. C) full-employment deficit exceeds the cyclical deficit. D) cyclical deficit exceeds the full-employment deficit. Answer: A

Type: G Topic: 5 E: 219 MA: 219 113. Refer to the above diagram where T is tax revenues and G is government expenditures. All figures are in billions of dollars. If the full-employment GDP and actual GDP are each $400 billion, this economy will realize a: A) full-employment deficit of $20 billion. C) cyclical surplus of $20 billion. B) cyclical deficit of $20 billion. D) full-employment deficit of zero. Answer: A

Type: G Topic: 5 E: 219 MA: 219 114. Refer to the above diagram where T is tax revenues and G is government expenditures. All figures are in billions of dollars. If the full-employment and actual GDP are each $400 billion, government can balance its budget by: A) increasing T by $40 billion. B) reducing G by $20 billion. C) reducing T by $20 billion. D) increasing T by $10 billion and reducing G by $20 billion. Answer: B

Type: D Topic: 5 E: 221 MA: 221 115. Economists refer to a budget deficit that exists when the economy is achieving full employment as a: A) cyclical deficit. B) full-employment budget. C) natural deficit. D) nonrecurring deficit. Answer: B

Type: A Topic: 5 E: 221 MA: 221 116. When the economy is at full employment: A) one cannot generalize in comparing the actual and the full-employment budgets. B) the full-employment budget will show a surplus and the actual budget will show a deficit. C) the actual budget will show a surplus and the full-employment budget will show a deficit. D) the actual and the full-employment budgets will be equal. Answer: D

McConnell/Brue: Economics, 16/e

Page 392

Chapter 12: Fiscal Policy

Type: A Topic: 5 E: 221 MA: 221 117. If government increases the size of its full-employment surplus, we can: A) assume that government is causing interest rates to rise. B) not determine government's impact on the economy without also knowing the status of the actual budget. C) assume that government is having a contractionary effect on the economy. D) assume that government is having an expansionary effect on the economy. Answer: C

Problems, criticisms, complications

Use the following to answer questions 118-120: Answer the next question(s) on the basis of the following sequence of events involving fiscal policy: 1) The composite index of leading indicators turns downward for three consecutive months; (2) Economists reach agreement that the economy is moving into a recession; (3) A tax cut is proposed in Congress; (4) The tax cut is passed by Congress and signed by the President; (5) Consumption spending begins to rise, aggregate demand increases, and the economy begins to recover.

Type: A Topic: 6 E: 223 MA: 223 118. Refer to the above information. The operational lag of fiscal policy is reflected in events: A) 1 and 2. B) 2 and 3. C) 3 and 4. D) 4 and 5. Answer: D

Type: A Topic: 6 E: 223 MA: 223 119. Refer to the above information. The recognition lag of fiscal policy is reflected in events: A) 1 and 2. B) 2 and 3. C) 3 and 4. D) 4 and 5. Answer: A

Type: A Topic: 6 E: 223 MA: 223 120. Refer to the above information. The administrative lag of fiscal policy is reflected in events: A) 1 and 2. B) 2 and 3. C) 3 and 4. D) 4 and 5. Answer: C

Type: A Topic: 6 E: 223 MA: 223 121. Which of the following best describes the idea of a political business cycle? A) Politicians are more willing to cut taxes and increase government spending than they are to do the reverse. B) Fiscal policy will result in alternating budget deficits and surpluses. C) Politicians will use fiscal policy to cause output, real incomes, and employment to be rising prior to elections. D) Despite good intentions, various timing lags will cause fiscal policy to reinforce the business cycle. Answer: C

McConnell/Brue: Economics, 16/e

Page 393

Chapter 12: Fiscal Policy

Type: D Topic: 6 E: 223 MA: 223 122. The political business cycle refers to the possibility that: A) incumbent politicians will be reelected regardless of the state of the economy. B) politicians will manipulate the economy to enhance their chances of being reelected. C) there is more inflation during Democratic administrations than during Republican administrations. D) recessions coincide with election years. Answer: B

Type: D Topic: 6 E: 224 MA: 224 123. The crowding-out effect of expansionary fiscal policy suggests that: A) tax increases are paid primarily out of saving and therefore are not an effective fiscal device. B) increases in government spending financed through borrowing will increase the interest rate and thereby reduce investment. C) it is very difficult to have excessive aggregate spending in the U.S. economy. D) consumer and investment spending always vary inversely. Answer: B

Type: A Topic: 6 E: 224 MA: 224 124. The crowding-out effect of expansionary fiscal policy suggests that: A) government spending is increasing at the expense of private investment. B) imports are replacing domestic production. C) private investment is increasing at the expense of government spending. D) saving is increasing at the expense of investment. Answer: A

Type: D Topic: 6 E: 224 MA: 224 125. Assume the government purposely incurs a budget deficit that is financed by borrowing. As a result, interest rates rise and the amount of private investment spending declines. This illustrates: A) the equation-of-exchange effect. C) the crowding-out effect. B) the paradox of thrift. D) the wealth effect. Answer: C

Type: D Topic: 6 E: 224 MA: 224 126. The financing of a government deficit increases interest rates and, as a result, reduces investment spending. This statement describes: A) the supply-side effects of fiscal policy. C) the crowding-out effect. B) built-in stability. D) the net export effect. Answer: C

McConnell/Brue: Economics, 16/e

Page 394

Chapter 12: Fiscal Policy

Use the following to answer questions 127-131:

AS
Price level Price level

AS

P1

P1

AD1 0 X Z Real GDP (billions) 0

AD1 X Y Z Real GDP (billions)

Type: G Topic: 6 E: 215 MA: 215 Status: New 127. Suppose real GDP is X, as shown in graph (a). Appropriate government fiscal policy would be to: A) increase taxes. B) reduce government spending. C) reduce government spending and taxes by equal-sized amounts. D) reduce taxes or increase government spending. Answer: D

Type: G Topic: 6 E: 216 MA: 216 Status: New 128. Suppose real GDP is X, as shown in graph (a). If the economy's MPC is .75, X is $100 billion and fullemployment real GDP Y is $140 billion, an appropriate fiscal policy would be to: A) reduce taxes by $100 billion. B) increase government expenditures by $100 billion. C) reduce taxes by $10 billion. D) increase government expenditures by $10 billion. Answer: D

Type: G Topic: 6 E: 216 MA: 216 Status: New 129. Suppose real GDP is X, as shown in graph (a). If the economy's MPC is .8, X is $200 billion and fullemployment real GDP Y is $300 billion, an appropriate fiscal policy would be to reduce taxes by: A) $100 billion. B) $20 billion. C) $25 billion. D) $164 billion. Answer: C

Type: G Topic: 6 E: 226 MA: 226 130. Refer to the above diagrams. Suppose that government undertakes fiscal policy designed to increase aggregate demand from AD1 to AD2 and thereby to increase GDP from X to Z. In terms of graph a, which of the following might explain why GDP increases to Y rather than to Z? A) inflation B) an increase in stock prices C) a net export effect D) a ratchet effect Answer: C

McConnell/Brue: Economics, 16/e

Page 395

Chapter 12: Fiscal Policy

Type: G Topic: 6 E: 226 MA: 226 131. Refer to the above diagrams. A second correct answer to the previous question would be a: A) depreciation of the dollar. B) reduction in tariffs imposed by our trading partners. C) decrease in the saving schedule. D) crowding-out effect. Answer: D

Fiscal policy: AE model

Use the following to answer questions 132-133:

Type: G Topic: 7 E: 179 MA: 179 132. Refer to the above diagram. If the full-employment level of GDP is D, then it would be appropriate fiscal policy for government to: A) decrease spending and increase taxes. C) increase spending and increase taxes. B) decrease spending and decrease taxes. D) increase spending and decrease taxes. Answer: D

Type: G Topic: 7 E: 179 MA: 179 133. Refer to the above diagram. If the full-employment level of GDP is A, then it would be appropriate fiscal policy for government to: A) decrease spending and increase taxes. C) increase spending and increase taxes. B) decrease spending and decrease taxes. D) increase spending and decrease taxes. Answer: A

McConnell/Brue: Economics, 16/e

Page 396

Chapter 12: Fiscal Policy

Use the following to answer questions 134-137:

Type: G Topic: 7 E: 179 MA: 179 134. In the above diagram it is assumed that investment, net exports, and government purchases: A) are leakages from the circular flow. C) vary inversely with GDP. B) are independent of the level of GDP. D) vary directly with GDP. Answer: B

Type: G Topic: 7 E: 179 MA: 179 135. Refer to the above diagram. The equilibrium level of GDP is: A) Y5. B) Y4. C) Y3. D) Y2. Answer: B

Type: G Topic: 7 E: 215 MA: 215 136. Refer to the above diagram. If the full-employment GDP is Y5, government should: A) incur neither a deficit nor a surplus. B) cut taxes and government spending by equal amounts. C) reduce taxes and increase government spending. D) increase taxes and reduce government spending. Answer: C

Type: G Topic: 7 E: 217 MA: 217 137. Refer to the above diagram. If the full-employment GDP is Y3, government should: A) incur neither a deficit nor a surplus. B) increase taxes and government spending by equal amounts. C) reduce taxes and increase government spending. D) increase taxes and reduce government spending. Answer: D

McConnell/Brue: Economics, 16/e

Page 397

Chapter 12: Fiscal Policy

Fiscal policy: open economy complications

Type: A Topic: 8 E: 226 MA: 226 138. An expansionary U.S. fiscal policy that drives up U.S. interest rates is most likely to: A) decrease the foreign demand for dollars and appreciate the international value of the dollar. B) decrease the foreign demand for dollars and depreciate the international value of the dollar. C) increase the foreign demand for dollars and appreciate the international value of the dollar. D) increase the foreign demand for dollars and depreciate the international value of the dollar. Answer: C

Type: A Topic: 8 E: 226 MA: 226 139. A contractionary U.S. fiscal policy that reduces domestic interest rates is most likely to: A) depreciate the international value of the dollar and increase U.S. net exports. B) depreciate the international value of the dollar and decrease U.S. net exports. C) appreciate the international value of the dollar and increase U.S. net exports. D) appreciate the international value of the dollar and decrease U.S. net exports. Answer: A

Type: A Topic: 8 E: 226 MA: 226 140. Which one of the following best describes the net export effect associated with an expansionary U.S. fiscal policy? A) domestic interest rate falls, foreign demand for dollars rises, dollar appreciates, and net exports increase. B) domestic interest rate falls, foreign demand for dollars rises, dollar appreciates, and net exports fall. C) domestic interest rate rises, foreign demand for dollars falls, dollar depreciates, and net exports increase. D) domestic interest rate rises, foreign demand for dollars increases, dollar appreciates, and net exports decline. Answer: D

Type: A Topic: 8 E: 226 MA: 226 141. The higher domestic interest rate resulting from an expansionary U.S. fiscal policy will tend to: A) increase domestic investment spending. C) increase domestic consumption spending. B) increase U.S. exports. D) decrease U.S. exports. Answer: D

Type: A Topic: 8 E: 226 MA: 226 142. An expansionary U.S. fiscal policy might unintentionally cause demand-pull inflation if: A) the dollar unexpectedly appreciates while the expansionary policy is in place. B) the dollar unexpectedly depreciates while the expansionary policy is in place. C) the policy produces severe crowding out. D) our trading partners experience recession during the time of the fiscal policy action. Answer: B

McConnell/Brue: Economics, 16/e

Page 398

Chapter 12: Fiscal Policy

Type: A Topic: 8 E: 226 MA: 226 143. International flows of financial capital in response to interest rate changes in the United States: A) weaken domestic fiscal policy through an offsetting net export effect. B) strengthen domestic fiscal policy through a supporting net export effect. C) strengthen domestic fiscal policy through an offsetting net export effect. D) do none of the above. Answer: A

Type: A Topic: 8 E: 226 MA: 226 144. All else equal, a contractionary U.S. fiscal policy which reduces domestic interest rates tends to: A) increase U.S. imports. C) reduce the foreign demand for U.S. dollars. B) increase the international value of the dollar. D) aggravate an existing U.S. trade deficit. Answer: C

Last Word Questions

Type: A E: 227 MA: 227 145. (Last Word) Which of the following is not an item in the list of leading economic indicators? A) changes in mutual fund balances B) the length of the average work week C) the money supply D) the value of the index of consumer expectations Answer: A

Type: A E: 227 MA: 227 146. (Last Word) The composite index of leading indicators is useful for: A) predicting potential GDP. C) developing discretionary fiscal policy. B) determining the natural rate of unemployment. D) forecasting aggregate supply shocks. Answer: C

Type: A E: 227 MA: 227 147. (Last Word) Which one of the following is one of the leading economic indicators? A) index of consumer expectations C) the consumer price index B) the unemployment rate D) Federal income tax collections Answer: A

True/False Questions

Type: A E: 215 MA: 215 148. Expansionary fiscal policy is so-named because it involves an expansion of the nation's money supply. Answer: False

Type: D E: 216 MA: 216 149. If the MPC in the economy is .75, government could shift the aggregate demand curve rightward by $30 billion by cutting taxes by $10 billion. Answer: True

McConnell/Brue: Economics, 16/e

Page 399

Chapter 12: Fiscal Policy

Type: A E: 216 MA: 216 150. A contractionary fiscal policy shifts the aggregate demand curve leftward. Answer: True

Type: A E: 216 MA: 216 151. The greatest impact of a tax cut will occur when government borrows from the public rather than prints money to finance the resulting budget deficit. Answer: False

Type: A E: 216 MA: 216 152. Demand-pull inflation can be restrained by increasing government spending and reducing taxes. Answer: False

Type: A E: 218 MA: 218 153. Built-in stability is synonymous with discretionary fiscal policy. Answer: False

Type: A E: 220 MA: 220 154. The actual budget may be in deficit while the full-employment budget is in surplus. Answer: True

Type: A E: 220 MA: 220 155. An increase in the cyclical deficits will automatically increase the full-employment budget deficit. Answer: False

Type: A E: 219 MA: 219 156. Tax revenues automatically increase during economic expansions and decrease during recessions. Answer: True

Type: A E: 223 MA: 223 157. The operational lag of fiscal policy refers to the time that elapses between the beginning of a recession or inflation and the certain awareness that it is actually happening. Answer: False

Type: D E: 224 MA: 224 158. The crowding-out effect refers to the possibility that deficit spending may motivate people to increase their saving in anticipation of higher future taxes. Answer: False

Type: D E: 226 MA: 226 159. If fiscal policy leads to higher interest rates, the dollar may appreciate and net exports may fall. Answer: True

McConnell/Brue: Economics, 16/e

Page 400

Chapter 12: Fiscal Policy

Type: D E: 214 MA: 214 Status: New 160. Fiscal policy is mainly undertaken by the Federal Reserve. Answer: False

Type: F E: 234 MA: 234 Status: New 161. Tax increases and government spending cuts by state governments during recessions often reduce the expansionary impact of fiscal policy by the Federal government. Answer: True

Type: F E: 216 MA: 216 Status: New 162. Permanent tax reductions are more likely to be expansionary than temporary tax reductions. Answer: True

Type: F E: 220 MA: 220 Status: New 163. As measured by the full-employment budget, the U.S. government engaged in a contractionary fiscal policy in 2001 and 2002. Answer: False

Type: F E: 223 MA: 223 Status: New 164. Fiscal policy is complicated by political considerations and political motivations. Answer: True

McConnell/Brue: Economics, 16/e

Page 401

CHAPTER 13

Money and Banking

Topic 1. 2. 3. 4. 5. 6. Functions of money Supply of money (definition, value, etc.) Demand for money Money market U.S. financial system Recent developments and reform Consider This Last Word True-False

Question numbers 1-11 12-63 64-81 82-117 118-143 144-156 157-159 160-162 163-176

____________________________________________________________

_______________________________________

____________________________________________________________

_______________________________________

Multiple Choice Questions Functions of money

Type: A Topic: 1 E: 232 MA: 232 1. To say money is socially defined means that: A) money has been defined in a Constitutional amendment. B) whatever performs the functions of money extremely well is considered to be money. C) the money supply includes all public and private securities purchased by society. D) society, acting through Congress, specifies what shall be included in the money supply. Answer: B

Type: D Topic: 1 E: 232-233 MA: 232-233 2. Money functions as: A) a store of value. B) a unit of account. C) a medium of exchange. Answer: D

D) all of the above.

Type: A Topic: 1 E: 233 MA: 233 3. If you are estimating your total expenses for school next semester, you are using money primarily as: A) a medium of exchange. B) a store of value. C) a unit of account. D) an economic investment. Answer: C

Type: A Topic: 1 E: 233 MA: 233 4. If you place a part of your summer earnings in a savings account, you are using money primarily as a: A) medium of exchange. B) store of value. C) unit of account. D) standard of value. Answer: B

Chapter 13: Money and Banking

Type: A Topic: 1 E: 232 MA: 232 5. If you write a check on a bank to purchase a used Honda Civic, you are using money primarily as: A) a medium of exchange. B) a store of value. C) a unit of account. D) an economic investment. Answer: A

Type: A Topic: 1 E: 233 MA: 233 6. A $70 price tag on a sweater in a department store window is an example of money functioning as a: A) unit of account. B) standard of deferred payments. C) store of value. D) medium of exchange. Answer: A

Type: A Topic: 1 E: 233 MA: 233 7. Stock market price quotations best exemplify money serving as a: A) store of value. B) unit of account. C) medium of exchange. Answer: B

D) index of satisfaction.

Type: A Topic: 1 E: 232 MA: 232 8. Purchasing common stock by writing a check best exemplifies money serving as a: A) store of value. B) unit of account. C) medium of exchange. D) index of satisfaction. Answer: C

Type: A Topic: 1 E: 232 MA: 232 9. When economists say that money serves as a medium of exchange, they mean that it is: A) a way to keep wealth in a readily spendable form for future use. B) a means of payment. C) a monetary unit for measuring and comparing the relative values of goods. D) declared as legal tender by the government. Answer: B

Type: A Topic: 1 E: 233 MA: 233 10. When economists say that money serves as a unit of account, they mean that it is: A) away to keep wealth in a readily spendable form for future use. B) a means of payment. C) a monetary unit for measuring and comparing the relative values of goods. D) declared as legal tender by the government. Answer: C

Type: A Topic: 1 E: 233 MA: 233 11. When economists say that money serves as a store of value, they mean that it is: A) a way to keep wealth in a readily spendable form for future use. B) a means of payment. C) a monetary unit for measuring and comparing the relative values of goods. D) declared as legal tender by the government. Answer: A

McConnell/Brue: Economics, 16/e

Page 404

Chapter 13: Money and Banking

Supply of money

Type: F Topic: 2 E: 234 MA: 234 12. The paper money used in the United States is: A) National Bank Notes. B) Treasury Notes. Answer: D

C) United States Notes.

D) Federal Reserve Notes.

Type: F Topic: 2 E: 233 MA: 233 13. The largest component of the money supply (M1) is: A) gold certificates. B) checkable deposits. C) paper money in circulation. Answer: C

D) coins.

Type: D Topic: 2 E: 233 MA: 233 14. In the United States, the money supply (M1) is comprised of: A) coins, paper currency, and checkable deposits. B) currency, checkable deposits, and Series E bonds. C) coins, paper currency, checkable deposits, and credit balances with brokers. D) paper currency, coins, gold certificates, and time deposits. Answer: A

Type: D Topic: 2 E: 237 MA: 237 15. Fiat money is: A) composed only of checkable deposits. B) money because the government asserts that it is. C) money that is in a commercial bank vault. D) money that can be redeemed for a valuable commodity such as gold. Answer: B

Type: A Topic: 2 E: 237 MA: 237 16. The purchasing power of the dollar: A) has been increasing in recent years because of economic growth. B) varies directly with the cost-of-living index. C) is inversely related to the level of aggregate demand. D) is the reciprocal of the price level. Answer: D

Type: F Topic: 2 E: 238 MA: 238 17. The money supply is backed: A) by the government's ability to control the supply of money and therefore to keep its value relatively stable. B) by government bonds. C) dollar-for-dollar with gold and silver. D) dollar-for-dollar with gold bullion. Answer: A

McConnell/Brue: Economics, 16/e

Page 405

Chapter 13: Money and Banking

Type: A Topic: 2 E: 234 MA: 234 18. Checkable deposits are classified as money because: A) they can be readily used in purchasing goods and paying debts. B) banks hold currency equal to the value of their checkable deposits. C) they are ultimately the obligations of the Treasury. D) they earn interest income for the depositor. Answer: A

Type: A Topic: 2 E: 237 MA: 237 19. The value of money varies: A) inversely with the price level. B) directly with the volume of employment. Answer: A

C) directly with the price level. D) directly with the interest rate.

Type: F Topic: 2 E: 233 MA: 233 20. Currency (paper money plus coins) constitutes about: A) 81 percent of the U.S. Ml money supply. C) 92 percent of the U.S. M1 money supply. B) 52 percent of the U.S. M1 money supply. D) 11 percent of the U.S. M1 money supply. Answer: B

Type: F Topic: 2 E: 233 MA: 233 21. In 2002, the supply of money (M1) in the United States was about: A) $247 billion. B) $1600 billion. C) $203 billion. D) $1236 billion. Answer: D

Type: F Topic: 2 E: 234 MA: 234 22. Nearly one-half the money in the U.S. economy is created by: A) the receipt of gold bullion through international trade and finance. B) commercial banks and thrift institutions. C) the Federal mint. D) the Federal Treasury. Answer: B

Type: A Topic: 2 E: 235 MA: 235 23. In defining money as M1, economists exclude time deposits because: A) the intrinsic value of time deposits is nil. B) the purchasing power of time deposits is much less stable than that of checkable deposits and currency. C) they are not directly or immediately a medium of exchange. D) they are not recognized by the Federal government as legal tender. Answer: C

Type: A Topic: 2 E: 237 MA: 237 24. The purchasing power of money and the price level vary: A) inversely. B) directly during recessions, but inversely during inflations. C) directly, but not proportionately. D) directly and proportionately. Answer: A

McConnell/Brue: Economics, 16/e

Page 406

Chapter 13: Money and Banking

Type: A Topic: 2 E: 237 MA: 237 25. If the price index rises from 100 to 120, the purchasing power value of the dollar: A) may either rise or fall. C) will fall by one-sixth. B) will rise by one-sixth. D) will rise by 20 percent. Answer: C

Type: C Topic: 2 E: 237 MA: 237 26. Other things equal, an excessive increase in the money supply will : A) increase the purchasing power of each dollar. B) decrease the purchasing power of each dollar. C) have no impact on the purchasing power of the dollar. D) reduce the price level. Answer: B

Type: E Topic: 2 E: 237 MA: 237 27. If P equals the price level expressed as an index number and D equals the value of the dollar, then: A) P = D - 1. B) D = 1/P. C) 1 = D/P. D) D = P - 1. Answer: B

Use the following to answer questions 28-30: Answer the next question(s) on the basis of the following table:
Year 1 2 3 4 Price level 1.00 1.25 .80 .50 Value of dollar $1.00

Type: T Topic: 2 E: 237 MA: 237 28. Refer to the above table. The value of the dollar in year 2 is: A) $1.25. B) $1.33. C) $.80. D) $1.00. Answer: C

Type: T Topic: 2 E: 237 MA: 237 29. Refer to the above table. The value of the dollar in year 3 is: A) $1.00. B) $1.25. C) $.80. D) $.50. Answer: B

Type: T Topic: 2 E: 237 MA: 237 30. Refer to the above table. The value of the dollar in year 4 is: A) $.25. B) $.33. C) $.50. D) $2.00. Answer: D

McConnell/Brue: Economics, 16/e

Page 407

Chapter 13: Money and Banking

Type: D Topic: 2 E: 235 MA: 235 31. Which of the following is not part of the M2 money supply? A) money market mutual fund balances C) currency B) money market deposit accounts D) large ($100,000 or more) time deposits Answer: D

Type: D Topic: 2 E: 235 MA: 235 32. The M2 money supply includes: A) stock certificates. B) corporate bond certificates. C) the cash value of life insurance policies. D) individual shares in money market mutual funds. Answer: D

Type: A Topic: 2 E: 234 MA: 234 33. A checking account entry is money because it: A) is ensured by the Federal Deposit Insurance Corporation. B) has been declared as such by the Federal government. C) performs the functions of money. D) can be sold for currency. Answer: C

Type: A Topic: 2 E: 233-235 MA: 233-235 34. Currency in circulation is part of: A) M1 only. B) M2 only. C) M3 only. D) M1, M2, and M3. Answer: D

Type: D Topic: 2 E: 233-235 MA: 233-235 35. Money market deposit accounts are included in: A) M1 only. B) both M1 and M2. C) both M2 and M3. Answer: C

D) M3 only.

Type: D Topic: 2 E: 233-234 MA: 233-234 36. Checkable deposits are: A) included in M1. B) not included in either Ml or M2. Answer: A

C) considered to be a near money. D) also called time deposits.

Type: D Topic: 2 E: 233-235 MA: 233-235 37. Checkable deposits are: A) included in M1 but not in M2. B) considered to be a near-money. Answer: C

C) included in M1 and in M2. D) also called time deposits.

McConnell/Brue: Economics, 16/e

Page 408

Chapter 13: Money and Banking

Type: F Topic: 2 E: 233-235 MA: 233-235 38. The amount of money reported as M2: A) is smaller than the amount reported as M1. B) is larger than the amount reported as M1. C) excludes coins and currency. D) includes large ($100,000 or more) certificates of deposit. Answer: B

Type: F Topic: 2 E: 233-234 MA: 233-234 39. The largest component of the money supply is: A) coins. B) paper money. C) checkable deposits. Answer: B

D) stock certificates.

Type: F Topic: 2 E: 234 MA: 234 40. Paper money (currency) in the United States is issued by the: A) United States Mint. B) Federal Reserve Banks. C) United States Treasury. Answer: B

D) national banks.

Type: A Topic: 2 E: 234 MA: 234 41. A $20 bill is a: A) gold certificate. B) Treasury note. Answer: D

C) Treasury bill.

D) Federal Reserve Note.

Type: D Topic: 2 E: 233-235 MA: 233-235 42. Coins in people's pockets and purses are: A) included in M1, but not in M2. B) included in both M1 and in M2. C) included in M2, but not in M1. D) excluded from M1 and M2 because people can exchange them for Federal Reserve notes. Answer: B

Type: D Topic: 2 E: 234 MA: 234 43. Coins held in commercial banks are: A) included in M1, but not in M2. B) included both in M1 and in M2. Answer: D

C) included in M2, but not in M1. D) not part of the nation's money supply.

Type: D Topic: 2 E: 234 MA: 234 44. Checkable deposits include: A) both large and small time deposits. B) the deposits of banks and thrifts on which checks can be written. C) only the checkable deposits of commercial banks. D) only the checkable deposits of thrift institutions. Answer: B

McConnell/Brue: Economics, 16/e

Page 409

Chapter 13: Money and Banking

Type: D Topic: 2 E: 233-235 MA: 233-235 45. The difference between M1 and M2 is that: A) the former includes time deposits. B) the latter includes small time deposits, noncheckable savings accounts, money market deposit accounts, and money market mutual fund balances. C) the latter includes negotiable government bonds. D) the latter includes cash held by commercial banks and the U.S. Treasury. Answer: B

Type: A Topic: 2 E: 233-234 MA: 233-234 46. Assuming no other changes, if checkable deposits increase by $40 billion and currency and coins in circulation decrease by $40 billion, the: A) M1 money supply will decline. C) M2 money supply will decline. B) M1 money supply will not change. D) M3 money supply will increase. Answer: B

Type: A Topic: 2 E: 233-235 MA: 233-235 47. Assuming no other changes, if checkable deposits decrease by $40 billion and balances in money market mutual funds increase by $40 billion, the: A) M1 money supply will decline and M2 money supply will remain unchanged. B) M1 and M2 money supplies will not change. C) M2 and M3 money supplies will increase. D) M1, M2, and M3 money supplies will decline. Answer: A

Type: A Topic: 2 E: 233-235 MA: 233-235 48. Assuming no other changes, if balances in money market mutual funds increase by $40 billion and large time deposits decrease by $40 billion, the: A) M1 and M2 money supplies will not change. B) M2 and M3 money supplies will increase. C) M1, M2, and M3 money supplies will decline. D) M2 money supply will increase and M3 money supply will remain unchanged. Answer: D

Use the following to answer questions 49-51:

Money market mutual fund balances Currency and coins in banks Currency and coins in circulation Saving deposits, including money market deposit accounts Large ($100,000 or more) time deposits Small (less than $100,000) time deposits Checkable deposits

$220 10 60 50 180 80 70

Type: A Topic: 2 E: 233-234 MA: 233-234 49. Refer to the above information. Money supply M1 for this economy is: A) $60. B) $70. C) $130. D) $140. Answer: C

McConnell/Brue: Economics, 16/e

Page 410

Chapter 13: Money and Banking

Type: A Topic: 2 E: 233, 235 MA: 233, 235 50. Refer to the above information. Money supply M2 for this economy is: A) $480. B) $130. C) $490. D) $660. Answer: A

Type: A Topic: 2 E: 233, 235 MA: 233, 235 51. Refer to the above information. Money supply M3 for this economy is: A) $480. B) $130. C) $490. D) $660. Answer: D

Use the following to answer questions 52-55: Answer the next question(s) on the basis of the following list of assets: 1. Large ($100,000 and over) time deposits 2. Noncheckable savings deposits 3. Currency (coins and paper money) 4. Small (under $100,000) time deposits 5. Stock certificates 6. Checkable deposits 7. Money market deposit accounts 8. Money market mutual fund balances

Type: A Topic: 2 E: 233-234 MA: 233-234 52. Refer to the above list. The M1 definition of money comprises item(s): A) 6 only. B) 3, 4, and 6. C) 3 and 6. D) 2, 3, and 6. Answer: C

Type: A Topic: 2 E: 233-235 MA: 233-235 53. Refer to the above list. The M2 definition of money comprises: A) items 1, 2, 3, and 6. C) items 2, 3, 4, 6, 7, and 8. B) items 3, 4, 5, and 6. D) items 1, 2, 3, and 4. Answer: C

Type: A Topic: 2 E: 233-235 MA: 233-235 54. Refer to the above list. The M3 definition of money comprises: A) items 1, 2, 3, 4, 6, 7, and 8. C) items 1, 3, 4, 6, 7, and 8. B) items 2, 3, 4, 6, 7, and 8. D) all of the eight items listed. Answer: A

Type: A Topic: 2 E: 233 MA: 233 55. Refer to the above list. Which of the following is not included in any of the three official definitions of money? A) item 2 B) item 5 C) item 4 D) items 1 and 4 Answer: B

McConnell/Brue: Economics, 16/e

Page 411

Chapter 13: Money and Banking

Type: A Topic: 2 E: 233, 235 MA: 233, 235 Status: New 56. Large time deposits of $100,000 or more are: A) a component of M1. C) a component of M3 but not of M2. B) a component of M2 but not of M1. D) not a component of M1, M2, or M3. Answer: C

Type: A Topic: 2 E: 235 MA: 235 57. A basic argument for using the M1 concept of money is that: A) it includes all of the important financial assets that have any degree of liquidity. B) the government collects data for the components of M1, but does not do so for M2 and M3. C) its components are superior to other financial assets as a store of value. D) its components are directly and immediately spendable. Answer: D

Type: A Topic: 2 E: 234 MA: 234 58. Currency and coins held within banks are part of: A) the M3 definition of the money supply. B) the M2 definition of the money supply. C) the M1 definition of the money supply. D) none of the above definitions of the money supply. Answer: D

Demand for money

Use the following to answer questions 59-63:
M1 Currency and coins in circulation Currency and coins in banks Small (less than $100,000) time deposits M2 Savings deposits, including money market deposit accounts Large (more than $100,000) time deposits $300 30 10 15 350 20 40

Type: A Topic: 2 E: 233-234 MA: 233-234 Status: New 59. Refer to above information. Checkable deposits in this economy are: A) $320. B) $230. C) $270. D) $260. Answer: C

Type: A Topic: 2 E: 233-234 MA: 233-234 Status: New 60. Refer to above information. Money market mutual fund balances in this economy are: A) $15. B) $20. C) $0. D) $10. Answer: A

Type: A Topic: 2 E: 233-234 MA: 233-234 Status: New 61. Refer to above information. M3 in this economy is: A) $360. B) $410. C) $370. D) $390. Answer: D

McConnell/Brue: Economics, 16/e

Page 412

Chapter 13: Money and Banking

Type: F Topic: 2 E: 233-234 MA: 233-234 Status: New 62. Which of the following is a component of the M1 money supply in the United States? A) money market mutual fund balances. B) checkable deposits C) savings deposits, including money market mutual fund accounts. D) small (less than $100,000) time deposits. Answer: B

Type: F Topic: 2 E: 233-234 MA: 233-234 Status: New 63. Which of the following is a component of the M1 money supply in the United States? A) money market mutual fund balances. B) small (less than $100,000) time deposits. C) currency in circulation D) savings deposits, including money market mutual fund accounts. Answer: C

Demand for money

Type: A Topic: 3 E: 238 MA: 238 64. The transactions demand for money is most closely related to money functioning as a: A) unit of account. B) medium of exchange. C) store of value. D) measure of value. Answer: B

Type: A Topic: 3 E: 239-240 MA: 239-240 65. The asset demand for money is most closely related to money functioning as a: A) unit of account. B) medium of exchange. C) store of value. D) measure of value. Answer: C

Type: A Topic: 3 E: 240 MA: 240 66. The asset demand for money: A) is unrelated to both the interest rate and the level of GDP. B) varies inversely with the rate of interest. C) varies inversely with the level of real GDP. D) varies directly with the level of nominal GDP. Answer: B

Type: A Topic: 3 E: 241 MA: 241 67. On a diagram where the interest rate and the quantity of money demanded are shown on the vertical and horizontal axes respectively, the transactions demand for money can be represented by: A) a line parallel to the horizontal axis. C) a downsloping line or curve from left to right. B) a vertical line. D) an upsloping line or curve from left to right. Answer: B

McConnell/Brue: Economics, 16/e

Page 413

Chapter 13: Money and Banking

Type: A Topic: 3 E: 241 MA: 241 68. On a diagram where the interest rate and the quantity of money demanded are shown on the vertical and horizontal axes respectively, the asset demand for money can be represented by: A) a line parallel to the horizontal axis. C) a downsloping line or curve from left to right. B) a vertical line. D) an upsloping line or curve from left to right. Answer: C

Type: A Topic: 3 E: 241 MA: 241 69. On a diagram where the interest rate and the quantity of money demanded are shown on the vertical and horizontal axes respectively, the total demand for money can be found by: A) horizontally adding the transactions and the asset demand for money. B) vertically subtracting the transactions demand from the asset demand for money. C) horizontally subtracting the asset demand from the transactions demand for money. D) vertically adding the transactions and the asset demand for money. Answer: A

Type: A Topic: 3 E: 240 MA: 240 70. The total demand for money curve will shift to the right as a result of: A) an increase in nominal GDP. C) a decline in the interest rate. B) an increase in the interest rate. D) a decline in nominal GDP. Answer: A

Type: A Topic: 3 E: 240-241 MA: 240-241 71. Which of the following statements is correct? Other things equal: A) a decline in real output will shift both the transactions demand curve for money and the total money demand curve to the right. B) a decline in the interest rate will shift the asset demand curve for money to the right, but leave the total money demand curve unchanged. C) deflation will shift both the transactions demand curve for money and the total money demand curve to the left. D) inflation will shift the transactions demand curve for money to the right, but leave the total money demand curve unchanged. Answer: C

Type: A Topic: 3 E: 240 MA: 240 72. If nominal GDP is $600 billion and, on the average, each dollar is spent three times per year, then the amount of money demanded for transactions purposes will be: A) $1800 billion. B) $600 billion. C) $200 billion. D) $1200 billion. Answer: C

Type: A Topic: 3 E: 240 MA: 240 73. In which of the following situations is it certain that the quantity of money demanded by the public will decrease? A) nominal GDP decreases and the interest rate decreases B) nominal GDP increases and the interest rate decreases C) nominal GDP decreases and the interest rate increases D) nominal GDP increases and the interest rate increases Answer: C

McConnell/Brue: Economics, 16/e

Page 414

Chapter 13: Money and Banking

Type: A Topic: 3 E: 240 MA: 240 74. It is costly to hold money because: A) deflation may reduce its purchasing power. B) in doing so one sacrifices interest income. Answer: B

C) bond prices are highly variable. D) the velocity of money may decline.

Type: A Topic: 3 E: 240 MA: 240 75. An increase in nominal GDP increases the demand for money because: A) interest rates will rise. B) more money is needed to finance a larger volume of transactions. C) bond prices will fall. D) the opportunity cost of holding money will decline. Answer: B

Type: A Topic: 3 E: 240 MA: 240 76. Which of the following is correct? A) The asset demand for money is downsloping because the opportunity cost of holding money declines as the interest rate rises. B) The asset demand for money is downsloping because the opportunity cost of holding money increases as the interest rate rises. C) The transactions demand for money is downsloping because the opportunity cost of holding money varies inversely with the interest rate. D) The asset demand for money is downsloping because bond prices and the interest rate are directly related. Answer: B

Type: A Topic: 3 E: 238-239 MA: 238-239 77. The transactions demand for money will shift to the: A) right when the interest rate increases. C) right when aggregate income increases. B) left when the interest rate decreases. D) right when aggregate income decreases. Answer: C

Type: A Topic: 3 E: 240 MA: 240 78. The opportunity cost of holding money: A) is zero because money is not an economic resource. B) varies inversely with the interest rate. C) varies directly with the interest rate. D) varies inversely with the level of economic activity. Answer: C

Type: A Topic: 3 E: 240 MA: 240 79. The total demand for money will shift to the left as a result of: A) a decline in nominal GDP. C) a change in the interest rate. B) an increase in the price level. D) an increase in nominal GDP. Answer: A

McConnell/Brue: Economics, 16/e

Page 415

Chapter 13: Money and Banking

Type: A Topic: 3 E: 240 MA: 240 80. The asset demand for money is downsloping because: A) the opportunity cost of holding money increases as the interest rate rises. B) it is more attractive to hold money at high interest rates than at low interest rates. C) bond prices rise as interest rates rise. D) the opportunity cost of holding money declines as the interest rate rises. Answer: A

Type: E Topic: 3 E: 240-241 MA: 240-241 81. (Advanced analysis) Assume the equation for the total demand for money is L = 0.4Y + 80 - 4 i, where L is the amount of money demanded, Y is gross domestic product, and i is the interest rate. If gross domestic product is $200 and the interest rate is 10 (percent), what amount of money will society want to hold? A) $200 B) $120 C) $320 D) $160 Answer: B

Money market

Type: A Topic: 4 E: 240-241 MA: 240-241 82. If the quantity of money demanded exceeds the quantity supplied: A) the supply-of-money curve will shift to the left. B) the demand-for-money curve will shift to the right. C) the interest rate will rise. D) the interest rate will fall. Answer: C

Type: A Topic: 4 E: 241 MA: 241 83. The equilibrium rate of interest in the money market is determined by the intersection of the: A) supply of money curve and the asset demand for money curve. B) supply of money curve and the transactions demand for money curve. C) supply of money curve and the total demand for money curve. D) investment demand curve and total demand for money curve. Answer: C

Type: A Topic: 4 E: 240-241 MA: 240-241 84. If the demand for money and the supply of money both decrease, the equilibrium: A) interest rate will decline, but we cannot predict the change in the equilibrium quantity of money. B) quantity of money and the equilibrium interest rate will both increase. C) quantity of money will increase, but we cannot predict the change in the equilibrium interest rate. D) quantity of money will decline, but we cannot predict the change in the equilibrium interest rate. Answer: D

Type: A Topic: 4 E: 241 MA: 241 85. If in the money market the quantity of money demanded exceeds the money supply, the interest rate will: A) fall, causing households and businesses to hold less money. B) rise, causing households and businesses to hold less money. C) rise, causing households and businesses to hold more money. D) fall, causing households and businesses to hold more money. Answer: B

McConnell/Brue: Economics, 16/e

Page 416

Chapter 13: Money and Banking

Type: A Topic: 4 E: 241 MA: 241 86. If in the money market the amount of money supplied exceeds the amount of money households and businesses want to hold, the interest rate will: A) fall, causing households and businesses to hold less money. B) rise, causing households and businesses to hold less money. C) rise, causing households and businesses to hold more money. D) fall, causing households and businesses to hold more money. Answer: D

Use the following to answer questions 87-91:

Type: A Topic: 4 E: 240 MA: 240 87. Refer to the above diagram of the money market. The downward slope of the money demand curve Dm is best explained in terms of the: A) transactions demand for money. B) direct or positive relationship between bond prices and interest rates. C) asset demand for money. D) wealth or real-balances effect. Answer: C

Type: A Topic: 4 E: 241 MA: 241 88. Refer to the above diagram of the money market. The vertical money supply curve Sm reflects the fact that: A) bond prices and interest rates are inversely related. B) the stock of money is determined by the Federal Reserve System and does not change when the interest rate changes. C) the velocity of money is zero. D) lower interest rates result in lower opportunity costs of supplying money. Answer: B

Type: A Topic: 4 E: 241 MA: 241 89. Refer to the above diagram of the money market. The equilibrium interest rate is: A) i1. B) i2. C) i3. D) not determinable without additional information. Answer: B

McConnell/Brue: Economics, 16/e

Page 417

Chapter 13: Money and Banking

Type: A Topic: 4 E: 241 MA: 241 90. Refer to the above diagram of the money market. Given Dm and Sm, an interest rate of i3 is not sustainable because the: A) supply of bonds in the bond market will decline and the interest rate will rise. B) supply of bonds in the bond market will increase and the interest rate will decline. C) demand for bonds in the bond market will decline and the interest rate will rise. D) demand for bonds in the bond market will rise and the interest rate will fall. Answer: D

Type: A Topic: 4 E: 238-239 MA: 238-239 91. Refer to the above diagram of the money market. Other things equal, the money demand curve in the diagram would shift leftward if: A) the asset demand for money increased. C) nominal GDP decreased. B) the transactions demand for money increased. D) the overall price level rose. Answer: C

Use the following to answer questions 92-93: Answer the next question(s) on the basis of the following information for a bond having no expiration date: bond price = $1000; bond fixed annual interest payment = $100; bond annual interest rate = 10 percent.

Type: A Topic: 4 E: 241 MA: 241 92. Refer to the above information. If the price of this bond falls by $200, the interest rate will: A) rise by 2.5 percentage points. C) fall by 2.5 percentage points. B) rise by 5 percentage points. D) fall by 5 percentage points. Answer: A

Type: A Topic: 4 E: 241 MA: 241 93. Refer to the above information. If the price of this bond increases to $1250, the interest rate will: A) fall to 9 percent. B) fall to 8 percent. C) rise to 11 percent. D) rise to 12 percent. Answer: B

Type: A Topic: 4 E: 240 MA: 240 94. Which of the following statements is correct? A) Interest rates and bond prices vary directly. B) Interest rates and bond prices vary inversely. C) Interest rates and bond prices are unrelated. D) Interest rates and bond prices vary directly during inflations and inversely during recessions. Answer: B

McConnell/Brue: Economics, 16/e

Page 418

Chapter 13: Money and Banking

Use the following to answer questions 95-101:

Rate of interest (percent)

S 10 8 6 4 2

D1

100 200 300 400 Amount of money demanded (billions of dollars)

Type: G Topic: 4 E: 239 MA: 239 95. Refer to the above money market diagrams. The asset demand for money is shown by: A) D1. B) D2. C) D3. D) S. Answer: B

Type: G Topic: 4 E: 239 MA: 239 96. Refer to the above money market diagrams. Curve D1 represents the: A) speculative demand for money. C) asset demand for money. B) transactions demand for money. D) stock of money. Answer: B

Type: G Topic: 4 E: 239 MA: 239 97. Refer to the above money market diagrams. The total demand for money is shown by: A) D1. B) D2. C) D3. D) S. Answer: C

Type: G Topic: 4 E: 239 MA: 239 98. Refer to the above money market diagrams. If each dollar held for transactions is spent four times per year on the average, we can infer that the: A) real GDP is $800. C) money supply must be $800. B) nominal GDP is $800. D) nominal GDP is $1200. Answer: B

Type: G Topic: 4 E: 239 MA: 239 99. Refer to the above money market diagrams. If the interest rate was at 3 percent, people would: A) sell bonds, which would cause bond prices to fall and the interest rate to rise. B) buy bonds, which would cause bond prices to fall and the interest rate to rise. C) sell bonds, which would cause bond prices to rise and the interest rate to rise. D) buy bonds, which would cause bond prices to rise but have an uncertain effect upon the interest rate. Answer: A

McConnell/Brue: Economics, 16/e

Page 419

Chapter 13: Money and Banking

Type: G Topic: 4 E: 239 MA: 239 100. Refer to the above money market diagrams. If the interest rate was at 8 percent, people would: A) sell bonds, which would cause bond prices to fall and the interest rate to fall. B) buy bonds, which would cause bond prices to rise and the interest rate to fall. C) have insufficient liquidity, which would cause them to reduce their spending on consumer goods. D) buy bonds, which would cause bond prices to fall and the interest rate to rise. Answer: B

Type: G Topic: 4 E: 239 MA: 239 101. Refer to the above money market diagrams. If the Federal Reserve increased the stock of money, the: A) S curve would shift leftward and the equilibrium interest rate would rise. B) S curve would shift rightward and the equilibrium interest rate would fall. C) D3 would shift leftward and the equilibrium interest rate would fall. D) D3 curve would shift leftward and the equilibrium interest rate would rise. Answer: B

Type: A Topic: 4 E: 239 MA: 239 102. Suppose the demand for money and the supply of money increase simultaneously. We can: A) expect the interest rate to rise and bond prices to fall. B) expect the interest rate to fall and bond prices to rise. C) the nominal GDP to expand. D) not predict what will happen to interest rates or bond prices. Answer: D

Type: A Topic: 4 E: 239 MA: 239 103. When the money market is in equilibrium: A) the quantity of money demanded equals the quantity of money supplied. B) the interest rate is increasing. C) bond prices are falling. D) the interest rate is declining. Answer: A

Type: A Topic: 4 E: 239 MA: 239 104. Other things equal, if there is an increase in nominal GDP: A) the demand for money will decrease. C) bond prices will rise. B) the interest rate will rise. D) consumption spending will fall. Answer: B

Type: A Topic: 4 E: 239 MA: 239 105. Other things equal, if the supply of money is reduced: A) the demand for money will increase. C) bond prices will fall. B) the interest rates will fall. D) investment spending will increase. Answer: C

McConnell/Brue: Economics, 16/e

Page 420

Chapter 13: Money and Banking

Use the following to answer questions 106-108: Answer the next question(s) on the basis of the following table in which columns (1) and (2) indicate the transactions demand (Dt) for money and columns (1) and (3) show the asset demand (Da) for money:

(1) Interest rate 12% 10 8 6 4 2

(2) Dt $100 100 100 100 100 100

(3) Da $ 0 20 40 60 80 100

Type: T Topic: 4 E: 238 MA: 238 106. The above data suggest that the amount of money demanded for transactions: A) varies directly with the interest rate. C) varies inversely with nominal GDP. B) varies inversely with the interest rate. D) is independent of the interest rate. Answer: D

Type: T Topic: 4 E: 240 MA: 240 107. The above data suggest that the amount of money that society wishes to hold as an asset: A) varies directly with the interest rate. C) varies inversely with nominal GDP. B) varies inversely with the interest rate. D) is independent of the interest rate. Answer: B

Type: T Topic: 4 E: 239 MA: 239 108. Refer to the above data. If the money supply is $160, the equilibrium interest rate will be: A) 10 percent. B) 8 percent. C) 6 percent. D) 4 percent. Answer: C

Use the following to answer questions 109-110: Answer the next question(s) on the basis of the following information. For transactions, households and businesses want to hold an amount of money equal to one half of nominal GDP. The table shows the amounts of money they want to hold as an asset at various interest rates.
Interest rate 10% 8 6 4 2 Amount of money demanded $ 20 40 60 80 100

Type: T Topic: 4 E: 238-239 MA: 238-239 109. Refer to the above information. If nominal GDP is $200 and the interest rate is 6 percent, the total amount of money that households and businesses will want to hold is: A) $120 B) $140 C) $160 D) $180 Answer: C

McConnell/Brue: Economics, 16/e

Page 421

Chapter 13: Money and Banking

Type: T Topic: 4 E: 238-239 MA: 238-239 110. Refer to the above information. If nominal GDP is $300 and the supply of money is $230, the equilibrium interest rate will be: A) 8 percent. B) 6 percent. C) 4 percent. D) 2 percent. Answer: C

Type: A Topic: 4 E: 241 MA: 241 111. The price of a bond having no expiration date is originally $8,000 and has a fixed annual interest payment of $800. A fall in the price of the bond by $3,000 will provide a new buyer of the bond an interest rate of: A) 10 percent. B) 12 percent. C) 14 percent. D) 16 percent. Answer: D

Use the following to answer questions 112-117: Answer the next question(s) on the basis of the following table:

Interest rate 2% 4 6 8 10

Transaction demand for money $220 220 220 220 220

Asset demand for money $300 280 260 240 220

Money supply $460 460 460 460 460

Type: T Topic: 4 E: 239 MA: 239 112. The transactions demand for money in the above money market would graph as a: A) vertical line. C) line sloping downward and to the right. B) horizontal line. D) line sloping upward and to the right. Answer: A

Type: T Topic: 4 E: 239 MA: 239 113. The total demand for money curve in the above money market would graph as a: A) vertical line. C) line sloping upward to the right. B) horizontal line. D) line sloping downward to the right. Answer: D

Type: T Topic: 4 E: 239 MA: 239 114. At equilibrium in the above money market, the total amount of money demanded is: A) $500. B) $480. C) $460. D) $440. Answer: C

Type: T Topic: 4 E: 239 MA: 239 115. Refer to the above table. The equilibrium interest rate is: A) 2 percent. B) 4 percent. C) 6 percent. D) 8 percent. Answer: D

McConnell/Brue: Economics, 16/e

Page 422

Chapter 13: Money and Banking

Type: T Topic: 4 E: 239 MA: 239 116. Refer to the above table. An increase in the money supply of $20 billion will cause the equilibrium interest rate to: A) fall by 4 percentage points. C) rise by 4 percentage points. B) fall by 2 percentage points. D) rise by 2 percentage points. Answer: B

Type: T Topic: 4 E: 239 MA: 239 117. All else equal, the transaction demand for money in the above table would increase if: A) nominal GDP increased. C) the supply of money increased. B) the interest rate fell. D) the economy's MPC declined. Answer: A

U.S. financial system

Type: F Topic: 5 E: 242 MA: 242 118. The basic policy-making body in the U.S. banking system is the: A) Federal Open Market Committee (FOMC). C) Federal Monetary Authority. B) Board of Governors of the Federal Reserve. D) Council of Economic Advisers. Answer: B

Type: F Topic: 5 E: 244 MA: 244 119. The twelve Federal Reserve Banks: A) are owned and operated by the U.S. Treasury. B) were created in 1776. Answer: C

C) hold the reserve deposits of commercial banks. D) are also known as national banks.

Type: F Topic: 5 E: 242 MA: 242 120. The Federal Reserve System was created in: A) 1926. B) 1946. C) 1895. D) 1913. Answer: D

Type: F Topic: 5 E: 242 MA: 242 121. In the U.S. economy the money supply is controlled by the: A) U.S. Treasury. C) Senate Committee on Banking and Finance. B) Federal Reserve System. D) Congress. Answer: B

Type: F Topic: 5 E: 243 MA: 243 Status: New 122. As it relates to Federal Reserve activities, the acronym FOMC describes the: A) Federal Open Market Committee. C) Federal Organization for Monetary Control. B) Federal Options Market Committee. D) Federal Organization for Money Creation. Answer: A

McConnell/Brue: Economics, 16/e

Page 423

Chapter 13: Money and Banking

Type: F Topic: 5 E: 243 MA: 243 Status: New 123. The Federal Open Market Committee (FOMC) is made up of: A) the chair of the Board of Governors along with the 12 presidents of the Federal Reserve Banks. B) the seven members of the Board of Governors along with the president of the New York Federal Reserve Bank. C) the seven members of the Board of Governors of the Federal Reserve System along with the three members of the Council of Economic Advisers. D) the seven member of the Board of Governors of the Federal Reserve System along with the president of the New York Federal Reserve Bank and four other Federal Reserve Banks presidents on a rotating basis. Answer: D

Type: F Topic: 5 E: 243 MA: 243 124. The group that sets the Federal Reserve Systems policy on buying and selling government securities (bills, notes, and bonds) is the: A) Federal Deposit Insurance Corporation (FDIC). C) Council of Economic Advisers. B) Federal Bond Sale Authority. D) Federal Open Market Committee (FOMC). Answer: D

Type: F Topic: 5 E: 244 MA: 244 Status: New 125. How many commercial banks are now operating in the United States? A) about 140,000 B) about 7,800 C) about 2,000 D) about 6,000 Answer: B

Type: F Topic: 5 E: 242 MA: 242 126. Which one of the following is true about the U. S. Federal Reserve System? A) There are 10 regional Federal Reserve Banks. B) The head of the U.S. Treasury also chairs the Federal Reserve Board. C) There are seven members of the Federal Reserve Board. D) The Open Market Committee is smaller in size than the Federal Reserve Board. Answer: C

Type: F Topic: 5 E: 243 MA: 243 127. Which one of the following is true about the U. S. Federal Reserve System? A) The head of the U.S. Treasury also chairs the Federal Reserve Board. B) There are 10 regional Federal Reserve Banks. C) There are 14 members of the Federal Reserve Board. D) The Federal Open Market Committee (FOMC) has more members than does the Federal Reserve Board. Answer: D

Type: F Topic: 5 E: 243 MA: 243 128. Which one of the following is true about the U. S. Federal Reserve System? A) There are 12 regional Federal Reserve Banks. B) The head of the U.S. Treasury also chairs the Federal Reserve Board. C) There are 14 members of the Federal Reserve Board. D) The Open Market Committee is smaller in size than the Federal Reserve Board. Answer: A

McConnell/Brue: Economics, 16/e

Page 424

Chapter 13: Money and Banking

Type: F Topic: 5 E: 243 MA: 243 129. In which of the following U. S. cities is one of the twelve Federal Reserve Banks located? A) Miami B) New Orleans C) San Francisco D) Denver Answer: C

Type: F Topic: 5 E: 243 MA: 243 130. In which of the following U. S. cities is one of the twelve Federal Reserve Banks located? A) New York City B) Seattle C) Miami D) Denver Answer: A

Type: F Topic: 5 E: 242 MA: 242 131. The Board of Governors of the Federal Reserve has ____ members. A) 5 B) 7 C) 9 D) 14 Answer: B

Type: F Topic: 5 E: 242-243 MA: 242-243 132. The members of the Federal Reserve Board: A) serve seven-year terms. B) are appointed by the American Economic Association. C) are elected by votes of the 12 presidents of the Federal Reserve Banks. D) serve 14-year terms. Answer: D

Type: F Topic: 5 E: 244-245 MA: 244-245 133. An important routine function of the Federal Reserve Bank is to: A) supervise the liquidation of the assets of bankrupt state banks. B) help large commercial banks develop correspondent relationships with smaller commercial banks. C) advise commercial banks as to the most profitable ways of reinvesting profits. D) provide facilities by which commercial banks and thrift institutions may collect checks. Answer: D

Type: A Topic: 5 E: 243 MA: 243 134. Which of the following statements best describes the twelve Federal Reserve Banks? A) They are privately owned and privately controlled central banks whose basic goal is to provide an ample and orderly market for U.S. Treasury securities. B) They are privately owned and publicly controlled central banks whose basic function is to minimize the risks in commercial banking in order to make it a reasonably profitable industry. C) They are privately owned and publicly controlled central banks whose basic goal is to control the money supply and interest rates in promoting the general economic welfare. D) They are privately owned and publicly controlled central banks whose basic goal is to earn profits for their owners. Answer: C

McConnell/Brue: Economics, 16/e

Page 425

Chapter 13: Money and Banking

Type: F Topic: 5 E: 242 MA: 242 135. The seven members of the Board of Governors of the Federal Reserve System are: A) appointed by the President with the confirmation of the Senate. B) elected by Congress from a slate of nominees provided by the President. C) appointed by the Senate Finance Committee. D) appointed by the presidents of the twelve Federal Reserve Banks. Answer: A

Type: D Topic: 5 E: 243 MA: 243 136. To say that the Federal Reserve Banks are quasi-public banks means that: A) they are privately owned, but managed in the public interest. B) they deal only with banks of foreign nations and do not have direct business contact with U.S. banks. C) they deal only with commercial banks, and not the public. D) they are publicly owned, but privately managed. Answer: A

Type: A Topic: 5 E: 245 MA: 245 137. Which of the following is the basic economic policy function of the Federal Reserve Banks? A) holding the deposits or reserves of commercial banks B) acting as fiscal agents for the Federal government C) controlling the supply of money D) the collection or clearing of checks among commercial banks Answer: C

Type: A Topic: 5 E: 243 MA: 243 138. The Federal Reserve System: A) is basically an independent agency. B) has the same status as the Supreme Court. C) has the status of a Congressional committee. D) is an agency of the executive branch of the Federal government. Answer: A

Type: F Topic: 5 E: 245 MA: 245 139. Research for industrially advanced countries indicates that: A) the more independent the central bank, the lower the average annual rate of inflation. B) the more independent the central bank, the higher the average annual rate of inflation. C) there is no relationship between the degree of independence of a country's central bank and its inflation rate. D) the more independent the central bank, the higher the average annual rate of unemployment. Answer: A

Type: F Topic: 5 E: 245 MA: 245 140. Research involving industrially advanced countries suggests that: A) the more independent the central bank, the lower the average annual growth of real GDP. B) the more independent the central bank, the higher the average annual growth of real GDP. C) there is no relationship between the degree of independence of a country's central bank and the growth rate of its real GDP. D) the less independent the central bank, the higher the average annual rate of inflation. Answer: D

McConnell/Brue: Economics, 16/e

Page 426

Chapter 13: Money and Banking

Type: A Topic: 5 E: 244 MA: 244 141. Commercial banks and thrift institutions: A) differ because thrifts cannot make loans. B) differ because thrifts cannot offer checkable deposits. C) have become less similar in recent years. D) have become increasingly similar in recent years. Answer: D

Type: F Topic: 5 E: 245 MA: 245 142. The traditional role of savings and loan associations has been to: A) finance business purchases of capital goods. B) purchase corporate stocks on behalf of their depositors. C) make installment loans to consumers. D) make mortgage loans on houses. Answer: D

Type: D Topic: 5 E: 244 MA: 244 143. The term thrift institution or thrifts includes: A) savings and loan associations, mutual savings banks, and credit unions. B) savings and loan associations, mutual savings banks, credit unions, and commercial banks. C) commercial banks and the twelve Federal Reserve Banks. D) any institution offering savings accounts. Answer: A

Recent developments and reform

Type: D Topic: 6 E: 246 MA: 246 144. Firms whose central business is to offer security advice and buy and sell individual stocks and bonds for clients are known as: A) thrifts. B) pension fund companies. C) securities firms. D) insurance companies. Answer: C

Type: D Topic: 6 E: 246 MA: 246 145. Firms whose central business is providing individual account shares of collections of stocks, bonds, or both are known as: A) insurance companies. B) thrifts. C) commercial banks. D) mutual funds companies. Answer: D

Type: C Topic: 6 E: 245 MA: 245 146. Which of these pairs of financial institutions are most alike in terms of their main lines of business? A) commercial banks and thrifts B) insurance companies and mutual fund companies. C) thrifts and securities firms. D) pension fund companies and commercial banks. Answer: A

McConnell/Brue: Economics, 16/e

Page 427

Chapter 13: Money and Banking

Type: F Topic: 6 E: 237 MA: 237 147. The Federal Deposit Insurance Corporation (FDIC) insures deposits up to $100,000 in: A) mutual fund companies and pension fund companies. B) thrifts and insurance companies. C) commercial banks and thrifts. D) securities firms and insurance companies. Answer: C

Type: F Topic: 6 E: 245 MA: 245 148. Banks and thrifts have responded to their relative declines by: A) expanding their services and merging with one another. B) merging with computer and software manufacturers. C) selling their ATMs to new startup firms. D) asking for trade protection against imported of financial services. Answer: A

Type: F Topic: 6 E: 245-246 MA: 245-246 149. Within the past decade, Congress has: A) permitted banks and thrifts to "self-insure" rather than participate in the FDIC system. B) allowed holders of government bonds to add these bonds to their insured checking accounts. C) ended restrictions on banks' merging with insurance companies, securities firms, and other firms offering financial services.. D) ended restrictions on banks' buying of nonbank firms such as manufacturers, corporate farms, and real estate companies. Answer: C

Type: F Topic: 6 E: 245 MA: 245 150. The bank and thrift share of total financial assets has: A) declined significantly since 1980. B) increased significantly since 1980. C) remained quite constant since the Second World War. D) increased in the United States but declined abroad. Answer: A

Type: F Topic: 6 E: 245 MA: 245 151. The share of total financial assets held by insurance companies, pension funds, mutual funds companies, and security-related firms has: A) declined significantly since 1980. B) increased significantly since 1980. C) has remained quite constant since the Second World War. D) decreased in the United States but increased abroad. Answer: B

McConnell/Brue: Economics, 16/e

Page 428

Chapter 13: Money and Banking

Type: F Topic: 6 E: 247 MA: 247 152. Which of the following is a true statement? A) The bank and thrift share of total financial assets has increased dramatically since 1980. B) The vast bulk of investment in the major nations is financed, not from internal saving, but from funds from abroad. C) The world's financial markets have become increasingly integrated. D) International stock and bond funds cannot be sold in the United States. Answer: C

Type: F Topic: 6 E: 246-247 MA: 246-247 153. In the 1990s, Congress passed legislation that: A) will eventually replace the $1 bill with a $1 coin. B) allows nonbank firms such as Chrysler and IBM to own large commercial banks or thifts. C) replaces the twelve Federal Reserve Banks with a single Central Bank. D) ends the legal separation of the banking industry and securities firms. Answer: D

Type: F Topic: 6 E: 246 MA: 246 154. The Financial Services Modernization Act of 1999: A) set limits on the fees that banks can charge for automatic teller machine (ATM) withdrawals. B) established a new dollar coin that will replace the dollar bill in 2005. C) permitted banks, thrifts, pension companies, and securities firms to merge and to sell each other's products. D) outlawed "payday loans" that are advanced against forthcoming payroll checks. Answer: C

Type: F Topic: 6 E: 247 MA: 247 155. Electronic money is: A) closely associated with smart cards. B) issued in real terms so that it is immune from the effects of inflation. C) the money dispensed by automatic teller machines (ATMs). D) also called share-draft money. Answer: A

Type: D Topic: 6 E: 247 MA: 247 156. Plastic cards that contain computer chips that store account balances are known as: A) credit cards. B) smart cards. C) debit cards. D) E-cards. Answer: B

Consider This Questions

Type: F E: 236 MA: 236 Status: New 157. (Consider This) Credits cards are: A) the fastest growing component of the M1 money supply. B) near-monies that are part of the M3 money supply but not part of the M2 or M1 money supplies. C) not money, officially defined. D) also known as time deposits. Answer: C

McConnell/Brue: Economics, 16/e

Page 429

Chapter 13: Money and Banking

Type: F E: 236 MA: 236 Status: New 158. (Consider This) Which of the following is not part of the M2 money supply? A) currency in circulation. C) small time deposits of less than $100,000. B) credit card balances. D) checkable deposits. Answer: B

Type: F E: 236 MA: 236 Status: New 159. (Consider This) Credit card balances are: A) a component of M1. B) a component of M2 but not of M1 Answer: D

C) a component of M3 but not of M2 or M1. D) not a component of M1, M2, or M3.

Last Word Questions

Type: F E: 248 MA: 248 160. (Last Word) The major countries in which citizens hold and use large quantities of U.S. dollars are: A) Germany, England, and France. C) Canada, Australia, and New Zealand. B) Russia, Argentina, and Poland. D) Egypt, Spain, and Italy. Answer: B

Type: F E: 248 MA: 248 161. (Last Word) The use of U.S. dollars in foreign countries: A) is illegal under international law. B) helps foreign buyers and sellers overcome problems with their domestic currencies. C) varies directly (positively) with U.S. interest rates. D) is less in volume than the use of foreign currencies in the United States. Answer: B

Type: F E: 248 MA: 248 162. (Last Word) The use of U.S. dollars in foreign countries: A) is illegal under international law. B) actually benefits the United States because each dollar costs less than a dollar to produce. C) varies directly (positively) with U.S. interest rates. D) is less in volume than the use of foreign currencies in the United States. Answer: B

True/False Questions

Type: F E: 233 MA: 233 163. The M2 money supply is larger than the M1 money supply. Answer: True

Type: F E: 237 MA: 237 164. Fiat money refers to all near monies. Answer: False

McConnell/Brue: Economics, 16/e

Page 430

Chapter 13: Money and Banking

Type: D E: 243 MA: 243 165. The twelve Federal Reserve Banks are governmentally owned but privately controlled. Answer: False

Type: F E: 244 MA: 244 166. The United States Treasury is the only source of money in the U.S. economy. Answer: False

Type: F E: 238-239 MA: 238-239 167. The higher the interest rate, the larger will be the amount of money demanded for transaction purposes. Answer: False

Type: A E: 240 MA: 240 168. The asset demand for money varies inversely with the nominal GDP. Answer: False

Type: A E: 245 MA: 245 169. Depository institutions are a major source of money in the U. S. economy. Answer: True

Type: F E: 233-234 MA: 233-234 170. Checkable deposits held in saving and loan institutions, mutual savings banks, and credit unions are part of the M1 definition of the money supply. Answer: True

Type: F E: 240 MA: 240 171. Bond prices and interest rates are directly or positively related. Answer: False

Type: A E: 234 MA: 234 172. Currency and coins held by banks are part of the M1 definition of money supply. Answer: False

Type: F E: 245 MA: 245 173. The percentage share of total U.S. financial assets held by commercial banks and thrifts has increased since 1980. Answer: False

Type: F E: 245 MA: 245 174. The number of U.S. banks has increased since 1990. Answer: False

McConnell/Brue: Economics, 16/e

Page 431

Chapter 13: Money and Banking

Type: F E: 246 MA: 246 175. Fidelity, Putnam, Dreyfus, and Kemper are examples of mutual fund companies. Answer: True

Type: F E: 244 MA: 244 176. Thrifts are known as "banker's banks" because they lend money to commercial banks. Answer: False

McConnell/Brue: Economics, 16/e

Page 432

CHAPTER 14

How Banks and Thrifts Create Money

Topic 1. 2. 3. 4. Bank balance sheets Banks and money creation Monetary multiplier Combined topics Last Word True-False

Question numbers 1-9 10-95 96-106 107-113 114-117 118-135

____________________________________________________________

_______________________________________

____________________________________________________________

_______________________________________

Multiple Choice Questions Bank balance sheets

Type: A Topic: 1 E: 252 MA: 252 1. Which of the following statements is correct? A) The actual reserves of a commercial bank equal its excess reserves minus its required reserves. B) A bank's liabilities plus its net worth equal its assets. C) When borrowers repay bank loans, the supply of money increases. D) A single commercial bank can safely lend a multiple amount of its excess reserves. Answer: B

Type: A Topic: 1 E: 252 MA: 252 2. A bank that has assets of $85 billion and a net worth of $10 billion must have: A) liabilities of $75 billion. C) liabilities of $10 billion. B) excess reserves of $10 billion. D) excess reserves of $75 billion. Answer: A

Type: A Topic: 1 E: 252 MA: 252 3. A bank that has liabilities of $150 billion and a net worth of $20 billion must have: A) excess reserves of $130 billion. C) excess reserves of $150 billion. B) assets of $150 billion. D) assets of $170 billion. Answer: D

Chapter 14: How Banks and Thrifts Create Money

Type: A Topic: 1 E: 252 MA: 252 4. If a bank has liabilities that exceed its net worth it: A) will not be able to meet the legal reserve ratio. B) is considered to be insolvent. C) most likely is a heavy borrower from its district Federal Reserve Bank. D) may or may not be a profitable firm. Answer: D

Type: D Topic: 1 E: 252 MA: 252 5. Which of the following describes the identity embodied in a balance sheet? A) Net Worth plus Assets equal Liabilities C) Assets equal Liabilities plus Net Worth B) Assets plus Liabilities equal Net Worth D) Assets plus Reserves equal Net Worth Answer: C

Type: D Topic: 1 E: 252 MA: 252 6. The claims of the owners of a firm against the firm's assets are called: A) working capital. B) assets. C) net worth. D) liabilities. Answer: C

Type: A Topic: 1 E: 252 MA: 252 7. Which of the following are all assets to a commercial bank? A) demand deposits, capital stock, and reserves C) vault cash, property, and capital stock B) vault cash, property, and reserves D) vault cash, capital stock, and demand deposits Answer: B

Type: D Topic: 1 E: 253 MA: 253 8. The reserves of a commercial bank consist of: A) the amount of money market funds it holds. B) deposits at the Federal Reserve Bank and vault cash. C) government securities that the bank holds. D) the bank's net worth. Answer: B

Type: A Topic: 1 E: 254 MA: 254 9. A commercial bank's reserves are: A) liabilities to both the commercial bank and the Federal Reserve Bank holding them. B) liabilities to the commercial bank and assets to the Federal Reserve Bank holding them. C) assets to both the commercial bank and the Federal Reserve Bank holding them. D) assets to the commercial bank and liabilities to the Federal Reserve Bank holding them. Answer: D

McConnell/Brue: Economics, 16/e

Page 434

Chapter 14: How Banks and Thrifts Create Money

Banks and money creation

Type: F Topic: 2 E: 253 MA: 253 10. The goldsmith's ability to create money was based on the fact that: A) withdrawals of gold tended to exceed deposits of gold in any given time period. B) consumers and merchants preferred to use gold for transactions, rather than paper money. C) the goldsmith was required to keep 100 percent gold reserves. D) paper money in the form of gold receipts was rarely redeemed for gold. Answer: D

Type: F Topic: 2 E: 253 MA: 253 11. When the receipts given by goldsmiths to depositors were used to make purchases: A) the gold standard was created. B) existing banking laws were violated. C) the receipts became in effect paper money. D) a fractional reserve banking system was created. Answer: C

Type: A Topic: 2 E: 255-256 MA: 255-256 12. The primary purpose of the legal reserve requirement is to: A) prevent banks from hoarding too much vault cash. B) provide a means by which the monetary authorities can influence the lending ability of commercial banks. C) prevent commercial banks from earning excess profits. D) provide a dependable source of interest income for commercial banks. Answer: B

Type: A Topic: 2 E: 255 MA: 255 13. The ABC Commercial Bank has $5,000 in excess reserves and the reserve ratio is 30 percent. The bank must have: A) $90,000 in outstanding loans and $35,000 in reserves. B) $90,000 in checkable deposit liabilities and $32,000 in reserves. C) $20,000 in checkable deposit liabilities and $10,000 in reserves. D) $90,000 in checkable deposit liabilities and $35,000 in reserves. Answer: B

Type: A Topic: 2 E: 253 MA: 253 14. Which one of the following is presently a major deterrent to bank panics in the United States? A) the legal reserve requirement C) the gold standard B) the fractional reserve system D) deposit insurance Answer: D

Type: A Topic: 2 E: 258 MA: 258 15. Commercial banks monetize claims when they: A) collect checks through the Federal Reserve System. B) make loans to the public. C) accept repayment of outstanding loans. D) borrow from the Federal Reserve Banks. Answer: B

McConnell/Brue: Economics, 16/e

Page 435

Chapter 14: How Banks and Thrifts Create Money

Type: F Topic: 2 E: 252 MA: 252 16. Most modern banking systems are based on: A) money of intrinsic value. B) commodity money. Answer: D

C) 100 percent reserves. D) fractional reserves.

Type: A Topic: 2 E: 259 MA: 259 17. Money is destroyed when: A) loans are made. B) checks written on one bank are deposited in another bank. C) loans are repaid. D) the net worth of the banking system declines. Answer: C

Type: D Topic: 2 E: 254 MA: 254 18. Checkable deposits are also called: A) checking accounts. B) high-powered money. Answer: A

C) savings balances.

D) Federal Reserve Notes.

Type: A Topic: 2 E: 255 MA: 255 19. Suppose a commercial bank has checkable deposits of $100,000 and the legal reserve ratio is 10 percent. If the bank's required and excess reserves are equal, then its actual reserves: A) are $30,000. B) are $10,000. C) are $20,000. D) cannot be determined from the given information. Answer: C

Type: A Topic: 2 E: 258 MA: 258 20. Banks create money when they: A) add to their reserves in the Federal Reserve Bank. B) accept deposits of cash. C) sell government bonds. D) exchange checkable deposits for the IOU's of businesses and individuals. Answer: D

McConnell/Brue: Economics, 16/e

Page 436

Chapter 14: How Banks and Thrifts Create Money

Use the following to answer questions 21-24: Answer the next question(s) on the basis of the following table for a commercial bank or thrift:
Reserve requirement, percent W 8 12 20 Checkable deposits $100,000 X 200,000 300,000 Actual reserves $ 10,000 20,000 Y 70,000 Excess reserves $ 0 12,000 8,000 Z

(1) (2) (3) (4)

Type: A Topic: 2 E: 254-255 MA: 254-255 Status: New 21. Refer to row 1 in the above table. The number appropriate for space W is: A) 4. B) 6. C) 10. D) 12. Answer: C

Type: A Topic: 2 E: 254-255 MA: 254-255 Status: New 22. Refer to row 2 in the above table. The number appropriate for space X is: A) $20,000. B) $60,000. C) $200,000. D) $100,000. Answer: D

Type: A Topic: 2 E: 254-255 MA: 254-255 Status: New 23. Refer to row 3 in the above table. The number appropriate for space Y is: A) $24,000. B) $32,000. C) $48,000. D) $96,000. Answer: B

Type: A Topic: 2 E: 254-255 MA: 254-255 Status: New 24. Refer to row 4 in the above table. The number appropriate for space Z is: A) $10,000. B) $70,000. C) $48,000. D) zero. Answer: A

Type: A Topic: 2 E: 256 MA: 256 25. When a check is drawn and cleared, the A) reserves and deposits of both the bank against which the check is cleared and the bank receiving the check are unchanged by this transaction. B) bank against which the check is cleared loses reserves and deposits equal to the amount of the check. C) bank receiving the check loses reserves and deposits equal to the amount of the check. D) bank against which the check is cleared acquires reserves and deposits equal to the amount of the check. Answer: B

Type: A Topic: 2 E: 255 MA: 255 26. Suppose the ABC bank has excess reserves of $4,000 and outstanding checkable deposits of $80,000. If the reserve requirement is 25 percent, what is the size of the bank's actual reserves? A) $16,000 B) $84,000 C) $24,000 D) $20,000 Answer: C

McConnell/Brue: Economics, 16/e

Page 437

Chapter 14: How Banks and Thrifts Create Money

Type: D Topic: 2 E: 255 MA: 255 27. Excess reserves refer to the: A) difference between a bank's vault cash and its reserves deposited at the Federal Reserve Bank. B) minimum amount of actual reserves a bank must keep on hand to back up its customers deposits. C) difference between actual reserves and loans. D) difference between actual reserves and required reserves. Answer: D

Type: A Topic: 2 E: 255 MA: 255 28. Suppose the reserve requirement is 10 percent. If a bank has $5 million of checkable deposits and actual reserves of $500,000, the bank: A) can safely lend out $500,000. C) can safely lend out $50,000 B) can safely lend out $5 million. D) cannot safely lend out more money Answer: D

Type: A Topic: 2 E: 255 MA: 255 29. A reserve requirement of 20 percent means a bank must have $1000 of reserves if its checkable deposits are: A) $100. B) $1,000. C) $5,000. D) $12,000. Answer: C

Type: A Topic: 2 E: 255 MA: 255 30. Assume that a bank initially has no excess reserves. If it receives $5,000 in cash from a depositor and the bank finds that it can safely lend out $4,500, the reserve requirement must be: A) zero. B) 10 percent. C) 20 percent. D) 25 percent Answer: B

Type: A Topic: 2 E: 258-259 MA: 258-259 31. Suppose the reserve requirement is 20 percent. If a bank has checkable deposits of $4 million and actual reserves of $1 million, it can safely lend out: A) $1 million. B) $1.2 million. C) $200,000. D) $800,000. Answer: C

Type: A Topic: 2 E: 255 MA: 255 32. Suppose that a bank's actual reserves are $5 million, its checkable deposits are $5 million, and its excess reserves are $3 million. The reserve requirement must be: A) 40 percent. B) 20 percent. C) 10 percent. D) 5 percent. Answer: A

McConnell/Brue: Economics, 16/e

Page 438

Chapter 14: How Banks and Thrifts Create Money

Type: T Topic: 2 E: 255 MA: 255 33. Assume the Continental National Bank's balance statement is as follows:

Assets Reserves $ 40,000 Loans 25,000 Securities 110,000

Liabilities and net worth Checkable deposits $130,000 Capital stock 45,000

Assuming a legal reserve ratio of 20 percent, how much excess reserves would this bank have after a check for $10,000 was drawn and cleared against it? A) $3,000 B) $24,000 C) $6,000 D) $16,000 Answer: C

Type: D Topic: 2 E: 255 MA: 255 34. The reserve ratio refers to the ratio of a bank's: A) reserves to its liabilities and net worth. B) capital stock to its total assets. C) checkable deposits to its total liabilities. D) reserves and vault cash to its checkable deposits. Answer: D

Type: A Topic: 2 E: 258 MA: 258 35. The amount that a commercial bank can lend is determined by its: A) required reserves. B) excess reserves. C) outstanding loans. Answer: B

D) outstanding checkable deposits.

Type: A Topic: 2 E: 259 MA: 259 36. A commercial bank can expand its excess reserves by: A) demanding and receiving payment on an overdue loan. B) buying bonds from a Federal Reserve Bank. C) buying bonds from the public. D) paying back money borrowed from a Federal Reserve Bank. Answer: A

Type: D Topic: 2 E: 258 MA: 258 37. Commercial banks create money when they: A) accept cash deposits from the public. B) purchase government securities from the central banks. C) create checkable deposits in exchange for IOUs. D) raise their interest rates. Answer: C

Type: A Topic: 2 E: 259 MA: 259 38. Banks destroy money when they: A) buy government bonds. C) fail to reissue loans that are paid off. B) accept deposits of cash into checkable accounts. D) clear checks against another bank. Answer: C

McConnell/Brue: Economics, 16/e

Page 439

Chapter 14: How Banks and Thrifts Create Money

Type: A Topic: 2 E: 258 MA: 258 39. Individual commercial banks are limited in their ability to create money by lending because: A) lending is likely to result in the loss of reserves to other banks. B) only the Treasury and the Federal Reserve Banks are authorized to create new money. C) the Board of Governors prohibits bank lending when the result is an expansion of the money supply. D) banking is a highly competitive industry. Answer: A

Type: A Topic: 2 E: 258 MA: 258 40. Assume Company X deposits $100,000 in cash in commercial Bank A. If no excess reserves exist at the time this deposit is made and the reserve ratio is 20 percent, Bank A can increase the money supply by a maximum of: A) $50,000. B) $180,000. C) $80,000. D) $500,000. Answer: C

Type: C Topic: 2 E: 258 MA: 258 41. Assume that Smith deposits $600 in currency into her checking account in the XYZ Bank. Later that same day Jones negotiates a loan for $1,200 at the same bank. In what direction and by what amount has the supply of money changed? A) decreased by $600 B) increased by $1,800 C) increased by $600 D) increased by $1,200 Answer: D

Type: A Topic: 2 E: 258 MA: 258 42. Assume the Standard Internet Company negotiates a loan for $5,000 from the Metro National Bank and receives a checkable deposit for that amount in exchange for its promissory note (IOU). As a result of this transaction: A) the supply of money is increased by $5,000. B) the supply of money declines by the amount of the loan. C) a claim has been "demonetized." D) the Metro Bank acquires reserves from other banks. Answer: A

Use the following to answer questions 43-47: Use the following balance sheet for the ABC National Bank in answering the next question(s). Assume the required reserve ratio is 20 percent.
Assets Reserves $ 27,000 Loans 50,000 Securities 33,000 Property 200,000 Liabilities and net worth Checkable deposits $110,000 Capital stock 200,000

Type: T Topic: 2 E: 255 MA: 255 43. Refer to the above data. This commercial bank has excess reserves of: A) $0. B) $3,000. C) $12,000. D) $5,000. Answer: D

McConnell/Brue: Economics, 16/e

Page 440

Chapter 14: How Banks and Thrifts Create Money

Type: T Topic: 2 E: 258 MA: 258 44. Refer to the above data. This bank can safely expand its loans by a maximum of: A) $7,000. B) $25,000. C) $12,000. D) $5,000. Answer: D

Type: T Topic: 2 E: 258-259 MA: 258-259 45. Refer to the above data. Assuming the bank has a check cleared against it for the amount loaned in the previous question, its reserves and checkable deposits will now be: A) $25,000 and $122,000 respectively. C) $32,000 and $115,000 respectively. B) $22,000 and $110,000 respectively. D) $22,000 and $105,000 respectively. Answer: B

Type: T Topic: 2 E: 256 MA: 256 46. Refer to the above data. After the transaction described in the previous question is completed, the bank will now have excess reserves of: A) $0. B) $3,000. C) $12,000. D) $5,000. Answer: A

Type: T Topic: 2 E: 261 MA: 261 47. Refer to the above data. If the original balance sheet was for the commercial banking system, rather than a single bank, loans and checkable deposits could have been expanded by a maximum of: A) $8,000. B) $15,000. C) $48,000. D) $25,000. Answer: D

Type: A Topic: 2 E: 258 MA: 258 48. A single commercial bank must meet a 25 percent reserve requirement. If the bank has no excess reserves initially and $5,000 of cash is deposited in the bank, it can increase its loans by a maximum of: A) $1,250. B) $120,000. C) $5,000. D) $3,750. Answer: D

Type: A Topic: 2 E: 259 MA: 259 49. When a bank loan is repaid the supply of money: A) is constant, but its composition will have changed. B) is decreased. C) is increased. D) may either increase or decrease. Answer: B

Type: A Topic: 2 E: 255 MA: 255 50. If you deposit a $50 bill in a commercial bank that has a 10 percent legal reserve requirement the bank will: A) have $45 of additional excess reserves. C) be capable of lending an additional $50. B) be capable of lending an additional $500. D) have $50 of required reserves. Answer: A

McConnell/Brue: Economics, 16/e

Page 441

Chapter 14: How Banks and Thrifts Create Money

Type: A Topic: 2 E: 258 MA: 258 51. When a commercial bank has excess reserves: A) it is in a position to make additional loans. B) its actual reserves are less than its required reserves. C) it is charging too high an interest rate on its loans. D) its reserves exceed its assets. Answer: A

Type: A Topic: 2 E: 256 MA: 256 52. If we both have checking accounts in the same commercial bank and I write a check in your favor for $200, the bank's: A) balance sheet will be unchanged. B) reserves and checkable deposits will both decline by $200. C) liabilities will decline by $200, but its net worth will increase by $200. D) assets and liabilities will both decline by $200. Answer: A

Type: A Topic: 2 E: 258 MA: 258 53. Which of the following is correct? A) Both the granting and repaying of bank loans expand the aggregate money supply. B) Granting and repaying bank loans do not affect the money supply. C) Granting a bank loan destroys money; repaying a bank loan creates money. D) Granting a bank loan creates money; repaying a bank loan destroys money. Answer: D

Type: A Topic: 2 E: 255 MA: 255 54. If the reserve requirement is 10 percent, how much excess reserves does a bank acquire when a business deposits a $500 check drawn on another bank? A) $450 B) $550 C) $5000 D) $500 Answer: A

Type: A Topic: 2 E: 254 MA: 254 55. The amount of reserves that a commercial bank is required to hold is equal to: A) the amount of its checkable deposits. B) the sum of its checkable deposits and time deposits. C) its checkable deposits multiplied by the reserve requirement. D) its checkable deposits divided by its total assets. Answer: C

Type: A Topic: 2 E: 259 MA: 259 56. Banks create money when they: A) allow loans to mature. B) accept deposits of cash. Answer: C

C) buy government bonds from households. D) sell government bonds from households.

McConnell/Brue: Economics, 16/e

Page 442

Chapter 14: How Banks and Thrifts Create Money

Type: A Topic: 2 E: 260 MA: 260 57. In prosperous times banks are likely to hold very small amounts of excess reserves because: A) the Fed wants commercial banks to increase the money supply during economic expansions. B) it is very costly to transfer funds between commercial banks and the central banks. C) the Federal Reserve Banks do not pay interest on bank reserves. D) the Federal Reserve Banks want to minimize their interest payments on such deposits. Answer: C

Type: D Topic: 2 E: 255 MA: 255 58. Which of the following is correct? A) Required reserves minus actual reserves equal excess reserves. B) Required reserves equal excess reserves minus actual reserves. C) Required reserves equal actual reserves plus excess reserves. D) Actual reserves minus required reserves equal excess reserves. Answer: D

Use the following to answer questions 59-60: Answer the next question(s) on the basis of the following balance sheet for the First National Bank of Bunco. All figures are in millions.

Assets Reserves Securities Loans Property

$20 25 15 90

Liabilities and net worth Checkable Deposits $100 Capital Stock 50

Type: T Topic: 2 E: 255 MA: 255 59. Refer to the above data. If this bank has excess reserves of $6 million, the legal reserve ratio must be: A) 10 percent. B) 12 percent. C) 14 percent. D) 20 percent. Answer: C

Type: T Topic: 2 E: 255 MA: 255 60. Refer to the above data. Suppose that customers of this bank collectively write checks for cash at the bank in the amount of $6 million. As a result, the bank's excess reserves diminish to: A) $0. B) $6 million. C) $.72 million. D) $.84 million. Answer: D

Type: A Topic: 2 E: 256 MA: 256 61. When a bank has a check drawn and cleared against it: A) excess reserves in the banking system decline. B) the nation's total money supply falls. C) the bank's balance sheet does not change. D) the amount of required reserves the bank must have will fall. Answer: D

McConnell/Brue: Economics, 16/e

Page 443

Chapter 14: How Banks and Thrifts Create Money

Type: A Topic: 2 E: 255 MA: 255 62. A bank's actual reserves can be found by: A) adding its required and excess reserves. B) subtracting its required reserves from its excess reserves. C) multiplying its excess reserves by the reserve ratio. D) multiplying its checkable deposits by the reserve ratio. Answer: A

Type: A Topic: 2 E: 255 MA: 255 63. Suppose a savings and loan association has checkable deposits of $500,000 and the legal reserve ratio is 10 percent. If the institution has excess reserves of $4,000, then its actual reserves are: A) $46,000. B) $50,000. C) $54,000. D) $4,000. Answer: C

Type: F Topic: 2 E: 254 MA: 254 64. The legal reserve ratio applies to checkable deposits at: A) national banks. B) credit unions. C) savings and loans. Answer: D

D) institutions of all of the above types.

Type: A Topic: 2 E: 258 MA: 258 65. Suppose a commercial banking system has $100,000 of outstanding checkable deposits and actual reserves of $35,000. If the reserve ratio is 20 percent, the banking system can expand the supply of money by the maximum amount of: A) $122,000. B) $175,000. C) $300,000. D) $75,000. Answer: D

Type: A Topic: 2 E: 258 MA: 258 66. If a portion of the loans extended by commercial banks is taken as cash rather than as checkable deposits, the maximum money-creating potential of the commercial banking system will: A) be equal to twice the reciprocal of the reserve ratio. B) be unaffected. C) increase. D) decrease. Answer: D

Type: A Topic: 2 E: 261 MA: 261 67. The basic reason why the commercial banking system can increase its checkable deposits by a multiple of its excess reserves is that: A) reserves lost by any particular bank will be gained by some other bank. B) the central banks follow policies that prevent reserves from falling below the level required by law. C) the MPC of borrowers is greater than zero, but less than 1. D) the banking system must keep reserves equal to 100 percent of its checkable-deposit liabilities. Answer: A

McConnell/Brue: Economics, 16/e

Page 444

Chapter 14: How Banks and Thrifts Create Money

Use the following to answer questions 68-70: Answer the next question(s) on the basis of the following consolidated balance sheet for the commercial banking system. Assume the required reserve ratio is 30 percent. All figures are in billions. Assets Reserves Securities Loans Property $ 51 100 109 10 Liabilities and net worth Checkable Deposits $140 Capital Stock 130

Type: T Topic: 2 E: 255 MA: 255 68. Refer to the above data. The commercial banking system has excess reserves of: A) $9 billion. B) $7 billion. C) $6.1 billion. D) $5 billion. Answer: A

Type: T Topic: 2 E: 261 MA: 261 69. Refer to the above data. The maximum amount by which the commercial banking system can expand the supply of money by lending is: A) $30 billion. B) $23.1 billion. C) $27 billion. D) $15 billion. Answer: A

Type: T Topic: 2 E: 258 MA: 258 70. Refer to the above data. If the commercial banking system actually loans the maximum amount it is able to lend: A) reserves and deposits equal to that amount will be gained. B) excess reserves will be $2.6 billion. C) excess reserves will fall to $1.7 billion. D) excess reserves will be reduced to zero. Answer: D

Type: A Topic: 2 E: 258 MA: 258 71. If borrowers take a portion of their loans as currency rather than checkable deposits, the maximum amount by which the commercial banking system can increase the money supply by lending will: A) decrease because the legal reserve ratio varies directly with the amount of currency in circulation. B) increase because currency is the basis for all checkable-deposit creation. C) decrease because the amount of reserves transferred to other banks will diminish. D) increase because commercial banks can lend by the reciprocal of the amount of currency in circulation. Answer: C

Type: A Topic: 2 E: 259 MA: 259 72. Given a 25 percent reserve ratio, assume the commercial banking system is loaned up. Now assume the reserve ratio is reduced to 20 percent. As a result of this reduction: A) we can expect bank lending and bank profits to decline. B) each dollar of bank reserves will now support a maximum of $5 of checkable deposits. C) the banking system must now reduce outstanding loans by 5 percent. D) the banking system can now increase lending by 5 percent. Answer: B

McConnell/Brue: Economics, 16/e

Page 445

Chapter 14: How Banks and Thrifts Create Money

Type: A Topic: 2 E: 259 MA: 259 73. When commercial banks use excess reserves to buy government securities from the public: A) new money is created. C) the money supply falls. B) commercial bank reserves increase. D) checkable deposits decline. Answer: A

Type: A Topic: 2 E: 260 MA: 260 74. Which of the following would reduce the money supply? A) Commercial banks use excess reserves to buy government bonds from the public. B) Commercial banks loan out excess reserves. C) Commercial banks sell government bonds to the public. D) A check clears from Bank A to Bank B. Answer: C

Type: A Topic: 2 E: 264 MA: 264 75. In an unregulated banking system there will be a tendency for: A) interest rates to vary directly with the rate of increase in the money supply. B) the money supply to grow at a constant rate through time. C) the money supply to decrease during recession. D) the money supply to increase during recession. Answer: C

Use the following to answer questions 76-78: Answer the next question(s) on the basis of the following consolidated balance sheet for the commercial banking system. Assume the required reserve ratio is 10 percent. All figures are in billions. Assets Reserves $ 30 Securities 70 Loans 130 Property 200 Liabilities and net worth Checkable Deposits $300 Capital Stock 130

Type: T Topic: 2 E: 255 MA: 255 76. Refer to the above data. The commercial banking system has excess reserves of: A) $0 billion. B) $30 billion. C) $60 billion. D) $70 billion. Answer: A

Type: T Topic: 2 E: 255 MA: 255 77. Refer to the above data. After a deposit of $10 billion of new currency into a checking account in the banking system, excess reserves will increase by: A) $0 billion. B) $7 billion. C) $9 billion. D) $10 billion. Answer: C

Type: T Topic: 2 E: 261 MA: 261 78. Refer to the above data. After the deposit, the maximum amount by which this commercial banking system can expand the supply of money by lending is: A) $9 billion. B) $45 billion. C) $36 billion. D) $90 billion. Answer: D

McConnell/Brue: Economics, 16/e

Page 446

Chapter 14: How Banks and Thrifts Create Money

Use the following to answer questions 79-82: Answer the next question(s) on the basis of the following information about a banking system: new currency deposited in the system = $40 billion; legal reserve ratio = 0.20; excess reserves prior to the currency deposit = $0.

Type: A Topic: 2 E: 254 MA: 254 79. Refer to the above information. The $40 billion deposit of currency into checking accounts will initially create: A) $8 billion of new checkable deposits. C) $40 billion of new checkable deposits. B) $10 billion of new checkable deposits. D) $160 billion of new checkable deposits. Answer: C

Type: A Topic: 2 E: 255 MA: 255 80. Refer to the above information. The $40 billion deposit of currency into checking accounts will create excess reserves of: A) $20 billion. B) $32 billion. C) $40 billion. D) $0. Answer: B

Type: A Topic: 2 E: 261 MA: 261 81. Refer to the above information. The banking system will be able to expand the money supply through loans by: A) $160 billion. B) $200 billion. C) $40 billion. D) $128 billion. Answer: A

Type: A Topic: 2 E: 263 MA: 263 82. Refer to the above information. The $40 billion deposit of new currency will support total checkable deposits of: A) $160 billion. B) $200 billion. C) $40 billion. D) $128 billion. Answer: B

Use the following to answer questions 83-87: Answer the next question(s) on the basis of the following table for a commercial bank or thrift:
(1) Legal reserve ratio, percent 10 20 25 30 (2) Checkable deposits $40,000 40,000 40,000 40,000 (3) Actual reserves $10,000 10,000 10,000 10,000

Type: T Topic: 2 E: 255 MA: 255 83. Refer to the above table. When the legal reserve ratio is 25 percent, the excess reserves of this single bank are: A) $0. B) $1,000. C) $5,000. D) $30,000. Answer: A

McConnell/Brue: Economics, 16/e

Page 447

Chapter 14: How Banks and Thrifts Create Money

Type: T Topic: 2 E: 258 MA: 258 84. Refer to the above table. When the legal reserve ratio is 10 percent, the money creating potential of this single bank is: A) $0. B) $6,000. C) $30,000. D) $60,000. Answer: B

Type: T Topic: 2 E: 261 MA: 261 85. Refer to the above table. When the legal reserve ratio is 20 percent, the money creating potential of the entire banking system is: A) $4,000. B) $6,000. C) $8,000. D) $10,000. Answer: D

Type: T Topic: 2 E: 262 MA: 262 86. Refer to the above table. When the legal reserve ratio is 30 percent, the monetary multiplier is: A) 5. B) 4. C) 3.33. D) 2.5. Answer: C

Type: T Topic: 2 E: 255 MA: 255 87. Refer to the above table. If the legal reserve ratio falls from 25 percent to 10 percent, excess reserves of this single bank will: A) rise by $6,000 and the monetary multiplier will increase from 4 to 10. B) rise by $60,000 and the monetary multiplier will increase from 4 to 10. C) fall by $6,000 and the monetary multiplier will decline from 30 to 10. D) fall by $2,000 and the monetary multiplier will decline from 10 to 4. Answer: A

Type: A Topic: 2 E: 255 MA: 255 88. If excess reserves in the banking system are $4,000, checkable deposits are $40,000, and the legal reserve ratio is 10 percent, then actual reserves are: A) $4,000. B) $6,000. C) $8,000. D) $5,000. Answer: C

Type: A Topic: 2 E: 255 MA: 255 89. If actual reserves in the banking system are $8,000, checkable deposits are $70,000, and the legal reserve ratio is 10 percent, then excess reserves are: A) zero. B) $1,000. C) $2,000. D) $500. Answer: B

Type: A Topic: 2 E: 255 MA: 255 90. If actual reserves in the banking system are $40,000, excess reserves are $10,000, and checkable deposits are $240,000, then the legal reserve requirement is: A) 10 percent. B) 12.5 percent. C) 20 percent. D) 5 percent. Answer: B

McConnell/Brue: Economics, 16/e

Page 448

Chapter 14: How Banks and Thrifts Create Money

Type: C Topic: 2 E: 262-263 MA: 262-263 91. If actual reserves in the banking system are $50,000, excess reserves are $5,000, and checkable deposits are $225,000, then the monetary multiplier is: A) 10. B) 4. C) 5. D) 10. Answer: C

Type: D Topic: 2 E: 260 MA: 260 92. Overnight loans from one bank to another for reserve purposes entail an interest rate called the: A) prime rate. B) discount rate. C) Federal funds rate. D) treasury bill rate. Answer: C

Type: A Topic: 2 E: 260 MA: 260 93. A bank temporarily short of required reserves may be able to remedy this situation by: A) borrowing funds in the Federal funds market. B) granting new loans. C) shifting some of its vault cash to its reserve account at the Federal Reserve. D) buying bonds from the public. Answer: A

Type: D Topic: 2 E: 260 MA: 260 94. The market for immediately available reserve balances at the Federal Reserve is known as the: A) money market. C) short-term bond market. B) long-term bond market. D) Federal funds market. Answer: D

Type: D Topic: 2 E: 260 MA: 260 95. The Federal funds market is the market in which: A) banks borrow from the Federal Reserve Banks. B) U.S. securities are bought and sold. C) banks borrow reserves from one another on an overnight basis. D) Federal Reserve Banks borrow from one another. Answer: C

Monetary multiplier Type: A Topic: 3 E: 262 MA: 262 96. The multiple by which the commercial banking system can expand the supply of money is equal to the reciprocal of: A) the MPS. B) its actual reserves. C) its excess reserves. D) the reserve ratio. Answer: D

Type: A Topic: 3 E: 263 MA: 263 97. The multiple by which the commercial banking system can expand the supply of money on the basis of excess reserves: A) is larger the smaller the legal reserve ratio. B) is the reciprocal of the bank's actual reserves. C) is directly or positively related to the size of the required reserve ratio. D) will be zero when the required reserve ratio is 100 percent. Answer: A

McConnell/Brue: Economics, 16/e

Page 449

Chapter 14: How Banks and Thrifts Create Money

Type: A Topic: 3 E: 263 MA: 263 98. The multiple by which the commercial banking system can increase the supply of money on the basis of each dollar of excess reserves is equal to: A) the reciprocal of the legal reserve ratio. C) the reciprocal of the income velocity of money. B) 1 minus the legal reserve ratio. D) 1/MPS. Answer: A

Type: E Topic: 3 E: 263 MA: 263 99. If m equals the maximum number of new dollars that can be created for a single dollar of excess reserves and R equals the required reserve ratio, then for the banking system: A) m = R - 1. B) R = m/1. C) R = m - 1. D) m = 1/R. Answer: D

Type: E Topic: 3 E: 263 MA: 263 100. If D equals the maximum amount of a new demand-deposit money that can be created by the banking system on the basis of any given amount of excess reserves; E equals the amount of excess reserves; and m is the monetary multiplier, then: A) m = E/D. B) D = E × m. C) D = E - 1/m. D) D = m/E. Answer: B

Type: A Topic: 3 E: 263 MA: 263 101. If the reserve ratio is 15 percent and commercial bankers decide to hold additional excess reserves equal to 5 percent of any newly acquired checkable deposits, then the relevant monetary multiplier for the banking system will be: A) 31/2. B) 4. C) 5. D) 10. Answer: C

Type: A Topic: 3 E: 263 MA: 263 102. Other things equal, if the required reserve ratio was lowered: A) banks would have to reduce their lending. B) the size of the monetary multiplier would increase. C) the actual reserves of banks would increase. D) the Federal funds interest rate would rise. Answer: B

Type: A Topic: 3 E: 263 MA: 263 103. If the monetary authorities want to reduce the monetary multiplier, they should: A) lower the legal reserve ratio. C) increase bank reserves. B) raise the legal reserve ratio. D) lower interest rates. Answer: B

Type: A Topic: 3 E: 263 MA: 263 104. If the reserve ratio is 100 percent, the value of the monetary multiplier is: A) 0. B) 1. C) 10. D) 100. Answer: B

McConnell/Brue: Economics, 16/e

Page 450

Chapter 14: How Banks and Thrifts Create Money

Type: E Topic: 3 E: 262 MA: 262 105. The value of the monetary multiplier is: A) 1/MPS. B) 1/Excess Reserves. C) 1/MPC. Answer: D

D) 1/Required Reserve Ratio.

Type: A Topic: 3 E: 263 MA: 263 106. The greater the legal reserve ratio, the: A) higher is the income multiplier. B) lower is the income multiplier. Answer: C

C) lower is the monetary multiplier. D) higher is the monetary multiplier.

Combined topics

Use the following to answer questions 107-113: Answer the next question(s) on the basis of the following information for the Moola Bank.

Reserves Checkable deposits Loans (to customers) Property Securities (owned) Capital stock

$ 100 1000 300 400 300 0

Type: A Topic: 4 E: 253-254 MA: 253-254 107. Assume that the listed amounts constitute this bank's complete set of accounts. Moola's: A) assets are $1000. B) liabilities are $1000. C) net worth is zero. D) profit is $1000. Answer: B

Type: A Topic: 4 E: 253-254 MA: 253-254 108. Assume that the listed amounts constitute this bank's complete set of accounts. Moola's: A) assets are $1100. B) liabilities are $1100. C) net worth is $300. D) profit is $1000. Answer: A

Type: C Topic: 4 E: 253-254 MA: 253-254 109. Assume that the listed amounts constitute this bank's complete set of accounts. Moola's: A) assets are $1000. B) liabilities are $300. C) net worth is $100. D) annual profit is $200. Answer: C

Type: C Topic: 4 E: 254 MA: 254 110. Refer to the above information. If Moola Bank is legally "loaned up," the reserve requirement must be: A) 10 percent. B) 15 percent. C) 20 percent. D) 25 percent. Answer: A

McConnell/Brue: Economics, 16/e

Page 451

Chapter 14: How Banks and Thrifts Create Money

Type: C Topic: 4 E: 262-263 MA: 262-263 111. Refer to the above information. If Moola Bank is legally "loaned up," the banking system's monetary multiplier must be: A) 5. B) 8. C) 10. D) 20. Answer: C

Type: A Topic: 4 E: 262-263 MA: 262-263 112. Refer to the above information and assumed that Moola bank is "loaned up." If it receives a $100 deposit of currency, it could safely expand its loans by: A) $100. B) $90. C) $900. D) $1000. Answer: B

Type: A Topic: 4 E: 263 MA: 263 113. Refer to the above information and assume that Moola Bank is "loaned up." If it receives a $100 deposit of currency, the banking system of which Moola is a part could expand loans by: A) $100. B) $90. C) $900. D) $1000. Answer: C

Last Word Questions

Type: F E: 265 MA: 265 114. (Last Word) The bank panics of 1930-1933: A) resulted in the passage of the Smoot-Hawley Act. B) boosted the nation's money supply, causing inflation. C) directly resulted in the Federal insured deposit program. D) caused a significant outflow of gold from the United States. Answer: C

Type: F E: 265 MA: 265 115. (Last Word) A "national bank holiday" that closed all banks for a week and resulted in Federal deposit insurance occurred in the United States in: A) 1903, following the "Louisiana stampede." B) 1987, following the collapse of numerous savings and loan associations. C) 1945, following the end of the Second World War. D) 1933, following the bank panics of 1930-1933. Answer: D

Type: F E: 265 MA: 265 116. (Last Word) Which of the following represents a change in today's banking policies that should prevent a recurrence of the bank panics of 1930-1933? A) banks are more cautious lenders B) banks keep large amounts of excess reserves on hand C) the FDIC insures bank deposits and therefore depositors do not panic and rush to withdraw money when individual banks have financial problems D) the President now has the authority to close banks whenever panics occur Answer: C

McConnell/Brue: Economics, 16/e

Page 452

Chapter 14: How Banks and Thrifts Create Money

Type: F E: 265 MA: 265 117. (Last Word) The bank panics of 1930-1933 and the resulting failures of many banks were caused by: A) the widespread conversion of checkable deposits to cash by the public. B) the raising of the reserve requirement by the Board of Governors. C) a massive inflow of gold bullion to the United States. D) a massive inflow of cash into bank deposits by citizens who feared their money was losing its value. Answer: A

True/False Questions

Type: A E: 255 MA: 255 118. Excess reserves are the amount by which required reserves exceed actual reserves. Answer: False

Type: D E: 259 MA: 259 119. The supply of money increases when the public buys government securities from commercial banks. Answer: False

Type: A E: 255 MA: 255 120. Commercial bank reserves are an asset to commercial banks but a liability to the Federal Reserve Bank holding them. Answer: True

Type: A E: 258-259 MA: 258-259 121. Commercial banks increase the supply of money when they purchase either personal IOU's or government bonds from businesses and households. Answer: True

122.

Type: A E: 252 MA: 252 Balance sheets always balance because reserves must always equal liabilities plus net worth Answer: False

Type: A E: 259 MA: 259 123. When commercial banks retire outstanding loans, the supply of money is increased. Answer: False

Type: A E: 260 MA: 260 124. Commercial banks monetize claims when they sell securities to Federal Reserve Banks. Answer: False

Type: A E: 255 MA: 255 125. Actual reserves equal required reserves plus excess reserves. Answer: True

McConnell/Brue: Economics, 16/e

Page 453

Chapter 14: How Banks and Thrifts Create Money

Type: D E: 262 MA: 262 126. The banking system can lend by a multiple of its excess reserves because lending does not result in a loss of reserves to the banking system as a whole. Answer: True

Type: A E: 262 MA: 262 127. The monetary multiplier and the income multiplier are two ways of referring to the same concept. Answer: False

Type: D E: 264 MA: 264 128. In an uncontrolled or unregulated system commercial bank lending will tend to intensify the business cycle. Answer: True

Type: F E: 258 MA: 258 Status: New 129. Loans made to customers are a liability on a bank's balance sheet. Answer: False

Type: F E: 254 MA: 254 Status: New 130. Checkable deposits are a liability on a bank's balance sheet. Answer: True

Type: A E: 259 MA: 259 Status: New 131. An individual bank can safely lend out a multiply of its excess reserves, but the banking system can safely lend out only an amount equal to the excess reserves in the banking system. Answer: False

Type: F E: 263 MA: 263 Status: New 132. If the reserve requirement is 10 percent, the monetary multiplier will be 10. Answer: True

Type: F E: 263 MA: 263 Status: New 133. If the reserve requirement is 20 percent, the monetary multiplier will be 4. Answer: False

Type: A E: 263 MA: 263 Status: New 134. The higher the reserve requirement, the lower is the monetary multiplier. Answer: True

McConnell/Brue: Economics, 16/e

Page 454

CHAPTER 15

Monetary Policy

Topic 1. 2. 3. 4. 5. 6. 7. 8. Fed balance sheet and general Open-market operations Reserve ratio Discount rate Monetary policy and the economy Assessment/recent policy International complications AD-AS/policy summary Consider This Last Word True-False

Question numbers 1-14 15-39 40-53 54-62 63-97 98-135 136-151 152-156 157-158 159-162 163-176

____________________________________________________________

_______________________________________

____________________________________________________________

_______________________________________

Multiple Choice Questions Fed balance sheet and general

Type: A Topic: 1 E: 269 MA: 269 1. Which of the following is an asset on the consolidated balance sheet of the Federal Reserve Banks? A) loans to commercial banks C) Treasury deposits B) Federal Reserve Notes in circulation D) reserves of commercial banks Answer: A

Type: A Topic: 1 E: 269 MA: 269 2. Reserves must be deposited in the Federal Reserve Banks by: A) only commercial banks which are members of the Federal Reserve System. B) all depository institutions, that is, all commercial banks and thrift institutions. C) state chartered commercial banks only. D) federally chartered commercial banks only. Answer: B

Type: F Topic: 1 E: 269 MA: 269 3. The securities held as assets by the Federal Reserve Banks consist mainly of: A) corporate bonds. C) common stock. B) Treasury bills and Treasury bonds. D) certificates of deposit. Answer: B

Chapter 15: Monetary Policy

Type: A Topic: 1 E: 270 MA: 270 4. Federal Reserve Notes in circulation are: A) an asset as viewed by the Federal Reserve Banks. B) a liability as viewed by the Federal Reserve Banks. C) neither an asset nor a liability as viewed by the Federal Reserve Banks. D) part of M1, but not of M2 or M3. Answer: B

Type: A Topic: 1 E: 270 MA: 270 5. Which of the following will increase commercial bank reserves? A) the purchase of government bonds in the open market by the Federal Reserve Banks B) a decrease in the reserve ratio C) an increase in the discount rate D) the sale of government bonds in the open market by the Federal Reserve Banks Answer: A

Type: A Topic: 1 E: 274 MA: 274 6. When a commercial bank borrows from a Federal Reserve Bank: A) the supply of money automatically increases. B) it indicates that the commercial bank is unsound financially. C) the commercial bank's lending ability is increased. D) the commercial bank's reserves are reduced. Answer: C

Type: A Topic: 1 E: 272 MA: 272 7. The Federal Reserve Banks sell government securities to the public. As a result, the checkable deposits: A) of commercial banks are unchanged, but their reserves increase. B) and reserves of commercial banks both decrease. C) of commercial banks are unchanged, but their reserves decrease. D) of commercial banks are both unchanged. Answer: B

Type: A Topic: 1 E: 270 MA: 270 8. The Federal Reserve Banks buy government securities from commercial banks. As a result, the checkable deposits: A) of commercial banks are unchanged, but their reserves increase. B) and reserves of commercial banks both decrease. C) of commercial banks are unchanged, but their reserves decrease. D) and reserves of commercial banks are both unchanged. Answer: A

Type: A Topic: 1 E: 274 MA: 274 9. The commercial banking system borrows from the Federal Reserve Banks. As a result, the checkable deposits: A) of commercial banks are unchanged, but their reserves increase. B) and reserves of commercial banks both decrease. C) of commercial banks are unchanged, but their reserves decrease. D) and reserves of commercial banks are both unchanged. Answer: A

McConnell/Brue: Economics, 16/e

Page 456

Chapter 15: Monetary Policy

Type: D Topic: 1 E: 270 MA: 270 10. Which of the following is a tool of monetary policy? A) open market operations C) changes in tax rates B) changes in banking laws D) changes in government spending Answer: A

Type: A Topic: 1 E: 275 MA: 275 11. Commercial banks and thrifts usually hold only small amounts of excess reserves because: A) the presence of such reserves tends to boost interest rates and reduce investment. B) the Fed constantly uses open market operations to eliminate excess reserves. C) the Fed does not pay interest on reserves. D) the Fed does not want commercial banks and thrifts to be too liquid. Answer: C

Type: F Topic: 1 E: 270 MA: 270 12. In the United States monetary policy is the responsibility of the: A) U.S. Treasury. B) Department of Commerce. C) Board of Governors of the Federal Reserve System. D) U.S. Congress. Answer: C

Type: A Topic: 1 E: 270 MA: 270 13. The three main tools of monetary policy are: A) tax rate changes, the discount rate, and open-market operations. B) tax rate changes, changes in government expenditures, and open-market operations. C) the discount rate, the reserve ratio, and open-market operations. D) changes in government expenditures, the reserve ratio, and the discount rate. Answer: C

Type: A Topic: 1 E: 270 MA: 270 14. The Fed can change the money supply by: A) changing bank reserves through the sale or purchase of government securities. B) changing the quantities of required and excess reserves by altering the legal reserve ratio. C) changing the discount rate so as to encourage or discourage commercial banks in borrowing from the central banks. D) doing all of the above. Answer: D

McConnell/Brue: Economics, 16/e

Page 457

Chapter 15: Monetary Policy

Open-market operations

Type: A Topic: 2 E: 270-271 MA: 270-271 15. Assume the reserve ratio is 25 percent and Federal Reserve Banks buy $4 million of U.S. securities from the public, which deposits this amount into checking accounts. As a result of these transactions, the supply of money is: A) not directly affected, but the money-creating potential of the commercial banking system is increased by $12 million. B) directly increased by $4 million and the money-creating potential of the commercial banking system is increased by $16 million. C) directly reduced by $4 million and the money-creating potential of the commercial banking system is decreased by $12 million. D) directly increased by $4 million and the money-creating potential of the commercial banking system is increased by $12 million. Answer: D

Type: A Topic: 2 E: 274-275 MA: 274-275 16. Assume the legal reserve ratio is 25 percent and the Fourth National Bank borrows $10,000 from the Federal Reserve Bank in its district. As a result: A) commercial bank reserves are increased by $10,000. B) the supply of money automatically declines by $7,500. C) commercial bank reserves are increased by $7,500. D) the supply of money is automatically increased by $10,000. Answer: A

Type: D Topic: 2 E: 270 MA: 270 17. Open-market operations refer to: A) purchases of stocks in the New York Stock Exchange. B) the purchase or sale of government securities by the Fed. C) central bank lending to commercial banks. D) the specifying of loan maximums on stock purchases. Answer: B

Type: A Topic: 2 E: 270-271 MA: 270-271 18. If the Federal Reserve System buys government securities from commercial banks and the public: A) commercial bank reserves will decline. B) commercial bank reserves will be unaffected. C) it will be easier to obtain loans at commercial banks. D) the money supply will contract. Answer: C

Type: A Topic: 2 E: 271 MA: 271 19. The purchase of government securities from the public by the Fed will cause: A) commercial bank reserves to decrease. C) demand deposits to decrease B) the money supply to increase. D) the interest rate to increase. Answer: B

McConnell/Brue: Economics, 16/e

Page 458

Chapter 15: Monetary Policy

Type: A Topic: 2 E: 271-272 MA: 271-272 20. Assume that a single commercial bank has no excess reserves and that the reserve ratio is 20 percent. If this bank sells a bond for $1,000 to a Federal Reserve Bank, it can expand its loans by a maximum of: A) $1,000. B) $2,000. C) $800. D) $5,000. Answer: A

Type: A Topic: 2 E: 272-273 MA: 272-273 21. Suppose the Federal Reserve Banks sell $2 billion of government bonds to the public which pays for them by drawing checks. As a result, commercial bank reserves will: A) increase by $10 billion. C) decrease by $2 billion. B) remain unchanged. D) increase by $2 billion. Answer: C

Type: A Topic: 2 E: 271 MA: 271 22. Which of the following statements is correct? A) The supply of money decreases when the Federal Reserve Banks buy government securities from households or businesses. B) Excess reserves are the amount by which actual reserves exceed required reserves. C) Commercial banks decrease the supply of money when they purchase government bonds from households or businesses. D) Commercial bank reserves are a liability to commercial banks but an asset to the Federal Reserve Banks. Answer: B

Use the following to answer questions 23-26:
CONSOLIDATED BALANCE SHEET: COMMERCIAL BANKING SYSTEM Liabilities and net worth Assets $ 72 Checkable deposits $240 110 Loans from Federal 60 Reserve Banks 2

Reserves Securities Loans

CONSOLIDATED BALANCE SHEET: FEDERAL RESERVE BANKS Assets Liabilities and net worth Securities $240 Reserves of commercial Loans to commercial Banks $72 banks 2 Treasury deposits 30 Federal Reserve Notes 140

Type: T Topic: 2 E: 261 MA: 261 23. Refer to the above balance sheets. The commercial banks have excess reserves of: A) $12. B) $22. C) $16. D) $24. Answer: A

McConnell/Brue: Economics, 16/e

Page 459

Chapter 15: Monetary Policy

Type: T Topic: 2 E: 261 MA: 261 24. Refer to the above balance sheets. The maximum money-creating potential of the commercial banking system is: A) $36. B) $17. C) $48. D) $24. Answer: C

Type: T Topic: 2 E: 270-271 MA: 270-271 25. Refer to the above balance sheets. Suppose the Federal Reserve Banks buy $2 in securities from the public, which deposits this amount into checking accounts. As a result of these transactions, the supply of money will: A) be unaffected but the money-creating potential of the commercial banking system will increase by $6. B) directly decrease by $2 and the money-creating potential of the commercial banking system will be unaffected. C) directly increase by $8 and the money-creating potential of the commercial banking system will increase by $32. D) directly increase by $2 and the money-creating potential of the commercial banking system will increase by $6. Answer: D

Type: T Topic: 2 E: 271-272 MA: 271-272 26. Refer to the above balance sheets. Suppose the Federal Reserve Banks sell $2 in securities directly to the commercial banks. As a result of this transaction the supply of money: A) will decrease by $2, but the money-creating potential of the commercial banking system will not be affected. B) is not directly affected, but the money-creating potential of the commercial banking system will decrease by $8. C) will directly increase by $2 and the money-creating potential of the commercial banking system will decrease by $8. D) will directly increase by $2 and the money-creating potential of the commercial banking system will increase by $8. Answer: B

Type: A Topic: 2 E: 275 MA: 275 27. The Federal Reserve System regulates the money supply primarily by: A) controlling the production of coins at the United States mint. B) altering the reserve requirements of commercial banks and thereby the ability of banks to make loans. C) altering the reserves of commercial banks, largely through sales and purchases of government bonds. D) restricting the issuance of Federal Reserve Notes because paper money is the largest portion of the money supply. Answer: C

Type: A Topic: 2 E: 270-271 MA: 270-271 28. Assuming no currency drains, when the Federal Reserve Banks purchase government securities the reserves of commercial banks are: A) decreased by a multiple of the amount of the purchase. B) decreased by the amount of the purchase. C) increased by a multiple of the amount of the purchase. D) increased by the amount of the purchase. Answer: D

McConnell/Brue: Economics, 16/e

Page 460

Chapter 15: Monetary Policy

Type: A Topic: 2 E: 270-271 MA: 270-271 29. Which of the following is correct? When the Federal Reserve buys government securities from the public, the money supply: A) contracts and commercial bank reserves increase. B) expands and commercial bank reserves decrease. C) contracts and commercial bank reserves decrease. D) expands and commercial bank reserves increase. Answer: D

Type: A Topic: 2 E: 271 MA: 271 30. Which of the following will happen when the Federal Reserve buys bonds from the public in the open market and cash in the hands of the public does not change? A) the required reserve ratio will increase C) the deposits of commercial banks will decline B) the money supply will decrease D) commercial bank reserves will increase Answer: D

Use the following to answer questions 31-32: Answer the next question(s) on the assumption that the legal reserve ratio is 20 percent. Suppose that the Fed sells $500 of government securities to commercial banks and buys $500 of securities from individuals, who deposit the cash in checking accounts.

Type: C Topic: 2 E: 270-272 MA: 270-272 31. As a result of the above transactions, reserves in the banking system will: A) remain unchanged. B) rise by $100. C) fall by $100. D) rise by $1000. Answer: A

Type: C Topic: 2 E: 270-272 MA: 270-272 32. As a result of the above transactions, the supply of money in the economy will: A) remain unchanged. B) rise by $500. C) fall by $100. D) fall by $500. Answer: A

Type: A Topic: 2 E: 270-273 MA: 270-273 33. Open-market operations change: A) the size of the monetary multiplier, but not commercial bank reserves. B) commercial bank reserves, but not the size of the monetary multiplier. C) neither commercial bank reserves nor the size of the monetary multiplier. D) both commercial bank reserves and the size of the monetary multiplier. Answer: B

McConnell/Brue: Economics, 16/e

Page 461

Chapter 15: Monetary Policy

Use the following to answer questions 34-39: Answer the next question(s) on the basis of the following consolidated balance sheet of the commercial banking system. Assume that the reserve requirement is 10 percent. All figures are in billions and each question should be answered independently of changes specified in the preceding ones.
Assets Reserves $ 60 Securities 140 Loans 260 Property 400 Liabilities and net worth Checkable deposits $600 Capital stock 260

Type: A Topic: 2 E: 261 MA: 261 Status: New 34. Refer to the above data. The commercial banking system has excess reserves of: A) $10 billion. B) $5 billion. C) $2 billion. D) zero. Answer: D

Type: A Topic: 2 E: 261 MA: 261 Status: New 35. Refer to the above data. The monetary multiplier for the commercial banking system is: A) 5. B) 10. C) 12.5. D) 20. Answer: B

Type: A Topic: 2 E: 271-272 MA: 271-272 Status: New 36. Refer to the above data. Suppose the Fed sold $10 billion of U.S. securities to the banks. This would: A) increase bank reserves to $70 billion, reduce bank-held securities to $130 billion, and increase the money supply (checkable deposits) by $100 billion. B) increase bank reserves to $70 billion, reduce bank-held securities to $130 billion, and decrease the money supply (checkable deposits) by $100 billion. C) reduce bank reserves to $50 billion, increase bank-held securities to $150 billion, and increase the money supply (checkable deposits) by $100 billion. D) reduce bank reserves to $50 billion, increase bank-held securities to $150 billion, and decrease the money supply (checkable deposits) by $100 billion. Answer: D

Type: A Topic: 2 E: 270 MA: 270 Status: New 37. Refer to the above data. Suppose the Fed bought $20 billion of U.S. securities from the banks. This would: A) increase bank reserves to $80 billion, reduce bank-held securities to $120 billion, and increase the money supply (checkable deposits) by $200 billion. B) increase bank reserves to $80 billion, reduce bank-held securities to $120 billion, and decrease the money supply (checkable deposits) by $200 billion. C) reduce bank reserves to $40 billion, increase bank-held securities to $160 billion, and increase the money supply (checkable deposits) by $200 billion. D) reduce bank reserves to $40 billion, increase bank-held securities to $160 billion, and decrease the money supply (checkable deposits) by $200 billion. Answer: A

McConnell/Brue: Economics, 16/e

Page 462

Chapter 15: Monetary Policy

Type: A Topic: 2 E: 270 MA: 270 Status: New 38. Refer to the above data. Suppose the Fed wants to increase the money supply by $400 billion to drive down interest rates and stimulate the economy. To accomplish this it could: A) sell $20 billion of U.S. securities to the banks. B) buy $20 billion of U.S. securities from the banks. C) sell $40 billion of U.S. securities to the banks. D) buy $40 billion of U.S. securities from the banks. Answer: D

Type: A Topic: 2 E: 271-272 MA: 271-272 Status: New 39. Refer to the above data. Suppose the Fed wants to reduce the money supply by $400 billion to drive up interest rates and dampen inflation. To accomplish this it could: A) sell $20 billion of U.S. securities to the banks. B) buy $20 billion of U.S. securities from the banks. C) sell $40 billion of U.S. securities to the banks. D) buy $40 billion of U.S. securities from the banks. Answer: C

Reserve ratio

Type: A Topic: 3 E: 273-274 MA: 273-274 40. If the Fed were to increase the legal reserve ratio, we would expect: A) lower interest rates, an expanded GDP, and depreciation of the dollar. B) lower interest rates, an expanded GDP, and appreciation of the dollar. C) higher interest rates, a contracted GDP, and appreciation of the dollar. D) higher interest rates, a contracted GDP, and depreciation of the dollar. Answer: C

Type: A Topic: 3 E: 273-274 MA: 273-274 41. An increase in the legal reserve ratio: A) increases the money supply by increasing excess reserves and increasing the monetary multiplier. B) decreases the money supply by decreasing excess reserves and decreasing the monetary multiplier. C) increases the money supply by decreasing excess reserves and decreasing the monetary multiplier. D) decreases the money supply by increasing excess reserves and decreasing the monetary multiplier. Answer: B

Type: A Topic: 3 E: 273-274 MA: 273-274 42. When the reserve requirement is increased: A) required reserves are changed into excess reserves. B) the excess reserves of member banks are increased. C) a single commercial bank can no longer lend dollar-for-dollar with its excess reserves. D) the excess reserves of member banks are reduced. Answer: D

McConnell/Brue: Economics, 16/e

Page 463

Chapter 15: Monetary Policy

Type: C Topic: 3 E: 273-274 MA: 273-274 43. Assume that the commercial banking system has checkable deposits of $10 billion and excess reserves of $1 billion at a time when the reserve requirement is 20 percent. If the reserve requirement is now raised to 30 percent, the banking system then has: A) excess reserves of $2 billion. C) a deficiency of reserves of $.5 billion. B) neither an excess nor a deficiency of reserves. D) excess reserves of only $.5 billion. Answer: B

Type: A Topic: 3 E: 273-274 MA: 273-274 44. When the required reserve ratio is increased, the excess reserves of member banks are: A) reduced, but the multiple by which the commercial banking system can lend is unaffected. B) reduced and the multiple by which the commercial banking system can lend is increased. C) increased and the multiple by which the commercial banking system can lend is increased. D) reduced and the multiple by which the commercial banking system can lend is reduced. Answer: D

Type: A Topic: 3 E: 274 MA: 274 45. When the required reserve ratio is decreased, the excess reserves of member banks are: A) reduced, but the multiple by which the commercial banking system can lend is unaffected. B) reduced and the multiple by which the commercial banking system can lend is increased. C) increased and the multiple by which the commercial banking system can lend is increased. D) increased and the multiple by which the commercial banking system can lend is reduced. Answer: C

Type: A Topic: 3 E: 274 MA: 274 46. A decrease in the reserve ratio increases the: A) amount of actual reserves in the banking system. B) amount of excess reserves in the banking system. C) number of government securities held by the Federal Reserve Banks. D) ratio of coins to paper currency in the economy. Answer: B

Type: A Topic: 3 E: 273-274 MA: 273-274 47. An increase in the reserve ratio: A) increases the size of the spending income multiplier. B) decreases the size of the spending income multiplier. C) increases the size of the monetary multiplier. D) decreases the size of the monetary multiplier. Answer: D

McConnell/Brue: Economics, 16/e

Page 464

Chapter 15: Monetary Policy

Use the following to answer questions 48-53: Answer the next question(s) on the basis of the following consolidated balance sheet of the commercial banking system. Assume that the reserve requirement is 20 percent. All figures are in billions and each question should be answered independently of changes specified in all preceding ones.
Assets Reserves $ 200 Securities 300 Loans 500 Property 400 Liabilities and net worth Checkable deposits $1000 Capital stock 400

Type: A Topic: 3 E: 261 MA: 261 Status: New 48. Refer to the above data. The commercial banking system has excess reserves of: A) zero. B) $2 billion. C) $5 billion. D) $10 billion. Answer: A

Type: A Topic: 3 E: 262 MA: 262 Status: New 49. Refer to the above data. The monetary multiplier for the commercial banking system is: A) 5. B) 10. C) 15. D) 20. Answer: A

Type: A Topic: 3 E: 273-274 MA: 273-274 Status: New 50. Refer to the above data. If the Fed increased the reserve requirement from 20 percent to 25 percent, a deficiency of reserves in the commercial banking system of _____ would occur and the monetary multiplier would fall to ____. A) $50 billion; 5. B) $10 billion; 4 C) $50 billion; 4 D) $10 billion; 8 Answer: C

Type: A Topic: 3 E: 274 MA: 274 Status: New 51. Refer to the above data. If the Fed reduced the reserve requirement from 20 percent to 16 percent, excess reserves in the commercial banking system would increase by _____ and the monetary multiplier would rise to ____. A) $10 billion; 5 B) $40 billion; 6.25 C) $10 billion; 10 D) $40 billion; 12.5 Answer: B

Type: A Topic: 3 E: 274 MA: 274 Status: New 52. Refer to the above data. Suppose the Fed wants to increase the money supply by $1000 billion to drive down interest rates and stimulate the economy. To accomplish this it could lower the reserve requirement from 20 percent to: A) 10 percent. B) 12 percent. C) 14 percent. D) 12 percent. Answer: A

Type: A Topic: 3 E: 273-274 MA: 273-274 Status: New 53. Refer to the above data. Suppose the Fed wants to reduce the money supply by $200 billion to drive up interest rates and dampen inflation. To accomplish this it could increase the reserve requirement from 20 percent to: A) 22 percent. B) 25 percent.. C) 30 percent. D) 33 percent. Answer: B

McConnell/Brue: Economics, 16/e

Page 465

Chapter 15: Monetary Policy

Discount rate

Type: D Topic: 4 E: 274 MA: 274 54. The discount rate is the interest: A) rate at which the central banks lend to the U.S. Treasury. B) rate at which the Federal Reserve Banks lend to commercial banks. C) yield on long-term government bonds. D) rate at which commercial banks lend to the public. Answer: B

Type: A Topic: 4 E: 274 MA: 274 55. A commercial bank can add to its actual reserves by: A) lending money to bank customers. B) buying government securities from the public. C) buying government securities from a Federal Reserve Bank. D) borrowing from a Federal Reserve Bank. Answer: D

Type: D Topic: 4 E: 274 MA: 274 56. The interest rate at which the Federal Reserve Banks lend to commercial banks is called the: A) prime rate. B) short-term rate. C) discount rate. D) Federal funds rate. Answer: C

Type: D Topic: 4 E: 274 MA: 274 57. The discount rate is the rate of interest at which: A) Federal Reserve Banks lend to commercial banks. B) savings and loan associations lend to some builders. C) Federal Reserve Banks lend to large corporations. D) commercial banks lend to large corporations. Answer: A

Type: D Topic: 4 E: 274 MA: 274 Status: New 58. Projecting that it might temporarily fall short of legally required reserves in the coming days, the Bank of Beano decides to borrow money from its regional Federal Reserve Bank. The interest rate on the loan is called the: A) prime rate. B) Federal funds rate. C) Treasury bill rate. D) discount rate. Answer: D

Type: D Topic: 4 E: 274 MA: 274 Status: New 59. When the Fed lends money to a commercial bank, the bank: A) increases its reserves and enhances its ability to extend credit to bank customers. B) decreases its reserves and reduces its ability to extend credit to bank customers. C) pays the Federal funds interest rate on the loan. D) pays the prime rate interest rate on the loan. Answer: A

McConnell/Brue: Economics, 16/e

Page 466

Chapter 15: Monetary Policy

Type: C Topic: 4 E: 274 MA: 274 Status: New 60. Suppose that, for every 1-percentage point decline in the discount rate, commercial banks collectively borrow an additional $2 billion from Federal Reserve banks. Also assume that reserve ratio is 10 percent. If the Fed lowers the discount rate from 4.0 percent to 3.5 percent, bank reserves will: A) increase by $1 billion and the money supply will increase by $5 billion. B) decline by $1 billion and the money supply will decline by $10 billion. C) increase by $1 billion and the money supply will increase by $10 billion. D) increase by $10 billion and the money supply will increase by $100 billion. Answer: C

Type: C Topic: 4 E: 274 MA: 274 Status: New 61. Suppose that, for every 1-percentage point decline of the discount rate, commercial banks collectively borrow an additional $2 billion from Federal Reserve banks. Also assume that reserve ratio is 20 percent. If the Fed increases the discount rate from 4.0 percent to 4.25 percent, bank reserves will: A) increase by $.5 billion and the money supply will increase by $2.5 billion. B) decline by $.5 billion and the money supply will decline by $2.5 billion. C) increase by $.75 billion and the money supply will increase by $3.75 billion. D) increase by $1 billion and the money supply will increase by $5 billion. Answer: A

Type: F Topic: 4 E: 275 MA: 275 Status: New 62. Changes in the discount rate are: A) the most powerful and useful tool of monetary policy. B) less frequent than changes in the reserve requirement. C) more important than open-market operations. D) less important than open-market operations in implementing monetary policy. Answer: D

Monetary policy and the economy

Type: A Topic: 5 E: 277 MA: 277 63. Which of the following best describes the cause-effect chain of an easy money policy? A) A decrease in the money supply will lower the interest rate, increase investment spending, and increase aggregate demand and GDP. B) A decrease in the money supply will raise the interest rate, decrease investment spending, and decrease aggregate demand and GDP. C) An increase in the money supply will raise the interest rate, decrease investment spending, and decrease aggregate demand and GDP. D) An increase in the money supply will lower the interest rate, increase investment spending, and increase aggregate demand and GDP. Answer: D

Type: A Topic: 5 E: 279 MA: 279 64. Upon which of the following industries is a tight money policy likely to be most effective? A) furniture B) clothing C) food processing D) residential construction Answer: D

McConnell/Brue: Economics, 16/e

Page 467

Chapter 15: Monetary Policy

Type: C Topic: 5 E: 277 MA: 277 65. Assuming government wishes to either increase or decrease the level of aggregate demand, which of the following pairs are not consistent policy measures? A) a tax increase and an increase in the money supply B) a tax reduction and an increase in the money supply C) a reduction in government expenditures and a decline in the money supply D) a tax increase and an increase in the interest rate Answer: A

Type: A Topic: 5 E: 279 MA: 279 66. If the Federal Reserve authorities were attempting to reduce demand-pull inflation, the proper policies would be to: A) sell government securities, raise reserve requirements, and raise the discount rate. B) buy government securities, raise reserve requirements, and raise the discount rate. C) sell government securities, lower reserve requirements, and lower the discount rate. D) sell government securities, raise reserve requirements, and lower the discount rate. Answer: A

Type: A Topic: 5 E: 279 MA: 279 67. A contraction of the money supply: A) increases the interest rate and decreases aggregate demand. B) increases both the interest rate and aggregate demand. C) lowers the interest rate and increases aggregate demand. D) lowers both the interest rate and aggregate demand. Answer: A

Type: C Topic: 5 E: 278-279 MA: 278-279 68. If the Fed were to purchase government securities in the open market, we would anticipate: A) lower interest rates, an expanded GDP, and depreciation of the dollar. B) lower interest rates, an expanded GDP, and appreciation of the dollar. C) higher interest rates, a contracted GDP, and depreciation of the dollar. D) lower interest rates, a contracted GDP, and appreciation of the dollar. Answer: A

Type: D Topic: 5 E: 279 MA: 279 69. The purpose of a tight money policy is to: A) alleviate recessions. B) raise interest rates and restrict the availability of bank credit. C) increase aggregate demand and GDP. D) increase investment spending. Answer: B

Type: A Topic: 5 E: 278 MA: 278 70. Monetary policy is expected to have its greatest impact on: A) Xg. B) Ig. C) C. D) G. Answer: B

McConnell/Brue: Economics, 16/e

Page 468

Chapter 15: Monetary Policy

Type: A Topic: 2 E: 278 MA: 278 71. Which of the following actions by the Fed would cause the money supply to increase? A) purchases of government bonds from banks. C) an increase in the discount rate. B) an increase in the reserve requirement. D) sales of government bonds to the public. Answer: A

Type: C Topic: 5 E: 278-279 MA: 278-279 72. Assume the economy is operating at less than full employment. An easy money policy will cause interest rates to ________. which will ___________ investment spending. A) decrease; decrease B) decrease; increase C) increase; increase D) increase; decrease Answer: B

Type: A Topic: 5 E: 276 MA: 276 73. Which of the following best describes the cause-effect chain of a tight money policy? A) A decrease in the money supply will lower the interest rate, increase investment spending, and increase aggregate demand and GDP. B) A decrease in the money supply will raise the interest rate, decrease investment spending, and decrease aggregate demand and GDP. C) An increase in the money supply will raise the interest rate, decrease investment spending, and decrease aggregate demand and GDP. D) An increase in the money supply will lower the interest rate, decrease investment spending, and increase aggregate demand and GDP. Answer: B

Type: C Topic: 5 E: 279 MA: 279 74. If the economy were encountering a severe recession, proper monetary and fiscal policies would call for: A) selling government securities, raising the reserve ratio, lowering the discount rate, and a budgetary surplus. B) buying government securities, reducing the reserve ratio, reducing the discount rate, and a budgetary deficit. C) buying government securities, raising the reserve ratio, raising the discount rate, and a budgetary surplus. D) buying government securities, reducing the reserve ratio, raising the discount rate, and a budgetary deficit. Answer: B

Type: C Topic: 5 E: 279 MA: 279 75. If severe demand-pull inflation was occurring in the economy, proper government policies would involve a government: A) deficit and the purchase of securities in the open market, a higher discount rate, and higher reserve requirements. B) deficit and the sale of securities in the open market, a higher discount rate, and lower reserve requirements. C) surplus and the sale of securities in the open market, a higher discount rate, and higher reserve requirements. D) surplus and the purchase of securities in the open market, a lower discount rate, and lower reserve requirements. Answer: C

McConnell/Brue: Economics, 16/e

Page 469

Chapter 15: Monetary Policy

Type: A Topic: 5 E: 277 MA: 277 76. If the amount of money demanded exceeds the amount supplied, the: A) demand-for-money curve will shift to the left. C) interest rate will rise. B) money supply curve will shift to the right. D) interest rate will fall. Answer: C

Use the following to answer questions 77-79:
10
Interest rate (%) Price level

8 6 4 2 0 $10

Investment demand

AS

AD3 (I = 40) AD2 (I = 30) AD1 (I = 20)

20 30 40 Investment ($)

50

0 Real GDP ($)

Type: G Topic: 5 E: 276-277 MA: 276-277 77. Refer to the above diagrams. The numbers in parentheses after the AD1, AD2, and AD3, labels indicate the levels of investment spending associated with each curve, respectively. All numbers are in billions of dollars. If the interest rate is 8 percent and the goal of the Fed is full-employment output of Qf, it should: A) increase the interest rate from 8 percent to 10 percent. B) decrease the interest rate from 8 to 4 percent. C) decrease the interest rate from 8 to 6 percent. D) maintain the interest rate at 8 percent. Answer: C

Type: G Topic: 5 E: 276-277 MA: 276-277 78. Refer to the above diagrams. The numbers in parentheses after the AD1, AD2, and AD3 labels indicate the levels of investment spending associated with each curve, respectively. All numbers are in billions of dollars. If the interest rate is 4 percent and the Fed desires to undo demand-pull inflation, it should: A) increase the interest rate from 4 percent to 6 percent. B) decrease the interest rate from 4 to 2 percent. C) increase investment spending by $20 billion. D) maintain the interest rate at 4 percent. Answer: A

Type: G Topic: 5 E: 276-277 MA: 276-277 79. Refer to the above diagrams. The numbers in parentheses after the AD1, AD2, and AD3 labels indicate the levels of investment spending associated with each curve, respectively. All numbers are in billions of dollars. If the interest rate is 6 percent and the goal of the Fed is full-employment output of Qf, it should: A) increase the interest rate from 6 percent to 8 percent. B) decrease the interest rate from 6 to 4 percent. C) decrease the interest rate from 6 to 2 percent. D) maintain the interest rate at 6 percent. Answer: D

McConnell/Brue: Economics, 16/e

Page 470

Chapter 15: Monetary Policy

Type: A Topic: 5 E: 275 MA: 275 80. The purpose of an easy money policy is to shift the: A) aggregate demand curve leftward. C) aggregate supply curve leftward. B) aggregate demand curve rightward. D) investment demand curve leftward. Answer: B

Use the following to answer questions 81-84:
Ms1 MS0 MS3

Interest rate (%)

6 3 Dm 0
$80 100 120 Amount of money demanded and supplied

Interest rate

9

Investment demand

0

$40 50 Investment

60

AS

Price level

AD1 (1=$40) AD3 (1=$60) AD2 (1=$50)

0

Real GDP

Qf

Type: G Topic: 5 E: 276-277 MA: 276-277 81. Refer to the above diagrams. The numbers in parentheses after the AD1, AD2, and AD3 labels indicate the levels of investment spending associated with each curve. All figures are in billions. If the money supply is Ms1 and the goal of the monetary authorities is full-employment output Qf, they should: A) increase the money supply from $80 to $100. C) maintain the money supply at $80. B) increase the money supply from $80 to $120. D) decrease the money supply from $80 to $60. Answer: A

Type: G Topic: 5 E: 276-277 MA: 276-277 82. Refer to the above diagrams. The numbers in parentheses after the AD1, AD2, and AD3 labels indicate the levels of investment spending associated with each curve. All figures are in billions. If aggregate demand is AD3 and the monetary authorities desire to reduce it to AD2, they should: A) increase the interest rate from 3 percent to 9 percent. B) increase the money supply from $100 to $120. C) decrease the money supply from $120 to $100. D) decrease the interest rate from 3 percent to 9 percent. Answer: C

McConnell/Brue: Economics, 16/e

Page 471

Chapter 15: Monetary Policy

Type: G Topic: 5 E: 276-277 MA: 276-277 83. Refer to the above diagrams. The numbers in parentheses after the AD1, AD2, and AD3 labels indicate the levels of investment spending associated with each curve. All figures are in billions. Which of the following would shift the money supply curve from Ms1 to Ms3? A) an increase in the discount rate B) purchases of U.S. securities by the Fed in the open market C) sales of U.S. securities by the Fed in the open market D) an increase in the reserve ratio Answer: B

Type: G Topic: 5 E: 276-277 MA: 276-277 84. Refer to the above diagrams. The numbers in parentheses after the AD1, AD2, and AD3 labels indicate the levels of investment spending associated with each curve. All figures are in billions. If the MPC for the economy described by the figures is .8: A) an increase in the money supply from $80 to $100 will shift the aggregate demand curve rightward by $50 billion at each price level. B) an increase in the money supply from $80 to $100 will shift the aggregate demand curve leftward by $40 billion at each price level. C) a decrease in the interest rate from 9 percent to 6 percent will shift the aggregate demand curve leftward by $100 billion at each price level. D) a decrease in the interest rate from 6 percent to 3 percent will shift the aggregate demand curve leftward by $50 billion at each price level. Answer: A

Type: A Topic: 5 E: 279 MA: 279 85. An increase in the money supply will: A) lower interest rates and lower the equilibrium GDP. B) lower interest rates and increase the equilibrium GDP. C) increase interest rates and increase the equilibrium GDP. D) increase interest rates and lower the equilibrium GDP. Answer: B

Type: A Topic: 5 E: 279 MA: 279 86. All else equal, when the Federal Reserve Banks engage in a tight money policy, the prices of government bonds usually: A) fall. B) rise. C) remain constant. D) move in the same direction as the bonds' interest rate yield. Answer: A

Type: A Topic: 5 E: 279 MA: 279 87. All else equal, when the Federal Reserve Banks engage in an easy money policy, the interest rates received on government bonds usually: A) fall. B) rise. C) remain constant. D) move in the same direction as the bonds' price. Answer: A

McConnell/Brue: Economics, 16/e

Page 472

Chapter 15: Monetary Policy

Use the following to answer questions 88-91: Answer the next question(s) on the basis of the information in the following table.

Money supply $400 400 400 400 400

Money demand $600 500 400 300 200

Interest rate 2% 3 4 5 6

Investment (at interest rate shown) $700 600 500 300 200

Type: T Topic: 5 E: 276 MA: 276 88. Refer to the above table. The equilibrium interest rate in this economy is: A) 3 percent. B) 4 percent. C) 5 percent. D) 6 percent. Answer: B

Type: T Topic: 5 E: 277 MA: 277 89. Refer to the above table. An interest rate of 2 percent is not sustainable because: A) the demand for bonds in the bond market will fall and the interest rate will fall. B) the demand for bonds in the bond market will rise and the interest rate will fall. C) the supply of bonds in the bond market will decline and the interest rate will rise. D) the supply of bonds in the bond market will rise and the interest rate will rise. Answer: D

Type: T Topic: 5 E: 276 MA: 276 90. Refer to the above table. The amount of investment that will be forthcoming in this economy is: A) $700. B) $600. C) $500. D) $300. Answer: C

Type: T Topic: 5 E: 271-272 MA: 271-272 91. Refer to the above table. Suppose the legal reserve requirement is 10 percent and initially there are no excess reserves in the banking system. If the Fed wished to reduce the interest rate by 1 percentage point, it would: A) sell $10 of government bonds in the open market. B) buy $100 of government bonds in the open market. C) sell $100 of government bonds in the open market. D) buy $10 of government bonds in the open market. Answer: A

Type: A Topic: 5 E: 272 MA: 272 92. The price of government bonds and the interest rate received by a bond buyer are: A) positively related. B) unrelated. C) negatively related. D) independent of Federal Reserve open-market operations. Answer: C

McConnell/Brue: Economics, 16/e

Page 473

Chapter 15: Monetary Policy

Type: A Topic: 5 E: 275 MA: 275 93. A tight money policy is designed to shift the: A) aggregate demand curve rightward. B) aggregate demand curve leftward. Answer: B

C) aggregate supply curve rightward. D) aggregate supply curve leftward.

Type: A Topic: 5 E: 277 MA: 277 94. If the economy is operating in the relatively steep (upper) portion of its aggregate supply curve, a reduction in the money supply will: A) increase the interest rate and increase employment. B) reduce the interest rate and increase employment. C) increase the interest rate and reduce the price level, assuming it is flexible downward. D) reduce the interest rate and increase the price level. Answer: C

Type: A Topic: 5 E: 277 MA: 277 95. The sale of government bonds by the Federal Reserve Banks to commercial banks will: A) increase aggregate supply. C) increase aggregate demand. B) decrease aggregate supply. D) decrease aggregate demand. Answer: D

Type: C Topic: 5 E: 276-277 MA: 276-277 96. Assume that the price level is flexible both upward and downward and that the Fed's policy is to keep the price level from either rising or falling. If aggregate supply increases in the economy, the Fed: A) will have to increase interest rates to keep the price level from falling. B) will have to reduce the money supply to keep the price level from rising. C) will have to increase the money supply to keep the price level from falling. D) can keep the price level stable without altering the money supply or interest rate. Answer: C

Type: C Topic: 5 E: 278 MA: 278 97. If the demand for money increases and the Fed wants interest rates to remain unchanged, which of the following would be appropriate policy? A) recall Federal Reserve Notes from circulation C) buy bonds in the open market B) raise the legal reserve requirement D) raise the discount rate Answer: C

Assessment/recent policy

Type: F Topic: 6 E: 281 MA: 281 98. Which of the following has bolstered the case for active monetary policy? A) budget surpluses B) increasing globalization of financial markets C) the success of monetary policy in helping the economy emerge from the 1990-1991 recession and sustain economic growth through the 1990s D) a decreasing role of banks and thrifts in the financial industry Answer: C

McConnell/Brue: Economics, 16/e

Page 474

Chapter 15: Monetary Policy

Type: F Topic: 6 E: 281 MA: 281 99. According to studies and recent experience: A) globalization of financial markets has undermined the Fed's ability to change interest rates through monetary policy. B) the shrinking market share of banks and thrifts relative to other financial institutions has caused the Fed to lose control over the money supply and therefore its ability to affect interest rates. C) tight money policies work; easy money policies do not. D) the Fed has retained its ability to control the money supply and affect interest rates, even in the face of globalization of financial markets and the declining role of banks and thrifts in financial markets. Answer: D

Type: F Topic: 6 E: 281 MA: 281 100. One of the strengths of monetary policy relative to fiscal policy is that monetary policy: A) can be implemented more quickly. B) is subject to closer political scrutiny. C) does not produce a net export effect. D) entails a larger spending income multiplier effect on real GDP. Answer: A

Type: D Topic: 6 E: 281-282 MA: 281-282 101. The problem of cyclical asymmetry refers to the idea that: A) a tight money policy can force a contraction of the money supply, but an easy money policy may not achieve an expansion of the money supply. B) the monetary authorities have been less willing to use an easy money policy than they have a tight money policy. C) cyclical downswings are typically of longer duration than cyclical upswings. D) an easy money policy can force an expansion of the money supply, but a tight money policy may not achieve a contraction of the money supply. Answer: A

Type: A Topic: 6 E: 282 MA: 282 102. An easy money policy may be less effective than a tight money policy because: A) the Federal Reserve Banks are always willing to make loans to commercial banks which are short of reserves. B) fiscal policy always works at cross purposes with an easy money policy. C) changes in exchange rates complicate an easy money policy more than it does a tight money policy. D) commercial banks may not be able to find loan customers. Answer: D

Type: A Topic: 6 E: 282 MA: 282 103. An easy money policy may be frustrated if the: A) demand-for-money curve shifts to the left. B) investment-demand curve shifts to the left. Answer: B

C) saving schedule shifts downward. D) investment-demand curve shifts to the right.

McConnell/Brue: Economics, 16/e

Page 475

Chapter 15: Monetary Policy

Type: A Topic: 6 E: 281 MA: 281 104. Some economists contend that the velocity of money often changes in such a way as to frustrate monetary policy. Which of the following statements is consistent with this thinking? A) An increase in the money supply will increase the interest rate and reduce the amount of money held as an asset. B) A decrease in the money supply will decrease the interest rate and increase the amount of money held as an asset. C) A decrease in the money supply will increase the interest rate and reduce the amount of money held as an asset. D) An increase in the money supply will lower the interest rate and reduce the amount of money held as an asset. Answer: C

Type: A Topic: 6 E: 281 MA: 281 105. A tight money policy could be offset by: A) a deterioration in the profit expectations of businesses. B) a budget surplus. C) a decline in the velocity of money. D) an increase in the velocity of money. Answer: D

Type: A Topic: 6 E: 282 MA: 282 106. Monetary policy is thought to be: A) equally effective in moving the economy out of a depression as in controlling demand-pull inflation. B) more effective in moving the economy out of a depression than in controlling demand-pull inflation. C) more effective in controlling demand-pull inflation than in moving the economy out of a depression. D) only effective in moving the economy out of a depression. Answer: C

Type: A Topic: 6 E: 281 MA: 281 107. Monetary policy: A) is less politically acceptable than is fiscal policy. B) will be weakened if the velocity of money changes in the same direction as the money supply. C) will be weakened if the velocity of money changes in the opposite direction as the money supply. D) is designed primarily to alter the velocity of money. Answer: C

Type: A Topic: 6 E: 282 MA: 282 108. The impact of monetary policy on investment spending may be weakened: A) because of the Treasury's desire for high interest rates. B) if velocity changes in the same direction as the money supply. C) if the investment-demand curve shifts to the right during inflation and to the left during recession. D) if the investment-demand curve is very flat. Answer: C

McConnell/Brue: Economics, 16/e

Page 476

Chapter 15: Monetary Policy

Type: F Topic: 6 E: 282-283 MA: 282-283 109. Since 1980, U.S. monetary policy has been: A) highly erratic, causing rising inflation and unemployment. B) relatively successful in controlling inflation and promoting full employment. C) partly responsible for the increase in the natural rate of unemployment. D) of secondary importance to fiscal policy in stabilizing the economy. Answer: B

Type: A Topic: 6 E: 280 MA: 280 110. The Fed directly sets: A) the prime interest rate but not the Federal funds rate. B) both the Federal funds rate and the prime interest rate. C) neither the Federal funds rate nor the prime interest rate. D) the discount rate and the prime interest rate. Answer: C

Type: A Topic: 6 E: 280 MA: 280 111. Which of the following will likely accompany an easy money policy? A) a higher prime interest rate C) a higher discount rate B) a lower Federal funds rate D) higher income tax rates Answer: B

Type: A Topic: 6 E: 280 MA: 280 112. A Federal funds rate reduction that is caused by monetary policy will: A) increase the prime interest rate. C) increase the Fed's discount rate. B) decrease the size of the monetary multiplier. D) decrease the prime interest rate. Answer: D

Type: A Topic: 6 E: 281 MA: 281 113. To reduce the Federal funds rate, the Fed can: A) buy government bonds from the public. B) increase the discount rate. Answer: A

C) increase the prime interest rate. D) sell government bonds to commercial banks.

Type: A Topic: 6 E: 280 MA: 280 114. Generally, the prime interest rate: A) moves in the opposite direction as the Federal funds rate. B) remains constant over long periods of time. C) is highly inflexible downward. D) moves in the same direction as the Federal funds rate. Answer: D

Type: A Topic: 6 E: 280 MA: 280 115. To increase the Federal funds rate, the Fed can: A) buy government bonds from the public. B) decrease the discount rate. Answer: D

C) decrease the prime interest rate. D) sell government bonds to commercial banks.

McConnell/Brue: Economics, 16/e

Page 477

Chapter 15: Monetary Policy

Type: A Topic: 6 E: 280 MA: 280 116. Recently, the Fed has communicated changes in its monetary policy by announcing changes in its policy targets for the: A) growth of the money supply. C) prime interest rate. B) Federal funds rate. D) U.S. dollar-foreign currency exchange rate. Answer: B

Type: A Topic: 6 E: 280 MA: 280 117. The prime interest rate: A) affects investment spending while the Federal funds rate affects consumption spending. B) affects consumption spending while the Federal funds rate affects investment spending. C) has no affect on exchange rates and net exports. D) affects investment spending while the Federal funds rate affects overnight borrowing of bank reserves. Answer: D

Type: F Topic: 6 E: 280 MA: 280 118. The Federal funds rate is: A) higher than the prime interest rate. B) lower than the prime interest rate. C) always equal to the Fed's discount rate. D) equal to the prime interest rate minus the Fed's discount rate. Answer: B

Type: F Topic: 6 E: 282 MA: 282 119. In recent years, the Federal Reserve has: A) paid closer attention to M1 than M2 in setting monetary targets. B) relied more on changes in the discount rate than open-market operations in establishing monetary policy. C) has increased M2 at a fixed annual rate, regardless of the health of the economy. D) taken an activist, pragmatic approach to monetary policy, paying close attention to interest rates. Answer: D

Type: A Topic: 6 E: 280 MA: 280 120. If the Fed wants to lower the Federal funds rate, it should: A) increase the discount rate. C) buy government securities in the open market. B) increase the reserve ratio. D) sell government securities in the open market. Answer: C

Type: A Topic: 6 E: 280 MA: 280 121. Other things equal, which of the following would increase the Federal funds rate? A) a decrease in loan demand in the Federal funds market B) a decrease in the reserve ratio C) Fed purchases of government securities from banks D) a decline in excess reserves in the banking system Answer: D

McConnell/Brue: Economics, 16/e

Page 478

Chapter 15: Monetary Policy

Type: D Topic: 6 E: 280 MA: 280 Status: New 122. The benchmark interest rate that banks use as a reference point for a variety of consumer and business loans is the: A) Federal funds rate. B) prime interest rate. C) discount rate. D) Treasury bill rate. Answer: B

Type: A Topic: 6 E: 280 MA: 280 123. The prime interest rate usually: A) rises when the Federal funds rate rises. B) rises when the discount rate falls. C) falls when the Federal funds rate rises. D) falls when the Fed sells bonds in the open market. Answer: A

Type: F Topic: 6 E: 281 MA: 281 124. In 1999 and 2000 the Fed increased the Federal funds rate several times. The Fed's purpose was to: A) prevent rising inflation. C) stimulate economic growth. B) reduce the unemployment rate. D) strengthen the international value of the dollar. Answer: A

Type: F Topic: 6 E: 281 MA: 281 Status: New 125. Between January 2001 and June 2003, the Fed reduced the Federal funds rate from 6 percent to 1 percent. The Fed's purpose was to: A) prevent rising inflation. C) promote recovery from recession. B) reduce the public debt. D) strengthen the international value of the dollar. Answer: C

Type: A Topic: 6 E: 281 MA: 281 126. To reduce the Federal funds rate, the Fed would: A) increase the discount rate. B) increase the reserve requirement. Answer: D

C) sell government securities. D) buy government securities.

Type: A Topic: 6 E: 280 MA: 280 127. To increase the Federal funds rate, the Fed would: A) sell government securities. B) buy government securities. Answer: A

C) reduce the discount rate. D) decrease the reserve requirement.

Type: F Topic: 6 E: 286 MA: 286 128. In the last-half of the 1990s, Japan: A) successfully used monetary policy to overcome recession. B) had poor success using monetary policy to overcome recession. C) abandoned monetary policy in favor of fiscal policy in fighting inflation. D) successfully used monetary policy to reduce rapid inflation. Answer: B

McConnell/Brue: Economics, 16/e

Page 479

Chapter 15: Monetary Policy

Type: F Topic: 6 E: 286 MA: 286 129. Compared with fiscal policy, monetary policy is: A) quicker and easier to implement. B) slower and more cumbersome to implement. C) more dependent on Congressional action. D) more likely to produce an offsetting net export effect. Answer: A

Use the following to answer questions 130-132:

(1) Interest 4% 5 6 7 8

(2) Investment $100 90 80 70 60

(3) Investment $80 70 60 50 40

Type: T Topic: 6 E: 276 MA: 276 130. Refer to the above table, in which investment is in billions. Suppose the Fed reduces the interest rate from 6 percent to 5 percent. Given columns (1) and (2), investment will: A) decline by $20 billion. C) decline by $10 billion. B) increase by $20 billion. D) increase by $10 billion. Answer: D

Type: T Topic: 6 E: 276 MA: 276 131. Refer to the above table, in which investment is in billions. Suppose the Fed reduces the interest rate from 6 to 5 percent at a time when the investment demand declines from that shown by columns (1) and (2) to that shown by columns (1) and (3). As a result of these two occurrences, investment will: A) increase by $10 billion. C) increase by $20 billion. B) decrease by $10 billion. D) decrease by $20 billion. Answer: B

Type: T Topic: 6 E: 280 MA: 280 132. The result demonstrated in the previous question illustrates: A) the crowding out problem. C) a potential weakness of monetary policy. B) the net export effect. D) a potential strength of monetary policy. Answer: C

McConnell/Brue: Economics, 16/e

Page 480

Chapter 15: Monetary Policy

Type: D Topic: 6 E: 282 MA: 282 Status: New 133. Inflation targeting consists of the Fed: A) using monetary policy to reduce the annual inflation rate by a set amount each year until the rate of inflation is zero. B) using monetary policy to hold the price of a fixed basket of commodities (wheat, gold, pork, and so on) to a 1 to 2 percent annual increase. C) identifying the sources of inflation and recommending structural changes in the economy that would relieve upward price pressures. D) regularly stating an explicit goal for the rate of inflation over some future period, such as the following two years. Answer: D

Type: F Topic: 6 E: 283 MA: 283 Status: New 134. Proponents of inflation targeting say it would: A) remove the need for countercyclical fiscal policy. B) increase the transparency of monetary policy and increase Fed accountability. C) do away with the need for the Fed to engage in open market operations. D) help coordinate fiscal and monetary policy. Answer: B

Type: F Topic: 6 E: 283 MA: 283 Status: New 135. Proponents of inflation targeting say it would: A) remove the need for countercyclical fiscal policy. B) increase the transparency of monetary policy and increase Fed accountability. C) do away with the need for the Fed to engage in open market operations. D) help coordinate fiscal and monetary policy. Answer: B

International complications

Type: C Topic: 7 E: 283 MA: 283 136. Other things equal, a tight money policy will: A) decrease the international value of the dollar. B) increase GDP. Answer: C

C) reduce net exports. D) increase net exports.

McConnell/Brue: Economics, 16/e

Page 481

Chapter 15: Monetary Policy

Use the following to answer questions 137-138:

AS

Price level

AD3 AD2 AD1

0

Real GDP

Type: G Topic: 7 E: 277 MA: 277 137. Refer to the above diagram. The shift of the aggregate demand curve from AD1 to AD2 might result from the Fed: A) selling bonds in the open market. C) increasing the reserve ratio. B) increasing the discount rate. D) buying bonds in the open market. Answer: D

Type: C Topic: 7 E: 277 MA: 277 138. Refer to the above diagram. Which of the following would explain why the policy described in the previous question might shift the aggregate demand curve to AD3 rather than to AD2? A) the domestic interest rate rises, the dollar depreciates, and net exports fall B) the domestic interest rate rises, the dollar appreciates, and net exports fall C) the domestic interest rate falls, the dollar depreciates, and net exports rise D) the domestic interest rate falls, the dollar appreciates, and net exports rise Answer: C

Type: A Topic: 7 E: 283, 286 MA: 283, 286 139. Other things equal, an easy money policy will: A) reduce net exports. B) increase interest rates. Answer: C

C) reduce the international value of the dollar. D) reduce GDP.

Type: C Topic: 7 E: 283, 286 MA: 283, 286 140. A particular change in money supply will produce a smaller net export effect: A) the steeper is the money demand curve. C) the steeper is the money supply curve. B) the flatter is the money demand curve. D) the flatter is the money supply curve. Answer: A

McConnell/Brue: Economics, 16/e

Page 482

Chapter 15: Monetary Policy

Type: A Topic: 7 E: 283, 286 MA: 283, 286 141. Under some conditions, proper domestic monetary policy may be at odds with the goal of correcting a trade imbalance because: A) changes in the domestic interest rate cause changes in domestic investment spending. B) changes in the domestic interest rate tend to cause changes in the international value of the dollar. C) the domestic interest rate varies inversely with the value of the dollar. D) changes in the interest rate cause changes in domestic saving. Answer: B

Type: A Topic: 7 E: 283, 286 MA: 283, 286 142. The net export effect: A) strengthens the stimulative effect of an expansionary fiscal policy. B) weakens the stimulative effect of an easy money policy. C) strengthens the stimulative effect of an easy money policy. D) has no perceptible impact on stabilization policies. Answer: C

Type: A Topic: 7 E: 283, 286 MA: 283, 286 143. An easy money policy in the United States is most likely to: A) decrease the foreign demand for dollars and appreciate the international value of the dollar. B) decrease the foreign demand for dollars and depreciate the international value of the dollar. C) increase the foreign demand for dollars and appreciate the international value of the dollar. D) increase the foreign demand for dollars and depreciate the international value of the dollar. Answer: B

Type: A Topic: 7 E: 283, 286 MA: 283, 286 144. A tight money policy in the United States is most likely to: A) depreciate the international value of the dollar and increase American net exports. B) depreciate the international value of the dollar and decrease American net exports. C) appreciate the international value of the dollar and increase American net exports. D) appreciate the international value of the dollar and decrease American net exports. Answer: D

Type: A Topic: 7 E: 286 MA: 286 145. International flows of financial capital in response to interest rate changes in the United States: A) weaken domestic monetary policy through an offsetting net export effect. B) strengthen domestic monetary policy through a supporting net export effect. C) strengthen domestic fiscal policy through an offsetting net export effect. D) weaken domestic monetary policy through an offsetting real wealth effect. Answer: B

Type: A Topic: 7 E: 286 MA: 286 146. All else equal, an easy money policy in the United States: A) increases U.S. imports. C) reduces the foreign demand for U.S. dollars. B) increases the international value of the dollar. D) aggravates an existing U.S. trade deficit. Answer: C

McConnell/Brue: Economics, 16/e

Page 483

Chapter 15: Monetary Policy

Type: A Topic: 7 E: 286 MA: 286 147. An easy money policy is appropriate for the alleviation of domestic: A) unemployment and compatible with the goal of correcting a trade deficit. B) unemployment and compatible with the goal of correcting a trade surplus. C) inflation and compatible with the goal of correcting a trade deficit. D) inflation and compatible with the goal of correcting a trade surplus. Answer: A

Type: A Topic: 7 E: 286 MA: 286 148. Assume the United States is experiencing a 6 percent annual rate of inflation and is also incurring a trade deficit. All else equal, the use of appropriate monetary policy to reduce inflation would: A) cause the dollar to depreciate in value. C) decrease the U.s. trade deficit. B) have no impact on the U.S. trade deficit. D) increase the U.S. trade deficit. Answer: D

Type: A Topic: 7 E: 286 MA: 286 149. Suppose the United States is experiencing an 8 percent rate of unemployment with stable prices and a trade deficit. All else equal, the use of appropriate monetary policy to reduce unemployment would: A) cause the dollar to appreciate in value. C) decrease the U.S. trade deficit. B) have no impact on the U.S. trade deficit. D) increase the U.S. trade deficit. Answer: C

Type: A Topic: 7 E: 286 MA: 286 150. Which of the following is correct? A) An easy money policy will cause the dollar to depreciate and will increase U.S. net exports. B) An easy money policy will cause the dollar to depreciate and will decrease U.S. net exports. C) An easy money policy will cause the dollar to appreciate and will increase U.S. net exports. D) An easy money policy will cause the dollar to appreciate and will decrease U.S. net exports. Answer: A

Type: A Topic: 7 E: 286 MA: 286 151. Which of the following is correct? A) A tight money policy will cause the dollar to appreciate and U.S. net exports to increase. B) A tight money policy will cause the dollar to appreciate and U.S. net exports to decrease. C) A tight money policy will cause the dollar to depreciate and U.S. net exports to increase. D) A tight money policy will cause the dollar to depreciate and U.S. net exports to decrease. Answer: B

AD-AS/policy summary

Type: A Topic: 8 E: 284-285 MA: 284-285 152. Other things equal, an increase in productivity will: A) reduce aggregate supply and increase real output. B) reduce both the interest rate and the international value of the dollar. C) increase both aggregate supply and real output. D) increase net exports, increase investment, and reduce aggregate demand. Answer: C

McConnell/Brue: Economics, 16/e

Page 484

Chapter 15: Monetary Policy

Type: A Topic: 8 E: 284-285 MA: 284-285 153. Other things equal, an increase in input prices will: A) reduce aggregate supply and reduce real output. B) increase the interest rate and lower the international value of the dollar. C) increase aggregate supply and increase the price level. D) increase net exports, increase investment, and reduce aggregate demand. Answer: A

Type: A Topic: 8 E: 284-285 MA: 284-285 154. Other things equal, a tight money policy during a period of demand-pull inflation will: A) lower the interest rate, increase investment, and reduce net exports. B) lower the price level, increase investment, and increase aggregate demand. C) increase productivity, aggregate supply, and real output. D) increase the interest rate, reduce investment, and reduce aggregate demand. Answer: D

Type: A Topic: 8 E: 284-285 MA: 284-285 155. Other things equal, a reduction in income taxes would: A) reduce productivity and reduce aggregate supply. B) increase consumption and increase aggregate demand. C) increase the supply of money and reduce investment. D) increase government spending and increase aggregate demand. Answer: B

Type: A Topic: 8 E: 284-285 MA: 284-285 156. Other things equal, a depreciation of the U.S. dollar would: A) increase the price of imported resources and decrease aggregate supply. B) decrease net exports and aggregate demand. C) increase consumption, investment, net export, and government spending. D) decrease aggregate supply and decrease aggregate demand. Answer: A

Consider This Questions

Type: F E: 282 MA: 282 Status: New 157. (Consider This) The possible asymmetry of monetary policy is the central idea of the: A) invisible hand concept. B) ratchet analogy. C) pushing-on-a-string analogy. D) bandwagon effect. Answer: C

Type: F E: 282 MA: 282 Status: New 158. (Consider This) The pushing-on-a-string analogy makes the point that, monetary policy may be better at: A) controlling demand-pull inflation than cost-push inflation. B) pulling the aggregate demand curve leftward than pushing it rightward . C) pulling the unemployment rate downward than pushing the economic growth rate upward. D) keeping rapid inflation from occurring than reducing it once it has begun. Answer: B

McConnell/Brue: Economics, 16/e

Page 485

Chapter 15: Monetary Policy

Last Word Questions

159.

Type: F E: 287 MA: 287 (Last Word) The current chair of the Board of Governors of the Federal Reserve System is: A) Lawrence Summers. B) John B. Taylor. C) Alan Greenspan. D) Regis Philbin. Answer: C

Type: A E: 287 MA: 287 160. (Last Word) Which of the following metaphors best fits the Fed's role in using monetary policy to stabilize the economy? A) Fed as a warrior B) Fed as a mechanic C) Fed as a fall guy D) Fed as a cosmic force Answer: B

Type: A E: 287 MA: 287 161. (Last Word) Which of the following metaphors best fits the tendency of the administration and congressional representatives to blame the Fed for the economy's difficulties? A) Fed as a warrior B) Fed as a mechanic C) Fed as a fall guy D) Fed as a cosmic force Answer: C

Type: A E: 287 MA: 287 162. (Last Word) Which of the following metaphors best fits the Fed's goal of maintaining price stability? A) Fed as a warrior B) Fed as a mechanic C) Fed as a fall guy D) Fed as a cosmic force Answer: A

True/False Questions

Type: A E: 279 MA: 279 163. The Fed reduces interest rates mainly by selling government securities. Answer: False

Type: A E: 279 MA: 279 164. The Fed increases interest rates mainly by selling government securities. Answer: True

Type: A E: 277 MA: 277 165. If the economy is operating in the relative flat (lower) part of its aggregate supply curve, a tight money policy will increase real output but not the price level. Answer: False

Type: F E: 281-282 MA: 281-282 166. In the last half of the 1990s monetary policy was highly effective in the United States but highly ineffective in Japan. Answer: True

McConnell/Brue: Economics, 16/e

Page 486

Chapter 15: Monetary Policy

Type: F E: 282 MA: 282 167. Alan Greenspan is the current chair of the Council of Economic Advisers. Answer: False

Type: F E: 273-284 MA: 273-284 168. A change in the reserve ratio will affect both the amount of the banking system's excess reserves and the multiple by which the system can lend on the basis of excess reserves. Answer: True

Type: A E: 280 MA: 280 169. The prime interest rate and the Federal funds rate normally change in opposite directions. Answer: False

Type: D E: 269 MA: 269 170. The largest single liability of the Federal Reserve Banks is their outstanding loans to commercial banks. Answer: False

Type: F E: 275 MA: 275 171. An easy money policy is one that reduces the supply of money. Answer: False

Type: A E: 278 MA: 278 172. Changes in the interest rate are more likely to affect investment spending than consumer spending. Answer: True

Type: A E: 279 MA: 279 173. The job of the Fed in limiting the supply of money may be made more complex if commercial banks initially have substantial excess reserves. Answer: True

Type: A E: 278 MA: 278 174. Other things equal, an easy money policy will shift the economy's aggregate demand curve to the right. Answer: True

Type: A E: 279 MA: 279 175. A tight money policy may be frustrated if the investment-demand curve shifts to the left. Answer: False

Type: A E: 279 MA: 279 176. A tight money policy reduces investment spending and shifts the economy's aggregate demand curve to the right. Answer: False

McConnell/Brue: Economics, 16/e

Page 487

CHAPTER 16

Extending the Analysis of Aggregate Supply

Topic 1. 2. 3. 4. 5. Short-run and long-run aggregate supply Extended AD-AS model Phillips Curve Long-run Phillips Curve Taxation and aggregate supply Consider This Last Word True-False

Question numbers 1-31 32-49 50-68 69-95 96-106 107-108 109-111 112-125

____________________________________________________________

_______________________________________

____________________________________________________________

_______________________________________

Multiple Choice Questions Short-run and long-run aggregate supply

Type: D Topic: 1 E: 292 MA: 292 1. In terms of aggregate supply, a period in which nominal wages and other resource prices are unresponsive to price-level changes is called the: A) long run. B) short run. C) immediate market period. D) very long run. Answer: B

Type: D Topic: 1 E: 294 MA: 294 2. In terms of aggregate supply, a period in which nominal wages and other resource prices are fully responsive to price-level changes is called the: A) long run. B) short run. C) immediate market period. D) very long run. Answer: A

Type: D Topic: 1 E: 293 MA: 293 3. In the extended analysis of aggregate supply, the short-run aggregate supply curve is: A) vertical and the long-run aggregate supply curve is horizontal. B) horizontal and the long-run aggregate supply curve is vertical. C) upward sloping and the long-run aggregate supply curve is vertical. D) horizontal and the long-run aggregate supply curve is upward sloping. Answer: C

Chapter 16: Extending the Analysis of Aggregate Supply

Type: D Topic: 1 E: 294 MA: 294 4. In the extended analysis of aggregate supply, the long-run aggregate supply curve is: A) vertical and the short-run aggregate supply curve is horizontal. B) horizontal and the short-run aggregate supply curve is vertical. C) horizontal and the short-run aggregate supply curve is upward sloping. D) vertical and the short-run aggregate supply curve is upward sloping. Answer: D

Type: D Topic: 1 E: 292 MA: 292 5. In terms of aggregate supply, the short run is a period in which: A) the price level is constant. B) employment is constant. C) real output is constant. D) nominal wages and other resource prices are unresponsive to price-level changes. Answer: D

Type: D Topic: 1 E: 294 MA: 294 6. In terms of aggregate supply, the difference between the long run and the short run is that in the long run: A) the price level is variable. B) employment is variable. C) real output is variable. D) nominal wages and other input prices are fully responsive to price-level changes. Answer: D

Type: D Topic: 1 E: 294 MA: 294 7. The long-run aggregate supply curve is vertical: A) because the rate of inflation is steady in the long run. B) because resource prices eventually rise and fall with product prices. C) because product prices always increase at a faster rate than resource prices. D) only when the money supply increases at the same rate as real GDP. Answer: B

Type: D Topic: 1 E: 293 MA: 293 8. The short-run aggregate supply curve is upsloping because: A) of the interest rate effect. B) higher price levels create incentives to expand output when resource prices are unresponsive to pricelevel changes. C) of the net export effect. D) higher price levels create an expectation among producers of still higher price levels. Answer: B

Type: A Topic: 1 E: 293 MA: 293 9. Other things equal, an increase in the price level will: A) shift the aggregate supply curve to the right. B) shift the aggregate demand curve to the right. C) cause a movement up along a short-run aggregate supply curve. D) cause a movement down an aggregate demand curve. Answer: C

McConnell/Brue: Economics, 16/e

Page 490

Chapter 16: Extending the Analysis of Aggregate Supply

Type: A Topic: 1 E: 293 MA: 293 10. Other things equal, a decrease in the price level will: A) shift the aggregate supply curve to the left. B) shift the aggregate demand curve to the left. C) cause a movement up a short-run aggregate supply curve. D) cause a movement down an aggregate supply curve. Answer: D

Use the following to answer questions 11-14: Suppose the natural level of real output (Q) for a hypothetical economy is $500, the price level (P) initially is 100, and that prices and wages are flexible both upward and downward. Use the following short-run aggregate supply schedules to answer the next question(s).
(a) AS(P100) P Q 125 $560 100 500 75 440 (b) AS(P125) P Q 125 $500 100 440 75 380 (c) AS(P75) P Q 125 $620 100 560 75 500

Type: T Topic: 1 E: 295 MA: 295 11. Refer to the information above. If the price level unexpectedly increases from 100 to 125, the level of real output in the short run will: A) rise from $500 to $560. C) fall from $560 to $500. B) fall from $500 to $440. D) rise from $440 to $500. Answer: A

Type: T Topic: 1 E: 295 MA: 295 12. Refer to the information above. In the long run, an increase in the price level from 100 to 125 will: A) increase real output from $500 to $560. B) decrease real output from $500 to $440. C) change the aggregate supply schedule from (a) to (c) and result in an equilibrium level of real output of $560. D) change the aggregate supply schedule from (a) to (b) and result in an equilibrium level of real output of $500. Answer: D

Type: T Topic: 1 E: 295 MA: 295 13. Refer to the information above. If the price level unexpectedly declines from 100 to 75, the level of real output in the short run will: A) rise from $500 to $560. C) fall from $560 to $500. B) fall from $500 to $440. D) rise from $440 to $500. Answer: B

McConnell/Brue: Economics, 16/e

Page 491

Chapter 16: Extending the Analysis of Aggregate Supply

Type: T Topic: 1 E: 294-295 MA: 294-295 14. Refer to the information above. In the long run, a fall in the price level from 100 to 75 will: A) decrease real output from $500 to $440. B) increase real output from $500 to $620. C) change the aggregate supply schedule from (a) to (c) and produce an equilibrium level of real output of $500. D) change the aggregate supply schedule from (a) to (b) and produce an equilibrium level of real output of $500. Answer: C

Type: D Topic: 1 E: 294-295 MA: 294-295 15. The: A) short-run aggregate supply curve is downward sloping. B) short-run aggregate supply curve is vertical. C) long-run aggregate supply curve is vertical. D) long-run aggregate supply curve is upsloping. Answer: C

Use the following to answer questions 16-21:

Type: G Topic: 1 E: 293 MA: 293 16. Refer to the above diagram. Assume that nominal wages initially are set on the basis of the price level P2 and that the economy initially is operating at its full-employment level of output Qf. In the short run, an increase in the price level from P2 to P3 will: A) change aggregate supply from AS2 to AS3. C) change aggregate supply from AS2 to AS1. B) increase real output from Q1 to Q2. D) increase real output from Qf to Q2. Answer: D

McConnell/Brue: Economics, 16/e

Page 492

Chapter 16: Extending the Analysis of Aggregate Supply

Type: G Topic: 1 E: 293 MA: 293 17. Refer to the above diagram. Assume that nominal wages initially are set on the basis of the price level P2 and that the economy initially is operating at its full-employment level of output Qf. In the long run, an increase in the price level from P2 to P3 will: A) increase real output from Qf to Q2. C) decrease real output from Q2 to Q1. B) change aggregate supply from AS2 to AS1. D) move the economy from b to d. Answer: D

Type: G Topic: 1 E: 294 MA: 294 18. Refer to the above diagram. Assume that nominal wages initially are set on the basis of the price level P2 and that the economy initially is operating at its full-employment level of output Qf. In terms of this diagram, the long-run aggregate supply curve: A) is AS2. B) is a vertical line extending from Qf upward through e, b, and d . C) may be either AS1, AS2, or AS3 depending on whether the price level is P1, P2, or P3. D) is a horizontal line extending from P2 rightward through f , b, and g. Answer: B

Type: G Topic: 1 E: 293 MA: 293 19. Refer to the above diagram. Assume that nominal wages initially are set on the basis of the price level P2 and that the economy initially is operating at its full-employment level of output Qf. In the short run, demand-pull inflation could best be shown as: A) a move from b to c on AS2. C) a change of aggregate supply from AS2 to AS3. B) a move from b to c to d. D) a move from b to d. Answer: A

Type: G Topic: 1 E: 294 MA: 294 20. Refer to the above diagram. Assume that nominal wages initially are set on the basis of the price level P2 and that the economy initially is operating at its full-employment level of output Qf. In the long run, demand-pull inflation could best be shown as: A) a move from b to c on AS2. C) a change of aggregate supply from AS2 to AS1. B) a move from b to f to d. D) a move from b to d. Answer: D

Type: G Topic: 1 E: 293 MA: 293 21. Refer to the above diagram. Assume that nominal wages initially are set on the basis of the price level P2 and that the economy initially is operating at its full-employment level of output Qf. In the short run, costpush inflation could best be shown as: A) a leftward shift of aggregate supply from AS2 to AS3. B) a move from b to c on AS2. C) a move from b to c to d. D) a move from b to f to d. Answer: A

McConnell/Brue: Economics, 16/e

Page 493

Chapter 16: Extending the Analysis of Aggregate Supply

Type: A Topic: 1 E: 293 MA: 293 22. Other things equal, the short-run aggregate supply curve shifts positions when: A) the price level changes. C) nominal wages and other input prices change. B) the rate of inflation changes. D) aggregate demand changes. Answer: C

Use the following to answer questions 23-26:
A B

Price level

P1

C

D
0 Q1 Real output

Type: G Topic: 1 E: 294 MA: 294 23. Refer to the above diagram relating to short-run and long-run aggregate supply. The A) short-run aggregate supply curve is A. C) long-run aggregate supply curve is B. B) short-run aggregate supply curve is B. D) long-run aggregate supply curve is D. Answer: B

Type: G Topic: 1 E: 294 MA: 294 24. Refer to the above diagram. If the price level rises above P1 because of an increase in aggregate demand, the: A) economy will move up along curve B and output will temporarily increase. B) long-run aggregate supply curve C will shift upward. C) short-run aggregate supply curve B will automatically shift to the right. D) economy's output first will decline, then increase, and finally return to Q1. Answer: A

Type: G Topic: 1 E: 294 MA: 294 25. Refer to the above diagram. The long-run aggregate supply curve is: A) A. B) B. C) C. D) D. Answer: A

Type: G Topic: 1 E: 294 MA: 294 26. Refer to the above diagram. The short-run aggregate supply is: A) A. B) B. C) D. D) not represented in the diagram. Answer: B

McConnell/Brue: Economics, 16/e

Page 494

Chapter 16: Extending the Analysis of Aggregate Supply

Use the following to answer questions 27-31:
AS3 P3
Price level

u t v w

x

AS2 AS1 z P2 P1

y

0

Q1

Qf

Q2

Real output

Type: G Topic: 1 E: 293 MA: 293 Status: New 27. Refer to the above diagram and assume the economy is operating at equilibrium point w. In the short run, an increase in the price level from P2 to P3 would move the economy from point w to point: A) v. B) x. C) u. D) v. Answer: B

Type: G Topic: 1 E: 294 MA: 294 Status: New 28. Refer to the above diagram and assume the economy is operating at equilibrium point w. In the long run, an increase in the price level from P2 to P3 would move the economy from point w to point: A) v. B) x. C) u. D) y. Answer: C

Type: G Topic: 1 E: 293 MA: 293 Status: New 29. Refer to the above diagram and assume the economy is operating at equilibrium point w. In the short run, a decrease in the price level from P2 to P1 would move the economy from point w to point: A) v. B) x. C) t. D) y. Answer: A

Type: G Topic: 1 E: 294 MA: 294 Status: New 30. Refer to the above diagram and assume the economy is operating at equilibrium point w. If wages and other resource prices are flexible downward, in the long run a decrease in the price level from P2 to P1 would move the economy from point w to point: A) v. B) x. C) t. D) y. Answer: D

Type: G Topic: 1 E: 294 MA: 294 Status: New 31. Refer to the above diagram. If drawn, the long-run aggregate supply curve would include points: A) v, w, and u. B) w, u and y. C) t, w, and z. D) y, w, and x. Answer: B

McConnell/Brue: Economics, 16/e

Page 495

Chapter 16: Extending the Analysis of Aggregate Supply

Extended AD-AS model

Type: D Topic: 2 E: 294 MA: 294 32. The extended AD-AS model: A) distinguishes between short-run and long-run aggregate demand. B) explains inflation but not recession. C) includes G and Xn whereas the simple AD-AD model does not. D) distinguishes between short-run and long-run aggregate supply. Answer: D

Type: A Topic: 2 E: 294 MA: 294 33. In the extended aggregate demand-aggregate supply model: A) long-run equilibrium occurs wherever the aggregate demand curve intersects the short-run aggregate supply curve. B) the long-run aggregate supply curve is horizontal. C) the price level is the same regardless of the location of the aggregate demand curve. D) long-run equilibrium occurs at the intersection of the aggregate demand curve, the short-run aggregate supply curve, and the long run aggregate supply. Answer: D

Type: A Topic: 2 E: 294 MA: 294 34. In the extended aggregate demand-aggregate supply model: A) long-run equilibrium occurs wherever the aggregate demand curve intersects the short-run aggregate supply curve. B) the long-run aggregate supply curve is horizontal. C) the level of real output is the same in the long run regardless of the location of the aggregate demand curve. D) the short-run aggregate supply curve is downsloping. Answer: C

McConnell/Brue: Economics, 16/e

Page 496

Chapter 16: Extending the Analysis of Aggregate Supply

Use the following to answer questions 35-40:

Type: G Topic: 2 E: 295 MA: 295 35. Refer to the above diagram. The initial aggregate demand curve is AD1 and the initial aggregate supply curve is AS1. Demand-pull inflation in the short run is best shown as: A) a shift of the aggregate demand curve from AD1 to AD2. B) a move from d to b to a. C) a move directly from d to a. D) a shift of the aggregate supply curve from AS1 to AS2. Answer: A

Type: G Topic: 2 E: 295 MA: 295 36. Refer to the above diagram. The initial aggregate demand curve is AD1 and the initial aggregate supply curve is AS1. In the long run, demand-pull inflation is best shown as: A) a shift of aggregate demand from AD1 to AD2 followed by a shift of aggregate supply from AS1 to AS2. B) a move from d to b to a. C) a shift of aggregate supply from AS1 to AS2 followed by a shift of aggregate demand from AD1 to AD2. D) a move from a to d. Answer: A

Type: G Topic: 2 E: 295 MA: 295 37. Refer to the above diagram. The initial aggregate demand curve is AD1 and the initial aggregate supply curve is AS1. In the long run, the aggregate supply curve is vertical in the diagram because: A) nominal wages and other input prices are assumed to be fixed. B) real output level Qf is the potential level of output. C) price level increases produce perfectly offsetting changes in nominal wages and other input prices. D) higher than expected rates of actual inflation reduce real output only temporarily. Answer: C

McConnell/Brue: Economics, 16/e

Page 497

Chapter 16: Extending the Analysis of Aggregate Supply

Type: G Topic: 2 E: 295 MA: 295 38. Refer to the above diagram. The initial aggregate demand curve is AD1 and the initial aggregate supply curve is AS1. Cost-push inflation in the short run is best represented as a: A) leftward shift of the aggregate supply curve from AS1 to AS2. B) rightward shift of the aggregate demand curve from AD1 to AD2. C) move from d to b to a. D) move from d directly to a. Answer: A

Type: G Topic: 2 E: 295 MA: 295 39. Refer to the above diagram. The initial aggregate demand curve is AD1 and the initial aggregate supply curve is AS1. Assuming no change in aggregate demand, the long-run response to a recession caused by cost-push inflation is best depicted as a: A) move from a to d along the long-run aggregate supply curve. B) rightward shift of the aggregate supply curve from AS2 to AS1. C) move from a to c to d. D) leftward shift of the aggregate supply curve from AS1 to AS2. Answer: B

Type: G Topic: 2 E: 295 MA: 295 40. Refer to the above diagram. The initial aggregate demand curve is AD1 and the initial aggregate supply curve is AS1. If government offsets the decline in real output resulting from short-run cost-push inflation by increasing aggregate demand from AD1 to AD2: A) real output will rise above Qf. B) the price level will rise from P1 to P2. C) it is possible that aggregate supply will shift rightward from AS2 because nominal wage demands will rise. D) the price level will rise from P2 to P 3. Answer: D

Type: A Topic: 2 E: 296 MA: 296 41. If government uses fiscal policy to restrain cost-push inflation, we can expect: A) the unemployment rate to rise. B) the unemployment rate to fall. C) the aggregate demand curve to shift rightward. D) tax-rate declines and increases in government spending. Answer: A

Type: A Topic: 2 E: 296 MA: 296 42. One policy dilemma posed by cost-push inflation is that: A) an increase in aggregate demand will increase inflation and the unemployment rate simultaneously. B) tax rates can be reduced without lowering tax revenues. C) the reduction of aggregate demand to restrain inflation will cause a further reduction in the real GDP. D) the adjustment of aggregate demand can neither increase real GDP nor reduce inflation. Answer: C

McConnell/Brue: Economics, 16/e

Page 498

Chapter 16: Extending the Analysis of Aggregate Supply

Type: A Topic: 2 E: 296 MA: 296 43. If government uses its stabilization policies to maintain full employment under conditions of cost-push inflation: A) a deflationary spiral is likely to occur. C) stagflation is likely to occur. B) an inflationary spiral is likely to occur. D) the Phillips Curve is likely to shift inward. Answer: B

Use the following to answer questions 44-49:

Type: G Topic: 2 E: 295 MA: 295 44. Refer to the above diagram and assume that prices and wages are flexible both upward and downward in the economy. In the extended AD-AS model: A) demand-pull inflation would involve a rightward shift of curve A, followed by a leftward shift of curve C. B) cost-push inflation would involve a rightward shift of curve A, followed by a leftward shift of curve C. C) recession would involve a leftward shift of curve A followed by a leftward shift of curve C. D) recession would involve a rightward shift of curve D, followed by leftward shifts of curves A and C. Answer: A

Type: G Topic: 2 E: 296 MA: 296 45. Refer to the above diagram and assume that prices and wages are flexible both upward and downward in the economy. In the extended AD-AS model: A) demand-pull inflation would involve a rightward shift of curve A, followed by a rightward shift of curve C. B) cost-push inflation would involve first a leftward shift of curve C, then a rightward shift of curve C. C) recession would involve a leftward shift of curve A followed by a leftward shift of curve C. D) recession would involve a rightward shift of curve D, followed by leftward shifts of curves A and C. Answer: B

McConnell/Brue: Economics, 16/e

Page 499

Chapter 16: Extending the Analysis of Aggregate Supply

Type: G Topic: 2 E: 296-297 MA: 296-297 46. Refer to the above diagram and assume that prices and wages are flexible both upward and downward in the economy. In the extended AD-AS model: A) demand-pull inflation would involve a rightward shift of curve A, followed by a rightward shift of curve C. B) cost-push inflation would involve a rightward shift of curve A, followed by a leftward shift of curve C. C) recession would involve a leftward shift of curve A followed by a rightward shift of curve C. D) recession would involve a rightward shift of curve D, followed by leftward shifts of curves A and C. Answer: C

Type: G Topic: 2 E: 296-297 MA: 296-297 47. Refer to the above diagram and assume that prices and wages are flexible both upward and downward in the economy. In the extended AD-AS model: A) demand-pull inflation would involve a rightward shft of curve A, followed by a rightward shift of curve C. B) cost-pust inflation would involve a rightward shift of curve A, followed by a leftward shift of curve C. C) recession would involve a leftward shift of curve A followed by a leftward shift of curves C and D. D) recession could be caused by either a leftward shift of curve A or a leftward shift of curve C. Answer: D

Type: G Topic: 2 E: 296-297 MA: 296-297 48. Refer to the above diagram and assume that prices and wages are flexible both upward and downward in the economy. In the extended AD-AS model: A) demand-pull inflation would involve a shift of curve D to the right. B) cost-push inflation would involve a shift of curve B downward. C) recession would involve a leftward shift of curve A. D) frictional unemployment would be zero in the long run. Answer: C

Type: G Topic: 2 E: 296-297 MA: 296-297 49. Refer to the above diagram. Assume both upward and downward price and wage flexibility in the economy. In the extended AD-AS model: A) demand-pull inflation would involve a rightward shift of curve A, followed by a rightward shift of curve C. B) cost-push inflation would involve a leftward shift of curve C, followed by an upward shift of curve B. C) recession would involve a leftward shift of curve A. D) a rightward shift of curve D would be equivalent to an outward shift of the nation's production possibilities curve. Answer: D

Phillips Curve

Type: A Topic: 3 E: 297 MA: 297 50. The traditional Phillips Curve suggests a tradeoff between: A) price level stability and income equality. B) the level of unemployment and price level stability. C) unemployment and income equality. D) economic growth and full employment. Answer: B

McConnell/Brue: Economics, 16/e

Page 500

Chapter 16: Extending the Analysis of Aggregate Supply

Type: A Topic: 3 E: 297 MA: 297 51. The basic problem portrayed by the traditional Phillips Curve is: A) that a level of aggregate demand sufficiently high to result in full employment may also cause inflation. B) that changes in the composition of total labor demand tend to be deflationary. C) that unemployment rises at the same time the general price level is rising. D) the possibility that automation will increase the level of noncyclical unemployment. Answer: A

Type: A Topic: 3 E: 298 MA: 298 52. The traditional Phillips Curve suggests that, if government uses an expansionary fiscal policy to stimulate output and employment: A) unemployment may actually increase because of the crowding-out effect. B) tax revenues may increase even though tax rates have been reduced. C) inflation may result. D) the natural rate of unemployment may fall. Answer: C

Use the following to answer questions 53-58:

Type: G Topic: 3 E: 298 MA: 298 53. Refer to the above diagram for a specific economy. The curve on this graph is known as a: A) Laffer Curve. B) Phillips Curve. C) labor demand curve. D) production possibilities curve. Answer: B

Type: G Topic: 3 E: 298 MA: 298 54. Refer to the above diagram for a specific economy. Which of the following best describes the relationship shown by this curve? A) The demand for labor is large when the rate of inflation is small. B) When the rate of unemployment is high, the rate of inflation is high. C) The rate of inflation and the rate of unemployment are inversely related. D) The rate of inflation and the rate of unemployment are directly related. Answer: C

McConnell/Brue: Economics, 16/e

Page 501

Chapter 16: Extending the Analysis of Aggregate Supply

Type: G Topic: 3 E: 299 MA: 299 55. Refer to the above diagram for a specific economy. A reduction in structural unemployment or bottleneck problems in labor markets will: A) shift this curve to the right. C) move this economy southeast along the curve. B) shift this curve to the left. D) move this economy northwest along the curve. Answer: B

Type: A Topic: 3 E: 299 MA: 299 56. Refer to the above diagram for a specific economy. An increase in aggregate demand will: A) shift this curve to the right. C) move this economy southeast along the curve. B) shift this curve to the left. D) move this economy northwest along the curve. Answer: D

Type: G Topic: 3 E: 299 MA: 299 57. Refer to the above diagram for a specific economy. Which of the following best describes a decision by policymakers that moves this economy from point b to point a? A) Policymakers have instituted an easy money policy and/or a budgetary deficit, thereby accepting more unemployment to reduce the rate of inflation. B) Policymakers have instituted a tight money policy and/or a budgetary surplus, thereby accepting a higher rate of inflation to reduce unemployment. C) Policymakers have instituted an easy money and/or a budgetary deficit, thereby accepting a higher rate of inflation to reduce unemployment. D) Policymakers have instituted a tight money policy and/or a budgetary surplus, thereby accepting more unemployment to reduce the rate of inflation. Answer: C

Type: G Topic: 3 E: 298 MA: 298 58. Refer to the above diagram for a specific economy. The shape of this curve suggests that: A) the price level rises at a diminishing rate as the level of aggregate demand increases. B) full employment and price stability are compatible goals only when aggregate demand is falling. C) each successive unit of decline in the unemployment rate is accompanied by a smaller increase in the rate of inflation. D) each successive unit of decline in the unemployment rate is accompanied by a larger increase in the rate of inflation. Answer: D

Type: D Topic: 3 E: 299 MA: 299 59. Stagflation refers to: A) an increase in inflation accompanied by decreases in real output and employment. B) a decline in the price level accompanied by increases in real output and employment. C) a simultaneous increase in real output and the price level. D) a simultaneous reduction in real output and the price level. Answer: A

Type: D Topic: 3 E: 299 MA: 299 60. Inflation accompanied by falling real output and employment is known as: A) Laffer's law. B) Okun's law. C) stagflation. D) the Phillips Curve. Answer: C

McConnell/Brue: Economics, 16/e

Page 502

Chapter 16: Extending the Analysis of Aggregate Supply

Type: F Topic: 3 E: 299 MA: 299 61. Which of the following allegedly contributed to the stagflation in the mid-1970s? A) appreciation of the dollar C) a dramatic increase in oil prices B) a sharp drop in the prices of farm products D) rising productivity in manufacturing. Answer: C

Type: F Topic: 3 E: 300 MA: 300 62. Statistical data for the 1970s and 1980s suggest that: A) the Phillips Curve was stable. B) the Phillips Curve was unstable. C) low levels of unemployment were consistently associated with high rates of inflation. D) the inflation rate was highly stable. Answer: B

Type: A Topic: 3 E: 298 MA: 298 63. A rightward shift of the traditional Phillips Curve would suggest that: A) the productivity of labor increased. B) the rate of inflation is now higher at each rate of unemployment. C) cost-push inflation decreased. D) the rate of inflation is now lower at each rate of unemployment. Answer: B

Type: C Topic: 3 E: 299-300 MA: 299-300 64. A major adverse aggregate supply shock: A) automatically shifts the aggregate demand curve rightward. B) causes the Phillips Curve to shift rightward and upward. C) can be caused by rising productivity. D) can be caused by falling wages. Answer: B

Type: A Topic: 3 E: 299-300 MA: 299-300 65. Rightward and upward shifts of the Phillips Curve in the 1970s and early 1980s were caused by: A) adverse shocks to aggregate supply. C) an increase in the misery index. B) adverse shocks to aggregate demand. D) the Vietnam War. Answer: A

Type: A Topic: 3 E: 299 MA: 299 66. An adverse aggregate supply shock could result from: A) a sharp rise in productivity. C) a decline in wages. B) a rapid rise in oil prices. D) an appreciation of the dollar. Answer: B

Type: A Topic: 3 E: 299 MA: 299 67. An adverse aggregate supply shock: A) automatically shifts the aggregate demand curve rightward. B) causes the Phillips Curve to shift leftward and downward. C) can be caused by a boost in the rate of growth of productivity. D) can cause stagflation. Answer: D

McConnell/Brue: Economics, 16/e

Page 503

Chapter 16: Extending the Analysis of Aggregate Supply

Type: F Topic: 3 E: 300 MA: 300 68. The last few years of the 1990s in the United States were characterized by: A) low inflation and high unemployment. C) low inflation and low unemployment. B) stagflation. D) a high misery index. Answer: C

Long-run Phillips Curve

Type: A Topic: 4 E: 300 MA: 300 69. Which of the following is a true statement? A) Under normal conditions there is a short-run tradeoff between inflation and unemployment. B) There is a long-run tradeoff between inflation and unemployment. C) The short-run Phillips Curve is vertical. D) The long-run Phillips Curve is horizontal. Answer: A

Type: A Topic: 4 E: 299 MA: 299 70. Which of the following is a true statement? A) There is a long-run tradeoff between inflation and unemployment. B) The short-run Phillips Curve is vertical. C) The long-run Phillips Curve is horizontal. D) Adverse aggregate supply shocks can simultaneously worsen unemployment and inflation. Answer: D

Type: A Topic: 4 E: 300 MA: 300 71. Which of the following is a true statement? A) There is a long-run tradeoff between inflation and unemployment. B) There is no tradeoff between inflation and unemployment in the long run. C) The short-run Phillips Curve is horizontal. D) The long-run Phillips Curve is horizontal. Answer: B

Type: A Topic: 4 E: 299 MA: 299 72. Which of the following is a true statement? A) The short-run Phillips Curve is horizontal. B) The long-run Phillips Curve is horizontal. C) There is a long-run tradeoff between inflation and unemployment. D) The short-run Phillips Curve is downward sloping. Answer: D

Type: A Topic: 4 E: 301 MA: 301 73. Which of the following is a true statement? A) There is a long-run tradeoff between inflation and unemployment. B) There is no tradeoff between inflation and unemployment in the short-run. C) The short-run Phillips Curve is horizontal. D) The long-run Phillips Curve is vertical. Answer: D

McConnell/Brue: Economics, 16/e

Page 504

Chapter 16: Extending the Analysis of Aggregate Supply

Type: F Topic: 4 E: 300 MA: 300 74. In the last half of the 1990s, the usual short-run tradeoff between inflation and unemployment did not arise because: A) the Fed held interest rates constant. B) the Federal government balanced its budget. C) the U.S. personal savings rate rose. D) productivity (and thus aggregate supply) grew faster than previously. Answer: D

Type: C Topic: 4 E: 300 MA: 300 75. Suppose that the CPI for a particular economy rose from 110 to 120 in year 1, 120 to 130 in year 2, and 130 to 140 in year 3. We could conclude that this economy is experiencing: A) accelerating inflation. B) deflation. C) disinflation. D) a constant rate of inflation. Answer: C

Type: D Topic: 4 E: 302 MA: 302 76. Disinflation occurs when: A) the price level is falling. B) investment plans exceed saving. Answer: D

C) a speculative investment "bubble" is bursting. D) the inflation rate is declining.

Type: D Topic: 4 E: 302 MA: 302 77. As distinct from reductions in the price level, reductions in the rate of inflation are referred to as: A) dollar depreciation. B) stagflation. C) deflation. D) disinflation. Answer: D

Type: A Topic: 4 E: 302 MA: 302 78. When the actual rate of inflation is less than the expected rate: A) the unemployment rate will temporarily rise. B) firms will increase their output to recoup their falling profits. C) the unemployment rate will temporarily fall. D) firms will experience rising profits and thus increase their employment. Answer: A

Type: A Topic: 4 E: 302 MA: 302 79. When the actual rate of inflation exceeds the expected rate: A) the unemployment rate will temporarily rise. B) firms will experience rising profits and thus increase their employment. C) the actual rate of inflation will fall. D) nominal wages will decline. Answer: B

McConnell/Brue: Economics, 16/e

Page 505

Chapter 16: Extending the Analysis of Aggregate Supply

Use the following to answer questions 80-85:

Annual rate of inflation (%)

9

c3

b4 b3

6

c2 c1

3

b2

b1 7 3 5 6 2 4 Unemployment rate (%)
Type: G Topic: 4 E: 301 MA: 301 80. The above diagram is the basis for explaining: A) the traditional Phillips Curve. B) the long-run Phillips Curve. C) how central planning can make full employment and price level stability compatible goals. D) new policies for eliminating unemployment. Answer: B

Type: G Topic: 4 E: 301 MA: 301 81. The natural rate of unemployment for this economy is: A) 3 percent. B) 5 percent. C) 6 percent. D) 4 percent. Answer: B

Type: G Topic: 4 E: 301 MA: 301 82. Refer to the above diagram. Assume the economy is initially at point b1. With a time lag between price and nominal wage adjustments, an increase in aggregate demand will temporarily move the economy from: A) b2 to b1. B) c1 to b2. C) b1 to c1. D) b1 to b2. Answer: C

Type: G Topic: 4 E: 301 MA: 301 83. Refer to the above diagram and assume the economy is initially at point b1. Which of the following movements is consistent with the traditional Phillips Curve? A) the movement from b1 to b2 C) the movement from c1 to b2 B) the movement from b1 to c1 D) the movement from b2 to b1 Answer: B

Type: G Topic: 4 E: 301 MA: 301 84. Refer to the above diagram and assume the economy is initially at point b1. Point c1 represents: A) a stable position because reality and expectations are consistent. B) a stable position because full employment and a constant annual inflation rate are represented. C) an unstable situation because government will undertake contractionary policies D) an unstable situation because nominal wage rates will increase. Answer: D

McConnell/Brue: Economics, 16/e

Page 506

Chapter 16: Extending the Analysis of Aggregate Supply

Type: G Topic: 4 E: 301 MA: 301 85. Refer to the above diagram and assume the economy is initially at point b1. The long-run relationship between the unemployment rate and the rate of inflation is represented by: A) the line connecting b1 and c1. C) the line connecting c1 and b2. B) the line through b1, b2, b3 , and b4. D) any line parallel to the horizontal axis. Answer: B

Type: C Topic: 4 E: 301 MA: 301 86. Government can push the unemployment rate below the natural rate only by: A) instituting supply-side economic policies. B) producing a higher rate of inflation than people expect. C) balancing the federal budget. D) achieving zero inflation. Answer: B

Type: A Topic: 4 E: 301 MA: 301 87. In the long run: A) attempts to "fine tune" the economy cause the rate of unemployment to accelerate. B) there is no long-run inflation-unemployment tradeoff. C) there is an inflation-unemployment tradeoff and the terms of that tradeoff have worsened in recent years. D) there is an inflation-unemployment tradeoff, but the terms of that tradeoff have improved in recent years. Answer: B

McConnell/Brue: Economics, 16/e

Page 507

Chapter 16: Extending the Analysis of Aggregate Supply

Use the following to answer questions 88-91:

PCLR

Inflation rate (%)

6

d

a

4

c PC1

b

PC2

0

4 5 7 Unemployment rate (%)

Type: G Topic: 4 E: 301 MA: 301 88. Refer to the above diagram. Assume that the natural rate of unemployment is 5 percent and that the economy is initially operating at point a where the expected and actual rates of inflation are each 6 percent. If the actual rate of inflation unexpectedly falls from 6 percent to 4 percent, then the unemployment rate will: A) temporarily fall from 5 percent to 4 percent. C) temporarily rise from 5 percent to 7 percent. B) permanently fall from 5 percent to 4 percent. D) permanently rise from 5 percent to 7 percent. Answer: C

Type: G Topic: 4 E: 301 MA: 301 89. Refer to the above diagram. Assume that the natural rate of unemployment is 5 percent and that the economy is initially operating at point a where the expected and actual rates of inflation are each 6 percent. In the long run, the decline in the actual rate of inflation from 6 percent to 4 percent will: A) reduce the unemployment rate. C) have no effect on the unemployment rate. B) reduce corporate profits in real terms. D) reduce real domestic output. Answer: C

Type: G Topic: 4 E: 301 MA: 301 90. Refer to the above diagram. Assume that the natural rate of unemployment is 5 percent and that the economy is initially operating at point c where the expected and actual rates of inflation are each 4 percent. If the actual rate of inflation unexpectedly rises from 4 percent to 6 percent, the economy will: A) move from a to b and eventually to c. C) remain at a. B) move directly from c to b. D) move from c to d and eventually to a. Answer: D

Type: G Topic: 4 E: 301 MA: 301 91. In the above diagram: A) any rate of inflation is consistent with the natural rate of unemployment in the long run. B) inflation can occur but disinflation cannot occur. C) unemployment rates exceeding the natural rate are permanent. D) unemployment rates less than the natural rate are permanent. Answer: A

McConnell/Brue: Economics, 16/e

Page 508

Chapter 16: Extending the Analysis of Aggregate Supply

Use the following to answer questions 92-95:

Type: G Topic: 4 E: 301 MA: 301 92. Refer to the above diagram. Point b on short-run Phillips Curve PC1 represents a rate of: A) inflation below the natural rate. C) unemployment above the natural rate. B) inflation above the natural rate. D) unemployment below the natural rate. Answer: D

Type: G Topic: 4 E: 301 MA: 301 93. Refer to the above diagram. Point b would be explained by: A) an actual rate of inflation that exceeds the expected rate. B) an actual rate of inflation that is less than the expected rate. C) cost-push inflation. D) an increase in long-run aggregate supply. Answer: A

Type: G Topic: 4 E: 301 MA: 301 94. Refer to the above diagram. Point b would not be permanent because the: A) economy would move from b to a on PC1. B) short-run Phillips Curve would shift from PC1 to PC2 and unemployment would increase to the natural rate at c. C) economy would immediately move from b to c to d. D) economy would move from b directly to d. Answer: B

Type: G Topic: 5 E: 301 MA: 301 95. Refer to the above diagram. The move of the economy from c to e on short-run Phillips Curve PC2 would be explained by an: A) increase in aggregate demand in the economy. B) increase in aggregate supply in the economy. C) actual rate of inflation that is less than the expected rate. D) actual rate of inflation that exceeds the expected rate. Answer: C

McConnell/Brue: Economics, 16/e

Page 509

Chapter 16: Extending the Analysis of Aggregate Supply

Taxation and aggregate supply

Type: D Topic: 5 E: 302 MA: 302 96. Which of the following is a tenet of supply-side economics? A) High marginal tax rates severely discourage work, saving, and investment. B) Increases in social security taxes and other business taxes shift the aggregate supply curve to the right. C) The Federal Reserve should adhere to a monetary rule that limits increases in the money supply to a 5 percent annual rate. D) Transfer payments increase incentives to work. Answer: A

Type: F Topic: 5 E: 303 MA: 303 97. The Laffer Curve is a central concept in: A) monetarism. B) Keynesianism. C) welfare economics. Answer: D

D) supply-side economics.

Use the following to answer questions 98-101:

Type: G Topic: 5 E: 303 MA: 303 98. The above curve is known as the: A) Taylor rule. B) Okun Curve. C) Laffer Curve. Answer: C

D) Phillips Curve.

Type: G Topic: 5 E: 303 MA: 303 99. Refer to the above diagram. Supply-side economists believe that tax rates are: A) such that an increase in tax rates will increase tax revenues. B) at some level below b. C) at some level above b. D) at d. Answer: C

Type: G Topic: 5 E: 303 MA: 303 Status: New 100. In the above curve, a decline in the tax rate from c to b would: A) greatly increase tax revenue. C) leave tax revenue about the same as before. B) greatly decrease tax revenue. D) shift the curve to the left. Answer: A

McConnell/Brue: Economics, 16/e

Page 510

Chapter 16: Extending the Analysis of Aggregate Supply

Type: G Topic: 5 E: 303 MA: 303 Status: New 101. If the current tax rate is currently c and the government wants to maximize tax revenue, it should: A) leave the tax rate at c. C) reduce the tax rate to b. B) increase the tax rate to d. D) reduce the tax rate to a. Answer: C

Type: A Topic: 5 E: 303 MA: 303 102. Supply-side economist Arthur Laffer has argued that: A) there is no empirically proven relationship between tax rates and incentives. B) large reductions in personal and corporate income taxes will increase aggregate supply much more than aggregate demand. C) the only way to eliminate inflation is to increase taxes to induce a recession severe enough to eliminate inflationary expectations. D) large cuts in income taxes will increase aggregate demand more than aggregate supply. Answer: B

Type: A Topic: 5 E: 303 MA: 303 103. A basic criticism of supply-side economics is that: A) empirical research clearly shows that incentives to work and invest vary directly with marginal tax rates. B) lower taxes will increase aggregate supply much more than they will increase aggregate demand. C) lower taxes will increase aggregate demand much more than they will increase aggregate supply. D) higher taxes will reduce incentives to work, invest, and innovate. Answer: C

Type: A Topic: 5 E: 304 MA: 304 104. Critics of supply-side economics: A) argue that a tax cut will increase aggregate supply by more than it increases aggregate demand. B) contend that the relationship between tax rates and economic incentives is small and of uncertain direction. C) believe that a decline in tax rates will increase tax revenues. D) point out that tax cuts enable households to "buy more leisure" by working less. Answer: B

Use the following to answer questions 105-106:

Average Tax Rate 20% 40 60 80

Tax Revenue (billions of dollars) $250 300 250 200

Type: D Topic: 5 E: 304 MA: 304 105. If graphed, the relationship shown above would depict this economy's: A) Laffer Curve. B) Lorenz Curve. C) Tax Freedom Curve. D) Phillips Curve. Answer: A

McConnell/Brue: Economics, 16/e

Page 511

Chapter 16: Extending the Analysis of Aggregate Supply

Type: F Topic: 5 E: 304-305 MA: 304-305 106. In 1993 the Federal government boosted income tax rates. In the seven years that followed: A) tax revenues fell slightly. C) the unemployment rate increased. B) productivity growth slowed. D) tax revenues expanded rapidly. Answer: D

Consider This Questions

Type: F E: 304 MA: 304 Status: New 107. (Consider This) The ideas of economist Arthur Laffer became the centerpiece for tax policy during the: A) Ford administration. C) Nixon administration. B) Clinton administration. D) Reagan administration. Answer: D

Type: F E: 304 MA: 304 Status: New 108. (Consider This) Economist Arthur Laffer equated Robin Hood to: A) government and equated the people passing through Sherwood Forest to taxpayers. B) charitable organizations and equated the people passing through Sherwood Forest to poor people. C) businesses and equated the people passing through Sherwood Forest to consumers. D) government and equated the people passing through Sherwood Forest to importers of goods and services. Answer: A

Last Word Questions

Type: F E: 305 MA: 305 109. (Last Word) Which of the following is a reason why changes in the price of imported oil have less of an effect on the U. S. economy than in the 1970s and early 1980s? A) The United States is now more reliant on domestic oil and less reliant on imported oil. B) The amount of energy consumed in producing each dollar of GDP has greatly declined. C) The United States has vastly expanded its hydroelectric capacity (dams and reservoirs). D) The United States has greatly expanded its passenger train services. Answer: B

Type: F E: 305 MA: 305 110. (Last Word) In recent years: A) significant changes in the price of oil have had much less effect on the U. S. economy than did similar changes in oil prices in previous decades. B) large increases in the price of oil have reduced U. S. aggregate supply and caused cost-push inflation. C) large decreases in the price of oil have increased U. S. aggregate supply and caused deflation. D) the United States has become a net exporter of oil. Answer: A

McConnell/Brue: Economics, 16/e

Page 512

Chapter 16: Extending the Analysis of Aggregate Supply

Type: F E: 305 MA: 305 111. (Last Word) Relative to previous decades, the U.S. economy is less affected by changes in the price of oil partly because: A) the composition of GDP has changed from larger, heavier items such as earth movers and steel products toward smaller, lighter items such as software and microchips. B) The United States is now more reliant on domestic oil and less reliant on imported oil. C) The United States has vastly expanded its hydroelectric capacity (dams and reservoirs). D) The ratio of passenger cars to passenger trucks has increased. Answer: A

True/False Questions

Type: A E: 293 MA: 293 112. The short-run aggregate supply curve is vertical and the long-run aggregate supply curve is horizontal. Answer: False

Type: A E: 293 MA: 293 113. The short-run aggregate supply curve shifts to the left when nominal wages rise in response to price level increases. Answer: True

Type: A E: 295 MA: 295 114. The extended AD-AS model distinguishes between short-run and long-run aggregate supply Answer: True

Type: A E: 295 MA: 295 115. In the extended AD-AS model, the long-run aggregate supply curve is vertical. Answer: True

Use the following to answer questions 116-119: Answer the next question(s) on the basis of the following economic data for a hypothetical economy:

Year 1997 1998 1999 2000

Average hourly Index of Price Rate of wage industrial Unemployment level increase in rates production rate index productivity $6.40 197 5.5% 130 3.0% 6.72 199 5.8 133 2.9 7.24 196 7.2 139 3.1 8.02 192 8.3 147 2.8

Type: A E: 295 MA: 295 116. The above data indicate that the economy has entered a period of demand-pull inflation. Answer: False

McConnell/Brue: Economics, 16/e

Page 513

Chapter 16: Extending the Analysis of Aggregate Supply

Type: T E: 295 MA: 295 117. Refer to the above data. It would be appropriate stabilization policy to raise interest rates, raise taxes, and reduce government expenditures. Answer: False

Type: T E: 296 MA: 296 118. Refer to the above data. There is evidence that cost-push inflationary pressure is present in this economy. Answer: True

Type: A E: 299 MA: 299 119. Refer to the above data. This economy has encountered stagflation. Answer: True

Type: A E: 294-295 MA: 294-295 120. Demand-pull inflation and cost-push inflation are identical concepts because both involve lower unemployment rates and rising prices. Answer: False

Type: A E: 297-298 MA: 297-298 121. The Phillips Curve suggests an inverse relationship between increases in the price level and the level of employment. Answer: False

Type: A E: 298 MA: 298 122. A shift in the Phillips Curve to the left will improve the inflation-unemployment choices available to society. Answer: True

Type: A E: 298 MA: 298 123. A rightward and upward shift of the Phillips Curve is consistent with the occurrence of stagflation. Answer: True

Type: A E: 303 MA: 303 124. The Laffer Curve shows the tradeoff between the price level and tax rates. Answer: False

Type: A E: 303 MA: 303 125. The Laffer Curve underlies the contention that lower tax rates need not reduce tax revenues. Answer: True

McConnell/Brue: Economics, 16/e

Page 514

CHAPTER 17

Economic Growth

Topic 1. 2. 3. 4. Ingredients and graphical analysis Growth record; growth accounting The New Economy Growth debate Consider This Last Word True-False

Question numbers 1-35 36-48 49-65 66-69 70-71 72-74 75-88

____________________________________________________________

_______________________________________

____________________________________________________________

_______________________________________

Multiple Choice Questions Ingredients and graphical analysis

Use the following to answer questions 1-5: Use the following list to answer the following questions: 1. 2. 3. 4. 5. 6. Improvements in technology Increases in the supply (stock) of capital goods Purchases of rising output. Obtaining the optimal combination of goods, each at least-cost production Increases in the quantity and quality of natural resources Increases in the quantity and quality of human resources

Type: A Topic: 1 E: 308-309 MA: 308-309 1. Refer to the above list. As distinct from the demand and efficiency factors of economic growth, the supply factors of economic growth are: A) 2, 5, and 6 only. B) 2, 4, 5, and 6 only. C) 1, 2, 5, and 6 only. D) 1, 3, 4 only. Answer: C

Type: A Topic: 1 E: 308 MA: 308 2. Refer to the above list. As distinct from the supply factors and efficiency factor of economic growth, the demand factor of economic growth is: A) 1 only. B) 4 only. C) 1 and 3 only. D) 3 only. Answer: D

Chapter 17: Economic Growth

Type: A Topic: 1 E: 309 MA: 309 3. Refer to the above list. As distinct from the supply factors and demand factor of economic growth, the efficiency factor of economic growth is: A) 1 only. B) 4 only. C) 1 and 3 only. D) 3 only. Answer: B

Type: A Topic: 1 E: 309 MA: 309 4. Which set of items in the above list would shift an economy's production possibilities curve outward? A) 2, 5, and 6 only. B) 2, 4, 5, and 6 only. C) 1, 2, 5, and 6 only. D) 1, 3, 4 only. Answer: C

Type: A Topic: 1 E: 309 MA: 309 5. Which set of items in the above list would shift an economy's long-run aggregate supply curve to the right? A) 1, 2, 5, and 6 only. B) 2, 5, and 6 only. C) 2, 4, 5, and 6 only. D) 1, 3, 4 only. Answer: A

Type: A Topic: 1 E: 309 MA: 309 6. Which of the following is not a supply factor in economic growth? A) the stock of capital C) the size and quality of the labor force B) technological advance D) aggregate expenditures Answer: D

Type: A Topic: 1 E: 309 MA: 309 7. The achievement of full employment through time will: A) diminish labor productivity. B) reduce the level of investment as a percentage of GDP. C) increase the realized rate of economic growth. D) have no impact on the rate of economic growth. Answer: C

Type: A Topic: 1 E: 309 MA: 309 8. Economic growth can be portrayed as a: A) outward shift of the production possibilities curve. B) inward shift of the production possibilities curve. C) movement from a point on to a point inside a production possibilities curve. D) movement from one point to another point on a fixed production possibilities curve. Answer: A

Type: C Topic: 1 E: 310 MA: 310 9. Suppose that an economy's labor productivity and total worker-hours each grew by 3 percent between year 1 and year 2. We could conclude that this economy's: A) real GDP remained constant. B) capital stock increased by 3 percent. C) production possibilities curve shifted inward. D) long-run aggregate supply curve shifted to the right. Answer: D

McConnell/Brue: Economics, 16/e

Page 516

Chapter 17: Economic Growth

Type: C Topic: 1 E: 310 MA: 310 10. Suppose that an economy's labor productivity and total worker-hours each grew by 4 percent between year 1 and year 2. We could conclude that this economy's: A) long-run aggregate supply curve shifted to the left. B) real GDP remained constant. C) production possibilities curve shifted outward. D) capital stock increased by 4 percent. Answer: C

Type: C Topic: 1 E: 310 MA: 310 11. Suppose that an economy's labor productivity fell by 3 percent and its total worker-hours remained constant between year 1 and year 2. We could conclude that this economy's: A) real GDP declined B) capital stock increased. C) production possibilities curve shifted outward. D) long-run aggregate supply curve shifted to the right. Answer: A

Type: C Topic: 1 E: 310 MA: 310 12. Suppose that an economy's labor productivity rose by 3 percent and its total worker-hours remained constant between year 1 and year 2. We could conclude that this economy's: A) capital stock increased. B) real GDP increased. C) production possibilities curve shifted inward. D) long-run aggregate supply curve shifted to the left. Answer: B

Use the following to answer questions 13-16:

X

Y

C
Capital goods Price level

A

0

D

0

Q2

Consumer goods

Real GDP

Type: G Topic: 1 E: 310 MA: 310 13. Curve AB is a: A) production possibilities curve and curve X is a long-run aggregate supply curve. B) consumer demand curve and curve X is a long-run aggregate supply curve. C) long-run aggregate supply curve and Y is potential real GDP curve. D) long-run aggregate supply curve and X is a production possibilities curve. Answer: A

McConnell/Brue: Economics, 16/e

Page 517

Chapter 17: Economic Growth

Type: G Topic: 1 E: 310 MA: 310 14. Refer to the above graphs. Growth of production capacity is shown by: A) the shift from AB to CD only. B) the shift from X to Y only. C) both the shift from AB to CD and the shift from X to Y. D) both the shift from AB to CD and the shift from Y to X. Answer: C

Type: G Topic: 1 E: 311 MA: 311 15. Refer to the above graphs. An increase in an economy's labor productivity would shift curve: A) AB to CD and shift curve Y to X. C) AB to CD and shift curve X to Y. B) CD to AB and shift curve X to Y. D) X to Y while leaving curve AB in place. Answer: C

Type: G Topic: 1 E: 310 MA: 310 16. Refer to the above graphs. An increase in the economy's human capital would shift curve: A) AB to CD and curve Y to X. C) X to Y while leaving curve AB in place. B) CD to AB and curve X to Y. D) AB to CD and curve X to Y. Answer: D

Use the following to answer questions 17-18:

C

A

Z Y
X

Capital goods

B 0 Consumer goods

D

Type: G Topic: 1 E: 310-311 MA: 310-311 17. Refer to the above diagram. Realized economic growth is best represented by a: A) shift in the production possibilities curve from AB to CD. B) move from X on AB to Y on CD. C) shift in the production possibilities curve from CD to AB. D) move from X to Z along AB. Answer: B

McConnell/Brue: Economics, 16/e

Page 518

Chapter 17: Economic Growth

Type: G Topic: 1 E: 311 MA: 311 18. Refer to the above diagram. The most likely cause of a shift from AB to CD would be a(n): A) increase in productivity. C) decrease in the size of the labor force. B) increase in the price level. D) recession. Answer: A

Type: A Topic: 1 E: 311 MA: 311 19. An outward shift of a nation's production possibilities curve: A) ensures the nation of an increase in real GDP per capita. B) ensures the nation of an increase in real GDP, but not of real GDP per capita. C) neither ensures a nation of an increase in real GDP nor of an increase in real GDP per capita. D) corresponds to a leftward shift a nation's long-run aggregate supply curve. Answer: C

Type: D Topic: 1 E: 313 MA: 313 20. Labor productivity is measured by: A) the ratio of capital to labor. B) real output per worker hour. Answer: B

C) real output per capita. D) the ratio of worker hours to real GDP.

Type: A Topic: 1 E: 313 MA: 313 21. Which of the following would not be expected to increase labor productivity? A) technological advance B) the acquisition of more education and training by the labor force C) an increase in the size of the labor force D) the realization of economies of scale Answer: C

Type: D Topic: 1 E: 313 MA: 313 22. Labor productivity is defined as: A) total output/worker-hours. B) nominal GDP minus real GDP. C) the ratio of real capital to worker-hours. D) the annual increase in nominal GDP per worker. Answer: A

Type: E Topic: 1 E: 313 MA: 313 23. Which of the following is correct? A) total output = labor productivity/worker-hours B) labor productivity = worker-hours/total output C) total output = worker-hours × labor productivity D) worker-hours = labor productivity × total output Answer: C

Type: E Topic: 1 E: 313 MA: 313 24. If the number of worker-hours in an economy is 100 and its labor productivity is 5 units of output per worker-hour, the economy's real GDP: A) is $20. B) is $500. C) is $5000. D) cannot be calculated. Answer: B

McConnell/Brue: Economics, 16/e

Page 519

Chapter 17: Economic Growth

Type: E Topic: 1 E: 313 MA: 313 25. Suppose total output (real GDP) is $4000 and labor productivity is 8. We can conclude that: A) real GDP per capita must be $500. C) nominal GDP must be $500. B) the price-level index must be greater than 100. D) the number of worker-hours must be 500. Answer: D

Type: E Topic: 1 E: 313 MA: 313 26. Suppose total output (real GDP) is $10,000 and worker-hours are 20,000. We can conclude that: A) real GDP per capita must be $200,000. B) the price-level index must be less than 100. C) labor productivity must be 0.5. D) nominal GDP must be between $10,000 and $20,000. Answer: C

Type: D Topic: 1 E: 313 MA: 313 Status: New 27. The percentage of the working-age population in the labor force (= employed + officially unemployed) is called the: A) labor force participation rate. C) work-activity rate. B) employment-population ratio. D) work-nonwork ratio. Answer: A

Type: A Topic: 1 E: 313 MA: 313 Status: New 28. Other things equal, which of the following would decrease the rate of economic growth, as measured by changes in real GDP? A) An increase in the educational attainment of the labor force B) A permanent decrease in frictional unemployment C) An increase in the amount of capital per worker D) A decrease in the labor force participation rate. Answer: D

Type: A Topic: 1 E: 312 MA: 312 Status: New 29. Other things equal, which of the following would increase the rate of economic growth, as measured by changes in real GDP? A) A decline in the average length of the work week. B) A decrease in the labor force participation rate. C) An increase in the size of the working age population. D) A decline in the amount of capital per worker. Answer: C

McConnell/Brue: Economics, 16/e

Page 520

Chapter 17: Economic Growth

Use the following to answer questions 30-31:

Type: G Topic: 1 E: 313-314 MA: 313-314 30. Refer to the above diagram. The shifts in long-run and short-run aggregate supply curves from AS1 and AS'1 to AS2 and AS'2 would most likely result from: A) an increase in the price level. C) an improvement in technology. B) a reduction in aggregate demand. D) deterioration of the infrastructure. Answer: C

Type: G Topic: 1 E: 313-314 MA: 313-314 31. Refer to the above diagram. Suppose that in a specific year the long-run and short-run aggregate supply curves shift from AS1 and AS'1 to AS2 and AS'2. If the aggregate demand curve also shifts rightward from AD1 to AD2, the rates of economic growth and inflation for the year will be: A) 4 percent each. C) 8 percent and 3 percent, respectively. B) 6 percent and 5 percent, respectively. D) 5 percent and 8 percent, respectively. Answer: D

Type: A Topic: 1 E: 311 MA: 311 32. In the aggregate demand-aggregate supply model, economic growth is represented by a: A) leftward shift of the long-run aggregate supply curve. B) leftward shift of the aggregate demand curve. C) rightward shift of the long-run aggregate supply curve. D) rightward shift of the short-run aggregate supply curve resulting from a decline in the price level. Answer: C

Type: C Topic: 1 E: 311 MA: 311 33. A rightward shift of a nation's long-run aggregate supply curve is equivalent to: A) a rightward shift of the nation's aggregate demand curve. B) a downward shift of the nation's aggregate expenditures curve. C) a rightward shift of the nation's investment demand curve. D) an outward shift of the nation's production possibilities curve. Answer: D

McConnell/Brue: Economics, 16/e

Page 521

Chapter 17: Economic Growth

Type: C Topic: 1 E: 311 MA: 311 34. An outward shift of an nation's production possibilities curve is equivalent to a: A) rightward shift of the nation's aggregate demand curve. B) downward shift of the nation's aggregate expenditure curve. C) rightward shift of the nation's long-run aggregate supply curve. D) rightward shift of the nation's investment demand curve. Answer: C

Type: A Topic: 1 E: 311 MA: 311 35. Which of the following, other things equal, would shift a nation's long-run aggregate supply curve to the right? A) a decrease in the total hours of work C) an increase in the labor force participation rate B) a decline in productivity D) an increase in net exports Answer: C

Growth record; growth accounting Type: F Topic: 2 E: 312 MA: 312 36. Which of the following statements is correct? A) Between 1950 and 2000, U.S. real GDP grew at about 2.3 percent per year and real GDP per capita grew at about 3.5 percent per year. B) Between 1950 and 2000, U.S. real GDP grew at about 3.5 percent per year and real GDP per capita grew at about 2.3 percent per year. C) Between 1950 and 2000, U.S. real GDP and real GDP per capita both grew at approximately 4 percent per year. D) Between 1950 and 2000, U.S. real GDP and real GDP per capita both grew at approximately 1 percent per year. Answer: B

Type: F Topic: 2 E: 312 MA: 312 37. Empirical studies suggest that: A) labor productivity has declined throughout U.S. history. B) real GDP per capita in the United States grew about 2.3 percent per year between 1950 and 2000. C) the achieving of economies of scale is the most important factor in U.S. economic growth. D) the U.S.'s real GDP has grown the slowest of all industrialized nations, particularly since 1995. Answer: B

Type: F Topic: 2 E: 312 MA: 312 38. Between 1950 and 2000, the long-run average annual rate of growth of real GDP in the United States was about: A) 8.2 percent. B) 5.1 percent. C) 3.5 percent. D) 1.8 percent. Answer: C

Type: F Topic: 2 E: 312 MA: 312 39. In the United States real GDP: A) has grown faster than real GDP per capita. B) has grown faster in recent years than has nominal GDP. C) per capita has grown faster than real GDP. D) and real GDP per capita have grown at nearly identical rates. Answer: A

McConnell/Brue: Economics, 16/e

Page 522

Chapter 17: Economic Growth

Type: F Topic: 2 E: 313-314 MA: 313-314 40. The largest contributor to increases in the productivity of American labor is: A) the reallocation of labor from agriculture to manufacturing. B) improvements in labor quality. C) increases in the quantity of capital. D) technological advance. Answer: D

Type: F Topic: 2 E: 314 MA: 314 41. Which of the following statements is correct? A) The U.S. population has increased more rapidly than real GDP in recent decades. B) Improved education and training of labor is the most important source of U.S. productivity growth. C) The average American factory worker has about 16 years of formal education. D) The amount of real capital used per worker has increased historically in the United States. Answer: D

Type: A Topic: 2 E: 314 MA: 314 42. The historical reallocation of labor from agriculture to manufacturing in the United States has: A) been inflationary. B) had no effect on the average productivity of labor. C) increased the average productivity of labor. D) reduced the average productivity of labor. Answer: C

Type: F Topic: 2 E: 314 MA: 314 43. More than half the growth of real GDP in the United States is caused by: A) a falling price level. B) the reallocation of labor from manufacturing to agriculture. C) increases in the productivity of labor. D) the use of fewer inputs of labor. Answer: C

Type: F Topic: 2 E: 313-314 MA: 313-314 44. Which of the following is the largest contribution to the growth of labor productivity in the United States? A) technological advance C) economies of scale B) education and training of labor D) improved resource allocation Answer: A

Type: D Topic: 2 E: 314 MA: 314 45. A nation's infrastructure refers to: A) its ability to realize economies of scale. B) its stock of technological knowledge. C) public capital goods such as highways and sanitation systems. D) the productivity of its labor force. Answer: C

McConnell/Brue: Economics, 16/e

Page 523

Chapter 17: Economic Growth

Type: D Topic: 2 E: 315 MA: 315 46. Economies of scale refer to: A) the idea that proprietorships are less bureaucratic and therefore more efficient than corporations. B) public investments in highways, schools, utilities, and such. C) the fact that large producers may be able to use more efficient technologies than smaller producers. D) the reallocation of labor from less-productive to more-productive uses. Answer: C

Type: A Topic: 2 E: 314 MA: 314 47. Other things equal, if a full-employment economy reallocated a substantial quantity of its resources to capital goods, we would expect: A) present consumption to rise. C) a lower rate of growth of real GDP. B) future consumption to fall. D) labor productivity to rise. Answer: D

Type: A Topic: 2 E: 314 MA: 314 48. Other things equal, in which of the following instances would the increase in labor productivity be the greatest? A) the stock of real capital and inputs of labor increase proportionately B) the increase in the stock of real capital exceeds the increase in inputs of labor C) the increase in inputs of labor exceeds the increase in the stock of real capital D) inputs of labor increase and the stock of real capital remains constant Answer: B

The New Economy

Type: A Topic: 3 E: 317 MA: 317 49. If the secular trend of labor productivity is 3 percent per year, the number of years that it will take for the standard of living to double will be about: A) 15 years. B) 17 years. C) 20 years. D) 23 years. Answer: D

Type: C Topic: 3 E: 317 MA: 317 50. If the secular trend of labor productivity rises from 2 percent per year to 4 percent, the number of years that it will take for the standard of living to double will decline by about: A) 5 years. B) 10 years. C) 17 years. D) 23 years. Answer: C

Type: F Topic: 3 E: 316 MA: 316 51. The annual growth of U.S. labor productivity: A) was greater between 1973 and 1995 than between 1995 and 2002. B) was greater between 1995 and 2002 than between 1973 and 1995. C) was negative in the late 1990s. D) averaged nearly 5 percent in the 1990s. Answer: B

McConnell/Brue: Economics, 16/e

Page 524

Chapter 17: Economic Growth

Type: D Topic: 3 E: 317 MA: 317 52. The New Economy is characterized by: A) a higher trend rate of saving. B) a higher natural rate of unemployment. Answer: C

C) a higher trend rate of productivity growth. D) the end of the business cycle.

Type: D Topic: 3 E: 317 MA: 317 53. The view that the trend rate of U.S. productivity growth accelerated between 1995 and 2002 is closely associated with the: A) idea of the New Economy. B) distinction between short-run and long-run aggregate supply. C) idea of the invisible hand. D) theory of comparative advantage. Answer: A

Type: D Topic: 3 E: 318 MA: 318 54. Increases in the value of a product to each user, including existing users, as the total number of users rises are called: A) information cascades. B) learning effects. C) network effects. D) scale economies. Answer: C

Type: D Topic: 3 E: 318 MA: 318 55. Network effects are: A) rises in the value of a product to each user, including existing users, as the total number of users rises. B) reductions in per unit production cost as firms learn by doing. C) increases in demand resulting from products being mentioned positively in a television program. D) the change in real GDP resulting from a change in investment or government spending. Answer: A

Type: A Topic: 3 E: 318 MA: 318 56. All of the following are sources of increasing returns and economies of scale except: A) network effects. C) more specialized inputs. B) spreading of development costs. D) coordination problems in large organizations. Answer: D

Type: A Topic: 3 E: 318 MA: 318 57. All of the following are sources of increasing returns and economies of scale except: A) network effects. B) the multiplier effect. C) learning-by-doing. D) simultaneous consumption. Answer: B

Type: F Topic: 3 E: 317 MA: 317 58. The fundamental invention underpinning the New Economy is the: A) microchip. B) fuel cell. C) Internet D) personal computer. Answer: A

McConnell/Brue: Economics, 16/e

Page 525

Chapter 17: Economic Growth

Type: F Topic: 3 E: 318 MA: 318 59. All of the following are economic implications of the New Economy except: A) a lower natural rate of unemployment. C) an end to the business cycle. B) higher rates of productivity advance. D) a greater rate of economic growth. Answer: C

Type: A Topic: 3 E: 320 MA: 320 60. Critics of the idea of the New Economy say that: A) it is too soon to judge whether the high productivity advances between 1995 and 2002 are permanent or transitory. B) the difficulties of the dot.com companies in 2001 disproves the idea of the New Economy. C) between 1995 and 2002 the economy moved below its natural rate of unemployment and paid the price in the form of accelerating inflation. D) the improved growth performance of the U.S. economy between 1995 and 2002 resulted from adroit monetary policy and not from increases in productivity. Answer: A

Type: F Topic: 3 E: 317 MA: 317 61. According to the adherents of the New Economy view, the above-normal economic growth in the United States between 1995 and 2002 was caused by: A) increases in the rate of personal saving. B) increased entrepreneurial activity, application of information technology, and global competition. C) rising Federal budget surpluses that reduced real interest rates. D) expansionary monetary policy. Answer: B

Type: A Topic: 3 E: 317 MA: 317 62. Proponents of the idea of a New Economy say that: A) the United States is entering an era of high structural unemployment due to rapid technological change. B) technological advance creates its own supply, which in turn creates its own demand. C) innovations in computers and communications, together with global capitalism, are greatly boosting U.S. productivity and the economy's potential economic growth rate. D) technological change will require more central planning and government regulation. Answer: C

Type: A Topic: 3 E: 320 MA: 320 63. Critics of the idea of a New Economy say that: A) it is too early to predict whether recent boosts in U.S. productivity are permanent or transitory. B) global capitalism is a media fiction, not an economic reality. C) the growth of service productivity is severely understated. D) many of the factors that slowed productivity in the 1973-1995 period have now reversed themselves. Answer: A

McConnell/Brue: Economics, 16/e

Page 526

Chapter 17: Economic Growth

Type: F Topic: 3 E: 316-317 MA: 316-317 64. Between 1995 and 2002, the U.S. productivity rate: A) was slightly negative, mainly because of record levels of employment growth. B) picked up substantially compared to prior years, leading some economists to predict a long-lasting resurgence of productivity growth. C) slowed considerably relative to the high rates between 1990 and 1995. D) reached record low levels for the United States' economy, leading some economists to talk of "secular stagnation." Answer: B

Type: A Topic: 3 E: 317 MA: 317 65. The claim that innovations in information technology, together with global capitalism, are leading to a long-term increase in U.S. productivity growth is known as the: A) rational expectations theory. C) new-Keynesian perspective. B) theory of creative destruction. D) New Economy view. Answer: D

Growth debate

Type: A Topic: 4 E: 321 MA: 321 66. Which of the following is a true statement? A) Economists who support economic growth say that it is the most practical route to the higher standards of living that the vast majority of people desire. B) Adherents of the view that the United States has achieved a New Economy contend that the business cycle is dead. C) Most economists believe that increases in real GDP actually produce decreases in overall economic well-being because of spillover costs. D) Mainstream economists disagree as whether the rate or productivity growth was higher between 1995 and 2002 than between 1973 and 1995. Answer: A

Type: A Topic: 4 E: 321 MA: 321 67. Proponents of economic growth say that pollution: A) is an inevitable by-product of growth. B) occurs, not because of growth, but because common properties are treated as free goods. C) declines as a country moves from agriculture to industry. D) is detrimental to economic growth. Answer: B

Type: A Topic: 4 E: 320-321 MA: 320-321 68. Critics of economic growth: A) contend that growth and industrialization reduce pollution. B) argue that economic growth does not resolve socioeconomic problems such as income inequality. C) point out that growth results in greater economic security for workers. D) say that its benefits accrue nearly exclusively to white males. Answer: B

McConnell/Brue: Economics, 16/e

Page 527

Chapter 17: Economic Growth

Type: A Topic: 4 E: 321 MA: 321 69. Proponents of economic growth make all of the following arguments except: A) Growth is the basic means of improving living standards. B) It is easier to reduce poverty when the economy is growing than when it is not. C) There is a direct relationship between a growing real GDP and rising pollution. D) Growth provides an economic environment favorable to education and self-fulfillment. Answer: C

Consider This Questions

Type: F E: 313 MA: 313 Status: New 70. (Consider This) The main point of the Consider This box about hypothetical countries Alpha, Bravo, and Charlie is that over several decades differing: A) inflation rates create large differences in real GDP per capita. B) economic growth rates create large differences in real GDP per capita. C) ratios of defense spending to GDP create large differences in real GDP per capita. D) unemployment rates create large differences in real GDP per capita. Answer: B

Type: F E: 313 MA: 313 Status: New 71. (Consider This) According to the Consider This box about hypothetical countries Alpha, Bravo, and Charlie, small differences in __________ make for large differences in _________ over several decades, assuming the same growth of population for each country. A) inflation rates; unemployment rates C) economic growth rates: real GDP per capita B) unemployment rates; economic growth rates D) tax rates; real GDP per capita Answer: C

Last Word Questions

Type: A E: 322 MA: 322 Status: New 72. (Last Word) The rapid rise in the number of women in the paid U.S. workforce over the past several decades has: A) shifted the U.S. production possibilities curve inward (to the left). B) moved the U.S. economy from a point inside its production possibilities curve to a point on the curve. C) reduced income inequality in the United States. D) shifted the U.S. production possibilities curve outward (to the right) and expanded real GDP. Answer: D

Type: F E: 322-323 MA: 322-323 Status: New 73. (Last Word) Over the past several decades, the percentage of women in the paid U.S. workforce has: A) increased in spite of declining wages for women. B) decreased because relatively more women are staying home to raise their children. C) increased due to higher wages, expanded job accessibility, changing preferences and attitudes, and other factors. D) increased for unmarried women, but decreased for married women. Answer: C

McConnell/Brue: Economics, 16/e

Page 528

Chapter 17: Economic Growth

Type: F E: 323 MA: 323 Status: New 74. (Last Word) Rising wages for women in the United States have: A) increased the proportion of women working part time compared to working full time. B) increased labor costs and thus shifted the nation's production possibilities curve inward. C) increased average family size in the United States. D) increased the percentage of married women in the workforce. Answer: D

True/False Questions

Type: F E: 312 MA: 312 75. In the United States, real GDP per capita has increased more rapidly than real GDP. Answer: False

Type: F E: 309 MA: 309 76. A rightward shift of a nation's production possibilities curve is a necessary but not sufficient condition for economic growth. Answer: True

Type: A E: 309 MA: 309 77. Economic growth can be shown as a movement from a point on one production possibility curve to a point on a curve located farther from the origin. Answer: True

Type: A E: 311 MA: 311 78. A rightward shift of an economy's long-run aggregate supply curve is equivalent to a movement along an existing production possibilities curve. Answer: False

Type: F E: 312 MA: 312 79. Growth of real GDP in the United States has averaged slightly more than 5 percent annually since 1950. Answer: False

Type: F E: 313 MA: 313 80. Between 1929 and 2002, rising labor productivity contributed more to U.S. economic growth than did increases in inputs Answer: True

Type: F E: 310 MA: 310 81. Real GDP = worker-hours x labor productivity. Answer: True

Type: E E: 310 MA: 310 82. Labor productivity = worker-hours/real GDP. Answer: False

McConnell/Brue: Economics, 16/e

Page 529

Chapter 17: Economic Growth

Type: E E: 314 MA: 314 83. Improvements in education and training explain about 80 percent of the historical growth of U.S. labor productivity. Answer: False

Type: F E: 316-317 MA: 316-317 84. Output-per worker in the United States declined between 1995 and 2002. Answer: False

Type: F E: 316 MA: 316 85. Productivity growth was greater between 1973 and 1995 than between 1995 and 2002. Answer: False

Type: F E: 317 MA: 317 86. The New Economy of 1995-2002 was characterized by greater productivity growth and greater economic growth than in the immediately preceding two decades. Answer: True

Type: F E: 317 MA: 317 87. According to proponents of the New Economy, the business cycle is dead. Answer: False

Type: F E: 320-321 MA: 320-321 88. Critics of economic growth say studies show that people are not interested in achieving higher standards of living. Answer: False

McConnell/Brue: Economics, 16/e

Page 530

CHAPTER 18

Deficits, Surpluses, and the Public Debt

Topic 1. 2. 3. 4. 5. Definitions: deficits, surpluses, debt Budget philosophies Public debt causes and facts Domestic economic implications Deficits and surpluses: 1992-2012 Last Word True-False

Question numbers 1-19 20-30 31-47 48-60 61-74 75-78 79-89

____________________________________________________________

_______________________________________

____________________________________________________________

_______________________________________

Multiple Choice Questions Definitions: deficits, surpluses, debt

Type: D Topic: 1 E: 325 MA: 325 1. The U.S. public debt: A) refers to the debts of all units of government--Federal, state, and local. B) consists of the total debt of U.S. households, businesses, and government. C) refers to the collective amount that U.S. citizens and businesses owe to foreigners. D) consists of the historical accumulation of all Federal government deficits and surpluses. Answer: D

Type: D Topic: 1 E: 325 MA: 325 2. The Federal budget deficit is found by: A) subtracting government tax revenues plus government borrowing from government spending in a particular year. B) subtracting government tax revenues from government spending in a particular year. C) cumulating the differences between government spending and tax revenues over all years since the nation's founding. D) subtracting government revenues from the noninvestment-type government spending in a particular year. Answer: B

Type: D Topic: 1 E: 325 MA: 325 3. The amount by which government expenditures exceed revenues during a particular year is the: A) public debt. B) budget deficit. C) full-employment. D) GDP gap. Answer: B

Chapter 18: Deficits, Surpluses, and the Public Debt

Type: D Topic: 1 E: 325 MA: 325 4. The amount by which Federal tax revenues exceed Federal government expenditures during a particular year is the: A) Federal reserve. B) budget deficit. C) budget surplus. D) public debt. Answer: C

Type: D Topic: 1 E: 325 MA: 325 5. The Federal budget surplus is found by: A) subtracting government revenue plus government borrowing from government spending in a particular year. B) cumulating the difference between government spending and tax revenues over all years since the nation's founding. C) subtracting government revenues from government spending on noninvestment goods in a particular year. D) subtracting government spending from government tax revenue in a particular year. Answer: D

Type: A Topic: 1 E: 325 MA: 325 6. The public debt is the amount of money that: A) state and local governments owe to the Federal government. B) Americans owe to foreigners. C) the Federal government owes to holders of U.S. securities. D) the Federal government owes to taxpayers. Answer: C

Type: A Topic: 1 E: 325 MA: 325 7. The public debt is held as: A) U.S. securities, corporate bonds, and common stock. B) Federal Reserve Notes. C) U.S. gold certificates. D) Treasury bills, Treasury notes, Treasury bonds, and U.S. savings bonds. Answer: D

Use the following to answer questions 8-10:
Security Treasury bills Corporate bonds Treasury notes Corporate stock U.S. saving bonds Treasury bonds Amount (in billions) $220 140 80 200 60 100

Type: A Topic: 1 E: 325 MA: 325 Status: New 8. The public debt for the above economy is: A) $540 billion. B) $400 billion. C) $580 billion. D) $460 billion. Answer: D

McConnell/Brue: Economics, 16/e

Page 532

Chapter 18: Deficits, Surpluses, and the Public Debt

Type: A Topic: 1 E: 325 MA: 325 Status: New 9. Other things equal, an increase of Treasury bonds from $100 billion to $120 billion in the above economy would: A) not change the size of the public debt. B) increase the public debt from $460 billion to $480 billion. C) increase the public debt from $400 billion to $420 billion. D) decrease the public debt by $20 billion. Answer: B

Type: A Topic: 1 E: 325 MA: 325 Status: New 10. Other things equal, an increase of corporate bonds from $140 billion to $150 billion in the above economy would: A) not change the size of the public debt. B) increase the public debt from $460 billion to $470 billion. C) increase the public debt from $600 billion to $610 billion. D) decrease the public debt by $20 billion. Answer: A

Type: A Topic: 1 E: 325 MA: 325 11. Suppose the Federal government had budget deficits of $40 billion in year 1 and $50 billion in year 2 but had budget surpluses of $20 billion in year 3 and $50 billion in year 4. Also assume that it used its budget surpluses to pay down the public debt. At the end of these four years, the Federal government's public debt would have: A) increased by $90 billion. C) decreased by $70 billion. B) increased by $20 billion. D) decreased by $20 billion. Answer: B

Type: A Topic: 1 E: 325 MA: 325 12. Suppose the Federal government had budget surpluses of $80 billion in year 1 and $120 billion in year 2 but had budget deficits of $10 billion in year 3 and $40 billion in year 4. Also assume that it used its budget surpluses to pay down the public debt. At the end of these four years, the Federal government's public debt would have: A) increased by $50 billion. C) decreased by $200 billion. B) increased by $150 billion. D) decreased by $150 billion. Answer: D

McConnell/Brue: Economics, 16/e

Page 533

Chapter 18: Deficits, Surpluses, and the Public Debt

Use the following to answer questions 13-18: Answer the next question(s) using the following budget information for a hypothetical economy. Assume that all budget surpluses are use to pay down the public debt.
Year 1 Year 2 Year 3 Year 4 Year 5 Year 6 Government spending $450 500 600 640 680 600 Tax revenues $425 450 500 620 580 620 GDP $2000 3000 4000 5000 4800 5000

Type: T Topic: 1 E: 325 MA: 325 13. Refer to the above data. If year 1 is the first year of this nation's existence and year 6 is the present year, this nation's public debt is: A) $275 billion. B) $100 billion. C) $3540 billion. D) $230 billion. Answer: A

Type: T Topic: 1 E: 325 MA: 325 14. Refer to the above data. The budget deficit in year 3 is: A) $175 billion. B) $3050 billion. C) $100 billion. Answer: C

D) $295 billion.

Type: A Topic: 1 E: 325 MA: 325 15. Refer to the above data. If year 1 is the first year of this nation's existence and year 4 is the present year, the public debt as a percentage of GDP in year 4 is: A) 7.5 percent. B) 1.39 percent. C) 2.5 percent. D) 3.9 percent. Answer: D

Type: T Topic: 1 E: 325 MA: 325 16. Refer to the above data. A budget surplus occurred in year: A) 2 B) 3 C) 4 D) 6 Answer: D

Type: T Topic: 1 E: 325 MA: 325 17. Refer to the above data. The public debt declined in year: A) 6 B) 5 C) 4 D) 3 Answer: A

Type: T Topic: 1 E: 325 MA: 325 18. Refer to the above data. As a percentage of GDP, the: A) budget deficit was 3.9 percent in year 4. B) budget surplus was less than 1 percent in year 6. C) public debt was 3 percent in year 6. D) public debt was 12.5 percent in year 1. Answer: B

McConnell/Brue: Economics, 16/e

Page 534

Chapter 18: Deficits, Surpluses, and the Public Debt

Type: D Topic: 1 E: 325 MA: 325 19. Which of the following statements is not correct? A) State and local governments in the aggregate have realized budget surpluses in some years. B) A Federal deficit of $20 billion in 2002 would increase the public debt by $20 billion. C) The public debt is the accumulation of all the Federal government's deficits and surpluses. D) The public debt refers to the debts of all units of government--Federal, state, and local. Answer: D

Budget philosophies

Type: A Topic: 2 E: 326 MA: 326 20. Fiscal conservatives favor an annually balanced budget primarily because they: A) believe the private sector is inherently unstable. B) believe the public debt will soon cause the Federal government to go bankrupt. C) believe deficit financing leads to an undesirable expansion of the public sector of the economy. D) fear interest payments on the debt will increase income inequality. Answer: C

Type: A Topic: 2 E: 326 MA: 326 21. An annually balanced budget intensifies the business cycle because tax revenue declines associated with: A) prosperity will require cuts in government spending. B) recession will require increases in government spending. C) prosperity will require cuts in government spending. D) recession will require cuts in government spending. Answer: D

Type: A Topic: 2 E: 326 MA: 326 22. If government adhered strictly to an annually balanced budget, then during a recession: A) expenditures and tax collections would remain unchanged. B) both expenditures and tax rates would have to be decreased. C) both expenditures and tax rates would have to be increased. D) expenditures would have to be reduced and/or tax rates increased. Answer: D

Type: A Topic: 2 E: 326 MA: 326 23. If government adhered strictly to an annually balanced budget, the government's budget would: A) reduce the size of the public debt. C) have no impact on real GDP. B) intensify the business cycle. D) stabilize the economy. Answer: B

Type: D Topic: 2 E: 326 MA: 326 24. The idea of a cyclically balanced budget is for government to: A) balance the economy, not worrying about expansion of the public debt. B) balance the budget over ten-year periods. C) exactly match budget deficits accruing in the recession phase of the business cycle with surpluses occurring in the late recovery phase. D) make sure the full-employment deficit is always zero. Answer: C

McConnell/Brue: Economics, 16/e

Page 535

Chapter 18: Deficits, Surpluses, and the Public Debt

Type: A Topic: 2 E: 326 MA: 326 25. A major criticism of a cyclically balanced budget is that: A) tax revenues automatically fall in recessions, but do not automatically increase in expansions. B) automatic (built-in) stabilizers are so effective that discretionary fiscal policy is unnecessary. C) such a policy has an inflationary bias. D) upswings and downswings of the business cycle are not always of equal magnitude and duration. Answer: D

Type: D Topic: 2 E: 326 MA: 326 26. Functional finance: A) is designed to increase the MPC to achieve greater macroeconomic stability. B) requires annually balanced budgets. C) views the public budget primarily as a way to stabilize the economy. D) is supported mainly by monetarists. Answer: C

Type: D Topic: 2 E: 326 MA: 326 27. Functional finance contends that: A) the Federal budget should be balanced, not annually, but rather over the business cycle. B) the Federal budget should be used primarily to alter the allocation of resources and not to stabilize the economy. C) automatic stabilizers are so effective that discretionary fiscal policy is unnecessary. D) the primary purpose of the Federal budget is to stabilize the economy and balancing the budget as such is of far lesser importance. Answer: D

Type: D Topic: 2 E: 342 MA: 326 28. 326The idea that the basic purpose of the Federal budget is to stabilize the economy regardless of any resulting changes in the size of the public debt best describes: A) a socially optimal budget. C) a cyclically balanced budget. B) functional finance. D) an annually balanced budget. Answer: B

Type: A Topic: 2 E: 326 MA: 326 29. A proponent of functional finance would say that: A) the budget must balanced, but only over the course of an entire business cycle. B) balancing the budget is not the most important objective of fiscal policy. C) the public debt should be reduced during recessions and increased during inflations. D) balancing the budget is the primary goal of fiscal policy. Answer: B

Type: A Topic: 2 E: 326 MA: 326 30. The problems associated with a growing public debt are modest when compared with the costs entailed by recession and unemployment. This statement reflects the idea of: A) the balanced-budget multiplier. C) functional finance. B) the full-employment budget. D) the crowding-out effect. Answer: C

McConnell/Brue: Economics, 16/e

Page 536

Chapter 18: Deficits, Surpluses, and the Public Debt

Public debt causes and facts

Type: F Topic: 3 E: 327-328 MA: 327-328 31. Which of the following is not a significant contributor to the U.S. public debt? A) war financing C) recessions B) tax cuts and expenditure increases in the 1980s D) demand-pull inflation Answer: D

Type: F Topic: 3 E: 327 MA: 327 32. Recessions have contributed to the public debt by: A) reducing national income and therefore tax revenues. B) increasing real interest rates. C) increasing the international value of the dollar. D) increasing national saving. Answer: A

Type: F Topic: 3 E: 328 MA: 328 33. The deep cut in tax rates in the early 1980s: A) increased tax revenues, reduced budget deficits, and slowed the growth of the public debt. B) were unaccompanied by expenditure reductions and therefore increased the public debt. C) was accompanied by equally deep cuts in Federal spending, thus neither increased nor decreased the public debt. D) contributed to rising budget deficits, but not to a rising public debt. Answer: B

Type: F Topic: 3 E: 327 MA: 327 34. Which of the following statements is correct? A) Federal deficits were larger in the late 1990s than in the early 1990s. B) Deep tax cuts always expand tax revenues and reduce the public debt. C) The public debt has usually declined during wartime. D) There is a tendency for the public debt to grow during recessions. Answer: D

Type: F Topic: 3 E: 327 MA: 327 35. In 2002, the public debt was: A) held largely by foreign governments. B) about four times as large as the GDP. Answer: D

C) about twice as large as the GDP. D) about 59 percent as large as the GDP.

Type: F Topic: 3 E: 327 MA: 327 36. In 2002, the public debt was about: A) $6.2 trillion. B) $1 trillion. C) $470 billion. Answer: A

D) $320 billion.

McConnell/Brue: Economics, 16/e

Page 537

Chapter 18: Deficits, Surpluses, and the Public Debt

Type: F Topic: 3 E: 327, 328 MA: 327, 328 37. Between 1995 and 2000, the public debt: A) remained constant absolutely, but declined as a percentage of the GDP. B) declined absolutely, but remained constant as a percentage of the GDP. C) declined as a percentage of the GDP. D) increased as a percentage of the GDP. Answer: C

Type: A Topic: 3 E: 328 MA: 328 38. The average tax rate required to service the public debt is roughly measured by: A) the absolute size of the debt. B) the debt as a fraction of the GDP. C) interest on the debt as a percentage of the GDP. D) the ratio of government spending to the GDP. Answer: C

Type: F Topic: 3 E: 328 MA: 328 39. As a percent of GDP, the United States public debt is: A) the highest among major industrial nations. B) the lowest among major industrial nations. C) lower than the public debts of several other major industrial nations. D) higher than the percentages for Canada, Germany, and Italy. Answer: C

Type: F Topic: 3 E: 328 MA: 328 40. What percentage of the U.S. public debt is held by Federal agencies and the Federal Reserve? A) 50 percent B) 67 percent C) 43 percent D) 11 percent Answer: C

Type: F Topic: 3 E: 328 MA: 328 41. What percentage of the public debt is held by foreign individuals and institutions? A) 18 percent B) 2 percent C) 33 percent D) 50 percent Answer: A

Type: F Topic: 3 E: 329 MA: 329 42. The portion of the public debt held outside Federal agencies and the Federal Reserve is: A) substantially larger than the portion held by Federal Agencies and the Federal Reserve. B) about half as large as the portion held by Federal Agencies and the Federal Reserve. C) equally split between U.S. and foreign lenders. D) includes U.S. saving bonds, but not Treasury bills, Treasury notes, and Treasury bonds. Answer: A

Type: F Topic: 3 E: 329 MA: 329 43. The largest proportion of the public debt is held by: A) the U.S. public (individuals, businesses, financial institutions, etc.) and state and local governments. B) foreign individuals and institutions. C) the Federal Reserve System. D) the U.S. Treasury. Answer: A

McConnell/Brue: Economics, 16/e

Page 538

Chapter 18: Deficits, Surpluses, and the Public Debt

Type: A Topic: 3 E: 329 MA: 329 Status: New 44. If monies added to, or subtracted from, the Social Security trust fund were excluded from Federal budget calculations, the current Federal budget: A) deficit would nearly disappear. C) surplus would nearly disappear. B) deficit would be substantially larger. D) surplus would be substantially smaller. Answer: B

Type: A Topic: 3 E: 329 MA: 329 Status: New 45. The Social Security trust fund currently is in: A) deficit, and it inclusion in the Federal budget increases the stated size of the budget deficit. B) deficit, and it inclusion in the Federal budget reduces the stated size of the budget deficit. C) surplus, and its inclusion in the Federal budget reduces the stated size of the budget deficit. D) surplus, and it inclusion in the Federal budget increases the stated size of the Federal budget deficit. Answer: C

Type: A Topic: 3 E: 329 MA: 329 Status: New 46. About ____ percent of the U.S. public debt is held by people and institutions abroad. A) 40 B) 8 C) 18 D) 33 Answer: C

Type: A Topic: 3 E: 329 MA: 329 Status: New 47. About ____ percent of the U.S. public debt is held by the Federal government and Federal Reserve. A) 10 B) 21 C) 62 D) 43 Answer: D

Domestic economic implications

Type: A Topic: 4 E: 330 MA: 330 48. To say that "the U.S. public debt is also a public credit" is to say that: A) only interest payments on the public debt are an economic burden. B) official figures understate the size of the public debt. C) the bulk of the public debt is owned by U.S. citizens and institutions. D) the public debt is equal to the land and buildings assets owned by the Federal government. Answer: C

Type: A Topic: 4 E: 328 MA: 328 49. Payment of interest on the U.S. public debt: A) increases the current domestic standard of living in the United States. B) has no effect on the distribution of income. C) is thought to decrease income inequality. D) is thought to increase income inequality. Answer: D

Type: A Topic: 4 E: 330 MA: 330 50. The most likely way the public debt burdens future generations, if at all, is by: A) reducing the current level of investment. C) causing deflation. B) causing future unemployment. D) reducing real interest rates. Answer: A

McConnell/Brue: Economics, 16/e

Page 539

Chapter 18: Deficits, Surpluses, and the Public Debt

Type: A Topic: 4 E: 331 MA: 331 51. Other things equal, the stock of capital inherited by future generations is likely to be smaller when government spending: A) increases during a period of recession, rather than prosperity. B) is primarily for capital-type goods. C) is financed by borrowing. D) is financed by taxation. Answer: C

Type: D Topic: 4 E: 331 MA: 331 52. The crowding-out effect suggests that: A) tax increases are paid primarily out of saving and therefore are not an effective fiscal device. B) government borrowing to finance the public debt increases the real interest rate and reduces private investment. C) it is very difficult to have excessive aggregate spending in a capitalist economy. D) consumer and investment spending always vary inversely. Answer: B

Type: D Topic: 4 E: 331 MA: 331 53. The Federal government has a large public debt that it finances through borrowing. As a result, real interest rates are higher than otherwise and the volume of private investment spending is lower. This illustrates the: A) equation-of-exchange effect. B) paradox of thrift. C) crowding-out effect. D) net export effect. Answer: C

Type: A Topic: 4 E: 331 MA: 331 54. The real burden of an increase in the public debt: A) may be very small or conceivably zero when the economy is in a severe depression. B) will be smaller when full employment exists than when the economy has large quantities of idle resources. C) can be shifted to future generations if the debt is internally financed. D) can best be measured by the dollar increase in the size of the debt. Answer: A

Type: A Topic: 4 E: 331 MA: 331 55. Which one of the following might offset a crowding-out effect of financing a large public debt? A) a decline in net exports C) a decrease in the money supply B) an increase in public investment D) a decline in public investment Answer: B

McConnell/Brue: Economics, 16/e

Page 540

Chapter 18: Deficits, Surpluses, and the Public Debt

Use the following to answer questions 56-57:

16 14

Interest rate (percent)

12 10

8 6 4 2 0 5
ID1 ID2

10 15 20 25 30 35 40 Investment (billions of dollars)

Type: G Topic: 4 E: 331 MA: 331 56. Refer to the above diagram. Initially assume that the investment demand curve is ID1. The crowding-out effect of a large public debt would be shown as a(n): A) shift of the investment demand curve from ID1 to ID2. B) leftward shift of the investment demand curve. C) increase in the interest rate from 4 percent to 6 percent and a decline in investment spending of $5 billion. D) increase in the interest rate from 6 percent to 8 percent and a decline in investment spending of $40 billion. Answer: C

Type: G Topic: 4 E: 331 MA: 331 57. Refer to the above diagram. Initially assume that the investment demand curve is ID1. Which of the following effects of financing a large public debt might shift the investment demand curve from ID1 to ID2, wholly offsetting any crowding-out effect? A) an improvement in profit expectations by businesses B) a decrease in saving C) a decline in the interest rate D) an increase in the marginal propensity to consume Answer: A

Type: A Topic: 4 E: 331 MA: 331 58. Which of the following is the best example of public investment? A) salaries of Senators and Representatives C) construction of highways B) government expenditures on food stamps D) funding of regulatory agencies Answer: C

McConnell/Brue: Economics, 16/e

Page 541

Chapter 18: Deficits, Surpluses, and the Public Debt

Type: F Topic: 4 E: 329 MA: 329 59. The true size of Federal budget deficits may be understated because: A) a portion of government spending is public investment. B) inflation reduces the real value of the public debt. C) Social Security surpluses are included as government tax revenues in measuring the budget deficit. D) foreign holdings of the debt have recently increased. Answer: C

Type: F Topic: 4 E: 329 MA: 329 60. Which of the following allegedly understates the true size of the Federal budget deficit? A) inclusion of government spending on the savings and loan (S&L) bailout B) inclusion of the Social Security surplus C) inclusion of Federal excise tax receipts D) inclusion of current transfer payments Answer: B

Deficits and surpluses: 1992 - 2012

Type: F Topic: 5 E: 332 MA: 332 61. The Deficit Reduction Act of 1993: A) imposed Federal property taxes for the first time. B) increased the excise tax on gasoline by 50 cents a gallon. C) increased the top marginal tax rate on personal income to 39.6 percent. D) reduced government spending by $2 for each $1 increase in taxes. Answer: C

Type: F Topic: 5 E: 332 MA: 332 62. The Deficit Reduction Act of 1993: A) equalized the top marginal tax rates on personal and corporate income. B) slightly raised the corporate income tax rate. C) replaced the corporate income tax with a national sales tax. D) cut government spending programs, but did not increase taxes. Answer: B

Type: F Topic: 5 E: 332 MA: 332 63. The main focus of U.S. economic policy in the first half of the 1990s was: A) creating an expansionary fiscal policy. B) reducing Federal budget deficits and lowering real interest rates. C) enacting a contractionary monetary policy. D) reducing the trade deficit. Answer: B

Type: F Topic: 5 E: 332 MA: 332 64. Between 1990 and 1997, the U.S. had annual _____ _____, whereas between 1998 and 2000 it had annual _____ _____. A) trade deficits; trade surpluses C) budget deficits; budget surpluses B) trade surpluses, trade deficits D) budget surpluses; budget deficits. Answer: C

McConnell/Brue: Economics, 16/e

Page 542

Chapter 18: Deficits, Surpluses, and the Public Debt

Type: F Topic: 5 E: 332 MA: 332 65. In 2001, Federal budget surpluses were projected to cumulate to: A) $2 trillion by 2010. B) $5 trillion by 2010 C) $10 trillion by 2005. D) $20 trillion by 2020. Answer: B

Type: A Topic: 5 E: 333 MA: 333 66. Advocates of using the actual and projected budget surpluses of the late 1990s to pay down the public debt said that this policy would: A) reduce interest rates, increase private investment, and increase economic growth. B) increase take-home pay, reduce consumption spending, and increase economic growth. C) reduce exports and increase imports. D) enhance the Fed's ability to conduct monetary policy through open-market operations. Answer: A

Type: A Topic: 5 E: 333 MA: 333 67. Advocates of using the actual and projected budget surpluses of the late 1990s to reduce taxes said that this policy would: A) reduce interest rates, increase private investment, and increase economic growth. B) return surplus revenue to taxpayers and limit the growth of government spending. C) preserve the long-term health of the social security system by bolstering the social security trust fund. D) lower the natural rate of unemployment. Answer: B

Type: A Topic: 5 E: 333 MA: 333 68. Advocates of using the actual and projected budget surpluses of the late 1990s to pay for new social programs such as Medicare coverage for prescription drugs said these programs would: A) reduce interest rates, increase private investment, and increase economic growth. B) lower the natural rate of unemployment. C) boost consumer and business confidence and promote economic growth. D) enhance the nation's well-being. Answer: D

Type: A Topic: 6 E: 333 MA: 333 Status: New 69. Advocates of using the actual and projected budget surpluses of the late 1990s to bolster the social security trust fund said that: A) the number of older Americans will increase substantially over the next several decades, eventually straining the Social Security system. B) this policy would boost interest rates, increase saving, and increase private investment. C) this policy would enable deep cuts in the payroll tax used to finance social security. D) without such a policy, the Social Security system will go bankrupt in 2010. Answer: A

Type: F Topic: 5 E: 332 MA: 332 70. Annual budget deficits gave way to annual budget surpluses beginning in: A) 1982. B) 1989. C) 1993. D) 1998. Answer: D

McConnell/Brue: Economics, 16/e

Page 543

Chapter 18: Deficits, Surpluses, and the Public Debt

Type: F Topic: 5 E: 333 MA: 333 71. In 2002 and 2003, the United States had: A) large Federal budget surpluses. B) large Federal budget deficits. Answer: B

C) modest trade surpluses. D) a rising natural rate of unemployment.

Type: F Topic: 5 E: 333 MA: 333 Status: New 72. Which of the following is a true statement? A) Surpluses in the Federal budget between 1998 and 2001 gave way to a budget deficit in 2002. B) Surpluses in the Federal budget between 1992 and 1997 gave way to budget deficits between 1998 and 2002. C) The swing from budget surpluses in the late 1990s to budget deficits in the early 2000s resulted exclusively because of a downturn in the economy. D) The swing from budget surpluses in the late 1990s to budget deficits in the early 2000s resulted exclusively from deep tax cuts by the Bush administration. Answer: A

Type: F Topic: 5 E: 332 MA: 332 Status: New 73. The immediate primary cause of the swing from Federal budget surpluses between 1998 and 2001 to a budget deficit in 2002 was: A) the tax cuts of 2001. B) spending increases relating to the war in Afghanistan. C) the recession of 2001. D) the acceleration of inflation in 2001 and 2002. Answer: C

Type: F Topic: 5 E: 333-334 MA: 333-334 Status: New 74. Which of the following did not contributed to the large Federal budget deficits in 2002 and 2003? A) spending on the wars in Afghanistan and Iraq. C) Federal tax cuts. B) low interest rates. D) the recession of 2001 and its aftermath. Answer: B

Last Word Questions

Type: F E: 334 MA: 334 Status: New 75. (Last Word) The Social Security program (excluding Medicare) in the United States is projected to: A) grow from 4.4 percent of GDP today to 6.5 percent of GDP in 2035. B) grow from 1.0 percent of GDP today to 4.4 percent of GDP in 2035. C) decline from 4.4 percent of GDP today to 1.0 percent in 2035. D) stay relatively constant at 3 percent of GDP into the future. Answer: A

Type: F E: 334 MA: 334 Status: New 76. (Last Word) In the past few years, the Social Security trust fund in the United States has been: A) drawn down to cover payouts that currently exceed payroll tax revenues. B) built up in anticipation of payroll revenue shortfalls beginning in 2018. C) draw down to eliminate unnecessarily large surpluses in the fund. D) built up because of greater than expected immigration to the United States. Answer: B

McConnell/Brue: Economics, 16/e

Page 544

Chapter 18: Deficits, Surpluses, and the Public Debt

Type: F E: 334 MA: 334 Status: New 77. (Last Word) The Social Security program (excluding Medicare) in the United States is expected to use up the monies in the Social Security trust fund in: A) 2008. B) 2010. C) 2018. D) 2042. Answer: C

Type: F E: 334 MA: 334 Status: New 78. (Last Word) The Social Security program in the United States faces a long-term funding crisis because: A) the Social Security trust fund was exhausted in 2002. B) the number of retirees receiving benefits is rising more rapidly than the number of workers paying payroll taxes. C) recent tax cuts have reduced payroll tax revenues. D) the war on terrorism has drained funds away from Social Security. Answer: B

True/False Questions

Type: A E: 326 MA: 326 79. If government was required by law to balance its budget annually, it would be obligated to increase its spending during a recession. Answer: False

Type: A E: 327 MA: 327 80. Financing wartime expenditures by increasing internally held public debt permits a nation to defer a part of the economic cost of war. Answer: False

Type: A E: 327 MA: 327 81. The per capita public debt doubled between 1990 and 2000. Answer: False

Type: A E: 327 MA: 327 82. It is more meaningful economically to measure the public debt relative to the GDP than to measure it in absolute terms. Answer: True

Type: A E: 332 MA: 332 83. The United States has experienced budget surpluses in recent years. Answer: True

Type: F E: 326 MA: 326 84. The functional finance philosophy says that the Federal government should balance its budget over the course of the business cycle. Answer: False

McConnell/Brue: Economics, 16/e

Page 545

Chapter 18: Deficits, Surpluses, and the Public Debt

Type: D E: 325 MA: 325 85. The public debt is the accumulation of all deficits and surpluses that have occurred through time. Answer: True

Type: D E: 325 MA: 325 86. The public debt is held as Treasury bills, Treasury notes, Treasury bonds, and U.S. savings bonds. Answer: True

Type: F E: 332 MA: 332 87. Between 1994 and 2000, annual budget deficits gave way to annual budget surpluses. Answer: True

Type: A E: 331 MA: 331 88. The crowding-out effect of the public debt may be dampened if the investment-demand curve is shifting to the right. Answer: True

Type: A E: 332 MA: 332 89. The Deficit Reduction Act of 1993 limits the public debt to 100 percent of the GDP. Answer: False

McConnell/Brue: Economics, 16/e

Page 546

CHAPTER 19

Disputes over Macro Theory and Policy

Topic 1. Classics vs. Keynes: AD/AS 2. Mainstream view of instability 3. Monetarism/equation of exchange 4. Real-business cycle view 5. Coordination failures 6. New classical view of self-correction 7. Mainstream view of self-correction 8. Efficiency wages; insider-outsider theory 9. Policy rules; critiques of stabilization policy 10. Defense of discretionary stabilization policy 11. Absorption into the mainstream Consider This Last Word True-False

Question numbers 1-9 10-14 15-53 54-60 61-64 65-83 84-88 89-100 101-116 117-122 123-126 127-128 129-133 134-152

____________________________________________________________

_______________________________________

____________________________________________________________

_______________________________________

Multiple Choice Questions Classics vs. Keynes: AD/AS

Type: A Topic: 1 E: 340 MA: 340 1. In its simplest form, Keynesian theory views the aggregate supply curve as being: A) upsloping and quite steep. B) horizontal. C) vertical. D) downsloping. Answer: B

Type: A Topic: 1 E: 340 MA: 340 2. In its simplest form, Keynesian theory holds that a decline in aggregate demand will: A) increase the price level, but lower output and employment. B) reduce output and employment but not the price level. C) reduce both the price level and the levels of output and employment. D) reduce the price level, but not the levels of output and employment. Answer: B

Type: A Topic: 1 E: 338 MA: 338 3. Classical theory sees the aggregate supply curve as being: A) upsloping, but quite flat. B) downsloping. C) vertical. Answer: C

D) horizontal.

Chapter 19: Disputes over Macro Theory and Policy

Type: A Topic: 1 E: 339 MA: 339 4. In classical theory a decline in aggregate demand will: A) increase the price level, but lower output and employment. B) reduce output and employment but not the price level. C) reduce both the price level and the levels of output and employment. D) reduce the price level, but not the levels of output and employment. Answer: D

Type: A Topic: 1 E: 338 MA: 338 5. The classical aggregate supply curve suggests that: A) real output is unrelated to the price level. B) businesses must receive higher prices to produce more output. C) real output can be increased without affecting the price level. D) idle capital goods and unemployed workers are available in the economy. Answer: A

Use the following to answer questions 6-7:

AS
Price level Price level

AS AS AD1 AD2 Qu Qf Real GDP Figure B
Price level

P1 AD1 Qf Real GDP Figure A

P1 P2 Qu Qf Real GDP Figure C

AD1 AD2

P2

Type: G Topic: 1 E: 339 MA: 339 6. Refer to the above diagrams. Classical theory is best portrayed by: A) Figure A. B) Figure B. C) Figure C. D) none of the above. Answer: A

Type: G Topic: 1 E: 339 MA: 339 7. Refer to the above diagrams. Keynesian theory is best portrayed by: A) Figure A. B) Figure B. C) Figure C. D) none of the above. Answer: B

Type: G Topic: 1 E: 339 MA: 339 8. Classical theory views the aggregate demand curve as being: A) horizontal. B) vertical. C) stable. D) unstable. Answer: C

McConnell/Brue: Economics, 16/e

Page 548

Chapter 19: Disputes over Macro Theory and Policy

Type: A Topic: 1 E: 340 MA: 340 9. If there is full employment, and a decline in aggregate demand occurs: A) Keynesian analysis would predict a decline in prices and wages. B) Keynesian analysis would predict a decline in real output and employment. C) Classical analysis would predict a decline in real output and employment. D) Classical analysis would predict declines in both real GDP and the price level. Answer: B

Mainstream view of instability

Type: F Topic: 2 E: 340 MA: 340 10. The equation underlying the mainstream view of macroeconomics is: A) MV = PQ. B) Ca + Ig + Xn + G = GDP. C) S = a - bY. D) GDP = P x Q. Answer: B

Type: F Topic: 2 E: 340 MA: 340 11. The mainstream view is that macro instability is caused by: A) erratic growth of the nation's money supply. C) significant changes in investment spending. B) government interference in the economy. D) consumption "booms" and "busts." Answer: C

Type: F Topic: 2 E: 340 MA: 340 12. The mainstream view of macroeconomic instability is based mainly on: A) Keynesian ideas. C) the theory of rational expectations. B) monetarist ideas. D) the equation of exchange. Answer: A

Type: A Topic: 2 E: 340 MA: 340 13. According to mainstream macroeconomists, U.S. macro instability has resulted from: A) investment "booms" and "busts" and, occasionally, adverse aggregate supply shocks. B) adherence by the Fed to a monetary rule. C) government's attempts to balance its budget. D) wide fluctuations in net exports. Answer: A

Type: A Topic: 2 E: 340 MA: 340 14. The mainstream view of macro instability is that: A) changes in the money supply directly cause changes in aggregate demand and thus cause changes in real GDP. B) changes in investment shift the aggregate demand curve and thus cause changes in real GDP. C) bursts of innovation put the economy on an unsustainable growth path, eventually producing recession. D) changes in technology and resource availability are the two main sources of fluctuations of real GDP. Answer: B

McConnell/Brue: Economics, 16/e

Page 549

Chapter 19: Disputes over Macro Theory and Policy

Monetarism/equation of exchange

Type: F Topic: 3 E: 342 MA: 342 15. Economist Milton Friedman is most closely associated with: A) Keynesian economics. C) supply-side economics. B) the rational expectations theory. D) monetarism. Answer: D

Type: F Topic: 3 E: 341 MA: 341 16. The intellectual roots of monetarism are based on: A) the idea that prices and wages are inflexible or sticky. B) the idea that both product and resource markets are monopolistic. C) classical economics. D) Keynesian economics. Answer: C

Type: F Topic: 3 E: 341 MA: 341 17. According to monetarists: A) changes in the money supply are the primary cause of changes in real output and the price level. B) an expansionary fiscal policy will lower interest rates and overstimulate the economy. C) changes in the velocity of money are more important than changes in the money supply in causing the level of economic activity to change. D) the supply of money changes in response to changes in the levels of real output and prices. Answer: A

Type: F Topic: 3 E: 341 MA: 341 18. The basic equation of monetarism is: A) MV = PQ. B) Sa + T + M = I g + G + Xn C) V = M/PQ. D) Ca + Ig + Xn + G = GDP. Answer: A

Type: D Topic: 3 E: 341 MA: 341 19. The equation of exchange indicates that: A) MV = PQ. B) other things equal, an increase in the demand for money will increase P and/or Q. C) the velocity and the supply of money vary directly with one another. D) MP = VQ. Answer: A

Type: E Topic: 3 E: 341 MA: 341 20. If M is $300, P is $4, and Q is 200, then V must be: A) 4. B) 22/ 3. C) 1. D) 3. Answer: B

Type: A Topic: 3 E: 341 MA: 341 21. In the equation of exchange the level of aggregate expenditures is indicated by: A) MV. B) MV/Q. C) PM. D) MV/P. Answer: A

McConnell/Brue: Economics, 16/e

Page 550

Chapter 19: Disputes over Macro Theory and Policy

Type: A Topic: 3 E: 341 MA: 341 22. According to the equation of exchange, changes in the money supply can affect: A) only the velocity of money. C) only real output and employment. B) both the price level and real output. D) only the price level. Answer: B

Type: D Topic: 3 E: 341 MA: 341 23. The velocity of money is the: A) relationship between the money supply and the price level. B) number of times per year the average dollar is spent on final goods and services. C) relationship between asset and transactions demands for money. D) price level divided by aggregate supply. Answer: B

Type: A Topic: 3 E: 341 MA: 341 24. If a certain household earns and spends $24,000 per year and, on the average, holds a money balance of $6,000, then the velocity of money for this household is: A) 6. B) 1/6. C) 4. D) 1/4. Answer: C

Type: D Topic: 3 E: 341 MA: 341 25. Which of the following is a component of the equation of exchange? A) consumption B) the interest rate C) investment D) the velocity of money Answer: D

Type: D Topic: 3 E: 341 MA: 341 26. The velocity of money measures the: A) proportion of the money supply held as an asset. B) ratio of the transactions demand to the asset demand for money. C) average annual rate of increase in the money supply. D) number of times per year the average dollar is spent on final goods and services. Answer: D

Type: D Topic: 3 E: 341 MA: 341 27. In the equation of exchange V indicates the: A) value or purchasing power of the dollar. B) number of times per year the average dollar is spent. C) quantity of real output. D) reciprocal of the price level. Answer: B

Type: D Topic: 3 E: 341 MA: 341 28. The velocity of money: A) is defined as the number of times per year the average dollar is spent on final goods and services. B) is defined as the multiple by which a final increase in GDP exceeds an initial increase in spending. C) is equal to the reciprocal of the monetary multiplier. D) has been nearly constant during the past several decades. Answer: A

McConnell/Brue: Economics, 16/e

Page 551

Chapter 19: Disputes over Macro Theory and Policy

Type: A Topic: 3 E: 341 MA: 341 29. At the equilibrium level of GDP: A) MV = nominal Ca + Ig + Xn + G. B) MV = real GDP. Answer: A

C) M = nominal GDP.

D) V = 1/MPS.

Type: A Topic: 3 E: 341 MA: 341 30. If the price level does not change from one period to the next: A) the money supply must have been constant during the period. B) velocity must have been constant during the period. C) nominal GDP must have remained constant during the period. D) none of the above responses are valid. Answer: D

Type: A Topic: 3 E: 342 MA: 342 31. The velocity of money is equal to: A) 1/MPS. B) nominal GDP/M. C) 1/reserve ratio. Answer: B

D) nominal GDP/P.

Type: A Topic: 3 E: 341 MA: 341 32. The velocity of money is equal to: A) 1/MPS. B) 1/reserve ratio. C) M/GDP. Answer: D

D) none of the above

Type: A Topic: 3 E: 341 MA: 341 33. The equation of exchange suggests that, if the supply and velocity of money remain unchanged, an increase in the physical volume of goods and services produced will cause: A) the unemployment rate to rise. B) the Federal Reserve Banks to sell securities in the open market. C) a decline in the price level. D) an automatic budget deficit. Answer: C

Type: A Topic: 3 E: 341 MA: 341 34. If the amount of money in circulation is $180 billion and the value of the economy's total output is $540 billion, then the: A) circulation period of money must be one-fourth of a year. B) velocity of money is 4. C) average price per final good sold is $3. D) velocity of money is 3. Answer: D

McConnell/Brue: Economics, 16/e

Page 552

Chapter 19: Disputes over Macro Theory and Policy

Use the following to answer questions 35-37: Answer the next question(s) on the basis of the following information for a hypothetical economy. All values are in nominal terms. M = $100 V=2 Ca = $160 Xn = $10 G = $10

Type: E Topic: 3 E: 341 MA: 341 35. Refer to the above information. Nominal GDP is: A) $100. B) $200. C) $180. D) $50. Answer: B

Type: E Topic: 3 E: 341 MA: 341 36. Refer to the above information. If the price level P is 4, Q is: A) 50. B) 100. C) 200. D) 500. Answer: A

Type: E Topic: 3 E: 341 MA: 341 37. Refer to the above information. In equilibrium, Ig is: A) $20. B) $10. C) $5. D) $50. Answer: A

Type: A Topic: 3 E: 341 MA: 341 38. Most monetarists would say that: A) the MV = PQ equation provides a better understanding of the macroeconomy than does the Ca + Ig + Xn + G = GDP equation. B) most changes in the price level are explainable by changes in the money supply. C) the velocity of money is quite stable. D) all of the above are true. Answer: D

Type: A Topic: 3 E: 341 MA: 341 39. Monetarist say that the relationship between the amount of money which households and businesses want to hold and the level of national output and income: A) has decreased historically because of increased accessibility to credit. B) rises during recession and falls during periods of full employment. C) falls during recession and rises during periods of full employment. D) is relatively stable. Answer: D

McConnell/Brue: Economics, 16/e

Page 553

Chapter 19: Disputes over Macro Theory and Policy

Type: A Topic: 3 E: 341 MA: 341 40. Monetarist say: A) that, because P is stable, a change in M will change Q proportionately in the opposite direction. B) a change in the money supply will change aggregate demand and therefore the nominal GDP. C) a change in the money supply will change velocity, which in turn will change nominal GDP. D) a change in the money supply will change the interest rate, which will change investment spending and nominal GDP. Answer: B

Type: A Topic: 3 E: 341 MA: 341 41. To determine the velocity of money you would need to know: A) nominal GDP and real GDP. C) nominal GDP and the money supply. B) the money supply and the price level. D) nominal GDP and the interest rate. Answer: C

Type: A Topic: 3 E: 341 MA: 341 42. In a full-employment economy a rise in M will cause inflation unless: A) V rises in proportion to the increase in M . B) the quantity of goods produced declines proportionately. C) tax reductions accompany the increase in the money supply. D) the velocity of money diminishes. Answer: D

Type: F Topic: 3 E: 341 MA: 341 43. As monetarists view the equation of exchange: A) V changes erratically and unpredictably. B) V is quite stable. C) V usually changes in the same direction of any given change in M . D) V usually changes in the opposite direction of any given change in M . Answer: B

Type: E Topic: 3 E: 342 MA: 342 44. The velocity of money may be stated as: A) PM/V . B) nominal GDP/M . C) QM/P. Answer: B

D) PM/Q.

Type: F Topic: 3 E: 341 MA: 341 45. Monetarists believe the private economy is inherently: A) unstable and the public sector should be small. B) unstable and the public sector should be large. C) stable, but that the public sector should be large. D) stable and that the government sector should be small. Answer: D

Type: D Topic: 3 E: 341 MA: 341 46. In the equation of exchange the nominal GDP is designated by: A) PQ/M . B) MV/P. C) PQ. D) MV. Answer: C

McConnell/Brue: Economics, 16/e

Page 554

Chapter 19: Disputes over Macro Theory and Policy

Type: E Topic: 3 E: 342 MA: 342 47. The term V in the equation of exchange is equal to: A) M /nominal GDP. C) the reciprocal of the price level. B) nominal GDP/M . D) nominal GDP/Q. Answer: B

Type: A Topic: 3 E: 342 MA: 342 48. If the money supply is constant when both nominal and real GDP are rising, we can conclude that: A) tax rates have been increased. C) interest rates are falling. B) the velocity of money is rising. D) the unemployment rate is rising. Answer: B

Type: A Topic: 3 E: 342 MA: 342 49. If the nominal GDP is $477 billion and the velocity of money is 4.5, then the money supply is: A) $122 billion. B) $98 billion. C) $106 billion. D) $477 billion. Answer: C

Type: A Topic: 3 E: 342 MA: 342 50. According to monetarists, a change in the money supply changes: A) the velocity of money which in turn changes the nominal GDP. B) investment spending which in turn changes the nominal GDP. C) the interest rate which in turn changes the nominal GDP. D) aggregate demand which in turn changes the nominal GDP. Answer: D

Type: A Topic: 3 E: 342 MA: 342 51. Assume monetary equilibrium exists--that is, the desired and the actual supply of money are equal--when nominal GDP equals $480 billion and the money supply is $160 billion. According to a strict monetarist view, an increase in the money supply of $10 billion will increase the nominal GDP by: A) $30 billion. B) $25 billion. C) $20 billion. D) $10 billion. Answer: A

Type: A Topic: 2 E: 342 MA: 342 52. According to monetarists, the Great Depression in the United States largely resulted from: A) contractionary fiscal policy. B) excessive imports relative to exports. C) significant changes in technology and resource availability. D) inappropriate monetary policy. Answer: D

Type: A Topic: 3 E: 342 MA: 342 53. The view that inappropriate monetary policy was a main reason for the depth of the Great Depression in the United States is most closely associated with: A) monetarism. C) the rational expectations theory. B) Keynesianism D) the real-business-cycle theory. Answer: A

McConnell/Brue: Economics, 16/e

Page 555

Chapter 19: Disputes over Macro Theory and Policy

Real-business cycle view

Type: D Topic: 4 E: 342 MA: 342 54. The real-business-cycle theory holds that business fluctuations are caused by: A) factors affecting aggregate demand. B) incorrectly anticipated government stabilization policies. C) factors affecting aggregate supply. D) "stop-and-go" monetary policies. Answer: C

Type: D Topic: 4 E: 342 MA: 342 55. According to real business cycle theory: A) monetary factors affecting aggregate demand cause macroeconomic instability. B) recessions result from declines in long-run aggregate supply, rather than decreases in aggregate demand. C) when real wages fall during recessions, "real" unemployment rates rise. D) the net long-run costs of business fluctuations are severe. Answer: B

Type: D Topic: 4 E: 342 MA: 342 56. In the real business cycle theory: A) declines in real output cause declines in the money supply and thus aggregate demand. B) decreases in long-run aggregate supply are fully anticipated and therefore do not reduce real output. C) technology is constant. D) economic instability results from inappropriate monetary policy. Answer: A Use the following to answer questions 57-58:

Type: G Topic: 4 E: 343 MA: 343 57. Refer to the above diagram. A decline of aggregate supply from ASLR1 to ASLR2, followed by a decline of aggregate demand from AD1 to AD2, would best describe the: A) direct relationship between aggregate supply and aggregate demand. B) real-business-cycle view of recession. C) monetarist view of recession. D) mainstream, Keynesian-based, view of recession. Answer: B

McConnell/Brue: Economics, 16/e

Page 556

Chapter 19: Disputes over Macro Theory and Policy

Type: G Topic: 4 E: 343 MA: 343 58. Refer to the above diagram. The real-business cycle view of recession would best be described by: A) a decrease of aggregate demand from AD1 to AD2, followed by a decrease in aggregate supply from ASLR1 to ASLR2. B) an increase in aggregate demand from AD1 to AD2, which in turn caused a decrease in aggregate supply from ASLR1 to ASLR2. C) a decrease in aggregate supply from ASLR1 to ASLR2, followed by a decrease in aggregate demand from AD1 to AD2. D) a decrease in aggregate supply from ASLR1 to ASLR2, followed by an increase in aggregate demand from AD2 to AD1. Answer: C

Type: G Topic: 4 E: 343 MA: 343 59. The real-business-cycle theory: A) is a monetarist view of the business cycle. B) is a Keynesian-view of the business cycle. C) assumes that the supply of money is constant. D) says that macro instability results from shifts in the long-run aggregate supply curve. Answer: D

Type: A Topic: 4 E: 342 MA: 342 60. Which of the following is not an aggregate-demand-side explanation of business cycles? A) the real-business-cycle theory B) the idea of coordination failures C) mainstream (Keynesian-based) macroeconomics D) monetarism Answer: A

Coordination failures

Type: A Topic: 5 E: 343 MA: 343 61. When most consumers and firms reduce spending only because they expect other consumers and firms to reduce spending, and a recession results: A) a self-correction has occurred. B) an adverse aggregate supply shock has occurred. C) a coordination failure has occurred. D) a real-business-downturn has occurred. Answer: C

Type: D Topic: 5 E: 343 MA: 343 62. A coordination failure: A) is a real-business-cycle event. B) is a self-fulfilling prophesy. C) results from the spending-income multiplier. D) is a direct outcome of inappropriate fiscal policy. Answer: B

McConnell/Brue: Economics, 16/e

Page 557

Chapter 19: Disputes over Macro Theory and Policy

Type: A Topic: 5 E: 343 MA: 343 63. The idea that an economy can get stuck in either an unemployment equilibrium or an inflation equilibrium is most closely associated with: A) new classical economics. C) monetarism. B) the real-business-cycle theory. D) the idea of coordination failures. Answer: D

Type: A Topic: 5 E: 343 MA: 343 64. Assume that many households and businesses reduce their spending only because they expect other households and consumers to reduce their spending. Also suppose that all households and consumers would be better off if they did not reduce their spending. This situation best describes the: A) real-business-cycle theory. C) idea of coordination failures. B) rational expectations theory. D) adaptive expectations theory. Answer: C

New classical view of self-correction

Type: A Topic: 6 E: 344 MA: 344 65. New classical economists: A) stress the importance of federal budget deficits in stimulating aggregate demand. B) hold that, left alone, the economy gravitates to its full employment level of output. C) emphasize tax custs as means of increasing aggregate supply. D) advocate active use of monetary policy to stabilize the economy. Answer: B

Type: A Topic: 6 E: 344 MA: 344 66. Rational expectations theory is based on the assumption that: A) wages and prices are flexible upward, but inflexible downward. B) both product and resource markets are very competitive. C) product markets are competitive, but resource markets are monopolistic. D) both product and resource markets are monopolistic. Answer: B

Type: A Topic: 5 E: 344 MA: 344 67. Rational expectations theory implies that the: A) aggregate demand curve is vertical. B) long-run aggregate supply curve is vertical. Answer: B

C) long-run aggregate supply curve is horizontal. D) long-run aggregate supply curve is quite flat.

McConnell/Brue: Economics, 16/e

Page 558

Chapter 19: Disputes over Macro Theory and Policy

Use the following to answer questions 68-70:

Type: G Topic: 6 E: 344 MA: 344 68. Refer to the above diagram. Rational expectations theory say that a fully anticipated shift in aggregate demand from AD1 to AD2 will: A) move the economy from a to b to c. B) move the economy directly from a to c. C) move the economy from a to new equilibrium at b. D) shift the AS curve to the right. Answer: B

Type: G Topic: 6 E: 344 MA: 344 69. Refer to the above diagram. Rational expectations theory says that a fully anticipated decrease in aggregate demand from AD2 to AD1 will: A) move the economy from a to b to c. C) move the economy from c to new equilibrium b. B) shift the AS curve to the left. D) move the economy directly from c to a. Answer: D

Type: G Topic: 6 E: 344 MA: 344 70. Refer to the above diagram. Suppose that, as expected, aggregate demand declines from AD2 to AD1. A direct move of the economy from c to a would best reflect: A) new classical economics. C) the real-business-cycle theory. B) mainstream (Keynesian-based) economics. D) a coordination failure. Answer: A

Type: F Topic: 6 E: 344 MA: 344 71. The rational expectations theory assumes that: A) people behave rationally and that all product and resource prices are flexible both upward and downward. B) firms pay above-market wages to elicit work effort. C) markets fail to coordinate the actions of households and businesses. D) markets are dominated by monopolistic firms. Answer: A

McConnell/Brue: Economics, 16/e

Page 559

Chapter 19: Disputes over Macro Theory and Policy

Type: A Topic: 6 E: 345 MA: 345 72. According to new classical economists, the: A) short-run demand for labor curve is vertical. B) short-run aggregate demand curve is vertical. Answer: D

C) long-run aggregate supply curve is horizontal. D) long-run aggregate supply curve is vertical.

Type: A Topic: 6 E: 345 MA: 345 73. New classical economist say that an unanticipated increase in aggregate demand first: A) increases the price level and real output, and then reduces short-run aggregate supply such that the economy returns to the full-employment level of output. B) increases the price level and real output, and then increases long-run aggregate supply. C) increases long-run aggregate supply, and then increases the price level and real output. D) reduces short-run aggregate supply, and then reduces long-run aggregate supply. Answer: A

Type: A Topic: 6 E: 345 MA: 345 74. New classical economists say that an unanticipated decrease in aggregate demand first: A) decreases the price level and real output, and then decreases long-run aggregate supply. B) decreases long-run aggregate supply, and then decreases the price level and real output. C) reduces short-run aggregate supply, and then reduces long-run aggregate supply. D) decreases the price level and real output, and then increases short-run aggregate supply such that the economy returns to the full employment level of output. Answer: D

Type: A Topic: 6 E: 346 MA: 346 75. New classical economists say that a fully anticipated increase in aggregate demand: A) shifts the long-run aggregate supply curve to the right. B) shifts the long-run aggregate supply curve to the left. C) moves the economy up along its vertical long-run aggregate supply curve. D) eventually results in a self-correcting decrease in aggregate demand. Answer: C

Type: A Topic: 6 E: 346 MA: 346 76. New classical economist say that an a fully anticipated decrease in aggregate demand: A) shifts the long-run aggregate supply curve to the right. B) shifts the long-run aggregate supply curve to the left. C) moves the economy down along its vertical long-run aggregate supply curve. D) eventually results in a self-correcting increase in aggregate demand. Answer: C

McConnell/Brue: Economics, 16/e

Page 560

Chapter 19: Disputes over Macro Theory and Policy

Use the following to answer questions 77-80:

Type: G Topic: 6 E: 345 MA: 345 77. Refer to the above figure and assume the economy initially is in equilibrium at point a. In the new classical theory, an unanticipated increase in aggregate demand from AD2 to AD1 would move the economy: A) directly from a to d. B) from a to b to d. C) from a to e to d. D) directly from a to f. Answer: C

Type: G Topic: 6 E: 346 MA: 346 78. Refer to the above figure and assume the economy initially is in equilibrium at point a. In the new classical theory, a fully anticipated increase in aggregate demand from AD2 to AD1 would move the economy: A) directly from a to d. B) from a to b to d. C) from a to e to d. D) directly from a to f. Answer: A

Type: G Topic: 6 E: 345 MA: 345 79. Refer to the above figure and assume the economy initially is in equilibrium at point a. In the new classical theory, an unanticipated decrease in aggregate demand from AD2 to AD3 would move the economy: A) directly from a to h. B) from a to g to h. C) directly from a to d. D) from a to c to h. Answer: D

Type: G Topic: 6 E: 346 MA: 346 80. Refer to the above figure and assume the economy initially is in equilibrium at point a. In the new classical theory, a fully anticipated decrease in aggregate demand from AD2 to AD3 would move the economy: A) directly from a to h. B) from a to g to h. C) directly from a to d. D) from a to c to h. Answer: A

Type: A Topic: 6 E: 345 MA: 345 81. In new classical economics, "a price-level surprise": A) has no effect on the economy. B) causes a temporary change in real output. C) causes a permanent change in real output. D) can never occur since people correctly anticipate the future. Answer: B

McConnell/Brue: Economics, 16/e

Page 561

Chapter 19: Disputes over Macro Theory and Policy

Type: A Topic: 6 E: 345 MA: 345 82. In new classical economics, the change in output caused by a "price-level surprise": A) is shown as a shift of the long-run aggregate supply curve. B) does not alter the rate of unemployment, even in the short-run. C) is soon reversed through a shift of the short-run aggregate supply curve. D) permanently reduces the rate of unemployment. Answer: C

Type: A Topic: 6 E: 345-346 MA: 345-346 83. Suppose that, as expected, aggregate demand in the economy sharply declines. New classical economists say that the price level will _____________ and real output will ____________. A) fall; remain constant. B) rise; remain constant. C) remain constant; fall. D) remain constant; rise. Answer: A

Mainstream view of self correction

Type: A Topic: 7 E: 346 MA: 346 84. Mainstream economists question the new classical assumption that: A) excessive growth of the money supply is a cause of inflation. B) the price level is determined by aggregate demand and aggregate supply. C) demand creates its own supply. D) wages and prices are equally flexible upward and downward. Answer: D

Type: A Topic: 7 E: 346 MA: 346 85. Suppose aggregate demand in the economy sharply declines. Mainstream economists say that the price level (at least for a time) will _______ and real output will _________. A) decrease; remain constant. C) remain constant; decrease. B) increase; remain constant. D) remain constant; increase. Answer: C

McConnell/Brue: Economics, 16/e

Page 562

Chapter 19: Disputes over Macro Theory and Policy

Use the following to answer questions 86-87:

Type: G Topic: 7 E: 346 MA: 346 86. Refer to the above diagram and assume the economy initially is in equilibrium at point a. In the mainstream view, a decline in aggregate demand from AD1 to AD2 would likely move the economy: A) directly from a to d. C) from a to c, then quickly from c to d. B) directly from a to b. D) from a to c, then eventually from c to b. Answer: B

Type: G Topic: 7 E: 346 MA: 346 87. Refer to the above diagram and assume the economy initially is in equilibrium at point a. Suppose the aggregate demand declines from AD1 to AD2 and the economy moves from a to c In the mainstream view, the resulting decline in the price level need not shift the short-run aggregate supply curve from AS1 to AS2 because: A) supply creates its own demand. B) nominal wages are (at least for a time) inflexible downward. C) firms misperceive the price-level decline as being permanent. D) deflation reduces the purchasing power of the dollar. Answer: B

Type: A Topic: 7 E: 346 MA: 346 88. If prices and wages are inflexible downward, a decrease in aggregate demand will: A) reduce the price level but not real output. C) decrease short-run aggregate supply. B) increase short-run aggregate supply. D) reduce real output but not the price level. Answer: D

Efficiency wages; insider-outsider theory

Type: D Topic: 8 E: 346 MA: 346 89. An efficiency wage is: A) a below-market wage. B) an above-market wage. C) a "wage" that contains a profit-sharing component. D) a wage that is free to rise or fall from day to day, depending on labor supply and demand. Answer: B

McConnell/Brue: Economics, 16/e

Page 563

Chapter 19: Disputes over Macro Theory and Policy

Type: D Topic: 8 E: 346 MA: 346 90. An efficiency wage is: A) a wage payment necessary to compensate workers for risk of injury on the job. B) a "wage" that contains a profit-sharing component as well as traditional hourly pay. C) an above-market wage that minimizes a firm's labor cost per unit of output. D) a wage that automatically rises with the national index of labor productivity. Answer: C

Use the following to answer questions 91-92:
Hourly wage rate $10 9 8 7 6 Output per hour of work 6 6 4 2 1

Type: T Topic: 8 E: 346 MA: 346 91. Refer to the above table. At the $8 wage, labor cost per-unit of output is: A) $1.25. B) $1.50. C) $2.00. D) $1.67. Answer: C

Type: T Topic: 8 E: 346 MA: 346 92. Refer to the above table. The efficiency wage is: A) $10. B) 9. C) 8 D) 6. Answer: B

Type: T Topic: 8 E: 346-347 MA: 346-347 93. A higher wage could result in a lower labor cost per unit of output than a lower wage if the higher wage: A) brings forth greater work effort. C) increases job turnover. B) increases supervision costs. D) increases worker absenteeism. Answer: A

Type: A Topic: 8 E: 347 MA: 347 94. A higher wage could result in a lower labor cost per unit of output than a lower wage if the higher wage: A) is accompanied by an offsetting decline in fringe benefits. B) increases supervision costs. C) reduces job turnover. D) increases worker absenteeism. Answer: C

Type: A Topic: 8 E: 347 MA: 347 95. If firms are paying efficiency wages, they: A) may be reluctant to increase nominal wages when aggregate demand increases. B) are highly vulnerable to import competition. C) may be targeted for takeover by firms paying market wages. D) may be reluctant to cut wages when aggregate demand declines. Answer: D

McConnell/Brue: Economics, 16/e

Page 564

Chapter 19: Disputes over Macro Theory and Policy

Type: D Topic: 8 E: 347 MA: 347 96. In the insider-outsider theory: A) insiders are workers who retain employment during recession. B) insiders are managers who have more information abut their firms' performance than outsiders. C) insiders are "principals" and outsiders are "agents." D) outsiders are foreigners. Answer: A

Type: D Topic: 8 E: 347 MA: 347 97. In the insider-outsider theory: A) outsiders are workers who retain employment during recession. B) insiders are managers who have more information about their firms' performance than outsiders. C) insiders are "principals" and outsiders are "agents." D) outsiders are laid off workers and other qualified unemployed workers. Answer: D

Type: A Topic: 8 E: 347 MA: 347 98. Suppose laid off workers and other qualified unemployed workers offer to work for less than the wages being paid existing employed workers, but employers do not hire these workers for fear that existing workers will refuse to cooperate with them. This situation best describes the: A) efficiency wage theory. C) insider-outsider theory. B) theory of compensating wage differentials. D) rational expectations theory. Answer: C

Type: A Topic: 8 E: 347 MA: 347 99. The insider-outsider theory implies that: A) wages are flexible both upward and downward. B) unemployment quickly reduces market wages. C) agents pursue their own agendas, sometimes at the expense of principals. D) wages may be inflexible downward. Answer: D

Type: A Topic: 8 E: 346-347 MA: 346-347 100. Which of the following pairs help explain why self-correction from a decline in aggregate demand in the economy may be slow rather than rapid? A) theory of compensation wage differentials; theory of derived demand for labor. B) efficiency wage theory; insider-outsider theory. C) insider-outsider theory; principle-agent problem. D) externalities; efficiency wage theory. Answer: B

McConnell/Brue: Economics, 16/e

Page 565

Chapter 19: Disputes over Macro Theory and Policy

Policy rules; critiques of stabilization policy

Type: D Topic: 9 E: 348 MA: 348 101. The traditional monetary rule is the idea that: A) the annual rate of increase in the money supply should be equal to the potential annual growth rate of real GDP. B) the annual rate of increase in the money supply should be equal to the long-term increase in the price level. C) an expansionary fiscal policy should always be accompanied by an easy monetary policy. D) monetary policy only affects the economy 6 to 9 months after the money supply is changed. Answer: A

Type: F Topic: 9 E: 348 MA: 348 102. Adherents of the traditional monetary rule advocate that the: A) functional finance approach to fiscal policy be adopted. B) money supply should be increased by a constant rate year after year. C) money supply should be reduced during inflation and increased during recession. D) money supply should be increased during inflation and reduced during recession. Answer: B

Type: F Topic: 9 E: 348 MA: 348 103. Adherents of the traditional monetary rule say that the supply of money should be: A) increased at a constant rate each year. B) decreased during recession and increased during inflation. C) held constant over time. D) increased during recession and decreased during inflation. Answer: A

Use the following to answer question 104:

Type: G Topic: 9 E: 348 MA: 348 104. Answer this question on the basis of the above diagram and the equation of exchange. Assume that the velocity of money is constant at 4. Suppose that the increase of aggregate supply from AS1 to AS2 indicates the economy's average increase in real output per year. According to monetarists, the proper monetary rule for price stability would be to increase the money supply by: A) zero percent per year. B) 4 percent per year. C) 10 percent per year. D) 30 percent per year. Answer: D

McConnell/Brue: Economics, 16/e

Page 566

Chapter 19: Disputes over Macro Theory and Policy

Type: A Topic: 9 E: 349 MA: 349 105. Monetarists and rational expectations theorists generally agree that: A) the Federal Reserve should adhere to a monetary rule. B) the rate of interest and the price of bonds are positively or directly related. C) the money supply cannot be measured and therefore cannot be controlled by the Federal Reserve. D) prices and wages are inflexible downward. Answer: A

Type: A Topic: 9 E: 349 MA: 349 106. According to monetarists, an expansionary fiscal policy: A) will be ineffective because the interest rate will rise and crowd out private investment spending. B) should not be permitted so long as a public debt exists. C) should be used only when unemployment exceeds 6 percent of the labor force. D) will be effective, provided the money supply is held constant. Answer: A

Type: D Topic: 9 E: 349 MA: 349 107. The crowding-out effect refers to the possibility that: A) when used simultaneously, expansionary fiscal and monetary policies are counterproductive. B) the asset demand for money varies inversely with the interest rate. C) deficit financing will increase the interest rate and reduce investment. D) an increase in the supply of money will result in a decline in velocity. Answer: C

Type: A Topic: 9 E: 349 MA: 349 108. According to monetarists, an expansionary fiscal policy is a weak stabilization tool because: A) the asset demand for money varies inversely with the rate of interest. B) government borrowing to finance a deficit will raise the interest rate and reduce private investment. C) government borrowing will reduce the supply of money in circulation and depress the GDP. D) government borrowing to finance a deficit will lower interest rates, increase money balances, and lower velocity. Answer: B

Type: A Topic: 9 E: 349 MA: 349 109. Assume there is an increase in government spending and a reduction in net taxes. With a specific money supply, the consequent: A) contractionary impact might be lessened by the resulting increase in the interest rate. B) expansionary impact might be lessened by the resulting increase in the interest rate. C) contractionary impact might be enhanced by the resulting decline in the interest rate. D) expansionary impact might be enhanced by the resulting decline in the interest rate. Answer: B

Type: A Topic: 9 E: 349 MA: 349 110. According to monetarists, a fiscal deficit will be associated with an increase in real output: A) regardless of the character of accompanying changes in M or V . B) only if it is accompanied by an increase in the demand for money. C) only if it is accompanied by an increase in the supply of money. D) only if it is financed by selling government bonds to the public. Answer: C

McConnell/Brue: Economics, 16/e

Page 567

Chapter 19: Disputes over Macro Theory and Policy

Type: A Topic: 9 E: 349 MA: 349 111. The rational expectations perspective suggests that: A) fiscal policy is more powerful than monetary policy. B) monetary policy is more powerful than fiscal policy. C) fiscal and monetary policy are not likely to achieve their stated aims. D) fiscal policy works only to the extent that it is accompanied by fully anticipated changes in the money supply. Answer: C

Type: A Topic: 9 E: 349 MA: 349 112. The theory of rational expectations concludes that: A) the public's expectations can influence the outcome of monetary policy, but not of fiscal policy. B) the public's expectations can influence the outcome of fiscal policy, but not of monetary policy. C) the public's expectations as to the effects of economic policies tends to reinforce the effectiveness of those policies. D) by reacting in its self-interest to the expected effects of stabilization policy, the public tends to negate the impact of those policies. Answer: D

Type: A Topic: 9 E: 349 MA: 349 113. In comparing monetarism and rational expectations theory we find that: A) both favor policy rules and for the same reasons. B) both favor policy rules, but for different reasons. C) both favor discretionary policies. D) the former favors discretionary policy, while the latter favors policy rules. Answer: B

Type: F Topic: 9 E: 349 MA: 349 Status: New 114. Over recent years, economists holding monetarist views have replaced their call for a monetary rule with a call for: A) artful Fed management of interest rates. C) nominal GDP targeting. B) inflation targeting. D) inflationary and recessionary gap analysis. Answer: B

Type: A Topic: 9 E: 349 MA: 349 Status: New 115. Proponents of inflation targeting generally think that: A) the economy will have fewer, shorter, and less severe business cycles if the Fed holds the rate of inflation to low, targeted levels from year to year. B) low interest rates are inflationary and high interest rates are deflationary. C) fiscal policy is more effective in stabilizing the economy than monetary policy. D) the Fed should strive to achieve zero inflation. Answer: A

Type: A Topic: 9 E: 349 MA: 349 Status: New 116. Which of the following groups of economists is most likely to favor annually balanced Federal budgets? A) Keynesians C) Rational expectations economists. B) Supply-side economists. D) Functional finance economists. Answer: C

McConnell/Brue: Economics, 16/e

Page 568

Chapter 19: Disputes over Macro Theory and Policy

Defense of discretionary stabilization policy

Type: A Topic: 10 E: 350 MA: 350 117. According to mainstream economists, the Fed's adherence to a traditional monetary rule rather than to discretionary monetary policy is likely to: A) reduce the severity of business cycles. B) increase the amount of instability in the economy. C) increase the rate of inflation. D) crowd out much needed investment spending during times of rapid inflation. Answer: B

Type: A Topic: 10 E: 350 MA: 350 118. According to mainstream economists, a tight money policy might be frustrated, wholly or in part, by: A) Treasury sales of gold bullion. B) a Treasury surplus. C) the desire of households and businesses to hold smaller money balances. D) a decrease in V. Answer: C

Type: A Topic: 10 E: 350 MA: 350 119. Mainstream economists favor: A) the use of discretionary monetary policy and fiscal policy. B) a monetary rule. C) a balance-budget amendment. D) wage and price controls. Answer: A

Type: A Topic: 10 E: 350 MA: 350 120. Mainstream economists contend that, as stabilization tools: A) discretionary fiscal policy is effective, but discretionary monetary policy is not. B) discretionary monetary policy is effective, but discretionary fiscal policy is not. C) both discretionary fiscal policy and monetary policy can be effective if appropriately used. D) discretionary fiscal policy and discretionary monetary policy cause more instability than they cure. Answer: C

Type: A Topic: 10 E: 350 MA: 350 121. Most mainstream macroeconomists oppose a strict requirement to balance the Federal budget annually because they conclude that such a requirement would: A) increase real interest rates and drive out investment spending. B) eliminate monetary policy as a stabilization tool. C) force government to undertake expansionary fiscal policy during inflation and contractionary fiscal policy during recession. D) expand the size of the Federal government. Answer: C

McConnell/Brue: Economics, 16/e

Page 569

Chapter 19: Disputes over Macro Theory and Policy

Type: F Topic: 10 E: 350 MA: 350 122. Since 1946: A) the U.S. economy has been more stable than in the 1895-1945 period. B) the U.S. economy has been less stable than in the 1985-1945 period. C) fiscal policy has largely replaced monetary policy as the United States' main stabilization tool. D) new classical economics has replaced Keynesian-based economics as the mainstream macroeconomic view. Answer: A

Absorption into the mainstream

Type: A Topic: 11 E: 351 MA: 351 123. Which of the following ideas of the rational expectations theory has been absorbed into mainstream macroeconomics? A) the monetary rule C) the monetary multiplier B) the idea that "money doesn't matter" D) the idea that "expectations are important." Answer: D

Type: F Topic: 11 E: 351 MA: 351 124. The view that excessive growth to the money supply over long periods leads to inflation: A) is accepted by the monetarists but not by mainstream macroeconomists. B) is the main contribution of the rational expectations theory. C) had been absorbed into the mainstream of macroeconomics. D) is known as the monetary rule. Answer: C

Type: F Topic: 11 E: 351 MA: 351 125. Mainstream macroeconomics has embraced the: A) rational expectations view that stabilization policy is totally ineffective. B) monetarist view that the Fed should increase the money supply at a fixed annual rate. C) rational expectations view that expectations can shift the aggregate demand and aggregate supply curves. D) monetarist view that an increase in government spending crowds out an equal amount of investment spending. Answer: C

Type: F Topic: 11 E: 351 MA: 351 126. Modern mainstream macroeconomists agree with the monetarists that: A) the Fed should increase the money supply at a fixed annual rate. B) velocity is highly stable. C) fiscal policy is largely ineffective. D) "money matters" in the macroeconomy. Answer: D

McConnell/Brue: Economics, 16/e

Page 570

Chapter 19: Disputes over Macro Theory and Policy

Consider This Questions

Type: A E: 348 MA: 348 Status: New 127. (Consider This) According to economist Abba Lerner (1903-1982), fiscal and monetary policy is analogous to: A) automobile brakes. C) a string that can be pushed or pulled. B) a steering wheel in an automobile. D) highway guard rails. Answer: B

Type: A E: 348 MA: 348 Status: New 128. (Consider This) According to economist Milton Friedman (b. 1912), the source of instability in the economy could be thought of as a: A) baseball manager (the Fed) that removes his starting pitcher too soon and sees a 5-run lead evaporate in a single inning. B) duck hunter (the Fed) who starts shooting at ducks well before they fly over. C) a camp councilor (the Fed) who is wearing a baseball cap that has two bills and says "I am the leader, which way did they go?" D) backseat car passenger (the Fed) who occasionally leans over the front seat and abruptly jerks the steering wheel to the left or to the right. Answer: D

Last Word Questions

Type: D E: 351 MA: 351 129. (Last Word) The proposed monetary rule that would specify how the Fed should respond to changes in GDP and inflation rates is called the: A) Keynesian rule. B) Friedman rule. C) Taylor rule. D) Lucas rule. Answer: C

Type: F E: 351 MA: 351 130. (Last Word) The key policy target in the Taylor rule is the: A) money supply. B) Federal funds interest rate. C) average tax rate. D) full-employment budget. Answer: B

Type: A E: 351 MA: 351 131. (Last Word) According to the Taylor rule, if real GDP rises 1 percent above potential GDP, the Fed should raise the Federal funds rate, relative to the current rate of inflation, by: A) .5 percentage points. B) 1 percentage points. C) 1.5 percentage points. D) 2 percentage points. Answer: A

Type: A E: 351 MA: 351 132. (Last Word) According to the Taylor rule, if inflation rises 1 percent above a target rate of 2 percent, the Fed should raise the Federal funds rate, relative to the current rate of inflation, by: A) .5 percentage points. B) 1 percentage point. C) 1.5 percentage points. D) 2 percentage points. Answer: A

McConnell/Brue: Economics, 16/e

Page 571

Chapter 19: Disputes over Macro Theory and Policy

Type: D E: 351 MA: 351 133. (Last Word) Suppose that real GDP falls to 2 percent below potential GDP. The, according to the Taylor rule, the Fed should reduce the Federal funds, relative to the current rate of inflation, by: A) 1/2 percentage point. B) 1 percentage point. C) 2 percentage points. D) 4 percentage points. Answer: B

True/False Questions

Type: A E: 339 MA: 339 134. In classical theory, the full-employment output is determined by the location of the aggregate supply curve while the price level is determined by the location of the aggregate demand curve. Answer: True

Type: F E: 340 MA: 340 135. In its simplest form, Keynesian theory views the aggregate demand curve as stable and the aggregate supply curve as vertical Answer: False

Type: A E: 341 MA: 341 136. Monetarists say the velocity of money is highly variable and there is no close link between the money supply and the level of economic activity. Answer: False

Type: A E: 341 MA: 341 137. According to monetarists, discretionary monetary policy has been a major source of economic instability. Answer: True

Type: F E: 341 MA: 341 138. The equation of exchange is MV = PQ. Answer: True

Type: F E: 346 MA: 346 139. Mainstream macroeconomists see two main sources of macroeconomic instability: changes in investment spending and, occasionally, adverse aggregate supply shocks. Answer: True

Type: F E: 343 MA: 343 140. In the theory of coordination failures, shifts of the nation's long-run aggregate supply curve are the main cause of business cycles. Answer: False

Type: A E: 342-343 MA: 342-343 141. The real-business cycle theorists see aggregate supply as the "active" factor in causing business cycles and aggregate demand as a "passive" factor. Answer: True

McConnell/Brue: Economics, 16/e

Page 572

Chapter 19: Disputes over Macro Theory and Policy

Type: A E: 342 MA: 342 142. The "real" factors in the real-business cycle theory include resource availability and technology. Answer: True

Type: F E: 343 MA: 343 143. The idea of coordination failures suggests the possibility of less-than-desirable price-level and real-output equilibriums in the economy. Answer: True

Type: A E: 344 MA: 344 144. The idea that the economy will "self-correct" when confronted with changes in aggregate demand is associated with new classical economics. Answer: True

Type: A E: 346 MA: 346 145. In the new classical theory, a fully anticipated change in aggregate demand and the price level will temporarily change real output, but an unanticipated change will not. Answer: False

Type: A E: 346 MA: 346 146. Mainstream economists say the recessions are unlikely to occur today because prices and wages are highly flexible downward. Answer: False

Type: A E: 346 MA: 346 147. Efficiency wage theory says that an above-market wage can reduce labor costs per unit of output by eliciting greater work effort, lowering supervision costs, and reducing job turnover. Answer: True

Type: F E: 347 MA: 347 148. In the insider-outsider theory, insiders are agents and outsiders are principals. Answer: False

Type: A E: 348 MA: 348 149. Nearly all modern economists support the idea of a monetary rule. Answer: False

Type: A E: 349 MA: 349 150. Monetarist say that fiscal policy, such as a tax cut, will only affect the level of real GDP if it entails a change in the supply of money. Answer: True

McConnell/Brue: Economics, 16/e

Page 573

Chapter 19: Disputes over Macro Theory and Policy

Type: A E: 351 MA: 351 151. Mainstream economists contend that use of stabilization policy has helped increase macro stability in the U.S. economy over the past 40 years. Answer: True

Type: F E: 351 MA: 351 152. Mainstream macroeconomics has incorporated some aspects of monetarism and rationale expectations theory. Answer: True

McConnell/Brue: Economics, 16/e

Page 574

CHAPTER 20

Elasticity of Demand and Supply

Topic 1. 2. 3. 4. 5. 6. Price elasticity of demand Total revenue test Determinants of price elasticity Elasticity of supply Applications of price elasticity Cross and income elasticity Consider This Last Word True-False

Question numbers 1-39 40-75 76-93 94-125 126-144 145-167 168-169 170-172 173-192

____________________________________________________________

_______________________________________

____________________________________________________________

_______________________________________

Multiple Choice Questions Price elasticity of demand

Type: D Topic: 1 E: 356 MI: 112 1. The price elasticity of demand coefficient measures: A) buyer responsiveness to price changes. B) the extent to which a demand curve shifts as incomes change. C) the slope of the demand curve. D) how far business executives can stretch their fixed costs. Answer: A

Type: E Topic: 1 E: 356 MI: 112 2. The basic formula for the price elasticity of demand coefficient is: A) absolute decline in quantity demanded/absolute increase in price. B) percentage change in quantity demanded/percentage change in price. C) absolute decline in price/absolute increase in quantity demanded. D) percentage change in price/percentage change in quantity demanded. Answer: B

Type: D Topic: 1 E: 356 MI: 112 3. The demand for a product is inelastic with respect to price if: A) consumers are largely unresponsive to a per unit price change. B) the elasticity coefficient is greater than 1. C) a drop in price is accompanied by a decrease in the quantity demanded. D) a drop in price is accompanied by an increase in the quantity demanded. Answer: A

Chapter 20: Elasticity of Demand and Supply

Type: A Topic: 1 E: 357 MI: 113 4. If the price elasticity of demand for a product is 2.5, then a price cut from $2.00 to $1.80 will: A) increase the quantity demanded by about 2.5 percent. B) decrease the quantity demanded by about 2.5 percent. C) increase the quantity demanded by about 25 percent. D) increase the quantity demanded by about 250 percent. Answer: C

Type: A Topic: 1 E: 357 MI: 113 5. Suppose that as the price of Y falls from $2.00 to $1.90 the quantity of Y demanded increases from 110 to 118. Then the price elasticity of demand is: A) 4.00. B) 2.09. C) 1.37. D) 3.94. Answer: C

Type: D Topic: 1 E: 357 MI: 113 6. Which of the following is not characteristic of the demand for a commodity that is elastic? A) The relative change in quantity demanded is greater than the relative change in price. B) Buyers are relatively sensitive to price changes. C) Total revenue declines if price is increased. D) The elasticity coefficient is less than one. Answer: D

Type: A Topic: 1 E: 357 MI: 113 7. If the demand for product X is inelastic, a 4 percent increase in the price of X will: A) decrease the quantity of X demanded by more than 4 percent. B) decrease the quantity of X demanded by less than 4 percent. C) increase the quantity of X demanded by more than 4 percent. D) increase the quantity of X demanded by less than 4 percent. Answer: B

Type: A Topic: 1 E: 357 MI: 113 8. If a firm can sell 3,000 units of product A at $10 per unit and 5,000 at $8, then: A) the price elasticity of demand is 0.44. C) the price elasticity of demand is 2.25. B) A is a complementary good. D) A is an inferior good. Answer: C

Type: A Topic: 1 E: 357-358 MI: 113-114 9. A perfectly inelastic demand schedule: A) rises upward and to the right, but has a constant slope. B) can be represented by a line parallel to the vertical axis. C) cannot be shown on a two-dimensional graph. D) can be represented by a line parallel to the horizontal axis. Answer: B

McConnell/Brue: Economics, 16/e

Page 576

Chapter 20: Elasticity of Demand and Supply

Type: C Topic: 1 E: 357 MI: 113 10. The larger the coefficient of price elasticity of demand for a product, the: A) larger the resulting price change for an increase in supply. B) more rapid the rate at which the marginal utility of that product diminishes. C) less competitive will be the industry supplying that product. D) smaller the resulting price change for an increase in supply. Answer: D

Type: A Topic: 1 E: 359 MI: 115 11. Most demand curves are relatively elastic in the upper-left portion because the original price: A) and quantity from which the percentage changes in price and quantity are calculated are both large. B) and quantity from which the percentage changes in price and quantity are calculated are both small. C) from which the percentage price change is calculated is small and the original quantity from which the percentage change in quantity is calculated is large. D) from which the percentage price change is calculated is large and the original quantity from which the percentage change in quantity is calculated is small. Answer: D

Type: A Topic: 1 E: 357 MI: 113 12. The price elasticity of demand for widgets is 0.80. Assuming no change in the demand curve for widgets, a 16 percent increase in sales implies a: A) 1 percent reduction in price. C) 40 percent reduction in price. B) 12 percent reduction in price. D) 20 percent reduction in price. Answer: D

Type: A Topic: 1 E: 359 MI: 115 13. Suppose Aiyanna's pizzeria currently faces a linear demand curve and is charging a very high price per pizza and doing very little business. Aiyanna now decides to lower pizza prices by 5 percent per week for an indefinite period of time. We can expect that each successive week: A) demand will become more price elastic. B) price elasticity of demand will not change as price is lowered. C) demand will become less price elastic. D) the elasticity of supply will increase. Answer: C

Type: A Topic: 1 E: 358 MI: 114 14. The price elasticity of demand of a straight-line demand curve is: A) elastic in high-price ranges and inelastic on low-price ranges. B) elastic, but does not change at various points on the curve. C) inelastic, but does not change at various points on the curve. D) 1 at all points on the curve. Answer: A

Type: C Topic: 1 E: 359 MI: 115 15. A leftward shift in the supply curve of product X will increase equilibrium price to a greater extent the: A) more elastic the supply curve. C) more elastic the demand for the product. B) larger the elasticity of demand coefficient. D) more inelastic the demand for the product. Answer: D

McConnell/Brue: Economics, 16/e

Page 577

Chapter 20: Elasticity of Demand and Supply

Type: A Topic: 1 E: 359 MI: 115 16. If the demand for bacon is relatively elastic, a 10 percent decline in the price of bacon will: A) decrease the amount demanded by more than 10 percent. B) increase the amount demanded by more than 10 percent. C) decrease the amount demanded by less than 10 percent. D) increase the amount demanded by less than 10 percent. Answer: B

Type: A Topic: 1 E: 357 MI: 113 17. The price elasticity of demand is: A) negative, but the minus sign is ignored. B) positive, but the plus sign is ignored. C) positive for normal goods and negative for inferior goods. D) positive because price and quantity demanded are inversely related. Answer: A

Type: A Topic: 1 E: 359 MI: 115 18. For a linear demand curve: A) elasticity is constant along the curve. B) elasticity is unity at every point on the curve. Answer: D

C) demand is elastic at low prices. D) demand is elastic at high prices.

Type: A Topic: 1 E: 359 MI: 115 19. The price of product X is reduced from $100 to $90 and, as a result, the quantity demanded increases from 50 to 60 units. Therefore demand for X in this price range: A) has declined. B) is of unit elasticity. C) is inelastic. D) is elastic. Answer: D

Use the following to answer question 20:

Type: G Topic: 1 E: 359 MI: 115 20. The above diagram shows two product demand curves. On the basis of this diagram we can say that: A) over range P1P2 price elasticity of demand is greater for D1 than for D2. B) over range P1P2 price elasticity of demand is greater for D2 than for D1. C) over range P1P2 price elasticity is the same for the two demand curves. D) not enough information is given to compare price elasticities. Answer: A

McConnell/Brue: Economics, 16/e

Page 578

Chapter 20: Elasticity of Demand and Supply

Type: A Topic: 1 E: 359 MI: 115 21. Suppose we find that the price elasticity of demand for a product is 3.5 when its price is increased by 2 percent. We can conclude that quantity demanded: A) increased by 7 percent. C) decreased by 9 percent. B) decreased by 7 percent. D) decreased by 12 percent. Answer: B

Type: A Topic: 1 E: 359 MI: 115 22. The price elasticity of demand for beef is about 0.60. Other things equal, this means that a 20 percent increase in the price of beef will cause the quantity of beef demanded to: A) increase by approximately 12 percent. C) decrease by approximately 32 percent. B) decrease by approximately 12 percent. D) decrease by approximately 26 percent. Answer: B

Type: A Topic: 1 E: 359 MI: 115 23. The elasticity of demand: A) is infinitely large for a perfectly inelastic demand curve. B) tends to be inelastic in high-price ranges and elastic in low-price ranges. C) tends to be elastic in high-price ranges and inelastic in low-price ranges. D) is the same at each price-quantity combination on a stable demand curve. Answer: C

Type: D Topic: 1 E: 357 MI: 113 24. If a demand for a product is elastic, the value of the price elasticity coefficient is: A) zero. B) greater than one. C) equal to one. D) less than one. Answer: B

Type: D Topic: 1 E: 356 MI: 112 25. The concept of price elasticity of demand measures: A) the slope of the demand curve. B) the number of buyers in a market. C) the extent to which the demand curve shifts as the result of a price decline. D) the sensitivity of consumer purchases to price changes. Answer: D

McConnell/Brue: Economics, 16/e

Page 579

Chapter 20: Elasticity of Demand and Supply

Use the following to answer questions 26-28:

Type: G Topic: 1 E: 359 MI: 115 26. Refer to the above diagram. Between prices of $5.70 and $6.30: A) D1 is more elastic than D2. C) D1 and D2 have identical elasticities. B) D2 is an inferior good and D1 is a normal good. D) D2 is more elastic than D1 Answer: A

Type: G Topic: 1 E: 359 MI: 115 27. Refer to the above diagram and assume a single good. If the price of the good decreases from $6.30 to $5.70, consumer spending would: A) decrease if demand were D1 only. C) decrease if demand were either D1 or D2. B) decrease if demand were D2 only. D) increase if demand were either D1 or D2. Answer: B

Type: G Topic: 1 E: 359 MI: 115 28. Refer to the above diagram and assume a single good. If the price of the good increased from $5.70 to $6.30 along D1, the price elasticity of demand along this portion of the demand curve would be: A) 0.8. B) 1.0. C) 1.2. D) 2.0. Answer: C

Type: A Topic: 1 E: 359 MI: 115 29. Suppose the price of local cable TV service increased from $16.20 to $19.80 and as a result the number of cable subscribers decreased from 224,000 to 176,000. Along this portion of the demand curve, price elasticity of demand is: A) 0.8. B) 1.2. C) 1.6. D) 8.0 Answer: B

Type: A Topic: 1 E: 359 MI: 115 30. If the price of hand calculators falls from $10 to $9 and, as a result, the quantity demanded increases from 100 to 125, then: A) demand is elastic. B) demand is inelastic. C) demand is of unit elasticity. D) not enough information is given to make a statement about elasticity. Answer: A

McConnell/Brue: Economics, 16/e

Page 580

Chapter 20: Elasticity of Demand and Supply

Type: A Topic: 1 E: 358 MI: 114 31. A perfectly inelastic demand curve: A) has a price elasticity coefficient greater than unity. B) has a price elasticity coefficient of unity throughout. C) graphs as a line parallel to the vertical axis. D) graphs as a line parallel to the horizontal axis. Answer: C

Type: A Topic: 1 E: 359 MI: 115 32. Moving upward on a downward-sloping straight-line demand curve, we find that price elasticity: A) is constant. C) decreases continuously. B) increases continuously. D) may either increase or decrease. Answer: B

Use the following to answer questions 33-37: Answer the next question(s) on the basis of the following demand schedule:
Price $6 5 4 3 2 1 Quantity demanded 1 2 3 4 5 6

Type: T Topic: 1 E: 359 MI: 115 33. Refer to the above data. If this demand schedule were graphed, we would find that: A) its slope diminishes as we move southeast down the curve. B) its slope diminishes as we move northwest up the curve. C) its slope is constant throughout. D) the data is inconsistent with the law of demand. Answer: C

Type: T Topic: 1 E: 359 MI: 115 34. Refer to the above data. The price elasticity of demand is relatively elastic: A) in the $6-$4 price range. C) in the $3-$1 price range. B) over the entire $6-$1 price range. D) in the $6-$5 price range only. Answer: A

Type: T Topic: 1 E: 359 MI: 115 35. Refer to the above data. The price elasticity of demand is relatively inelastic: A) in the $6-$4 price range. C) in the $3-$1 price range. B) over the entire $6-$1 price range. D) in the $6-$5 price range only. Answer: C

McConnell/Brue: Economics, 16/e

Page 581

Chapter 20: Elasticity of Demand and Supply

Type: T Topic: 1 E: 359 MI: 115 36. Refer to the above data. The price elasticity of demand is unity: A) throughout the entire price range because the slope of the demand curve is constant. B) in the $4-$3 price range only. C) over the entire $3-$1 price range. D) over the entire $6-$4 price range. Answer: B

Type: T Topic: 1 E: 359 MI: 115 37. Refer to the above data. Which of the following is correct? A) Although the slope of the demand curve is constant, price elasticity declines as we move from high to low price ranges. B) Although the slope of the demand curve is constant, price elasticity increases as we move from high to low price ranges. C) Although the demand curve is concave to the origin, price elasticity of demand is constant throughout. D) A steep slope means demand is inelastic; a flat slope means demand is elastic. Answer: A

Type: A Topic: 1 E: 359 MI: 115 38. If the price elasticity of demand for gasoline is 0.20: A) the demand for gasoline is linear. B) a rise in the price of gasoline will reduce total revenue. C) a 10 percent rise in the price of gasoline will decrease the amount purchased by 2 percent. D) a 10 percent fall in the price of gasoline will increase the amount purchased by 20 percent. Answer: C

Type: T Topic: 1 E: 359 MI: 115 39. In which price range of the accompanying demand schedule is demand elastic?
Quantity demanded 2 4 6 8 C) $2-$1 D) below $1

Price $4 3 2 1

A) $4-$3 B) $3-$2 Answer: A

Total revenue test

Type: A Topic: 2 E: 361 MI: 117 40. When the percentage change in price is greater than the resulting percentage change in quantity demanded: A) a decrease in price will increase total revenue. C) an increase in price will increase total revenue. B) demand may be either elastic or inelastic. D) demand is elastic. Answer: C

McConnell/Brue: Economics, 16/e

Page 582

Chapter 20: Elasticity of Demand and Supply

Type: A Topic: 2 E: 360-361 MI: 116-117 41. Suppose the price elasticity coefficients of demand are 1.43, 0.67, 1.11, and 0.29 for products W, X, Y, and Z respectively. A 1 percent decrease in price will increase total revenue in the case(s) of: A) W and Y. B) Y and Z. C) X and Z. D) Z and W. Answer: A

Use the following to answer questions 42-44:

$20 18 16
Total revenue

14 12 10 8 6 4 2
1 2 3 4 5 6 7 Quantity demanded 8 TR

Type: G Topic: 2 E: 361 MI: 117 42. Suppose that the above total revenue curve is derived from a particular linear demand curve. That demand curve must be: A) inelastic for price declines that increase quantity demanded from 6 units to 7 units. B) elastic for price declines that increase quantity demanded from 6 units to 7 units. C) inelastic for price increases that reduce quantity demanded from 4 units to 3 units. D) elastic for price increases that reduce quantity demanded from 8 units to 7 units. Answer: A

Type: G Topic: 2 E: 360 MI: 116 43. Suppose that the above total revenue curve is derived from a particular linear demand curve. That demand curve must be: A) inelastic for price declines that increase quantity demanded from 2 units to 3 units. B) elastic for price declines that increase quantity demanded from 5 units to 6 units. C) inelastic for price increases that reduce quantity demanded from 4 units to 3 units. D) elastic for price increases that reduce quantity demanded from 4 units to 3 units. Answer: D

McConnell/Brue: Economics, 16/e

Page 583

Chapter 20: Elasticity of Demand and Supply

Type: G Topic: 2 E: 361 MI: 117 44. Suppose that the above total revenue curve is derived from a particular linear demand curve. That demand curve must be: A) inelastic for price declines that increase quantity demanded from 2 units to 3 units. B) elastic for price declines that increase quantity demanded from 5 units to 6 units. C) unit elastic for price increases that reduce quantity demanded from 5 units to 4 units.. D) inelastic for price increases that reduce quantity demanded from 4 units to 3 units. Answer: C

Type: A Topic: 2 E: 360 MI: 116 45. Which of the following statements is not correct? A) If the relative change in price is greater than the relative change in the quantity demanded associated with it, demand is inelastic. B) In the range of prices in which demand is elastic, total revenue will diminish as price decreases. C) Total revenue will not change if price varies within a range where the elasticity coefficient is unity. D) Demand tends to be elastic at high prices and inelastic at low prices. Answer: B

Type: A Topic: 2 E: 360 MI: 116 46. In which of the following instances will total revenue decline? A) price rises and supply is elastic C) price rises and demand is inelastic B) price falls and demand is elastic D) price rises and demand is elastic Answer: D

Type: A Topic: 2 E: 360-361 MI: 116-117 47. If a firm's demand for labor is elastic, a union-negotiated wage increase will: A) necessarily be inflationary. C) cause the firm's total payroll to decline. B) cause the firm's total payroll to increase. D) cause a shortage of labor. Answer: C

Type: A Topic: 2 E: 360-361 MI: 116-117 48. The Illinois Central Railroad once asked the Illinois Commerce Commission for permission to increase its commuter rates by 20 percent. The railroad argued that declining revenues made this rate increase essential. Opponents of the rate increase contended that the railroad's revenues would fall because of the rate hike. It can be concluded that: A) both groups felt that the demand was elastic but for different reasons. B) both groups felt that the demand was inelastic but for different reasons. C) the railroad felt that the demand for passenger service was inelastic and opponents of the rate increase felt it was elastic. D) the railroad felt that the demand for passenger service was elastic and opponents of the rate increase felt it was inelastic. Answer: C

McConnell/Brue: Economics, 16/e

Page 584

Chapter 20: Elasticity of Demand and Supply

Type: A Topic: 2 E: 360 MI: 116 49. If a firm finds that it can sell $13,000 of a product when its price is $5 per unit and $11,000 of it when its price is $6, then: A) the demand for the product is elastic in the $6-$5 price range. B) the demand for the product must have increased. C) elasticity of demand is 0.74. D) the demand for the product is inelastic in the $6-$5 price range. Answer: A

Type: C Topic: 2 E: 361 MI: 117 50. Suppose the price elasticity of demand for bread is 0.20. If the price of bread falls by 10 percent, the quantity demanded will increase by: A) 2 percent and total expenditures on bread will rise. B) 2 percent and total expenditures on bread will fall. C) 20 percent and total expenditures on bread will fall. D) 20 percent and total expenditures on bread will rise. Answer: B

Use the following to answer questions 51-52:

Type: G Topic: 2 E: 361 MI: 117 51. Refer to the above diagram which is a rectangular hyperbola, that is, a curve such that each rectangle drawn from any point on the curve will be of identical area. If this rectangular hyperbola was a demand curve, we could say that it would be: A) elastic at high prices and inelastic at low prices. C) impossible to generalize about its elasticity. B) elastic at low prices and inelastic at high prices. D) of unit elasticity throughout. Answer: D

Type: G Topic: 2 E: 361 MI: 117 52. Refer to the above diagram which is a rectangular hyperbola, that is, a curve such that each rectangle drawn from any point on the curve will be of identical area. In comparing the price elasticity and the slope of this demand curve we can conclude that the: A) slope of a demand curve measures its elasticity. B) elasticity of a demand curve measures its slope. C) slope and elasticity of the curve are both constant throughout. D) slope of the curve varies, but its elasticity is constant. Answer: D

McConnell/Brue: Economics, 16/e

Page 585

Chapter 20: Elasticity of Demand and Supply

Type: A Topic: 2 E: 361 MI: 117 53. Gigantic State University raises tuition for the purpose of increasing its revenue so that more faculty can be hired. GSU is assuming that the demand for education at GSU is: A) decreasing. B) relatively elastic. C) perfectly elastic. D) relatively inelastic. Answer: D

Type: A Topic: 2 E: 361 MI: 117 54. If the demand for farm products is price inelastic, a good harvest will cause farm revenues to: A) increase. B) decrease. C) be unchanged. D) either increase or decrease, depending on what happens to supply. Answer: B

Type: A Topic: 2 E: 360 MI: 116 55. Other things the same, if a price change causes total revenue to change in the opposite direction, demand is: A) perfectly inelastic. B) relatively elastic. C) relatively inelastic. D) of unit elasticity. Answer: B

Type: A Topic: 2 E: 361 MI: 117 56. If the price elasticity of demand for a product is unity, a decrease in price will: A) have no effect upon the amount purchased. B) increase the quantity demanded and increase total revenue. C) increase the quantity demanded, but decrease total revenue. D) increase the quantity demanded, but total revenue will be unchanged. Answer: D

Type: A Topic: 2 E: 361 MI: 117 57. If a price reduction reduces a firm's total revenue: A) the demand for the product is inelastic in this price range. B) the product is an inferior good. C) in this price range the elasticity coefficient of demand is greater than 1. D) this price decline will increase the firm's profits. Answer: A

Type: A Topic: 2 E: 361 MI: 117 58. In which of the following cases will total revenue increase? A) price falls and demand is inelastic C) price rises and demand is inelastic B) price falls and supply is elastic D) price rises and demand is elastic Answer: C

Type: A Topic: 2 E: 360-361 MI: 116-117 59. A manufacturer of frozen pizzas found that total revenue decreased when price was lowered from $5 to $4. It was also found that total revenue decreased when price was raised from $5 to $6. Thus, A) the demand for pizza is elastic above $5 and inelastic below $5. B) the demand for pizza is elastic both above and below $5. C) the demand for pizza is inelastic above $5 and elastic below $5. D) $5 is not the equilibrium price of pizza. Answer: A

McConnell/Brue: Economics, 16/e

Page 586

Chapter 20: Elasticity of Demand and Supply

Use the following to answer questions 60-61:

Type: G Topic: 2 E: 360 MI: 116 60. Refer to the above diagram. In the P1P2 price range demand is: A) of unit elasticity. B) relatively inelastic. C) relatively elastic. Answer: C

D) perfectly elastic.

Type: G Topic: 2 E: 360 MI: 116 61. Refer to the above diagram. In the P3P4 price range demand is: A) of unit elasticity. B) relatively inelastic. C) relatively elastic. Answer: B

D) perfectly elastic.

Type: A Topic: 2 E: 360 MI: 116 62. If the demand for a product is elastic, then total revenue will: A) increase whether price increases or decreases. C) fall as price falls. B) be constant in response to a price change. D) rise as price falls. Answer: D

Type: A Topic: 2 E: 361 MI: 117 63. The total-revenue test for elasticity: A) is equally applicable to both demand and supply. B) does not apply to demand because price and quantity are inversely related. C) does not apply to supply because price and quantity are directly related. D) applies to the short-run supply curve, but not to the long-run supply curve. Answer: C

Type: A Topic: 2 E: 361 MI: 117 64. If the University Chamber Music Society decides to raise ticket prices to provide more funds to finance concerts, the Society is assuming that the demand for tickets is: A) parallel to the horizontal axis. B) shifting to the left. C) inelastic. D) elastic. Answer: C

McConnell/Brue: Economics, 16/e

Page 587

Chapter 20: Elasticity of Demand and Supply

Type: A Topic: 2 E: 361 MI: 117 65. The state legislature has cut Gigantic State University's appropriations. GSU's Board of Regents decides to increase tuition and fees to compensate for the loss of revenue. The board is assuming that the: A) demand for education at GSU is elastic. B) demand for education at GSU is inelastic. C) coefficient of price elasticity of demand for education at GSU is unity. D) coefficient of price elasticity of demand for education at GSU is greater than unity. Answer: B

Type: A Topic: 2 E: 360 MI: 116 66. Which of the following is correct? A) If demand is elastic, an increase in price will increase total revenue. B) If demand is elastic, a decrease in price will decrease total revenue. C) If demand is elastic, a decrease in price will increase total revenue. D) If demand is inelastic, an increase in price will decrease total revenue. Answer: C

Type: A Topic: 2 E: 361 MI: 117 67. Suppose that the price of peanuts falls from $3 to $2 per bushel and that, as a result, the total revenue received by peanut farmers changes from $16 to $14 billion. Thus: A) the demand for peanuts is elastic. B) the demand for peanuts is inelastic. C) the demand curve for peanuts has shifted to the right. D) no inference can be made as to the elasticity of demand for peanuts. Answer: B

Type: A Topic: 2 E: 361 MI: 117 68. Which of the following is correct? A) If the demand for a product is inelastic, a change in price will cause total revenue to change in the opposite direction. B) If the demand for a product is inelastic, a change in price will cause total revenue to change in the same direction. C) If the demand for a product is inelastic, a change in price may cause total revenue to change in either the opposite or the same direction. D) The price elasticity coefficient applies to demand, but not to supply. Answer: B

McConnell/Brue: Economics, 16/e

Page 588

Chapter 20: Elasticity of Demand and Supply

Use the following to answer questions 69-72:

Type: G Topic: 2 E: 360 MI: 116 69. Refer to the above diagram. Total revenue at price P1 is indicated by area(s): A) C + D B) A + B C) A + C D) A Answer: B

Type: G Topic: 2 E: 360 MI: 116 70. Refer to the above diagram. If price falls from P1 to P2, total revenue will become area(s): A) B + D B) C + D C) A + C D) C Answer: A

Type: G Topic: 2 E: 360 MI: 116 71. Refer to the above diagram. The decline in price from P1 to P2 will: A) increase total revenue by D. C) decrease total revenue by A. B) increase total revenue by B + D. D) increase total revenue by D - A. Answer: D

Type: G Topic: 2 E: 360 MI: 116 72. Refer to the above diagram. In the P1 to P2 price range, we can say: A) that consumer purchases are relatively insensitive to price changes. B) nothing concerning price elasticity of demand. C) that demand is inelastic with respect to price. D) that demand is elastic with respect to price. Answer: D

McConnell/Brue: Economics, 16/e

Page 589

Chapter 20: Elasticity of Demand and Supply

Use the following to answer questions 73-75:

Type: G Topic: 2 E: 361 MI: 117 73. Refer to the above diagram. If price falls from $10 to $2, total revenue: A) rises from A + B to A+ B + D + C and demand is elastic. B) falls from A + D to B + C and demand is inelastic. C) rises from C + D to B + A and demand is elastic. D) falls from A + B to B + C and demand is inelastic. Answer: D

Type: G Topic: 2 E: 361 MI: 117 74. Refer to the above diagram and assume that price increases from $2 to $10. The coefficient of price elasticity of demand (midpoints formula) relating to this change in price is about: A) .25 and demand is inelastic. C) 1 and demand is unit elastic. B) 1.5 and demand is elastic. D) .67 and demand is inelastic. Answer: A

Type: G Topic: 2 E: 361 MI: 117 75. Refer to the above diagram and assume that price declines from $10 to $2. The coefficient of price elasticity of demand (midpoints formula) relating to this change in price is about: A) .25 and demand is inelastic. C) 1 and demand is unit elastic. B) 1.5 and demand is elastic. D) .67 and demand is inelastic. Answer: A

Determinants of price elasticity

Type: A Topic: 3 E: 362 MI: 118 76. The demands for such products as salt, bread, and electricity tend to be: A) perfectly price elastic. C) relatively price inelastic. B) of unit price elasticity. D) relatively price elastic. Answer: C

McConnell/Brue: Economics, 16/e

Page 590

Chapter 20: Elasticity of Demand and Supply

Type: A Topic: 3 E: 362-363 MI: 118-119 77. The elasticity of demand for a product is likely to be greater: A) if the product is a necessity, rather than a luxury good. B) the greater the amount of time over which buyers adjust to a price change. C) the smaller the proportion of one's income spent on the product. D) the smaller the number of substitute products available. Answer: B

Type: A Topic: 3 E: 362 MI: 118 78. We would expect: A) the demand for Coca-Cola to be less elastic than the demand for soft drinks in general. B) the demand for Coca-Cola to be more elastic than the demand for soft drinks in general. C) no relationship between the elasticity of demand for Coca-Cola and the elasticity of demand for soft drinks in general. D) none of the above to hold true. Answer: B

Type: A Topic: 3 E: 362 MI: 118 79. The narrower the definition of a product: A) the larger the number of substitutes and the larger the price elasticity of demand. B) the smaller the number of substitutes and the larger the price elasticity of demand. C) the larger the number of substitutes and the smaller the price elasticity of demand. D) the smaller the number of substitutes and the smaller the price elasticity of demand. Answer: A

Type: A Topic: 3 E: 362 MI: 118 80. Which of the following statements is correct? A) Supply is more elastic in the short run than in the long run. B) Demand is more elastic in the short run than in the long run. C) Demand is more elastic when a large number of substitute goods are available. D) Supply is more elastic when there are a small number of producers in the industry. Answer: C

Type: A Topic: 3 E: 362-363 MI: 118-119 81. The more time consumers have to adjust to a change in price: A) the smaller will be the price elasticity of demand. B) the greater will be the price elasticity of demand. C) the more likely the product is a normal good. D) the more likely the product is an inferior good. Answer: B

Type: A Topic: 3 E: 362 MI: 118 82. The demand for autos is likely to be: A) less elastic than the demand for Honda Accords. B) more elastic than the demand for Honda Accords. C) of the same elasticity as the demand for Honda Accords. D) perfectly inelastic. Answer: A

McConnell/Brue: Economics, 16/e

Page 591

Chapter 20: Elasticity of Demand and Supply

Type: A Topic: 3 E: 362-363 MI: 118-119 83. Price elasticity of demand is generally: A) greater in the long run than in the short run. B) greater in the short run than in the long run. C) the same in both the short run and the long run. D) greater for "necessities" than it is for "luxuries." Answer: A

Type: A Topic: 3 E: 362 MI: 118 84. Which of the following generalizations is not correct? A) The larger an item is in one's budget, the greater the price elasticity of demand. B) The price elasticity of demand is greater for necessities than it is for luxuries. C) The larger the number of close substitutes available, the greater will be the price elasticity of demand for a particular product. D) The price elasticity of demand is greater the longer the time period under consideration. Answer: B

Type: D Topic: 3 E: 358 MI: 114 85. A demand curve which is parallel to the vertical axis is: A) perfectly inelastic B) perfectly elastic C) relatively inelastic Answer: A

D) relatively elastic

Type: A Topic: 3 E: 358 MI: 114 86. If quantity demanded is completely unresponsive to price changes, demand is: A) perfectly inelastic B) perfectly elastic C) relatively inelastic D) relatively elastic Answer: A

Type: A Topic: 3 E: 362 MI: 118 87. If price and total revenue vary in opposite directions, demand is: A) perfectly inelastic B) perfectly elastic C) relatively inelastic Answer: D

D) relatively elastic

Type: D Topic: 3 E: 362 MI: 118 88. If the coefficient of price elasticity is less than 1 but greater than zero, demand is: A) perfectly inelastic B) perfectly elastic C) relatively inelastic D) relatively elastic Answer: C

Type: A Topic: 3 E: 362 MI: 118 89. The demand for a luxury good whose purchase would exhaust a significant portion of one's income is: A) perfectly inelastic B) perfectly elastic C) relatively inelastic D) relatively elastic Answer: D

Type: A Topic: 3 E: 362 MI: 118 90. The coefficient of price elasticity is 0.2. Demand is thus: A) perfectly inelastic B) perfectly elastic C) relatively inelastic Answer: D

D) relatively elastic

McConnell/Brue: Economics, 16/e

Page 592

Chapter 20: Elasticity of Demand and Supply

Type: A Topic: 3 E: 358 MI: 114 91. A firm can sell more or less output at a constant price. Demand is thus: A) perfectly inelastic B) perfectly elastic C) relatively inelastic D) relatively elastic Answer: B

Type: A Topic: 3 E: 362 MI: 118 92. The demand for a necessity whose cost is a small component of one's total income is: A) perfectly inelastic B) perfectly elastic C) relatively inelastic D) relatively elastic Answer: C

Type: D Topic: 3 E: 358 MI: 114 93. A demand curve which is parallel to the horizontal axis is: A) perfectly inelastic B) perfectly elastic C) relatively inelastic Answer: B

D) relatively elastic

Elasticity of supply

Type: D Topic: 4 E: 364 MI: 120 94. The price elasticity of supply measures how: A) easily labor and capital can be substituted for one another in the production process. B) responsive the quantity supplied of X is to changes in the price of X. C) responsive the quantity supplied of Y is to changes in the price of X. D) responsive quantity supplied is to a change in incomes. Answer: B

Type: A Topic: 4 E: 365 MI: 121 95. The main determinant of elasticity of supply is the: A) number of close substitutes for the product available to consumers. B) amount of time the producer has to adjust inputs in response to a price change. C) urgency of consumer wants for the product. D) number of uses for the product. Answer: B

Type: A Topic: 4 E: 366 MI: 122 96. Suppose the supply of product X is perfectly inelastic. If there is an increase in the demand for this product, equilibrium price: A) will decrease but equilibrium quantity will increase. B) and quantity will both decrease. C) will increase but equilibrium quantity will decline. D) will increase but equilibrium quantity will be unchanged. Answer: D

McConnell/Brue: Economics, 16/e

Page 593

Chapter 20: Elasticity of Demand and Supply

Use the following to answer questions 97-100:

Price $10 8 6 4 2

Quantity supplied 10 9 8 7 6

Type: T Topic: 4 E: 365 MI: 121 Status: New 97. Over the $6-$4 price range, supply is: A) perfectly elastic. B) elastic. C) perfectly inelastic. D) inelastic. Answer: D

Type: T Topic: 4 E: 365 MI: 121 Status: New 98. Over the $8-$6 price range, supply is: A) inelastic. B) elastic. C) perfectly inelastic. D) perfectly elastic. Answer: A

Type: T Topic: 4 E: 365 MI: 121 Status: New 99. Over the $10-$8 price range, the elasticity coefficient of supply is: A) 1. B) zero. C) less than 1. D) greater than 1. Answer: C

Type: T Topic: 4 E: 365 MI: 121 Status: New 100. Over the $4-$2 price range, the elasticity coefficient of supply is: A) 1. B) zero. C) less than 1. D) greater than 1. Answer: C

Type: A Topic: 4 E: 365 MI: 121 Status: New 101. The supply of product X is elastic if the price of X rises by: A) 5 percent and quantity supplied rises by 7 percent. B) 8 percent and quantity supplied rises by 8 percent. C) 10 percent and quantity supplied remains the same. D) 7 percent and quantity supplied rises by 5 percent. Answer: A

Type: A Topic: 4 E: 365 MI: 121 Status: New 102. The supply of product X is inelastic if the price of X rises by: A) 5 percent and quantity supplied rises by 7 percent. B) 8 percent and quantity supplied rises by 8 percent. C) 10 percent and quantity supplied remains the same. D) 7 percent and quantity supplied rises by 5 percent. Answer: D

McConnell/Brue: Economics, 16/e

Page 594

Chapter 20: Elasticity of Demand and Supply

Type: A Topic: 4 E: 365 MI: 121 Status: New 103. The elasticity of supply of product X is unitary if the price of X rises by: A) 5 percent and quantity supplied rises by 7 percent. B) 8 percent and quantity supplied rises by 8 percent. C) 10 percent and quantity supplied stays the same. D) 7 percent and quantity supplied rises by 5 percent. Answer: B

Type: A Topic: 4 E: 365 MI: 121 Status: New 104. The supply of product X is perfectly inelastic if the price of X rises by: A) 5 percent and quantity supplied rises by 7 percent. B) 8 percent and quantity supplied rises by 8 percent. C) 10 percent and quantity supplied stays the same. D) 7 percent and quantity supplied rises by 5 percent. Answer: C

Use the following to answer question 105:

Type: G Topic: 4 E: 365 MI: 121 105. The above diagram shows two product supply curves. It indicates that: A) over range Q1Q2 price elasticity of supply is greater for S1 than for S2. B) over range Q1Q2 price elasticity of supply is greater for S2 than for S1. C) over range Q1Q2 price elasticity of supply is the same for the two curves. D) not enough information is given to compare price elasticities. Answer: A

Type: A Topic: 4 E: 365 MI: 121 106. It takes a considerable amount of time to increase the production of pork. This implies that: A) a change in the demand for pork will not affect its price in the short run. B) the short-run supply curve for pork is less elastic than the long-run supply curve for pork. C) an increase in the demand for pork will elicit a larger supply response in the short run than in the long run. D) the long-run supply curve for pork is less elastic than the short-run supply curve for pork. Answer: B

McConnell/Brue: Economics, 16/e

Page 595

Chapter 20: Elasticity of Demand and Supply

Type: A Topic: 4 E: 365 MI: 121 107. Suppose that the price of product X rises by 20 percent and the quantity supplied of X increases by 15 percent. The coefficient of price elasticity of supply for good X is: A) negative and therefore X is an inferior good. C) less than 1 and therefore supply is inelastic. B) positive and therefore X is a normal good. D) more than 1 and therefore supply is elastic. Answer: C

Type: A Topic: 4 E: 365 MI: 121 108. Assume that the price of product X rises by 5 percent and the quantity supplied of X increases by 15 percent. The coefficient of price elasticity of supply for good X is: A) negative and therefore X is an inferior good. C) less than 1 and therefore supply is inelastic. B) positive and therefore X is a normal good. D) more than 1 and therefore supply is elastic. Answer: D

Use the following to answer questions 109-110:

Type: G Topic: 4 E: 365 MI: 121 109. Refer to the above diagram and assume that price increases from $2 to $10. The coefficient of the price elasticity of supply (midpoints formula) relating to this price change is about: A) 5 and supply is elastic. C) .25 and supply is inelastic. B) 1 and supply is unit elastic. D) 2.5 and supply is elastic. Answer: C

Type: G Topic: 4 E: 365 MI: 121 110. Refer to the above diagram and assume that price decreases from $10 to $2. The coefficient of the price elasticity of supply (midpoints formula) relating to this price change is about: A) 4 and supply is elastic. C) .5 and supply is inelastic. B) 1 and supply is unit elastic. D) .25 and supply is inelastic. Answer: D

McConnell/Brue: Economics, 16/e

Page 596

Chapter 20: Elasticity of Demand and Supply

Use the following to answer questions 111-114:

Type: G Topic: 4 E: 365-366 MI: 121-122 111. The above diagram concerns supply adjustments to an increase in demand (D1 to D 2) in the immediate market period, the short run, and the long run. Supply curves S1 , S2, and S3 apply to the: A) immediate market period, long run, and short run respectively. B) immediate market period, short run, and long run respectively. C) long run, short run, and immediate market period respectively. D) short run, long run, and immediate market period respectively. Answer: C

Type: G Topic: 4 E: 365-366 MI: 121-122 112. The above diagram concerns supply adjustments to an increase in demand (D1 to D 2) in the immediate market period, the short run, and the long run. In the immediate market period the increase in demand will: A) have no effect on either equilibrium price or quantity. B) increase equilibrium price, but not equilibrium quantity. C) increase equilibrium quantity, but not equilibrium price. D) increase both equilibrium price and quantity. Answer: B

Type: G Topic: 4 E: 365-366 MI: 121-122 113. The above diagram concerns supply adjustments to an increase in demand (D1 to D 2) in the immediate market period, the short run, and the long run. In the long run the increase in demand will: A) have no effect on either equilibrium price or quantity. B) increase equilibrium price, but not equilibrium quantity. C) increase equilibrium quantity, but not equilibrium price. D) increase both equilibrium price and quantity. Answer: D

Type: G Topic: 4 E: 365-366 MI: 121-122 114. The above diagram concerns supply adjustments to an increase in demand (D1 to D 2) in the immediate market period, the short run, and the long run. On the basis of this illustration we can conclude that: A) short-run adjustments are more economically efficient than are long-run adjustments. B) the amount of time producers have to adjust to a change in demand is not a determinant of supply elasticity. C) supply is more elastic the greater the amount of time producers have to adjust to a change in demand. D) supply is less elastic the greater the amount of time producers have to adjust to a change in demand. Answer: C

McConnell/Brue: Economics, 16/e

Page 597

Chapter 20: Elasticity of Demand and Supply

Type: C Topic: 4 E: 365 MI: 121 115. If the supply of product X is perfectly elastic, an increase in the demand for it will increase: A) equilibrium quantity but reduce equilibrium price. B) equilibrium quantity but equilibrium price will be unchanged. C) equilibrium price but reduce equilibrium quantity. D) equilibrium price but equilibrium quantity will be unchanged. Answer: B

Type: A Topic: 4 E: 366 MI: 122 116. Price elasticity of supply is: A) positive in the short run but negative in the long run. B) greater in the long run than in the short run. C) greater in the short run than in the long run. D) independent of time. Answer: B

Type: C Topic: 4 E: 365 MI: 121 117. Suppose the price of a product rises and the total revenue of sellers increases. A) It can be concluded that the demand for the product is elastic. B) It can be concluded that the supply of the product is elastic. C) It can be concluded that the supply of the product is inelastic. D) No conclusion can be reached with respect to the elasticity of supply. Answer: D

Type: A Topic: 4 E: 366 MI: 122 118. Supply curves tend to be: A) perfectly elastic in the long run because consumer demand will have sufficient time to adjust fully to changes in supply. B) more elastic in the long run because there is time for firms to enter or leave the industry. C) perfectly inelastic in the long run because the law of scarcity imposes absolute limits on production. D) less elastic in the long run because there is time for firms to enter or leave an industry. Answer: B

Type: A Topic: 4 E: 366 MI: 122 119. For an increase in demand the price effect is smallest and the quantity effect is largest: A) when supply is least elastic. C) in the short run. B) in the long run. D) in the immediate market period. Answer: B

Type: A Topic: 4 E: 365 MI: 121-122 120. A supply curve that is a vertical straight line indicates that: A) production costs for this product cannot be calculated. B) the relationship between price and quantity supplied is inverse. C) a change in price will have no effect on the quantity supplied. D) an unlimited amount of the product will be supplied at a constant price. Answer: C

McConnell/Brue: Economics, 16/e

Page 598

Chapter 20: Elasticity of Demand and Supply

Type: C Topic: 4 E: 365 MI: 121 121. A supply curve that is parallel to the horizontal axis suggests that: A) the industry is organized monopolistically. B) the relationship between price and quantity supplied is inverse. C) a change in demand will change price in the same direction. D) a change in demand will change the equilibrium quantity but not price. Answer: D

Type: C Topic: 4 E: 365 MI: 121 122. An increase in demand will increase equilibrium price to a greater extent: A) if the product is a normal good. C) the less elastic the supply curve. B) if the product is an inferior good. D) the more elastic the supply curve. Answer: C

Type: C Topic: 4 E: 365 MI: 121 123. Assume there is an increase in the demand for hand calculators. The subsequent: A) increase in price will be greater in the long run than in the short run. B) increase in price will be greater the greater the inelasticity of supply. C) increase in price will be greater the greater the elasticity of supply. D) decline in price will be greater the greater the elasticity of supply. Answer: B

Type: D Topic: 4 E: 365 MI: 121 124. If the supply of a product is inelastic, the price elasticity coefficient of supply is: A) zero. B) greater than one. C) equal to one. D) less than one. Answer: D

Type: A Topic: 4 E: 366 MI: 122 125. The supply of known Monet paintings is: A) perfectly elastic. B) perfectly inelastic. Answer: B

C) relatively elastic.

D) relatively inelastic.

McConnell/Brue: Economics, 16/e

Page 599

Chapter 20: Elasticity of Demand and Supply

Applications of price elasticity

Use the following to answer questions 126-134:
Price per ticket $13 11 9 7 5 3 Quantity demanded 1,000 2,000 3,000 4,000 5,000 6,000

Type: T Topic: 5 E: 360 MI: 116 126. Refer to the above information and assume the stadium capacity is 5000. If the Mudhens' management charges $7 per ticket: A) some fans who want to see the game will find that tickets are not available. B) there will be 2,000 empty seats. C) there will be 1,000 empty seats. D) the game will be sold out. Answer: C

Type: A Topic: 5 E: 365 MI: 121 127. Refer to the above information and assume the stadium capacity is 5000. The supply of seats for the game: A) varies inversely with ticket prices. C) is perfectly inelastic. B) varies directly with ticket prices. D) is perfectly elastic. Answer: C

Type: A Topic: 5 E: 360 MI: 116 128. Refer to the above information and assume the stadium capacity is 5000. If the Mudhens' management wanted a full house for the game, it would: A) set price so as to maximize its total revenue. B) encourage scalpers to sell their tickets for more than $7. C) set ticket prices at $5. D) set ticket prices at $9. Answer: C

Type: T Topic: 5 E: 360-361 MI: 116-117 Status: New 129. Refer to the above information. Over the $13-$11 price range, demand is: A) perfectly elastic. B) perfectly inelastic. C) elastic. D) inelastic. Answer: C

Type: T Topic: 5 E: 360-361 MI: 116-117 Status: New 130. Refer to the above information. Over the $11-$9 price range, demand is: A) perfectly elastic. B) perfectly inelastic. C) elastic. D) inelastic. Answer: C

McConnell/Brue: Economics, 16/e

Page 600

Chapter 20: Elasticity of Demand and Supply

Type: T Topic: 5 E: 360-361 MI: 116-117 Status: New 131. Refer to the above information. Over the $9-$7 price range, demand is: A) perfectly elastic. B) perfectly inelastic. C) elastic. D) inelastic. Answer: C

Type: T Topic: 5 E: 361 MI: 117 Status: New 132. Refer to the above information. Over the $7-$5 price range, demand is: A) perfectly elastic. B) perfectly inelastic. C) elastic. D) inelastic. Answer: D

Type: T Topic: 5 E: 361 MI: 117 Status: New 133. Refer to the above information. Over the $5-$3 price range, demand is: A) perfectly elastic. B) perfectly inelastic. C) elastic. D) inelastic. Answer: D

Type: T Topic: 5 E: 361 MI: 117 Status: New 134. Refer to the above information. If the Mudhens' management wanted to maximize total revenue from the game, it would set the ticket price at: A) $5. B) $7. C) $9. D) $13. Answer: B

Type: A Topic: 5 E: 367 MI: 123 135. An antidrug policy which reduces the supply of heroin might: A) increase street crime because the addict's demand for heroin is highly inelastic. B) reduce street crime because the addict's demand for heroin is highly elastic. C) reduce street crime because the addict's demand for heroin is highly inelastic. D) increase street crime because the addict's demand for heroin is highly elastic. Answer: A

Type: A Topic: 5 E: 364 MI: 120 136. Studies of the minimum wage suggest that the price elasticity of demand for teenage workers is relatively inelastic. This means that: A) an increase in the minimum wage would increase the total incomes of teenage workers as a group. B) an increase in the minimum wage would decrease the total incomes of teenage workers as a group. C) the unemployment effect of an increase in the minimum wage would be relatively large. D) the cross elasticity of demand between teenage and adult workers is positive and very large. Answer: A Type: A Topic: 5 E: 363-364 MI: 119-120 137. In 1991 the Federal government imposed a 10 percent excise tax on yachts costing more than $100,000. This tax: A) failed to bring in the expected amount of tax revenue because the demand for yachts was more price elastic than the government estimated. B) brought in more tax revenue than expected because the demand for yachts was less price elastic than the government estimated. C) failed to bring in the expected amount of tax revenue because the demand for yachts was less income elastic than the government estimated. D) brought about an increase in the demand for yachts and stimulated employment in the boat-building industry. Answer: A

McConnell/Brue: Economics, 16/e

Page 601

Chapter 20: Elasticity of Demand and Supply

Type: A Topic: 5 E: 363 MI: 119 138. Farmers often find that large bumper crops are associated with declines in their gross incomes. This suggests that: A) farm products are normal goods. B) farm products are inferior goods. C) the price elasticity of demand for farm products is less than 1. D) the price elasticity of demand for farm products is greater than 1. Answer: C

Use the following to answer questions 139-144:

Quantity demanded 45 50 56 61 67

Price $10 8 6 4 2

Quantity supplied 77 73 68 61 57

Type: T Topic: 5 E: 48-49 MI: 48-49 Status: New 139. Refer to the above data: Equilibrium price is: A) $8. B) $6. C) $4. D) $2. Answer: C

Type: T Topic: 5 E: 365 MI: 121 Status: New 140. Refer to the above data. Suppose quantity demanded increased by 12 units at each price, changing the equilibrium price in a direction and an amount for you to determine. Over that price range, supply is: A) perfectly elastic. B) perfectly inelastic. C) elastic. D) inelastic. Answer: D

Type: T Topic: 5 E: 365 MI: 121 Status: New 141. Refer to the above data. Suppose quantity supplied declined by 23 units at each price, changing the equilibrium price in a direction and amount for you to determine. Over that price range, demand is: A) elastic. B) inelastic. C) perfectly elastic. D) perfectly inelastic. Answer: B

Type: T Topic: 5 E: 366 MI: 122 Status: New 142. The price of old baseball cards rises rapidly with increases in demand because: A) the supply of old baseball cards is inelastic. C) the demand for old baseball cards is inelastic. B) the supply of old baseball cards in elastic. D) the demand for old baseball cards is elastic. Answer: A

Type: T Topic: 5 E: 366 MI: 122 Status: New 143. The supply curve of a one-of-a-kind original painting is: A) relatively elastic. B) relatively inelastic. C) perfectly inelastic. D) perfectly elastic. Answer: C

McConnell/Brue: Economics, 16/e

Page 602

Chapter 20: Elasticity of Demand and Supply

Type: T Topic: 5 E: 366 MI: 122 Status: New 144. The supply curve of antique reproductions is: A) relatively elastic. B) relatively inelastic. C) perfectly inelastic. D) unit elastic. Answer: A

Cross and income elasticity

Type: A Topic: 6 E: 368 MI: 124 145. Suppose the income elasticity of demand for toys is +2.00. This means that: A) a 10 percent increase in income will increase the purchase of toys by 20 percent. B) a 10 percent increase in income will increase the purchase of toys by 2 percent. C) a 10 percent increase in income will decrease the purchase of toys by 2 percent. D) toys are an inferior good. Answer: A

Type: A Topic: 6 E: 368 MI: 124 146. If the income elasticity of demand for lard is -3.00, this means that: A) lard is a substitute for butter. B) lard is a normal good. C) lard is an inferior good. D) more lard will be purchased when its price falls. Answer: C

Type: A Topic: 6 E: 367 MI: 123 147. The formula for cross elasticity of demand is percentage change in: A) quantity demanded of X/percentage change in price of X. B) quantity demanded of X/percentage change in income. C) quantity demanded of X/percentage change in price of Y. D) price of X/percentage change in quantity demanded of Y. Answer: C

Type: A Topic: 6 E: 367 MI: 123 148. Cross elasticity of demand measures how sensitive purchases of a specific product are to changes in: A) the price of some other product. C) income. B) the price of that same product. D) the general price level. Answer: A

Type: A Topic: 6 E: 367 MI: 123 149. The larger the positive cross elasticity coefficient of demand between products X and Y, the: A) stronger their complementariness. B) greater their substitutability. C) smaller the price elasticity of demand for both products. D) the less sensitive purchases of each are to increases in income. Answer: B

McConnell/Brue: Economics, 16/e

Page 603

Chapter 20: Elasticity of Demand and Supply

Type: A Topic: 6 E: 367 MI: 123 150. We would expect the cross elasticity of demand between Pepsi and Coke to be: A) positive, indicating normal goods. C) positive, indicating substitute goods. B) positive, indicating inferior goods. D) negative, indicating substitute goods. Answer: C

Type: A Topic: 6 E: 367 MI: 123 151. We would expect the cross elasticity of demand between dress shirts and ties to be: A) positive, indicating normal goods. C) negative, indicating substitute goods. B) positive, indicating inferior goods. D) negative, indicating complementary goods. Answer: D

Use the following to answer question 152:

Type: G Topic: 6 E: 367 MI: 123 152. The above diagram suggests that: A) X and Y are both inferior goods. B) X and Y are both normal goods. Answer: D

C) X and Y are substitute goods. D) X and Y are independent goods.

Type: A Topic: 6 E: 367 MI: 123 153. Compared to coffee, we would expect the cross elasticity of demand for: A) tea to be negative, but positive for cream. C) both tea and cream to be negative. B) tea to be positive, but negative for cream. D) both tea and cream to be positive. Answer: B

Type: A Topic: 6 E: 367 MI: 123 154. We would expect the cross elasticity of demand for Pepsi in relation to other soft drinks to be greater than that for soft drinks generally because: A) soft drinks are normal goods. B) the income effect always exceeds the substitution effect. C) there are fewer good substitutes for soft drinks generally than for Pepsi. D) there are more good substitutes for soft drinks generally than for Pepsi. Answer: C

McConnell/Brue: Economics, 16/e

Page 604

Chapter 20: Elasticity of Demand and Supply

Type: A Topic: 6 E: 368 MI: 124 155. Suppose that when your income increases from $28,000 to $30,000 per year, your purchases of X increase from 4 to 5 units because of that income increase. Thus: A) X is an inferior good. B) X is a substitute good. C) the income effect exceeds the substitution effect. D) the demand for X is elastic with respect to income. Answer: D

Use the following to answer questions 156-163:

Type: G Topic: 6 E: 367 MI: 123 156. Refer to the above diagrams. The case of substitute goods is represented by figure: A) A. B) B. C) C. D) D. Answer: D

Type: G Topic: 6 E: 368 MI: 124 157. Refer to the above diagrams. The case of a normal good is represented by figure(s): A) A. B) B. C) C. D) D. Answer: A

Type: G Topic: 6 E: 368 MI: 124 158. Refer to the above diagrams. The case of an inferior good is represented by figure(s): A) A only. B) B only. C) C. D) D. Answer: B

Type: G Topic: 6 E: 367 MI: 123 159. Refer to the above diagrams. The case of complementary goods is represented by figure: A) A. B) B. C) C. D) D. Answer: C

McConnell/Brue: Economics, 16/e

Page 605

Chapter 20: Elasticity of Demand and Supply

Type: G Topic: 6 E: 368 MI: 124 160. Refer to the above diagrams. In which case would the coefficient of income elasticity be positive? A) A. B) B. C) C. D) D. Answer: A

Type: G Topic: 6 E: 368 MI: 124 161. Refer to the above diagrams. In which case would the coefficient of income elasticity be negative? A) A. B) B. C) C. D) D. Answer: B

Type: G Topic: 6 E: 368 MI: 124 162. Refer to the above diagrams. In which case would the coefficient of cross elasticity of demand be positive? A) A. B) B. C) C. D) D. Answer: D

Type: G Topic: 6 E: 368 MI: 124 163. Refer to the above diagrams. In which case would the coefficient of cross elasticity of demand be negative? A) A B) B C) C D) D Answer: C

Type: A Topic: 6 E: 367 MI: 123 164. Suppose that a 10 percent increase in the price of normal good Y causes a 20 percent increase in the quantity demanded of normal good X. The coefficient of cross elasticity of demand is: A) negative and therefore these goods are substitutes. B) negative and therefore these goods are complements. C) positive and therefore these goods are substitutes. D) positive and therefore these goods are complements. Answer: C

Type: A Topic: 6 E: 367 MI: 123 165. Suppose that a 20 percent increase in the price of normal good Y causes a 10 percent decline in the quantity demanded of normal good X. The coefficient of cross elasticity of demand is: A) negative and therefore these goods are substitutes. B) negative and therefore these goods are complements. C) positive and therefore these goods are substitutes. D) positive and therefore these goods are complements. Answer: B

Type: A Topic: 6 E: 368 MI: 124 166. Assume that a 4 percent increase in income in the economy produces an 8 percent increase in the quantity demanded of good X. The coefficient of income elasticity of demand is: A) negative and therefore X is an inferior good. C) positive and therefore X is an inferior good. B) negative and therefore X is a normal good. D) positive and therefore X is a normal good. Answer: D

McConnell/Brue: Economics, 16/e

Page 606

Chapter 20: Elasticity of Demand and Supply

Type: A Topic: 6 E: 368 MI: 124 167. Assume that a 3 percent increase in income in the economy produces a 1 percent decline in the quantity demanded of good X. The coefficient of income elasticity of demand for good X is: A) negative and therefore X is an inferior good. C) positive and therefore X is an inferior good. B) negative and therefore X is a normal good. D) positive and therefore X is a normal good. Answer: A

Consider This Questions

Type: F E: 358 MI: 114 Status: New 168. (Consider This) Elastic demand is analogous to a __________ and inelastic demand to a _________. A) normal wrench; socket wrench C) Ace bandage; firm rubber tie-down B) tight rubber band; loose rubber band D) one-foot ruler; tape measure Answer: C

Type: F E: 358 MI: 114 Status: New 169. (Consider This) Elasticity can be thought of as degree of relative: A) video brightness. B) price bounce. C) audio volume. D) quantity stretch. Answer: D

Last Word Questions

Type: A E: 369 MI: 125 Status: New 170. (Last Word) Suppose that a firm has "pricing power" and can segregate its market into two distinct groups based on differences in elasticities of demand. The firm might charge: A) a lower price to the group that has the less elastic demand. B) a higher price to the group that has the less elastic demand. C) the same price to both groups but include a "free" related product for the group that has an inelastic demand. D) the same price to both groups but make it difficult for the group with the more elastic demand to gain access to the product. Answer: B

Type: A E: 369 MI: 125 Status: New 171. (Last Word) Microsoft charges a substantially lower price for a software upgrade than for the initial purchase of the software. This implies that Microsoft views the demand curve for the software upgrade to be: A) more elastic than the demand for the original software. B) upsloping rather than downsloping. C) less elastic than the demand for the original software. D) of less value than the original software. Answer: A

McConnell/Brue: Economics, 16/e

Page 607

Chapter 20: Elasticity of Demand and Supply

Type: A E: 369 MI: 125 Status: New 172. (Last Word) Which of the following is not an example of pricing based on group differences in elasticity of demand? A) Senior-citizen discounts at restaurants and motels. B) Cash rebates for purchases of automobiles. C) Children discounts for admissions to theme parks. D) Discounted student prices for visits to museums. Answer: B

True/False Questions

Type: A E: 359 MI: 115 Status: New 173. A linear demand curve has a constant elasticity over the full range of the curve. Answer: False

Type: A E: 365 MI: 121 Status: New 174. The greater the ease of shifting resources from product X to Y in the production process, the greater is the elasticity of supply of product Y. Answer: True

Type: E E: 365 MI: 121 Status: New 175. If the elasticity coefficient of supply is 0.7, supply is elastic. Answer: False

Type: F E: 366 MI: 122 Status: New 176. Antiques tend to have highly inelastic supply curves. Answer: True

Type: A E: 362 MI: 118 177. The smaller the number of good substitutes for a product, the greater will be the price elasticity of demand for it. Answer: False

Type: A E: 363 MI: 119 178. If the demand for wheat is highly price inelastic, an extraordinarly large crop may reduce farm incomes. Answer: True

Type: A E: 362 MI: 118 179. Generally speaking, the demand for luxury goods is more price elastic than is the demand for necessities. Answer: True

Type: A E: 362 MI: 118 180. Generally speaking, the smaller the percentage of one's total budget devoted to a particular product, the more price elastic will be the demand for that product. Answer: False

McConnell/Brue: Economics, 16/e

Page 608

Chapter 20: Elasticity of Demand and Supply

Type: A E: 361 MI: 117 181. If price and total revenue are directly related, demand is inelastic. Answer: True

Type: A E: 361-362 MI: 117-118 182. If price changes and total revenue changes in the opposite direction, demand is relatively elastic. Answer: True

Use the following to answer questions 183-185: Answer the next question(s) on the basis of the following demand and supply data:
Quantity demanded per month Price 30 $8 36 7 42 6 50 5 Quantity supplied per month 44 38 30 20

Type: A E: 48-49 MI: 48-49 183. Refer to the above data. The equilibrium price of this product is somewhere between $6 and $7. Answer: True

Type: T E: 359 MI: 115 184. Refer to the above data. The demand for this product is elastic in the $8-$7 price range. Answer: True

Type: T E: 365 MI: 121 185. Refer to the above data. The supply of this product is inelastic in the $6-$5 price range. Answer: False

Type: T E: 367 MI: 123 186. Cross elasticity of demand measures the effect of a change in the price of one product on the quantity demanded of another product. Answer: True

Type: D E: 368 MI: 124 187. Income elasticity measures the effect of a change in income on the purchases of some good or service. Answer: True

Type: D E: 368 MI: 124 188. If the coefficient of income elasticity of demand is positive, the product is an inferior good. Answer: False

McConnell/Brue: Economics, 16/e

Page 609

Chapter 20: Elasticity of Demand and Supply

Type: A E: 367 MI: 123 189. If the coefficient of cross elasticity of demand is positive, the two products are complementary goods. Answer: False

Type: A E: 368 MI: 124 190. An income elasticity coefficient of -1.8 means the product is a normal good. Answer: False

Type: A E: 367 MI: 123 191. A cross elasticity of demand coefficient of +2.5 indicates that the two products are substitutes. Answer: True

Type: A E: 367 MI: 123 192. We would expect the coefficient of cross elasticity of demand for DVD players and DVDs to be positive. Answer: False

McConnell/Brue: Economics, 16/e

Page 610

CHAPTER 21

Consumer Behavior and Utility Maximization

Topic 1. 2. 3. 4. 5. Income and substitution effects Utility; law of diminishing marginal utility Utility-maximizing rule Marginal utility and product demand Applications; extensions Consider This Last Word True-False

Question numbers 1-9 10-36 37-64 65-74 75-87 88-89 90-92 93-105

____________________________________________________________

_______________________________________

Appendix 6. Budget lines 7. Indifference curves 8. Consumer equilibrium; changes in equilibrium Consider This True-False

106-131 132-141 142-160 161-162 163-174

____________________________________________________________

_______________________________________

Multiple Choice Questions Income and substitution effects

Type: D Topic: 1 E: 372 MI: 128 1. The income effect indicates that: A) a rise in money income will cause consumers to buy smaller quantities of normal goods. B) when the price of a product falls, the lower price will induce the consumer to buy more of that product now that it is relatively cheaper. C) consumers should substitute among various products until the marginal utility from the last unit of each product purchased is the same. D) when the price of a product falls, a consumer will be able to buy more of it with a specific money income. Answer: D

Chapter 21: Consumer Behavior and Utility Maximization

Type: A Topic: 1 E: 372-373 MI: 128-129 2. If the price of normal good X rises, the income: A) and substitution effects will both induce the consumer to buy less of X. B) and substitution effects will both induce the consumer to buy more of X. C) effect will induce the consumer to buy more of X and the substitution effect will induce him to buy less. D) effect will induce the consumer to buy less of X and the substitution will induce him to buy more. Answer: A

Type: A Topic: 1 E: 373 MI: 129 3. If the price of a product falls, that product becomes cheaper and people will want to purchase more of it in place of other goods. This statement best describes: A) the income effect. B) the substitution effect. C) a complementary good. D) an inferior good. Answer: B

Type: A Topic: 1 E: 372 MI: 128 4. A fall in the price of a good increases the real income or purchasing power of consumers so that they are able to buy more of the product. This statement best describes: A) the income effect. B) a complementary good. C) the substitution effect. D) an inferior good. Answer: A

Type: A Topic: 1 E: 372-373 MI: 128-129 5. Which of the following is correct? When the price of normal good Z falls: A) both income and substitution effects cause the consumer to buy more. B) both income and substitution effects cause the consumer to buy less. C) the income effect causes the consumer to buy less, but the substitution effect causes her to buy more. D) the income effect causes the consumer to buy more, but the substitution effect causes her to buy less. Answer: A

Type: A Topic: 1 E: 372-373 MI: 128-129 6. If steak is a normal good and its price rises: A) the amount purchased may either increase or decrease depending on the relative importance of the income and substitution effects. B) both the income and substitution effects suggest that less will be purchased. C) the substitution effect suggests more will be purchased, but the income effect suggests less will be purchased. D) the income effect suggests more will be purchased, but the substitution effect suggests less will be purchased. Answer: B

Type: D Topic: 1 E: 373 MI: 129 7. The substitution effect indicates that: A) a decline in money income will cause the consumer to buy more inferior goods and fewer superior goods. B) consumer equilibrium can only be achieved when the consumer is buying substitute goods. C) when the price of a product falls, the lower price will induce the consumer to buy more of that product at the expense of other products. D) when the price of a product falls, a consumer will be able to buy more of it with a specific money income. Answer: C

McConnell/Brue: Economics, 16/e

Page 612

Chapter 21: Consumer Behavior and Utility Maximization

Type: A Topic: 1 E: 373 MI: 129 8. The substitution effect causes a consumer to buy less of a product when its price rises because the: A) consumer's real income has decreased. B) consumer's real income has increased. C) product is now less expensive compared to other products. D) product is now more expensive compared to other products. Answer: D

Type: A Topic: 1 E: 372-373 MI: 128-129 9. Suppose that Jean normally orders three tacos, but on seeing that their price has gone up, decides to buy only two. Jean's decision is best explained by: A) income and substitution effects. C) the principle of comparative advantage. B) the law of supply. D) the law of increasing opportunity costs. Answer: A

Utility; law of diminishing marginal utility

Type: F Topic: 2 E: 373 MI: 129 Status: New 10. Utility: A) is synonymous with usefulness. B) is want-satisfying power. Answer: B

C) is easy to quantify. D) rarely varies from person to person.

Type: F Topic: 2 E: 373 MI: 129 Status: New 11. Marginal utility can be: A) positive, but not negative. B) positive or negative, but not zero. Answer: C

C) positive, negative, or zero. D) decreasing, but not negative.

Type: A Topic: 2 E: 373-374 MI: 129-130 Status: New 12. Brenda says, "You would have to pay me $50 to attend that pro wrestling event." For Brenda, the marginal utility of the event is: A) zero. B) positive, but declines rapidly. C) negative. D) positive, but less than the ticket price. Answer: C

Type: A Topic: 2 E: 373 MI: 129 Status: New 13. The ability of a good or service to satisfy wants is called: A) utility maximization. B) opportunity cost. C) revenue potential. D) utility. Answer: D

McConnell/Brue: Economics, 16/e

Page 613

Chapter 21: Consumer Behavior and Utility Maximization

Use the following to answer questions 14-19:
Units consumed 0 1 2 3 4 Total utility 0 W 35 Y 40 Marginal utility 20 X 10 Z

Type: T Topic: 2 E: 373-374 MI: 129-130 Status: New 14. Refer to the above data. The value for Y is: A) 25. B) 30. C) 40. D) 45. Answer: D

Type: T Topic: 2 E: 373-374 MI: 129-130 Status: New 15. Refer to the above data. The value for X is: A) 15. B) 5. C) 55. D) 10. Answer: A

Type: T Topic: 2 E: 373-374 MI: 129-130 Status: New 16. Refer to the above data. The value for W is: A) 15. B) 20. C) 25. D) 30. Answer: B

Type: T Topic: 2 E: 373-374 MI: 129-130 Status: New 17. Refer to the above data. The value for Z is: A) -5. B) + 5. C) -10. D) zero. Answer: A

Type: T Topic: 2 E: 375 MI: 131 Status: New 18. The above data illustrate the: A) law of comparative advantage. B) utility-maximizing rule. Answer: C

C) law of diminishing marginal utility. D) law of increasing opportunity costs.

Type: T Topic: 2 E: 373-374 MI: 129-130 Status: New 19. Refer to above data. Marginal utility becomes negative beginning with the: A) first unit. B) second unit. C) third unit. D) fourth unit. Answer: D

Type: D Topic: 2 E: 373 MI: 129 20. A product has utility if it: A) takes more and more resources to produce successive units of it. B) violates the law of demand. C) satisfies consumer wants. D) is useful. Answer: C

McConnell/Brue: Economics, 16/e

Page 614

Chapter 21: Consumer Behavior and Utility Maximization

Type: D Topic: 2 E: 375 MI: 131 21. The law of diminishing marginal utility states that: A) total utility is maximized when consumers obtain the same amount of utility per unit of each product consumed. B) beyond some point additional units of a product will yield less and less extra satisfaction to a consumer. C) price must be lowered to induce firms to supply more of a product. D) it will take larger and larger amounts of resources beyond some point to produce successive units of a product. Answer: B

Type: A Topic: 2 E: 374 MI: 130 22. The first Pepsi yields Craig 18 units of utility and the second yields him an additional 12 units of utility. His total utility from three Pepsis is 38 units of utility. The marginal utility of the third Pepsi is: A) 26 units of utility. B) 6 units of utility. C) 8 units of utility. D) 38 units of utility. Answer: C

Type: A Topic: 2 E: 375 MI: 131 23. If the price of product X rises, then the resulting decline in the amount purchased will: A) necessarily increase the consumer's total utility from his total purchases. B) increase the marginal utility of this good. C) increase the total utility from purchases of this good. D) reduce the marginal utility of this good. Answer: B

Type: D Topic: 2 E: 373 MI: 129 24. Marginal utility is the: A) sensitivity of consumer purchases of a good to changes in the price of that good. B) change in total utility obtained by consuming one more unit of a good. C) change in total utility obtained by consuming another unit of a good divided by the change in the price of that good. D) total utility associated with the consumption of a certain number of units of a good divided by the number of units consumed. Answer: B

Type: D Topic: 2 E: 373 MI: 129 25. Utility refers to the: A) satisfaction that a consumer derives from a good or service. B) rate of decline in a product demand curve. C) relative scarcity of a product. D) usefulness of a product. Answer: A

Type: A Topic: 2 E: 373 MI: 129 26. Total utility may be determined by: A) multiplying the marginal utility of the last unit consumed by the number of units consumed. B) summing the marginal utilities of each unit consumed. C) multiplying the marginal utility of the last unit consumed by product price. D) multiplying the marginal utility of the first unit consumed by the number of units consumed. Answer: B

McConnell/Brue: Economics, 16/e

Page 615

Chapter 21: Consumer Behavior and Utility Maximization

Use the following to answer questions 27-29:

Type: G Topic: 2 E: 374 MI: 130 27. Refer to the above diagram. The marginal utility of the third unit of X is: A) 5. B) 4. C) 2. D) 1. Answer: B

Type: G Topic: 2 E: 374 MI: 130 28. Refer to the above diagram. The total utility yielded by 4 units of X is: A) 4. B) 15. C) 17. D) 18. Answer: C

Type: G Topic: 2 E: 374 MI: 130 29. Refer to the above diagram. Total utility is at a maximum at _____ units of X. A) 2 B) 3 C) 4 D) 6 Answer: D

McConnell/Brue: Economics, 16/e

Page 616

Chapter 21: Consumer Behavior and Utility Maximization

Use the following to answer questions 30-31:

Type: G Topic: 2 E: 374 MI: 130 30. Refer to the above diagram. Total utility: A) increases so long as additional units of Y are purchased. B) becomes negative at 4 units. C) increases at a diminishing rate, reaches a maximum, and then declines. D) is maximized at 2 units. Answer: C

Type: G Topic: 2 E: 374 MI: 130 31. Refer to the above diagram. Marginal utility: A) increases at an increasing rate. B) becomes zero at 4 units of output. C) is found by dividing total utility by the number of units purchased. D) cannot be calculated from the total utility information. Answer: B

Type: A Topic: 2 E: 373-375 MI: 129-131 32. If total utility is increasing, marginal utility: A) is positive, but may be either increasing or decreasing. B) must also be increasing. C) may be either positive or negative. D) will be increasing at an increasing rate. Answer: A

Type: A Topic: 2 E: 373-374 MI: 129-130 33. Where total utility is at a maximum, marginal utility is: A) negative. B) positive and increasing. C) zero. D) positive but decreasing. Answer: C

McConnell/Brue: Economics, 16/e

Page 617

Chapter 21: Consumer Behavior and Utility Maximization

Type: A Topic: 2 E: 373 MI: 129 34. Marginal utility: A) is equal to total utility divided by the number of units consumed. B) is equal to total utility if the demand curve is linear. C) increases as more of a product is consumed. D) diminishes as more of a product is consumed. Answer: D

Type: A Topic: 2 E: 373-374 MI: 129-130 35. Which of the following is correct? A) There is no firm mathematical relationship between marginal utility and total utility. B) Total utility is equal to the change in marginal utility from consuming an additional unit of a product. C) If marginal utility is diminishing and is a positive amount, total utility will increase. D) If marginal utility is diminishing, total utility must also be diminishing. Answer: C

Type: A Topic: 2 E: 373 MI: 129 36. Which of the following statements is correct? A) Utility and usefulness are synonymous. B) The marginal utility derived from successive units of a product tends to be similar for all consumers. C) Because utility is not measurable, the utility-maximizing rule provides no useful insights as to consumer behavior. D) A product may yield utility, but not be functionally useful. Answer: D

Utility-maximizing rule
Type: D Topic: 3 E: 376 MI: 132 37. The theory of consumer behavior assumes that : A) consumers behave rationally, maximizing their satisfactions. B) consumers have unlimited money incomes. C) consumers do not know how much marginal utility they obtain from succesive units of various products. D) marginal utility is constant. Answer: A

Type: A Topic: 3 E: 376 MI: 132 38. To maximize utility a consumer should allocate money income so that the: A) elasticity of demand on all products purchased is the same. B) marginal utility obtained from the last dollar spent on each product is the same. C) total utility derived from each product consumed is the same. D) marginal utility of the last unit of each product consumed is the same. Answer: B

Type: A Topic: 3 E: 377 MI: 133 39. Suppose that MUx/Px exceeds MUy/Py. To maximize utility the consumer who is spending all her money income should buy: A) less of X only if its price rises. C) more of Y and less of X. B) more of Y only if its price rises. D) more of X and less of Y. Answer: D

McConnell/Brue: Economics, 16/e

Page 618

Chapter 21: Consumer Behavior and Utility Maximization

Type: A Topic: 3 E: 377 MI: 133 40. Mrs. Green is spending all her money income by buying bottles of soda and bags of pretzels in such amounts that the marginal utility of the last bottle is 60 utils and the marginal utility of the last bag is 30 utils. The prices of soda and pretzels are $.60 per bottle and $.40 per bag respectively. It can be concluded that: A) the two commodities are substitute goods. B) Mrs. Green should spend more on pretzels and less on soda. C) Mrs. Green should spend more on soda and less on pretzels. D) Mrs. Green is buying soda and pretzels in the utility-maximizing amounts. Answer: C

Use the following to answer questions 41-43: Answer the next question(s) on the basis of the following two schedules which show the amounts of additional satisfaction (marginal utility) which a consumer would get from successive quantities of products J and K.

Units of J MUj 1 56 2 48 3 32 4 24 5 20 6 16 7 12

Units of K MUk 1 32 28 2 24 3 20 4 12 5 10 6 7 8

Type: T Topic: 3 E: 376-377 MI: 132-133 41. Refer to the above data. If the consumer has a money income of $52 and the prices of J and K are $8 and $4 respectively, the consumer will maximize her utility by purchasing: A) 2 units of J and 7 units of K. C) 4 units of J and 5 units of K. B) 5 units of J and 5 units of K. D) 6 units of J and 3 units of K. Answer: C

Type: T Topic: 3 E: 376-377 MI: 132-133 42. Refer to the above data. What level of total utility is realized from the equilibrium combination of J and K determined in the previous question? A) 156 utils B) 124 utils C) 276 utils D) 36 utils Answer: C

Type: T Topic: 3 E: 377 MI: 133 43. Refer to the above data. If the consumer's money income were cut from $52 to $28, she would maximize her satisfaction by purchasing: A) 3 units of J and 3 units of K. C) 4 units of J and 1 unit of K. B) 1 unit of J and 3 units of K. D) 2 units of J and 3 units of K. Answer: D

McConnell/Brue: Economics, 16/e

Page 619

Chapter 21: Consumer Behavior and Utility Maximization

Type: A Topic: 3 E: 377 MI: 133 44. Ben is exhausting his money income consuming products A and B in such quantities that MUa/Pa = 5 and MUb/Pb = 8. Ben should purchase: A) more of A and less of B. C) more of both A and B. B) more of B and less of A. D) less of both A and B. Answer: B

Type: A Topic: 3 E: 377 MI: 133 45. The marginal utility of the last unit of A consumed is 12 and the marginal utility of the last unit of B consumed is 8. What set of prices for A and B respectively would be consistent with consumer equilibrium? A) $4 and $6 B) $6 and $4 C) $8 and $12 D) $16 and $9 Answer: B

Type: A Topic: 3 E: 377 MI: 133 46. Suppose you have a limited money income and you are purchasing products A and B whose prices happen to be the same. To maximize your utility you should purchase A and B in such amounts that: A) their marginal utilities are the same. B) their total utilities are the same. C) their marginal and total utilities are proportionate. D) the income and substitution effects associated with each are equal. Answer: A

Type: A Topic: 3 E: 378 MI: 134 47. A consumer is maximizing her utility with a particular money income when: A) the total utility derived from each product consumed is the same. B) MUa/Pa = MUb/Pb = MUc/Pc = ... = MUn/Pn. C) MUa = MUb = MUc = ... = MUn. D) Pa = Pb = Pc = ... = Pn. Answer: B

Type: A Topic: 3 E: 376-377 MI: 132-133 48. Suppose that Ms. Spencer is currently exhausting her money income by purchasing 10 units of A and 8 units of B at prices of $2 and $4 respectively. The marginal utility of the last units of A and B are 16 and 24 respectively. These data suggest that Ms. Spencer: A) has preferences that are at odds with the principle of diminishing marginal utility. B) considers A and B to be complementary goods. C) should buy less A and more B. D) should buy less B and more A. Answer: D

Type: A Topic: 3 E: 376-377 MI: 132-133 49. If a rational consumer is in equilibrium, which of the following conditions will hold true? A) MUa = MUb = MUc = ... = MUn. B) The marginal utility of each good purchased will be zero. C) The marginal utility of the last dollar spent on each good purchased will be the same. D) The total utility obtained from each good purchased will be the same. Answer: C

McConnell/Brue: Economics, 16/e

Page 620

Chapter 21: Consumer Behavior and Utility Maximization

Type: A Topic: 3 E: 378 MI: 134 50. Assume MUc and MUd represent the marginal utility that a consumer gets from products C and D, the respective prices of which are Pc and P d. The consumer will increase his total utility from a specific money outlay by spending more on C and less on D if initially: A) MUd < MUc B)

C)

D) MUc > MUd Answer: C

Type: A Topic: 3 E: 378 MI: 134 51. In purchasing products A and B, a consumer is in equilibrium when: A) MUa/Pa = MUb/Pb B) MUa/Pb = MUb/Pa C) MUa - MUb = Pa/Pb D) MUa × Pa = MUb × Pb Answer: A

Type: A Topic: 3 E: 378 MI: 134 52. A consumer who has a limited budget will maximize utility or satisfaction when the: A) ratios of the marginal utility of each product purchased divided by its price are equal. B) total utility derived from each product purchased is the same. C) marginal utility of each product purchased is the same. D) price of each product purchased is the same. Answer: A

Type: A Topic: 3 E: 378 MI: 134 53. Mr. Chan has an income of $20 that he is spending on donuts and cheese in such amounts that he derives 25 utils of satisfaction from the donuts and 25 utils of satisfaction from the cheese. On the basis of this information we: A) cannot say whether or not Chan is buying donuts and cheese in equilibrium amounts. B) can say that Chan should buy more cheese and fewer donuts. C) can say that Chan should buy more donuts and less cheese. D) can say that Chan is buying the utility-maximizing amounts of donuts and cheese. Answer: A

Type: A Topic: 3 E: 378 MI: 134 54. If MUa/Pa = 100/$35 = MUb/Pb = 300/? = MUc/Pc = 400/?, the prices of products b and c in consumer equilibrium: A) cannot be determined from the information given. B) are $105 and $140 respectively. C) are $105 and $175 respectively. D) are $100 and $200 respectively. Answer: B

McConnell/Brue: Economics, 16/e

Page 621

Chapter 21: Consumer Behavior and Utility Maximization

Type: A Topic: 3 E: 378 MI: 134 55. Assume that a consumer purchases products A, B, and C in quantities such that the last dollar spent on each yields the same marginal utility and the consumer's income is totally spent. We can conclude that: A) total utility is being minimized. C) marginal utility exceeds total utility. B) production costs are being minimized. D) total utility is being maximized. Answer: D

Use the following to answer questions 56-57: Answer the next question(s) on the basis of the following total utility data for products L and M. Assume that the prices of L and M are $3 and $4 respectively and that the consumer's income is $18.

Units of L 1 2 3 4 5

Total utility 9 15 18 20 21

Units of M 1 2 3 4 5

Total utility 16 28 36 40 42

Type: A Topic: 3 E: 378 MI: 134 56. Refer to the above data. How many units of the two products will the consumer purchase? A) 3 of L and none of M B) 4 of L and 2 of M C) 3 of L and 5 of M D) 2 of L and 3 of M Answer: D

Type: A Topic: 3 E: 376 MI: 132 57. Refer to the above data. What level of total utility does the consumer realize in equilibrium? A) 87 utils B) 114 utils C) 51 utils D) 58 utils Answer: C

Type: A Topic: 3 E: 376 MI: 132 58. An increase in the price of product A will: A) increase the marginal utility per dollar spent on A. B) decrease the marginal utility per dollar spent on A. C) not affect the marginal utility per dollar spent on A. D) cause utility-maximizing consumers to buy more of A. Answer: B

Type: A Topic: 3 E: 377 MI: 133 59. Rosenbaum is purchasing products C and D in utility-maximizing amounts. If the price of C is $4 and the price of D is $2, then: A) the marginal utility of D is twice that of C. B) the marginal utility of D is the same as that of C. C) the marginal utility of C is twice that of D. D) the marginal utility of C is four times that of D. Answer: C

McConnell/Brue: Economics, 16/e

Page 622

Chapter 21: Consumer Behavior and Utility Maximization

Type: D Topic: 3 E: 376 MI: 132 60. The theory of consumer behavior assumes that consumers attempt to maximize: A) the difference between total and marginal utility. B) total utility. C) average utility. D) marginal utility. Answer: B

Type: A Topic: 3 E: 376-377 MI: 132-133 61. When a consumer is maximizing total utility, A) the average utility from each dollar spent is the same. B) total utility cannot be increased by reallocating expenditures among various products. C) the total utility obtainable from each product is at a maximum. D) the marginal utility of the last unit of each product purchased is zero. Answer: B

Type: A Topic: 3 E: 376-377 MI: 132-133 62. When a consumer shifts purchases from product X to product Y the marginal utility of: A) X falls and the marginal utility of Y rises. C) both X and Y rises. B) X rises and the marginal utility of Y falls. D) both X and Y falls. Answer: B

Type: A Topic: 3 E: 376-377 MI: 132-133 63. Sam decides to buy a $75 ticket to a particular New York professional hockey game rather than a $50 ticket for a particular Broadway play. We can conclude that Sam: A) is relatively unappreciative of the arts. B) obtains more marginal utility from the play than from the hockey game. C) has a higher "marginal utility to price ratio" for the hockey game than for the play. D) has recently attended several other Broadway plays. Answer: C

Type: A Topic: 3 E: 376 MI: 132 64. Suppose that Dave normally orders two tacos, but on seeing they are on sale, decides to buy three. Dave's decision is best explained by the: A) law of increasing opportunity costs. C) principle of comparative advantage. B) law of supply. D) the principle of utility maximization. Answer: D

Marginal utility and product demand

Type: A Topic: 4 E: 379 MI: 135 65. Diminishing marginal utility explains why: A) the income effect exceeds the substitution effect. B) the substitution effect exceeds the income effect. C) supply curves are upsloping. D) demand curves are downsloping. Answer: D

McConnell/Brue: Economics, 16/e

Page 623

Chapter 21: Consumer Behavior and Utility Maximization

Type: A Topic: 4 E: 379 MI: 135 66. What do the income effect, the substitution effect, and diminishing marginal utility have in common? A) All are required to explain the utility-maximizing position of a consumer. B) They are all empirically measurable. C) They all help explain the upsloping supply curve. D) They all help explain the downsloping demand curve. Answer: D

Type: A Topic: 4 E: 379 MI: 135 67. A consumer's demand curve for a product is downsloping because: A) total utility falls below marginal utility as more of a product is consumed. B) marginal utility diminishes as more of a product is consumed. C) time becomes less valuable as more of a product is consumed. D) the income and substitution effects precisely offset each other. Answer: B

Type: A Topic: 4 E: 378 MI: 134 68. The utility-maximizing rule: A) is inconsistent with the law of demand. B) implies a perfectly elastic demand curve. Answer: D

C) implies a leftward shifting demand curve. D) is consistent with the law of demand.

Use the following to answer questions 69-72: Answer the next question(s) on the basis of the following marginal utility data for products X and Y. Assume that the prices of X and Y are $4 and $2 respectively and that the consumer's income is $18.
Units of X 1 2 3 4 5 6 Marginal utility, X 20 16 12 8 6 4 Units of Y 1 2 3 4 5 6 Marginal utility, Y 16 14 12 10 8 6

Type: T Topic: 4 E: 376-377 MI: 132-133 69. Refer to the above data. What quantities of X and Y should be purchased to maximize utility? A) 2 of X and 1 of Y B) 4 of X and 5 of Y C) 2 of X and 5 of Y D) 2 of X and 6 of Y Answer: C

Type: T Topic: 4 E: 376 MI: 132 70. Refer to the above data. What level of total utility will the utility-maximizing consumer realize? A) 96 utils B) 108 utils C) 72 utils D) 142 utils Answer: A

McConnell/Brue: Economics, 16/e

Page 624

Chapter 21: Consumer Behavior and Utility Maximization

Type: T Topic: 4 E: 376 MI: 132 71. Refer to the above data. If the price of X decreases to $2, then the utility-maximizing combination of the two products is: A) 2 of X and 5 of Y. B) 4 of X and 6 of Y. C) 6 of X and 3 of Y. D) 4 of X and 5 of Y. Answer: D

Type: T Topic: 4 E: 379 MI: 135 72. Refer to the above data. Which of the following represents the demand schedule for X?

(a) P $4 2 Qd 2 5 P $4 2

(b) Qd 2 4 P $4 2

(c) Qd 3 6 P $4 2

(d) Qd 3 5

Answer: B

Use the following to answer questions 73-74:
Units of X 1 2 3 4 5 6 MUx 20 18 16 14 12 11 MUx/ Px = $2 ___ ___ ___ ___ ___ ___ MUx/ Px = $1 ___ ___ ___ ___ ___ ___ Units of Y 1 2 3 4 5 6 MUy 48 40 36 32 24 12 MUy/ Py = $4 ___ ___ ___ ___ ___ ___

Type: T Topic: 4 E: 378 MI: 134 73. If the price of X and Y are $2 and $4 per unit, respectively, to maximize total utility this consumer should buy: A) 1 units of X and 1 units of Y. C) 1 units of X and 2 units of Y. B) 2 units of X and 2 units of Y. D) 5 units of X and no units of Y. Answer: C

McConnell/Brue: Economics, 16/e

Page 625

Chapter 21: Consumer Behavior and Utility Maximization

Type: G Topic: 4 E: 378-379 MI: 134-135 74.

Refer to the above table and graph. Suppose that the price of X falls from $2 to $1, while the price of Y remains at $4. Which of the following represents the demand curve for X? A) D1 B) D2 C) D3 D) D4 Answer: B

Applications; extensions

Type: A Topic: 5 E: 380-381 MI: 136-137 75. Some modern theories of consumer behavior have: A) emphasized that consumption is basically an instantaneous act. B) contended that in the MUx/Px = MUy/Py equation MU is understated for time-intensive goods. C) introduced the opportunity cost of time as a component of product price. D) argued that inflationary expectations negate the theory of consumer behavior. Answer: C

Type: A Topic: 5 E: 380-381 MI: 136-137 76. In introducing the opportunity cost of time into the theory of consumer behavior we find that, all else equal: A) one should consume less of time-intensive goods. B) one should consume more of time-intensive goods. C) the consumer's equilibrium position is not altered. D) the marginal utility derived from each product must be multiplied by consumption time in determining equilibrium. Answer: A

Type: C Topic: 5 E: 380-381 MI: 136-137 77. Assume you are spending your full budget and purchasing such amounts of X and Y that the marginal utility from the last units consumed is 40 and 20 utils respectively. Assume (a) the prices of X and Y are $8 and $4 respectively; (b) it takes 3 hours to consume a unit of X and 1 hour to consume a unit of Y; and (c) your time is worth $2 per hour. You A) should substitute X for Y until the marginal utility per hour is the same for both products. B) are consuming X and Y in the optimal amounts. C) should consume less of Y and more of X. D) should consume less of X and more of Y. Answer: D

McConnell/Brue: Economics, 16/e

Page 626

Chapter 21: Consumer Behavior and Utility Maximization

Type: A Topic: 5 E: 380 MI: 136 78. Which of the following has been a significant factor in DVDs replacing video cassettes (VCs) in the retail home video market? A) DVDs are now less than one-half the price of VCs. B) A scarcity of production capacity has curtailed the manufacture of VCs. C) Most consumers perceive DVD sound and video reproduction to be of higher quality. D) The price of DVD players has increased dramatically. Answer: C

Type: A Topic: 5 E: 380 MI: 136 79. Consumer demand for DVDs has increased over time because the price of DVD players has: A) decreased, and DVD players and video cassette players are substitute goods. B) decreased, and DVD players and video cassette players are complementary goods. C) increased, and DVD players and video cassette players are substitute goods. D) increased, and DVD players and video cassette players are complementary goods. Answer: B

Type: D Topic: 5 E: 380 MI: 136 80. The diamond-water paradox arises because: A) essential goods may be cheap while nonessential goods may be expensive. B) the marginal utility of certain products increases, rather than diminishes. C) essential goods are always higher priced than nonessential goods. D) we sometimes fail to use money as a standard of value. Answer: A

Type: A Topic: 5 E: 380 MI: 136 81. The diamond-water paradox occurs because: A) the price of a product is related to its total utility, not its marginal utility. B) the price of a product is related to its marginal utility, not its total utility. C) water is, in fact, very scarce in certain regions of the world. D) diamonds are more useful than water. Answer: B

Type: A Topic: 5 E: 380 MI: 136 82. "Essential" water is cheaper than "nonessential" diamonds because: A) new industrial uses for diamonds have been discovered. B) the supply of water is great relative to demand and the supply of diamonds is small relative to demand. C) although the total utility of diamonds is greater, their marginal utility is small. D) the supply of diamonds is great relative to demand and the supply of water is small relative to demand. Answer: B

Type: A Topic: 5 E: 381 MI: 137 83. The fact that most medical care purchases are financed through insurance: A) has no effect on health care consumption because aggregate costs are the same regardless of payment method. B) reduces the amount of health care consumed. C) has decreased health care costs and therefore reduced aggregate health care expenditures. D) increases the amount of health care consumed. Answer: D

McConnell/Brue: Economics, 16/e

Page 627

Chapter 21: Consumer Behavior and Utility Maximization

Type: A Topic: 5 E: 381 MI: 137 84. Most economists contend that: A) noncash transfers are more efficient than cash transfers. B) noncash transfers are less efficient than cash transfers. C) noncash and cash transfers are equally efficient. D) government can assess consumer preferences better than can consumers themselves. Answer: B

Type: A Topic: 5 E: 381 MI: 137 85. Which of the following statements is correct? A) Both cash and noncash gift-giving cause value losses. B) Neither cash nor noncash gift-giving cause value losses. C) Noncash gift-giving create a value loss, but cash gifts do not. D) Cash gifts creates a value loss, but noncash gifts do not. Answer: C

Type: A Topic: 5 E: 381 MI: 137 86. Noncash gifts: A) increase the utility of recipients by introducing them to products they have not consumed before. B) reduce recipient utility relative to a cash gift because noncash gifts often fail to match recipient preferences. C) entail as much utility as do cash gifts. D) increase the utility of recipients because many people are uncertain of their own preferences. Answer: B

Type: A Topic: 5 E: 381 MI: 137 87. If you receive a gift whose market price is $20, but you consider it to be worth only $10, then: A) there is a $10 or 50 percent value gain. B) there may or may not be a value loss. C) there is a $10 or 50 percent value loss. D) you can be relatively certain the giver was a sibling or other close relative. Answer: C

Consider This Questions Type: A E: 375 MI: 131 Status: New 88. (Consider This) Newspapers dispensing devices seemingly "trust" people to take only a single paper but the devices actually rely on the law of: A) supply. B) increasing opportunity costs. C) demand. D) diminishing marginal utility. Answer: D

Type: A E: 375 MI: 131 Status: New 89. (Consider This) Unlike newspaper dispensing devices, soft drink dispensing machines do not permit people to take more than one can or bottle with each payment. The reason is that the: A) opportunity cost of additional cans or bottles of soft drink increase very rapidly. B) marginal utility of extra soft drink cans or bottles declines slowly, particularly because they are storable and can be consumed later. C) marginal utility of extra soft drink cans or bottles declines quite rapidly. D) opportunity cost of additional cans or bottles of soft drink increase very slowly. Answer: B

McConnell/Brue: Economics, 16/e

Page 628

Chapter 21: Consumer Behavior and Utility Maximization

Last Word Questions

Type: A E: 382 MI: 138 90. (Last Word) Theft and burglary: A) can be viewed as attempts to maximize utility, given certain marginal costs and marginal benefits. B) are examples of irrational behavior. C) are applications of the law of increasing opportunity costs. D) are examples of noneconomic behavior. Answer: A

Type: A E: 382 MI: 138 91. (Last Word) Most people do not steal because: A) the marginal utilities of stolen goods diminish as more of them are obtained. B) the marginal utilities of stolen goods are negative. C) their marginal costs, including guilt costs, are too high. D) stolen goods can be sold only at deep discounts. Answer: C

Type: A E: 382 MI: 138 92. (Last Word) All of the following would reduce property crime by increasing its "price," except: A) imposing greater penalties for those who are caught and convicted. B) using more sophisticated security systems. C) enhancing the legitimate earnings of potential criminals. D) cutting out the middlemen ("fences") by selling stolen goods via Internet auction sites. Answer: D

True/False Questions

Type: D E: 372 MI: 128 93. The income effect explains an exception to the law of demand. Answer: False

Type: A E: 373-374 MI: 129-130 94. If marginal utility is diminishing, total utility must also be declining. Answer: False

Type: A E: 373 MI: 129 95. The substitution effect suggests that, when consumers judge product quality by price, they will substitute high-priced products for low-priced products. Answer: False

Type: A E: 372-373 MI: 128-129 96. When the price of a product falls, the income effect induces the consumer to purchase more of it while the substitution effect prompts her to buy less. Answer: False

McConnell/Brue: Economics, 16/e

Page 629

Chapter 21: Consumer Behavior and Utility Maximization

Type: A E: 376 MI: 132 97. A rational consumer will cease purchasing a product at that quantity where marginal utility begins to diminish. Answer: False

Type: D E: 373 MI: 129 98. Marginal utility is total utility divided by the number of units consumed. Answer: False

Type: F E: 373-374 MI: 129-130 99. When total utility is at a maximum, marginal utility is zero. Answer: True

Type: A E: 376 MI: 132 100. The limited money income of consumers results in a so-called budget constraint. Answer: True

Type: A E: 376 MI: 132 101. When a consumer is maximizing total utility, he or she cannot increase total utility by reallocating expenditures among different products. Answer: True

Type: A E: 379 MI: 135 102. The consumer demand curve for a product is downsloping because marginal utility is constant when price declines. Answer: False

Type: A E: 380 MI: 136 103. Water has greater marginal utility than diamonds, yet diamonds have greater total utility than water. Answer: False

Type: A E: 381 MI: 137 104. Noncash gift giving involves value loss when the marginal utility of the gift to the receiver is less than the product price. Answer: True

Type: A E: 379 MI: 135 105. When a consumer shifts purchases from X to Y, the marginal utility of X falls and the marginal utility of Y rises. Answer: False

McConnell/Brue: Economics, 16/e

Page 630

Chapter 21: Consumer Behavior and Utility Maximization

Appendix Questions Multiple Choice Questions Budget lines

Type: D Topic: 6 E: 386 MI: 142 106. The budget line shows: A) the amount of product A that a consumer is willing to give up to obtain one more unit of product B. B) all possible combinations of two goods that can be purchased, given money income and the prices of the goods. C) all equilibrium points on an indifference map. D) all possible combinations of two goods that yield the same level of utility to the consumer. Answer: B

Type: D Topic: 6 E: 386 MI: 142 107. Which of the following statements is not correct? A) A reduction in money income will shift the budget line to the right. B) A reduction in money income accompanied by an increase in product prices will necessarily shift the budget line to the left. C) An increase in product prices will shift the budget line to the left. D) An increase in money income will shift the budget line to the right. Answer: A

Use the following to answer questions 108-110:

Type: A Topic: 6 E: 386 MI: 142 108. Refer to the budget line shown in the diagram above. If the consumer's money income is $20, the: A) prices of C and D cannot be determined. B) price of C is $2 and the price of D is $4. C) consumer can obtain a combination of 5 units of both C and D. D) price of C is $4 and the price of D is $2. Answer: D

McConnell/Brue: Economics, 16/e

Page 631

Chapter 21: Consumer Behavior and Utility Maximization

Type: G Topic: 6 E: 386-387 MI: 142-143 109. Refer to the budget line shown in the diagram above. Given the same money income, reductions in the prices of both products C and D will: A) shift the budget line outward on the horizontal axis, but leave it anchored at "10" on the vertical axis. B) shift the budget line to the left. C) shift the budget line to the right. D) have no effect on the budget line. Answer: C

Type: G Topic: 6 E: 387 MI: 143 110. Refer to the budget line shown in the diagram above. The absolute value of the slope of the budget line is: A) MUC/MUD. B) one-half. C) PD/PC. D) PC/PD. Answer: D

Type: E Topic: 6 E: 386 MI: 142 111. In moving along a given budget line: A) the prices of both products and money income are assumed to be constant. B) each point on the line will be equally satisfactory to consumers. C) money income varies, but the prices of the two goods are constant. D) the prices of both products are assumed to vary, but money income is constant. Answer: A

Type: A Topic: 6 E: 386-387 MI: 142-143 112. Increases in product prices shift the consumer's: A) budget line to the right. B) budget line to the left. Answer: B

C) indifference curves to the left. D) indifference curves to the right.

Type: A Topic: 6 E: 386 MI: 142 113. An increase in money income shifts the consumer's: A) budget line to the right. C) indifference curves to the left. B) budget line to the left. D) indifference curves to the right. Answer: A

Type: A Topic: 6 E: 387 MI: 143 114. A change in the slope of a budget line is solely the result of a change in: A) consumer preferences. C) money income. B) the price of one or both goods. D) the marginal rate of substitution. Answer: B

Type: A Topic: 6 E: 386 MI: 142 115. In drawing a budget line it is assumed that: A) consumer preferences are fixed. B) the prices of the two products are variable. C) money income is fixed. D) consumer willingness to substitute between the two products is fixed. Answer: C

McConnell/Brue: Economics, 16/e

Page 632

Chapter 21: Consumer Behavior and Utility Maximization

Type: D Topic: 6 E: 386 MI: 142 116.

The shift of the budget line from cd to ab in the above figure is consistent with: A) decreases in the prices of both M and N . B) an increase in the price of M and a decrease in the price of N . C) a decrease in money income. D) an increase in money income. Answer: C

Type: G Topic: 6 E: 386 MI: 142 117.

The budget line shift from ab to cd in the above figure is consistent with: A) decreases in the prices of both M and N . B) an increase in the price of M and a decrease in the price of N . C) a decrease in money income. D) an increase in money income. Answer: B

Type: G Topic: 6 E: 386 MI: 142 118. Any combination of goods lying outside of the budget line: A) implies that the consumer is not spending all his income. B) yields less utility than any point on the budget line. C) yields less utility than any point inside the budget line. D) is unobtainable, given the consumer's income. Answer: D

McConnell/Brue: Economics, 16/e

Page 633

Chapter 21: Consumer Behavior and Utility Maximization

Type: A Topic: 6 E: 386-387 MI: 142-143 119. If money income increases and the prices of products A and B both increase, then the budget line: A) must shift to the right. B) must shift to the left. C) may shift either to the right or the left. D) will no longer be tangent to an indifference curve. Answer: C

Type: C Topic: 6 E: 386-387 MI: 142-143 120.

The movement of the budget line from BB to bb in the above figure suggests that income has: A) increased and the price of X has decreased. B) fallen and the price of Y has increased. C) fallen and the price of X has decreased. D) decreased but there have been no price changes. Answer: B

Type: G Topic: 6 E: 386 MI: 142 121.

Suppose you have a money income of $10 all of which you spend on Coke and popcorn. In the above diagram, the prices of Coke and popcorn respectively are: A) $.50 and $1.00. B) $1.00 and $.50. C) $1.00 and $2.00. D) $.40 and $.50. Answer: A

McConnell/Brue: Economics, 16/e

Page 634

Chapter 21: Consumer Behavior and Utility Maximization

Type: G Topic: 6 E: 386-387 MI: 142-143 122.

If the budget line shifts from BB to bb in the above diagram we can infer that the: A) price of Y has increased and the price of X has decreased. B) price of Y has decreased and the price of X has increased. C) prices of both X and Y have increased. D) prices of both X and Y have decreased. Answer: A

Type: G Topic: 6 E: 386 MI: 142 123. A budget line shows the: A) alternative combinations of two goods that a consumer can purchase with a given money income. B) alternative combinations of two goods that will yield the same level of total utility to a consumer. C) quantities of a particular good that a consumer will buy at various prices. D) ratio of money income to product price. Answer: A

Type: D Topic: 6 E: 386 MI: 142 124. Other things equal, an increase in a consumer's money income: A) shifts her indifference curves rightward because she can now satisfy more of her wants. B) shifts her budget line rightward because she can now purchase more of both products. C) will be subject to the substitution effect, but not the income effect. D) will not alter the location of consumer equilibrium. Answer: B

Type: A Topic: 6 E: 386 MI: 142 125. The slope of a budget line reflects the: A) elasticity of demand for the two products. B) price ratio of the two products. Answer: B

C) amount of the consumer's income. D) utility ratio of the two products.

Type: D Topic: 6 E: 386 MI: 142 126. If the price of A is $12 and the price of B is $3, the budget line tells us that a consumer in effect can trade: A) 12 units of A for 3 of B. C) 1 unit of A for 3 of B. B) 1 unit of A for 4 of B. D) 1 unit of B for 4 of A. Answer: B

McConnell/Brue: Economics, 16/e

Page 635

Chapter 21: Consumer Behavior and Utility Maximization

Type: A Topic: 6 E: 386-387 MI: 142-143 127. Assume the price of product Y (the quantity of which is plotted on the vertical axis) is initially $15 and the price of X (the quantity of which is plotted on the horizontal axis) is initially $3. Assume money income is initially $60. If the prices of Y and X now increase to $30 and $6 respectively and money income increases to $120, then the budget line will: A) shift rightward and become steeper. C) shift rightward, but its slope will not change. B) shift rightward and become flatter. D) be unchanged. Answer: D

Type: A Topic: 6 E: 386-387 MI: 142-143 128. Assume initially that the price of X (measured on the horizontal axis) is $9 and the price of Y (measured on the vertical axis) is $4. If the price of X now declines to $6, the budget line will: A) be unaffected. C) shift inward on the horizontal axis. B) shift outward on the vertical axis. D) shift outward on the horizontal axis. Answer: D

Type: A Topic: 6 E: 386 MI: 142 129.

Suppose Elroy's budget line is as shown on the above diagram. If his tastes change in favor of Coke and against popcorn, the budget line will: A) become steeper. B) become flatter. C) shift rightward. D) be unaffected. Answer: D

Type: G Topic: 6 E: 386 MI: 142 130. Edith is buying products X and Y with her money income. Suppose her budget line shifts rightward (outward). This might be the result of: A) the prices of X and Y increasing while her money income remains constant. B) her money income decreasing while the prices of X and Y remain constant. C) her money income increasing more than proportionately to increases in the prices of X and Y. D) none of the above. Answer: C

Type: A Topic: 6 E: 386 MI: 142 131. Assume the price of product Y (the quantity of which is on the vertical axis) is $15 and the price of product X (the quantity of which is on the horizontal axis) is $3. Also assume that money income is $60. The absolute value of the slope of the resulting budget line: A) is 5. B) is 1/5. C) is 4. D) is 20. Answer: B

McConnell/Brue: Economics, 16/e

Page 636

Chapter 21: Consumer Behavior and Utility Maximization

Indifference curves

Type: A Topic: 7 E: 387 MI: 143 132.

The indifference curve in the above diagram yields Juan 100 units of utility. If Juan's money income were to increase by 20 percent, the indifference curve would: A) shift leftward. B) shift rightward. C) become steeper. D) not be affected. Answer: D

Type: G Topic: 7 E: 387 MI: 143 133. At each point on an indifference curve: A) money income is the same. B) the prices of the two products are the same. Answer: C

C) total utility is the same. D) marginal utility is the same.

Type: A Topic: 7 E: 387 MI: 143 134. An indifference curve shows all: A) possible equilibrium positions on an indifference map. B) equilibrium combinations of two products that are obtainable with a given money income. C) combinations of two products yielding the same total utility to a consumer. D) possible combinations of two products that a consumer can purchase, given her income and the prices of the products. Answer: C

Type: D Topic: 7 E: 387 MI: 143 135. An indifference curve: A) may be either upsloping or downsloping, depending on whether the two products are complements or substitutes. B) is downsloping and convex to the origin. C) is upsloping and has a constant slope. D) is downsloping and concave to the origin. Answer: B

McConnell/Brue: Economics, 16/e

Page 637

Chapter 21: Consumer Behavior and Utility Maximization

Type: D Topic: 7 E: 388 MI: 144 136. An indifference map implies that: A) money income is constant, but the prices of the two products vary directly with the quantities purchased. B) the two products under consideration are perfectly substitutable for one another. C) a consumer is better off to be at some point high on a given curve as opposed to a point low on the same curve. D) curves farther from the origin yield higher levels of total utility. Answer: D

Type: A Topic: 7 E: 387 MI: 143 137. The marginal rate of substitution measures the: A) magnitude of the substitution effect. B) total utility received by a consumer when equilibrium is achieved. C) extra utility that a consumer derives from successive units of a product. D) consumer's willingness to substitute one product for another so that total utility will remain constant. Answer: D

Type: D Topic: 7 E: 387 MI: 143 138. The marginal rate of substitution: A) may increase or decrease on a given indifference curve, depending on whether the substitution or the income effect is dominant. B) increases as one moves southeast along an indifference curve. C) is constant at all points on the budget line. D) declines as one moves southeast along an indifference curve. Answer: D

Type: A Topic: 7 E: 387 MI: 143 139. Which of the following is not characteristic of indifference curves? A) They are downsloping. B) They are convex to the origin. C) Their slope diminishes as we move from northwest to southeast on a given curve. D) Curves closer to the origin reflect higher levels of total utility. Answer: D

Type: A Topic: 7 E: 386-387 MI: 142-143 140. Which of the following is correct? A) Budget lines are linear and upsloping; indifference curves are downsloping and concave to the origin. B) Budget lines are linear and downsloping; indifference curves are downsloping and concave to the origin. C) Budget lines are linear and downsloping; indifference curves are downsloping and convex to the origin. D) Budget lines are downsloping and convex to the origin; indifference curves are linear and downsloping. Answer: C

McConnell/Brue: Economics, 16/e

Page 638

Chapter 21: Consumer Behavior and Utility Maximization

Type: A Topic: 7 E: 387 MI: 143 141. Indifference curve analysis: A) presumes, as does utility analysis, that satisfaction is numerically measurable. B) presumes, unlike utility analysis, that satisfaction is numerically measurable. C) presumes only that the consumer can say one combination of two goods yields more or less utility than some other combination. D) is in conflict with the idea of a downsloping demand curve. Answer: C

Consumer equilibrium; changes in equilibrium

Use the following to answer questions 142-145:

Type: A Topic: 8 E: 388-389 MI: 144-145 142. Refer to the above diagram where xy is the relevant budget line and I1 , I2, and I3 are indifference curves. The equilibrium position for the consumer is at: A) any point on xy. B) point M. C) point K. D) point J. Answer: C

Type: G Topic: 8 E: 388-389 MI: 144-145 143. Refer to the above diagram where xy is the relevant budget line and I1 , I2, and I3 are indifference curves. If the consumer is initially at point L, he or she should: A) strive for point N by obtaining a larger money income. B) purchase more of X and less of Y . C) remain at that point to maximize utility. D) purchase more of Y and less of X . Answer: D

McConnell/Brue: Economics, 16/e

Page 639

Chapter 21: Consumer Behavior and Utility Maximization

Type: G Topic: 8 E: 389 MI: 145 144. Refer to the above diagram where xy is the relevant budget line and I1 , I2, and I3 are indifference curves. Point M : A) is the consumer's equilibrium position. B) is unobtainable. C) is inferior to point N . D) entails the highest attainable level of total utility. Answer: B

Type: G Topic: 8 E: 389 MI: 145 145. Refer to the above diagram where xy is the relevant budget line and I1 , I2, and I3 are indifference curves. At point K: A) MUx = MUy. B) MRS = Px/Py. C) MRS = Py/Px. D) Px exceeds Py. Answer: B

Type: G Topic: 8 E: 390 MI: 146 146.

AA is Al's indifference curve and BB is Betty's. Al and Betty have the same budget line, LL. This information implies that: A) Al's demand for X is stronger than Betty's. B) Al's demand for Y is stronger than Betty's. C) Al and Betty have the same demand for both products. D) Al will buy some of X, but Betty will not. Answer: B

McConnell/Brue: Economics, 16/e

Page 640

Chapter 21: Consumer Behavior and Utility Maximization

Type: G Topic: 8 E: 390 MI: 146 147.

In the above diagram: A) the consumer is indifferent between points A and B, but neither point maximizes his utility. B) the consumer is indifferent between points A and B and either point will maximize his utility. C) any combination of X and Y entailing more of Y and less of X than shown at B would be preferred. D) any combination of X and Y entailing more of X and less of Y than shown at A would be preferred. Answer: A

Type: G Topic: 8 E: 386 MI: 142 148. If a consumer is initially in equilibrium, an increase in money income will: A) move him to a new equilibrium on a lower indifference curve. B) make his indifference curves steeper, but will not alter the equilibrium position. C) have no effect on the equilibrium position because product prices have not changed. D) move him to a new equilibrium on a higher indifference curve. Answer: D

Type: A Topic: 8 E: 388-389 MI: 144-145 149. Assume a diagram in which a budget line is imposed on an indifference map. A consumer will maximize her utility: A) at any point where the budget line and an indifference curve intersect. B) at either point where the budget line intersects the horizontal and vertical axes. C) where the budget line is tangent to an indifference curve. D) where the ratio of the two product prices equals the reciprocal of the consumer's income. Answer: C

Type: A Topic: 8 E: 388-389 MI: 144-145 150. Indifference curve analysis indicates that consumer equilibrium exists: A) where an indifference curve has a slope of 1. B) where any two indifference curves intersect. C) at any point where the budget line intersects an indifference curve. D) where the budget line touches the highest possible indifference curve. Answer: D

McConnell/Brue: Economics, 16/e

Page 641

Chapter 21: Consumer Behavior and Utility Maximization

Use the following to answer questions 151-153:

Type: A Topic: 8 E: 389 MI: 145 151. Refer to the above diagram in which the downsloping linear lines are budget lines and I1, I 2, and I3 comprise an indifference map. The combinations of products M and N indicated by points 1, 2, and 5 are such that: A) point 2 yields more utility than either 1 or 5. B) points 1 and 5 yield more utility than point 2. C) points 1, 2, and 5 yield equal amounts of utility. D) the levels of utility associated with these three points cannot be compared. Answer: C

Type: G Topic: 8 E: 389 MI: 145 152. Refer to the above diagram in which the downsloping linear lines are budget lines and I1, I 2, and I3 comprise an indifference map. The combinations of products M and N indicated by points 1, 3, and 5 are such that: A) all three imply the same level of utility. B) 1 and 5 imply a higher level of utility than does 3. C) 3 implies a higher level of utility than does 1 or 5. D) the person is indifferent among the three combinations. Answer: C

Type: G Topic: 8 E: 389 MI: 145 153. Refer to the above diagram in which the downsloping linear lines are budget lines and I1, I 2, and I3 comprise an indifference map. With respect to points 2, 3, and 4, we can say that: A) point 4 entails more utility than 3, but less than 2. B) point 4 entails more utility than 2, but less than 3. C) point 4 yields more utility than either 2 or 3. D) the levels of utility associated with these three points cannot be compared. Answer: C

McConnell/Brue: Economics, 16/e

Page 642

Chapter 21: Consumer Behavior and Utility Maximization

Type: G Topic: 8 E: 386 MI: 142 154. If a consumer chooses a combination of goods that lies inside of her budget line, the consumer: A) is maximizing utility. B) is spending in excess of her current income. C) could obtain more goods with her money income. D) has upsloping indifference curves. Answer: C

Use the following to answer questions 155-157:

Type: A Topic: 8 E: 389 MI: 145 155. Refer to the above diagram. If the budget line shifts from ab to ac the: A) price of K has increased. C) price of K has decreased. B) consumer's money income has fallen. D) price of J has increased. Answer: A

Type: G Topic: 8 E: 390 MI: 146 156. Refer to the above diagram. If the budget line shifts from ab to ac the: A) consumer's level of total utility will increase. B) consumer will purchase more of both J and K. C) consumer will purchase less of both J and K. D) consumer will purchase more of J and less of K. Answer: D

Type: G Topic: 8 E: 390 MI: 146 157. Refer to the above diagram. The equilibrium points shown in the diagram along with the price change that produced the shift of the budget line from ab to ac: A) are consistent with a downsloping demand curve for product K. B) imply that the consumer's money income has declined, but his or her real income has increased. C) imply consumer irrationality since the dearer product is being substituted for the cheaper product. D) suggest that K is an inferior good. Answer: A

McConnell/Brue: Economics, 16/e

Page 643

Chapter 21: Consumer Behavior and Utility Maximization

Use the following to answer questions 158-160:

Type: G Topic: 8 E: 390 MI: 146 158. Refer to the above diagram. Suppose the budget line shifts so that the consumer's equilibrium changes from point A to point B. This means that the: A) price of Y has increased. C) price of X has increased. B) price of Y has decreased. D) consumer's money income has increased. Answer: B

Type: G Topic: 8 E: 390 MI: 146 159. Refer to the above diagram. The budget line shift that moves the consumer's equilibrium from point A to point B suggests: A) an increase in the demand for product X . C) no change in the demand for product X . B) a decrease in the demand for product X . D) that X is an inferior good. Answer: B

Type: G Topic: 8 E: 390 MI: 146 160. Refer to the above diagram. The budget line shift which moves the consumer's equilibrium position from point A to point B suggests: A) an increase in the quantity of Y demanded. C) a leftward shift in the demand curve for Y . B) a decrease in the quantity of Y demanded. D) a rightward shift in the demand curve for Y . Answer: A

Consider This Questions

Type: F E: 389 MI: 145 Status: New 161. (Consider This) The lines on a topographical map are analogous to a(n): A) series of demand curves. C) indifference map. B) series of supply curves. D) series of budget constraints. Answer: C

Type: A E: 389 MI: 145 Status: New 162. (Consider This) A topographical map shows successively higher equal-elevation lines, whereas an indifference map shows successively higher levels of total: A) utility. B) revenue. C) profit. D) cost. Answer: A

McConnell/Brue: Economics, 16/e

Page 644

Chapter 21: Consumer Behavior and Utility Maximization

True/False Questions

Type: G E: 387 MI: 143 163. Indifference curves are linear and budget lines are convex to the origin. Answer: False

Type: A E: 388-389 MI: 144-145 164. Graphically, the consumer maximizes total utility where the budget line is tangent to an indifference curve. Answer: True

Type: A E: 386-387 MI: 142-143 165. In drawing a particular budget line, money income and the prices of the two products are fixed. Answer: True

Type: A E: 386-387 MI: 142-143 166. With fixed money income, an increase in the price of one good and a decrease in the price of the other will cause the new budget line to intersect the original budget line. Answer: True

Type: A E: 387-388 MI: 143-144 167. In moving northeasterly from the origin we encounter indifference curves that reflect higher and higher levels of total utility. Answer: True

Type: A E: 387 MI: 143 168. Each point on a single indifference curve reflects the same level of total utility for a consumer. Answer: True

Type: A E: 388 MI: 144 169. It is possible for a consumer's indifference curves to intersect. Answer: False

Type: A E: 388 MI: 144 170. A rational consumer will try to achieve the highest indifference curve that his or her income will allow. Answer: True

Type: A E: 387 MI: 143 171. Indifference analysis assumes that utility is numerically measurable. Answer: False

Type: A E: 388 MI: 144 172. As a consumer moves down a given indifference curve, his or her total utility will diminish. Answer: False

McConnell/Brue: Economics, 16/e

Page 645

Chapter 21: Consumer Behavior and Utility Maximization

Type: A E: 390 MI: 146 173. Marginal utility theory and indifference curve analysis are both consistent with the law of demand. Answer: True

Type: A E: 386 MI: 142 174. The lower the consumer's income, the higher his or her budget line. Answer: False

McConnell/Brue: Economics, 16/e

Page 646

CHAPTER 22

The Costs of Production

Topic 1. 2. 3. 4. 5. 6. Costs: explicit and implicit Profits Short run versus long run Law of diminishing returns Short-run costs Long-run costs Last Word True-False

Question numbers 1-9 10-23 24-31 32-55 56-157 158-193 194-196 197-210

____________________________________________________________

_______________________________________

____________________________________________________________

_______________________________________

Multiple Choice Questions Costs: explicit and implicit

Type: D Topic: 1 E: 392 MI: 148 1. Economic cost can best be defined as: A) any contractual obligation that results in a flow of money expenditures from an enterprise to resource suppliers. B) any contractual obligation to labor or material suppliers. C) compensations that must be received by resource owners to insure their continued supply. D) all costs exclusive of payments to fixed factors of production. Answer: C

Type: A Topic: 1 E: 393 MI: 149 2. Which of the following constitutes an implicit cost to the Johnston Manufacturing Company? A) payments of wages to its office workers B) rent paid for the use of equipment owned by the Schultz Machinery Company C) depreciation charges on company-owned equipment D) economic profits resulting from current production Answer: C

Type: A Topic: 1 E: 393 MI: 149 3. Which of the following is most likely to be an implicit cost for Company X? A) depreciation charges on company-owned equipment B) rental payments on IBM equipment C) payments for raw materials purchased from Company Y D) transportation costs paid to a nearby trucking firm Answer: A

Chapter 22: The Costs of Production

Type: A Topic: 1 E: 393 MI: 149 4. Costs to an economist: A) consist only of explicit costs. B) may or may not involve monetary outlays. Answer: B

C) never reflect monetary outlays. D) always reflect monetary outlays.

Type: A Topic: 1 E: 393 MI: 149 5. What do wages paid to blue-collar workers, interest paid on a bank loan, forgone interest, and the purchase of component parts have in common? A) None are either implicit or explicit costs. C) All are implicit costs. B) All are opportunity costs. D) All are explicit costs. Answer: B

Type: A Topic: 1 E: 393 MI: 149 6. To the economist total cost includes: A) explicit and implicit costs, including a normal profit. B) neither implicit nor explicit costs. C) implicit, but not explicit, costs. D) explicit, but not implicit, costs. Answer: A

Type: D Topic: 1 E: 393 MI: 149 7. Implicit and explicit costs are different in that: A) explicit costs are relevant only in the short run. B) implicit costs are relevant only in the short run. C) the latter refer to nonexpenditure costs and the former to out-of-pocket costs. D) the former refer to nonexpenditure costs and the latter to out-of-pocket costs. Answer: D

Type: D Topic: 1 E: 393 MI: 149 8. Implicit costs are: A) regarded as costs by accountants but not by economists. B) payments that a firm makes to other firms or individuals who supply resources to it. C) nonexpenditure costs. D) costs that vary proportionately with output. Answer: C

Type: D Topic: 1 E: 393 MI: 149 9. An explicit cost is: A) omitted when accounting profits are calculated. B) a money payment made for resources not owned by the firm itself. C) an implicit cost to the resource owner who receives that payment. D) always in excess of a resource's opportunity cost. Answer: B

McConnell/Brue: Economics, 16/e

Page 648

Chapter 22: The Costs of Production

Profits

Type: A Topic: 2 E: 393 MI: 149 10. Accounting profits are typically: A) greater than economic profits because the former do not take explicit costs into account. B) equal to economic profits because accounting costs include all opportunity costs. C) smaller than economic profits because the former do not take implicit costs into account. D) greater than economic profits because the former do not take implicit costs into account. Answer: D

Type: A Topic: 2 E: 393 MI: 149 11. Economic profits are calculated by subtracting: A) explicit costs from total revenue. B) implicit costs from total revenue. Answer: D

C) implicit costs from normal profits. D) explicit and implicit costs from total revenue.

Type: D Topic: 2 E: 393 MI: 149 12. Normal profit is: A) determined by subtracting implicit costs from total revenue. B) determined by subtracting explicit costs from total revenue. C) the return to the entrepreneur when economic profits are zero. D) the average profitability of an industry over the preceding 10 years. Answer: C

Type: A Topic: 2 E: 393 MI: 149 13. Which of the following definitions is correct? A) Accounting profit + economic profit = normal profit. B) Economic profit - accounting profit = explicit costs. C) Economic profit = accounting profit - implicit costs. D) Economic profit - implicit costs = accounting profits. Answer: C

Type: A Topic: 2 E: 393 MI: 149 14. Suppose that a business incurred implicit costs of $200,000 and explicit costs of $1 million in a specific year. If the firm sold 4,000 units of its output at $300 per unit, its accounting profits were: A) $100,000 and its economic profits were zero. B) $200,000 and its economic profits were zero. C) $100,000 and its economic profits were $100,000. D) zero and its economic loss was $200,000. Answer: B

Type: A Topic: 2 E: 393 MI: 149 15. Suppose that a business incurred implicit costs of $500,000 and explicit costs of $5 million in a specific year. If the firm sold 100,000 units of its output at $50 per unit, its accounting: A) profits were $100,000 and its economic profits were zero. B) losses were $500,000 and its economic losses were zero. C) profits were $500,000 and its economic profits were $1 million. D) profits were zero and its economic losses were $500,000. Answer: D

McConnell/Brue: Economics, 16/e

Page 649

Chapter 22: The Costs of Production

Use the following to answer questions 16-23: Use the following cost information for the Creamy Crisp Donut Company to answer questions 16-23: Entrepreneur's potential earnings as a salaried worker = $50,000 Annual lease on building = $22,000 Annual revenue from operations = $380,000 Payments to workers = $120,000 Utilities (electricity, water, disposal) costs = $8,000 Entrepreneur's potential economic profit from the next best entrepreneurial activity = $80,000 Entrepreneur's forgone interest on personal funds used to finance the business = $6,000

Type: A Topic: 2 E: 393 MI: 149 16. Refer to the above data. Creamy Crisp's explicit costs are: A) $286,000. B) $150,000. C) $94,000. D) $156,000. Answer: B

Type: A Topic: 2 E: 393 MI: 149 17. Refer to the above data. Creamy Crisp's implicit costs, including a normal profit are: A) $136,000. B) $150,000. C) $94,000. D) $156,000. Answer: A

Type: A Topic: 2 E: 393 MI: 149 18. Refer to the above data. Creamy Crisp's total economic costs (explicit + implicit costs, including a normal profit) are: A) $286,000. B) $150,000. C) $94,000. D) $156,000. Answer: A

Type: A Topic: 2 E: 393 MI: 149 19. Refer to the above data. Creamy Crisp's accounting profit is: A) $150,000. B) $380,000. C) $230,000. D) $294,000. Answer: C

Type: A Topic: 2 E: 393 MI: 149 20. Refer to the above data. Creamy Crisp's economic profit is: A) $150,000. B) $80,000. C) $230,000. D) $94,000. Answer: D

Type: A Topic: 2 E: 393 MI: 149 21. Refer to the above data. Creamy Crisp's total revenues exceed its total costs, including a normal profit, by: A) $150,000. B) $94,000. C) $80,000. D) $230,000. Answer: B

McConnell/Brue: Economics, 16/e

Page 650

Chapter 22: The Costs of Production

Type: A Topic: 2 E: 393 MI: 149 22. Refer to the above data. Creamy Crisp: A) has lower implicit costs, including a normal profit, than its explicit costs. B) is earning a normal profit but not an economic profit. C) is earning an economic profit. D) is suffering an economic loss, when implicit costs are considered. Answer: C

Type: A Topic: 2 E: 393 MI: 149 23. Refer to the above data. If, other things equal, Creamy Crisp's revenue fell to $286,000: A) its implicit costs, including a normal profit, would exceed its explicit costs. B) it would earn a normal profit but not an economic profit. C) it would suffer an economic loss. D) its accounting profit would fall to zero. Answer: B

Short run versus long run

Type: A Topic: 3 E: 394 MI: 150 24. The basic characteristic of the short run is that: A) barriers to entry prevent new firms from entering the industry. B) the firm does not have sufficient time to change the size of its plant. C) the firm does not have sufficient time to cut its rate of output to zero. D) a firm does not have sufficient time to change the amounts of any of the resources it employs. Answer: B

Type: A Topic: 3 E: 394 MI: 150 25. Which of the following represents a long-run adjustment? A) a farmer uses an extra dose of fertilizer on his corn crop B) unable to meet foreign competition, a U.S. watch manufacturer sells one of its branch plants C) a steel manufacturer cuts back on its purchases of coke and iron ore D) a supermarket hires four additional clerks Answer: B

Type: A Topic: 3 E: 394 MI: 150 26. Which of the following is a short-run adjustment? A) A local bakery hires two additional bakers. B) Six new firms enter the plastics industry. C) The number of farms in the United States declines by 5 percent. D) BMW constructs a new assembly plant in South Carolina. Answer: A

Type: D Topic: 3 E: 394 MI: 150 27. To economists the main difference between the short run and the long run is that: A) the law of diminishing returns applies in the long run, but not in the short run. B) in the long run all resources are variable, while in the short run at least one resource is fixed. C) fixed costs are more important to decision making in the long run than they are in the short run. D) in the short run all resources are fixed, while in the long run all resources are variable. Answer: B

McConnell/Brue: Economics, 16/e

Page 651

Chapter 22: The Costs of Production

Type: A Topic: 3 E: 394 MI: 150 28. The amount of calendar time associated with the long run: A) is less than that associated with the immediate market period. B) varies from industry to industry. C) is the same for all firms. D) is one year by definition. Answer: B

Type: A Topic: 3 E: 394 MI: 150 29. The basic difference between the short run and the long run is that: A) all costs are fixed in the short run, but all costs are variable in the long run. B) the law of diminishing returns applies in the long run, but not in the short run. C) at least one resource is fixed in the short run, while all resources are variable in the long run. D) economies of scale may be present in the short run, but not in the long run. Answer: C

Type: A Topic: 3 E: 394 MI: 150 30. The short run is characterized by: A) plenty of time for firms to either enter or leave the industry. B) increasing, but not diminishing returns. C) at least one fixed resource. D) zero fixed costs. Answer: C

Type: A Topic: 3 E: 394 MI: 150 31. The long run is characterized by: A) the relevance of the law of diminishing returns. B) at least one fixed input. C) insufficient time for firms to enter or leave the industry. D) the ability of the firm to change its plant size. Answer: D

Law of diminishing returns

Type: D Topic: 4 E: 395 MI: 151 32. Marginal product is: A) the increase in total output attributable to the employment of one more worker. B) the increase in total revenue attributable to the employment of one more worker. C) the increase in total cost attributable to the employment of one more worker. D) total product divided by the number of workers employed. Answer: A

McConnell/Brue: Economics, 16/e

Page 652

Chapter 22: The Costs of Production

Type: D Topic: 4 E: 395 MI: 151 33. The law of diminishing returns indicates that: A) as extra units of a variable resource are added to a fixed resource, marginal product will decline beyond some point. B) because of economies and diseconomies of scale a competitive firm's long-run average total cost curve will be U-shaped. C) the demand for goods produced by purely competitive industries is downsloping. D) beyond some point the extra utility derived from additional units of a product will yield the consumer smaller and smaller extra amounts of satisfaction. Answer: A

Type: A Topic: 4 E: 395-396 MI: 151-152 34. Which of the following statements concerning the relationships between total product (TP), average product (AP), and marginal product (MP) is not correct? A) AP continues to rise so long as TP is rising. B) AP reaches a maximum before TP reaches a maximum. C) TP reaches a maximum when the MP of the variable input becomes zero. D) MP cuts AP at the maximum AP. Answer: A

Type: D Topic: 4 E: 396 MI: 152 35. Which of the following best expresses the law of diminishing returns? A) Because large-scale production allows the realization of economies of scale, the real costs of production vary directly with the level of output. B) Population growth automatically adjusts to that level at which the average product per worker will be at a maximum. C) As successive amounts of one resource (labor) are added to fixed amounts of other resources (property), beyond some point the resulting extra output will decline. D) Proportionate increases in the inputs of all resources will result in a less-than-proportionate increase in total output. Answer: C

Use the following to answer questions 36-38: Answer the next question(s) on the basis of the following output data for a firm. Assume that the amounts of all nonlabor resources are fixed.
Number of workers 0 1 2 3 4 5 6 Units of output 0 40 90 126 150 165 180

Type: T Topic: 4 E: 396 MI: 152 36. Refer to the above data. Diminishing marginal returns become evident with the addition of the: A) sixth worker. B) fourth worker. C) third worker. D) second worker. Answer: C

McConnell/Brue: Economics, 16/e

Page 653

Chapter 22: The Costs of Production

Type: T Topic: 4 E: 396 MI: 152 37. Refer to the above data. The marginal product of the sixth worker is: A) 180 units of output. B) 30 units of output. C) 15 units of output. Answer: C

D) negative.

Type: T Topic: 4 E: 396 MI: 152 38. Refer to the above data. Average product is at a maximum when: A) five workers are hired. C) three workers are hired. B) four workers are hired. D) two workers are hired. Answer: D

Type: A Topic: 4 E: 395 MI: 151 39. Marginal product: A) diminishes at all levels of production. B) may initially increase, then diminish, but never become negative. C) may initially increase, then diminish, and ultimately become negative. D) is always less than average product. Answer: C

Type: A Topic: 4 E: 395-396 MI: 151-152 40. The first, second, and third workers employed by a firm add 24, 18, and 9 units to total product respectively. Therefore, the: A) marginal product of the third worker is 9. C) average product of the three workers is 18. B) total product of the three workers is 54. D) marginal product of the second worker is 18. Answer: A

Type: D Topic: 4 E: 395 MI: 151 41. If a variable input is added to some fixed input, beyond some point the resulting extra output will decline. This statement describes: A) economies and diseconomies of scale. C) the law of diminishing returns. B) X-inefficiency. D) the law of diminishing marginal utility. Answer: C

Type: A Topic: 4 E: 395 MI: 151 42. If in the short run a firm's total product is increasing, then its: A) marginal product must also be increasing. B) marginal product must be decreasing. C) marginal product could be either increasing or decreasing. D) average product must also be increasing. Answer: C

Type: A Topic: 4 E: 396-397 MI: 152-153 43. The law of diminishing returns results in: A) an eventually rising marginal product curve. B) a total product curve that eventually increases at a decreasing rate. C) an eventually falling marginal cost curve. D) a total product curve that rises indefinitely. Answer: B

McConnell/Brue: Economics, 16/e

Page 654

Chapter 22: The Costs of Production

Type: A Topic: 4 E: 395 MI: 151 44. The law of diminishing returns describes the: A) relationship between total costs and total revenues. B) profit-maximizing position of a firm. C) relationship between resource inputs and product outputs in the short run. D) relationship between resource inputs and product outputs in the long run. Answer: C

Type: A Topic: 4 E: 396 MI: 152 45. Which of the following is correct? A) When total product is rising, both average product and marginal product must also be rising. B) When marginal product is falling, total product must be falling. C) When marginal product is falling, average product must also be falling. D) Marginal product rises faster than average product and also falls faster than average product. Answer: D

Type: A Topic: 4 E: 396 MI: 152 46. Which of the following is not correct? A) Where marginal product is greater than average product, average product is rising. B) Where total product is at a maximum, average product is also at a maximum. C) Where marginal product is zero, total product is at a maximum. D) Marginal product becomes negative before average product becomes negative. Answer: B

Use the following to answer questions 47-48:

Product

3

2 1 Variable input
Type: G Topic: 4 E: 397 MI: 153 47. In the above diagram curves 1, 2, and 3 represent the: A) average, marginal, and total product curves respectively. B) marginal, average, and total product curves respectively. C) total, average, and marginal product curves respectively. D) total, marginal, and average product curves respectively. Answer: B

McConnell/Brue: Economics, 16/e

Page 655

Chapter 22: The Costs of Production

Type: G Topic: 4 E: 397 MI: 153 48. The above diagram suggests that: A) when marginal product is zero, total product is at a minimum. B) when marginal product lies above average product, average product is rising. C) when marginal product lies below average product, average product is rising. D) when total product is at a maximum, so is marginal product and average product. Answer: B

Type: G Topic: 4 E: 397 MI: 153 49. The total output of a firm will be at a maximum where: A) MP is at a maximum. B) AP is at a minimum. C) MP is zero. Answer: C

D) AP is at a maximum.

Use the following to answer questions 50-51: Answer the next question(s) on the basis of the following information:

Number of workers 0 1 2 3 4 5 6

Total product 0 8 25 30

Marginal product -8 10

3 34

Type: T Topic: 4 E: 396 MI: 152 50. Refer to the above data. When two workers are employed: A) total product is 20. B) total product is 18. C) average product is 10. D) total product cannot be determined from the information given. Answer: B

Type: T Topic: 4 E: 396 MI: 152 51. Refer to the above data. The marginal product of the fourth worker: A) is 5. B) is 7. C) is 71/2. D) cannot be calculated from the information given. Answer: A

McConnell/Brue: Economics, 16/e

Page 656

Chapter 22: The Costs of Production

Type: G Topic: 4 E: 397 MI: 153

Marginal and average product

52.

Marginal product Average product

0

Q1

Q2 Inputs of labor

Q3

In the above diagram the range of diminishing marginal returns is: A) 0Q3. B) 0Q2. C) Q1Q2. D) Q1Q3. Answer: D

Use the following to answer questions 53-55: Use the following data to answer the next question(s):
Inputs of labor 0 1 2 3 4 5 6 7 Total product 0 8 18 25 30 33 34 32

Type: T Topic: 4 E: 397 MI: 153 53. Refer to the above data. When total product is increasing at an increasing rate, marginal product is: A) positive and increasing. B) positive and decreasing. C) constant. D) negative. Answer: A

Type: T Topic: 4 E: 397 MI: 153 54. Refer to the above data. When total product is increasing at a decreasing rate, marginal product is: A) positive and increasing. B) positive and decreasing. C) constant. D) negative. Answer: B

Type: T Topic: 4 E: 397 MI: 153 55. Refer to the above data. When total product is diminishing, marginal product is: A) positive and increasing. B) positive and decreasing. C) constant. D) negative. Answer: D

McConnell/Brue: Economics, 16/e

Page 657

Chapter 22: The Costs of Production

Short-run costs

Type: D Topic: 5 E: 398 MI: 154 56. Fixed cost is: A) the cost of producing one more unit of capital, say, machinery. B) any cost which does not change when the firm changes its output. C) average cost multiplied by the firm's output. D) usually zero in the short run Answer: B

Type: A Topic: 5 E: 398 MI: 154 57. Which of the following is most likely to be a fixed cost? A) shipping charges C) wages for unskilled labor B) property insurance premiums D) expenditures for raw materials Answer: B

Type: A Topic: 5 E: 398 MI: 154 58. If you owned a small farm, which of the following would be a fixed cost? A) harvest labor B) hail insurance C) fertilizer D) seed Answer: B

Type: A Topic: 5 E: 398 MI: 154 59. Which of the following is most likely to be a variable cost? A) fuel and power payments C) rental payments on IBM equipment B) interest on business loans. D) real estate taxes Answer: A

Type: A Topic: 5 E: 398 MI: 154 60. If you operated a small bakery, which of the following would be a variable cost in the short run? A) baking ovens C) annual lease payment for use of the building B) interest on business loans D) baking supplies (flour, salt, etc.) Answer: D

Type: D Topic: 5 E: 400 MI: 156 61. Marginal cost is the: A) rate of change in total fixed cost that results from producing one more unit of output. B) change in total cost that results from producing one more unit of output. C) change in average variable cost that results from producing one more unit of output. D) change in average total cost that results from producing one more unit of output. Answer: B

Type: A Topic: 5 E: 400 MI: 156 62. For most producing firms: A) marginal cost rises as output is carried to a certain level, and then begins to decline. B) total costs rise as output is carried to a certain level, and then begin to decline. C) average total costs decline as output is carried to a certain level, and then begin to rise. D) average total costs rise as output is carried to a certain level, and then begin to decline. Answer: C

McConnell/Brue: Economics, 16/e

Page 658

Chapter 22: The Costs of Production

Type: A Topic: 5 E: 399 MI: 155 63. Average fixed cost: A) equals marginal cost when average total cost is at its minimum. B) may be found for any output by adding average variable cost and average total cost. C) graphs as a U-shaped curve. D) declines continually as output increases. Answer: D

Type: A Topic: 5 E: 402 MI: 158 64. Which of the following is correct as it relates to cost curves? A) Average variable cost intersects marginal cost at the latter's minimum point. B) Marginal cost intersects average total cost at the latter's minimum point. C) Average fixed cost intersects marginal cost at the latter's minimum point. D) Marginal cost intersects average fixed cost at the latter's minimum point. Answer: B

Use the following to answer questions 65-70:

Type: G Topic: 5 E: 399-400 MI: 155-156 65. Refer to the above diagram. At output level Q total variable cost is: A) 0BEQ. B) BCDE. C) 0CDQ. D) 0AFQ. Answer: A

Type: G Topic: 5 E: 399-400 MI: 155-156 66. Refer to the above diagram. At output level Q total fixed cost is: A) 0BEQ. B) BCDE. C) 0BEQ-0AFQ. D) 0CDQ. Answer: B

Type: G Topic: 5 E: 399-400 MI: 155-156 67. Refer to the above diagram. At output level Q total cost is: A) 0BEQ. B) BCDE. C) 0BEQ plus BCDE. D) 0AFQ plus BCDE. Answer: C

McConnell/Brue: Economics, 16/e

Page 659

Chapter 22: The Costs of Production

Type: G Topic: 5 E: 399 MI: 155 68. Refer to the above diagram. At output level Q average fixed cost: A) is equal to EF. B) is equal to QE. C) is measured by both QF and ED. D) cannot be determined from the information given. Answer: C

Type: G Topic: 5 E: 401 MI: 157 69. Refer to the above diagram. At output level Q: A) marginal product is falling. B) marginal product is rising. C) marginal product is negative. D) one cannot determine whether marginal product is falling or rising. Answer: A

Type: G Topic: 5 E: 399-400 MI: 155-156 70. Refer to the above diagram. The vertical distance between ATC and AVC reflects: A) the law of diminishing returns. C) marginal cost at each level of output. B) the average fixed cost at each level of output. D) the presence of economies of scale. Answer: B

Type: A Topic: 5 E: 402 MI: 158 71. Marginal cost: A) equals both average variable cost and average total cost at their respective minimums. B) is the difference between total cost and total variable cost. C) rises for a time, but then begins to decline when diminishing returns set in. D) declines continuously as output increases. Answer: A

Type: A Topic: 5 E: 399-400 MI: 155-156 72. When average fixed costs are falling: A) average total cost must be falling. B) average variable cost may be either rising or falling. C) marginal cost must be falling. D) average variable costs must be rising. Answer: B

Type: A Topic: 5 E: 400 MI: 156 73. Which of the following statements is correct? A) Average total cost is the difference between average variable cost and average fixed cost. B) Marginal cost measures the cost per unit of output associated with any level of production. C) When marginal product rises, marginal cost must also rise. D) Marginal cost is the price or cost of an extra variable input (for example, an additional worker) divided by its marginal product. Answer: D

McConnell/Brue: Economics, 16/e

Page 660

Chapter 22: The Costs of Production

Type: A Topic: 5 E: 400 MI: 156 74. Assume that in the short run a firm is producing 100 units of output, has average total costs of $200, and average variable costs of $150. The firm's total fixed costs are: A) $5,000. B) $500. C) $.50. D) $50. Answer: A

Type: A Topic: 5 E: 400 MI: 156 75. If average total cost is declining, then: A) marginal cost must be greater than average total cost. B) the average fixed cost curve must lie above the average variable cost curve. C) marginal cost must be less than average total cost. D) total cost must also be declining. Answer: C

Type: A Topic: 5 E: 401 MI: 157 76. The relationship between the marginal cost and the average total cost schedule is such that: A) the behavior of one schedule does not affect the other. B) if ATC exceeds MC, MC must be rising. C) if MC is declining, ATC may be either declining or rising. D) if MC is declining, ATC must also be declining. Answer: D

Type: A Topic: 5 E: 402 MI: 158 77. Other things equal, if the prices of a firm's variable inputs were to fall: A) one could not predict how unit costs of production would be affected. B) marginal cost, average variable cost, and average fixed cost would all fall. C) marginal cost, average variable cost, and average total cost would all fall. D) average variable cost would fall, but marginal cost would be unchanged. Answer: C

Type: A Topic: 5 E: 398-400 MI: 154-156 78. Other things equal, if the fixed costs of a firm were to increase by $100,000 per year, which of the following would happen? A) Marginal costs and average variable costs would both rise. B) Average fixed costs and average variable costs would rise. C) Average fixed costs and average total costs would rise. D) Average fixed costs would rise, but marginal costs would fall. Answer: C

Type: A Topic: 5 E: 398 MI: 154 79. If a firm decides to produce no output in the short run, its costs will be: A) its marginal costs. B) its fixed plus its variable costs. C) its fixed costs. Answer: C

D) zero.

McConnell/Brue: Economics, 16/e

Page 661

Chapter 22: The Costs of Production

Use the following to answer questions 80-84: Answer the next question(s) on the basis of the following cost data:

Output 0 1 2 3 4 5 6

Total cost $24 33 41 48 54 61 69

Type: T Topic: 5 E: 398 MI: 154 80. Refer to the above data. The total variable cost of producing 5 units is: A) $61. B) $48. C) $37. D) $24. Answer: C

Type: T Topic: 5 E: 400 MI: 156 81. Refer to the above data. The average total cost of producing 3 units of output is: A) $14. B) $12. C) $13.50. D) $16. Answer: D

Type: T Topic: 5 E: 399 MI: 155 82. Refer to the above data. The average fixed cost of producing 3 units of output is: A) $8. B) $7.40. C) $5.50. D) $6. Answer: A

Type: T Topic: 5 E: 400 MI: 156 83. Refer to the above data. The marginal cost of producing the sixth unit of output is: A) $24. B) $12. C) $16. D) $8. Answer: D

Type: T Topic: 5 E: 400 MI: 156 84. Refer to the above data. The profit-maximizing output for this firm: A) is 3. B) is 4. C) is 5. D) cannot be determined from the information given. Answer: D

Type: A Topic: 5 E: 398 MI: 154 85. In comparing the changes in TVC and TC associated with an additional unit of output, we find that: A) no generalization about the changes in TC and TVC can be made. B) the changes in TC and TVC are equal. C) the change in TC is greater than the change in TVC. D) the change in TVC is greater than the change in TC. Answer: B

McConnell/Brue: Economics, 16/e

Page 662

Chapter 22: The Costs of Production

Use the following to answer questions 86-89: Answer the next question(s) on the basis of the following information: TFC = total fixed cost MC = marginal cost TVC = total variable cost

Q = quantity of output P = product price

Type: E Topic: 5 E: 399 MI: 155 86. Refer to the above information. Average fixed cost is: A) TVC - MC B) MC Q C) TFC Q D) TVC Q Answer: C

Type: E Topic: 5 E: 400 MI: 156 87. Refer to the above information. Average total cost is: A) TVC - MC B) TVC - TFC Q C) TVC Q D) TFC + TVC Q Answer: D

Type: E Topic: 5 E: 400 MI: 156 88. Refer to the above information. Marginal cost is: A) change in TVC Q B) change in TVC change in Q C) P•Q change in Q D) change in TFC change in Q Answer: D

Type: E Topic: 5 E: 398 MI: 154 89. Refer to the above information. Total cost is: A) the change in marginal cost. B) TVC - TFC C) TFC + TVC D) TFC + TVC Q Answer: C

McConnell/Brue: Economics, 16/e

Page 663

Chapter 22: The Costs of Production

Use the following to answer questions 90-92:
Marginal and average product

Marginal product
Average product

0

Q1

Q2 Inputs of labor

Q3

Type: G Topic: 5 E: 402 MI: 158 90. Refer to the above diagram, where variable inputs of labor are being added to a constant amount of property resources. The total output of this firm will cease to expand: A) if a labor force in excess of Q1 is employed. B) if a labor force in excess of Q2 is employed. C) if a labor force in excess of Q3 is employed. D) only if the marginal product curve becomes negative at all levels of output. Answer: C

Type: G Topic: 5 E: 402 MI: 158 91. Refer to the above diagram, where variable inputs of labor are being added to a constant amount of property resources. Marginal cost will be at a minimum for this firm when it is hiring: A) Q3 workers. B) Q2 workers. C) Q1 workers. D) more than Q3 workers. Answer: C

Type: G Topic: 5 E: 402 MI: 158 92. Refer to the above diagram, where variable inputs of labor are being added to a constant amount of property resources. Average variable cost will be at a minimum when the firm is hiring: A) Q3 workers. B) Q2 workers. C) Q1 workers. D) more than Q3 workers. Answer: B

McConnell/Brue: Economics, 16/e

Page 664

Chapter 22: The Costs of Production

Type: G Topic: 5 E: 402 MI: 158 93.

In the above figure, curves 1, 2, 3, and 4 represent the: A) ATC, MC, AFC, and AVC curves respectively. C) MC, ATC, AVC, and AFC curves respectively. B) AFC, MC, AVC, and ATC curves respectively. D) ATC, AVC, AFC, and MC curves respectively. Answer: C

Type: A Topic: 5 E: 403 MI: 159 94. If a technological advance reduces the amount of variable resources needed to produce any level of output, then the: A) AVC curve will shift upward. C) ATC curve will shift upward. B) MC curve will shift downward. D) AFC curve will shift downward. Answer: B

Type: A Topic: 5 E: 398-399 MI: 154-155 95. In the short run which of the following statements is correct? A) The marginal cost curve intersects the average variable and average fixed cost curves at their minimum points. B) Average variable cost declines continuously as total output is expanded. C) Total cost will exceed variable cost. D) If the inputs of all resources are increased by equal amounts, total output will expand by diminishing amounts. Answer: C

Type: A Topic: 5 E: 398 MI: 154 96. Total fixed cost (TFC): A) falls as the firm expands output from zero, but eventually rises. B) falls continuously as total output expands. C) varies directly with total output. D) does not change as total output increases or decreases. Answer: D

McConnell/Brue: Economics, 16/e

Page 665

Chapter 22: The Costs of Production

Type: A Topic: 5 E: 398 MI: 154 97. Fixed costs are associated with: A) highly adjustable inputs such as labor. B) both the short run and the long run. Answer: C

C) the short run only. D) the long run only.

Type: G Topic: 5 E: 399 MI: 155 98.

In the above diagram curves 1, 2, and 3 represent: A) average variable cost, marginal cost, and average fixed cost respectively. B) total variable cost, total fixed cost, and total cost respectively. C) total fixed cost, total variable cost, and total cost respectively. D) marginal product, average variable cost, and average total cost respectively. Answer: C

Type: A Topic: 5 E: 401-402 MI: 157-158 99. Which of the following is correct?. A) There is no relationship between MP and MC. B) When AP is rising MC is falling, and when AP is falling MC is rising. C) When MP is rising MC is rising, and when MP is falling MC is falling. D) When MP is rising MC is falling, and when MP is falling MC is rising. Answer: D

Type: A Topic: 5 E: 399 MI: 155 100. Which of the following curves is not U-shaped? A) MC B) AFC C) AVC D) ATC Answer: B

Type: A Topic: 5 E: 400 MI: 156 101. If a firm wanted to know how much it would save by producing one less unit of output, it would look to: A) MC. B) ATC. C) AVC. D) AFC. Answer: A

McConnell/Brue: Economics, 16/e

Page 666

Chapter 22: The Costs of Production

Type: A Topic: 5 E: 402 MI: 158 102. Which of the following holds true? A) There is no relationship between AP and AVC. B) When MP is rising AVC is falling, and when MP is falling AVC is rising. C) When AP is rising AVC is falling, and when AP is falling AVC is rising. D) When AP is rising AVC is rising, and when AP is falling AVC is falling. Answer: C

Type: A Topic: 5 E: 399 MI: 155 103. In the short run it is impossible for an expansion of output to increase: A) average total cost. B) average fixed cost. C) marginal cost. D) average variable cost. Answer: B

Type: A Topic: 5 E: 400 MI: 156 104. Average fixed costs can be determined graphically by: A) summing the marginal costs of any number of units of output and dividing the sum by that output. B) the vertical distance between TC and TVC. C) the vertical distance between AVC and MC. D) the vertical distance between ATC and AVC. Answer: D

Type: A Topic: 5 E: 398-399 MI: 154-155 105. The vertical distance between the total cost and the total variable cost curves differs by an amount which: A) initially increases, but then decreases, as output increases. B) is constant as output changes. C) decreases as output increases. D) increases as output increases. Answer: B

Type: A Topic: 5 E: 400 MI: 156 106. The vertical distance between a firm's ATC and AVC curves represents: A) AFC, which increases as output increases. B) AFC, which decreases as output increases. C) marginal costs, which decrease as output decreases. D) marginal costs, which increase as output increases. Answer: B

Type: A Topic: 5 E: 400 MI: 156 107. If a profitable firm's fixed costs somehow were zero: A) MC and ATC would be equal at all levels of output. B) AFC would become negative as output increases. C) AVC and ATC would coincide. D) ATC would be zero at all output levels. Answer: C

McConnell/Brue: Economics, 16/e

Page 667

Chapter 22: The Costs of Production

Use the following to answer questions 108-111:

Type: G Topic: 5 E: 402 MI: 158 108. Refer to the above short-run production and cost data. In Figure A curve (1) is: A) total product and curve (2) is average product. B) total product and curve (2) is marginal product. C) average product and curve (2) is marginal product. D) marginal product and curve (2) is average product. Answer: C

Type: G Topic: 5 E: 402 MI: 158 109. Refer to the above short-run production and cost data. In Figure B curve (3) is: A) AVC and curve (4) is MC. C) MC and curve (4) is AFC. B) MC and curve (4) is AVC. D) AFC and curve (4) is MC. Answer: B

Type: G Topic: 5 E: 402 MI: 158 110. Refer to the above short-run production and cost data. The curves of Figures A and B suggest that: A) marginal product and marginal cost reach their maximum points at the same output. B) marginal cost reaches a minimum where marginal product is at its maximum. C) marginal cost and marginal product reach their minimum points at the same output. D) AVC cuts MC at the latter's minimum point. Answer: B

McConnell/Brue: Economics, 16/e

Page 668

Chapter 22: The Costs of Production

Type: G Topic: 5 E: 402 MI: 158 111. Refer to the above short-run production and cost data. The curves of Figures A and B suggest that: A) average product and average variable cost reach their maximum points at the same output. B) AVC cuts MC at the latter's maximum point. C) AVC reaches a minimum where AP is at its maximum. D) AFC declines so long as output increases. Answer: C

Type: A Topic: 5 E: 398-399 MI: 154-155 112. In the short run: A) TVC will increase for a time at a diminishing rate, but then beyond some point will increase at an increasing rate. B) TVC will increase for a time at an increasing rate, but then beyond some point will increase at a diminishing rate. C) TVC will increase by the same absolute amount for each additional unit of output produced. D) one cannot generalize concerning the behavior of TVC as output increases. Answer: A

Type: A Topic: 5 E: 398-399 MI: 154-155 113. Total cost minus total variable cost equals: A) average fixed cost. B) total fixed cost. C) average variable cost. Answer: B

D) marginal cost.

Type: A Topic: 5 E: 398-399 MI: 154-155 114. As output increases, total variable cost: A) increases more rapidly than does total cost. B) increases continuously at a decreasing rate. C) increases at a decreasing rate and then at an increasing rate. D) increases at a constant rate. Answer: C

Type: A Topic: 5 E: 399 MI: 155 115. In the short run the Sure-Screen T-Shirt Company is producing 500 units of output. Its average variable costs are $2.00 and its average fixed costs are $.50. The firm's total costs: A) are $2.50. B) are $1250. C) are $750. D) are $1100. Answer: B

Type: A Topic: 5 E: 402 MI: 158 116. Because the marginal product of a variable resource at first increases and then decreases as the output of the firm is increased: A) total cost at first increases at a decreasing rate and then increases at an increasing rate. B) total variable cost at first increases at an increasing rate and then increases at a decreasing rate. C) average total cost at first increases and then diminishes. D) average fixed cost will rise beyond the point of diminishing returns. Answer: A

McConnell/Brue: Economics, 16/e

Page 669

Chapter 22: The Costs of Production

Type: A Topic: 5 E: 399 MI: 155 117. Suppose that, when producing 10 units of output, a firm's AVC is $22, its AFC is $5, and its MC is $30. This: A) firm's ATC is $35. C) firm's total cost is $270. B) firm's ATC is $57. D) firm's total cost is $30. Answer: C

Type: A Topic: 5 E: 399-400 MI: 155-156 118. The relationship between marginal cost and average fixed cost is such that: A) declines in MC cause AFC to decline as output increases. B) increases in MC cause AFC to increase as output increases. C) MC intersects AFC at that output where AFC is at a minimum. D) MC may either rise or fall as AFC declines. Answer: D

Type: A Topic: 5 E: 400 MI: 156 119. In comparing the changes in TC and TVC associated with an additional unit of output, we find that: A) the change in TVC is equal to MC, while the change in TC is equal to TFC. B) the change in TC exceeds the change in TVC. C) the change in TVC exceeds the change in TC. D) both are equal to MC. Answer: D

Type: A Topic: 5 E: 402 MI: 158 120. Which of the following is correct? A) When AP is rising, AVC is rising. B) When AP is rising, AVC is falling. Answer: B

C) When AP is rising, AP exceeds MP. D) There is no relationship between AP and AVC.

Use the following to answer questions 121-127: Answer the next question(s) on the basis of the following cost data:

Output 1 2 3 4 5 6 7 8

Average fixed cost $50.00 25.00 16.67 12.50 10.00 8.37 7.14 6.25

Average variable cost $100.00 80.00 66.67 65.00 68.00 73.33 80.00 87.50

Type: T Topic: 5 E: 399 MI: 155 121. Refer to the above data. Total fixed cost is: A) $6.25. B) $100.00. C) $150.00. D) $50.00. Answer: D

McConnell/Brue: Economics, 16/e

Page 670

Chapter 22: The Costs of Production

Type: T Topic: 5 E: 400 MI: 156 122. Refer to the above data. The average total cost of five units of output is: A) $69. B) $78. C) $3. D) $10. . Answer: B

Type: T Topic: 5 E: 399-400 MI: 155-156 123. Refer to the above data. The total cost of four units of output is: A) $260. B) $77.50. C) $310. D) $215. Answer: C

Type: T Topic: 5 E: 398-399 MI: 154-155 124. Refer to the above data. If the firm closed down and produced zero units of output, its total cost would be: A) zero. B) $50. C) $150. D) $100. Answer: B

Type: T Topic: 5 E: 399-400 MI: 155-156 125. Refer to the above data. The marginal cost of the fifth unit of output is: A) $3. B) $62. C) $80. D) $78. Answer: C

Type: T Topic: 5 E: 401-402 MI: 157-158 126. Refer to the above data. The marginal cost curve would intersect the average variable cost curve at about: A) 2 units of output. B) 4 units of output. C) 6 units of output. D) 7 units of output. Answer: B

Type: T Topic: 5 E: 398-399 MI: 154-155 127. Refer to the above data. If the firm decided to increase its output from 6 to 7 units, its total costs would rise by: A) $87.14. B) $80.00. C) $6.67. D) $120.00. Answer: D

Type: D Topic: 5 E: 398 MI: 154 128. A fixed cost is: A) associated with any productive resource whose price is fixed. B) any cost which increases proportionately with output. C) any cost which a firm would incur even if output was zero. D) associated with all inputs whose short-run supply is perfectly inelastic. Answer: C

Type: A Topic: 5 E: 402 MI: 158 129. Other things equal, if the wage rates paid to a firm's labor inputs were to rise, we would expect the: A) AFC, AVC, ATC, and MC curves all to rise. C) AFC and ATC curves to fall. B) AVC, ATC, and MC curves all to rise. D) MP curve to fall. Answer: B

McConnell/Brue: Economics, 16/e

Page 671

Chapter 22: The Costs of Production

Type: A Topic: 5 E: 403 MI: 159 130. If a technological advance increases a firm's labor productivity, we would expect its: A) average total cost curve to rise. C) total cost curve to rise. B) average total cost curve to fall. D) average total cost curve to be unaffected. Answer: B

Type: A Topic: 5 E: 398 MI: 154 131. As a firm produces successive units of output in the short run we would expect: A) TVC to increase initially by declining amounts, but eventually increase by increasing amounts. B) TVC to increase initially by increasing amounts, but eventually by decreasing amounts. C) TFC to increase by constant amounts. D) the sum of TVC and TFC to increase initially by increasing amounts, but eventually by decreasing amounts. Answer: A

Type: A Topic: 5 E: 398 MI: 154 132. Assume a firm closes down in the short run and produces no output. Under these conditions: A) TVC is positive, but TFC and TC are zero. C) TFC and TC are positive, but TVC is zero. B) TFC is positive, but TVC and TC are zero. D) TFC, TVC, and TC will all be positive. Answer: C

Type: A Topic: 5 E: 400-401 MI: 156-157 133. If marginal cost is: A) falling, then average total cost must also be falling. B) rising, then average total cost must also be rising. C) rising, then average total cost could be either falling or rising. D) falling, then average total cost could be either falling or rising. Answer: C

Type: A Topic: 5 E: 398-399 MI: 154-155 134. Which of the following is incorrect? A) Total fixed cost does not change with output in the short run. B) Fixed costs exist only in the short run. C) Total fixed cost must be added to total variable cost to determine total cost. D) Total fixed cost equals total variable cost in the long run. Answer: D

Type: A Topic: 5 E: 399-400 MI: 155-156 135. If the total variable cost of 9 units of output is $90 and the total variable cost of 10 units of output is $120, then: A) the average variable cost of 10 units is $10. B) the average variable cost of 9 units is $10. C) the marginal cost of the tenth unit is $90. D) the firm is operating in the range of increasing marginal returns. Answer: B

McConnell/Brue: Economics, 16/e

Page 672

Chapter 22: The Costs of Production

Type: A Topic: 5 E: 398 MI: 154 136. A firm's total variable cost will depend on: A) the prices of variable resources. B) the production techniques that are used. Answer: D

C) the level of output. D) all of the above.

Type: A Topic: 5 E: 400 MI: 156 137. The short-run average total cost curve is U-shaped because: A) average fixed costs decline continuously as output increases. B) of increasing and diminishing returns. C) of economies and diseconomies of scale. D) minimum efficient scale is encountered. Answer: B

Use the following to answer questions 138-142: The Sunshine Corporation finds that its costs are $40 when it produces no output. Its total variable costs (TVC) change with output as shown in the accompanying table. Use this information to answer the following question(s).
Output 1 2 3 4 5 TVC $ 30 50 65 85 110

Type: T Topic: 5 E: 398 MI: 154 138. Refer to the above information. The total cost of producing 3 units of output is: A) $65. B) $105. C) $145. D) $185. Answer: B

Type: T Topic: 5 E: 400 MI: 156 139. Refer to the above information. The average total cost of 3 units of output is: A) $65. B) $21.67. C) $40. D) $35. Answer: D

Type: T Topic: 5 E: 399 MI: 155 140. Refer to the above information. The average fixed cost of 3 units of output is: A) $13.33. B) $12.50. C) $40. D) $18.50. Answer: A

Type: T Topic: 5 E: 400 MI: 156 141. Refer to the above information. The marginal cost of the third unit of output is: A) $105. B) $25. C) $15. D) $20. Answer: C

McConnell/Brue: Economics, 16/e

Page 673

Chapter 22: The Costs of Production

Type: T Topic: 5 E: 393-394 MI: 149-150 142. Refer to the above information. This firm: A) is making an economic profit of $260. B) is realizing a loss of $125. Answer: C

C) may be either realizing a profit or a loss. D) is selling its output in a competitive market.

Use the following to answer questions 143-146:

Type: G Topic: 5 E: 399-400 MI: 155-156 143. Refer to the above diagram. This firm's average fixed costs are: A) not shown. C) the vertical distance between AVC and ATC. B) the vertical distance between AVC and MC. D) equal to the per unit change in MC. Answer: C

Type: G Topic: 5 E: 402 MI: 158 144. Refer to the above diagram. If labor is the only variable input, the marginal product of labor is at a: A) maximum at point a. B) minimum at point a. C) maximum at point b. D) maximum at point c. Answer: A

Type: G Topic: 5 E: 402 MI: 158 145. Refer to the above diagram. If labor is the only variable input, the average product of labor is at a: A) minimum at point b. B) maximum at point b. C) maximum at point a. D) maximum at point c. Answer: B

Type: G Topic: 5 E: 393-394 MI: 149-150 146. Refer to the above diagram. The profit-maximizing level of output for this firm: A) is at point a. B) is at point b. C) is at point c. D) cannot be determined from the information given. Answer: D

McConnell/Brue: Economics, 16/e

Page 674

Chapter 22: The Costs of Production

Use the following to answer questions 147-150: Answer the next question(s) on the basis of the accompanying table that shows average total costs (ATC) for a manufacturing firm whose total fixed costs are $10: Output 1 2 3 4 5 ATC $40 27 29 31 38

Type: T Topic: 5 E: 398-399 MI: 154-155 147. Refer to the above data. The total cost of producing 4 units of output is: A) $31. B) $87. C) $124. D) $108. Answer: C

Type: T Topic: 5 E: 399-400 MI: 155-156 148. Refer to the above data. The average variable cost of 4 units of output is: A) $33.50. B) $28.50. C) $19.00. D) $21.00. Answer: B

Type: T Topic: 5 E: 399-400 MI: 155-156 149. Refer to the above data. The marginal cost of the fourth unit of output is: A) $2. B) $12. C) $37. D) $16. Answer: C

Type: T Topic: 5 E: 393-394 MI: 149-150 150. Refer to the above data. The profit-maximizing level of output for this firm: A) is 3. B) is 4. C) is 5. D) cannot be determined from the information given. Answer: D

McConnell/Brue: Economics, 16/e

Page 675

Chapter 22: The Costs of Production

Use the following to answer questions 151-157:

Dollars

d e a b

f ATC1 c

0

Quantity

Type: G Topic: 5 E: 400 MI: 156 Status: New 151. Refer to the above graph. Which one of the following would cause a move from point b to point c along short-run average total cost curve ATC1? A) diminishing marginal returns C) a decrease in the wage rate B) an increase in the wage rate D) increasing marginal returns Answer: A

Type: G Topic: 5 E: 400 MI: 156 Status: New 152. Refer to the above graph. Which one of the following would cause a move from point d to point e along short-run average total cost curve ATC2? A) diminishing marginal returns C) a decrease in the wage rate B) an increase in the wage rate D) increasing marginal returns Answer: D

Type: G Topic: 5 E: 403 MI: 159 Status: New 153. Refer to the above graph. Which one of the following would cause a move from point b on short-run average total cost curve ATC1 to point e on short-run average cost curve ATC2? A) diminishing marginal returns C) a decrease in the wage rate B) an increase in the wage rate D) increasing marginal returns Answer: B

Type: G Topic: 5 E: 403 MI: 159 Status: New 154. Refer to the above graph. Which one of the following would cause a move from point e on short-run average total cost curve ATC2 to point b on short-run average cost curve ATC1? A) diminishing marginal returns C) a decrease in the wage rate B) an increase in the wage rate D) increasing marginal returns Answer: C

Type: G Topic: 5 E: 403 MI: 159 Status: New 155. Refer to the above graph. Which one of the following would cause a move from point e on short-run average total cost curve ATC2 to point b on short-run average cost curve ATC1? A) a decline in total fixed costs C) an increase in the wage rate B) diminishing marginal returns D) increasing marginal returns Answer: A

McConnell/Brue: Economics, 16/e

Page 676

Chapter 22: The Costs of Production

Type: G Topic: 5 E: 400 MI: 156 Status: New 156. Refer to the above graph. Diminishing marginal returns are reflected in: A) the shift of the of short-run average total cost curve from ATC2 to ATC1. B) a move along short-run average total cost curve ATC2 from point e to point f. C) a move along short-run average total cost curve ATC1 from point a to point b. D) the shift of the short-run average total cost curve from ATC1 to ATC2. Answer: B

Type: G Topic: 5 E: 403 MI: 159 Status: New 157. Refer to the above graph. A decrease in fixed costs is shown by: A) a move along short-run average total cost curve ATC2 from point e to point f. B) a move along short-run average total cost curve ATC1 from point a to point b. C) the shift of the short-run average total cost curve from ATC1 to ATC2. D) the shift of the of short-run average total cost curve from ATC2 to ATC1. Answer: D

Long-run costs

Type: A Topic: 6 E: 404 MI: 160 158. Economies and diseconomies of scale explain: A) the profit-maximizing level of production. B) why the firm's long-run average total cost curve is U-shaped. C) why the firm's short-run marginal cost curve cuts the short-run average variable cost curve at its minimum point. D) the distinction between fixed and variable costs. Answer: B

Type: D Topic: 6 E: 403 MI: 159 159. In the long run: A) all costs are variable costs. B) all costs are fixed costs. Answer: A

C) variable costs equal fixed costs. D) fixed costs are greater than variable costs.

McConnell/Brue: Economics, 16/e

Page 677

Chapter 22: The Costs of Production

Use the following to answer questions 160-164:

Type: G Topic: 6 E: 400 MI: 156 Status: New 160. The above diagram shows the short-run average total cost curves for five different plant sizes of a firm. The shape of each individual curve reflects: A) increasing returns, followed by diminishing returns. B) economies of scale, followed by diseconomies of scale. C) constant costs. D) increasing costs, followed by decreasing costs. Answer: A

Type: G Topic: 6 E: 404 MI: 160 Status: New 161. As the firm in the above diagram expands from plant size #1 to plant size #3, it experiences: A) diminishing returns. B) economies of scale. C) diseconomies of scale. D) constant costs. Answer: B

Type: G Topic: 6 E: 407 MI: 163 Status: New 162. As the firm in the above diagram expands from plant size #3 to plant size #5, it experiences: A) increasing returns. B) economies of scale. C) diseconomies of scale. D) constant costs. Answer: C

Type: G Topic: 6 E: 404 MI: 160 163. The above diagram shows the short-run average total cost curves for five different plant sizes of a firm. The position of these five curves in relation to one another reflects: A) economies and diseconomies of scale. B) the effect of fixed costs on ATC as output increases. C) the law of constant costs. D) the law of diminishing returns. Answer: A

Type: G Topic: 6 E: 403 MI: 159 164. The above diagram shows the short-run average total cost curves for five different plant sizes of a firm. In the long run the firm should produce output 0x with a plant of size: A) #4 B) #3. C) #2. D) #1. Answer: C

McConnell/Brue: Economics, 16/e

Page 678

Chapter 22: The Costs of Production

Type: A Topic: 6 E: 407 MI: 163 165. When diseconomies of scale occur: A) the long-run average total cost curve falls. B) marginal cost intersects average total cost. Answer: C

C) the long-run average total cost curve rises. D) average fixed costs will rise.

Type: A Topic: 6 E: 404-406 MI: 160-162 166. Which of the following is not a source of economies of scale? A) learning-by-doing. C) use of larger machines. B) labor specialization. D) inelastic resource supply curves. Answer: D

Type: A Topic: 6 E: 406 MI: 162 167. When a firm does more of something, it gets better at it. This learning-by-doing is: A) a source of diseconomies of scale. C) called the principle of natural progression. B) a source of economies of scale. D) called "spreading the overhead." Answer: B

Use the following to answer questions 168-171: Use the following data to answer the next question(s). The letters A, B, and C designate three successively larger plant sizes.

Output 10 20 30 40 50 60 70 80 90 100

ATC-A $ 6 5 4 5 7 10 14 19 25 32

ATC-B $13 9 6 4 3 4 5 7 10 16

ATC-C $44 35 27 20 14 11 8 6 5 7

Type: T Topic: 6 E: 403-404 MI: 159-160 168. Refer to the above data. In the long run the firm should use plant size "A" for: A) all possible levels of output. C) 30 to 60 units of output. B) 10 to 30 units of output. D) all outputs greater than 40. Answer: B

Type: T Topic: 6 E: 403-404 MI: 159-160 169. Refer to the above data. In the long run the firm should use plant size "C" for: A) all possible levels of output. C) 40 to 70 units of output. B) 10 to 30 units of output. D) all units of output greater than 80. Answer: D

McConnell/Brue: Economics, 16/e

Page 679

Chapter 22: The Costs of Production

Type: T Topic: 6 E: 404-405 MI: 160-161 170. Refer to the above data. Economies of scale are realized over the ___ to ___ levels of output; diseconomies of scale exist over the ___ to ___ levels of output. A) 10, 30; 40, 100 B) 10, 40; 80, 100 C) 10, 50; 60, 100 D) 10, 70; 80, 100 Answer: C

Type: T Topic: 6 E: 407 MI: 163 171. Refer to the above data. At what level of output is minimum efficient scale realized? A) 30 B) 40 C) 50 D) 60 Answer: C

Type: A Topic: 6 E: 404 MI: 160 172. Economies of scale are indicated by: A) the rising segment of the average variable cost curve. B) the declining segment of the long-run average total cost curve. C) the difference between total revenue and total cost. D) a rising marginal cost curve. Answer: B

Type: A Topic: 6 E: 407 MI: 163 173. Diseconomies of scale: A) pertain to the long run. B) pertain to the short run. Answer: A

C) are synonymous with diminishing returns. D) are synonymous with increasing returns.

Type: A Topic: 6 E: 403 MI: 159 174. The long-run average total cost curve: A) will rise if diminishing returns are encountered. B) will fall if diminishing returns are encountered. C) will rise if economies of scale are incurred. D) is based on the assumption that all resources are variable. Answer: D

Type: A Topic: 6 E: 404 MI: 160 175. If a firm doubles its output in the long run and its unit costs of production decline, we can conclude that: A) technological progress has occurred. C) the firm is encountering diminishing returns. B) economies of scale are being realized. D) diseconomies of scale are being encountered. Answer: B

McConnell/Brue: Economics, 16/e

Page 680

Chapter 22: The Costs of Production

Use the following to answer questions 176-178:

Type: G Topic: 6 E: 404 MI: 160 176. Refer to the above diagram. For output level Q, per unit costs of C are: A) unobtainable and imply the inefficient use of resources. B) unobtainable, given resource prices and the current state of technology. C) obtainable, but imply the inefficient use of resources. D) obtainable and imply that resources are being combined efficiently. Answer: C

Type: G Topic: 6 E: 404 MI: 160 177. Refer to the above diagram. For output level Q, per unit costs of B are: A) unobtainable and imply the inefficient use of resources. B) unobtainable, given resource prices and the current state of technology. C) obtainable, but imply the inefficient use of resources. D) obtainable and imply least-cost production of this output. Answer: D

Type: G Topic: 6 E: 404 MI: 160 178. Refer to the above diagram. For output level Q, per unit costs of A are: A) unobtainable and imply the inefficient use of resources. B) unobtainable, given resource prices and the current state of technology. C) obtainable, but imply the inefficient use of resources. D) obtainable and imply least-cost production of this output. Answer: B

Type: D Topic: 6 E: 407 MI: 163 179. The minimum efficient scale of a firm: A) is realized somewhere in the range of diseconomies of scale. B) occurs where marginal product becomes zero. C) is in the middle of the range of constant returns to scale. D) is the smallest level of output at which long-run average total cost is minimized. Answer: D

McConnell/Brue: Economics, 16/e

Page 681

Chapter 22: The Costs of Production

Type: A Topic: 6 E: 406-407 MI: 162-163 180. If an industry's long-run average total cost curve has an extended range of constant returns to scale, this implies that: A) technology precludes both economies and diseconomies of scale. B) the industry will be a natural monopoly. C) both relatively small and relatively large firms can be viable in the industry. D) the industry will be comprised of a very large number of small firms. Answer: C

Type: D Topic: 6 E: 407 MI: 163 181. A natural monopoly exists when: A) unit costs are minimized by having one firm produce an industry's entire output. B) several formerly competing producers merge to become the only firm in an industry. C) short-run average total cost curves are tangent to long-run average total cost curves. D) minimum efficient scale is attained at a small level of output. Answer: A

Type: A Topic: 6 E: 407 MI: 163 182. Diseconomies of scale arise primarily because: A) the short-run average total cost curve rises when marginal product is increasing. B) of the difficulties involved in managing and coordinating a large business enterprise. C) firms must be large both absolutely and relative to the market to employ the most efficient productive techniques available. D) beyond some point marginal product declines as additional units of a variable resource (labor) are added to a fixed resource (capital). Answer: B

Use the following to answer questions 183-186:

Type: G Topic: 6 E: 403 MI: 159 183. In the above diagram it is assumed that: A) some costs are fixed and other costs are variable. B) all costs are variable. C) the law of diminishing returns determines the shape of the cost curve. D) marginal product first falls, but ultimately rises as output is increased. Answer: B

McConnell/Brue: Economics, 16/e

Page 682

Chapter 22: The Costs of Production

Type: G Topic: 6 E: 407 MI: 163 184. Refer to the above diagram. Economies of scale: A) are evident over the entire range of output. B) occur over the 0Q1 range of output. Answer: B

C) begin at output Q3. D) occur only over the Q1Q3 range of output.

Type: G Topic: 6 E: 407 MI: 163 185. Refer to the above diagram. Diseconomies of scale: A) begin at output Q1. C) begin at output Q3. B) occur over the Q1Q3 range of output. D) are in evidence at all output levels. Answer: C

Type: G Topic: 6 E: 407 MI: 163 186. Refer to the above diagram. Minimum efficient scale: A) occurs at some output greater than Q3. C) is achieved at Q3. B) is achieved at Q1. D) cannot be identified in this diagram. Answer: B

Type: G Topic: 6 E: 407 MI: 163 187.

In the above long-run average total cost curve the: A) movement from A to B reflects diseconomies of scale. B) movement from B to C reflects diseconomies of scale. C) realization of economies of scale would shift the entire curve downward. D) movement from B to C reflects the law of diminishing returns. Answer: B

Type: D Topic: 6 E: 403-404 MI: 159-160 188. The long-run average total cost curve: A) displays declining unit costs so long as output is increasing. B) indicates the lowest unit costs achievable when a firm has had sufficient time to alter plant size. C) has a shape which is the inverse of the law of diminishing returns. D) can be derived by summing horizontally the average total cost curves of all firms in an industry. Answer: B

McConnell/Brue: Economics, 16/e

Page 683

Chapter 22: The Costs of Production

Type: A Topic: 6 E: 406 MI: 162 189. If a firm increases all of its inputs by 10 percent and its output increases by 15 percent, then: A) it is encountering diseconomies of scale. C) the law of diminishing returns is taking hold. B) it is encountering economies of scale. D) the firm's long-run ATC curve will be rising. Answer: B

Type: A Topic: 6 E: 407 MI: 163 190. If a firm increases all of its inputs by 10 percent and its output increases by 10 percent, then: A) it is encountering diseconomies of scale. C) it is encountering constant returns to scale. B) it is encountering economies of scale. D) the marginal products of all inputs are falling. Answer: C

Type: A Topic: 6 E: 407 MI: 163 191. The ABC Corporation decreases all of its inputs by 12 percent and finds that its output falls by only 8 percent. This means that initially it was producing: A) in the range of diseconomies of scale. C) where AP is less than MP. B) in the range of economies of scale. D) at the point of minimum efficient scale. Answer: A

Type: A Topic: 6 E: 407 MI: 163 192. Diseconomies of scale means that: A) a firm's long-run average total cost curve is declining. B) a firm's long-run average total cost curve is rising. C) the advantages of specialization are being more fully realized. D) a given increase in inputs results in a more-than-proportionate increase in output. Answer: B

Type: A Topic: 6 E: 406 MI: 162 193. Suppose a firm is in a range of production where it is experiencing economies of scale. Knowing this, we can predict that: A) the long-run average total cost curve is upsloping. B) a 10 percent increase in all inputs will increase output by less than 10 percent. C) a 10 percent increase in all inputs will increase output by more than 10 percent. D) the firm is encountering problems of managerial bureaucracy because of its size. Answer: C

Last Word Questions

Type: D E: 409 MI: 165 194. (Last Word) A cost that cannot be partly or fully recovered through any subsequent action is known as a: A) variable cost. B) fixed cost. C) marginal cost. D) sunk cost. Answer: D

McConnell/Brue: Economics, 16/e

Page 684

Chapter 22: The Costs of Production

Type: B E: 409 MI: 165 195. (Last Word) Which of the following is an example of a sunk cost, as it relates to a firm? A) an expenditure on raw materials used in the production process. B) an expenditure on a nonrefundable, nontransferable airline ticket. C) an expenditure to buy a delivery van. D) an expenditure for a new factory. Answer: B

Type: A E: 409 MI: 165 196. (Last Word) Which of the following sayings relates most closely to the idea of sunk costs: A) Don't cry over spilt milk. C) He who hesitates is lost. B) A bird in the hand is worth two in the bush. D) Show me the money. Answer: A

True/False Questions

Type: A E: 392 MI: 148 197. The real opportunity cost of producing product X is the amounts of products Y, Z, and so forth, that might have been produced if resources had not been used to produce X. Answer: True

Type: D E: 394 MI: 150 198. The short run is a period of time during which all costs are fixed costs. Answer: False

Type: D E: 398 MI: 154 199. Variable costs are costs that vary directly with output. Answer: True

Type: A E: 395 MI: 151 200. The law of diminishing returns explains why the long-run average total cost curve is U-shaped. Answer: False

Type: A E: 407 MI: 163 201. Diseconomies of scale stem primarily from the difficulties in managing and coordinating a large-scale business enterprise. Answer: True

Type: A E: 398 MI: 154 202. At zero units of output a firm's variable costs are zero. Answer: True

Type: A E: 399 MI: 155 203. Average fixed costs diminish continuously as output increases. Answer: True

McConnell/Brue: Economics, 16/e

Page 685

Chapter 22: The Costs of Production

Type: A E: 401 MI: 157 204. If the marginal-cost curve lies below the average-variable-cost curve, the average-variable-cost curve must be falling. Answer: True

Type: A E: 393 MI: 149 205. Economic profit is found by subtracting accounting costs from total revenue. Answer: False

Type: A E: 393 MI: 149 206. A firm's economic profit is usually higher than its accounting profit. Answer: False

Type: D E: 393 MI: 149 207. In economics, a firm earns a normal profit when its total revenue equals its total economic costs. Answer: True

Type: C E: 401 MI: 157 208. The law of diminishing returns explains why short-run marginal cost curves are upward sloping. Answer: True

Type: C E: 404 MI: 160 209. The law of diminishing returns explains diseconomies of scale. Answer: False

Type: C E: 407 MI: 163 210. Minimum efficient scale varies by industry. Answer: True

McConnell/Brue: Economics, 16/e

Page 686

CHAPTER 23

Pure Competition

Topic 1. 2. 3. 4. 5. Four market models Pure competition defined; demand curve Profit maximizing in short run Profit maximizing in long run Pure competition and efficiency Consider This Last Word True-False

Question numbers 1-10 11-34 35-147 148-187 188-205 206-207 208-210 211-233

____________________________________________________________

_______________________________________

____________________________________________________________

_______________________________________

Multiple Choice Questions Four market models

Type: A Topic: 1 E: 414 MI: 170 1. Economists would describe the U.S. automobile industry as: A) purely competitive. B) an oligopoly. C) monopolistically competitive. Answer: B

D) a pure monopoly.

Type: A Topic: 1 E: 414 MI: 170 2. In which of the following market structures is there clear-cut mutual interdependence with respect to priceoutput policies? A) pure monopoly B) oligopoly C) monopolistic competition D) pure competition Answer: B

Type: A Topic: 1 E: 414 MI: 170 3. Which of the following industries most closely approximates pure competition? A) agriculture B) farm implements C) clothing D) steel Answer: A

Type: D Topic: 1 E: 414 MI: 170 4. Economists use the term imperfect competition to describe: A) all industries which produce standardized products. B) any industry in which there is no nonprice competition. C) a pure monopoly only. D) those markets which are not purely competitive. Answer: D

Chapter 23: Pure Competition

Type: A Topic: 1 E: 414 MI: 170 5. In which of the following industry structures is the entry of new firms the most difficult? A) pure monopoly B) oligopoly C) monopolistic competition D) pure competition Answer: A

Type: A Topic: 1 E: 414 MI: 170 6. An industry comprised of 40 firms, none of which has more than 3 percent of the total market for a differentiated product is an example of: A) monopolistic competition B) oligopoly C) pure monopoly D) pure competition Answer: A

Type: A Topic: 1 E: 413-414 MI: 169-170 7. A one-firm industry is known as: A) monopolistic competition B) oligopoly C) pure monopoly D) pure competition Answer: C

Type: A Topic: 1 E: 414 MI: 170 8. An industry comprised of four firms, each with about 25 percent of the total market for a product is an example of: A) monopolistic competition B) oligopoly C) pure monopoly D) pure competition Answer: B

Type: A Topic: 1 E: 413-414 MI: 169-170 9. An industry comprised of a very large number of sellers producing a standardized product is known as: A) monopolistic competition B) oligopoly C) pure monopoly D) pure competition Answer: D

Type: A Topic: 1 E: 414 MI: 170 10. An industry comprised of a small number of firms, each of which considers the potential reactions of its rivals in making price-output decisions is called: A) monopolistic competition B) oligopoly C) pure monopoly D) pure competition Answer: B

Pure competition defined; demand curve

Type: A Topic: 2 E: 414 MI: 170 11. Which of the following statements applies to a purely competitive producer? A) It will not advertise its product. B) In long-run equilibrium it will earn an economic profit. C) Its product will have a brand name. D) Its product is slightly different from those of its competitors. Answer: A

McConnell/Brue: Economics, 16/e

Page 688

Chapter 23: Pure Competition

Type: A Topic: 2 E: 414 MI: 170 12. A purely competitive seller is: A) both a "price maker" and a "price taker." B) neither a "price maker" nor a "price taker." Answer: C

C) a "price taker." D) a "price maker."

Type: A Topic: 2 E: 414 MI: 170 13. Which of the following is not characteristic of pure competition? A) price strategies by firms C) no barriers to entry B) a standardized product D) a larger number of sellers Answer: A

Type: A Topic: 2 E: 414 MI: 170 14. Which of the following is not a basic characteristic of pure competition? A) considerable nonprice competition C) a standardized or homogeneous product B) no barriers to the entry or exodus of firms D) a large number of buyers and sellers Answer: A

Type: A Topic: 2 E: 415 MI: 171 15. The demand schedule or curve confronted by the individual purely competitive firm is: A) relatively elastic, that is, the elasticity coefficient is greater than unity. B) perfectly elastic. C) relatively inelastic, that is, the elasticity coefficient is less than unity. D) perfectly inelastic. Answer: B

Type: A Topic: 2 E: 415-416 MI: 171-172 16. Which of the following is characteristic of a purely competitive seller's demand curve? A) Price and marginal revenue are equal at all levels of output. B) Average revenue is less than price. C) Its elasticity coefficient is 1 at all levels of output. D) It is the same as the market demand curve. Answer: A

Use the following to answer questions 17-19: In answering the next question(s), assume a graph in which dollars are measured on the vertical axis and output on the horizontal axis.

Type: G Topic: 2 E: 415-416 MI: 171-172 17. Refer to the above information. For a purely competitive firm total revenue: A) graphs as a straight, upsloping line. C) is a straight line, parallel to the horizontal axis. B) is a straight line, parallel to the vertical axis. D) graphs as a straight, downsloping line. Answer: A

McConnell/Brue: Economics, 16/e

Page 689

Chapter 23: Pure Competition

Type: G Topic: 2 E: 416 MI: 172 18. Refer to the above information. For a purely competitive firm marginal revenue: A) graphs as a straight, upsloping line. C) is a straight line, parallel to the horizontal axis. B) is a straight line, parallel to the vertical axis. D) graphs as a straight, downsloping line. Answer: C

Type: G Topic: 2 E: 416 MI: 172 19. Refer to the above information. For a purely competitive firm: A) marginal revenue will graph as an upsloping line. B) the demand curve will lie above the marginal revenue curve. C) the marginal revenue curve will lie above the demand curve. D) the demand and marginal revenue curves will coincide. Answer: D

Type: A Topic: 2 E: 415-416 MI: 171-172 20. If a firm in a purely competitive industry is confronted with an equilibrium price of $5, its marginal revenue: A) may be either greater or less than $5. C) will be less than $5. B) will also be $5. D) will be greater than $5. Answer: B

Type: A Topic: 2 E: 414 MI: 170 21. Price is constant or given to the individual firm selling in a purely competitive market because: A) the firm's demand curve is downsloping. B) of product differentiation reinforced by extensive advertising. C) each seller supplies a negligible fraction of total supply. D) there are no good substitutes for its product. Answer: C

Type: A Topic: 2 E: 415-416 MI: 171-172 22. For a purely competitive seller, price equals: A) average revenue. B) marginal revenue. C) total revenue divided by output. Answer: D

D) all of the above.

Type: A Topic: 2 E: 415 MI: 171 23. For a purely competitive firm total revenue: A) is price times quantity sold. B) increases by a constant absolute amount as output expands. C) graphs as a straight upsloping line from the origin. D) has all of the above characteristics. Answer: D

Type: A Topic: 2 E: 416 MI: 172 24. The marginal revenue curve of a purely competitive firm: A) lies below the firm's demand curve. B) increases at an increasing rate as output expands. C) is horizontal at the market price. D) is downsloping because price must be reduced to sell more output. Answer: C

McConnell/Brue: Economics, 16/e

Page 690

Chapter 23: Pure Competition

Type: A Topic: 2 E: 415 MI: 171 25. The demand curve in a purely competitive industry is ______, while the demand curve to a single firm in that industry is ______. A) perfectly inelastic, perfectly elastic C) downsloping, perfectly inelastic B) downsloping, perfectly elastic D) perfectly elastic, downsloping Answer: B

Type: A Topic: 2 E: 415 MI: 171 26. A perfectly elastic demand curve implies that the firm: A) must lower price to sell more output. B) can sell as much output as it chooses at the existing price. C) realizes an increase in total revenue which is less than product price when it sells an extra unit. D) is selling a differentiated (heterogeneous) product. Answer: B

Type: A Topic: 2 E: 415-416 MI: 171-172 27. The vertical distance between the horizontal axis and any point on a pure competitor's demand curve measures: A) total revenue. B) total cost. C) product price, marginal revenue, and average revenue. D) the quantity demanded. Answer: C

Type: A Topic: 2 E: 416 MI: 172 28. The fact that a purely competitive firm's total revenue curve is linear and upsloping to the right implies that: A) product price increases as output increases. B) product price decreases as output increases. C) product price is constant at all levels of output. D) marginal revenue declines as more output is produced. Answer: C

Type: A Topic: 2 E: 415 MI: 171 29. Which of the following statements is correct? A) The demand curve for a purely competitive firm is perfectly elastic, but the demand curve for a purely competitive industry is downsloping. B) The demand curve for a purely competitive firm is downsloping, but the demand curve for a purely competitive industry is perfectly elastic. C) The demand curves are downsloping for both a purely competitive firm and a purely competitive industry. D) The demand curves are perfectly elastic for both a purely competitive firm and a purely competitive industry. Answer: A

McConnell/Brue: Economics, 16/e

Page 691

Chapter 23: Pure Competition

Use the following to answer questions 30-31:

Type: G Topic: 2 E: 416 MI: 172 30. Refer to the above diagram, which pertains to a purely competitive firm. Curve A represents: A) total revenue and marginal revenue. C) total revenue and average revenue. B) marginal revenue only. D) total revenue only. Answer: D

Type: G Topic: 2 E: 415-416 MI: 171-172 31. Refer to the above diagram, which pertains to a purely competitive firm. Curve C represents: A) total revenue and marginal revenue. C) total revenue and average revenue. B) marginal revenue only. D) average revenue and marginal revenue. Answer: D

Type: A Topic: 2 E: 416 MI: 172 32. A purely competitive seller's average revenue curve coincides with: A) its marginal revenue curve only. B) its demand curve only. C) both its demand and marginal revenue curves. D) neither its demand nor its marginal revenue curve. Answer: C

Type: D Topic: 2 E: 416 MI: 172 33. Marginal revenue is the: A) change in product price associated with the sale of one more unit of output. B) change in average revenue associated with the sale of one more unit of output. C) difference between product price and average total cost. D) change in total revenue associated with the sale of one more unit of output. Answer: D

Type: A Topic: 2 E: 416 MI: 172 34. Marginal revenue for a purely competitive firm: A) is greater than price. B) is less than price. Answer: C

C) is equal to price. D) may be either greater or less than price.

McConnell/Brue: Economics, 16/e

Page 692

Chapter 23: Pure Competition

Profit maximizing in short run

Type: A Topic: 3 E: 416 MI: 172 35. Firms seek to maximize: A) per unit profit. B) total revenue. Answer: C

C) total profit. D) market share.

Type: A Topic: 3 E: 418 MI: 174 36. A competitive firm in the short run can determine the profit-maximizing (or loss-minimizing) output by equating: A) price and average total cost. C) marginal revenue and marginal cost. B) price and average fixed cost. D) price and marginal revenue. Answer: C

Type: A Topic: 3 E: 417 MI: 173 37. In the short run a purely competitive firm that seeks to maximize profit will produce: A) where the demand and the ATC curves intersect. B) where total revenue exceeds total cost by the maximum amount. C) that output where economic profits are zero. D) at any point where the total revenue and total cost curves intersect. Answer: B

Use the following to answer questions 38-41:

Type: G Topic: 3 E: 417-418 MI: 173-174 38. Refer to the above short-run data. Total fixed cost for this firm is: A) about $67. B) $300. C) $200. D) $100. Answer: C

McConnell/Brue: Economics, 16/e

Page 693

Chapter 23: Pure Competition

Type: G Topic: 3 E: 417 MI: 173 39. Refer to the above short-run data. The shape of the total cost curve reflects: A) diminishing opportunity costs. C) increasing and diminishing returns. B) the law of rising fixed costs. D) economies and diseconomies of scale. Answer: C

Type: G Topic: 3 E: 418 MI: 174 40. Refer to the above short-run data. The profit-maximizing output for this firm is: A) above 440 units. B) 440 units. C) 320 units. D) 100 units. Answer: C

Type: G Topic: 3 E: 418 MI: 174 41. Refer to the above short-run data. Which of the following is correct? A) This firm will maximize its profit at 440 untis of output. B) Any level of output between 100 and 440 units will yield an economic profit. C) This firm's marginal revenue rises with output. D) Any level of output less than 100 units or greater than 440 units is profitable. Answer: B

Type: A Topic: 3 E: 417 MI: 173 42. A competitive firm will maximize profits at that output at which: A) total revenue exceeds total cost by the greatest amount. B) total revenue and total cost are equal. C) price exceeds average total cost by the largest amount. D) the difference between marginal revenue and price is at a maximum. Answer: A

Use the following to answer questions 43-48:

Type: G Topic: 3 E: 418 MI: 174 43. Curve (1) in the above diagram is a purely competitive firm's: A) total cost curve. C) marginal revenue curve B) total revenue curve. D) total economic profit curve. Answer: D

McConnell/Brue: Economics, 16/e

Page 694

Chapter 23: Pure Competition

Type: G Topic: 3 E: 416 MI: 172 44. Curve (2) in the above diagram is a purely competitive firm's A) total cost curve. C) marginal revenue curve B) total revenue curve. D) total economic profit curve. Answer: C

Type: G Topic: 3 E: 418 MI: 174 45. Curve (3) in the above diagram is a purely competitive firm's A) total cost curve. C) marginal revenue curve. B) total revenue curve. D) total economic profit curve. Answer: B

Type: G Topic: 3 E: 418 MI: 174 46. Curve (4) in the above diagram is a purely competitive firm's: A) total cost curve. B) total revenue curve. C) marginal revenue curve. D) total profit curve. Answer: A

Type: G Topic: 3 E: 418 MI: 174 47. Refer to the above diagram. Other things equal, an increase of product price would be shown as: A) an increase in the steepness of curve (3), an upward shift in curve (2), and upward shift in curve (1). B) a decrease in the steepness of curve (3), a downward shift in curve (2), and an upward shift in curve (1). C) an downward shift in curve (4) and an upward shift in curve (1), with no changes in lines (2) and (3). D) an upward shift in line (2) only. Answer: A

Type: G Topic: 3 E: 418 MI: 174 48. The firm represented by the above diagram would maximize its profit where: A) curves (2) and (1) intersect. B) curve (1) touches the horizontal axis for the second time. C) the vertical distance between curves (3) and (4) is the greatest. D) curves (3) and (4) intersect. Answer: C

Type: D Topic: 3 E: 417-418 MI: 173-174 49. A firm reaches a break-even point (normal profit position) where: A) marginal revenue cuts the horizontal axis. B) marginal cost intersects the average variable cost curve. C) total revenue equals total variable cost. D) total revenue and total cost are equal. Answer: D

Type: A Topic: 3 E: 419 MI: 175 50. The MR = MC rule applies: A) to firms in all types of industries. B) only when the firm is a "price taker." Answer: A

C) only to monopolies. D) only to purely competitive firms.

McConnell/Brue: Economics, 16/e

Page 695

Chapter 23: Pure Competition

Type: A Topic: 3 E: 418 MI: 174 51. When a firm is maximizing profit it will necessarily be: A) maximizing profit per unit of output. B) maximizing the difference between total revenue and total cost. C) minimizing total cost. D) maximizing total revenue. Answer: B

Type: A Topic: 3 E: 419 MI: 175 52. The MR = MC rule can be restated for a purely competitive seller as P = MC because: A) each additional unit of output adds exactly its price to total revenue. B) the firm's average revenue curve is downsloping. C) the market demand curve is downsloping. D) the firm's marginal revenue and total revenue curves will coincide. Answer: A

Type: A Topic: 3 E: 424 MI: 180 53. In the short run the individual competitive firm's supply curve is that segment of the: A) average variable cost curve lying below the marginal cost curve. B) marginal cost curve lying above the average variable cost curve. C) marginal revenue curve lying below the demand curve. D) marginal cost curve lying between the average total cost and average variable cost curves. Answer: B

Type: A Topic: 3 E: 424 MI: 180 54. Which of the following is not a valid generalization concerning the relationship between price and costs for a purely competitive seller in the short run? A) Price must be at least equal to average total cost. B) Price times quantity produced must be equal to or greater than total variable cost for some level of output or the firm will close down in the short run. C) Price may be equal to, greater than, or less than average total cost. D) Price must be equal to or greater than minimum average variable cost for the firm to continue producing. Answer: A

Type: A Topic: 3 E: 418 MI: 174 55. Assume the XYZ Corporation is producing 20 units of output. It is selling this output in a purely competitive market at $10 per unit. Its total fixed costs are $100 and its average variable cost is $3 at 20 units of output. This corporation: A) should close down in the short run. C) is realizing a loss of $60. B) is maximizing its profits. D) is realizing an economic profit of $40. Answer: D

McConnell/Brue: Economics, 16/e

Page 696

Chapter 23: Pure Competition

Type: D Topic: 3 E: 424 MI: 180 56. A purely competitive firm's short-run supply curve is: A) perfectly elastic at the minimum average total cost. B) upsloping and equal to the portion of the marginal cost curve that lies above the average variable cost curve. C) upsloping and equal to the portion of the marginal cost curve that lies above the average total cost curve. D) upsloping only when the industry has constant costs. Answer: B

Type: A Topic: 3 E: 424 MI: 180 57. Suppose you find that the price of your product is less than minimum AVC. You should: A) minimize your losses by producing where P = MC. B) maximize your profits by producing where P = MC. C) close down because, by producing, your losses will exceed your total fixed costs. D) close down because total revenue exceeds total variable cost. Answer: C

Type: A Topic: 3 E: 422 MI: 178 58. If a purely competitive firm shuts down in the short run: A) its loss will be zero. B) it will realize a loss equal to its total variable costs. C) it will realize a loss equal to its total fixed costs. D) it will realize a loss equal to its total costs. Answer: C

Type: A Topic: 3 E: 422 MI: 178 59. A purely competitive firm should produce in the short run if its total revenue is sufficient to cover its: A) total variable costs. B) total costs. C) total fixed costs. D) marginal costs. Answer: A

Use the following to answer questions 60-64: Answer the next question(s) on the basis of the following data confronting a firm:
Output 0 1 2 3 4 5 Marginal revenue -$16 16 16 16 16 Marginal cost -$10 9 13 17 21

Type: T Topic: 3 E: 416 MI: 172 60. Refer to the above data. This firm is selling its output in a(n): A) imperfectly competitive market. C) purely competitive market. B) monopolistic market. D) oligopolistic market. Answer: C

McConnell/Brue: Economics, 16/e

Page 697

Chapter 23: Pure Competition

Type: T Topic: 3 E: 424 MI: 180 61. Refer to the above data. If the firm's minimum average variable cost is $10, the firm's profit-maximizing level of output would be: A) 2. B) 3. C) 4. D) 5. Answer: B

Type: T Topic: 3 E: 417 MI: 173 62. Refer to the above data. At the profit-maximizing output the firm's total revenue is: A) $48. B) $32. C) $80. D) $64. Answer: A

Type: T Topic: 3 E: 417 MI: 173 63. Refer to the above data. At the profit-maximizing output the firm's total cost is: A) $48. B) $32. C) $80. D) $64. Answer: B

Type: T Topic: 3 E: 417 MI: 173 64. Refer to the above data. The firm's: A) economic profit is $12. B) economic profit is $16. Answer: B

C) loss is $14.

D) economic profit is $3.

Type: A Topic: 3 E: 424 MI: 180 65. In the short run a purely competitive firm will always make an economic profit if: A) P = ATC. B) P > AVC. C) P = MC. D) P > ATC. Answer: D

Type: A Topic: 3 E: 424 MI: 180 66. Suppose that at 500 units of output marginal revenue is equal to marginal cost. The firm is selling its output at $5 per unit and average total cost at 500 units of output is $6. On the basis of this information we: A) can say that the firm should close down in the short run. B) can say that the firm can produce and realize an economic profit in the short run. C) cannot determine whether the firm should produce or shut down in the short run. D) can assume the firm is not using the most efficient technology. Answer: C

Type: A Topic: 3 E: 424 MI: 180 67. If a firm is confronted with economic losses in the short run, it will decide whether or not to produce by comparing: A) marginal revenue and marginal cost. C) total revenue and total cost. B) price and minimum average variable cost. D) total revenue and total fixed cost. Answer: B

McConnell/Brue: Economics, 16/e

Page 698

Chapter 23: Pure Competition

Type: A Topic: 3 E: 424 MI: 180 68. A firm finds that at its MR = MC output, its TC = $1000, TVC = $800, TFC = $200, and total revenue is $900. This firm should: A) shut down in the short run. B) produce because the resulting loss is less than its TFC. C) produce because it will realize an economic profit. D) liquidate its assets and go out of business. Answer: B

Type: A Topic: 3 E: 424 MI: 180 69. The lowest point on a purely competitive firm's short-run supply curve corresponds to: A) the minimum point on its ATC curve. C) the minimum point on its AFC curve. B) the minimum point on its AVC curve. D) the minimum point on its MC curve. Answer: B

Use the following to answer questions 70-73:

Type: G Topic: 3 E: 423-424 MI: 179-180 70. Refer to the above diagram for a purely competitive producer. The lowest price at which the firm should produce (as opposed to shutting down) is: A) P1. B) P2. C) P3. D) P4. Answer: B

Type: G Topic: 3 E: 423-424 MI: 179-180 71. Refer to the above diagram for a purely competitive producer. The firm will produce at a loss at all prices: A) above P1. B) above P3. C) above P4. D) between P2 and P 3. Answer: D

Type: G Topic: 3 E: 424 MI: 180 72. Refer to the above diagram for a purely competitive producer. If product price is P3: A) the firm will maximize profit at point d. C) economic profits will be zero. B) the firm will earn an economic profit. D) new firms will enter this industry. Answer: C

McConnell/Brue: Economics, 16/e

Page 699

Chapter 23: Pure Competition

Type: G Topic: 3 E: 424 MI: 180 73. Refer to the above diagram for a purely competitive producer. The firm's short-run supply curve is: A) the abcd segment of the MC curve. C) the cd segment of the MC curve. B) the bcd segment of the MC curve. D) not shown. Answer: B

Type: A Topic: 3 E: 424 MI: 180 74. The short-run supply curve of a purely competitive producer is based on its: A) AVC curve. B) ATC curve. C) AFC curve. D) MC curve. Answer: D

Type: A Topic: 3 E: 424 MI: 180 75. On a per unit basis economic profit can be determined as the difference between: A) marginal revenue and product price. C) marginal revenue and marginal cost. B) product price and average total cost. D) average fixed cost and product price. Answer: B

Type: A Topic: 3 E: 423-424 MI: 179-180 76. In the short run a purely competitive seller will shut down if: A) it cannot produce at an economic profit. B) price is less than average variable cost at all outputs. C) price is less than average fixed cost at all outputs. D) there is no point at which marginal revenue and marginal cost are equal. Answer: B

Use the following to answer questions 77-81:

Type: G Topic: 3 E: 424 MI: 180 77. Refer to the above diagram. To maximize profit or minimize losses this firm will produce: A) K units at price C. B) D units at price J. C) E units at price A. D) E units at price B. Answer: C

McConnell/Brue: Economics, 16/e

Page 700

Chapter 23: Pure Competition

Type: G Topic: 3 E: 420 MI: 176 78. Refer to the above diagram. At the profit-maximizing output, total revenue will be: A) 0AHE. B) 0BGE. C) 0CFE. D) ABGE. Answer: A

Type: G Topic: 3 E: 420 MI: 176 79. Refer to the above diagram. At the profit-maximizing output, total fixed cost is equal to: A) 0AHE. B) 0BGE. C) 0CFE. D) BCFG. Answer: D

Type: G Topic: 3 E: 420 MI: 176 80. Refer to the above diagram. At the profit-maximizing output, total variable cost is equal to: A) 0AHE. B) 0CFE. C) 0BGE. D) ABGH. Answer: B

Type: G Topic: 3 E: 420 MI: 176 81. Refer to the above diagram. At the profit-maximizing output, the firm will realize: A) a loss equal to BCFG. C) an economic profit of ACFH. B) a loss equal to ACFH. D) an economic profit of ABGH. Answer: D

Type: A Topic: 3 E: 420 MI: 176 82. If a purely competitive firm is producing at some level less than the profit-maximizing output, then: A) price is necessarily greater than average total cost. B) fixed costs are large relative to variable costs. C) price exceeds marginal revenue. D) marginal revenue exceeds marginal cost. Answer: D

McConnell/Brue: Economics, 16/e

Page 701

Chapter 23: Pure Competition

Use the following to answer questions 83-87: Answer the next question(s) on the basis of the following cost data for a firm that is selling in a purely competitive market:

Total product 1 2 3 4 5 6 7 8 9 10 11 12

Average fixed cost $100.00 50.00 33.33 25.00 20.00 16.67 14.29 12.50 11.11 10.00 9.09 8.33

Average variable cost $17.00 16.00 15.00 14.25 14.00 14.00 15.71 17.50 19.44 21.60 24.00 26.67

Average total cost $117.00 66.00 48.33 39.25 34.00 30.67 30.00 30.00 30.55 31.60 33.09 35.00

Marginal cost $17 15 13 12 13 14 26 30 35 41 48 56

Type: T Topic: 3 E: 421 MI: 177 83. Refer to the above data. If the market price for the firm's product is $12, the competitive firm will produce: A) 4 units at a loss of $109. C) 8 units at a loss of $48.80. B) 4 units at an economic profit of $31.75. D) zero units at a loss of $100. Answer: D

Type: T Topic: 3 E: 420 MI: 176 84. Refer to the above data. If the market price for the firm's product is $32, the competitive firm will produce: A) 8 units at an economic profit of $16. B) 5 units at a loss of $10. C) 8 units at a loss equal to the firm's total fixed cost. D) 7 units at an economic profit of $41.50. Answer: A

Type: T Topic: 3 E: 421 MI: 177 85. Refer to the above data. If the market price for the firm's product is $28, the competitive firm will: A) produce 4 units at a loss of $17.40. C) close down in the short run. B) produce 7 units at a loss of $14.00. D) produce 6 units at a loss of $23.80. Answer: B

McConnell/Brue: Economics, 16/e

Page 702

Chapter 23: Pure Competition

Type: T Topic: 3 E: 423 MI: 179 86. Refer to the above data. Which of the following is the firm's short-run supply schedule?
(a) Price Qs $50 12 42 10 36 8 32 8 20 6 13 0 (b) Price Qs $50 12 42 11 36 9 32 8 20 6 13 5 (c) Price Qs $50 11 42 10 36 9 32 8 20 6 13 0 (d) Price Qs $50 11 42 10 36 9 32 8 20 6 13 5

Answer: C

Type: T Topic: 3 E: 425 MI: 181 87. Refer to the above data. If there were 1,000 identical firms in this industry and total or market demand is as shown below, equilibrium price will be:
Price $50 42 36 32 20 13 Quantity demanded 3,000 6,000 9,000 11,000 14,000 19,500

A) $32. B) $42. Answer: C

C) $36.

D) $13.

Type: A Topic: 3 E: 423-424 MI: 179-180 88. If at the MC = MR output, AVC exceeds price: A) new firms will enter this industry. B) the firm should produce the MC = MR output and realize an economic profit. C) the firm should shut down in the short run. D) the firm should expand its plant. Answer: C

McConnell/Brue: Economics, 16/e

Page 703

Chapter 23: Pure Competition

Use the following to answer questions 89-94:

Type: G Topic: 3 E: 424 MI: 180 89. Refer to the above diagram. The profit-maximizing output: A) is n. B) is k. C) is h. D) cannot be determined from the information given. Answer: A

Type: G Topic: 3 E: 424 MI: 180 90. Refer to the above diagram. At the profit-maximizing output, average variable cost is: A) ef. B) fg. C) na. D) ac. Answer: C

Type: G Topic: 3 E: 424 MI: 180 91. Refer to the above diagram. At the profit-maximizing output, total profit is: A) efbc. B) fgab. C) egac. D) 0fbn. Answer: A

Type: G Topic: 3 E: 424 MI: 180 92. Refer to the above diagram. For any level of output, total fixed cost: A) is fgab. B) is 0gan. C) is ba. D) is efbc. Answer: A

Type: G Topic: 3 E: 424 MI: 180 93. Refer to the above diagram. The short-run supply curve for this firm is the: A) entire MC curve. B) segment of the AVC curve lying to the right of the MC curve. C) segment of the MC curve lying above the ATC curve. D) segment of the MC curve lying above the AVC curve. Answer: D

McConnell/Brue: Economics, 16/e

Page 704

Chapter 23: Pure Competition

Type: G Topic: 3 E: 416 MI: 172 94. Refer to the above diagram. This firm is selling its product in a(n): A) purely competitive market. C) monopsonistic market. B) imperfectly competitive market. D) monopolistic market. Answer: A

Type: A Topic: 3 E: 423-424 MI: 179-180 95. In the short run a purely competitive seller will shut down if product price: A) equals average revenue. B) is greater than MC. C) is less than AVC. Answer: C

D) is less than ATC.

Type: A Topic: 3 E: 423-424 MI: 179-180 96. The short-run shut-down point for a purely competitive firm occurs: A) at any point where price is less than the minimum AVC. B) between the two break-even points. C) at any point where total revenue is less than total cost. D) at any point where the firm is not making an economic profit. Answer: A

Type: A Topic: 3 E: 427 MI: 183 97. In a purely competitive industry: A) there will be no economic profits in either the short run or the long run. B) economic profits may persist in the long run if consumer demand is strong and stable. C) there may be economic profits in the short run, but not in the long run. D) there may be economic profits in the long run, but not in the short run. Answer: C

Type: A Topic: 3 E: 424 MI: 180 98. The short-run supply curve for a purely competitive industry can be found by: A) multiplying the AVC curve of the representative firm by the number of firms in the industry. B) adding horizontally the AVC curves of all firms. C) summing horizontally the segments of the MC curves lying above the AVC curve for all firms. D) adding horizontally the immediate market period supply curves of each firm. Answer: C

Type: A Topic: 3 E: 419 MI: 175 99. DASH Airlines is considering the addition of a flight from Red Cloud to David City. The total cost of the flight would be $1100 of which fixed costs are $800. Expected revenues from the flight are $600. DASH should: A) not add this flight because only flights which cover their full costs are profitable. B) not add this flight because it is not profitable at the margin. C) add this flight because marginal revenue exceeds marginal costs. D) not add this flight because total costs exceed total revenue. Answer: C

McConnell/Brue: Economics, 16/e

Page 705

Chapter 23: Pure Competition

Type: A Topic: 3 E: 421 MI: 177 100. In contrast to American firms, Japanese firms frequently make lifetime employment commitments to their workers and agree not to lay them off when product demand is weak. Other things being equal, we would expect Japanese firms to: A) face more elastic product demand curves than American firms. B) have relatively greater variable costs than American firms. C) discontinue production at higher product prices than would American firms. D) continue to produce in the short run at lower prices than would American firms. Answer: D

Type: A Topic: 3 E: 421 MI: 177 101. Assume for a competitive firm that MC = AVC at $12, MC = ATC at $20, and MC = MR at $16. This firm will: A) realize a profit of $4 per unit of output. B) maximize its profit by producing in the short run. C) minimize its losses by producing in the short run. D) shut down in the short run. Answer: C

Type: A Topic: 3 E: 418 MI: 174 102. The principle that a firm should produce up to the point where the marginal revenue from the sale of an extra unit of output is equal to the marginal cost of producing it is known as the: A) output-maximizing rule. B) profit-maximizing rule. C) shut-down rule. D) break-even rule. Answer: B

Type: A Topic: 3 E: 419 MI: 175 103. If a purely competitive firm is producing at the P = MC output and realizing an economic profit, at that output: A) marginal revenue is less than price. C) ATC is being minimized. B) marginal revenue exceeds ATC. D) total revenue equals total cost. Answer: B

Type: A Topic: 3 E: 418 MI: 174 104. If a profit-seeking competitive firm is producing its profit-maximizing output and its total fixed costs fall by 25 percent, the firm should: A) use more labor and less capital to produce a larger output. B) not change its output. C) reduce its output. D) increase its output. Answer: B

McConnell/Brue: Economics, 16/e

Page 706

Chapter 23: Pure Competition

Use the following to answer questions 105-108:

Type: G Topic: 3 E: 424 MI: 180 105. Refer to the above diagram. At P2, this firm will: A) produce 44 units and realize an economic profit. B) produce 44 units and earn only a normal profit. C) produce 66 units and earn only a normal profit. D) shut down in the short run. Answer: B

Type: G Topic: 3 E: 424 MI: 180 106. Refer to the above diagram. At P1, this firm will produce: A) 47 units and break even. C) 66 units and earn only a normal profit. B) 47 units and realize an economic profit. D) 24 units and earn only a normal profit. Answer: B

Type: G Topic: 3 E: 423-424 MI: 179-180 107. Refer to the above diagram. At P4, this firm will: A) shut down in the short run. B) produce 30 units and incur a loss. Answer: A

C) produce 30 units and earn only a normal profit. D) produce 10 units and earn only a normal profit.

Type: G Topic: 3 E: 423-424 MI: 179-180 108. Refer to the above diagram. At P3, this firm will: A) produce 14 units and realize an economic profit. B) produce 62 units and earn only a normal profit. C) produce 40 units and incur a loss. D) shut down in the short run. Answer: C

Type: A Topic: 3 E: 423-424 MI: 179-180 109. The loss of a purely competitive firm which shuts down in the short run: A) is equal to its total variable costs. C) is equal to its total fixed costs. B) is zero. D) cannot be determined. Answer: C

McConnell/Brue: Economics, 16/e

Page 707

Chapter 23: Pure Competition

Type: A Topic: 3 E: 423-424 MI: 179-180 110. The Ajax Manufacturing Company is selling in a purely competitive market. Its output is 100 units which sell at $4 each. At this level of output total cost is $600, total fixed cost is $100, and marginal cost is $4. The firm should: A) reduce output to about 80 units. C) continue to produce 100 units. B) expand its production. D) produce zero units of output. Answer: D

Type: A Topic: 3 E: 419 MI: 175 111. The MR = MC rule can be restated for a purely competitive seller as P = MC because: A) each additional unit of output adds exactly its constant price to total revenue. B) the firm's average revenue curve is downsloping. C) the market demand curve is downsloping. D) the firm's marginal revenue and total revenue curves will coincide. Answer: A

Type: A Topic: 3 E: 421 MI: 177 112. If a purely competitive firm is maximizing economic profit: A) it is necessarily maximizing per-unit profit. B) it may or may not be maximizing per unit profit. C) then per-unit profit will be minimized. D) it is necessarily overallocating resources to its product. Answer: B

Use the following to answer questions 113-115: Answer the next question(s) on the basis of the following cost data for a purely competitive seller:
Output 0 1 2 3 4 5 6 7 Total cost $ 50 90 120 140 170 210 260 330

Type: T Topic: 3 E: 417 MI: 173 113. Refer to the above data. If product price is $60, the firm will: A) shut down. B) produce 4 units and realize a $120 economic profit. C) produce 6 units and realize a $100 economic profit. D) produce 3 units and incur a $40 loss. Answer: C

McConnell/Brue: Economics, 16/e

Page 708

Chapter 23: Pure Competition

Type: T Topic: 3 E: 417 MI: 173 114. Refer to the above data. If product price is $45, the firm will: A) shut down. B) produce 4 units and realize a $120 economic profit. C) produce 5 units and realize a $15 economic profit. D) produce 6 units and realize a $100 economic profit. Answer: C

Type: T Topic: 3 E: 422 MI: 178 115. Refer to the above data. If product price is $25, the firm will: A) shut down and incur a $90 loss. B) shut down and incur a $50 loss. C) produce 3 units and incur a $65 loss. D) produce 4 units and realize a $10 economic profit. Answer: B

Type: A Topic: 3 E: 422 MI: 178 116. If total revenue is less than total variable costs at the MR = MC output, a purely competitive firm should: A) shut down. C) produce and may or may not realize a profit. B) produce, but will necessarily realize a loss. D) increase its output. Answer: A

Type: A Topic: 3 E: 417 MI: 173 117. Assume a purely competitive firm is selling 200 units of output at $3 each. At this output its total fixed cost is $100 and its total variable cost is $350. This firm: A) is maximizing its profit. B) is making a profit, but not necessarily the maximum profit. C) is incurring losses. D) should shut down in the short run. Answer: B

McConnell/Brue: Economics, 16/e

Page 709

Chapter 23: Pure Competition

Use the following to answer questions 118-122:

Type: G Topic: 3 E: 424 MI: 180 118. Refer to the above diagram. This firm will earn only a normal profit if product price is: A) P1. B) P2. C) P3. D) P4. Answer: C

Type: G Topic: 3 E: 424 MI: 180 119. Refer to the above diagram. The firm will realize an economic profit if price is: A) P1. B) P2. C) P3. D) P4. Answer: D

Type: G Topic: 3 E: 423-424 MI: 179-180 120. Refer to the above diagram. The firm will produce at a loss if price is: A) P1. B) P2. C) P3. D) P4. Answer: B

Type: G Topic: 3 E: 423-424 MI: 179-180 121. Refer to the above diagram. The firm will shut down at any price less than: A) P1. B) P2. C) P3. D) P4. Answer: A

Type: G Topic: 3 E: 424 MI: 180 Status: New 122. Refer to the above diagram. The firm's supply curve is the segment of the: A) MC curve above its intersection with the AVC curve. B) MC curve above its intersection with the ATC curve. C) AVC curve above its intersection with the MC curve. D) ATC curve above its intersection with the MC curve. Answer: A

McConnell/Brue: Economics, 16/e

Page 710

Chapter 23: Pure Competition

Use the following to answer questions 123-132: Answer the next question(s) on the basis of the following cost data for a firm that is selling in a purely competitive market.

Total output 1 2 3 4 5 6 7 8 9 10

Average fixed cost $150.00 75.00 50.00 37.50 30.00 25.00 21.43 18.75 16.67 15.00

Average variable cost $25.00 23.00 20.00 21.00 23.00 25.00 28.00 33.00 39.00 48.00

Average total cost $175.00 98.00 70.00 58.50 53.00 50.00 49.43 51.76 55.67 63.00

Marginal cost $ 25.00 21.00 14.00 24.00 31.00 35.00 46.01 68.07 86.95 128.97

Type: G Topic: 3 E: 402 MI: 158 Status: New 123. Refer to the above data. The marginal cost column reflects: A) the law of diminishing returns. C) diseconomies of scale. B) the law of diminishing marginal utility. D) economies of scale. Answer: A

Type: G Topic: 3 E: 398-399 MI: 154-155 Status: New 124. Refer to the above data. At 6 units of output, total fixed cost is ____ and total cost is ____: A) $25; $50. B) $50; $300. C) $100, $200. D) $150; $300. Answer: D

Type: G Topic: 3 E: 398-399 MI: 154-155 Status: New 125. Refer to the above data. At 3 units of output, total variable cost is ____ and total cost is ____: A) $20; $70. B) $60; $210. C) $20, $210. D) $60; $350. Answer: B

Type: G Topic: 3 E: 398-399 MI: 154-155 Status: New 126. Refer to the above data. We can infer that, at zero output, this firm's total fixed, total variable, and total costs are: A) zero, zero, and zero, respectively. C) $150, $25, and $175, respectively. B) zero, $25, and $175, respectively. D) $150, zero, and $150, respectively. Answer: D

Type: G Topic: 3 E: 419-420 MI: 175-176 Status: New 127. Refer to the above data. If the market price for this firm's product is $86.95, it will produce: A) 9 units at an economic profit of zero. C) 9 units at an economic profit of $281.52. B) 6 units at a loss of $90. D) 8 units at an economic profit of $130.48. Answer: C

McConnell/Brue: Economics, 16/e

Page 711

Chapter 23: Pure Competition

Type: G Topic: 3 E: 419-420 MI: 175-176 Status: New 128. Refer to the above data. If the market price for this firm's product is $68.07, it will produce: A) 8 units at an economic profit of zero. C) 9 units at an economic profit of $281.52. B) 6 units at a loss of $90. D) 8 units at an economic profit of $130.48. Answer: D

Type: G Topic: 3 E: 421-422 MI: 177-178 Status: New 129. Refer to the above data. If the market price for this firm's product is $35, it will produce: A) 6 units at a loss of $150. C) 9 units at an economic profit of $281.52. B) 6 units at a loss of $90. D) 8 units at an economic profit of $130.48. Answer: B

Type: G Topic: 3 E: 421-422 MI: 177-178 Status: New 130. Refer to the above data. If the market price for this firm's product is $24, it will produce: A) 4 units at a loss of $150. C) 3 units at an economic profit of zero. B) 6 units at a loss of $90. D) 4 units at a loss of $138. Answer: D

Type: G Topic: 3 E: 422-423 MI: 178-179 Status: New 131. Refer to the above data. If the market price for this firm's product is $14, it will produce: A) 0 units at a loss of $150. C) 3 units at an economic profit of zero. B) 3 units at a loss of $168. D) 4 units at a loss of $138. Answer: A

Type: G Topic: 3 E: 424 MI: 180 Status: New 132. Refer to the above data. The firm's supply schedule is reflected in the: A) marginal cost data for the fourth through tenth units of output. B) marginal cost data for the first through tenth units of output. C) average total cost data for the seventh through tenth units of output. D) average variable cost data for fifth through tenth units of output. Answer: A

Type: A Topic: 3 E: 417 MI: 173 133. A purely competitive seller should produce (rather than shut down) in the short run: A) only if total revenue exceeds total cost. B) only if total cost exceeds total revenue. C) if total revenue exceeds total cost or if total cost exceeds total revenue by some amount less than total fixed cost. D) if total cost exceeds total revenue by some amount greater than total fixed cost. Answer: C

Type: A Topic: 3 E: 417-418 MI: 173-174 134. In the short run a purely competitive firm will maximize profit by producing that output at which: A) total revenue exceeds total cost by a maximum amount. B) total revenue exceeds total cost by a minimum amount. C) total revenue and total cost are equal. D) total fixed cost equals total variable cost. Answer: A

McConnell/Brue: Economics, 16/e

Page 712

Chapter 23: Pure Competition

Use the following to answer questions 135-139: Answer the next question(s) on the basis of the following cost data for a purely competitive seller:
Total product 0 1 2 3 4 5 6 Total fixed cost $50 50 50 50 50 50 50 Total variable cost $ 0 70 120 150 220 300 390 Total cost $ 50 120 170 200 270 350 440

Type: T Topic: 3 E: 417 MI: 173 135. The above data are for: A) the long run. B) the short run. Answer: B

C) both the short run and the long run. D) the intermediate market period only.

Type: T Topic: 3 E: 419-420 MI: 175-176 136. Refer to the above data. At 5 units of output average fixed cost, average variable cost, and average total cost are: A) $10, $60, and $70 respectively. C) $10, $70, and $80 respectively. B) $50, $40, and $90 respectively. D) $5, $25, and $30 respectively. Answer: A

Type: T Topic: 3 E: 419 MI: 175 137. Refer to the above data. The marginal cost of the fifth unit of output is: A) $80. B) $90. C) $50. D) $20. Answer: A

Type: T Topic: 3 E: 419-420 MI: 175-176 138. Refer to the above data. If product price is $75, the firm will produce: A) 3 units of output. B) 4 units of output. C) 5 units of output. D) 6 units of output. Answer: B

Type: T Topic: 3 E: 419-420 MI: 175-176 139. Refer to the above data. Given the $75 product price, at its optimal output the firm will: A) realize a $25 economic profit. C) incur a $25 loss. B) realize a $30 economic profit. D) realize a $30 loss. Answer: B

McConnell/Brue: Economics, 16/e

Page 713

Chapter 23: Pure Competition

Type: A Topic: 3 E: 424 MI: 180 140. A purely competitive firm's short-run supply curve is: A) the upward sloping portion of its marginal cost curve. B) the upward sloping portion of its average variable cost curve. C) its marginal cost curve above average variable cost. D) its average total cost curve. Answer: C

Type: A Topic: 3 E: 424 MI: 180 141. In the short run, a purely competitive firm will earn a normal profit when: A) P = AVC. B) P > MC. C) that firm's MR = market equilibrium price. D) P = ATC. Answer: D

Use the following to answer questions 142-147: The following table applies to a purely competitive industry composed of 100 identical firms.

Quantity Demanded 400,000 500,000 600,000 700,000 800,000

Price $5 4 3 2 1

Quantity Supplied 800,000 700,000 600,000 500,000 400,000

Type: T Topic: 3 E: 425 MI: 181 142. Refer to the above table. The equilibrium price in this purely competitive market is: A) $5. B) $4. C) $3. D) $2. Answer: C

Type: T Topic: 3 E: 425 MI: 181 143. Refer to the above table. At the equilibrium price, each of the 100 firms in this industry will produce: A) 600,000 units of output. C) 6,000 units of output B) 60,000 units of output. D) 600 units of output. Answer: C

Type: T Topic: 3 E: 425-426 MI: 181-182 144. Refer to the above table. For each of the 100 firms in this industry, marginal revenue and total revenue will be: A) $4 and $400, respectively. C) $4 and $20,000, respectively. B) $3 and $30,000, respectively. D) $3 and $18,000, respectively. Answer: D

Type: T Topic: 3 E: 425-426 MI: 181-182 145. Refer to the above table. If each of the 100 firms in the industry is maximizing its profit, each must have a marginal cost of: A) $5. B) $4. C) $3. D) $2. Answer: C

McConnell/Brue: Economics, 16/e

Page 714

Chapter 23: Pure Competition

Type: T Topic: 3 E: 425-426 MI: 181-182 146. Refer to the above table. If each of the 100 firms in the industry is maximizing its profit and earning only a normal profit, each must have a total cost of: A) $18,000. B) $20,000. C) $22,000. D) $24,000. Answer: A

Type: T Topic: 3 E: 425-426 MI: 181-182 147. Refer to the above table. If each of the 100 firms in the industry is maximizing its profit and earning only a normal profit, each must have an average total cost of: A) $2. B) $3. C) $4. D) $5. Answer: B

Profit maximizing in long run

Type: A Topic: 4 E: 427 MI: 183 148. Suppose a firm in a purely competitive market discovers that the price of its product is above its minimum AVC point but everywhere below ATC. Given this, the firm: A) minimizes losses by producing at the minimum point of its AVC curve. B) maximizes profits by producing where MR = ATC. C) should close down immediately. D) should continue producing in the short run, but leave the industry in the long run. Answer: D

Type: A Topic: 4 E: 427 MI: 183 149. Which of the following is true concerning purely competitive industries? A) There will be economic losses in the long run because of cut-throat competition. B) Economic profits will persist in the long run if consumer demand is strong and stable. C) In the short run, firms may incur economic losses or earn economic profits, but in the long run they earn normal profits. D) There are economic profits in the long run, but not in the short run. Answer: C

Type: A Topic: 4 E: 428 MI: 184 150. If a purely competitive firm is producing at the MR = MC output level and earning an economic profit, then: A) the selling price for this firm is above the market equilibrium price. B) new firms will enter this market. C) some existing firms in this market will leave. D) there must be price fixing by the industry's firms. Answer: B

Type: A Topic: 4 E: 427 MI: 183 151. Long-run competitive equilibrium: A) is realized only in constant-cost industries. B) will never change once it is realized. Answer: D

C) is not economically efficient. D) results in zero economic profits.

McConnell/Brue: Economics, 16/e

Page 715

Chapter 23: Pure Competition

Type: A Topic: 4 E: 428 MI: 184 152. We would expect an industry to expand if firms in that industry are: A) earning normal profits. C) incurring economic losses. B) earning economic profits. D) earning accounting profits. Answer: B

Type: A Topic: 4 E: 422-423 MI: 178-179 153. Which of the following statements is correct? A) Economic profits induce firms to enter an industry; losses encourage firms to leave. B) Economic profits induce firms to leave an industry; profits encourage firms to leave. C) Economic profits and losses have no significant impact on the growth or decline of an industry. D) Normal profits will cause an industry to expand. Answer: A

Type: A Topic: 4 E: 430 MI: 186 154. Suppose a purely competitive increasing-cost industry is in long-run equilibrium. Now assume that a decrease in consumer demand occurs. After all resulting adjustments have been completed, the new equilibrium price: A) and industry output will be less than the initial price and output. B) will be greater than the initial price, but the new industry output will be less than the original output. C) will be less than the initial price, but the new industry output will be greater than the original output. D) and industry output will be greater than the initial price and output. Answer: A

Type: A Topic: 4 E: 430 MI: 186 155. Which of the following statements is correct? A) The long-run supply curve for a purely competitive increasing-cost industry will be upsloping. B) The long-run supply curve for a purely competitive increasing-cost industry will be perfectly elastic. C) The long-run supply curve for a purely competitive industry will be less elastic than the industry's short-run supply curve. D) The long-run supply curve for a purely competitive decreasing-cost industry will be upsloping. Answer: A

Type: A Topic: 4 E: 429 MI: 185 156. A constant-cost industry is one in which: A) a higher price per unit will not result in an increased output. B) if 100 units can be produced for $100, then 150 can be produced for $150, 200 for $200, and so forth. C) the demand curve and therefore the unit price and quantity sold seldom change. D) the total cost of producing 200 or 300 units is no greater than the cost of producing 100 units. Answer: B

Type: A Topic: 4 E: 432 MI: 188 157. Which of the following will not hold true for a competitive firm in long-run equilibrium? A) P equals AFC B) P equals minimum ATC C) MC equals minimum ATC D) P equals MC Answer: A

McConnell/Brue: Economics, 16/e

Page 716

Chapter 23: Pure Competition

Type: A Topic: 4 E: 430 MI: 186 158. Assume a purely competitive increasing-cost industry is initially in long-run equilibrium and that an increase in consumer demand occurs. After all economic adjustments have been completed product price will be: A) lower, but total output will be larger than originally. B) higher and total output will be larger than originally. C) lower and total output will be smaller than originally. D) higher, but total output will be smaller than originally. Answer: B

Type: A Topic: 4 E: 430 MI: 186 159. Assume a purely competitive, increasing-cost industry is in long-run equilibrium. If a decline in demand occurs, firms will: A) leave the industry, price will decrease, and quantity produced will increase. B) enter the industry and price and quantity will both increase. C) leave the industry and price and output will both increase. D) leave the industry and price and output will both decline. Answer: D

Type: A Topic: 4 E: 427 MI: 183 160. When a purely competitive firm is in long-run equilibrium: A) marginal revenue exceeds marginal cost. B) price equals marginal cost. C) total revenue exceeds total cost. D) minimum average total cost is less than the product price. Answer: B

Type: A Topic: 4 E: 427 MI: 183 161. A purely competitive firm: A) must earn a normal profit in the short run. B) cannot earn economic profit in the long run. C) may realize either economic profit or losses in the long run. D) cannot earn economic profit in the short run. Answer: B

Type: A Topic: 4 E: 429 MI: 185 162. A constant-cost industry is one in which: A) resource prices fall as output is increased. B) resource prices rise as output is increased. C) resource prices remain unchanged as output is increased. D) small and large levels of output entail the same total costs. Answer: C

Type: A Topic: 4 E: 430 MI: 186 163. An increasing-cost industry is associated with: A) a perfectly elastic long-run supply curve. B) an upsloping long-run supply curve. Answer: B

C) a perfectly inelastic long-run supply curve. D) an upsloping long-run demand curve.

McConnell/Brue: Economics, 16/e

Page 717

Chapter 23: Pure Competition

Use the following to answer questions 164-166:

Type: G Topic: 4 E: 426 MI: 182 164. Refer to the above diagrams, which pertain to a purely competitive firm producing output q and the industry in which it operates. Which of the following is correct? A) The diagrams portray neither long-run nor short-run equilibrium. B) The diagrams portray both long-run and short-run equilibrium. C) The diagrams portray short-run equilibrium, but not long-run equilibrium. D) The diagrams portray long-run equilibrium, but not short-run equilibrium. Answer: C

Type: G Topic: 4 E: 427-428 MI: 183-184 165. Refer to the above diagrams, which pertain to a purely competitive firm producing output q and the industry in which it operates. In the long run we should expect: A) firms to enter the industry, market supply to rise, and product price to fall. B) firms to leave the industry, market supply to rise, and product price to fall. C) firms to leave the industry, market supply to fall, and product price to rise. D) no change in the number of firms in this industry. Answer: C

Type: G Topic: 4 E: 432 MI: 188 166. Refer to the above diagrams, which pertain to a purely competitive firm producing output q and the industry in which it operates. The predicted long-run adjustments in this industry might be offset by: A) a decline in product demand. B) an increase in resource prices. C) a technological improvement in production methods. D) entry of new firms into the industry. Answer: C

Type: A Topic: 4 E: 427 MI: 183 167. Assume a purely competitive firm is maximizing profit at some output at which long-run average total cost is at a minimum. Then: A) the firm is earning an economic profit. B) there is no tendency for the firm's industry to expand or contract. C) allocative but not productive efficiency is being achieved. D) other firms will enter this industry. Answer: B

McConnell/Brue: Economics, 16/e

Page 718

Chapter 23: Pure Competition

Type: A Topic: 4 E: 430 MI: 186 168. An increasing-cost industry is the result of: A) higher resource prices which occur as the industry expands. B) a change in the industry's minimum efficient scale. C) X-inefficiency. D) the law of diminishing returns. Answer: A

Type: A Topic: 4 E: 427 MI: 183 169. A purely competitive firm is precluded from making economic profit in the long run because: A) it is a "price taker." C) of unimpeded entry to the industry. B) its demand curve is perfectly elastic. D) it produces a differentiated product. Answer: C

Type: A Topic: 4 E: 428-429 MI: 184-185 170. If a purely competitive constant-cost industry is realizing economic profits, we can expect industry supply to: A) increase, output to increase, price to decrease, and profits to decrease. B) increase, output to increase, price to increase, and profits to decrease. C) decrease, output to decrease, price to increase, and profits to increase. D) increase, output to decrease, price to decrease, and profits to decrease. Answer: A

Type: A Topic: 4 E: 427 MI: 183 171. Assume that a decline in consumer demand occurs in a purely competitive industry which is initially in long-run equilibrium. We can: A) predict that the new price will be greater than the original price. B) predict that the new price will be less than the original price. C) predict that the new price will be the same as the original price. D) not compare the original and the new price without knowing about cost conditions in the industry. Answer: D

Type: D Topic: 4 E: 430-431 MI: 186-187 172. A decreasing-cost industry is one in which: A) contraction of the industry will decrease unit costs. B) input prices fall or technology improves as the industry expands. C) the long-run supply curve is perfectly elastic. D) the long-run supply curve is upsloping. Answer: B

Type: A Topic: 4 E: 430-431 MI: 186-187 173. When compact disc (CD) players first came on the market, they sold for over $1,000. Now they cost only $100. These facts imply that: A) the CD industry was once competitive, but is now monopolistic. B) fewer firms produce CD players than was the case five or ten years ago. C) the demand curve for CD players has shifted leftward. D) the CD player industry is a decreasing-cost industry. Answer: D

McConnell/Brue: Economics, 16/e

Page 719

Chapter 23: Pure Competition

Type: A Topic: 4 E: 431 MI: 187 174. Suppose that an industry's long-run supply curve is downsloping. This suggests that: A) it is an increasing-cost industry. B) relevant inputs have become more expensive as the industry has expanded. C) technology has become less efficient as a result of the industry's expansion. D) it is a decreasing-cost industry. Answer: D

Type: A Topic: 4 E: 430-431 MI: 186-187 175. Suppose an increase in product demand occurs in a decreasing-cost industry. As a result: A) the new long-run equilibrium price will be lower than the original long-run equilibrium price. B) equilibrium quantity will decline. C) firms will eventually leave the industry. D) the new long-run equilibrium price will be higher than the original price. Answer: A

Type: C Topic: 4 E: 429-430 MI: 185-186 176. Purely competitive industry X has constant costs and its product is an inferior good. The industry is currently in long-run equilibrium. The economy now goes into a recession and average incomes decline. The result will be: A) an increase in output and in the price of the product. B) an increase in output, but not in the price, of the product. C) a decrease in the output, but not in the price, of the product. D) a decrease in output and in the price of the product. Answer: B

Type: A Topic: 4 E: 430 MI: 186 177. Suppose losses cause industry X to contract and, as a result, the prices of relevant inputs decline. Industry X is: A) a constant-cost industry. C) an increasing-cost industry. B) a decreasing-cost industry. D) encountering X-inefficiency. Answer: C

McConnell/Brue: Economics, 16/e

Page 720

Chapter 23: Pure Competition

Use the following to answer questions 178-183:

Type: G Topic: 4 E: 420 MI: 176 178. Refer to the above diagram showing the average total cost curve for a purely competitive firm. Suppose this firm is maximizing its total profit and the market price is $15. The firm's per unit profit is: A) $5. B) $200. C) a positive amount less than $5. D) a positive amount more than $200. Answer: C

Type: G Topic: 4 E: 420 MI: 176 179. Refer to the above diagram showing the average total cost curve for a purely competitive firm. Suppose that total variable cost is $300 at 40 units of output. At that level of output, average fixed cost: A) is $2.50. B) is $4. C) is $100. D) cannot be determined from the information provided. Answer: A

Type: G Topic: 4 E: 420 MI: 176 180. Refer to the above diagram showing the average total cost curve for a purely competitive firm. Suppose that average variable cost is $8 at 40 units of output. At that level of output, total fixed cost: A) is $2. B) is $40. C) is $80. D) cannot be determined from the information provided. Answer: C

Type: G Topic: 4 E: 415-416, 427 MI: 171-172, 183 181. Refer to the above diagram showing the average total cost curve for a purely competitive firm. At the longrun equilibrium level of output, this firm's total revenue: A) is $10. B) is $40. C) is $400. D) cannot be determined from the information provided. Answer: C

Type: G Topic: 4 E: 420, 427 MI: 176, 183 182. Refer to the above diagram showing the average total cost curve for a purely competitive firm. At the longrun equilibrium level of output, this firm's total cost: A) is $10. B) is $40. C) is $400. D) cannot be determined from the information provided. Answer: C

Type: G Topic: 4 E: 427 MI: 183 183. Refer to the above diagram showing the average total cost curve for a purely competitive firm. At the longrun equilibrium level of output, this firm's economic profit: A) is zero. B) is $400. C) is $200. D) cannot be determined from the information provided. Answer: A

McConnell/Brue: Economics, 16/e

Page 721

Chapter 23: Pure Competition

Type: A Topic: 4 E: 427 MI: 183 184. The MR = MC rule applies: A) in the short run, but not in the long run. B) in the long run, but not in the short run. Answer: C

C) in both the short run and the long run. D) only to a purely competitive firm.

Type: A Topic: 4 E: 430 MI: 186 185. If the long-run supply curve of a purely competitive industry slopes upward, this implies that the prices of relevant resources: A) will fall as the industry expands. C) rise as the industry contracts. B) are constant as the industry expands. D) rise as the industry expands. Answer: D

Use the following to answer questions 186-187:

Type: G Topic: 4 E: 430 MI: 186 186. Refer to the above diagram. Line (1) reflects the long-run supply curve for: A) a constant-cost industry. C) an increasing-cost industry. B) a decreasing-cost industry. D) technologically progressive industry. Answer: C

Type: G Topic: 4 E: 429 MI: 185 187. Refer to the above diagram. Line (2) reflects the long-run supply curve for: A) a constant-cost industry. C) an increasing-cost industry. B) a decreasing-cost industry. D) technologically progressive industry. Answer: A

Pure competition and efficiency

Type: A Topic: 5 E: 432 MI: 188 188. Allocative efficiency is achieved when the production of a good occurs where: A) P = minimum ATC. B) P = MC. C) P = minimum AVC. D) total revenue is equal to TFC. Answer: B

McConnell/Brue: Economics, 16/e

Page 722

Chapter 23: Pure Competition

Type: A Topic: 5 E: 432 MI: 188 189. A firm is producing an output such that the benefit from one more unit is more than the cost of producing that additional unit. This means the firm is: A) producing more output than allocative efficiency requires. B) producing less output than allocative efficiency requires. C) achieving productive efficiency. D) producing an inefficient output, but we cannot say whether output should be increased or decreased. Answer: B

Type: A Topic: 5 E: 431 MI: 187 190. Resources are efficiently allocated when production occurs where: A) marginal cost equals average variable cost. C) price is equal to marginal cost. B) price is equal to average revenue. D) price is equal to average variable cost. Answer: C

Type: D Topic: 5 E: 432 MI: 188 191. The term productive efficiency refers to: A) any short-run equilibrium position of a competitive firm. B) the production of the product-mix most desired by consumers. C) the production of a good at the lowest average total cost. D) fulfilling the condition P = MC. Answer: C

Type: A Topic: 5 E: 432 MI: 188 192. If the price of product Y is $25 and its marginal cost is $18: A) Y is being produced with the least-cost combination of resources. B) society will realize a net gain if less of Y is produced. C) resources are being underallocated to Y. D) resources are being overallocated to Y. Answer: C

Type: D Topic: 5 E: 432 MI: 188 193. The term allocative efficiency refers to: A) the level of output that coincides with the intersection of the MC and AVC curves. B) minimization of the AFC in the production of any good. C) the production of the product-mix most desired by consumers. D) the production of a good at the lowest average total cost. Answer: C

Type: A Topic: 5 E: 433 MI: 189 194. Under pure competition in the long run: A) neither allocative efficiency nor productive efficiency are achieved. B) both allocative efficiency and productive efficiency are achieved. C) productive efficiency is achieved, but allocative efficiency is not. D) allocative efficiency is achieved, but productive efficiency is not. Answer: B

McConnell/Brue: Economics, 16/e

Page 723

Chapter 23: Pure Competition

Type: A Topic: 5 E: 432 MI: 188 195. If for a firm P = minimum ATC = MC, then: A) neither allocative efficiency nor productive efficiency is being achieved. B) productive efficiency is being achieved, but allocative efficiency is not. C) both allocative efficiency and productive efficiency are being achieved. D) allocative efficiency is being achieved, but productive efficiency is not. Answer: C

Use the following to answer questions 196-201:

Type: G Topic: 5 E: 431 MI: 187 196. The above diagram portrays: A) a competitive firm that should shut down in the short run. B) the equilibrium position of a competitive firm in the long run. C) a competitive firm that is realizing an economic profit. D) the loss-minimizing position of a competitive firm in the short run. Answer: B

Type: G Topic: 5 E: 431 MI: 187 Status: New 197. Refer to the above diagram. If this competitive firm produces output Q, it will: A) suffer an economic loss. B) earn a normal profit. C) earn an economic profit. D) achieve productive efficiency, but not allocative efficiency. Answer: B

Type: G Topic: 5 E: 433 MI: 189 198. Refer to the above diagram. By producing output level Q: A) neither productive nor allocative efficiency are achieved. B) both productive and allocative efficiency are achieved. C) allocative efficiency is achieved, but productive efficiency is not. D) productive efficiency is achieved, but allocative efficiency is not. Answer: B

McConnell/Brue: Economics, 16/e

Page 724

Chapter 23: Pure Competition

Type: G Topic: 5 E: 432 MI: 188 199. Refer to the above diagram. At output level Q1: A) neither productive nor allocative efficiency are achieved. B) both productive and allocative efficiency are achieved. C) allocative efficiency is achieved, but productive efficiency is not. D) productive efficiency is achieved, but allocative efficiency is not. Answer: A

Type: G Topic: 5 E: 432 MI: 188 200. Refer to the above diagram. At output level Q1: A) resources are overallocated to this product and productive efficiency is not realized. B) resources are underallocated to this product and productive efficiency is not realized. C) productive efficiency is achieved, but resources are underallocated to this product. D) productive efficiency is achieved, but resources are overallocated to this product. Answer: B

Type: G Topic: 5 E: 432-433 MI: 188-189 201. Refer to the above diagram. At output level Q2: A) resources are overallocated to this product and productive efficiency is not realized. B) resources are underallocated to this product and productive efficiency is not realized. C) productive efficiency is achieved, but resources are underallocated to this product. D) productive efficiency is achieved, but resources are overallocated to this product. Answer: A

Type: A Topic: 5 E: 432 MI: 188 202. Assume that society places a higher value on the last unit of X produced than the value of the resources used to produce that unit. With no spillovers, this information means that: A) total cost is greater than total revenue. C) marginal cost is greater than price. B) price is greater than marginal cost. D) resources are being overallocated to X. Answer: B

Type: A Topic: 5 E: 432-433 MI: 188-189 203. If production is occurring where marginal cost exceeds price, the purely competitive firm will: A) maximize profit, but resources will be underallocated to the product. B) maximize profit, but resources will be overallocated to the product. C) fail to maximize profit and resources will be overallocated to the product. D) fail to maximize profit and resources will be underallocated to the product. Answer: C

Type: A Topic: 5 E: 432 MI: 188 204. If a purely competitive firm is producing where price exceeds marginal cost, then: A) the firm will fail to maximize profit, but resources will be efficiently allocated. B) the firm will fail to maximize profit and resources will be overallocated to the product. C) the firm will fail to maximize profit and resources will be underallocated to the product. D) resources will be underallocated to the product, but the firm will maximize profit. Answer: C

McConnell/Brue: Economics, 16/e

Page 725

Chapter 23: Pure Competition

Type: A Topic: 5 E: 432 MI: 188 Status: New 205. Which of the following conditions is true for a purely competitive firm in long-run equilibrium? A) P > MC = minimum ATC. C) P = MC = minimum ATC. B) P > MC > minimum ATC. D) P < MC < minimum ATC. Answer: C

Consider This Questions

Type: A E: 423 MI: 179 Status: New 206. (Consider This) An unprofitable motel will stay open in the short-run if: A) price (average nightly room rate) exceeds average variable cost. B) marginal revenue exceeds marginal cost. C) price (average nightly room rate) exceeds average fixed cost. D) marginal revenue exceeds price. Answer: A

Type: A E: 423 MI: 179 Status: New 207. (Consider This) An otherwise unprofitable motel located on a largely abandoned roadway might be able to stay open for several years by: A) increasing its nightly room rates. B) reducing or eliminating its annual maintenance expenses. C) charging room rates that exceed marginal revenue. D) eliminating its fixed costs, including its opportunity costs. Answer: B

Last Word Questions

Use the following to answer questions 208-210:

Type: G E: 434 MI: 190 208. (Last Word) Refer to the above graph of the market for asparagus. At the market price of $2, area A + B represents: A) total consumer utility. C) consumer surplus. B) total revenue to sellers. D) returns to capital and to labor. Answer: A

McConnell/Brue: Economics, 16/e

Page 726

Chapter 23: Pure Competition

Type: G E: 434 MI: 190 209. (Last Word) Refer to the above graph of the market for asparagus. At the market price of $2, area A represents: A) total consumer utility. B) total revenue to sellers. C) consumer surplus. D) economic profit. Answer: C

Type: G E: 434 MI: 190 210. (Last Word) In long-run equilibrium, purely competitive markets: A) minimize total cost. B) maximize consumer surplus. C) yield economic profits to most sellers. D) inevitably degenerate into monopoly in increasing cost industries. Answer: B

True/False Questions

Type: A E: 416 MI: 172 211. In maximizing profit a firm will always produce that output where total revenues are at a maximum. Answer: False

Type: A E: 422 MI: 178 212. In the short run a competitive firm will always choose to shut down if product price is less than the lowest attainable average total cost. Answer: False

Type: A E: 427 MI: 183 213. After all long-run adjustments have been completed, a firm in a competitive industry will produce that level of output where average total cost is at a minimum. Answer: True

Type: A E: 430-431 MI: 186-187 214. The long-run supply curve for a decreasing-cost industry is downsloping. Answer: True

Type: A E: 420 MI: 176 215. A competitive firm will produce in the short run so long as its price exceeds its average fixed cost. Answer: False

Type: A E: 432 MI: 188 216. Marginal cost is a measure of the alternative goods which society forgoes in using resources to produce an additional unit of some specific product. Answer: True

McConnell/Brue: Economics, 16/e

Page 727

Chapter 23: Pure Competition

Type: A E: 415-416 MI: 171-172 217. Price and marginal revenue are identical for an individual purely competitive seller. Answer: True

Type: A E: 432 MI: 188 218. Because the equilibrium position of a purely competitive seller entails an equality of price and marginal costs, competition produces up to an efficient allocation of economic resources. Answer: True

Type: A E: 423-424 MI: 179-180 219. The short-run supply curve slopes upward because producers must be compensated for rising marginal costs. Answer: True

Type: A E: 425 MI: 181 220. The demand curve for a purely competitive industry is perfectly elastic, but the demand curves faced by individual firms in such an industry are downsloping. Answer: False

Type: A E: 417-418 MI: 173-174 221. The total revenue curve of a competitive seller graphs as a straight, upsloping line. Answer: True

Type: D E: 416 MI: 172 222. Marginal revenue is the addition to total revenue resulting from the sale of one more unit of output. Answer: True

Use the following to answer questions 223-231:

Type: G E: 419-420 MI: 175-176 223. Refer to the above diagram. This firm will maximize profits by producing output D. Answer: False

McConnell/Brue: Economics, 16/e

Page 728

Chapter 23: Pure Competition

Type: G E: 419-420 MI: 175-176 224. Refer to the above diagram. At the profit-maximizing output total revenue will be 0GLD. Answer: False

Type: G E: 420 MI: 176 225. Refer to the above diagram. At output C production will result in an economic profit. Answer: True

Type: G E: 420 MI: 176 226. Refer to the above diagram. At output C total variable cost is FGKJ. Answer: False

Type: G E: 420 MI: 176 227. Refer to the above diagram. At output C average fixed cost is GF. Answer: False

Type: G E: 424 MI: 180 228. Refer to the above diagram. At any price below R the firm will shut down in the short run. Answer: True

Type: G E: 424 MI: 180 229. Refer to the above diagram. If demand fell to the level of FNJ, there would be no output at which the firm could realize an economic profit. Answer: False

Type: G E: 424 MI: 180 230. Refer to the above diagram. If the firm produced D units of output at price G, it would earn a normal profit. Answer: True

Type: G E: 417 MI: 173 231. Refer to the above diagram. Total costs are minimized at output level B. Answer: False

Type: A E: 425 MI: 181 232. Although individual purely competitive firms can influence the price of their product, these firms as a group cannot influence market price. Answer: False

Type: A E: 414 MI: 170 233. In a purely competitive industry competition centers more on advertising and sales promotion than on price. Answer: False

McConnell/Brue: Economics, 16/e

Page 729

CHAPTER 24

Pure Monopoly

Topic 1. 2. 3. 4. 5. 6. 7. Monopoly concept; definition Barriers to entry Monopoly demand curve Profit maximization Economic implications Price discrimination Regulated monopolies Consider This Last Word True-False

Question numbers 1-7 8-13 14-73 74-116 117-145 146-164 165-179 180-181 182-184 185-208

____________________________________________________________

_______________________________________

____________________________________________________________

_______________________________________

Multiple Choice Questions Monopoly concept; definition

Type: D Topic: 1 E: 438 MI: 194 1. Pure monopoly means: A) any market in which the demand curve to the firm is downsloping. B) a standardized product being produced by many firms. C) a single firm producing a product for which there are no close substitutes. D) a large number of firms producing a differentiated product. Answer: C

Type: D Topic: 1 E: 438 MI: 194 2. Which of the following is correct? A) Both purely competitive and monopolistic firms are "price takers." B) Both purely competitive and monopolistic firms are "price makers." C) A purely competitive firm is a "price taker," while a monopolist is a "price maker." D) A purely competitive firm is a "price maker," while a monopolist is a "price taker." Answer: C

Chapter 24: Pure Monopoly

Type: A Topic: 1 E: 438-439 MI: 194-195 3. A purely monopolistic industry: A) has no entry barriers. B) has a downward sloping demand curve. C) produces a product or service for which there are many close substitutes. D) earns only a normal profit in the long run. Answer: D

Type: D Topic: 1 E: 438 MI: 194 4. A pure monopolist is: A) any firm realizing all existing economies of scale. B) any firm whose demand curve is downsloping. C) any firm which can engage in price discrimination. D) a one-firm industry. Answer: D

Type: A Topic: 1 E: 439 MI: 195 5. Pure monopolists may obtain economic profits in the long run because: A) of advertising. C) of barriers to entry. B) marginal revenue is constant as sales increase. D) of rising average fixed costs. Answer: C

Type: F Topic: 1 E: 439 MI: 195 6. Which of the following approximates a pure monopoly? A) the foreign exchange market C) the diamond market B) the Kansas City wheat market D) the soft drink market Answer: C

Type: A Topic: 1 E: 439 MI: 195 7. Which of the following is a characteristic of pure monopoly? A) close substitute products B) barriers to entry C) the absence of market power Answer: B

D) "price taking"

Barriers to entry

Type: A Topic: 2 E: 439-441 MI: 195-197 8. Which of the following is not a barrier to entry? A) patents B) X-inefficiency C) economies of scale Answer: B

D) ownership of essential resources

Type: A Topic: 2 E: 439 MI: 195 9. Barriers to entering an industry: A) are justified because they result in allocative efficiency. B) are justified because they result in productive efficiency. C) are the basis for monopoly. D) apply only to purely monopolistic industries. Answer: C

McConnell/Brue: Economics, 16/e

Page 732

Chapter 24: Pure Monopoly

Type: A Topic: 2 E: 440 MI: 196 10. Patents: A) give firms the exclusive right to produce or control a product for 100 years. B) discourage research and innovation. C) are a source of monopoly. D) are also called trademarks. Answer: D

Type: A Topic: 2 E: 440 MI: 196 11. A natural monopoly occurs when: A) long-run average costs decline continuously through the range of demand. B) a firm owns or controls some resource essential to production. C) long-run average costs rise continuously as output is increased. D) economies of scale are obtained at relatively low levels of output. Answer: A

Type: A Topic: 2 E: 440 MI: 196 12. Large minimum efficient scale of plant combined with limited market demand may lead to: A) natural monopoly. B) patent monopoly C) government franchise monopoly. D) shared monopoly. Answer: A

Type: A Topic: 2 E: 439-441 MI: 195-197 13. What do economies of scale, the ownership of essential raw materials, and patents have in common? A) They must all be present before price discrimination can be practiced. B) They are all barriers to entry. C) They all help explain why a monopolist's demand and marginal revenue curves coincide. D) They all help explain why the long-run average cost curve is U-shaped. Answer: B

Monopoly demand curve

Type: A Topic: 3 E: 441 MI: 197 14. The nondiscriminating pure monopolist's demand curve: A) is the industry demand curve. B) shows a direct or positive relationship between price and quantity demanded. C) tends to be inelastic at high prices and elastic at low prices. D) is identical to its marginal revenue curve. Answer: D

Type: A Topic: 3 E: 441-442 MI: 197-198 15. The nondiscriminating monopolist's demand curve: A) is less elastic than a purely competitive firm's demand curve. B) is perfectly elastic. C) coincides with its marginal revenue curve. D) is perfectly inelastic. Answer: A

McConnell/Brue: Economics, 16/e

Page 733

Chapter 24: Pure Monopoly

Type: A Topic: 3 E: 441-442 MI: 197-198 16. If a nondiscriminating imperfectly competitive firm is selling its 100th unit of output for $35, its marginal revenue: A) may be either greater or less than $35. C) will be less than $35. B) will also be $35. D) will be greater than $35. Answer: C

Type: A Topic: 3 E: 441-442 MI: 197-198 17. A nondiscriminating pure monopolist's demand curve: A) is perfectly inelastic. C) lies above its marginal revenue curve. B) coincides with its marginal revenue curve. D) lies below its marginal revenue curve. Answer: C

Type: A Topic: 3 E: 441-442 MI: 197-198 18. For an imperfectly competitive firm: A) total revenue is a straight, upsloping line because a firm's sales are independent of product price. B) the marginal revenue curve lies above the demand curve because any reduction in price applies to all units sold. C) the marginal revenue curve lies below the demand curve because any reduction in price applies to all units sold. D) the marginal revenue curve lies below the demand curve because any reduction in price applies only to the extra unit sold. Answer: C

Type: A Topic: 3 E: 442-443 MI: 198-199 19. For a nondiscriminating imperfectly competitive firm: A) the marginal revenue curve lies above the demand curve. B) the demand and marginal revenue curves coincide. C) the demand curve intersects the horizontal axis where total revenue is at a maximum. D) marginal revenue will become zero at that output where total revenue is at a maximum. Answer: D

Type: A Topic: 3 E: 443-444 MI: 199-200 20. When a firm is on the inelastic segment of its demand curve, it can: A) increase total revenue by reducing price. B) decrease total costs by decreasing price. C) increase profits by increasing price. D) increase total revenue by more than the increase in total cost by increasing price. Answer: C

McConnell/Brue: Economics, 16/e

Page 734

Chapter 24: Pure Monopoly

Use the following to answer questions 21-23:

Type: G Topic: 3 E: 443-444 MI: 199-200 21. Refer to the above diagram. If price is reduced from P1 to P2, total revenue will: A) increase by A minus C. C) decrease by A minus C. B) increase by C minus A. D) decrease by C minus A. Answer: B

Type: G Topic: 3 E: 443-444 MI: 199-200 22. Refer to the above diagram. The quantity difference between areas A and C for the indicated price reduction measures: A) marginal cost. B) marginal revenue. C) monopoly price. D) a welfare or efficiency loss. Answer: B

Type: G Topic: 3 E: 442-443 MI: 198-199 23. The above diagram implies that whenever a firm's demand curve is downsloping: A) price discrimination is not possible. B) monopolists will be more efficient than competitors. C) the demand and marginal revenue curves will coincide. D) marginal revenue is less than price. Answer: D

Use the following to answer questions 24-25: Answer the next question(s) on the basis of the demand schedule shown below:
Price $7 6 5 4 3 Quantity demanded 1 2 3 4 5

Type: T Topic: 3 E: 442 MI: 198 24. Refer to the above data. The marginal revenue obtained from selling the third unit of output is: A) $6. B) $1. C) $3. D) $5. Answer: C

McConnell/Brue: Economics, 16/e

Page 735

Chapter 24: Pure Monopoly

Type: T Topic: 3 E: 443 MI: 199 25. Refer to the above data. At the point where 3 units are being sold, the coefficient of price elasticity of demand: A) cannot be estimated. C) is less than unity (one). B) suggests that the market is purely competitive. D) is greater than unity (one). Answer: D

Type: A Topic: 3 E: 442 MI: 198 26. A monopolistic firm has a sales schedule such that it can sell 10 prefabricated garages per week at $10,000 each, but if it restricts its output to 9 per week it can sell these at $11,000 each. The marginal revenue of the tenth unit of sales per week is: A) -$1,000. B) $9,000. C) $10,000. D) $1,000. Answer: D

Use the following to answer questions 27-31:

Type: G Topic: 3 E: 416 MI: 172 27. Refer to the above two diagrams for individual firms. Figure 1 pertains to: A) an imperfectly competitive firm. C) an oligopolist. B) a purely competitive firm. D) a pure monopolist. Answer: B

Type: G Topic: 3 E: 416 MI: 172 28. Refer to the above two diagrams for individual firms. In Figure 1 line B represents the firm's: A) demand and marginal revenue curves. C) marginal revenue curve only. B) demand curve only. D) average revenue curve only. Answer: A

Type: G Topic: 3 E: 416 MI: 172 29. Refer to the above two diagrams for individual firms. In Figure 1, line A represents the firm's: A) demand and marginal revenue curves. C) marginal revenue curve only. B) demand curve only. D) total revenue curve only. Answer: D

McConnell/Brue: Economics, 16/e

Page 736

Chapter 24: Pure Monopoly

Type: G Topic: 3 E: 441 MI: 197 30. Refer to the above two diagrams for individual firms. Figure 2 pertains to: A) a market characterized by government regulation of price and output. B) either an imperfectly competitive or a purely competitive seller. C) a purely competitive seller. D) an imperfectly competitive seller. Answer: D

Type: G Topic: 3 E: 441-442 MI: 197-198 31. Refer to the above two diagrams for individual firms. In Figure 2 the firm's demand and marginal revenue curves are represented by: A) lines B and C respectively. C) lines A and B respectively. B) lines A and C respectively. D) line B. Answer: A

Type: A Topic: 3 E: 441-442 MI: 197-198 32. With respect to the pure monopolist's demand curve it can be said that: A) the stronger the barriers to entry, the more elastic is the monopolist's demand curve. B) price exceeds marginal revenue at all outputs greater than 1. C) demand is perfectly inelastic. D) marginal revenue equals price at all outputs. Answer: B

Type: A Topic: 3 E: 441 MI: 197 33. The pure monopolist's demand curve is: A) identical with the industry demand curve. B) of unit elasticity throughout. Answer: A

C) perfectly inelastic. D) perfectly elastic.

McConnell/Brue: Economics, 16/e

Page 737

Chapter 24: Pure Monopoly

Use the following to answer questions 34-37:

Type: G Topic: 3 E: 441-442 MI: 197-198 34. Refer to the above diagram. This firm is selling in: A) a market in which there are an extremely large number of other firms producing the same product. B) an imperfectly competitive market. C) a market in which demand is elastic at all prices. D) a purely competitive market. Answer: B

Type: G Topic: 3 E: 442-443 MI: 198-199 35. Refer to the above diagram. Demand is relatively elastic: A) in the P2P1 price range. C) in the P2P4 price range. B) in the 0P1 price range. D) only at price P2. Answer: C

Type: G Topic: 3 E: 442-443 MI: 198-199 36. Refer to the above diagram. Demand is relatively inelastic: A) at price P3. B) at any price below P2. C) in the P2P4 price range. Answer: B

D) in the P2P3 price range.

Type: G Topic: 3 E: 444 MI: 200 37. Refer to the above diagram. If this somehow was a costless product (that is, the total cost of any level of output was zero), the firm would maximize profits by: A) selling the product at the highest possible price at which a positive quantity will be demanded. B) producing Q1 units and charging a price of P1. C) producing Q3 units and charging a price of P3. D) producing Q2 units and charging a price of P2. Answer: D

McConnell/Brue: Economics, 16/e

Page 738

Chapter 24: Pure Monopoly

Type: A Topic: 3 E: 441-442 MI: 197-198 38. Price exceeds marginal revenue for the pure monopolist because the: A) law of diminishing returns is inapplicable. B) demand curve is downsloping. C) monopolist produces a smaller output than would a purely competitive firm. D) demand curve lies below the marginal revenue curve. Answer: B

Type: A Topic: 3 E: 442-443 MI: 198-199 39. The demand curve faced by a pure monopolist: A) may be either more or less elastic than that faced by a single purely competitive firm. B) is less elastic than that faced by a single purely competitive firm. C) has the same elasticity as that faced by a single purely competitive firm. D) is more elastic than that faced by a single purely competitive firm. Answer: B

Type: A Topic: 3 E: 443 MI: 199 40. The marginal revenue curve for a monopolist: A) is a straight, upward sloping curve. B) rises at first, reaches a maximum, and then declines. C) becomes negative when output increases beyond some particular level. D) is a straight line, parallel to the horizontal axis. Answer: C

Type: G Topic: 3 E: 442 MI: 198 41.

If the firm in the above diagram lowers price from P1 to P2, it will: A) lose P1P 2ba in revenue from the price cut but increase revenue by Q1bcQ2 from the increase in sales. B) lose P1P 2ca in revenue from the price cut but increase revenue by Q1acQ2 from the increase in sales. C) incur a decline in total revenue because it is operating on the elastic segment of the demand curve. D) incur an increase in total revenue because it is operating on the inelastic segment of the demand curve. Answer: A

Type: G Topic: 3 E: 442 MI: 198 42. The quantitative difference between areas Q1bcQ 2 and P1P2ba in the above diagram measures: A) marginal cost. B) total revenue. C) marginal revenue. D) average revenue. Answer: C

McConnell/Brue: Economics, 16/e

Page 739

Chapter 24: Pure Monopoly

Type: A Topic: 3 E: 441 MI: 197 43. Which of the following is characteristic of a pure monopolist's demand curve? A) Average revenue is less than price. B) Its elasticity coefficient is 1 at all levels of output. C) Price and marginal revenue are equal at all levels of output. D) It is the same as the market demand curve. Answer: D

Type: A Topic: 3 E: 441 MI: 197 44. Because the monopolist's demand curve is downsloping: A) MR will equal price. B) price must be lowered to sell more output. C) the elasticity coefficient will increase as price is lowered. D) its supply curve will also be downsloping. Answer: B

Type: A Topic: 3 E: 443 MI: 199 45. The pure monopolist's demand curve is relatively elastic: A) in the price range where total revenue is declining. B) at all points where the demand curve lies above the horizontal axis. C) in the price range where marginal revenue is negative. D) in the price range where marginal revenue is positive. Answer: D

Type: A Topic: 3 E: 443 MI: 199 46. When the pure monopolist's demand curve is elastic, marginal revenue: A) may be either positive or negative. B) is zero. C) is negative. D) is positive. Answer: D

Type: A Topic: 3 E: 443 MI: 199 47. When total revenue is increasing: A) marginal revenue may be either positive or negative. B) the demand curve is relatively inelastic. C) marginal revenue is positive. D) marginal revenue is negative. Answer: C

Type: A Topic: 3 E: 443-444 MI: 199-200 48. A nondiscriminating monopolist: A) will never produce in the output range where marginal revenue is positive. B) will never produce in the output range where demand is inelastic. C) will never produce in the output range where demand is elastic. D) may produce where demand is either elastic or inelastic, depending on the level of production costs. Answer: B

McConnell/Brue: Economics, 16/e

Page 740

Chapter 24: Pure Monopoly

Type: A Topic: 3 E: 443 MI: 199 49. For a pure monopolist the relationship between total revenue and marginal revenue is such that: A) marginal revenue is positive when total revenue is at a maximum. B) total revenue is positive when marginal revenue is increasing, but total revenue becomes negative when marginal revenue is decreasing. C) marginal revenue is positive when total revenue is increasing, but marginal revenue becomes negative when total revenue is decreasing. D) marginal revenue is positive so long as total revenue is positive. Answer: C

Type: A Topic: 3 E: 441 MI: 197 50. A pure monopolist's demand curve is: A) downsloping. B) upsloping. C) parallel to the vertical axis. Answer: A

D) parallel to the horizontal axis.

Type: A Topic: 3 E: 441-442 MI: 197-198 51. For a pure monopolist marginal revenue is less than price because: A) the monopolist's demand curve is perfectly elastic. B) the monopolist's demand curve is perfectly inelastic. C) when a monopolist lowers price to sell more output, the lower price applies to all units sold. D) the monopolist's total revenue curve is linear and slopes upward to the right. Answer: C

Use the following to answer questions 52-54:

Type: G Topic: 3 E: 443 MI: 199 52. Refer to the above diagram for a nondiscriminating monopolist. Demand is elastic: A) in the q1q 3 output range. C) for all levels of output less than q2. B) only for outputs greater than q4. D) for all levels of output greater than q2. Answer: C

Type: G Topic: 3 E: 442 MI: 198 53. Refer to the above diagram for a nondiscriminating monopolist. Marginal revenue will be zero at output: A) q4. B) q3. C) q2. D) q1. Answer: C

McConnell/Brue: Economics, 16/e

Page 741

Chapter 24: Pure Monopoly

Type: G Topic: 3 E: 444 MI: 200 54. Refer to the above diagram for a nondiscriminating monopolist. The profit-seeking monopolist will: A) always produce at output q2. C) never produce an output larger than q2. B) always produce more than q2. D) never produce an output larger than q1. Answer: C

Type: A Topic: 3 E: 444 MI: 200 55. Assume a pure monopolist is currently operating at a price-quantity combination on the inelastic segment of its demand curve. If the monopolist is seeking maximum profits, it should: A) retain its current price-quantity combination. C) charge a higher price. B) increase both price and quantity sold. D) charge a lower price. Answer: C

Type: A Topic: 3 E: 444 MI: 200 56. A pure monopolist should never produce in the: A) elastic segment of its demand curve because it can increase total revenue and reduce total cost by lowering price. B) inelastic segment of its demand curve because it can increase total revenue and reduce total cost by increasing price. C) inelastic segment of its demand curve because it can always increase total revenue by more than it increases total cost by reducing price. D) segment of its demand curve where the price elasticity coefficient is greater than one. Answer: B

Type: A Topic: 3 E: 441 MI: 197 57. Assuming no change in product demand, a pure monopolist: A) can increase price and increase sales simultaneously because it dominates the market. B) adds an amount to total revenue which is equal to the price of incremental sales. C) should produce in the range where marginal revenue is negative. D) must lower price to increase sales. Answer: D

Type: A Topic: 3 E: 443-444 MI: 199-200 58. If a monopolist were to produce in the inelastic segment of its demand curve: A) total revenue would be at a maximum. C) the firm would be maximizing profits. B) marginal revenue would be negative. D) it would necessarily incur a loss. Answer: D

Type: A Topic: 3 E: 443 MI: 199 59. If a pure monopolist is operating in a range of output where demand is elastic: A) it cannot possibly be maximizing profits. C) marginal revenue will be positive and rising. B) marginal revenue will be positive but declining. D) total revenue will be declining. Answer: B

McConnell/Brue: Economics, 16/e

Page 742

Chapter 24: Pure Monopoly

Type: A Topic: 3 E: 442-443 MI: 198-199 60. Suppose a pure monopolist is charging a price of $12 and the associated marginal revenue is $9. We thus know that: A) demand is inelastic at this price. C) the firm is maximizing profits. B) total revenue is increasing. D) total revenue is at a maximum. Answer: B

Type: A Topic: 3 E: 442-443 MI: 198-199 61. A pure monopolist is selling 6 units at a price of $12. If the marginal revenue of the seventh unit is $5, then: A) price of the seventh unit is $10. C) price of the seventh unit is greater than $12. B) price of the seventh unit is $11. D) firm's demand curve is perfectly elastic. Answer: B

Type: A Topic: 3 E: 442 MI: 198 62. The vertical distance between the horizontal axis and any point on a nondiscriminating monopolist's demand curve measures: A) the quantity demanded. C) total revenue. B) product price and average revenue. D) product price and marginal revenue. Answer: B

Type: G Topic: 3 E: 442 MI: 198 63.

The above diagram indicates that the marginal revenue of the sixth unit of output is: A) $1. B) -$1. C) $4. D) $24. Answer: B

Type: A Topic: 3 E: 427 MI: 183 64. Which of the following is incorrect? Imperfectly competitive producers: A) face downsloping demand curves. B) do not compete with one another. C) can alter their output by changing price. D) find that, when they reduce price, their total revenue increases by less than the new price. Answer: B

McConnell/Brue: Economics, 16/e

Page 743

Chapter 24: Pure Monopoly

Use the following to answer question 65: Answer the next question on the basis of the following table showing the demand schedule facing a nondiscriminating monopolist:
P $10 7 5 3 1 Qd 1 2 3 4 5

Type: T Topic: 3 E: 443 MI: 199 65. The monopolist will select its profit-maximizing level of output somewhere within the: A) 3-5 unit range of output. C) 1-4 unit range of output. B) 1-3 unit range of output. D) 2-4 unit range of output. Answer: B

Type: A Topic: 3 E: 442-443 MI: 198-199 66. A nondiscriminating pure monopolist finds that it can sell its fiftieth unit of output for $50. We can surmise that the marginal: A) cost of the fiftieth unit is also $50. C) revenue of the fiftieth unit is less than $50. B) revenue of the fiftieth unit is also $50. D) revenue of the fiftieth unit is greater than $50. Answer: C

Type: A Topic: 3 E: 442 MI: 198 67. If a nondiscriminating pure monopolist decides to sell one more unit of output, the marginal revenue associated with that unit will be: A) equal to its price. B) the price at which that unit is sold less the price reductions which apply to all other units of output. C) the price at which that unit is sold plus the price increases which apply to all other units of output. D) indeterminate unless marginal cost data are known. Answer: B

Type: A Topic: 3 E: 443 MI: 199 68. Assuming a pure monopolist's demand curve is downsloping, its total revenue: A) is rising. B) is falling. C) may be either rising or falling. D) must be negative. Answer: C

McConnell/Brue: Economics, 16/e

Page 744

Chapter 24: Pure Monopoly

Use the following to answer questions 69-71:

Type: G Topic: 3 E: 443 MI: 199 69. Which of the above diagrams correctly portray a nondiscriminating pure monopolist's demand (D) and marginal revenue (MR) curves? A) A B) B C) C D) D Answer: B

Type: G Topic: 3 E: 454 MI: 210 70. Which of the above diagrams correctly portray the demand (D) and marginal revenue (MR) curves of a pure monopolist that is practicing perfect price discrimination? A) A B) B C) C D) D Answer: A

Type: G Topic: 3 E: 415-416 MI: 171-172 71. Which of the above diagrams correctly portray the demand (D) and marginal revenue (MR) curves of a purely competitive seller? A) A B) B C) C D) D Answer: C

Type: A Topic: 3 E: 442 MI: 198 Status: New 72. Suppose that a pure monopolist can sell 20 units of output at $10 per unit and 21 units at $9.75 per unit. The marginal revenue of the twenty-first unit of output is: A) $9.75. B) $204.75. C) $4.75. D) $.25. Answer: C

Type: A Topic: 3 E: 442 MI: 198 Status: New 73. Suppose that a pure monopolist can sell 10 units of output at $5 per unit and 11 units at $4.90 per unit. The marginal revenue of the eleventh unit is: A) $3.90. B) $.10. C) $53.90. D) $4.90. Answer: A

McConnell/Brue: Economics, 16/e

Page 745

Chapter 24: Pure Monopoly

Profit maximization

Type: A Topic: 4 E: 444-445 MI: 200-201 74. The MR = MC rule: A) applies only to pure competition. B) applies only to pure monopoly. C) does not apply to pure monopoly because price exceeds marginal revenue. D) applies both to pure monopoly and pure competition. Answer: D

Type: A Topic: 4 E: 444-445 MI: 200-201 75. In the long run a pure monopolist will maximize profits by producing that output at which marginal cost is equal to: A) average total cost. B) marginal revenue. C) average variable cost. D) average cost. Answer: B

Type: A Topic: 4 E: 444-445 MI: 200-201 76. An unregulated pure monopolist will maximize profits by producing that output at which: A) P = MC. B) P = ATC. C) MR = MC. D) MC = AC. Answer: C

Type: C Topic: 4 E: 444-445 MI: 200-201 Status: New 77. Suppose that a pure monopolist can sell 5 units of output at $4 per unit and 6 units at $3.90 per unit. The monopolist will produce and sell the sixth unit if its marginal cost is: A) $4 or less. B) $3.90 or less. C) $3.50 or less.. D) $3.40 or less. Answer: D

Type: C Topic: 4 E: 444-445 MI: 200-201 Status: New 78. Suppose that a pure monopolist can sell 4 units of output at $2 per unit and 5 units at $1.75 per unit. The monopolist will produce and sell the fifth unit if its marginal cost is: A) $1 or less. B) $.75 or less. C) $1.75 or less. D) $2 or less. Answer: B

McConnell/Brue: Economics, 16/e

Page 746

Chapter 24: Pure Monopoly

Use the following to answer questions 79-84:

Total output 1 2 3 4 5 6 7 8 9 10

Price $100 90 80 70 60 50 40 30 20 10

Marginal revenue $100 80 60 40 20 0 -20 -40 -60 -80

Average total cost $100.00 63.00 52.67 49.50 49.60 50.00 52.29 55.75 60.67 67.60

Marginal cost $ 30 26 32 40 50 52 66 80 100 130

Type: G Topic: 4 E: 444-445 MI: 200-201 Status: New 79. Refer to the above data for a nondiscriminating monopolist. This firm will maximize its profit by producing: A) 3 units. B) 4 units. C) 5 units. D) 6 units. Answer: B

Type: G Topic: 4 E: 444-445 MI: 200-201 Status: New 80. Refer to the above data for a nondiscriminating monopolist. At its profit-maximizing output, this firm will be operating in the: A) perfectly elastic portion of its demand curve. C) elastic portion of its demand curve. B) perfectly inelastic portion of its demand curve. D) inelastic portion of its demand curve. Answer: C

Type: G Topic: 4 E: 444-445 MI: 200-201 Status: New 81. Refer to the above data for a nondiscriminating monopolist. At its profit-maximizing output, this firm's price will exceed its marginal cost by ____ and its average total cost by ____. A) $20; $27.33. B) $10; $10.40. C) $24; $27.33. D) $30; $20.50. Answer: D

Type: G Topic: 4 E: 444-445 MI: 200-201 Status: New 82. Refer to the above data for a nondiscriminating monopolist. At its profit-maximizing output, this firm's total costs will be: A) $300. B) $248. C) $198. D) $126. Answer: C

Type: G Topic: 4 E: 444-445 MI: 200-201 Status: New 83. Refer to the above data. At its profit-maximizing output, this firm's total revenue will be: A) $300. B) $198. C) $180. D) $280. Answer: D

McConnell/Brue: Economics, 16/e

Page 747

Chapter 24: Pure Monopoly

Type: G Topic: 4 E: 444-445 MI: 200-201 Status: New 84. Refer to the above data for a nondiscriminating monopolist. At its profit-maximizing output, this firm's total profit will be: A) $82. B) zero. C) $54. D) $27. Answer: A

Type: A Topic: 4 E: 444-445 MI: 200-201 85. A pure monopolist is producing an output such that ATC = $4, P = $5, MC = $2, and MR = $3. This firm is realizing: A) a loss that could be reduced by producing more output. B) a loss that could be reduced by producing less output. C) an economic profit that could be increased by producing more output. D) an economic profit that could be increased by producing less output. Answer: C

Type: A Topic: 4 E: 446 MI: 202 86. Which of the following statements is incorrect? A) A monopolist's 100 percent market share ensures economic profits. B) The monopolist's marginal revenue is less than price for any given output greater than 1. C) A monopolistic firm produces a product having no close substitutes. D) A pure monopolist's demand curve is the industry demand curve. Answer: A

Type: A Topic: 4 E: 444-445 MI: 200-201 87. If a monopolist's marginal revenue is $3.00 and its marginal cost is $4.50, it will increase its profits by: A) reducing output and raising price. C) increasing both price and output. B) reducing both output and price. D) raising price while keeping output unchanged. Answer: A

Use the following to answer questions 88-90: Answer the next question(s) on the basis of the following demand and cost data for a pure monopolist:

Demand Data Quantity demanded Price $5.50 3 5.00 4 4.50 5 3.85 6 3.35 7 2.90 8 2.50 9

Cost Data Total Output cost 3 $ 5 4 6 6.50 5 7.50 6 7 9 8 11 9 14

Type: T Topic: 4 E: 444-445 MI: 200-201 88. Refer to the above data. Equilibrium price for the monopolist will be: A) $5.00 B) $2.90. C) $3.35. D) $4.50. Answer: D

McConnell/Brue: Economics, 16/e

Page 748

Chapter 24: Pure Monopoly

Type: T Topic: 4 E: 444-445 MI: 200-201 89. Refer to the above data. The equilibrium level of output will be: A) 4 units. B) 7 units. C) 6 units. D) 5 units. Answer: D

Type: T Topic: 4 E: 444-445 MI: 200-201 90. Refer to the above data. The monopolist will realize a: A) profit of $8.50. B) profit of $7.50. C) profit of $16. Answer: C

D) loss of $14.

Use the following to answer questions 91-94:

Type: G Topic: 4 E: 445 MI: 201 91. Refer to the above diagram. To maximize profits or minimize losses this firm should produce: A) E units and charge price C. C) M units and charge price N. B) E units and charge price A. D) L units and charge price LK. Answer: B

Type: G Topic: 4 E: 445 MI: 201 92. Refer to the above diagram. In equilibrium total revenue will be: A) NM times 0M. B) 0AJE. C) 0EGC. D) 0EHB. Answer: B

Type: G Topic: 4 E: 445 MI: 201 93. Refer to the above diagram. In equilibrium total cost will be: A) NM times 0M. B) 0AJE. C) 0CGC. D) 0BHE. Answer: D

McConnell/Brue: Economics, 16/e

Page 749

Chapter 24: Pure Monopoly

Type: G Topic: 4 E: 445 MI: 201 94. Refer to the above diagram. In equilibrium the firm will realize: A) an economic profit of ABHJ. C) a loss of GH per unit. B) an economic profit of ACGJ. D) a loss of JH per unit. Answer: A

Type: A Topic: 4 E: 444 MI: 200 95. In equilibrium which of the following conditions are common to both unregulated monopoly and to pure competition? A) MC = P B) MC = ATC C) MR = MC D) P = MR Answer: C

Type: A Topic: 4 E: 444-445 MI: 200-201 96. A pure monopolist: A) will realize an economic profit if price exceeds ATC at the equilibrium output. B) will realize an economic profit if ATC exceeds MR at the equilibrium output. C) will realize an economic loss if MC intersects the downsloping portion of MR. D) always realizes an economic profit. Answer: A

Type: A Topic: 4 E: 444-445 MI: 200-201 97. If a pure monopolist is producing at that output where P = ATC, then: A) its economic profits will be zero. B) it will be realizing losses. C) it will be producing less than the profit-maximizing level of output. D) it will be realizing an economic profit. Answer: A

Type: A Topic: 4 E: 444 MI: 200 98. A pure monopolist's short-run profit-maximizing or loss-minimizing position is such that price: A) equals marginal revenue. C) will always equal ATC. B) may be greater or less than ATC. D) always exceeds ATC. Answer: B

Type: A Topic: 4 E: 444 MI: 200 99. The short-run profit maximizing position of an unregulated pure monopolist is characterized by: A) P = minimum ATC. B) P = MC. C) MR = MC. D) MC = ATC. Answer: C

McConnell/Brue: Economics, 16/e

Page 750

Chapter 24: Pure Monopoly

Use the following to answer questions 100-102:

Type: G Topic: 4 E: 445 MI: 201 100. Refer to the above diagram for a pure monopolist. Monopoly price will be: A) e. B) c. C) b. D) a. Answer: B

Type: G Topic: 4 E: 445 MI: 201 101. Refer to the above diagram for a pure monopolist. Monopoly output will be: A) between f and g. B) h. C) g. D) f . Answer: D

Type: G Topic: 4 E: 445 MI: 201 102. Refer to the above diagram for a pure monopolist. Monopoly profit: A) cannot be determined from the information given. B) will be ae per unit. C) will be bc per unit. D) will be ac per unit. Answer: A

Type: A Topic: 4 E: 446 MI: 202 103. In the short run a pure monopolist's profit: A) will be maximized where price equals average total cost. B) may be positive, zero, or negative. C) are always positive. D) will be zero. Answer: B

Type: A Topic: 4 E: 444 MI: 200 104. Purely competitive firms and pure monopolists are similar in that: A) the demand curves of both are perfectly elastic. C) both are price makers. B) significant entry barriers are common to both. D) both maximize profit where MR = MC. Answer: D

McConnell/Brue: Economics, 16/e

Page 751

Chapter 24: Pure Monopoly

Type: A Topic: 4 E: 446 MI: 202 105. In the short run, a monopolist's economic profits: A) are always positive because the monopolist is a price-maker. B) are usually negative because of government price regulation. C) are always zero because consumers prefer to buy from competitive sellers. D) may be positive or negative depending on market demand and cost. Answer: D

Type: A Topic: 4 E: 443-444 MI: 199-200 106. A profit-maximizing monopolist will set its price: A) as far above ATC as possible. B) along the elastic portion of its demand curve. C) where the marginal cost curve intersects the demand curve. D) as close as possible to the minimum point of ATC. Answer: B

Type: A Topic: 4 E: 444 MI: 200 107. Under which of the following situations would a monopolist increase profits by lowering price (and increasing output): A) if it discovered that it was producing where MC = MR B) if it discovered that it was producing where its MC curve intersects its demand curve C) if it discovered that it was producing where MC < MR D) under none of the above circumstances because a monopolist would never lower price Answer: C

Use the following to answer questions 108-109:

Type: G Topic: 4 E: 445 MI: 201 108. Refer to the above diagram. If this industry is purely competitive, the profit-maximizing price and quantity will be: A) P3 and Q3. B) P1 and Q1. C) P2 and Q2. D) indeterminate on the basis of the information given. Answer: C

McConnell/Brue: Economics, 16/e

Page 752

Chapter 24: Pure Monopoly

Type: G Topic: 4 E: 445 MI: 201 109. Refer to the above diagram. If this industry is comprised of only one seller, the profit-maximizing price and quantity will be: A) P3 and Q3. B) P1 and Q1. C) P2 and Q2. D) indeterminate on the basis of the information given. Answer: A

Type: A Topic: 4 E: 446 MI: 202 110. When a pure monopolist is producing its profit -maximizing output, price will: A) be less than MR. B) equal neither MC nor MR. C) equal MR. D) equal MC. Answer: B

Use the following to answer question 111:

Type: G Topic: 4 E: 443 MI: 199 111. Assume a pure monopolist is charging price P and selling output Q as shown on the above diagram. On the basis of this information we can say that: A) if marginal costs were somehow zero, the firm would be maximizing its profits. B) if marginal costs were positive the firm would increase profits by reducing price and selling more output. C) the firm is producing where the price elasticity coefficient is less than one. D) the firm is a "price taker." Answer: A

Type: A Topic: 4 E: 445-446 MI: 201-202 112. The supply curve for a monopolist is: A) perfectly elastic. B) upsloping. C) that portion of the marginal cost curve lying above minimum average variable cost. D) nonexistent. Answer: D

Type: A Topic: 4 E: 445-446 MI: 201-202 113. The supply curve of a pure monopolist: A) is that portion of its marginal cost curve which lies above average variable cost. B) is the same as that of a purely competitive industry. C) is its average variable cost curve. D) does not exist because prices are not "given" to a monopolist. Answer: D

McConnell/Brue: Economics, 16/e

Page 753

Chapter 24: Pure Monopoly

Type: A Topic: 4 E: 445 MI: 201 114. If the variable costs of a profit-maximizing pure monopolist decline, the firm should: A) produce more output and charge a higher price. C) reduce both output and price. B) produce more output and charge a lower price. D) raise both output and price. Answer: B

Type: A Topic: 4 E: 446-447 MI: 202-203 115. In the short run a pure monopolist: A) always earns an economic profit. B) always earns a normal profit. C) always realizes a loss. D) may realize an economic profit, a normal profit, or a loss. Answer: D

Type: A Topic: 4 E: 446 MI: 202 116. To maximize profit a pure monopolist must: A) maximize its total revenue. B) maximize the difference between marginal revenue and marginal cost. C) maximize the difference between total revenue and total cost. D) produce where average total cost is at a minimum. Answer: C

Economic implications

Use the following to answer questions 117-121:

Type: G Topic: 5 E: 416, 443 MI: 172, 199 117. Refer to the above diagrams. Firm A is a: A) pure competitor and Firm B is a pure monopoly. B) pure competitor, as is Firm B. C) pure monopoly and Firm B is a pure competitor. D) pure monopoly, as is Firm B. Answer: A

McConnell/Brue: Economics, 16/e

Page 754

Chapter 24: Pure Monopoly

Type: G Topic: 5 E: 416 MI: 172 118. Refer to the above diagrams. The demand for Firm A's product is: A) perfectly elastic over all ranges of output. B) perfectly inelastic over all ranges of output. C) elastic for prices above $1 and inelastic for prices below $1. D) inelastic for prices above $1 and inelastic for prices below $1. Answer: A

Type: G Topic: 5 E: 443 MI: 199 119. Refer to the above diagrams. The demand for Firm B's product is: A) perfectly elastic over all ranges of output. B) perfectly inelastic over all ranges of output. C) elastic for prices above $4 and inelastic for prices below $4. D) inelastic for prices above $4 and inelastic for prices below $4. Answer: C

Type: G Topic: 5 E: 443 MI: 199 120. If $4 is Firm B's profit-maximizing price, its: A) ATC must be $4. B) MC must be $4. C) MR must be $4. D) MC must be zero. Answer: D

Type: G Topic: 5 E: 416 MI: 172 121. Firm A's average revenue is: A) zero. B) $1. C) less than $1. D) more than $1. Answer: B

Type: A Topic: 5 E: 442-443 MI: 198-199 122. Economic profit in the long run is: A) possible for both a pure monopoly and a pure competitor. B) possible for a pure monopoly, but not for a pure competitor. C) impossible for both a pure monopolist and a pure competitor. D) only possible when barriers to entry are nonexistent. Answer: B

Type: A Topic: 5 E: 448 MI: 204 123. Which of the following statements is correct? A) The pure monopolist will maximize profit by producing at that point on the demand curve where elasticity is zero. B) In seeking the profit-maximizing output the pure monopolist underallocates resources to its production. C) The pure monopolist maximizes profits by producing that output at which the differential between price and average cost is the greatest. D) Purely monopolistic sellers earn only normal profits in the long run. Answer: B

McConnell/Brue: Economics, 16/e

Page 755

Chapter 24: Pure Monopoly

Type: A Topic: 5 E: 448 MI: 204 124. Confronted with the same unit cost data, a monopolistic producer will charge: A) the same price and produce the same output as a competitive firm. B) a higher price and produce a larger output than a competitive firm. C) a higher price and produce a smaller output than a competitive firm. D) a lower price and produce a smaller output than a competitive firm. Answer: C

Type: A Topic: 5 E: 448 MI: 204 125. An important economic problem associated with pure monopoly is that, at the profit maximizing outputs, resources are: A) overallocated because price exceeds marginal cost. B) overallocated because marginal cost exceeds price. C) underallocated because price exceeds marginal cost. D) underallocated because marginal cost exceeds price. Answer: C

Type: A Topic: 5 E: 448 MI: 204 126. A single-price monopoly is economically undesirable because, at the profit maximizing output: A) marginal revenue exceeds product price at all profitable levels of production. B) monopolists always price their products on the basis of the ability of consumers to pay rather than on costs of production. C) MC > P. D) society values additional units of the monopolized product more highly than it does the alternative products those resources could otherwise produce. Answer: D

Type: C Topic: 5 E: 445, 447-448 MI: 201, 203-204 127. If a pure monopolist is producing more output than the MR = MC output: A) the firm may, or may not, be maximizing profits. B) it will be in the interest of the firm, but not necessarily of society, to reduce output. C) it will be in the interest of the firm and society to increase output. D) it will be in the interest of the firm and society to reduce output. Answer: B

Type: A Topic: 5 E: 448 MI: 204 128. At its profit-maximizing output, a pure nondiscriminating monopolist achieves: A) neither productive efficiency nor allocative efficiency. B) both productive efficiency and allocative efficiency. C) productive efficiency but not allocative efficiency. D) allocative efficiency but not productive efficiency. Answer: A

Type: A Topic: 5 E: 448 MI: 204 129. The profit-maximizing output of a pure monopoly is economically inefficient because in equilibrium: A) price equals minimum average total cost. C) marginal cost exceeds price. B) marginal revenue equals marginal cost. D) price exceeds marginal cost. Answer: D

McConnell/Brue: Economics, 16/e

Page 756

Chapter 24: Pure Monopoly

Type: A Topic: 5 E: 448 MI: 204 130. A single-price pure monopoly is economically inefficient: A) only because it produces beyond the point of minimum average total cost. B) only because it produces short of the point of minimum average total cost. C) because it produces short of minimum average cost and price is greater than marginal cost. D) because it produces beyond minimum average total cost and marginal cost is greater than price. Answer: C

Type: A Topic: 5 E: 448 MI: 204 131. Comparing a pure monopoly and a purely competitive firm with identical costs, we would find in long-run equilibrium that the pure monopolist's: A) price, output, and average total cost would all be higher. B) price and average total cost would be higher, but output would be lower. C) price, output, and average total cost would all be lower. D) price and output would be lower, but average total cost would be higher. Answer: B

Use the following to answer questions 132-137:

Type: G Topic: 5 E: 447 MI: 203 132. Refer to the above diagrams. Diagram (A) represents: A) equilibrium price and quantity in a purely competitive industry. B) the pure monopoly model. C) an industry in which there is productive efficiency but not allocative efficiency. D) a single firm operating in a purely competitive industry. Answer: A

Type: G Topic: 5 E: 447 MI: 203 133. Refer to the above diagrams. Diagram (B) represents: A) the pure competition model. B) an industry in which there is allocative efficiency but not productive efficiency. C) the pure monopoly model. D) a long-run constant-cost industry. Answer: C

McConnell/Brue: Economics, 16/e

Page 757

Chapter 24: Pure Monopoly

Type: G Topic: 5 E: 447-448 MI: 203-204 134. Refer to the above diagrams. In diagram (B) the profit-maximizing quantity is: A) g and the profit-maximizing price is e. C) g and the profit-maximizing price is f. B) h and the profit-maximizing price is e. D) g and the profit-maximizing price is d. Answer: D

Type: G Topic: 5 E: 447-448 MI: 203-204 135. Refer to the above diagrams. With the industry structure represented by diagram: A) (A) there will be only a normal profit in the long run, while in (B) an economic profit can persist. B) (A) price exceeds marginal cost, resulting in allocative inefficiency. C) (B) price equals marginal cost, resulting in allocative efficiency. D) (B) equilibrium price and quantity will be e and h, respectively. Answer: A

Type: G Topic: 5 E: 447-448 MI: 203-204 136. Refer to the above diagrams. With the industry structure represented by diagram: A) (A) there will be allocative inefficiency. C) (B) output will be less than in diagram (A). B) (A) economic profit can persist in the long run. D) (B) output will be the same as in diagram (A). Answer: C

Type: G Topic: 5 E: 447-448 MI: 203-204 137. Refer to the above diagrams. The price will be _______ and the quantity will be _______ with the industry structure represented by diagram (B) compared to the one reprsented in (A). A) higher; higher. B) higher, lower. C) lower, lower. D) lower, higher. Answer: B

Type: D Topic: 5 E: 449-450 MI: 205-206 138. X-inefficiency refers to a situation in which a firm: A) is not as technologically progressive as it might be. B) encounters diseconomies of scale. C) fails to realize all existing economies of scale. D) fails to achieve the minimum average total costs attainable at each level of output. Answer: D

Type: A Topic: 5 E: 450 MI: 206 139. Which of the following is not a possible source of natural monopoly? A) large-scale network effects. C) greater use of specialized inputs. B) simultaneous consumption. D) rent-seeking behavior. Answer: D

Type: F Topic: 5 E: 450 MI: 206 140. There is some evidence to suggest that X-inefficiency is: A) absent whenever two or more producers are competing with one another. B) not encountered in either competitive or monopolistic firms. C) more likely to occur in monopolistic firms than in competitive firms. D) more likely to occur in competitive firms than in monopolistic firms. Answer: C

McConnell/Brue: Economics, 16/e

Page 758

Chapter 24: Pure Monopoly

Use the following to answer questions 141-143:

Type: G Topic: 5 E: 449-450 MI: 205-206 141. Refer to the above long-run cost diagram for a firm. If the firm produces output Q1 at an average total cost of ATC1, then the firm is: A) producing the potentially profit-maximizing output, but is failing to minimize production costs. B) incurring X-inefficiency, but is realizing all existing economies of scale. C) incurring X-inefficiency and is failing to realize all existing economies of scale. D) producing that output with the most efficient combination of inputs and is realizing all economies of scale. Answer: C

Type: G Topic: 5 E: 449-450 MI: 205-206 142. Refer to the above long-run cost diagram for a firm. If the firm produces output Q2 at an average cost of ATC 2, then the firm is: A) producing the potentially profit-maximizing output, but is failing to minimize production costs. B) incurring X-inefficiency, but is producing that output at which all existing economies of scale might be realized. C) incurring X-inefficiency and is failing to produce the output at which all economies of scale might be realized. D) producing that output with the most efficient combination of inputs and is realizing all existing economies of scale. Answer: B

Type: G Topic: 5 E: 449-450 MI: 205-206 143. Refer to the above long-run cost diagram for a firm. If the firm produces output Q2 at an average cost of ATC3, then the firm is: A) producing the potentially profit-maximizing output, but is failing to minimize production costs. B) incurring X-inefficiency, but is realizing all existing economies of scale. C) incurring X-inefficiency and is failing to realize all existing economies of scale. D) producing that output with the most efficient combination of inputs and is realizing all existing economies of scale. Answer: D

McConnell/Brue: Economics, 16/e

Page 759

Chapter 24: Pure Monopoly

Type: A Topic: 5 E: 450 MI: 206 144. In which one of the following market models is X-inefficiency most likely to be the greatest? A) pure competition B) oligopoly C) monopolistic competition D) pure monopoly Answer: D

Type: A Topic: 5 E: 450 MI: 206 145. In which one of the following market models is X-inefficiency least likely to be present? A) pure competition B) oligopoly C) monopolistic competition D) pure monopoly Answer: A

Price discrimination

Type: D Topic: 6 E: 451 MI: 207 146. Price discrimination refers to: A) selling a given product for different prices at two different points in time. B) any price above that which is equal to a minimum average total cost. C) the selling of a given product at different prices that do not reflect cost differences. D) the difference between the prices a purely competitive seller and a purely monopolistic seller would charge. Answer: C

Type: A Topic: 6 E: 452-453 MI: 208-209 147. If a monopolist engages in price discrimination, we can expect: A) profits to increase and output to fall. B) both profits and output to increase. C) both profits and output to decrease. D) the demand curve to lie below the marginal revenue curve. Answer: B

Type: A Topic: 6 E: 451-452 MI: 207-208 148. The practice of price discrimination is associated with pure monopoly because: A) it can be practiced whenever a firm's demand curve is downsloping. B) monopolists have considerable ability to control output and price. C) monopolists usually realize economies of scale. D) most monopolists sell differentiated products. Answer: B

Type: A Topic: 6 E: 452 MI: 208 149. Which of the following is not a precondition for price discrimination? A) The commodity involved must be a durable good. B) The good or service cannot be resold by original buyers. C) The seller must be able to segment the market, that is, to distinguish buyers with different elasticities of demand. D) The seller must possess some degree of monopoly power. Answer: A

McConnell/Brue: Economics, 16/e

Page 760

Chapter 24: Pure Monopoly

Type: A Topic: 6 E: 453 MI: 209 150. A perfectly discriminating pure monopolist will charge each buyer: A) different prices to compensate for differences in the characteristics of the product. B) the same price if per unit cost is constant for each unit of the product. C) that price which equals the buyer's marginal cost. D) the maximum price each would be willing to pay. Answer: D

Type: A Topic: 6 E: 453 MI: 209 151. Other things equal, in which of the following cases would economic profit be the greatest? A) an unregulated monopolist which is able to engage in price discrimination B) an unregulated monopolist C) a regulated monopolist charging a price equal to average total cost D) a regulated monopolist charging a price equal to marginal cost Answer: A

Type: A Topic: 6 E: 453 MI: 209 152. If a pure monopolist can engage in perfect price discrimination: A) the marginal revenue curve and the total revenue curve will now coincide. B) the marginal revenue curve will now shift to a position above the demand curve. C) the marginal revenue curve will now coincide with the demand curve. D) marginal revenue will become less at each level of output than it would be without price discrimination. Answer: C

Type: A Topic: 6 E: 453 MI: 209 153. If a monopolist engages in perfect price discrimination, it will: A) realize a smaller profit. B) charge a higher price where individual demand is inelastic and a lower price where individual demand is elastic. C) produce a smaller output than when it did not discriminate. D) charge a competitive price to all its customers. Answer: B

Type: A Topic: 6 E: 453 MI: 209 154. The vertical distance between the horizontal axis and any point on a perfectly discriminating monopolist's demand curve measures: A) the quantity demanded. C) product price and marginal revenue. B) total revenue. D) average revenue and average total cost. Answer: C

McConnell/Brue: Economics, 16/e

Page 761

Chapter 24: Pure Monopoly

Use the following to answer questions 155-160: Answer the next question(s) on the basis of the following information for a pure monopolist:
Output 0 1 2 3 4 5 Total cost $250 260 290 350 480 700 Product price $500 300 250 200 150 100

Type: T Topic: 6 E: 445-446 MI: 201-202 155. How many units would the above profit-maximizing monopolist produce? A) 1 B) 2 C) 3 D) 4 Answer: C

Type: T Topic: 6 E: 445-446 MI: 201-202 156. The above monopolist should set its price at: A) $300. B) $250. C) $200. D) $15. Answer: C

Type: T Topic: 6 E: 445-446 MI: 201-202 157. At its profit-maximizing output, the above monopolist: A) incurs a loss. C) earns a normal profit of $250. B) earns an economic profit of $250. D) earns an economic profit of $150. Answer: B

Type: T Topic: 6 E: 453 MI: 209 158. If the above monopolist could engage in perfect price discrimination, it would: A) not alter its level of output. C) decrease output by 1 unit. B) increase output by 1 unit. D) increase output by 2 units. Answer: B

Type: T Topic: 6 E: 453 MI: 209 159. If the above monopolist could engage in perfect price discrimination, its total revenue would be: A) $600. B) $1250. C) $800. D) $1400. Answer: D

Type: T Topic: 6 E: 453 MI: 209 160. If the above monopolist could engage in perfect price discrimination, its economic profit would be: A) $650. B) $920. C) $1000. D) $740. Answer: B

McConnell/Brue: Economics, 16/e

Page 762

Chapter 24: Pure Monopoly

Use the following to answer questions 161-163:

Type: G Topic: 6 E: 452-453 MI: 208-209 161. Assume the above figure applies to a perfectly discriminating pure monopolist, that is, to a monopolist who is able to extract from each buyer the maximum price that buyer is willing to pay. The profit-maximizing output: A) will be l. B) will be j. C) will be k. D) cannot be determined from the information given. Answer: C

Type: G Topic: 6 E: 452-453 MI: 208-209 162. Assume the above figure applies to a perfectly discriminating pure monopolist, that is, to a monopolist who is able to extract from each buyer the maximum price that buyer is willing to pay. Total revenue will be: A) 0efk. B) 0dfk. C) 0cgl. D) 0bhk. E) 0aij. Answer: A

Type: G Topic: 6 E: 452-453 MI: 208-209 163. Assume the above figure applies to a perfectly discriminating pure monopolist, that is, to a monopolist who is able to extract from each buyer the maximum price that buyer is willing to pay. The total economic profit for this monopolist: A) will be cdfg. B) will be bdfh. C) will be befh. D) cannot be determined from the information given. Answer: C

Type: A Topic: 6 E: 452-453 MI: 208-209 164. Other things equal, a perfectly discriminating monopolist will: A) realize a smaller economic profit than a nondiscriminating monopolist. B) produce a larger output than a nondiscriminating monopolist. C) produce the same output as a nondiscriminating monopolist. D) produce a smaller output than a nondiscriminating monopolist. Answer: B

McConnell/Brue: Economics, 16/e

Page 763

Chapter 24: Pure Monopoly

Regulated monopolies

Type: A Topic: 7 E: 454 MI: 210 165. If the long-run average total cost curve of an industry is declining at the point where it intersects the industry demand curve, we can expect: A) an overallocation of resources. B) the industry will be purely competitive. C) the industry will be monopolistically competitive. D) the industry will be a natural monopoly. Answer: D

Use the following to answer questions 166-169:

Type: A Topic: 7 E: 454 MI: 210 166. Refer to the above diagram for a pure monopolist. If the monopolist is unregulated, it will maximize profits by charging: A) a price above P3 and selling a quantity less than Q3. B) price P3 and producing output Q3. C) price P2 and producing output Q2. D) price P1 and producing output Q1. Answer: B

Type: G Topic: 7 E: 455 MI: 211 167. Refer to the above diagram for a pure monopolist. Suppose a regulatory commission is created to determine a legal price for the monopoly. If the commission seeks to provide the monopolist with a "fair return," it will set price at: A) P1. B) P3. C) P2. D) P4. Answer: A

Type: G Topic: 7 E: 454-455 MI: 210-211 168. Refer to the above diagram for a pure monopolist. If a regulatory commission seeks to achieve the most efficient allocation of resources to this line of production, it will set a price of: A) P1. B) P3. C) P2. D) P4. Answer: C

McConnell/Brue: Economics, 16/e

Page 764

Chapter 24: Pure Monopoly

Type: A Topic: 7 E: 454-455 MI: 210-211 169. Refer to the above diagram for a pure monopolist. If a regulatory commission sets price to achieve the most efficient allocation of resources, it will have to: A) tax the monopolist P3P1 per unit to prevent the monopolist from realizing an economic profit. B) subsidize the monopolist or the monopolist will go bankrupt in the long run. C) subsidize the monopolist P1P4 per unit to allow the monopolist to break even. D) tax the monopolist P1P2 per unit to prevent the monopolist from realizing an economic profit. Answer: B

Type: D Topic: 7 E: 455 MI: 211 170. The dilemma of regulation refers to the idea that: A) the regulated price which achieves allocative efficiency is also likely to result in persistent economic profits. B) the regulated price which results in a "fair return" restricts output by more than would unregulated monopoly. C) regulated pricing always conflicts with the "due process" provision of the Constitution. D) the regulated price which achieves allocative efficiency is also likely to result in losses. Answer: D

Type: A Topic: 7 E: 455 MI: 211 171. If a regulatory commission wants to provide a natural monopoly with a fair return, it should establish a price that is equal to: A) minimum average fixed cost. B) average total cost. C) marginal cost. D) marginal revenue. Answer: B

Type: A Topic: 7 E: 454-455 MI: 210-211 172. If a regulatory commission wants to establish a socially optimal price for a natural monopoly, it should select a price: A) at which the marginal cost curve intersects the demand curve. B) at which marginal revenue is zero. C) at which the average total cost curve intersects the demand curve. D) which corresponds with the equality of marginal cost and marginal revenue. Answer: A

Type: A Topic: 7 E: 455 MI: 211 173. Suppose for a regulated monopoly that price equals minimum ATC but price exceeds MC. This means that: A) both productive and allocative efficiency are being achieved. B) productive efficiency is being achieved, but not allocative efficiency. C) allocative efficiency is being achieved, but not productive efficiency. D) neither productive nor allocative efficiency is being achieved. Answer: B

McConnell/Brue: Economics, 16/e

Page 765

Chapter 24: Pure Monopoly

Type: A Topic: 7 E: 455 MI: 211 174. If a regulatory commission imposes upon a nondiscriminating natural monopoly a price that is equal to marginal cost and below average total cost at the resulting output, then: A) the firm will realize an economic profit. B) the firm will earn only a normal profit. C) allocative efficiency will be worsened. D) the firm must be subsidized or it will go bankrupt. Answer: D

Type: A Topic: 7 E: 454-455 MI: 210-211 175. If a regulatory commission forces a natural monopoly to charge a price equal to its marginal cost: A) the monopoly may incur a loss. C) output will decrease. B) resource allocation will be worsened. D) the firm will earn only a normal profit. Answer: A

Type: A Topic: 7 E: 454-455 MI: 210-211 176. If a regulatory commission forces a natural monopoly to charge a price equal to its average total cost: A) output will decrease. C) resource allocation will worsen. B) the monopolist will realize a normal profit. D) the firm will earn an economic profit. Answer: B

Use the following to answer questions 177-179:

Type: G Topic: 7 E: 454 MI: 210 177. Refer to the above diagram for a natural monopolist. If a regulatory commission were to set a maximum price of P3, the monopolist would: A) maximize profits. B) increase output beyond the profit-maximizing level. C) reduce output below the profit-maximizing level. D) be unable to make a normal profit. Answer: A

McConnell/Brue: Economics, 16/e

Page 766

Chapter 24: Pure Monopoly

Type: G Topic: 7 E: 454-455 MI: 210-211 178. Refer to the above diagram for a natural monopolist. If a regulatory commission set a maximum price of P2, the monopolist would: A) produce output Q1 and realize an economic profit. B) produce output Q3 and realize an economic profit. C) close down in the short run. D) produce output Q3 and realize a normal profit. Answer: D

Type: G Topic: 7 E: 454-455 MI: 210-211 179. Refer to the above diagram for a natural monopolist. If a regulatory commission set a maximum price of P1, the monopolist would produce output: A) Q2 and realize a normal profit. C) Q3 and realize an economic profit. B) Q4 and realize a normal profit. D) Q4 and realize a loss. Answer: D

Consider This Questions

Type: A E: 452 MI: 208 Status: New 180. (Consider This) Children are charged less than adults for admission to professional baseball games but are charged the same prices as adults at the concession stands. This pricing system occurs because: A) children have an elastic demand for game ticket but an inelastic demand for concession items. B) children have an inelastic demand for game tickets but an elastic demand for concession items. C) the seller can prevent children from buying game tickets for adults but cannot prevent children from buying concession items for adults. D) children can personally "consume" only a single game ticket, but can personally consume more than one concession item. Answer: C

Type: A E: 452 MI: 208 Status: New 181. (Consider This) Children are charged less than adults for admission to professional baseball games but are charged the same prices as adults at the concession stands. Which of the following conditions of price discrimination explain why this occurs? A) The seller must have some monopoly power; that is, it must be able to set the product price. B) The seller must be able to identify buyers by group characteristics such as age or income. C) Groups must have different elasticities of demand for the product. D) The items cannot be bought by people in the low-price group and transferred to members of the highprice group. Answer: D

Last Word Questions

Type: F E: 456 MI: 212 182. (Last Word) DeBeers Consolidated Mines markets about: A) 65 percent of the world's rough-cut diamonds. C) 50 percent of the world's rough-cut diamonds. B) 80 percent of the world's rough-cut diamonds. D) 33 percent of the world's rough-cut diamonds. Answer: A

McConnell/Brue: Economics, 16/e

Page 767

Chapter 24: Pure Monopoly

Type: F E: 456 MI: 212 183. (Last Word) Over a recently ended 66-year period, DeBeers: A) earned only a normal profit because of its high mining and marketing costs. B) operated substantially in accord with the predictions of the unregulated monopoly model. C) was subject to U.S. antimonopoly laws and therefore could not control diamond prices. D) was regulated by the South African government and thus had to limit prices to average total cost. Answer: B

Type: F E: 456 MI: 212 184. (Last Word) In a recent policy change, DeBeers has decided to: A) sell off its entire inventory of diamonds. B) abandon its policy of profit maximization. C) purchase the entire output of other mines and withhold diamonds from the market to bolster diamond prices. D) abandon its 66-year policy of trying to monopolize the sale of rough-cut diamonds. Answer: D

True/False Questions

Type: A E: 445 MI: 201 185. In the long run a pure monopolist must produce at that output where average total cost is at a minimum. Answer: False

Type: A E: 445 MI: 201 186. A pure monopolist will maximize profits by producing at that output where price and marginal cost are equal. Answer: False

Type: A E: 445 MI: 201 187. In the short run a pure monopolist will maximize profits by producing at that level of output where the difference between price and average total cost is at a maximum. Answer: False

Type: A E: 446 MI: 202 188. In the short run a pure monopolist will charge the highest price the market will bear for its product. Answer: False

Type: A E: 446-447 MI: 202-203 189. Pure monopolists always earn economic profits. Answer: False

Type: A E: 442 MI: 198 190. If the XYZ Company can sell 4 units per week at $10 per unit and 5 units per week at $9 per unit, the marginal revenue of the fifth unit is $5. Answer: True

McConnell/Brue: Economics, 16/e

Page 768

Chapter 24: Pure Monopoly

Type: A E: 447 MI: 203 191. Because of their large-scale level of production, pure monopolists overallocate resources to their industry by producing beyond the P = MC output. Answer: False

Type: A E: 442-443 MI: 198-199 192. Because of the ability to influence price, a pure monopolist can increase price and increase volume of sales simultaneously. Answer: False

Use the following to answer questions 193-200:

Type: A E: 444-445 MI: 200-201 193. Refer to the above diagram for a nondiscriminating monopolist. The profit-maximizing output for this firm is M . Answer: True

Type: G E: 445 MI: 201 194. Refer to the above diagram for a nondiscriminating monopolist. At the profit-maximizing output the firm's economic profit will be BAFG. Answer: True

Type: G E: 445 MI: 201 195. Refer to the above diagram for a nondiscriminating monopolist. At output R economic profits will be zero. Answer: True

Type: G E: 445 MI: 201 196. Refer to the above diagram for a nondiscriminating monopolist. At output Q production will be unprofitable. Answer: False

McConnell/Brue: Economics, 16/e

Page 769

Chapter 24: Pure Monopoly

Type: G E: 445 MI: 201 197. Refer to the above diagram for a nondiscriminating monopolist. The profit-maximizing price for this firm is J. Answer: False

Type: G E: 445 MI: 201 198. Refer to the above diagram for a nondiscriminating monopolist. At output M total variable cost will be 0CHM . Answer: False

Type: G E: 447-448 MI: 203-204 199. Refer to the above diagram for a nondiscriminating monopolist. From society's point of view it would be desirable to have the monopolist produce a larger output than M . Answer: True

Type: G E: 447-448 MI: 203-204 200. Refer to the above diagram for a nondiscriminating monopolist. At output Q average variable cost is QJ . Answer: False

Use the following to answer questions 201-206:

Type: G E: 443 MI: 199 201. Refer to the above diagrams. Both firms are selling their products in purely competitive markets. Answer: False

Type: G E: 416 MI: 172 202. Refer to the above diagrams. The demand for Firm A's product is perfectly elastic. Answer: True

Type: G E: 443 MI: 199 203. Refer to the above diagrams. The demand for Firm B's product is elastic at all prices in excess of $4. Answer: True

McConnell/Brue: Economics, 16/e

Page 770

Chapter 24: Pure Monopoly

Type: G E: 442-443 MI: 198-199 204. Refer to the above diagrams. Firm B's average revenue curve coincides with its marginal revenue curve. Answer: False

Type: G E: 443 MI: 199 205. Refer to the above diagrams. The demand for Firm B's product is inelastic at all prices below $4. Answer: True

Type: G E: 441-442 MI: 197-198 206. Refer to the above diagrams. If drawn, Firm A's average revenue curve would lie below its demand curve. Answer: False

Type: F E: 449 MI: 205 207. Natural monopoly may result where products produce substantial network effects and can be simultaneously consumed by a large number of consumers. Answer: True

Type: F E: 449 MI: 205 208. Extensive network effects may drive a market toward natural monopoly because consumers tend to choose a common, standard product that everyone else is using. Answer: True

McConnell/Brue: Economics, 16/e

Page 771

CHAPTER 25

Monopolistic Competition and Oligopoly

Topic 1. Monopolistic competition: definition; characteristics 2. Demand curve 3. Price-output behavior 4. Efficiency aspects 5. Oligopoly: definition; characteristics 6. Concentration ratio; Herfindahl Index 7. Game theory 8. Kinked-demand curve model 9. Collusion; cartels; price leadership 10. Advertising 11. Efficiency aspects 12. Review of four structures Consider This Last Word True-False Multiple Choice Questions Monopolistic competition: definition; characteristics

Question numbers 1-17 18-24 25-78 79-88 89-112 113-140 141-156 157-176 177-194 195-200 201-204 205-226 227-228 229-233 234-258

____________________________________________________________

_______________________________________

____________________________________________________________

_______________________________________

Type: D Topic: 1 E: 460 MI: 216 1. Monopolistic competition means: A) a market situation where competition is based entirely on product differentiation and advertising. B) a large number of firms producing a standardized or homogeneous product. C) many firms producing differentiated products. D) a few firms producing a standardized or homogeneous product. Answer: C

Type: A Topic: 1 E: 461 MI: 217 2. Monopolistic competition is characterized by a: A) few dominant firms and low entry barriers. B) large number of firms and substantial entry barriers. C) large number of firms and low entry barriers. D) few dominant firms and substantial entry barriers. Answer: C

Chapter 25: Monopolistic Competition and Oligopoly

Type: A Topic: 1 E: 461 MI: 217 3. Under monopolistic competition entry to the industry is: A) completely free of barriers. B) more difficult than under pure competition but not nearly as difficult as under pure monopoly. C) more difficult than under pure monopoly. D) blocked. Answer: B

Type: A Topic: 1 E: 461 MI: 217 4. Monopolistic competition resembles pure competition because: A) both industries emphasize nonprice competition. B) in both instances firms will operate at the minimum point on their long-run average total cost curves. C) both industries entail the production of differentiated products. D) barriers to entry are either weak or nonexistent. Answer: D

Type: A Topic: 1 E: 460-461 MI: 216-217 5. Which of the following is not a basic characteristic of monopolistic competition? A) the use of trademarks and brand names C) product differentiation B) recognized mutual interdependence D) a relatively large number of sellers Answer: B

Type: D Topic: 1 E: 461-462 MI: 217-218 6. Nonprice competition refers to: A) competition between products of different industries, for example, competition between aluminum and steel in the manufacture of automobile parts. B) price increases by a firm that are ignored by its rivals. C) advertising, product promotion, and changes in the real or perceived characteristics of a product. D) reductions in production costs that are not reflected in price reductions. Answer: C

Type: A Topic: 1 E: 462-463 MI: 218-219 7. Which of the following is not characteristic of monopolistic competition? A) relatively large numbers of sellers C) production at minimum ATC in the long-run B) product differentiation D) relatively easy entry to the industry Answer: C

Type: A Topic: 1 E: 462 MI: 218 8. The book publishing, furniture, and clothing industries are each illustrations of: A) countervailing power. C) monopolistic competition. B) homogeneous oligopoly. D) pure monopoly. Answer: C

McConnell/Brue: Economics, 16/e

Page 774

Chapter 25: Monopolistic Competition and Oligopoly

Type: A Topic: 1 E: 461 MI: 217 9. If the number of firms in a monopolistically competitive industry increases and the degree of product differentiation diminishes: A) the likelihood of realizing economic profits in the long run would be enhanced. B) individual firms would now be operating at outputs where their average total costs would be higher. C) the industry would more closely approximate pure competition. D) the likelihood of collusive pricing would increase. Answer: C

Type: A Topic: 1 E: 461 MI: 217 10. Economic analysis of a monopolistically competitive industry is more complicated than that of pure competition because: A) the number of firms in the industry is larger. B) monopolistically competitive firms cannot realize an economic profit in the long run. C) of product differentiation and consequent product promotion activities. D) monopolistically competitive producers use strategic pricing strategies to combat rivals. Answer: C

Type: A Topic: 1 E: 461 MI: 217 11. A monopolistically competitive industry combines elements of both competition and monopoly. The monopoly element results from: A) the likelihood of collusion. C) product differentiation. B) high entry barriers. D) mutual interdependence in decision making. Answer: C

Type: D Topic: 1 E: 462 MI: 218 12. Nonprice competition refers to: A) low barriers to entry. B) product development, advertising, and product packaging. C) the differences in information which consumers have regarding various products. D) an industry or firm in long-run equilibrium. Answer: B

Type: A Topic: 1 E: 461 MI: 217 13. A significant difference between a monopolistically competitive firm and a purely competitive firm is that the: A) former does not seek to maximize profits. B) latter recognizes that price must be reduced to sell more output. C) former sells similar, although not identical, products. D) former's demand curve is perfectly inelastic. Answer: C

Type: A Topic: 1 E: 460 MI: 216 14. A monopolistically competitive industry combines elements of both competition and monopoly. It is correct to say that the competitive element results from: A) a relatively large number of firms and the monopolistic element from product differentiation. B) product differentiation and the monopolistic element from high entry barriers. C) a perfectly elastic demand curve and the monopolistic element from low entry barriers. D) a highly inelastic demand curve and the monopolistic element from advertising and product promotion. Answer: A

McConnell/Brue: Economics, 16/e

Page 775

Chapter 25: Monopolistic Competition and Oligopoly

Type: A Topic: 1 E: 461 MI: 217 15. Monopolistically competitive and purely competitive industries are similar in that: A) both are assured of short-run economic profits. B) both produce differentiated products. C) the demand curves facing individual firms are perfectly elastic in both industries. D) there are few, if any, barriers to entry. Answer: D

Type: A Topic: 1 E: 462 MI: 218 16. The monopolistic competition model predicts that: A) allocative efficiency will be achieved. B) productive efficiency will be achieved. C) firms will engage in nonprice competition. D) firms will realize economic profits in the long run. Answer: C

Type: C Topic: 1 E: 460 MI: 216 17. Use your basic knowledge and your understanding of market structures to answer this question. Which of the following companies most closely approximates a monopolistic competitor? A) Subway Sandwiches B) Pittsburgh Plate Glass C) Ford Motor Company D) Microsoft. Answer: A

Demand curve

Type: D Topic: 2 E: 462 MI: 218 18. A monopolistically competitive firm has a: A) highly elastic demand curve. B) highly inelastic demand curve. Answer: A

C) perfectly inelastic demand curve. D) perfectly elastic demand curve.

Type: A Topic: 2 E: 462 MI: 218 19. The monopolistically competitive seller's demand curve will become more elastic the: A) more significant the barriers to entering the industry. B) greater the degree of product differentiation. C) larger the number of competitors. D) smaller the number of competitors. Answer: C

Type: A Topic: 2 E: 462 MI: 218 20. The larger the number of firms and the smaller the degree of product differentiation the: A) greater the divergence between the demand and the marginal revenue curves of the monopolistically competitive firm. B) larger will be the monopolistically competitive firm's fixed costs. C) less elastic is the monopolistically competitive firm's demand curve. D) more elastic is the monopolistically competitive firm's demand curve. Answer: D

McConnell/Brue: Economics, 16/e

Page 776

Chapter 25: Monopolistic Competition and Oligopoly

Type: A Topic: 2 E: 462 MI: 218 21. The demand curve of a monopolistically competitive producer is: A) less elastic than that of either a pure monopolist or a pure competitor. B) less elastic than that of a pure monopolist, but more elastic than that of a pure competitor. C) more elastic than that of a pure monopolist, but less elastic than that of a pure competitor. D) more elastic than that of either a pure monopolist or a pure competitor. Answer: C

Type: A Topic: 2 E: 462-463 MI: 218-219 22. A monopolistically competitive firm's marginal revenue curve: A) is downsloping and coincides with the demand curve. B) coincides with the demand curve and is parallel to the horizontal axis. C) is downsloping and lies below the demand curve. D) does not exist because the firm is a "price maker." Answer: C

Type: A Topic: 2 E: 462-463 MI: 218-219 23. In comparing the demand curve of a pure monopolist with that of a monopolistically competitive firm, we would expect the monopolistic competitor to have a: A) perfectly elastic demand curve and the monopolist to have a perfectly inelastic demand curve. B) generally more elastic demand curve. C) generally less elastic demand curve. D) demand curve whose elasticity coefficient is 1 at all possible prices. Answer: B

Type: A Topic: 2 E: 462 MI: 218 24. The price elasticity of a monopolistically competitive firm's demand curve varies: A) inversely with the number of competitors and the degree of product differentiation. B) directly with the number of competitors and the degree of product differentiation. C) directly with the number of competitors, but inversely with the degree of product differentiation. D) inversely with the number of competitors, but directly with the degree of product differentiation. Answer: C

Price-output behavior

Type: A Topic: 3 E: 464 MI: 220 Status: New 25. In short-run equilibrium, a monopolistically competitive firm sets it price: A) equal to marginal revenue. C) above marginal cost. B) equal to marginal cost. D) below marginal cost. Answer: C

Type: A Topic: 3 E: 464 MI: 220 Status: New 26. In long-run equilibrium, a monopolistically competitive firm sets it price: A) above marginal cost. C) equal to marginal revenue. B) below marginal cost. D) equal to marginal cost. Answer: A

McConnell/Brue: Economics, 16/e

Page 777

Chapter 25: Monopolistic Competition and Oligopoly

Type: A Topic: 3 E: 464 MI: 220 Status: New 27. In short-run equilibrium, the price charged by the monopolistically competitive firm: A) must be less than ATC. B) must be more than ATC. C) may be either equal to ATC, less than ATC, or more than ATC. D) must be equal to ATC. Answer: C

Type: A Topic: 3 E: 464 MI: 220 Status: New 28. In long-run equilibrium, the price charged by the monopolistically competitive firm: A) must be less than ATC. B) must be more than ATC. C) may be either equal to ATC, less than ATC, or more than ATC. D) will be equal to ATC. Answer: D

Type: A Topic: 3 E: 464 MI: 220 29. Monopolistically competitive firms: A) realize normal profits in the short run but losses in the long run. B) incur persistent losses in both the short run and long run. C) may realize either profits or losses in the short run, but realize normal profits in the long run. D) persistently realize economic profits in both the short run and long run. Answer: C

Type: A Topic: 3 E: 464 MI: 220 30. The monopolistically competitive seller maximizes profit by producing at the point where: A) total revenue is at a maximum. C) marginal revenue equals marginal cost. B) average costs are at a minimum. D) price equals marginal revenue. Answer: C

Type: A Topic: 3 E: 464 MI: 220 31. In long-run equilibrium a monopolistically competitive firm's price will: A) be less than both MC and ATC. C) exceed MC, but equal ATC. B) exceed ATC, but equal MC. D) exceed both MC and ATC. Answer: C

Type: A Topic: 3 E: 465 MI: 221 32. Which of the following is correct for a monopolistically competitive firm in long-run equilibrium? A) MC = ATC B) MC exceeds MR C) P exceeds minimum ATC D) P = MC Answer: C

Type: A Topic: 3 E: 465-466 MI: 221-222 33. In long-run equilibrium a monopolistically competitive firm will: A) earn an economic profit. C) equate price and marginal cost. B) realize all economies of scale. D) have excess production capacity. Answer: D

McConnell/Brue: Economics, 16/e

Page 778

Chapter 25: Monopolistic Competition and Oligopoly

Type: D Topic: 3 E: 465-466 MI: 221-222 34. Excess capacity refers to the: A) amount by which actual production falls short of the minimum ATC output. B) fact that entry barriers artificially reduce the number of firms in an industry. C) differential between price and marginal costs which characterizes monopolistically competitive firms. D) fact that most monopolistically competitive firms encounter diseconomies of scale. Answer: A

Use the following to answer questions 35-38:

Type: G Topic: 3 E: 463-464 MI: 219-220 35. Refer to the above diagram for a monopolistically competitive firm in short-run equilibrium. This firm's profit-maximizing price will be: A) $10. B) $13. C) $16. D) $19. Answer: C

Type: G Topic: 3 E: 463-464 MI: 219-220 36. Refer to the above diagram for a monopolistically competitive firm in short-run equilibrium. The profitmaximizing output for this firm will be: A) 210. B) 180. C) 160. D) 100. Answer: C

Type: G Topic: 3 E: 463-464 MI: 219-220 37. Refer to the above diagram for a monopolistically competitive firm in short-run equilibrium. This firm will realize an economic: A) loss of $320. B) loss of $280. C) profit of $480. D) profit of $600. E) profit of $360. Answer: C

Type: G Topic: 3 E: 464 MI: 220 38. Refer to the above diagram for a monopolistically competitive firm in short-run equilibrium. Assume the firm is part of an increasing-cost industry. In the long run firms will: A) leave this industry, causing both demand and the ATC curve to shift upward. B) enter this industry, causing demand to rise and the ATC curve to shift downward. C) enter this industry, causing demand to fall and the ATC curve to shift upward. D) enter this industry, causing both demand and the ATC curve to shift upward. Answer: C

McConnell/Brue: Economics, 16/e

Page 779

Chapter 25: Monopolistic Competition and Oligopoly

Type: A Topic: 3 E: 464 MI: 220 39. In the short run a monopolistically competitive firm's economic profit: A) will be maximized where price equals average total cost. B) may be positive, zero, or negative. C) are always positive. D) will always be zero. Answer: B

Use the following to answer questions 40-42:

Type: G Topic: 3 E: 463 MI: 219 Status: New 40. In short-run equilibrium, the monopolistically competitive firm shown above will set its price: A) below ATC. B) above ATC. C) below MC. D) below MR. Answer: A

Type: G Topic: 3 E: 463-464 MI: 219-220 41. The monopolistically competitive firm shown in the above figure: A) is in long-run equilibrium. B) might realize an economic profit or a loss, depending on its choice of output level. C) cannot operate profitably, at least in the short run. D) can realize an economic profit. Answer: C

Type: G Topic: 3 E: 464 MI: 220 Status: New 42. If all monopolistically competitive firms in the industry have profit circumstances similar to the firm shown above: A) new firms will enter the industry. C) all firms will exit the industry. B) some firms will exit the industry. D) no firms will exit the industry. Answer: B

McConnell/Brue: Economics, 16/e

Page 780

Chapter 25: Monopolistic Competition and Oligopoly

Use the following to answer questions 43-45:

Type: G Topic: 3 E: 463 MI: 219 Status: New 43. In short-run equilibrium, the monopolistically competitive firm shown in the above figure will set its price: A) below ATC. B) above ATC. C) below MC. D) below MR. Answer: B

Type: G Topic: 3 E: 463-464 MI: 219-220 44. The monopolistically competitive firm shown in the above figure: A) will realize allocative efficiency at its profit-maximizing output. B) cannot operate at a loss. C) is in long-run equilibrium. D) is realizing an economic profit. Answer: D

Type: G Topic: 3 E: 464 MI: 220 Status: New 45. If all monopolistically competitive firms in the industry have profit circumstances similar to the firm shown above: A) new firms will enter the industry. C) all firms will exit the industry. B) some firms will exit the industry. D) no firms will enter the industry. Answer: A

McConnell/Brue: Economics, 16/e

Page 781

Chapter 25: Monopolistic Competition and Oligopoly

Use the following to answer questions 46-48:

Type: G Topic: 3 E: 463 MI: 219 46. Refer to the above diagrams, which pertain to monopolistically competitive firms. Short-run equilibrium entailing economic loss is shown by: A) diagram a only. B) diagram b only. C) diagram c only. D) both diagrams a and c. Answer: C

Type: G Topic: 3 E: 463 MI: 219 47. Refer to the above diagrams, which pertain to monopolistically competitive firms. A short-run equilibrium entailing economic profits is shown by: A) diagram a only. B) diagram b only. C) diagram c only. D) both diagrams b and c. Answer: B

Type: G Topic: 3 E: 463 MI: 219 48. Refer to the above diagrams, which pertain to monopolistically competitive firms. Long-run equilibrium is shown by: A) diagram a only. B) diagram b only. C) diagram c only. D) both diagrams b and c. Answer: A

Type: A Topic: 3 E: 464 MI: 220 49. Which of the following is not characteristic of long-run equilibrium under monopolistic competition? A) price equals minimum average total cost C) price is equal to average total cost B) marginal cost equals marginal revenue D) price exceeds marginal cost Answer: A

McConnell/Brue: Economics, 16/e

Page 782

Chapter 25: Monopolistic Competition and Oligopoly

Use the following to answer questions 50-52:

Type: G Topic: 3 E: 463 MI: 219 50. Refer to the above diagram for a monopolistically competitive firm. Long-run equilibrium price will be: A) above A. B) EF. C) A. D) B. Answer: C

Type: G Topic: 3 E: 463 MI: 219 51. Refer to the above diagram for a monopolistically competitive firm. Long-run equilibrium output will be: A) greater than E. B) E. C) D. D) C. Answer: C

Type: G Topic: 3 E: 464 MI: 220 52. Refer to the above diagram for a monopolistically competitive firm. If more firms would enter the industry and product differentiation would weaken: A) resource misallocation would become more severe. B) the demand curve would become more elastic. C) equilibrium output would decline and equilibrium price would rise. D) equilibrium output would decline and equilibrium price would fall. Answer: B

Type: A Topic: 3 E: 464 MI: 220 53. Long-run equilibrium for a monopolistically competitive firm where economic profits are zero results from: A) rising marginal costs. C) relatively easy entry. B) a perfectly elastic product demand curve. D) product differentiation and development. Answer: C

McConnell/Brue: Economics, 16/e

Page 783

Chapter 25: Monopolistic Competition and Oligopoly

Use the following to answer questions 54-55:

Type: G Topic: 3 E: 464 MI: 220 54. In long-run equilibrium, the firm shown in the diagram above will: A) earn a normal profit. B) go bankrupt. C) incur a loss. D) realize an economic profit. Answer: A

Type: G Topic: 3 E: 465 MI: 221 55. In long-run equilibrium, production for the firm shown in the diagram above is: A) greater than would occur under pure competition. B) less efficient than in a purely competitive market. C) more efficient than in a purely competitive market. D) optimally efficient. Answer: B

Type: A Topic: 3 E: 463-464 MI: 219-220 56. When a monopolistically competitive firm is in long-run equilibrium: A) production takes place where ATC is minimized. B) marginal revenue equals marginal cost and price equals average total cost. C) normal profit is zero and price equals marginal cost. D) economic profit is zero and price equals marginal cost. Answer: B

Type: A Topic: 3 E: 464 MI: 220 57. In the long run, new firms will enter a monopolistically competitive industry: A) provided economies of scale are being realized. C) until minimum average total cost is achieved. B) even though losses are incurred in the short run. D) until economic profits are zero. Answer: D

Type: A Topic: 3 E: 464 MI: 220 58. If some firms leave a monopolistically competitive industry, the demand curves of the remaining firms will: A) be unaffected. B) shift to the left. C) become more elastic. D) shift to the right. Answer: D

McConnell/Brue: Economics, 16/e

Page 784

Chapter 25: Monopolistic Competition and Oligopoly

Type: A Topic: 3 E: 463-464 MI: 219-220 59. When a monopolistically competitive firm is in long-run equilibrium: A) P = MC = ATC. C) MR > MC and P = minimum ATC. B) MR = MC and minimum ATC > P. D) MR = MC and P > minimum ATC. Answer: D

Type: A Topic: 3 E: 464 MI: 220 60. Other things equal, if more firms enter a monopolistically competitive industry: A) the demand curves facing existing firms would shift to the right. B) the demand curves facing existing firms would shift to the left. C) the demand curves facing existing firms would become less elastic. D) losses would necessarily occur. Answer: B

Type: A Topic: 3 E: 464 MI: 220 61. Which of the following statements is correct? A) Purely competitive firms, monopolistically competitive firms, and pure monopolies all earn zero economic profits in the long run. B) Purely competitive firms, monopolistically competitive firms, and pure monopolies all earn positive economic profits in the long run. C) In the long run purely competitive firms and monopolistically competitive firms earn zero economic profits, while pure monopolies may or may not earn economic profits. D) Monopolistically competitive firms earn zero economic profits in both the short run and the long run. Answer: C

Type: A Topic: 3 E: 463 MI: 219 62. For a monopolistically competitive firm in long-run equilibrium: A) price will equal marginal cost. C) marginal revenue will exceed marginal cost. B) price will equal average total cost. D) economic profits will be some positive amount. Answer: B

Type: A Topic: 3 E: 463-464 MI: 219-220 63. In long-run equilibrium both purely competitive and monopolistically competitive firms will: A) produce at minimum average total cost. C) achieve allocative efficiency. B) earn economic profits. D) equate marginal cost and marginal revenue. Answer: D

Type: A Topic: 3 E: 465-466 MI: 221-222 64. In long-run equilibrium monopolistic competition entails: A) an efficient allocation of resources. B) an overallocation of resources. C) an underallocation of resources. D) production at the minimum attainable average total cost. Answer: C

McConnell/Brue: Economics, 16/e

Page 785

Chapter 25: Monopolistic Competition and Oligopoly

Type: A Topic: 3 E: 464 MI: 220 65. Which of the following statements concerning a monopolistically competitive industry is correct? A) If there are short-run losses, firms will leave the industry and the demand curves of the remaining firms will shift to the right. B) If there are short-run economic profits, firms will enter the industry and the demand curves of existing firms will shift to the right. C) If there are short-run losses, firms will leave the industry and the demand curves of the remaining firms will shift to the left. D) If there are short-run economic profits, firms will leave the industry and the demand curves of the remaining firms will shift to the right. Answer: A

Use the following to answer questions 66-71: Answer the next question(s) on the basis of the following demand and cost data for a specific firm:
(1) Price $11 9.99 9 8 7.10 6 5.15 Demand Data (2) Price $10 8.85 8 7 6.10 5 4.15 (3) Quantity 6 7 8 9 10 11 12 Cost Data Total Output cost 6 $61 7 62 8 64 9 67 10 72 11 79 12 86

Type: T Topic: 3 E: 464 MI: 220 66. If columns (1) and (3) of the demand data shown above are this firm's demand schedule, the profitmaximizing level of output will be: A) 12 units. B) 8 units. C) 10 units. D) 9 units. Answer: B

Type: T Topic: 3 E: 464 MI: 220 67. If columns (1) and (3) of the demand data shown above are this firm's demand schedule, the profitmaximizing price will be: A) $9. B) $7. C) $11. D) $6. Answer: A

Type: T Topic: 3 E: 464 MI: 220 68. If columns (1) and (3) of the demand data shown above are this firm's demand schedule, economic profit will be: A) $10. B) $19. C) $6. D) $8. Answer: D

Type: T Topic: 3 E: 464 MI: 220 69. Suppose that entry into the industry changes this firm's demand schedule from columns (1) and (3) shown above to columns (2) and (3). Economic profit will: A) fall by $10. B) fall to $6. C) increase by $10. D) decline to zero. Answer: D

McConnell/Brue: Economics, 16/e

Page 786

Chapter 25: Monopolistic Competition and Oligopoly

Type: T Topic: 3 E: 464 MI: 220 70. Suppose that entry into this industry changes this firm's demand schedule from columns (1) and (3) shown above to columns (2) and (3). We can conclude that this industry is: A) a pure monopoly. C) a constant cost industry. B) purely competitive. D) monopolistically competitive. Answer: D

Type: T Topic: 3 E: 464 MI: 220 71. With the demand schedule shown above by columns (2) and (3), in long-run equilibrium: A) price will equal average total cost. C) marginal cost will exceed price. B) total cost will exceed total revenue. D) price will equal marginal revenue. Answer: A

Type: A Topic: 3 E: 463-464 MI: 219-220 72. An important similarity between a monopolistically competitive firm and a purely competitive firm is that: A) both face perfectly elastic demand schedules. C) both realize productive efficiency. B) economic profit tends toward zero for both. D) both realize allocative efficiency. Answer: B

Type: A Topic: 3 E: 462 MI: 218 73. An important similarity between a monopolistically competitive firm and a pure monopolist is that both: A) realize an economic profit in the long run. B) achieve allocative efficiency. C) face demand curves which are less than perfectly elastic. D) achieve productive efficiency. Answer: C

Type: A Topic: 3 E: 462, 464 MI: 218, 220 74. The less elastic a monopolistic competitor's long-run demand curve, the: A) less its excess capacity. B) higher its price relative to that of a pure competitor having the same cost curves. C) higher its long-run profits. D) lower its average total cost at its equilibrium level of output. Answer: B

McConnell/Brue: Economics, 16/e

Page 787

Chapter 25: Monopolistic Competition and Oligopoly

Use the following to answer questions 75-77:

Type: G Topic: 3 E: 463-464 MI: 219-220 75. Refer to the above diagram for a monopolistically competitive producer. The firm is: A) minimizing losses in the long run. C) realizing a normal profit in the long run. B) minimizing losses in the short run. D) about to leave the industry. Answer: C

Type: G Topic: 3 E: 465-466 MI: 221-222 76. Refer to the above diagram for a monopolistically competitive producer. This firm is experiencing: A) a shortage of production capacity. C) excess capacity of DE. B) excess capacity of CD. D) diseconomies of scale. Answer: C

Type: G Topic: 3 E: 465 MI: 221 77. Refer to the above diagram for a monopolistically competitive producer. If this firm were to realize productive efficiency, it would: A) also realize an economic profit. C) also achieve allocative efficiency. B) incur a loss. D) have to produce a smaller output. Answer: B

Type: A Topic: 3 E: 463-464 MI: 219-220 78. In the long run a monopolistically competitive firm: A) earns an economic profit. C) produces where MR exceeds MC. B) produces where P = ATC. D) achieves allocative efficiency. Answer: B

Efficiency aspects

Type: D Topic: 4 E: 465-466 MI: 221-222 79. Monopolistically competitive industries are inefficient because: A) they realize diseconomies of scale. B) advertising costs retard technological advance and product development. C) monopolistically competitive industries are overpopulated with firms whose plants are underutilized. D) monopolistically competitive sellers engage in misleading advertising. Answer: C

McConnell/Brue: Economics, 16/e

Page 788

Chapter 25: Monopolistic Competition and Oligopoly

Type: A Topic: 4 E: 466 MI: 222 80. The economic inefficiencies of monopolistic competition may be offset by the fact that: A) advertising expenditures shift the average cost curve upward. B) available capacity is fully utilized. C) resources are optimally allocated to the production of the product. D) consumers have a number of variations of the product from which to choose. Answer: D

Type: A Topic: 4 E: 465-466 MI: 221-222 81. Inefficiencies occur under monopolistic competition because: A) each firm's demand curve becomes more elastic as we move down the curve. B) each firm's marginal revenue curve coincides with its demand curve. C) each firm's downsloping demand curve is tangent to the ATC curve in the long run. D) entry barriers greatly restrict the entry of new firms. Answer: C

Type: F Topic: 4 E: 466 MI: 222 82. A significant benefit of monopolistic competition compared with pure competition is: A) less likelihood of X-inefficiency. B) improved resource allocation. C) greater product variety. D) stronger incentives to achieve economies of scale. Answer: C

Type: A Topic: 4 E: 466 MI: 222 83. Product variety is likely to be greater in: A) monopolistic competition than in pure competition. B) pure competition than in monopolistic competition. C) homogenous oligopoly than in monopolistic competition. D) homogenous oligopoly than in differentiated oligopoly. Answer: A

Type: A Topic: 4 E: 466 MI: 222 84. Which of the following is correct? A) The excess capacity problem diminishes as the monopolistically competitive firm's demand curve becomes less elastic. B) The excess capacity problem means that monopolistically competitive firms typically produce at some point on the rising segment of their average total cost curve. C) The greater the degree of product variation, the lesser is the excess capacity problem. D) The greater the degree of product variation, the greater is the excess capacity problem. Answer: D

Type: A Topic: 4 E: 465 MI: 221 85. In monopolistically competitive markets resources are: A) overallocated because long-run equilibrium occurs where price exceeds marginal cost. B) underallocated because long-run equilibrium occurs where price exceeds marginal cost. C) overallocated because long-run equilibrium occurs where marginal cost exceeds price. D) underallocated because long-run equilibrium occurs where marginal cost exceeds price. Answer: B

McConnell/Brue: Economics, 16/e

Page 789

Chapter 25: Monopolistic Competition and Oligopoly

Type: A Topic: 4 E: 465 MI: 221 86. In long-run equilibrium a monopolistically competitive producer achieves: A) neither productive efficiency nor allocative efficiency. B) both productive efficiency and allocative efficiency. C) productive efficiency, but not allocative efficiency. D) allocative efficiency, but not productive efficiency. Answer: A

Type: A Topic: 4 E: 465-466 MI: 221-222 87. The less elastic a monopolistic competitor's long-run demand curve, the: A) greater its excess capacity. B) lower its price relative to that of a pure competitor having the same cost curves. C) higher its long-run economic profit. D) lower its average total cost at its equilibrium level of output. Answer: A

Type: A Topic: 4 E: 465 MI: 221 88. The more elastic a monopolistic competitor's long-run demand curve, the: A) greater its excess capacity. B) the higher its price relative to that of a pure competitor having the same cost curves. C) lower its long-run profit. D) lower its average total cost at its equilibrium level of output. Answer: D

Oligopoly: definition; characteristics

Type: A Topic: 5 E: 467 MI: 223 Status: New 89. In which of these continuums of degrees of competition (highest to lowest) is oligopoly properly placed? A) pure competition, oligopoly, pure monopoly, monopolistic competition B) oligopoly, pure competition, monopolistic competition, pure monopoly C) monopolistic competition, pure competition, pure monopoly, oligopoly D) pure competition, monopolistic competition, oligopoly, pure monopoly Answer: D

Type: D Topic: 5 E: 467 MI: 223 90. The term oligopoly indicates: A) a one-firm industry. B) many producers of a differentiated product. C) a few firms producing either a differentiated or a homogeneous product. D) an industry whose four-firm concentration ratio is low. Answer: C

Type: A Topic: 5 E: 467 MI: 223 91. In an oligopolistic market: A) one firm is always dominant. B) products may be standardized or differentiated. C) the four largest firms account for 20 percent or less of total sales. D) the industry is monopolistically competitive. Answer: B

McConnell/Brue: Economics, 16/e

Page 790

Chapter 25: Monopolistic Competition and Oligopoly

Type: D Topic: 5 E: 467-468 MI: 223-224 92. Oligopolistic industries are characterized by: A) a few dominant firms and substantial entry barriers. B) a few dominant firms and no barriers to entry. C) a large number of firms and low entry barriers. D) a few dominant firms and low entry barriers. Answer: A

Type: A Topic: 5 E: 467 MI: 223 93. The automobile, household appliance, and automobile tire industries are all illustrations of: A) homogeneous oligopoly. C) pure monopoly. B) monopolistic competition. D) differentiated oligopoly. Answer: D

Type: C Topic: 5 E: 467 MI: 223 94. Use your basic knowledge and your understanding of market structures to answer this question. Which of the following companies most closely approximates a differentiated oligopolist in a highly concentrated industry? A) Subway Sandwiches B) Pittsburgh Plate Glass C) Ford Motor Company D) Kaiser Aluminum. Answer: C

Type: C Topic: 5 E: 467 MI: 223 95. Use your basic knowledge and your understanding of market structures to answer this question. Which of the following companies most closely approximates a homogenous oligopolist in a highly concentrated industry? A) Kellogg B) Pittsburgh Plate Glass C) Ford Motor Company D) Starbucks Coffee. Answer: B

Type: A Topic: 5 E: 467 MI: 223 96. The mutual interdependence that characterizes oligopoly arises because: A) the products of various firms are homogeneous. B) the products of various firms are differentiated. C) a small number of firms produce a large proportion of industry output. D) the demand curves of firms are kinked at the prevailing price. Answer: C

Type: A Topic: 5 E: 467 MI: 223 97. Barriers to entry in oligopolistic industries may consist of: A) diseconomies of scale. C) ownership of essential resources. B) diminishing returns. D) patent expirations. Answer: C

Type: A Topic: 5 E: 467 MI: 223 98. The copper, aluminum, cement, and industrial alcohol industries are examples of: A) interproduct competition. C) monopolistic competition. B) homogeneous oligopoly. D) differentiated oligopoly. Answer: B

McConnell/Brue: Economics, 16/e

Page 791

Chapter 25: Monopolistic Competition and Oligopoly

Type: A Topic: 5 E: 467 MI: 223 99. Which of the following is the best example of oligopoly? A) women's dress manufacturing B) automobile manufacturing Answer: B

C) restaurants

D) cotton farming

Type: A Topic: 5 E: 467 MI: 223 100. If there are significant economies of scale in an industry, then: A) a firm that is large may be able to produce at a lower unit cost than can a small firm. B) a firm that is large will have to charge a higher price than will a small firm. C) entry to that industry will be easy. D) firms must differentiate their products to earn economic profits. Answer: A

Type: A Topic: 5 E: 472 MI: 228 101. In which of the following market models do demand and marginal revenue diverge? A) pure monopoly, oligopoly, and monopolistic competition B) pure monopoly, oligopoly, and pure competition C) pure monopoly only D) oligopoly only Answer: A

Type: A Topic: 5 E: 471 MI: 227 102. Oligopoly is difficult to analyze primarily because: A) the number of firms is too large to make collusion understandable. B) the price and output decisions of any one firm depend on the reactions of its rivals. C) output may be either homogenous or differentiated. D) neither allocative nor productive efficiency is achieved. Answer: B

Type: A Topic: 5 E: 471 MI: 227 103. Oligopoly is more difficult to analyze than other market models because: A) the number of firms is so large that market behavior cannot be accurately predicted. B) the marginal cost and marginal revenue curves of an oligopolist play no part in the determination of equilibrium price and quantity. C) of mutual interdependence and the fact that oligopoly outcomes are less certain than in other market models. D) unlike the firms of other market models, it cannot be assumed that oligopolists are profit maximizers. Answer: C

Type: A Topic: 5 E: 467 MI: 223 104. Which of the following is an illustration of differentiated oligopoly? A) the aluminum industry C) the soft drink industry B) the steel industry D) retail stores in large cities Answer: C

Type: A Topic: 5 E: 467 MI: 223 105. Which of the following industries is an illustration of homogeneous oligopoly? A) household laundry products B) personal computers C) aluminum D) the auto industry Answer: C

McConnell/Brue: Economics, 16/e

Page 792

Chapter 25: Monopolistic Competition and Oligopoly

Type: D Topic: 5 E: 467 MI: 223 106. Differentiated oligopoly exists where a small number of firms are: A) producing goods that differ in terms of quality and design. B) setting price and output collusively. C) setting price and output independently. D) producing virtually identical products. Answer: A

Type: D Topic: 5 E: 467 MI: 223 107. Homogeneous oligopoly exists where a small number of firms are: A) producing virtually identical products. C) setting price and output collusively. B) setting price and output independently. D) producing differentiated products. Answer: A

Type: D Topic: 5 E: 467 MI: 223 108. Oligopolistic industries: A) are characterized by a relatively large number of small sellers. B) may produce either standardized or differentiated products. C) always produce differentiated products. D) always produce standardized products. Answer: B

Type: A Topic: 5 E: 467 MI: 223 109. Which of the following is a unique feature of oligopoly? A) mutual interdependence C) product differentiation B) advertising expenditures D) nonprice competition Answer: A

Type: A Topic: 5 E: 467 MI: 223 110. Prices are likely to be least flexible: A) in oligopoly. B) in monopolistic competition. Answer: A

C) where product demand is inelastic. D) in pure competition.

Type: D Topic: 5 E: 467 MI: 223 111. Mutual interdependence means that each oligopolistic firm: A) faces a perfectly elastic demand for its product. B) must consider the reactions of its rivals when it determines its price policy. C) produces a product identical to those of its rivals. D) produces a product similar but not identical to the products of its rivals. Answer: B

Type: A Topic: 5 E: 467 MI: 223 112. Clear-cut mutual interdependence with respect to the price-output policies exists in: A) pure monopoly B) oligopoly C) monopolistic competition D) pure competition Answer: B

McConnell/Brue: Economics, 16/e

Page 793

Chapter 25: Monopolistic Competition and Oligopoly

Concentration ratio; Herfindahl Index

Type: D Topic: 6 E: 468 MI: 224 113. Concentration ratios measure the: A) geographic location of the largest corporations in each industry. B) degree to which product price exceeds marginal cost in various industries. C) percentage of total sales accounted for by the four largest firms in the industry. D) number of firms in an industry. Answer: C

Type: A Topic: 6 E: 468 MI: 224 114. If the four-firm concentration ratio for industry X is 80: A) the four largest firms account for 80 percent of total sales. B) each of the four largest firms accounts for 20 percent of total sales. C) the four largest firms account for 20 percent of total sales. D) the industry is monopolistically competitive. Answer: A

Type: A Topic: 6 E: 468 MI: 224 115. An industry having a four-firm concentration ratio of 85 percent: A) approximates pure competition. C) is a pure monopoly. B) is monopolistically competitive. D) is an oligopoly. Answer: D

Type: A Topic: 6 E: 468 MI: 224 116. As a general rule, oligopoly exists when the four-firm concentration ratio: A) exceeds the Herfindahl index. C) is 40 percent or more. B) is less than the Herfindahl index. D) is 15 percent or more. Answer: C

Type: A Topic: 6 E: 469 MI: 225 117. Aluminum competes with copper in the market for power transmission lines. This illustrates: A) mutual interdependence. C) interindustry competition. B) differentiated oligopoly. D) homogeneous oligopoly. Answer: C

Type: A Topic: 6 E: 469 MI: 225 118. The Herfindahl index for a pure monopolist is: A) 100. B) 10,000. C) 100,000. D) 10. Answer: B

McConnell/Brue: Economics, 16/e

Page 794

Chapter 25: Monopolistic Competition and Oligopoly

Type: A Topic: 6 E: 469 MI: 225 119. Industries X and Y both have four-firm concentration ratios of 65 percent, but the Herfindahl index for X is 1,500 while that for Y is 2,000. These data suggest: A) greater market power in X than in Y. B) greater market power in Y than in X. C) that X is more technologically progressive than Y. D) that price competition is stronger in Y than in X. Answer: B

Type: A Topic: 6 E: 468 MI: 224 120. Suppose that total sales in an industry in a particular year are $600 million and sales by the top four sellers are $200 million, $150 million, $100 million, and $50 million, respectively. We can conclude that: A) price leadership exists in this industry. B) the concentration ratio is more than 80 percent. C) this industry is a differentiated oligopoly. D) the firms in this industry face a kinked demand curve. Answer: B

Type: A Topic: 6 E: 468 MI: 224 121. The four-firm sales concentration ratio for an industry measures the: A) geographic concentration of firms. B) extent to which the four largest firms dominate the production of a good. C) percentage of the industry's capital facilities owned by the four largest firms. D) degree of X-inefficiency in the industry. Answer: B

Type: A Topic: 6 E: 468 MI: 224 122. Concentration ratios: A) may overstate the degree of competition because they ignore imported products. B) may overstate the degree of competition because interindustry competition is ignored. C) may understate the degree of competition because they ignore imported products. D) provide detailed insights as to the price and output behavior of firms which comprise the various industries. Answer: C

Type: A Topic: 6 E: 468-469 MI: 224-225 123. If a product such as cement or bricks is costly to ship and, therefore, markets are very localized, the national concentration ratio for that industry: A) will be greater than 50 percent. B) may understate the degree of monopoly. C) may overstate the degree of monopoly. D) will yield an accurate impression of the degree of monopoly. Answer: B

McConnell/Brue: Economics, 16/e

Page 795

Chapter 25: Monopolistic Competition and Oligopoly

Type: A Topic: 6 E: 469 MI: 225 124. Concentration ratios may be inaccurate indicators of the degree of monopoly power in an industry because: A) they include interindustry competition. B) foreign competition is not considered. C) they are only calculated for local and regional markets. D) they do not distinguish between normal and economic profit. Answer: B

Type: A Topic: 6 E: 468 MI: 224 125. If an industry evolves from monopolistic competition to oligopoly, we would expect: A) the four-firm concentration ratio to decrease. B) the four-firm concentration ratio to increase. C) the four-firm concentration ratio to remain the same. D) barriers to entry to weaken. Answer: B

Type: D Topic: 6 E: 469 MI: 225 126. Interindustry competition means that: A) in oligopolistic industries a few large firms compete with one another in bidding down product price. B) in some markets the producers of a particular product might face competition from products produced by other industries. C) firms that sell a product at one stage of production are faced with firms that buy the product at the next stage of production. D) in most industries there are usually a number of firms producing identical products. Answer: B

Type: A Topic: 6 E: 469 MI: 225 127. If you sum the squares of the market shares of each firm in an industry (as measured by percent of industry sales), you are calculating: A) the four-firm concentration ratio. C) the degree of collusion. B) the Herfindahl index. D) the Lerner index. Answer: B

Type: A Topic: 6 E: 469 MI: 225 128. The Herfindahl Index: A) measures the prices charged by oligopolistic manufacturers. B) is another name for the four-firm concentration ratio. C) tells us whether oligopolistic firms are engaging in collusion. D) gives much greater weight to larger firms than to smaller firms in an industry. Answer: D

Type: A Topic: 6 E: 469 MI: 225 129. If the four-firm concentration ratio in an oligopolistic industry is 100 percent and each firm has an equal percentage of sales, the Herfindahl Index is: A) 10,000. B) 2,500. C) 3,750. D) 1,000. Answer: B

McConnell/Brue: Economics, 16/e

Page 796

Chapter 25: Monopolistic Competition and Oligopoly

Type: A Topic: 6 E: 469 MI: 225 130. Assume six firms comprising an industry have market shares of 30, 30, 10, 10, 10, and 10 percent. The Herfindahl Index for this industry is: A) 2,525. B) 1,600. C) 2,200. D) 80. Answer: C

Type: A Topic: 6 E: 469 MI: 225 131. Suppose the Herfindahl Indexes for industries A, B, and C are 1,200, 5,000, and 7,500 respectively. These data imply that: A) market power is greatest in industry A. B) market power is greatest in industry B. C) market power is greatest in industry C. D) industry A is more monopolistic than industry C. Answer: C

Use the following to answer questions 132-136:

Firm A B C D E F

Market share (%) 20 20 20 20 10 10

Type: T Topic: 6 E: 468 MI: 224 132. The industry characterized by the above information is: A) an oligopoly. C) a purely competitive industry. B) a monopolistically competitive industry. D) a pure monopoly. Answer: A

Type: T Topic: 6 E: 468 MI: 224 133. The four-firm concentration ratio for the above industry is: A) 100 percent. B) indeterminate, since we don't know which four firms are included. C) 80 percent. D) 20 percent. Answer: C

Type: T Topic: 6 E: 469 MI: 225 134. The Herfindahl Index for the above industry is: A) 1,600. B) 1,800. C) 18,000. D) 80. Answer: B

Type: T Topic: 6 E: 469 MI: 225 135. If all the firms in the above industry merged into a single firm, the Herfindahl Index would become: A) 100. B) 1,000. C) 10,000. D) 100,000. Answer: C

McConnell/Brue: Economics, 16/e

Page 797

Chapter 25: Monopolistic Competition and Oligopoly

Type: T Topic: 6 E: 468-469 MI: 224-225 136. Suppose that firms in this industry miraculously split up such that there were 100 firms, each with a one percent market share. The four-firm concentration ratio and the Herfindahl Index respectively would be: A) 100 percent and 10,000. B) 4 percent and 4. C) 100 percent and 16. D) 4 percent and 16. Answer: D

Use the following to answer questions 137-140:
Firm A B C D E Market share (%) 40 30 20 5 5

Type: A Topic: 6 E: 468 MI: 224 Status: New 137. Refer to the above data. The four-firm concentration ratio for this industry is: A) 90 percent. B) 95 percent. C) 100 percent. D) indeterminate, because we don't know which four firms are included. Answer: B

Type: A Topic: 6 E: 469 MI: 225 Status: New 138. Refer to the above data. The Herfindahl Index for this industry is: A) 95. B) 1000. C) 2925. D) 2950. Answer: D

Type: A Topic: 6 E: 467-468 MI: 223-224 Status: New 139. Refer to the above data. This industry illustrates: A) pure competition. B) monopolistic competition. C) oligopoly. D) pure monopoly. Answer: C

Type: A Topic: 6 E: 468-469 MI: 224-225 Status: New 140. Refer to the data above. If Firm B merged with Firm C, the industry's four-firm concentration ratio would ____ and its Herfindahl Index would ____: A) rise; rise. B) fall; rise C) remain the same; rise. D) remain the same; fall. Answer: C

Game theory

Type: A Topic: 7 E: 469 MI: 225 Status: New 141. Game theory: A) is the analysis of how people (or firms) behave in strategic situations. B) is best suited for analyzing purely competitive markets. C) reveals that mergers between rival firms are self-defeating. D) reveals that price-fixing among firms reduces profits. Answer: A

McConnell/Brue: Economics, 16/e

Page 798

Chapter 25: Monopolistic Competition and Oligopoly

Type: A Topic: 7 E: 469 MI: 225 Status: New 142. The study of how people (or firms) behave in strategic situations is called: A) cost-benefit analysis. B) recursive analysis. C) normative economics. D) game theory. Answer: D

Type: A Topic: 7 E: 469-470 MI: 225-226 143. The terms strategic behavior and payoff matrix both relate directly to: A) the perfect competition model. C) game theory. B) the monopolistic competition model. D) the price leadership model. Answer: C

Type: A Topic: 7 E: 469 MI: 225 144. Game theory is best suited to analyze the pricing behavior of: A) pure monopolists. B) pure competitors. C) monopolistic competitors. D) oligopolists. Answer: D

Type: A Topic: 7 E: 470 MI: 226 145. Game theory can be used to demonstrate that oligopolists: A) rarely consider the potential reactions of rivals. B) experience economies of scale. C) that oligopolists can increase their profits through collusion. D) may be either homogeneous or differentiated. Answer: C

Use the following to answer questions 146-151:

Type: T Topic: 7 E: 470 MI: 226 146. The above matrix best illustrates: A) game theory. B) the principal-agent problem. C) product differentiation. D) price discrimination. Answer: A

McConnell/Brue: Economics, 16/e

Page 799

Chapter 25: Monopolistic Competition and Oligopoly

Type: T Topic: 7 E: 470 MI: 226 147. Refer to the above diagram where the numerical data show profits in millions of dollars. Beta's profits are shown in the northeast corner and Alpha's profits in the southwest corner of each cell. If both firms follow a high-price policy: A) Alpha will realize a $10 million profit and Beta a $30 million profit. B) each will realize a $20 million profit. C) Beta will realize a $10 million profit and Alpha a $30 million profit. D) each will realize a $15 million profit. Answer: B

Type: T Topic: 7 E: 470 MI: 226 148. Refer to the above diagram wherein the numerical data show profits in millions of dollars. Beta's profits are shown in the northeast corner and Alpha's profits in the southwest corner of each cell. If Beta commits to a high-price policy, Alpha will gain the largest profit by: A) also adopting a high-price policy. B) adopting a low-price policy. C) adopting a low-price policy, but only if Beta agrees to do the same. Answer: B

Type: T Topic: 7 E: 470 MI: 226 149. Refer to the above diagram where the numerical data show profits in millions of dollars. Beta's profits are shown in the northeast corner and Alpha's profits in the southwest corner of each cell. With independent pricing the outcome of this duopoly game will gravitate to cell: A) A. B) B. C) C. D) D. Answer: D

Type: T Topic: 7 E: 470-471 MI: 226-227 150. Refer to the above diagram where the numerical data show profits in millions of dollars. Beta's profits are shown in the northeast corner and Alpha's profits in the southwest corner of each cell. If Alpha and Beta engage in collusion, the outcome of the game will be at cell: A) A. B) B. C) C. D) D. Answer: A

Type: T Topic: 7 E: 470-471 MI: 226-227 151. Refer to the above diagram where the numerical data show profits in millions of dollars. Beta's profits are shown in the northeast corner and Alpha's profits in the southwest corner of each cell. If Alpha and Beta agree to a high-price policy through collusion, the temptation to cheat on that agreement is demonstrated by the fact that: A) Beta can increase its profit by lowering its price. B) Beta can increase its profit by increasing its price still further. C) both Alpha and Beta can earn even more profits if both agree to a low-price policy. D) Alpha can increase its profit by reducing its production costs. Answer: A

McConnell/Brue: Economics, 16/e

Page 800

Chapter 25: Monopolistic Competition and Oligopoly

Use the following to answer questions 152-154:

Type: T Topic: 7 E: 470 MI: 226 152. Refer to the above profits-payoff table for a duopoly. If the firms are acting independently and firm X sets its price at $6, firm Y will achieve the largest profit by selecting: A) a price higher than $6. B) a price between $5 and $6. C) $6. D) $4. Answer: D

Type: T Topic: 7 E: 470 MI: 226 153. Refer to the above profits-payoff table for a duopoly. If initially firms X and Y are charging $5 and $4 respectively: A) the two firms will be maximizing joint profits. B) Y will find it advantageous to raise its price if it was certain X would not alter its price. C) X will find it advantageous to raise its price if it was certain Y would not alter its price. D) both firms would find it advantageous to collude to raise their prices by $1 each. Answer: D

Type: T Topic: 7 E: 470 MI: 226 154. Refer to the above profits-payoff table for a duopoly. If initially firm X's price was $6 and Y's price was $5: A) X would find it profitable to cut price, provided Y also cut price. B) Y would find it profitable to cut price, provided X also cut price. C) Y would find it profitable to raise price by $1, provided X would also raise price by $1. D) both firms would profit by simultaneously lowering their prices by $1. Answer: C

McConnell/Brue: Economics, 16/e

Page 801

Chapter 25: Monopolistic Competition and Oligopoly

Use the following to answer questions 155-156:
Ajax’s advertising strategy Large Small budget budget
A B $800 $800

Acme’s advertising strategy

Large budget

$600

$1200 D $1200 $1000

C

Small budget $600

$1000

Type: A Topic: 7 E: 470 MI: 226 Status: New 155. Refer to the above game theory matrix where the numerical data show the profits resulting from alternative combinations of advertising strategies for Ajax and Acme. Ajax's profits are shown in the upper right part of each cell; Acme's profits are shown in the lower left. Without collusion, the outcome of the game is cell: A) A. B) B. C) C. D) D. Answer: A

Type: A Topic: 7 E: 470-471 MI: 226-227 Status: New 156. Refer to the above game theory matrix where the numerical data show the profits resulting from alternative combinations of advertising strategies for Ajax and Acme. Ajax's profits are shown in the upper right part of each cell; Acme's profits are shown in the lower left. With collusion and no cheating, the outcome of the game is cell: A) A. B) B. C) C. D) D. Answer: D

Kinked-demand curve model

Type: A Topic: 8 E: 472 MI: 228 157. Suppose an oligopolistic producer assumes its rivals will ignore a price increase but match a price cut. In this case the firm perceives its: A) demand curve as being of unit elasticity throughout. B) supply curve as kinked, being steeper below the going price than above. C) demand curve as kinked, being steeper below the going price than above. D) demand curve as kinked, being steeper above the going price than below. Answer: C

McConnell/Brue: Economics, 16/e

Page 802

Chapter 25: Monopolistic Competition and Oligopoly

Type: A Topic: 8 E: 472 MI: 228 158. The kinked-demand curve of an oligopolist is based on the assumption that: A) competitors will follow a price cut but ignore a price increase. B) competitors will match both price cuts and price increases. C) competitors will ignore a price cut but follow a price increase. D) there is no product differentiation. Answer: A

Type: A Topic: 8 E: 472 MI: 228 159. The kinked-demand curve describes a situation in which an oligopolist will be: A) interested in maintaining the going price unless there is a rather large change in costs. B) anxious to either increase or lower price. C) anxious to increase price but not to lower price. D) anxious to lower price but not to increase price. Answer: A

Type: A Topic: 8 E: 472 MI: 228 160. If an oligopoly is faced with a kinked-demand curve that is relatively elastic above, and relatively inelastic below, the going price, then it will: A) increase total revenue by increasing price, but lower total revenue by decreasing price. B) decrease total revenue by either increasing or decreasing price. C) increase total revenue by either increasing or decreasing price. D) increase total revenue by decreasing price, but lower total revenue by increasing price. Answer: B

Type: A Topic: 8 E: 472 MI: 228 161. The kinked-demand curve model of oligopoly is useful in explaining: A) the way that collusion works. B) why oligopolistic prices and outputs are extremely sensitive to changes in marginal cost. C) why oligopolistic prices might change only infrequently. D) the process by which oligopolists merge with one another. Answer: C

Type: A Topic: 8 E: 472 MI: 228 162. The kinked-demand curve model helps to explain price rigidity because: A) there is a gap in the marginal revenue curve within which changes in marginal cost will not affect output or price. B) demand is inelastic above and elastic below the going price. C) the model assumes firms are engaging in some form of collusion. D) the associated marginal revenue curve is perfectly elastic at the going price. Answer: A

Type: A Topic: 8 E: 472 MI: 228 163. If competing oligopolists completely ignore oligopolist X's price changes, then X's: A) demand curve will be less elastic than if the other oligopolists matched X's price changes. B) demand curve will be more elastic than if the other oligopolists matched X's price changes. C) marginal revenue curve will have a vertical gap. D) demand and marginal revenue curves will coincide. Answer: B

McConnell/Brue: Economics, 16/e

Page 803

Chapter 25: Monopolistic Competition and Oligopoly

Type: A Topic: 8 E: 472 MI: 228 164. If an oligopolist is faced with a marginal revenue curve that has a gap in it, we may assume that: A) it is colluding with its rivals to maximize joint profits. B) its demand curve is kinked. C) it is selling a standardized product. D) it is selling a differentiated product. Answer: B

Type: A Topic: 8 E: 472 MI: 228 165. The kinked-demand curve model of oligopoly: A) assumes a firm's rivals will ignore a price cut but match a price increase. B) embodies the possibility that changes in unit costs will have no effect on equilibrium price and output. C) assumes a firm's rivals will match any price change it may initiate. D) assumes a firm's rivals will ignore any price change it may initiate. Answer: B

Use the following to answer questions 166-169:

Type: G Topic: 8 E: 472-473 MI: 228-229 166. Refer to the above diagram for a noncollusive oligopolist. Suppose that the firm is initially in equilibrium at point E where the equilibrium price and quantity are P and Q. Which of the following statements is correct? A) Demand curve D1 assumes that rivals will match any price change initiated by this oligopolist. B) Demand curves D1 and D2 both assume that rivals will ignore any price change initiated by this oligopolist. C) Demand curves D1 and D2 both assume that rivals will match any price change initiated by this oligopolist. D) Demand curve D2 assumes that rivals will match any price change initiated by this oligopolist. Answer: A

Type: G Topic: 8 E: 472-473 MI: 228-229 167. Refer to the above diagram for a noncollusive oligopolist. Suppose that the firm is initially in equilibrium at point E where the equilibrium price and quantity are P and Q. If the firm's rivals will ignore any price increase but match any price reduction, then the firm's demand curve will be: A) D1ED2. B) D2ED1. C) D1ED1. D) D2ED2. Answer: B

McConnell/Brue: Economics, 16/e

Page 804

Chapter 25: Monopolistic Competition and Oligopoly

Type: G Topic: 8 E: 472-473 MI: 228-229 168. Refer to the above diagram for a noncollusive oligopolist. We assume that the firm is initially in equilibrium at point E where the equilibrium price and quantity are P and Q. If the firm's rivals will ignore any price increase but match any price reduction, the firm's marginal revenue curve will be: A) D1ED2. B) MR2abMR1. C) MR2aMR2. D) MR1bMR1. Answer: B

Type: G Topic: 8 E: 472-473 MI: 228-229 169. Refer to the above diagram for a noncollusive oligopolist. We assume that the firm is initially in equilibrium at point E where the equilibrium price and quantity are P and Q. If the firm's rivals will ignore any price increase but match any price reduction, over what range might marginal cost rise without disturbing equilibrium price and output? A) bE B) ab C) Qa D) Qb Answer: B

Use the following to answer questions 170-174:

Type: G Topic: 8 E: 473 MI: 229 170. The above diagram portrays: A) pure competition. B) monopolistic competition. Answer: C

C) noncollusive oligopoly.

D) pure monopoly.

Type: G Topic: 8 E: 473 MI: 229 171. Refer to the above diagram. Equilibrium output is: A) j. B) h. C) g. D) f . Answer: C

Type: G Topic: 8 E: 473 MI: 229 172. Refer to the above diagram. Equilibrium price is: A) e. B) d. C) c. D) b. Answer: B

McConnell/Brue: Economics, 16/e

Page 805

Chapter 25: Monopolistic Competition and Oligopoly

Type: G Topic: 8 E: 472-473 MI: 228-229 173. Refer to the above diagram. This firm's demand and marginal revenue curves are based on the assumption that: A) the firm has no immediate rivals. B) rivals will match both a price increase and a price decrease. C) rivals will match a price increase, but ignore a price decrease. D) rivals will ignore a price increase, but match a price decrease. Answer: D

Type: G Topic: 8 E: 472-473 MI: 228-229 174. Refer to the above diagram. In equilibrium the firm: A) is realizing an economic profit of ad per unit. C) is incurring a loss. B) should close down in the short run. D) is realizing an economic profit of bd per unit. Answer: A

Type: A Topic: 8 E: 472-473 MI: 228-229 175. According to the kinked-demand curve model, an oligopolistic firm will produce where: A) average total cost is minimized. B) price equals marginal cost. C) marginal revenue equals marginal cost. D) the demand curve intersects the average total cost curve. Answer: C

Type: A Topic: 8 E: 472-473 MI: 228 176. A kink may exist in an oligopolist's demand curve because: A) products are differentiated. B) an abrupt change in price elasticity occurs. C) the firm will ignore price cuts by rivals, but will match their price increases. D) there is a gap in marginal costs. Answer: B

Collusion; cartels; price leadership

Type: A Topic: 9 E: 475 MI: 231 177. OPEC provides an example of: A) a tacit understanding. B) noncollusive oligopoly. Answer: C

C) an international cartel. D) a monopolistically competitive industry.

Type: A Topic: 9 E: 474 MI: 230 178. Oligopolistic firms engage in collusion to: A) minimize unit costs of production. B) realize allocative efficiency, that is, the P = MC level of output. C) earn greater profits. D) increase production. Answer: C

McConnell/Brue: Economics, 16/e

Page 806

Chapter 25: Monopolistic Competition and Oligopoly

Type: A Topic: 9 E: 476 MI: 232 179. The likelihood of a cartel being successful is greater when: A) firms are producing a differentiated, rather than a homogeneous, product. B) cost and demand curves of various participants are very similar. C) the number of firms involved is relatively large. D) the economy is in the recession phase of the business cycle. Answer: B

Type: A Topic: 9 E: 476 MI: 232 180. Cartels are difficult to maintain in the long run because: A) they are illegal in all industrialized countries. B) individual members may find it profitable to cheat on agreements. C) it is more profitable for the industry to charge a lower price and produce more output. D) entry barriers are insignificant in oligopolistic industries. Answer: B

Type: A Topic: 9 E: 474-476 MI: 230-232 181. Three major means of collusion by oligopolists are: A) cartels, tacit understandings, and price leadership. B) market sharing, mutual interdependence, and product differentiation. C) cartels, kinked-demand pricing, and product differentiation. D) tacit understandings, P = MC pricing, and mutual interdependence. Answer: A

Type: A Topic: 9 E: 474 MI: 230 182. If the firms in an oligopolistic industry can establish an effective cartel, the resulting output and price will approximate those of: A) a purely competitive producer. B) a pure monopoly. C) a monopolistically competitive producer. D) an industry with a low four-firm concentration ratio. Answer: B

Type: F Topic: 9 E: 476 MI: 232 183. In the United States cartels are: A) quite common in industries that produce nondurable goods. B) in violation of the antitrust laws. C) concentrated in monopolistically competitive industries. D) encouraged by government policy so firms can achieve economies of scale. Answer: B

Type: A Topic: 9 E: 475 MI: 231 184. Assume the several manufacturers of ceramic tile in a city reach a verbal agreement to establish the price of their product at 55 cents per tile. This best describes: A) multiproduct pricing. B) a cartel. C) a tacit understanding. D) price leadership. Answer: C

McConnell/Brue: Economics, 16/e

Page 807

Chapter 25: Monopolistic Competition and Oligopoly

Type: A Topic: 9 E: 476 MI: 232 185. One would expect that collusion among oligopolistic producers would be easiest to achieve in which of the following cases? A) a rather large number of firms producing a differentiated product B) a very few firms producing a differentiated product C) a rather large number of firms producing a homogeneous product D) a very few firms producing a homogeneous product Answer: D

Type: D Topic: 9 E: 474 MI: 230 186. Suppose the only three existing manufacturers of video game players signed a written contract by which each agreed to charge the same price for products and to distribute their products only in the geographical area assigned them in the contract. This best describes: A) cost-plus pricing. B) multiproduct pricing. C) a cartel. D) price leadership. Answer: C

Type: D Topic: 9 E: 475 MI: 231 187. Which of the following statements is correct? A) A cartel is usually a formal agreement among oligopolists that sets product price and determines each firm's market share. B) The practice of price leadership is almost always based on a formal written agreement. C) All oligopolists heavily advertise their products. D) Active and frequent price competition between firms is a basic characteristic of oligopoly. Answer: A

Type: D Topic: 9 E: 477 MI: 233 188. Suppose firms in a collusive oligopoly decide to establish their prices at a level that discourages new rivals from entering the industry. This is called: A) mutual interdependence. B) pricing the demand curve. C) limit pricing. D) price leadership. Answer: C

Type: A Topic: 9 E: 474 MI: 230 189. If the several oligopolistic firms that comprise an industry behave collusively, the resulting price and output will most likely resemble those of: A) bilateral monopoly. B) pure monopoly. C) monopolistic competition. D) pure competition. Answer: B

Type: A Topic: 9 E: 476 MI: 232 190. Other things equal, cartels and similar collusive arrangements are easier to establish and maintain: A) when there are ample opportunities for the firms to make secret price concessions to selected buyers. B) during periods of cyclical stability and full employment. C) when the demand and cost conditions of the participating firms differ substantially. D) when the number of firms is relatively large. Answer: B

Type: D Topic: 9 E: 477 MI: 233 191. A break-down in price leadership leading to successive rounds of price cuts is known as: A) limit pricing. B) a price war. C) tacit pricing D) price discrimination Answer: B

McConnell/Brue: Economics, 16/e

Page 808

Chapter 25: Monopolistic Competition and Oligopoly

Type: F Topic: 9 E: 475 MI: 231 192. Which of the following nations is not a member of the OPEC oil cartel? A) Saudi Arabia. B) Iran. C) Venezuela. D) Norway. Answer: D

Type: F Topic: 9 E: 475 MI: 231 193. Which of the following companies was convicted in the 1990s as part of a conspiracy to fix prices? A) Archer Daniels Midland B) Boeing C) Wendy's D) IBM Answer: A

Type: F Topic: 9 E: 475 MI: 231 194. Secret conspiracies to fix prices are examples of: A) cartels. B) price leadership. C) overt collusion. D) covert collusion. Answer: D

Advertising

Type: F Topic: 10 E: 477 MI: 233 195. In recent years advertising expenditures in the United States have been: A) 10 to 12 percent of GDP per year. C) more than $215 billion per year. B) $1 to $2 billion per year. D) about $20 billion per year. Answer: C

Type: A Topic: 10 E: 478 MI: 234 196. Advertising can enhance economic efficiency when it: A) increases brand loyalty. B) raises entry barriers. C) increases consumer awareness of substitute products. D) boosts average total cost. Answer: C

Type: A Topic: 10 E: 478 MI: 234 197. Advertising can enhance economic efficiency when it: A) increases brand loyalty. B) expands sales such that firms achieve substantial economies of scale. C) keeps new firms from entering profitable industries. D) is undertaken by pure competitors. Answer: B

Type: A Topic: 10 E: 478-479 MI: 234-235 198. Advertising can impede economic efficiency when it: A) increases entry barriers. B) reduces brand loyalty. C) enables firms to achieve substantial economies of scale. D) increases consumer awareness of substitute products. Answer: A

McConnell/Brue: Economics, 16/e

Page 809

Chapter 25: Monopolistic Competition and Oligopoly

Type: A Topic: 10 E: 478-479 MI: 234-235 199. Advertising can impede economic efficiency when it: A) reduces entry barriers. B) reduces brand loyalty. C) leads to greater monopoly power. D) provides consumers with useful information about product quality. Answer: C

Type: A Topic: 10 E: 479 MI: 235 200. The presence of advertising in a particular market: A) tells us that the industry is an oligopoly. B) tells us that the industry is monopolistically competitive. C) means that barriers to entering the industry are high. D) may or may not mean substantial monopoly power in the industry. Answer: D

Efficiency aspects

Type: A Topic: 11 E: 479 MI: 235 201. We would expect a cartel to achieve: A) both allocative efficiency and productive efficiency. B) allocative efficiency, but not productive efficiency. C) productive efficiency, but not allocative efficiency. D) neither allocative efficiency nor productive efficiency. Answer: D

Type: C Topic: 11 E: 479 MI: 235 202. Suppose that a particular industry has a a four-firm concentration ratio of 85 and a Herfindahl Index of 3000. Most likely, this industry would achieve: A) both productive efficiency and allocative efficiency. B) allocative efficiency but not productive efficiency. C) neither productive efficiency nor productive efficiency. D) productive efficiency but not allocative efficiency. Answer: C

Type: A Topic: 11 E: 479 MI: 235 203. Suppose that an industry is characterized by a few firms and price leadership. We would expect that: A) price would equal marginal cost. B) price would equal average total cost. C) price would exceed both marginal cost and average total cost. D) marginal revenue would exceed marginal cost. Answer: C

McConnell/Brue: Economics, 16/e

Page 810

Chapter 25: Monopolistic Competition and Oligopoly

Type: A Topic: 11 E: 479 MI: 235 204. The conclusion that oligopoly is inefficient relative to the competitive ideal must be qualified because: A) industry price leaders often select a price equal to marginal cost. B) over time oligopolistic industries may promote more rapid product development and greater improvement of production techniques than if they were purely competitive. C) increased output due to persuasive advertising may perfectly offset the restriction of output caused by monopoly power. D) many oligopolists sell their products in monopolistically competitive or even purely competitive industries. Answer: B

Review of four structures

Type: A Topic: 12 E: 461, 467 MI: 217, 223 205. Monopolistic competition and oligopoly are alike in that: A) the number of firms is approximately the same in both cases. B) the kinked-demand analysis is applicable in both instances. C) strong mutual interdependence exists among firms in both market models. D) nonprice competition is common to both. Answer: D

Type: A Topic: 12 E: 461, 467 MI: 217, 223 206. Product differentiation is present in: A) purely competitive markets only. B) monopolistically competitive markets only. C) oligopolistic markets only. D) both monopolistically competitive and oligopolistic markets. Answer: D

Type: A Topic: 12 E: 479 MI: 235 207. Under which of the following market structures will the long-run equilibrium price be equal to marginal cost? A) oligopoly B) monopolistic competition C) pure monopoly D) pure competition Answer: D

Type: A Topic: 12 E: 439, 467 MI: 195, 223 208. In which of the following industry structures is the entry of new firms the most difficult? A) pure monopoly B) oligopoly C) monopolistic competition D) pure competition Answer: A

Type: A Topic: 12 E: 460 MI: 216 209. An industry comprised of 40 firms, none of which has more than 3 percent of the total market for a differentiated product is an example of: A) monopolistic competition B) oligopoly C) pure monopoly D) pure competition Answer: A

McConnell/Brue: Economics, 16/e

Page 811

Chapter 25: Monopolistic Competition and Oligopoly

Type: A Topic: 12 E: 413, 438 MI: 169, 194 210. A one-firm industry is known as: A) monopolistic competition B) oligopoly C) pure monopoly Answer: C

D) pure competition

Type: A Topic: 12 E: 467 MI: 223 211. An industry comprised of four firms, each with approximately 25 percent of the total market for a product, is an example of: A) monopolistic competition B) oligopoly C) pure monopoly D) pure competition Answer: B

Type: A Topic: 12 E: 413 MI: 169 212. An industry comprised of a very large number of sellers that are producing a homogeneous or standardized product is called: A) monopolistic competition B) oligopoly C) pure monopoly D) pure competition Answer: D

Type: A Topic: 12 E: 414, 467 MI: 170, 223 213. An industry comprised of a small number of firms, each of which considers the potential reactions of its rivals in making price-output decisions is called: A) monopolistic competition B) oligopoly C) pure monopoly D) pure competition Answer: B

Type: A Topic: 12 E: 468 MI: 224 214. An industry producing a differentiated product whose four-firm concentration ratio is 18 percent is an example of: A) monopolistic competition B) oligopoly C) pure monopoly D) pure competition Answer: A

Type: A Topic: 12 E: 468 MI: 224 215. An industry producing a homogeneous product whose four-firm concentration ratio is 76 percent is an example of: A) monopolistic competition B) oligopoly C) pure monopoly D) pure competition Answer: B

Type: A Topic: 12 E: 414 MI: 170 216. In which of the following market models do individual firms exert no control over product price? A) oligopoly B) pure monopoly C) monopolistic competition D) pure competition Answer: D

McConnell/Brue: Economics, 16/e

Page 812

Chapter 25: Monopolistic Competition and Oligopoly

Type: A Topic: 12 E: 414 MI: 170 217. Which of the following correctly arrays the various market structures in terms of their similarities to one another? A) pure competition, oligopoly, monopolistic competition, pure monopoly B) pure monopoly, oligopoly, monopolistic competition, pure competition C) pure competition, pure monopoly, monopolistic competition, oligopoly D) pure competition, oligopoly, pure monopoly, monopolistic competition Answer: B

Use the following to answer questions 218-226:

Type: G Topic: 12 E: 426 MI: 182 218. The purely competitive market model is portrayed in the above figures by: A) Figure A. B) Figure B. C) both Figures B and D. D) Figure C. Answer: B

Type: G Topic: 12 E: 414, 461 MI: 170, 217 219. Refer to the above figures. We would expect industry entry and exit to be relatively easy in: A) Figure A only. B) Figure C only. C) both Figures A and C. D) both Figures B and D. Answer: D

Type: G Topic: 12 E: 432 MI: 188 220. Refer to the above figures. Both allocative and productive efficiency are being realized in: A) all four figures. B) Figures B and D. C) Figure D only. D) Figure B only. Answer: D

Type: G Topic: 12 E: 474-475 MI: 230-231 221. Refer to the above figures. Collusion is most likely to occur in the industry(ies) represented by: A) Figure A. B) Figure B. C) Figure C. D) both Figures B and D. Answer: C

McConnell/Brue: Economics, 16/e

Page 813

Chapter 25: Monopolistic Competition and Oligopoly

Type: G Topic: 12 E: 461, 467 MI: 217, 223 222. Refer to the above figures. Product differentiation may be present in: A) Figure A only. B) Figure B only. C) Figure C only. D) both Figures C and D. Answer: D

Type: G Topic: 12 E: 453-454 MI: 209-210 223. Refer to the above figures. Government regulation of price and service is most likely to occur in: A) Figure A only. B) Figure D only. C) both Figures A and C. D) both Figures A and D. Answer: A

Type: G Topic: 12 E: 474 MI: 230 224. Refer to the above figures. Long-run economic profits are most likely to occur in: A) Figures A and B. B) Figure B only. C) Figure D. D) Figures A and C. Answer: D

Type: G Topic: 12 E: 414 MI: 170 225. Refer to the above figures. Industry entry is likely to be most difficult in: A) Figure A. B) Figure B. C) Figure C. D) Figure D. Answer: A

Type: G Topic: 12 E: 414 MI: 170 226. Refer to the above figures. A homogeneous or standardized product is most likely to be produced in: A) Figure A. B) Figure B. C) Figure C. D) Figure D. Answer: B

Consider This Questions

Type: A E: 470 MI: 226 Status: New 227. (Consider This) The story about three sellers of Native American arts and crafts best illustrates the idea of; A) strategic behavior. B) excess capacity. C) the role of advertising. D) product differentiation. Answer: A

Type: A E: 470 MI: 226 Status: New 228. (Consider This) The Native American arts and crafts story illustrates the twin ideas of: A) product differentiation and monopolistic competition. B) excess capacity and monopolistic competition. C) local oligopoly and strategic behavior. D) pure monopoly and price discrimination. Answer: C

McConnell/Brue: Economics, 16/e

Page 814

Chapter 25: Monopolistic Competition and Oligopoly

Last Word Questions

Type: F E: 480 MI: 236 229. (Last Word) The U.S. beer industry: A) has become monopolistically competitive as the result of new production technologies. B) has evolved from monopolistic competition to oligopoly in the past 50 years. C) is populated by hundreds of relatively large, independent brewers. D) approximates the purely competitive market model. Answer: B

Type: F E: 480-481 MI: 236-237 230. (Last Word) Increased concentration in the beer industry has been caused by: A) changes in consumer tastes from the strong beers of small breweries to the light beers of the large brewers. B) a shift of beer consumption from bars to homes. C) technological progress which has speeded up canning and bottling lines and lowered costs. D) all of the above factors. Answer: D

Type: F E: 480 MI: 236 231. (Last Word) The two largest U.S. brewers share about: A) 10 percent of the beer market. C) 50 percent of the beer market. B) 20 percent of the beer market. D) 70 percent of the beer market. Answer: D

Type: F E: 480 MI: 236 232. (Last Word) The leading seller of beer in the United States is: A) Coors. B) Anheuser-Busch. C) Miller Brewing Company. D) Corona. Answer: B

Type: F E: 469, 480 MI: 225, 236 233. (Last Word) The two leading producers of beer in the United States have market shares of 47 percent and 22 percent, respectively. On the basis of that information, we can conclude that: A) the four-firm concentration ratio in the beer industry is nearly 100 percent. B) the Herfindahl index in the beer industry exceeds 2500. C) firms practice overt collusion to set prices in the industry. D) the industry is monopolistically competitive. Answer: B

True/False Questions

Type: A E: 464 MI: 220 234. In the long run monopolistically competitive firms make normal profits because they are forced to operate at the minimum point on their average total cost curve. Answer: False

McConnell/Brue: Economics, 16/e

Page 815

Chapter 25: Monopolistic Competition and Oligopoly

Type: A E: 464-465 MI: 220-221 235. The monopolistically competitive seller maximizes profits by equating price and marginal cost. Answer: False

Type: A E: 464-465 MI: 220-221 236. Monopolistically competitive firms are inefficient because they produce at a point on the rising segment of their average cost curves. Answer: False

Type: A E: 462 MI: 218 237. The demand curve of a monopolistically competitive producer is less elastic than that of a purely competitive producer. Answer: True

Type: D E: 464 MI: 220 238. The larger the number of firms and the less the degree of product differentiation, the greater will be the elasticity of a monopolistically competitive seller's demand curve. Answer: True

Type: A E: 464 MI: 220 239. The economic profits earned by monopolistically competitive sellers are zero in the long run. Answer: True

Type: A E: 465-466 MI: 221-222 240. The excess capacity problem associated with monopolistic competition implies that fewer firms could produce the same industry output at a lower total cost. Answer: True

Type: A E: 462 MI: 218 241. The demand curve of a monopolistically competitive firm is more elastic than that of a pure monopolist. Answer: True

Type: A E: 464 MI: 220 242. The monopolistically competitive seller equates price and marginal cost in maximizing profits. Answer: False

Type: A E: 464 MI: 220 243. Monopolistically competitive sellers realize economic profits in the long run because entry barriers are significant. Answer: False

Type: A E: 464-465 MI: 220-221 244. Monopolistically competitive sellers produce efficiently because they obtain only normal profits in the long run. Answer: False

McConnell/Brue: Economics, 16/e

Page 816

Chapter 25: Monopolistic Competition and Oligopoly

Type: A E: 472 MI: 228 245. The oligopolist's kinked-demand curve is highly elastic below and highly inelastic above the going product price. Answer: False

Type: A E: 467 MI: 223 246. Mutual interdependence means that oligopolistic producers rely on price competition in determining their shares of the total market for their product. Answer: False

Type: D E: 472 MI: 228 247. If an oligopolist's several rivals exactly match any price changes it initiates, the demand curve will be less elastic than if its price changes are ignored by its rivals. Answer: True

Type: A E: 474 MI: 230 248. If three or four homogeneous oligopolists collude, the resulting price and production outcomes will be similar to those of pure monopoly. Answer: True

Type: A E: 476 MI: 232 249. Generally speaking, the larger the number of firms in an oligopolistic industry, the more difficult it is for those firms to collude. Answer: True

Type: A E: 467 MI: 223 250. Generally speaking, oligopolistic industries producing raw materials and semifinished goods usually offer differentiated products, while oligopolists producing consumer goods usually offer standardized products. Answer: False

Type: C E: 468-469 MI: 224-225 251. Two industries that have the same 4-firm concentration ratio can have significantly different Herfindahl indexes. Answer: True

Type: F E: 469 MI: 225 252. As it relates to oligopoly, game theory focuses on the strategic behavior of rival firms. Answer: True

Type: F E: 469 MI: 225 253. The highest possible value of the Herfindahl index is 1,000. Answer: False

McConnell/Brue: Economics, 16/e

Page 817

Chapter 25: Monopolistic Competition and Oligopoly

Type: D E: 467 MI: 223 Status: New 254. The market structure called "oligopoly" includes industries with one or a small number of firms. Answer: False

Type: F E: 467 MI: 223 Status: New 255. The U.S. breakfast cereal industry is an example of differentiated oligopoly. Answer: True

Type: F E: 467 MI: 223 Status: New 256. The U.S. steel industry is an example of homogeneous oligopoly. Answer: True

Type: F E: 472 MI: 228 Status: New 257. In the kinked demand curve model, the firm's marginal revenue curve and demand curve are identical. Answer: False

Type: F E: 467 MI: 223 Status: New 258. Homogenous oligopolists tend to advertise more than do differentiated oligopolists. Answer: False

McConnell/Brue: Economics, 16/e

Page 818

CHAPTER 26

Technology, R&D, and Efficiency

Topic

Question numbers 1-19 20-29 30-34 35-39 40-57 58-67 68-75 76-80 81-93 94-100 101-107 108-113 114-115 116-118 119-130

____________________________________________________________

_______________________________________

1. Technological advance: invention; innovation; diffusion 2. Entrepreneurship; scientific research 3. R&D: interest-rate cost of funds 4. R&D: expected rate of return 5. Optimal R&D 6. Product innovation and revenue enhancement 7. Process innovation and cost reduction 8. Imitation problem: "fast-second strategy" 9. Benefits of being first with innovation 10. Role of market structure 11. Inverted-U theory; role of scientific opportunity 12. Technological advance and efficiency Consider This Last Word True-False Multiple Choice Questions Technological advance: invention; innovation; diffusion

____________________________________________________________

_______________________________________

Type: D Topic: 1 E: 484 MI: 240 1. Broadly defined, technological advance: A) can occur in either the short run, long run, or very long run. B) comprises new and improved goods and services and new and improved ways of producing or distributing them. C) includes invention, but not innovation or diffusion. D) includes product innovation, but not process innovation. Answer: B

Type: F Topic: 1 E: 484 MI: 240 2. In economists' models, technological advance occurs in: A) the very long run. B) either the short run, long run, or very long run. C) manufacturing industries but not in service industries. D) pure competition, but not in monopolistic competition, oligopoly, and pure monopoly. Answer: A

Chapter 26: Technology, R&D, and Efficiency

Type: A Topic: 1 E: 484 MI: 240 3. Technological advance is shown as a(n): A) movement from a point inside a production possibilities curve to a point on the curve. B) movement along a production possibilities curve. C) outward shift of a production possibilities curve. D) inward shift of a production possibilities curve. Answer: C

Type: D Topic: 1 E: 484 MI: 240 4. Technological advance is a three-step process involving: A) invention, duplication, and diffusion. C) invention, innovation, and diffusion. B) duplication, innovation, and diversity. D) necessity, invention, and solution. Answer: C

Type: A Topic: 1 E: 484 MI: 240 5. Fiber-optic telephone lines are rapidly replacing copper telephone cable. This is an example of: A) capital-labor substitutability. C) market (or monopoly) power. B) economies of scale. D) technological advance. Answer: D

Type: D Topic: 1 E: 485 MI: 241 6. The first discovery (as distinct from first commercial application) of a product or process is called: A) innovation. B) invention. C) creative destruction. D) diffusion. Answer: B

Type: D Topic: 1 E: 485 MI: 241 7. The successful commercial introduction of a new product, the use of a new method, or the creation of a new form of business enterprise is called: A) innovation. B) invention. C) creative destruction. D) diffusion. Answer: A

Type: D Topic: 1 E: 485 MI: 241 8. The wide imitation and spread of an innovation is called: A) innovation. B) invention. C) creative destruction. Answer: D

D) diffusion.

Type: D Topic: 1 E: 485 MI: 241 9. The first successful commercial introduction of a new product refers to: A) invention. B) innovation. C) diffusion. D) diversification. Answer: B

Type: A Topic: 1 E: 485 MI: 241 10. The first commercial introduction, as opposed to first discovery, of transparent tape is an example of: A) innovation. B) invention. C) creative destruction. D) diffusion. Answer: A

McConnell/Brue: Economics, 16/e

Page 820

Chapter 26: Technology, R&D, and Efficiency

Type: A Topic: 1 E: 485 MI: 241 11. The first discovery, as opposed to first commercial application, of the water-soluble material used in contact lens is an example of: A) innovation. B) invention. C) creative destruction. D) diffusion. Answer: B

Type: D Topic: 1 E: 485 MI: 241 12. The spread of innovation through imitation refers to: A) invention. B) diffusion. C) duplication. D) diversification. Answer: B

Type: A Topic: 1 E: 485 MI: 241 13. As pizza topped with barbecue chicken became popular at specialty restaurants, Pizza Hut introduced a similar pizza. This imitation illustrates: A) innovation. B) invention. C) creative destruction. D) diffusion. Answer: D

Type: A Topic: 1 E: 485 MI: 241 14. Kodak introduced to the marketplace a digital camera which uses no film, but which takes photos that can be shown on personal computers. This is an example of: A) economies of scale. B) product innovation. C) process innovation. D) venture capital. Answer: B

Type: A Topic: 1 E: 485 MI: 241 15. Suppose firm X implements a new process for extracting copper from copper-bearing ore. This is an example of: A) product innovation. B) process innovation. C) economics of scale. D) the inverted-U theory. Answer: B

Type: C Topic: 1 E: 485 MI: 241 16. Which of the following is a true statement? A) innovation normally follows invention and precedes diffusion. B) invention normally follows diffusion and precedes innovation C) diffusion normally follows invention and precedes innovation. D) innovation normally follows diffusion and precedes invention. Answer: A

Type: A Topic: 1 E: 485 MI: 241 17. Innovation: A) is the first discovery of a product or process, rather than its first successful commercial introduction. B) includes new products, but not new production methods. C) is also known as diffusion. D) can either increase or decrease the market share of a large firm, depending on whether it is introduced by the large firm or one of its competitors. Answer: D

McConnell/Brue: Economics, 16/e

Page 821

Chapter 26: Technology, R&D, and Efficiency

Type: F Topic: 1 E: 485 MI: 241 18. In the United States, research and development spending as a percentage of GDP is: A) 1.5 to 2.0 percent, which is lower than that of most other industrial countries. B) 2.5 to 3.0 percent, which is higher than that of most other industrial countries. C) 4.5 to 5.0 percent, which is lower than that of most other industrial countries. D) 5.5 to 6.0 percent, which is higher than that of most other industrial countries. Answer: B

Type: F Topic: 1 E: 485 MI: 241 19. Which of the following correctly orders, highest to lowest, the relative magnitudes of U.S spending by businesses on components of R&D? A) invention, basic research, innovation. C) innovation, invention, basic research. B) invention, innovation, basic research. D) basic research, invention, innovation. Answer: C

Entrepreneurship; scientific research

Type: A Topic: 2 E: 485 MI: 241 20. When economists view technological change as internal to the economy, they mean that it: A) occurs randomly. B) occurs accidentally. C) arises deliberately from the profit motive and competition. D) arises mainly from government subsidies. Answer: C

Type: F Topic: 2 E: 485 MI: 241 21. U.S. firms collectively devote the largest portion of their total R&D spending to: A) applied research (pursuing invention). C) innovation and diffusion. B) basic scientific research. D) financing startup firms. Answer: C

Type: F Topic: 2 E: 485 MI: 241 22. About ____ percent of business R&D spending is for basic research. A) 1. B) 6. C) 13. D) 20. Answer: B

Type: A Topic: 2 E: 486 MI: 242 23. The modern view of technological advance is that it: A) is rooted in the independent advance of science, an element largely external to the market system. B) it is rarely carried out by oligopolists or pure monopolists. C) is an internal element of capitalism, occurring in responses to profit incentives. D) necessarily destroys existing monopoly power. Answer: C

McConnell/Brue: Economics, 16/e

Page 822

Chapter 26: Technology, R&D, and Efficiency

Type: A Topic: 2 E: 486 MI: 242 24. How do entrepreneurs differ from "other innovators?" A) entrepreneurs bear risk; "other innovators" do not. B) "other innovators" bear risk; entrepreneurs do not. C) entrepreneurs only invent; "other innovators" find new markets for inventions. D) entrepreneurs develop entirely new products; "other innovators" focus on product improvements. Answer: A

Type: F Topic: 2 E: 486 MI: 242 25. Entrepreneurship: A) can be carried out either by individuals or teams of individuals. B) is no longer important in a world of large corporate R&D laboratories. C) is, by definition, carried out by a single individual. D) is relevant to pure competition but not to the other market structures. Answer: A

Type: A Topic: 2 E: 486-487 MI: 242-243 26. Entrepreneurs: A) work exclusively in government and university R&D laboratories. B) often form small companies called startups. C) are less likely to exist in service industries than in manufacturing industries. D) are engaged mainly in basic scientific research. Answer: B

Type: A Topic: 2 E: 487 MI: 243 27. Entrepreneurs: A) include everyone engaged in R&D work. B) are located in small enterprises only. C) try to anticipate the future. D) work exclusively in government and university R&D laboratories. Answer: C

Type: F Topic: 2 E: 487 MI: 243 28. The major source of new scientific knowledge in the United States is: A) university and government research. B) R&D work in large corporations. C) entrepreneurs working alone. D) purely competitive and monopolistically competitive firms. Answer: A

Type: A Topic: 2 E: 487 MI: 243 29. New scientific knowledge mainly comes from university and government laboratories, not private firms, because: A) large corporations do not have funds available to channel toward basic research. B) government pays scientists higher salaries than do private firms. C) entrepreneurs find it difficult to secure venture capital to finance innovation. D) basic scientific principles, as such, cannot be patented and do not always have commercial applicability. Answer: D

McConnell/Brue: Economics, 16/e

Page 823

Chapter 26: Technology, R&D, and Efficiency

R&D: interest-rate cost of funds

Type: A Topic: 3 E: 488 MI: 244 30. As it relates to the R&D decision, the interest-rate-cost-of-funds curve: A) usually slopes downward. B) is the marginal cost element in the MB = MC decision framework. C) indicates a constant rate of return, r. D) reflects the interest rate on bank loans, but not the implicit interest rate on the use of retained earnings. Answer: B

Type: D Topic: 3 E: 488 MI: 244 31. Funds lent to startup firms in return for shares of the profit if the firms succeed are called: A) retained earnings. B) time deposits. C) venture capital. D) transfer payments. Answer: C

Type: C Topic: 3 E: 488 MI: 244 32. In exchange for a share of ZYX's profits if it succeeds, Firm ABC provides development funds to newly formed ZYX which is developing an innovative product. ABC funds are called ____________while ZYX is know as a ____________. A) venture capital; startup. C) mutual funds; startup. B) retained earnings; entrepreneurial firm. D) transfer payments; entrepreneurial firm. Answer: A

Type: D Topic: 3 E: 488 MI: 244 33. The retained earnings that corporations often use to finance R&D are also known as: A) venture capital. B) undistributed corporate profits. C) dividends. D) mutual funds. Answer: B

Type: F Topic: 3 E: 488 MI: 244 34. A major source of funding of R&D in large, established corporations is: A) venture capital. B) dividends. C) mutual funds. D) retained earnings. Answer: D

R&D: expected rate of return

Type: A Topic: 4 E: 488 MI: 244 35. The marginal benefit to a firm from its R&D expenditures is depicted by its: A) interest-rate-cost of funds curve. C) venture capital acquisition curve. B) expected-rate-of-return curve. D) retained earnings pay-out curve. Answer: B

McConnell/Brue: Economics, 16/e

Page 824

Chapter 26: Technology, R&D, and Efficiency

Type: A Topic: 4 E: 489 MI: 245 36. As it relates to R&D, the expected-rate-of-return curve, r: A) usually slopes upward. B) shows the cost of financing various levels of R&D. C) varies in location depending on the location of the interest-rate-cost-of-funds curve, i. D) represents the marginal benefit element in the MB = MC decision framework. Answer: D

Type: A Topic: 4 E: 489 MI: 245 37. Suppose a firm anticipates that a R&D expenditure of $100 million will result in a new production process that will reduce costs and thus create a one-time added profit of $112 million a year later. The firm's expected rate of return is: A) 0.12 percent. B) 112 percent. C) 12 percent. D) 2 percent. Answer: C

Type: A Topic: 4 E: 489 MI: 245 38. As it relates to R&D, a firm's expected-rate-of-return-curve, r: A) slopes downward because the firm arrays, highest to lowest, the rates of returns on R&D activities. B) slopes upward because of the law of diminishing returns. C) is a horizontal line. D) depends on whether it borrows from the bank or used retained earnings in financing R&D. Answer: A

Type: A Topic: 4 E: 490 MI: 246 39. The corporate decision on type and level of R&D activity is difficult because: A) the interest-rate cost of funds is difficult to estimate. B) much of corporate R&D is based on the pursuit of science, not on the profit motive. C) expected returns lie in the future and are highly uncertain. D) total returns and marginal returns greatly diverge. Answer: C

Optimal R&D

Type: A Topic: 5 E: 489-490 MI: 245-246 40. A profit-maximizing firm should not undertake a R&D project for which the: A) Expected rate of return exceeds its interest-rate cost of funds. B) interest-rate cost of funds exceeds the expected rate of return. C) expected returns are in the distant future. D) the expected returns, though potentially very large, are uncertain. Answer: B

Type: A Topic: 5 E: 488-489 MI: 244-245 41. In deciding on an optimal amount and type of research and development, firms should adhere to the rule: Expand R&D until: A) expected rate of return is zero. B) expected rate of return equals the interest rate. C) expected rate of return exceeds the interest rate by the greatest amount. D) the interest rate is constant. Answer: B

McConnell/Brue: Economics, 16/e

Page 825

Chapter 26: Technology, R&D, and Efficiency

Type: A Topic: 5 E: 488-489 MI: 244-245 42. Suppose a firm anticipates that a particular R&D expenditure of $100 million will result in a new product and thus create a one-time added profit of $108 million a year later. The firm will: A) undertake the R&D expenditure if its interest-rate-cost of borrowing is 12 percent. B) undertake the R&D expenditure if its interest-rate-cost of borrowing is10 percent. C) not undertake the R&D expenditure if its interest-rate-cost of borrowing is 9 percent. D) not undertake the R&D expenditure if its interest-rate-cost of borrowing is 7 percent. Answer: C

Type: A Topic: 5 E: 488-489 MI: 244-245 43. Suppose a firm anticipates that a particular R&D expenditure of $20 million will result in a new product and thus create a one-time added profit of $22 million a year later. The firm will: A) not undertake the R&D expenditure if its interest-rate-cost of borrowing is 8 percent. B) undertake the R&D expenditure if its interest-rate-cost of borrowing is 12 percent. C) undertake the R&D expenditure if its interest-rate-cost of borrowing is 20 percent. D) undertake the R&D expenditure if its interest-rate-cost of borrowing is 9 percent. Answer: D

Use the following to answer questions 44-48:
Expected rate of return (%) 25 20 15 10 5 R&D (millions of dollars) 20 40 60 80 100 Interest rate cost of funds (%) 10 10 10 10 10

Type: T Topic: 5 E: 488-489 MI: 244-245 44. If we plotted the above data on a graph with R&D expenditures on the horizontal axis, the: A) interest-rate-cost-of-funds-curve would be a vertical line. B) interest-rate-cost-of-funds-curve would be horizontal line. C) expected-rate-of-return-curve would slope upward. D) expected-rate-of-return-curve would be a horizontal line. Answer: B

Type: T Topic: 5 E: 488-489 MI: 244-245 45. If we plotted the above data on a graph with R&D expenditures on the horizontal axis, the: A) interest-rate-cost-of-funds-curve would be a vertical line. B) interest-rate-cost-of-funds-curve would slope downward. C) expected-rate-of-return-curve would slope downward. D) expected-rate-of-return-curve would be a horizontal line. Answer: C

Type: T Topic: 5 E: 490 MI: 246 46. Refer to the above data. The firm's optimal amount of R&D spending is: A) $20 million. B) $40 million. C) $60 million. D) $80 million. Answer: D

McConnell/Brue: Economics, 16/e

Page 826

Chapter 26: Technology, R&D, and Efficiency

Type: T Topic: 5 E: 490 MI: 246 47. Refer to the above data. At $100 million of R&D expenditures, the: A) marginal cost of R&D exceeds the marginal benefit. B) marginal benefit of R&D exceeds the marginal cost. C) expected rate of return from R&D is negative. D) firm has exceeded its affordable level of R&D. Answer: A

Type: T Topic: 5 E: 490 MI: 246 48. Refer to the above data. At $20 million of R&D expenditures, the: A) marginal cost of R&D exceeds the marginal benefit. B) expected total return from R&D is at a maximum. C) interest rate cost of funds is negative. D) marginal benefit of R&D exceeds the marginal cost. Answer: D

Use the following to answer questions 49-53:

Type: G Topic: 5 E: 490 MI: 246 49. In the above diagram, (1) is the: A) expected-rate-of-return curve and (2) is the average total cost curve. B) total revenue curve and (2) is the interest-rate-cost-of funds-curve. C) expected-rate-of-return curve and (2) is the interest-rate-cost-funds curve. D) marginal cost curve and (2) is the marginal benefit curve. Answer: C

Type: G Topic: 5 E: 490 MI: 246 50. In the above diagram, the optimal amount of R&D is: A) $20 million. B) $80 million. C) $40 million. D) $60 million. Answer: C

McConnell/Brue: Economics, 16/e

Page 827

Chapter 26: Technology, R&D, and Efficiency

Type: G Topic: 5 E: 490 MI: 246 51. In the above diagram, at $10 million of R&D expenditure the: A) expected rate of return exceeds the interest rate cost of funds. B) firm is spending an optimal amount on R&D. C) interest rate cost of funds exceeds the expected rate of return. D) marginal benefit of R&D is less than the marginal cost of R&D. Answer: A

Type: G Topic: 5 E: 490 MI: 246 52. In the above diagram, at $60 million of R&D expenditure the: A) expected rate of return exceeds the interest rate cost of funds. B) firm is spending an optimal amount on R&D. C) interest rate cost of funds exceeds the expected rate of return. D) expected rate of return on R&D is negative. Answer: C

Type: G Topic: 5 E: 490 MI: 246 53. In the above diagram, at $40 million of R&D expenditure, the expected rate of return: A) equals the interest rate cost of funds. C) is less than the interest rate cost of funds. B) is greater than the interest rate cost of funds. D) is negative. Answer: A

Type: C Topic: 5 E: 490 MI: 246 Status: New 54. Assume that a firm's interest-rate-cost of funds curve for R&D is perfectly elastic. Which of the following would increase a firm's optimal R&D expenditures and, in equilibrium, reduce the expected rate of return on the last dollar of R&D? A) a rightward shift of the expected-rate-of-return curve B) an upward shift of the interest-rate-cost of funds curve C) a leftward shift of the expected-rate-of-return curve D) a downward shift of the interest-rate-cost of funds curve Answer: D

Type: C Topic: 5 E: 490 MI: 246 Status: New 55. Assume that a firm's interest-rate-cost of funds curve for R&D is perfectly elastic. Which of the following would decrease a firm's optimal R&D expenditures and, in equilibrium, increase the expected rate of return on the last dollar of R&D? A) a rightward shift of the expected-rate-of-return curve B) an upward shift of the interest-rate-cost of funds curve C) a leftward shift of the expected-rate-of-return curve D) a downward shift of the interest-rate-cost of funds curve Answer: B

McConnell/Brue: Economics, 16/e

Page 828

Chapter 26: Technology, R&D, and Efficiency

Type: C Topic: 5 E: 490 MI: 246 Status: New 56. Assume that a firm's interest-rate-cost of funds curve for R&D is perfectly elastic. Which of the following would increase a firm's optimal R&D expenditures and, in equilibrium, leave the expected rate of return on R&D unchanged? A) a rightward shift of the expected-rate-of-return curve B) an upward shift of the interest-rate-cost of funds curve C) a leftward shift of the expected-rate-of-return curve D) a downward shift of the interest-rate-cost of funds curve Answer: A

Type: C Topic: 5 E: 490 MI: 246 Status: New 57. Assume that a firm's interest-rate-cost of funds curve for R&D is perfectly elastic. Which of the following would decrease a firm's optimal R&D expenditures and, in equilibrium, leave the expected rate of return on R&D unchanged? A) a rightward shift of the expected-rate-of-return curve B) an upward shift of the interest-rate-cost of funds curve C) a leftward shift of the expected-rate-of-return curve D) a downward shift of the interest-rate-cost of funds curve Answer: C

Product innovation and revenue enhancement

Type: A Topic: 6 E: 491 MI: 247 58. A consumer will buy a new product rather than an existing product: A) when the MU/P of the new product is less than the MU/P of the existing product. B) when the substitution of the new product for the old product increases the consumer's total utility. C) only if the new product has a lower price than the existing product. D) only if the MU of the new product exceeds the MU of the existing product. Answer: B

Type: A Topic: 6 E: 491 MI: 247 59. We know with certainty that a consumer will buy a newly introduced product rather than an existing product when the: A) MU/P of the new product exceeds the MU/P of the existing product. B) price of the new product is less than the price of the existing product. C) MU of the new product is more than the MU of the existing product. D) law of diminishing marginal utility applies to the existing product. Answer: A

McConnell/Brue: Economics, 16/e

Page 829

Chapter 26: Technology, R&D, and Efficiency

Use the following to answer questions 60-63: Consumer's income = $12

Units of product First Second Third Fourth Fifth Sixth Seventh

Marginal utility, X (price =$1) 12 10 8 6 4 2 0

Marginal utility, Y (price = $1) 16 14 12 10 8 6 4

Marginal utility, new product, Z (price = $1) 20 18 16 14 12 10 8

Type: T Topic: 6 E: 491 MI: 247 60. Refer to the above data for a utility-maximizing consumer. Assume that new product Z doesn't exist. How many units of X and Y will this consumer buy, given his or her $12 budget? A) 5 of X and 7 of Y. B) 7 of X and 5 of Y. C) 6 of X and 6 of Y. D) 5 of X and 6 of Y. Answer: A

Type: T Topic: 6 E: 491 MI: 247 61. Refer to the above data. Assume new product Z is introduced. How many units of Z will this consumer buy, given his or her $12 budget? A) zero units B) 2 unit. C) 4 units. D) 6 units. Answer: D

Type: T Topic: 6 E: 491 MI: 247 62. Refer to the above data. In equilibrium, the introduction of new product Z has increased this consumer's total utility by: A) 42 utils. B) 54 utils. C) 60 utils. D) 66 utils. Answer: C

Type: T Topic: 6 E: 491 MI: 247 63. Refer to the above data. Suppose the price of new product Z is $10 rather than $1. This consumer would purchase: A) some of Z but not as much as if the price were $1. B) none of Z. C) less of X, Y, and Z than if the price were $1. D) more of X, Y, and Z than if the price were $1. Answer: B

Type: A Topic: 6 E: 491 MI: 247 64. The commercial success of a new product depends on: A) its price only. C) both its price and its marginal utility. B) its marginal utility only. D) neither its price nor its marginal utility. Answer: C

McConnell/Brue: Economics, 16/e

Page 830

Chapter 26: Technology, R&D, and Efficiency

Type: A Topic: 6 E: 491 MI: 247 65. For a new product to be profitable, it must: A) enable customers to obtain greater total utility from their money income. B) be less expensive than existing substitute products. C) have greater marginal utility than existing substitute products. D) embody process innovation. Answer: A

Type: A Topic: 6 E: 491 MI: 247 66. Suppose that a firm introduces a highly profitable new product. If this new product is priced higher than existing substitute products, then the: A) new product has greater marginal utility than the existing products. B) laws of economics have been violated. C) new product must have increasing, not diminishing, marginal utility. D) existing products were unprofitable to produce. Answer: A

Type: A Topic: 6 E: 491 MI: 247 67. Suppose that a firm introduces a highly profitable new product. If this new product offers less marginal utility per unit to consumers than existing substitute products, then the: A) laws of economics have been violated. B) new product must have increasing, not diminishing, marginal utility. C) existing products were being produced at a loss. D) new product has a lower price than the existing substitute products. Answer: D

Process innovation and cost reduction

Type: D Topic: 7 E: 492 MI: 248 68. Process innovation refers to: A) development of new products. B) implementation of better methods of producing products. C) first discovery of new scientific principles. D) wide-spread imitation of innovations. Answer: B

Type: A Topic: 7 E: 492 MI: 248 69. Firm ABC designs and implements a lower-cost method of producing its product. This is an example of: A) product innovation. B) the inverted U-theory. C) economies of scale. D) process innovation. Answer: D

Type: A Topic: 7 E: 492-493 MI: 248-249 70. Process innovation causes an upward shift in a firm's total product curve and: A) a decrease in its average product. C) an upward shift in its average total cost curve. B) a downward shift in its average total cost curve. D) an upward shift in its marginal revenue curve. Answer: B

McConnell/Brue: Economics, 16/e

Page 831

Chapter 26: Technology, R&D, and Efficiency

Type: A Topic: 7 E: 492 MI: 248 71. Process innovation can be depicted as: A) an upward shift in a firm's total product curve. B) an upward shift in a firm's marginal cost curve. C) a downward shift in a firm's marginal revenue curve. D) an increase in product demand. Answer: A

Use the following to answer questions 72-73:

Type: G Topic: 7 E: 492-493 MI: 248-249 72. Refer to the above diagram which relates to Firm A. Which of the following would shift A's average total cost curve from ATC1 to ATC2? A) an increase in the price of a key component used by A in producing its product B) a decrease in the incomes of A's customers C) a move along A's total product curve (not shown) D) an improved production method that shifts A's total product curve upward Answer: D

Type: G Topic: 7 E: 492-493 MI: 248-249 73. Refer to the above diagram which relates to Firm A. Which of the following would shift A's average total cost curve from ATC1 to ATC2? A) replacement of old equipment with new, more productive equipment embodying technological advance. B) a decrease in the incomes of A's customers C) a move along A's total product curve (not shown) D) the increase in the price of one of the major inputs used to produce A's product. Answer: A

McConnell/Brue: Economics, 16/e

Page 832

Chapter 26: Technology, R&D, and Efficiency

Use the following to answer questions 74-75:

Type: G Topic: 7 E: 492-493 MI: 248-249 74. Refer to the above diagram that relates to Firm X. Which of the following would illustrate process innovation by X? A) a downshift in the total product curve from TP1 to TP2 B) an upshift in the total product curve from TP2 to TP1 C) a move from a to b on TP1 D) a move from c to d on TP2 Answer: B

Type: G Topic: 7 E: 492-493 MI: 248-249 75. Refer to the above diagram that relates to Firm X. Suppose X implements an innovative new production method that shifts its total product curve from TP2 to TP1. Other things equal: A) the average product of X's labor would fall. B) the average total cost of X's output would decline. C) X would supply less output at each product price than before. D) the demand curve for X's product would shift to the right. Answer: B

Imitation problem: "fast-second strategy"

Type: A Topic: 8 E: 493 MI: 249 76. As it relates to R&D, the imitation problem is that: A) patents, copyrights, and trademarks hinder imitation and thus limit economically desirable diffusion. B) brand names create entry barriers for would-be competitors. C) diffusion of innovation occurs more slowly than is desirable from society's perspective. D) a firm's rivals may be able to copy its new product or process innovation, reducing its return on R&D. Answer: D

Type: A Topic: 8 E: 493 MI: 249 77. Gigantic Corporation follows a strategy of waiting for rivals to innovate, then quickly imitating any successful innovations. This behavior is known as: A) collusion. C) a fast-second strategy. B) an entrepreneurial strategy. D) pricing the demand curve. Answer: C

McConnell/Brue: Economics, 16/e

Page 833

Chapter 26: Technology, R&D, and Efficiency

Type: A Topic: 8 E: 493 MI: 249 78. Fast-second strategies are more likely to be used by: A) dominant firms than by startup firms. C) startup firms rather than existing firms. B) pure competitors rather than oligopolists. D) entrepreneurs than by corporations. Answer: A

Type: A Topic: 8 E: 493 MI: 249 79. Other things equal, the prospect of imitation by others: A) decreases the expected rate of return on R&D expenditures. B) increases the expected rate of return on R&D expenditures. C) increases the interest-rate cost of funds used to finance R&D expenditures. D) decreases the interest-rate cost of funds used to finance R&D expenditures. Answer: A

Type: A Topic: 8 E: 493 MI: 249 80. All of the following increase the expected rate of return on R&D expenditures, except: A) patents. B) trademarks. C) imitation by others. D) trade secrets. Answer: C

Benefits of being first with innovation

Type: D Topic: 9 E: 493 MI: 249 Status: New 81. Legal protections against competitors producing and selling a product identical to the one you invented are called _________; legal protections against competitors using your product's name are called __________. A) patents; trademarks B) trademarks, copyrights C) copyrights, patents D) trademarks, patents Answer: A

Type: A Topic: 9 E: 493 MI: 249 Status: New 82. Suppose that Marlen Fisher has legal protection against anyone producing and selling a fishing lure identical to his unique-action "MarFish" lure, whatever the competitor might name the lure. This legal protection is most likely to be a: A) trademark. B) restraining order. C) patent. D) copyright. Answer: C

Type: A Topic: 9 E: 493 MI: 249 Status: New 83. Suppose that Marlen Fisher has legal protection against anyone producing and selling a fishing lure specifically named "MarFish." This legal protection is most likely to be a: A) trademark. B) restraining order. C) patent. D) copyright. Answer: A

Type: A Topic: 9 E: 493 MI: 249 Status: New 84. Suppose that Book-Cost Busters (TCB), without authorization, reproduced a best-selling novel and placed it for downloading on the TCB pay-for-use website. This action would violate the publisher's: A) profit rights. B) patent. C) copyright. D) trademark. Answer: C

McConnell/Brue: Economics, 16/e

Page 834

Chapter 26: Technology, R&D, and Efficiency

Type: A Topic: 9 E: 493 MI: 249 85. Other things equal, patents: A) decrease the expected rate of return on a R&D expenditure. B) increase the expected rate of return on a R&D expenditure. C) increase the interest-rate cost of funds used to finance a R&D expenditure. D) decrease the interest-rate cost of funds used to finance a R&D expenditure. Answer: B

Type: A Topic: 9 E: 493 MI: 249 86. Other things equal, trademarks and brand names: A) increase the interest-rate cost of funds used to finance R&D expenditures. B) decrease the interest-rate cost of funds used to finance R&D expenditures. C) decrease the expected rate of return on R&D expenditures. D) increase the expected rate of return on R&D expenditures. Answer: D

Type: C Topic: 9 E: 493 MI: 249 87. Suppose that a firm's legal staff concludes that a new product that a firm is developing is patentable. Graphically, this new information would shift the firm's expected rate of return curve on R&D to the: A) right and reduce its optimal amount of R&D. C) left and increase its optimal amount of R&D. B) right and increase its optimal amount of R&D. D) left and reduce its optimal amount of R&D. Answer: B

Type: C Topic: 9 E: 490 MI: 246 88. Suppose that a firm's legal staff concludes that a new product which a firm is developing is patentable. Graphically, this new information would shift the firm's expected rate of return curve on R&D to the: A) right and reduce its optimal amount of R&D. C) left and increase its optimal amount of R&D. B) right and increase its optimal amount of R&D. D) left and reduce its optimal amount of R&D. Answer: B

Type: A Topic: 9 E: 493 MI: 249 89. A patent on a new product benefits the firm securing it by: A) limiting the direct imitation of the product by rivals for many years. B) enabling the firm to retain "trade secrets" about the product. C) reducing the firm's legal expenses. D) increasing the speed of diffusion of the new product. Answer: A

Type: D Topic: 9 E: 493 MI: 249 90. Legal protections against direct copying of written material are called __________; legal protections against using a product's name are called __________. A) patents; trademarks. B) trademarks, patents. C) copyrights, trademarks. D) copyrights; patents. Answer: C

McConnell/Brue: Economics, 16/e

Page 835

Chapter 26: Technology, R&D, and Efficiency

Type: A Topic: 9 E: 493-494 MI: 249-250 91. Even where imitation is possible, a firm may gain advantage from being the first to introduce an innovative product because of: A) long-lasting brand-name recognition. B) a time lag between innovation and imitation by rivals. C) trade secrets that limit the ability of rivals to imitate the product. D) all of the above. Answer: D

Type: A Topic: 9 E: 493-494 MI: 249-250 92. Firm X develops a new product and gets a head start in its production. Other firms try to produce a similar product but discover they have higher average total costs than the existing firm. This situation illustrates: A) diseconomies of scale. C) learning-by-doing. B) diminishing marginal returns. D) spillover costs. Answer: C

Type: A Topic: 9 E: 494 MI: 250 93. Small innovative firm X that has developed a new product accepts an offer to be acquired by a larger firm. As it relates to new technology, this behavior or outcome is known as a: A) fast-second strategy. B) limit pricing. C) zero-sum game. D) a buyout. Answer: D

Role of market structure

Type: A Topic: 10 E: 495-496 MI: 251-252 94. Which of the following supports of the contention that pure competitors have a weak incentive to engage in R&D? A) Entry to purely competitive industries is easy and thus profit from innovation is quickly competed away. B) In pure competition, products are already highly differentiated. C) Most purely competitive industries are increasing-cost industries. D) Pure competitors are happy to earn only a normal profit. Answer: A

Type: A Topic: 10 E: 495-496 MI: 251-252 95. Which of the following supports the contention that pure competitors have a strong incentive to engage in R&D? A) Entry to purely competitive industries is easy and thus profit from innovation is quickly competed away. B) Pure competitors cannot risk being complacent about innovation, since a new product, production technique, or distribution method could undermine their normal profit and drive them out of the market. C) Most purely competitive industries are increasing-cost industries. D) Pure competitors are happy to earn only a normal profit. Answer: B

McConnell/Brue: Economics, 16/e

Page 836

Chapter 26: Technology, R&D, and Efficiency

Type: A Topic: 10 E: 496 MI: 252 96. Which of the following supports the contention that monopolistic competitors have a strong incentive to engage in R&D? A) Entry to monopolistic competitive industries is relatively easy and thus profit from innovation is quickly competed away. B) Most monopolistic competitive industries are decreasing cost industries. C) The desire to differentiate products from competitors may motivate monopolistic competitors to engage in R&D. D) Monopolistic competitors have large retained earnings that are available to finance R&D. Answer: C

Type: A Topic: 10 E: 496 MI: 252 97. Which pair of market structures provide firms with the greatest ability to finance R&D out of retained earnings? A) oligopolists and pure monopolists. B) pure competitors and pure monopolists. C) pure competitors and monopolistic competitors. D) monopolistic competitors and pure monopolists. Answer: A

Type: A Topic: 10 E: 496 MI: 252 98. Economists who contend that oligopolists have a strong incentive to engage in R&D say that: A) the undistributed profits of oligopolists give them a source of readily available, relatively low cost funds for financing R &D. B) entry barriers enable oligopolists to sustain the profit it gains from innovation. C) the large size of oligopolists' R&D departments allow them to use very specialized, expensive R&D equipment and employ teams of specialized researchers. D) all of the above are true. Answer: D

Type: A Topic: 10 E: 496 MI: 252 99. Those who contend that oligopolists are less likely than more competitive firms to engage in R&D say that: A) Oligopolists have little incentive to introduce costly new technology and produce new products when they currently are earning large economic profit using existing technology and selling existing products. B) the undistributed profits of oligopolists give them a source of readily available, relatively low cost funds for financing R &D. C) entry barriers enable oligopolists to sustain the profits they gain from innovation. D) the large size of oligopolists' R&D departments allow them to use very specialized, expensive R&D equipment and employ teams of specialized researchers. Answer: A

Type: A Topic: 10 E: 496 MI: 252 100. Which of the following market structures do economists generally agree is the least conducive to R&D and innovation? A) monopolistic competition B) pure competition C) pure monopoly D) oligopoly Answer: C

McConnell/Brue: Economics, 16/e

Page 837

Chapter 26: Technology, R&D, and Efficiency

Inverted-U theory; role of scientific opportunity

Type: A Topic: 11 E: 497-498 MI: 253-254 101. In the inverted-U theory: A) process innovation and product innovation are inversely related. B) technological change is inversely related to scientific discovery. C) R&D expenditures rise continuously as a percentage of firms' sales as industry concentration rises. D) R&D expenditures first rise as a percentage of firms' sales as industry concentration increases, but then fall as higher industry concentration occurs. Answer: D

Type: D Topic: 11 E: 497-498 MI: 253-254 102. The conjecture that R&D expenditures as a percentage of firms' sales first rise, reach a peak, and then fall as industry concentration rises is known as the. A) inverted-U theory. C) bell-shaped curve. B) average product of R&D theory. D) theory of increasing and diminishing returns. Answer: A

Type: A Topic: 11 E: 498 MI: 254 103. Which of the following ideas most directly relates to R&D expenditures? A) increasing cost industries. B) inverted-U theory. C) a cartel. D) kinked-demand curve. Answer: B

Type: C Topic: 11 E: 497 MI: 253 104. In the inverted-U theory, which of the following industry concentration ratios would be most conducive to R&D (as a percentage of firm sales)? A) 1 percent. B) 10 percent. C) 50 percent. D) 70 percent. Answer: C

Type: C Topic: 11 E: 497 MI: 253 105. Industry A has a 60 percent concentration ratio, while industry B has a 40 percent concentration ratio. According to the inverted-U theory, all else equal, we can conclude that: A) Industry A will be more technologically progressive than B. B) Industry C with a 10 percent concentration ratio will be more progressive than either industry A or B. C) Industry D with a 80 percent concentration ratio will be more technologically progressive than either industry A or B. D) that none of the above are necessarily true. Answer: D

Type: A Topic: 11 E: 497 MI: 253 106. Which among the following is the strongest determinant of an industry's technological progressiveness? A) the scientific character of its industry and the number of technological opportunities available B) the size of the industry concentration ratio; the lower the ratio, the greater the firm's technological progressiveness C) the Herfindahl index in the firm's industry; the higher the index value, the greater the firm's technological progressiveness D) the amount of retained earnings in the industry Answer: A

McConnell/Brue: Economics, 16/e

Page 838

Chapter 26: Technology, R&D, and Efficiency

Type: A Topic: 11 E: 497 MI: 253 107. In general, the: A) number of firms in the industry is far more important than the industry's scientific character and extent of technological opportunities. B) greater is an industry's concentration ratio, the higher its R&D expenditures in relation to sales. C) industry's scientific character and extent of technological opportunities often is more important than the industry's concentration ratio. D) higher the industry's interest cost of borrowing funds for R&D, the greater is the industry's progressiveness. Answer: C

Technological advance and efficiency

Type: A Topic: 12 E: 498 MI: 254 108. Technological advance improves productive efficiency by: A) lowering average total cost. C) enhancing monopoly power. B) increasing marginal utility. D) decreasing a nation's exports. Answer: A

Type: A Topic: 12 E: 498 MI: 254 109. Technological advance improves allocative efficiency by: A) enhancing monopoly power. B) reducing income inequality. C) giving society a more-preferred mix of goods and services. D) encouraging saving. Answer: C

Type: D Topic: 12 E: 499 MI: 255 110. The process by which new firms and new products replace existing dominant firms and products is called: A) monopolistic competition. C) process innovation. B) the inverted-U process. D) creative destruction. Answer: D

Type: D Topic: 12 E: 499-500 MI: 255-256 Status: New 111. Creative destruction is: A) the process by which large firms buy up small firms. B) the process by which new firms and new products replace existing dominant firms and products. C) a term coined many years ago by Adam Smith. D) is applicable to planned economies, but not to market economies. Answer: B

Type: F Topic: 12 E: 499 MI: 255 112. The theory of creative destruction was advanced many years ago by: A) Rocky Balboa. B) John Maynard Keynes. C) Joseph Schumpeter. Answer: C

D) Adam Smith.

McConnell/Brue: Economics, 16/e

Page 839

Chapter 26: Technology, R&D, and Efficiency

Type: A Topic: 12 E: 500 MI: 256 113. Creative destruction is not automatic because: A) there are major obstacles to the entry of new innovative firms into concentrated industries. B) consumer tastes are highly unstable. C) corporate takeovers increase dynamic competition. D) large firms rarely are technologically progressive. Answer: A

Consider This Questions

Type: D E: 494 MI: 250 Status: New 114. (Consider This) The central idea illustrated by the vignette on "catgut" used as violin strings is: A) patent rights. B) research and development activity. C) derived demand. D) trade secrets. Answer: D

Type: D E: 494 MI: 250 Status: New 115. (Consider This) Violin strings made from sheep intestines were first called "catgut" because: A) at the time it was thought to be extremely unlucky to kill sheep. B) the inventor wanted to establish a legally protected brand name. C) the inventor wanted to preserve his trade secret. D) the inventor thought that "catgut" would sound less offensive to buyers than "sheep intestines." Answer: C

Last Word Questions

Type: A E: 498-499 MI: 254-255 116. (Last Word) In 1981, IBM introduced its version of the personal computer to compete with existing personal computers offered by Apple and others. IBM's action best exemplifies: A) invention. B) scientific determinism. C) diffusion. D) technological lag. Answer: C

Type: A E: 498-499 MI: 254-255 117. (Last Word) Construction of the first working computer "mouse" best exemplifies: A) invention. B) scientific determinism. C) diffusion. D) creative destruction. Answer: A

Type: A E: 498-499 MI: 254-255 118. (Last Word) In 1995 Microsoft released its Windows 95 operating system which was a far better system than the Microsoft system it replaced. Microsoft's Window 95 best illustrates: A) innovation. B) scientific determinism. C) venture capitalism. D) creative destruction. Answer: A

McConnell/Brue: Economics, 16/e

Page 840

Chapter 26: Technology, R&D, and Efficiency

True/False Questions

Type: D E: 484 MI: 240 119. Technological advance includes new and improved products, new and improved production techniques, and new and improved distribution methods Answer: True

Type: D E: 485 MI: 241 120. Innovation is the first discovery of a new product or production process; invention is the first successful commercial introduction of the product or process. Answer: False

Type: D E: 485 MI: 241 121. Diffusion is the first successful commercial introduction of a product, the use of a new method, or the creation of new form of business enterprise. Answer: False

Type: D E: 488 MI: 244 122. Venture capital is another name for retained earnings. Answer: False

Type: A E: 488 MI: 244 123. The marginal cost to a firm of R&D expenditures is the market interest rate the firm must pay to obtain the needed financing. Answer: True

Type: A E: 489 MI: 245 124. A firm should undertake all the R&D activity that it can afford. Answer: False

Type: A E: 489 MI: 245 125. A firm's optimal amount of R&D occurs where the marginal benefit of this activity exceeds marginal cost by the greatest amount. Answer: False

Type: A E: 490 MI: 246 126. A firm's optimal amount of R&D occurs where the interest-rate cost of funds and the expected rate of return are equal. Answer: True

Type: A E: 491 MI: 247 127. Successful new products enable consumers to increase the total utility they obtain from a specific amount of their total spending. Answer: True

McConnell/Brue: Economics, 16/e

Page 841

Chapter 26: Technology, R&D, and Efficiency

Type: A E: 491-492 MI: 247-248 128. Process innovation is represented as a downshift in a firm's total product curve and its average total cost curve. Answer: False

Type: D E: 496-497 MI: 252-253 129. The theory that R&D expenditures as a percentage of firms' sales first rise, reach a peak, and then fall with increases in industry concentration is called the inverted-U theory. Answer: True

Type: D E: 499 MI: 255 130. The process by which new firms and new products destroy existing dominant firms and their products is called creative destruction. Answer: True

McConnell/Brue: Economics, 16/e

Page 842

CHAPTER 27

The Demand for Resources

Topic 1. 2. 3. 4. 5. 6. Derived demand Resource demand curve; optimal hiring Determinants of resource demand Elasticity of resource demand Optimal combination of resources Marginal productivity theory of income distribution Consider This Last Word True-False

Question numbers 1-8 9-59 60-97 98-114 115-145 146-149 150-151 152-154 155-170

____________________________________________________________

_______________________________________

____________________________________________________________

_______________________________________

Multiple Choice Questions Derived demand

Type: F Topic: 1 E: 504-505 MI: 260-261 1. Resource pricing is important because: A) resource prices are a major determinant of money incomes. B) resource prices allocate scarce resources among alternative uses. C) resource prices, along with resource productivity, are important to firms in minimizing their costs. D) of all of the above reasons. Answer: D

Type: A Topic: 1 E: 505 MI: 261 2. Which of the following statements best illustrates the concept of derived demand? A) As income goes up the demand for farm products will increase by a smaller relative amount. B) A decline in the price of margarine will reduce the demand for butter. C) A decline in the demand for shoes will cause the demand for leather to decline. D) When the price of gasoline goes up, the demand for motor oil will decline. Answer: C

Type: D Topic: 1 E: 505 MI: 261 3. When economists say that the demand for labor is a derived demand, they mean that it is: A) dependent on government expenditures for public goods and services. B) related to the demand for the product or service labor is producing. C) based on the desire of businesses to exploit labor by paying below equilibrium wage rates. D) based on the assumption that workers are trying to maximize their money incomes. Answer: B

Chapter 27: The Demand for Resources

Type: A Topic: 1 E: 505 MI: 261 4. The demand for airline pilots results from the demand for air travel. This fact is an example of: A) resource substitutability. C) elasticity of resource demand. B) rising marginal resource cost. D) the derived demand for labor. Answer: D

Type: D Topic: 1 E: 505 MI: 261 5. We say that the demand for labor is a derived demand because: A) labor is a necessary input in the production of every good or service. B) we demand the product that labor helps produce rather than labor service per se. C) the forces of supply and demand do not apply directly to labor markets. D) labor is hired using the MRP = MRC rule. Answer: B

Type: A Topic: 1 E: 505 MI: 261 6. The demand for a resource depends primarily on: A) the supply of that resource. B) the demand for the product or service that it helps produce. C) the price of that input. D) the elasticity of supply of substitute inputs. Answer: B

Type: A Topic: 1 E: 505 MI: 261 7. The demand for labor is derived from: A) the demands for other variable inputs. B) consumer demand for the product or service it is helping to produce. C) the cost-minimization rule. D) the supply of related inputs. Answer: B

Type: A Topic: 1 E: 505 MI: 261 8. In the United States professional football players earn much higher incomes than professional soccer players. This occurs because: A) most football players are good soccer players while the reverse is not true. B) consumers have a greater demand for football games than for soccer games. C) football and soccer games are highly substitutable products for most consumers. D) the marginal productivity of soccer players exceeds that of football players. Answer: B

Resource demand curve; optimal hiring

Type: D Topic: 2 E: 505 MI: 261 9. Marginal revenue product measures the: A) amount by which the extra production of one more worker increases a firm's total revenue. B) decline in product price that a firm must accept to sell the extra output of one more worker. C) increase in total resource cost resulting from the hire of one extra unit of a resource. D) increase in total revenue resulting from the production of one more unit of a product. Answer: A

McConnell/Brue: Economics, 16/e

Page 844

Chapter 27: The Demand for Resources

Type: A Topic: 2 E: 506-507 MI: 262-263 10. The marginal revenue product schedule is: A) the same whether the firm is selling in a purely competitive or imperfectly competitive market. B) the firm's resource demand schedule. C) the firm's resource supply schedule. D) upsloping. Answer: B

Type: A Topic: 2 E: 507 MI: 263 11. The purely competitive employer of resource A will maximize the profits from A by equating the: A) price of A with the MRP of A. C) marginal productivity of A with the price of A. B) marginal productivity of A with the MRC of A. D) price of A with the MRC of A. Answer: A

Type: A Topic: 2 E: 506-507 MI: 262-263 12. The MRP curve for labor: A) intersects the firm's labor demand curve from above. B) is the firm's labor demand curve. C) lies below the firm's labor demand curve. D) lies above the firm's labor demand curve. Answer: B

Type: D Topic: 2 E: 506 MI: 262 13. Marginal product is: A) the output of the least skilled worker. B) the amount an additional worker adds to the firm's total output. C) a worker's output multiplied by the price at which each unit can be sold. D) the amount any given worker contributes to the firm's total revenue. Answer: B

Type: A Topic: 2 E: 506-507 MI: 262-263 14. The labor demand curve of a purely competitive seller: A) slopes downward because the elasticity of demand is always less than unity. B) slopes downward because of diminishing marginal productivity. C) is perfectly elastic at the going wage rate. D) slopes downward because of diminishing marginal utility. Answer: B

Type: A Topic: 2 E: 506 MI: 262 15. Assume labor is the only variable input and that an additional input of labor increases total output from 72 to 78 units. If the product sells for $6 per unit in a purely competitive market, the MRP of this additional worker is: A) $6. B) $12. C) $36. D) $72. Answer: C

McConnell/Brue: Economics, 16/e

Page 845

Chapter 27: The Demand for Resources

Type: A Topic: 2 E: 506 MI: 262 16. If one worker can pick $30 worth of grapes and two workers together can pick $50 worth of grapes, the: A) marginal revenue product of each worker is $25. B) marginal revenue product of the first worker is $20. C) marginal revenue product of the second worker is $20. D) data given do not permit the determination of the marginal revenue product of either worker. Answer: C

Type: A Topic: 2 E: 506-507 MI: 262-263 17. A competitive employer should hire additional labor as long as: A) the MRP exceeds the wage rate. C) average product exceeds MP. B) the wage rate is less than MP. D) MC exceeds MR. Answer: A

Type: A Topic: 2 E: 506 MI: 262 18. A firm will find it profitable to hire workers up to the point at which their: A) marginal resource cost equals their wage rate. C) MP is equal to their MRP. B) wage rate equals product price. D) marginal resource cost is equal to their MRP. Answer: D

Type: A Topic: 2 E: 506 MI: 262 19. A profit-maximizing firm employs resources to the point where: A) MRC = MP. B) Resource price equals product price. C) MRP = MRC. D) MP = product price. Answer: C

Use the following to answer questions 20-22: Answer the next question(s) on the basis of the following information: Jones owns a barber shop and charges $6 per haircut. By hiring one barber at $10 per hour the shop can provide 24 haircuts per 8-hour day. By hiring a second barber at the same wage rate the shop can now provide a total of 42 haircuts per day.

Type: A Topic: 2 E: 506 MI: 262 20. Refer to the above information. The MP of the second barber is: A) $240. B) $108. C) 18. D) 42. Answer: C

Type: A Topic: 2 E: 506 MI: 262 21. Refer to the above information. The MRP of the second barber: A) is 18. B) is $108. C) is 42. D) is $24. Answer: B

Type: A Topic: 2 E: 506 MI: 262 22. Refer to the above information. Jones should: A) hire the second barber because he will add $28 to profits. B) hire the second barber because he will add $108 to profits. C) not hire the second barber because he is less productive than the first barber. D) not hire the second barber because he will diminish profits. Answer: A

McConnell/Brue: Economics, 16/e

Page 846

Chapter 27: The Demand for Resources

Type: A Topic: 2 E: 506-507 MI: 262-263 23. The labor demand schedule is identical with the: A) marginal product schedule. B) marginal resource cost schedule. Answer: C

C) marginal revenue product schedule. D) product demand schedule.

Type: A Topic: 2 E: 507 MI: 263 24. Assume the Apex Manufacturing company is purely competitive in both the hire of labor and in the sale of its product. Apex's labor demand curve would be: A) vertical at the current level of employment. C) upward sloping. B) horizontal at the "going" wage rate. D) downward sloping. Answer: D

Type: A Topic: 2 E: 506-507 MI: 262-263 25. The general rule for hiring any input (say, labor) in the profit-maximizing amount is MRC = MRP. This rule takes the special form W = MRP (where W is the wage rate) when the: A) labor supply curve is upsloping. B) supply of labor is inelastic. C) firm is hiring labor under purely competitive conditions. D) firm is hiring labor under imperfectly competitive conditions. Answer: C

Use the following to answer questions 26-30: Answer the next question(s) on the basis of the following information for Manfred's Shoe Shine Parlor. Assume Manfred hires labor, its only variable input, under purely competitive conditions. Shoe shines are also sold competitively.
Units of labor 0 1 2 3 4 5 6 7 Total product 0 14 30 35 39 44 2 Marginal product 14 10 Total revenue $ 42 90 117 126 132

Type: T Topic: 2 E: 506 MI: 262 26. Refer to the above data. How many units of output are produced when 2 workers are employed? A) 4 B) 16 C) 24 D) 10 Answer: C

Type: T Topic: 2 E: 506 MI: 262 27. Refer to the above data. What is the marginal product of the sixth worker? A) 2 units B) 3 units C) 4 units D) 5 units Answer: B

McConnell/Brue: Economics, 16/e

Page 847

Chapter 27: The Demand for Resources

Type: T Topic: 2 E: 506-507 MI: 262-263 28. Refer to the above data. At what price does each shoe shine sell? A) $1 B) $2 C) $3 D) $2.50 Answer: C

Type: T Topic: 2 E: 506-507 MI: 262-263 29. Refer to the above data. If the wage rate is $11, how many workers will Manfred hire to maximize profits? A) 1 B) 2 C) 3 D) 5 Answer: D

Type: T Topic: 2 E: 507 MI: 263 30. Refer to the above data. If Manfred's only fixed input is capital, the total cost of which is $30, then what will be his economic profit? A) $62 B) $42 C) $28 D) $32 Answer: D

Type: A Topic: 2 E: 506-507 MI: 262-263 31. Assume that a restaurant is hiring labor in an amount such that the MRC of the last worker is $16 and her MRP is $12. On the basis of this information we can say that: A) profits will be increased by hiring additional workers. B) profits will be increased by hiring fewer workers. C) marginal revenue product must exceed average revenue product. D) the restaurant is maximizing profits. Answer: B

Use the following to answer questions 32-35: Answer the next question(s) on the basis of the information given in the following table:
Employment 0 1 2 3 4 5 6 Total product 0 12 22 30 36 40 42 Product price $3 3 3 3 3 3 3

Type: T Topic: 2 E: 506 MI: 262 32. Refer to the above data. This firm is: A) selling its product in a purely competitive market. B) selling its product in an imperfectly competitive market. C) hiring workers in a purely competitive market. D) hiring workers in an imperfectly competitive market. Answer: A

McConnell/Brue: Economics, 16/e

Page 848

Chapter 27: The Demand for Resources

Type: T Topic: 2 E: 506-507 MI: 262-263 33. Refer to the above data. If the firm is hiring workers under purely competitive conditions at a wage rate of $22, it will employ: A) 1 worker. B) 2 workers. C) 3 workers. D) 4 workers. Answer: C

Type: T Topic: 2 E: 507 MI: 263 34. Refer to the above data. If the firm is hiring workers under purely competitive conditions at a wage rate of $10, it will employ: A) 2 workers. B) 3 workers. C) 4 workers. D) 5 workers. Answer: D

Type: T Topic: 2 E: 507 MI: 263 35. Refer to the above data. Which of the following best represents the labor demand schedule for this firm?
(a) WR $35 29 23 17 (b) WR $35 29 23 17 (c) WR $35 29 23 17 (d) WR $40 35 30 25

Qd 1 2 3 4

Qd 2 3 4 5

Qd 3 4 5 6

Qd 1 2 3 4

Answer: A

Type: A Topic: 2 E: 507 MI: 263 36. Assuming a firm is selling its output in a purely competitive market, its resource demand curve can be determined by: A) multiplying total product by product price. B) multiplying marginal product by product price. C) dividing total revenue by marginal product. D) comparing marginal product with various possible input prices. Answer: B

Type: D Topic: 2 E: 506 MI: 262 37. Marginal resource cost is: A) the increase in total resource cost associated with the production of one more unit of output. B) the increase in total resource cost associated with the hire of one more unit of the resource. C) total resource cost divided by the number of inputs employed. D) the change in total revenue associated with the employment of one more unit of the resource. Answer: B

McConnell/Brue: Economics, 16/e

Page 849

Chapter 27: The Demand for Resources

Use the following to answer questions 38-40: Answer the next question(s) on the basis of the data contained in the following table. Assume that the firm is hiring labor in a purely competitive market.

Units of labor 0 1 2 3 4 5 6

Total product 0 15 28 39 48 55 60

Product price $2.20 2.00 1.80 1.60 1.40 1.20 1.10

Type: T Topic: 2 E: 506-507 MI: 262-263 38. Refer to the above data. If the wage rate is $20, how many workers will the firm choose to employ? A) 5 B) 4 C) 3 D) 2 Answer: D

Type: T Topic: 2 E: 506-507 MI: 262-263 39. Refer to the above data. If the wage rate is $11, how many workers will the firm choose to employ? A) 5 B) 4 C) 3 D) 2 Answer: C

Type: T Topic: 2 E: 507-508 MI: 263-264 40. The above data reveal that: A) the firm is selling its product in a purely competitive market. B) the firm is selling its product in an imperfectly competitive market. C) there is no level of output at which this firm can operate at a profit. D) the law of diminishing returns is not applicable to this firm. Answer: B

Type: A Topic: 2 E: 507-508 MI: 263-264 41. Other things equal, the resource demand curve of an imperfectly competitive seller will: A) lie below its marginal revenue product curve. B) be subject to increasing marginal productivity. C) be less elastic than that of a purely competitive seller. D) be more elastic than that of a purely competitive seller. Answer: C

Type: A Topic: 2 E: 507 MI: 263 42. The MRP curve is the resource demand curve for: A) neither the purely competitive nor the imperfectly competitive seller. B) the imperfectly competitive seller, but not the purely competitive seller. C) the purely competitive seller, but not the imperfectly competitive seller. D) both the purely competitive and imperfectly competitive seller. Answer: D

McConnell/Brue: Economics, 16/e

Page 850

Chapter 27: The Demand for Resources

Type: A Topic: 2 E: 507-508 MI: 263-264 43. The labor demand curve of an imperfectly competitive seller is downward sloping: A) solely because of diminishing marginal utility. B) both because of diminishing returns and the necessity to lower price to sell more output. C) solely because product price must be reduced to sell more output. D) solely because of diminishing returns. Answer: B

Type: A Topic: 2 E: 507 MI: 263 44. If a firm is selling in an imperfectly competitive product market, then: A) average product will be less than marginal product for any number of workers hired. B) the marginal products of successive workers must be sold at lower prices. C) the marginal products of successive workers can be sold at higher prices. D) the marginal products of successive workers can be sold at a constant price. Answer: B

Type: A Topic: 2 E: 507-508 MI: 263-264 45. Other things the same, we would expect the labor demand curve of a purely competitive seller to be: A) of unitary elasticity. B) more elastic than that of an imperfectly competitive seller. C) less elastic than that of an imperfectly competitive seller. D) perfectly elastic. Answer: D

Type: A Topic: 2 E: 508 MI: 264 46. Other things equal, we would expect the labor demand curve of a monopolistic seller to: A) decline more rapidly than that of a purely competitive seller. B) decline less rapidly than that of a purely competitive seller. C) decline at the same rate as that of a purely competitive seller. D) be more elastic than that of a purely competitive seller. Answer: A

Type: D Topic: 2 E: 506 MI: 262 47. The change in a firm's total revenue that results from the hire of an additional worker is measured by: A) marginal product. C) marginal revenue product. B) marginal revenue. D) average revenue product. Answer: C

Type: A Topic: 2 E: 507 MI: 263 48. The labor demand curve of a purely competitive seller: A) slopes downward because the firm must lower price to sell more output. B) slopes downward because labor productivity increases as successive workers are hired. C) is perfectly elastic because the firm is hiring an insignificant portion of the total labor supply. D) slopes downward because the marginal product of successive workers declines. Answer: D

McConnell/Brue: Economics, 16/e

Page 851

Chapter 27: The Demand for Resources

Type: A Topic: 2 E: 507 MI: 263 49. The MRP curve for labor: A) is downsloping and shows the relationship between wage rates and the quantity of labor demanded. B) is perfectly elastic if the firm is selling its output competitively. C) is upsloping and lies above the labor supply curve. D) will shift location when the wage rate changes. Answer: A

Type: A Topic: 2 E: 507 MI: 263 50. If the wage rate increases: A) a purely competitive producer will hire less labor, but an imperfectly competitive producer will not. B) an imperfectly competitive producer will hire less labor, but a purely competitive producer will not. C) a purely competitive and an imperfectly competitive producer will both hire less labor. D) an imperfectly competitive producer may find it profitable to hire either more or less labor. Answer: C

Type: A Topic: 2 E: 506 MI: 262 51. The marginal revenue product of any input is the: A) cost of an additional unit of that input. B) added profits resulting from the use of one more unit of that input. C) additional output resulting from the use of one more unit of that input. D) additional revenue resulting from the use of one more unit of that input. Answer: D

Use the following to answer questions 52-55: Answer the next question(s) on the basis of the following information. A farmer who has fixed amounts of land and capital finds that total product is 24 for the first worker hired; 32 when two workers are hired; 37 when three are hired; and 40 when four are hired. The farmer's product sells for $3 per unit and the wage rate is $13 per worker.

Type: A Topic: 2 E: 505-506 MI: 261-262 52. Refer to the above information. The marginal product of the second worker is: A) 24. B) 8. C) 5. D) 1. Answer: B

Type: A Topic: 2 E: 506 MI: 262 53. Refer to the above information. The marginal revenue product of the second worker is: A) $24. B) $8. C) $15. D) $9. Answer: A

Type: A Topic: 2 E: 507 MI: 263 54. Refer to the above information. How many workers should the farmer hire? A) 1 B) 2 C) 3 D) 4 Answer: C

McConnell/Brue: Economics, 16/e

Page 852

Chapter 27: The Demand for Resources

Type: A Topic: 2 E: 506 MI: 262 55. Refer to the above information. What is the farmer's profit-maximizing output? A) 20 B) 32 C) 37 D) 40 Answer: C

Type: A Topic: 2 E: 506 MI: 262 56. A competitive employer is using labor in such an amount that labor's MRP is $10 and its wage rate is $8. This firm: A) should hire more labor because this will increase profits. B) should hire more labor, although this may either increase or decrease profits. C) is currently hiring the profit-maximizing amount of labor. D) is selling its product in an imperfectly competitive market. Answer: A

Type: A Topic: 2 E: 506 MI: 262 57. A firm is hiring the profit-maximizing amount of an input when: A) AVC = MC. B) MP = MRC. C) MRC = MR. D) MRP = MRC. Answer: D

Type: A Topic: 2 E: 506 MI: 262 Status: New 58. For a firm selling its product in a purely competitive market, the marginal revenue product of labor can be found by: A) adding marginal product to total product as one more unit of labor is employed. B) adding marginal revenue to total product as one more unit of labor is employed. C) multiplying marginal product by product price. D) dividing marginal product by product price. Answer: C

Type: A Topic: 2 E: 506 MI: 262 Status: New 59. For a firm selling its product in an imperfectly competitive market, the marginal revenue product of labor can be found by: A) adding marginal product to total product as one more unit of labor is employed. B) adding marginal revenue to total product as one more unit of labor is employed. C) multiplying marginal product by product price. D) multiplying marginal product by marginal revenue. Answer: D

McConnell/Brue: Economics, 16/e

Page 853

Chapter 27: The Demand for Resources

Determinants of resource demand

Use the following to answer questions 60-67:

Wage rate

a b D1 D2 D3

0

Quantity of labor
Type: G Topic: 3 E: 509 MI: 265 60. Refer to the above graph. An increase in the quantity of labor demanded (as distinct from an increase in demand) is shown by the: A) shift from labor demand curve D1 to D2. C) move from a to b along labor demand curve D1. B) shift from labor demand curve D3 to D2. D) move from b to a along labor demand curve D1. Answer: C

Type: G Topic: 3 E: 509 MI: 265 61. Refer to the above graph. An increase in labor demand (as distinct from an increase in the quantity of labor demanded) is shown by the: A) shift from labor demand curve D1 to D2. C) move from a to b along labor demand curve D1. B) shift from labor demand curve D3 to D2. D) move from b to a along labor demand curve D1. Answer: A

Type: G Topic: 3 E: 509 MI: 265 62. Refer to the above graph. A move from b to a along labor demand curve D1 would result from: A) a decrease in the price of a substitute resource, assuming that the substitution effect exceeds the output effect. B) an increase in the wage rate. C) a decrease in the wage rate. D) an increase in the demand for the product that this labor is helping to produce. Answer: B

Type: G Topic: 3 E: 507 MI: 263 63. Refer to the above graph. Each of the three labor demand curves shown slopes downward because of the: A) law of diminishing marginal utility. C) principal-agent problem. B) law of increasing opportunity costs. D) law of diminishing returns. Answer: D

McConnell/Brue: Economics, 16/e

Page 854

Chapter 27: The Demand for Resources

Type: G Topic: 3 E: 509-510 MI: 265-266 64. Refer to the above graph. Other things equal, an increase in labor productivity would cause a(n): A) move from a to b on D1. C) shift from D3 to D2. B) shift from D2 to D3. D) move from b to a on D1. Answer: B

Type: G Topic: 3 E: 510 MI: 266 65. Refer to the above graph. Other things equal, an increase in the price of a complementary resource would cause a(n): A) move from a to b on D1. C) shift from D3 to D2. B) shift from D2 to D3. D) move from b to a on D1. Answer: C

Type: G Topic: 3 E: 510 MI: 266 66. Refer to the above graph. Other things equal, a decrease in the price of a substitute resource would cause a(n): A) move from a to b on D1. B) shift from D2 to D3 assuming the output effect exceeds the substitution effect. C) shift from D3 to D2 assuming the output effect exceeds the substitution effect. D) move from b to a on D1. Answer: B

Type: G Topic: 3 E: 510 MI: 266 67. Refer to the above graph. Other things equal, an increase in the price of substitute resource would cause a(n): A) shift from D2 to D3 assuming the substitution effect exceeds the output effect. B) move from a to b on D1. C) move from b to a on D1. D) shift from D3 to D2 assuming the substitution effect exceeds the output effect. Answer: A

Type: A Topic: 3 E: 509 MI: 265 68. Suppose the demand for strawberries rises sharply, resulting in an increased price of strawberries. As it relates to strawberry pickers, we could expect the: A) MRP curve to shift to the right. C) MRC curve to shift downward. B) MRP curve to shift to the left. D) MP curve to shift downward. Answer: A

Type: A Topic: 3 E: 511 MI: 267 69. Which of the following will not cause a shift in the demand for resource X? A) a decline in the price of resource X B) an increase in the price of the product resource X is producing C) a decrease in the price of substitute resource Y D) an increase in the productivity of resource X Answer: A

McConnell/Brue: Economics, 16/e

Page 855

Chapter 27: The Demand for Resources

Type: A Topic: 3 E: 511 MI: 267 70. A decline in the price of resource A will: A) increase the demand for complementary resource B. B) shift the demand curve for A to the left. C) shift the demand curve for A to the right. D) reduce the demand for complementary resource B. Answer: A

Type: A Topic: 3 E: 510 MI: 266 71. Assume the price of capital falls relative to the price of labor and, as a result, the demand for labor increases. Therefore: A) the output effect is greater than the substitution effect. B) capital is very highly substitutable for labor. C) the income effect is greater than the output effect. D) the substitution effect is greater than the output effect. Answer: A

Type: A Topic: 3 E: 511 MI: 267 72. The labor demand curve of a firm: A) will shift to the left if the price of the product the labor is producing falls. B) is perfectly elastic if the firm is selling its product in a purely competitive market. C) reflects a direct relationship between the number of workers hired and the money wage rate. D) is the same as its marginal product curve. Answer: A

Type: A Topic: 3 E: 511 MI: 267 73. Which of the following will not shift the demand curve for labor? A) the use of a larger stock of capital with the labor force B) a change in the wage rate C) an increase in the price of the product which labor is helping to produce D) the adoption of a more efficient method of combining labor and capital in the production process Answer: B

Type: A Topic: 3 E: 509-510 MI: 265-266 74. Employers will hire more units of a resource if: A) the price of the resource increases. B) the productivity of the resource increases. Answer: B

C) the price of the good being produced declines. D) the price of a complementary resource rises.

Type: A Topic: 3 E: 510 MI: 266 75. Assume that a firm's production technique is such that varying combinations of labor and capital can be used to produce output. If the price of labor falls relative to the price of capital and the firm decides to use more labor in the production process, this decision is: A) solely the result of the substitution effect. B) solely the result of the output effect. C) probably the result of both the substitution and output effects. D) the result of neither the substitution nor the output effect. Answer: C

McConnell/Brue: Economics, 16/e

Page 856

Chapter 27: The Demand for Resources

Type: A Topic: 3 E: 510 MI: 266 76. If technology dictates that labor and capital must be used in fixed proportions, an increase in the price of capital will cause a firm to use: A) more labor as a consequence of the substitution effect. B) more labor as a consequence of the output effect. C) less labor as a consequence of the substitution effect. D) less labor as a consequence of the output effect. Answer: D

Type: A Topic: 3 E: 510 MI: 266 77. If resources A and B are complementary and employed in fixed proportions: A) a change in the price of A will have no effect on the quantity of B employed. B) an increase in the price of A may either increase or decrease the demand for B. C) an increase in the price of A will increase the demand for B. D) an increase in the price of A will decrease the demand for B. Answer: D

Type: A Topic: 3 E: 510 MI: 266 78. If two resources are highly substitutable for one another: A) a decrease in the price of one will increase unit costs of production. B) an increase in the price of one will increase the demand for the other. C) an increase in the price of one will reduce the demand for the other. D) a decline in the price of one will increase the demand for the other. Answer: B

Type: A Topic: 3 E: 510 MI: 266 79. The output effect occurs: A) only when wage elasticity of demand is greater than 1. B) because a change in the price of a resource will alter costs and therefore the equilibrium output. C) only when the inputs being employed are substitutes. D) only when the inputs being employed are complementary. Answer: B

Type: A Topic: 3 E: 510-511 MI: 266-267 80. Suppose complementary inputs A and B are being used by a firm in the profit-maximizing amounts. If the price of A now increases, the firm should use: A) more of B, provided the substitution effect exceeds the output effect. B) more of B because of the substitution effect. C) less of B because of the substitution effect. D) less of B because of the output effect. Answer: D

Type: D Topic: 3 E: 510 MI: 266 81. The substitution effect indicates that a profit-seeking firm will use: A) more of an input whose price has fallen and less of other inputs in producing a given output. B) more of all inputs if production costs fall. C) more of those inputs whose marginal productivity is the greatest. D) less of an input whose price has fallen and more of other inputs in producing a given output. Answer: A

McConnell/Brue: Economics, 16/e

Page 857

Chapter 27: The Demand for Resources

Type: A Topic: 3 E: 510-511 MI: 266-267 82. Suppose capital and labor are used in fixed proportions so that each machine requires only one worker. If a decline in the price of capital occurs, then the demand for labor will: A) decline solely because of the substitution effect. B) increase solely because of the substitution effect. C) increase solely because of the output effect. D) decrease solely because of the output effect. Answer: C

Type: A Topic: 3 E: 510 MI: 266 83. Assume the price of capital doubles and, as a result, firms make no change in the relative quantities of capital and labor they employ. This implies that: A) labor is not readily substitutable for capital. C) the firms are producing an inferior good. B) the law of diminishing returns is not applicable. D) the demand for capital is highly price elastic. Answer: A

Type: A Topic: 3 E: 509-510 MI: 265-266 84. The demand curve for labor would shift leftward as the result of: A) an increase in the price of the product labor is producing. B) a decrease in the productivity of labor. C) an increase in the price of labor. D) a decrease in the price of capital, provided the output effect exceeds the substitution effect. Answer: B

Type: A Topic: 3 E: 510 MI: 266 85. A firm will employ more of an input whose relative price has fallen and, conversely, will use less of an input whose relative price has risen. Thus a fall in the price of capital will increase the relative price of labor and thereby reduce the demand for labor. This describes the: A) output effect. B) substitution effect. C) idea of derived demand. D) law of diminishing returns. Answer: B

Type: A Topic: 3 E: 510 MI: 266 86. A change in an input price will alter both production costs and the profit-maximizing output. Thus a decline in the price of capital will reduce production costs, increase the profit-maximizing output, and thereby increase the demand for labor. This describes the: A) output effect. B) substitution effect. C) idea of derived demand. D) law of diminishing returns. Answer: A

Type: A Topic: 3 E: 510 MI: 266 87. Capital and labor: A) are always complementary. B) are always substitutable. Answer: C

C) may be either complementary or substitutable. D) are both normal inputs.

McConnell/Brue: Economics, 16/e

Page 858

Chapter 27: The Demand for Resources

Type: C Topic: 3 E: 510-511 MI: 266-267 88. If the price of capital declines, the consequent output effect would be: A) greater, the more elastic the demand for the product. B) greater, the less elastic the demand for the product. C) negative. D) of consequence only if capital and labor are used in fixed proportions. Answer: A

Type: C Topic: 3 E: 511 MI: 267 89. Suppose the productivity of labor increases and at the same time the price of capital, which is complementary to labor, increases. As a result, the demand for labor: A) will increase. B) will decrease. C) may either increase or decrease. D) will not change. Answer: C

Type: A Topic: 3 E: 510 MI: 266 90. A change in the price of an input will usually: A) shift a firm's cost curves. B) cause the firm to alter the combination of inputs it employs. C) induce the firm to change its level of output. D) do all of the above. Answer: D

Type: C Topic: 3 E: 509 MI: 265 91. Suppose a technological improvement increases the productivity of a firm's capital and, simultaneously, its workers' union negotiates a wage increase. We can predict that: A) the firm will use relatively more capital and relatively less labor. B) the firm will use relatively more labor and relatively less capital. C) inputs of capital and labor will be unchanged. D) the firm's equilibrium output will necessarily increase. Answer: A

Type: A Topic: 3 E: 510 MI: 266 92. A manufacturer using both capital and labor decides to use more labor and less capital because of an increase in the price of capital. This is likely the result of: A) capital and labor being complementary inputs. B) capital and labor being substitute inputs. C) the output effect being greater than the substitution effect. D) diminishing returns being applicable to capital but not to labor. Answer: B

Type: C Topic: 3 E: 510 MI: 266 93. Suppose the price of the product that labor is producing increases and simultaneously the price of capital, which is substitutable for labor, decreases. Assuming that the substitution effect is greater than the output effect, the demand for labor: A) will increase. B) will decrease. C) may either increase or decrease. D) will not change. Answer: C

McConnell/Brue: Economics, 16/e

Page 859

Chapter 27: The Demand for Resources

Type: C Topic: 3 E: 509-510 MI: 265-266 94. Suppose there is a decline in the demand for the product labor is producing. Furthermore, the price of capital, which is complementary to labor, increases. Thus the demand for labor: A) will increase. B) will decrease. C) may either increase or decrease. D) will not change. Answer: B

Type: A Topic: 3 E: 510 MI: 266 95. Suppose that the price of capital increases relative to the wage rate and, as a result, the demand for labor increases. This means that: A) the substitution effect is greater than the output effect. B) labor and capital are complementary resources. C) it is impossible to substitute labor for capital. D) the output effect is greater than the substitution effect. Answer: A

Type: F Topic: 3 E: 512 MI: 268 Status: New 96. Which of the following occupations is not among the ten projected fastest growing U.S. occupations in terms of percentage increases? A) school teachers C) computer support specialists B) computer software engineers D) personal and home care aides Answer: A

Type: F Topic: 3 E: 512 MI: 268 Status: New 97. Which of the following occupations is not among the ten projected fastest growing U.S. occupations in terms of percentage increases? A) medical assistants B) computer systems administrators C) desktop publishers D) loan interviewers Answer: D

Elasticity of resource demand

Type: D Topic: 4 E: 513 MI: 269 Status: New 98. Elasticity of resource demand is measured by the: A) absolute change in resource quantity demanded divided by the absolute change in resource price. B) percentage change in resource quantity demanded divided by the percentage change in resource price. C) absolute change in resource price divided by the absolute change in resource quantity demanded. D) percentage change in resource price divided by the percentage change in resource quantity demanded. Answer: B

Type: F Topic: 4 E: 513 MI: 269 Status: New 99. When the elasticity coefficient for resource demand is greater than one, resource demand is: A) inelastic. B) elastic. C) unit elastic. D) perfectly inelastic. Answer: B

Type: F Topic: 4 E: 513 MI: 269 Status: New 100. When the elasticity coefficient for resource demand is less than one, resource demand is: A) inelastic. B) elastic. C) unit elastic. D) infinitely elastic. Answer: A

McConnell/Brue: Economics, 16/e

Page 860

Chapter 27: The Demand for Resources

Type: A Topic: 4 E: 513 MI: 269 Status: New 101. Suppose that a union successfully negotiated a 10 percent wage increase and the quantity of labor demanded increased by 10 percent. We can conclude that: A) the labor demand curve must have independently shifted to the right. B) labor demand is highly elastic. C) the coefficient of labor demand elasticity is less than 1. D) labor demand is unit-elastic. Answer: A

Type: A Topic: 4 E: 513 MI: 269 Status: New 102. Suppose that a union successfully negotiated a 10 percent wage increase and the quantity of labor demanded decreased by 10 percent. Given a fixed labor demand curve, we can conclude that: A) the labor demand curve is upward sloping. B) labor demand is elastic. C) labor demand is unit-elastic. D) the coefficient of elasticity of labor demand is less than 1. Answer: C

Type: A Topic: 4 E: 513 MI: 269 Status: New 103. Resource X has many close substitutes whereas resource Y has no close substitutes. Other things equal, we would expect: A) the demand for resource Y to be more elastic than the demand for resource X. B) resources X and Y to be close substitutes. C) resource X to be more expensive than resource Y. D) the demand for resource X to be more elastic than the demand for resource Y. Answer: D

Type: D Topic: 4 E: 513 MI: 269 104. The elasticity of resource demand measures the: A) responsiveness of workers to changes in wage rates. B) responsiveness of producers to changes in resource prices. C) ratio of marginal revenue product to resource price. D) sensitivity of marginal revenue product to changes in product price. Answer: B

Use the following to answer questions 105-106:

Wage rate $16 14 12 10 8

Quantity of labor demanded 2000 1600 1200 1000 800

Type: T Topic: 4 E: 513 MI: 269 105. Refer to the above data. For the $16 to $14 range of wage rates, labor demand is: A) perfectly elastic. B) elastic. C) perfectly inelastic. D) inelastic. Answer: B

McConnell/Brue: Economics, 16/e

Page 861

Chapter 27: The Demand for Resources

Type: T Topic: 4 E: 513 MI: 269 106. Refer to the above data. Over the $10 to $8 range of wage rates, the demand for labor is: A) perfectly elastic. B) elastic. C) unit elastic. D) inelastic. Answer: C

Type: A Topic: 4 E: 513 MI: 269 107. The elasticity of resource demand will be greater the: A) smaller the portion of the product's total costs accounted for by the resource. B) less the elasticity of demand for the product it is producing. C) larger the number of good substitute resources that are available. D) greater the elasticity of resource supply. Answer: C

Type: A Topic: 4 E: 513 MI: 269 108. The relationship between the elasticity of product demand and the elasticity of demand for labor employed in its production is such that, other things being equal: A) the more elastic the demand for the product, the less elastic the demand for labor. B) the more elastic the demand for the product, the more elastic the demand for labor. C) the elasticity of product demand only affects the elasticity of labor demand when the product market is purely competitive. D) if product demand is perfectly elastic, labor demand will be perfectly inelastic. Answer: B

Type: A Topic: 4 E: 513 MI: 269 109. Other things equal, the relationship between the relative importance of a given type of labor in a firm's total costs and the elasticity of demand for that labor is such that the: A) demand for labor will be elastic only if labor accounts for less than 50 percent of total costs. B) demand for labor will be elastic only if labor accounts for 50 percent or more of total costs. C) larger the labor cost-total cost ratio, the smaller will be the elasticity of labor demand. D) larger the labor cost-total cost ratio, the greater will be the elasticity of labor demand. Answer: D

Type: A Topic: 4 E: 513 MI: 269 110. Other things equal, if wage rates increase by 20 percent, the greatest decline in employment will occur when labor costs are a: A) large proportion of total costs and product demand is elastic. B) small proportion of total costs and product demand is elastic. C) large proportion of total costs and product demand is inelastic. D) small proportion of total costs and product demand is inelastic. Answer: A

Type: A Topic: 4 E: 513 MI: 269 111. If a 10 percent wage increase in a particular labor market results in a 5 percent decline in employment in that market, labor demand is: A) unit elastic. B) elastic. C) inelastic. D) perfectly elastic. Answer: C

McConnell/Brue: Economics, 16/e

Page 862

Chapter 27: The Demand for Resources

Type: A Topic: 4 E: 513 MI: 269 112. Assume that the coefficient of elasticity of product demand is .5 in industry A and is 3.2 in industry B. Other things equal, labor demand will be: A) more elastic in industry A than in B. C) more elastic in industry B than in A. B) unit elastic in both industry A and B. D) relatively inelastic in both industry A and B. Answer: C

Type: A Topic: 4 E: 513 MI: 269 113. Suppose that the labor cost-total cost ratio in industry A is 82 percent while in industry B it is 21 percent. Other things equal, labor demand will be: A) more elastic in industry A than in B. C) more elastic in industry B than in A. B) unit elastic in both industry A and B. D) relatively elastic in both industry A and B. Answer: A

Type: A Topic: 4 E: 513 MI: 269 Status: New 114. Which of the following statements is true? Other things equal, the demand for labor will be less elastic the: A) the easier it is to substitute capital for labor. C) greater the elasticity of product demand. B) greater the elasticity of resource supply. D) smaller the ratio of labor costs to total costs. Answer: D

Optimal combination of resources

Type: A Topic: 5 E: 514 MI: 270 115. Assuming a competitive resource market, a firm is hiring several resources in the profit-maximizing amounts when the: A) firm's total outlay on resources is minimized. B) marginal revenue product of each resource is equal to its price. C) price of each resource employed is the same. D) marginal revenue product of the last unit of each resource hired is the same. Answer: B

Type: A Topic: 5 E: 514 MI: 270 116. Assume that an appliance manufacturer is employing variable resources X and Y in such amounts that the MRPs of the last units of X and Y employed are $100 and $60 respectively. Resource X can be hired at $50 per unit and resource Y at $20 per unit. The firm: A) should hire more of both X and Y. B) should hire more of Y and less of X. C) is producing with the least-costly combination of X and Y, but could increase its profits by employing more of X and less of Y. D) is using the least-cost combination of X and Y, but could increase its profits by employing less of both X and Y. Answer: A

McConnell/Brue: Economics, 16/e

Page 863

Chapter 27: The Demand for Resources

Type: A Topic: 5 E: 515 MI: 271 117. A firm is hiring resources X, Y, and Z in the profit-maximizing amounts when: A) MRPx/Px equals MRPy/Py equals MRPz/Pz equals 1. B) the sum of the MRPs of the three resources is at a minimum. C) the marginal revenue productivity of all three resources is the same. D) the marginal revenue product of the last dollar spent on each of the three resources is the same. Answer: A

Use the following to answer questions 118-121: Answer the next question(s) on the basis of the following marginal product data for resources a and b. The output of these independent resources sells in a purely competitive market at $1 per unit.
Inputs of a 1 2 3 4 5 6 7 MPa 25 20 15 10 5 2 1 Inputs of b l 2 3 4 5 6 7 MPb 40 36 32 24 20 16 8

Type: T Topic: 5 E: 514 MI: 270 118. Refer to the above data. Assuming the prices of resources a and b are $5 and $8 respectively, what is the least costly combination of resources for the firm to employ in producing 192 units of output? A) 2 of a and 6 of b B) 6 of a and 2 of b C) 4 of a and 3 of b D) 3 of a and 4 of b Answer: D

Type: T Topic: 5 E: 514-515 MI: 270-271 119. Refer to the above data. Assuming the prices of resources a and b are $5 and $8 respectively, what is the profit-maximizing combination of resources? A) 7 of a and 7 of b B) 6 of a and 4 of b C) 5 of a and 7 of b D) 4 of a and 4 of b Answer: C

Type: T Topic: 5 E: 515 MI: 271 120. Refer to the above data. When the firm hires the profit-maximizing combination of resources, its economic profit will be: A) $170 B) $76 C) $145 D) $138 Answer: A

Type: T Topic: 5 E: 515-516 MI: 271-272 121. Refer to the above data. Assume now that the prices of a and b rise to $15 and $20 respectively. To maximize profits what combination of a and b should the employer hire? A) 3 of a and 5 of b B) 5 of a and 7 of b C) 7 of a and 7 of b D) 6 of a and 2 of b Answer: A

McConnell/Brue: Economics, 16/e

Page 864

Chapter 27: The Demand for Resources

Type: A Topic: 5 E: 515 MI: 271 122. The equation MPL/PL = MPC/PC: A) designates the MR = MC level of output. B) assumes imperfect competition in the hiring of labor and capital. C) is a sufficient condition for the maximization of profits. D) is a necessary, but not sufficient, condition for the maximization of profits. Answer: D

Type: A Topic: 5 E: 514-515 MI: 270-271 123. Assume a firm purchases resources a and b under purely competitive conditions and combines these resources to produce X. Product X is sold in a purely competitive market. The MP of a and b are 6 and 3 respectively and the prices of a and b are $12 and $6 respectively. If equilibrium exists, the price of X will be: A) $1. B) $.50. C) $2. D) $5. Answer: C

Type: A Topic: 5 E: 515 MI: 271 124. Which of the following statements is correct? A) If the profit-maximizing rule is fulfilled, it necessarily follows that the cost-minimization rule is being fulfilled. B) The profit-maximizing and the cost-minimizing rules are such that the fulfilling of one has no bearing on the fulfilling of the other. C) If the profit-maximizing rule is fulfilled, the cost-minimization rule may or may not be fulfilled. D) If the cost-minimization rule is fulfilled, it necessarily follows that the profit-maximizing rule is being fulfilled. Answer: A

Type: A Topic: 5 E: 514-515 MI: 270-271 125. If MPa/Pa = MPb/Pb and MRPa/Pa = MRPb/Pb>1, this firm is: A) producing its output with the least costly combination of resources, but is not producing the profitmaximizing output. B) maximizing profits, but failing to minimize costs. C) neither maximizing profits nor minimizing costs. D) combining resources a and b so as to minimize costs and maximize profits. Answer: A

Type: A Topic: 5 E: 514 MI: 270 126. A firm that is motivated by self interest should: A) employ the combination of resources that will produce the profit-maximizing output at the minimum cost. B) hire each input so the productivity of each is equal at the margin. C) always use large amounts of the most productive inputs and small amounts of the least productive inputs in producing its output. D) always use large amounts of cheap inputs and small amounts of expensive inputs in producing its output. Answer: A

McConnell/Brue: Economics, 16/e

Page 865

Chapter 27: The Demand for Resources

Type: A Topic: 5 E: 515 MI: 271 127. Assuming pure competition, which of the following are equivalents? A) MRPL/PL = MRPC/PC and Px = 1/MC. C) Px = MC and MRPL/PL = MRPC/PC = 1. B) MRPL/PL = MRPC/PC and Px = AVC. D) Px = MC and MPL/PL = MPC/PC. Answer: C

Type: A Topic: 5 E: 514 MI: 270 128. Suppose a firm is hiring resources l and m under purely competitive conditions to produce product Y that sells for $2 in a purely competitive market. The prices of l and m are $10 and $4 respectively. In equilibrium the MPs of l and m, respectively, are: A) 1 and 1. B) 2 and 5. C) 10 and 4. D) 5 and 2. Answer: D

Type: A Topic: 5 E: 514-515 MI: 270-271 129. If a firm is employing quantities of resources J and K so that MRPJ/PJ = MRPK/PK = 1, then: A) MPJ/PJ may either exceed or be less than MPK/PK. B) MPJ/PJ will be less than MPK/PK. C) MPJ/PJ will exceed MPK/PK. D) MPJ/PJ = MPK/PK. Answer: D

Type: C Topic: 5 E: 514 MI: 270 130. If a firm is hiring variable resources D and F in perfectly competitive input markets, it will minimize the cost of producing any level of output by employing D and F in such amounts that: A) the price of each input equals its MP. C) MPD/PD = MPF/PF. B) MPD = MPF. D) MPD/PF = MPF/PD. Answer: C

Type: A Topic: 5 E: 515 MI: 271 131. Suppose a firm is employing all its inputs so that the MRP per dollar spent on each is the same. This suggests that the: A) amount of each resource employed will depend on both its price and its productivity. B) price of each input must be identical. C) firm is using the same quantity of each input. D) total expenditure on each input is identical. Answer: A

Type: A Topic: 5 E: 515-516 MI: 271-272 132. Assume a pencil manufacturer is employing resources C and D in such quantities that the MRPs of the last units hired are $80 and $50 respectively. The price of resource C is $90 and the price of D is $35. This firm: A) should hire less of C and more of D. C) should hire less of both C and D. B) should hire more of both C and D. D) is using the least-cost combination of C and D. Answer: A

McConnell/Brue: Economics, 16/e

Page 866

Chapter 27: The Demand for Resources

Use the following to answer questions 133-138: Answer the next question(s) on the basis of the following data:

Quantity of labor 1 2 3 4 5 6

MP of labor 15 12 9 6 3 1

MRP of labor $45 36 27 18 9 3

Quantity of capital 1 2 3 4 5 6

MP of capital 8 6 5 4 3 2

MRP of capital $24 18 15 12 9 6

Type: T Topic: 5 E: 515 MI: 271 133. Refer to the above data. This firm is selling its product in: A) an imperfectly competitive market at prices that decline as sales increase. B) a purely competitive market at $3 per unit. C) a purely competitive market at $2 per unit. D) an imperfectly competitive market at $3 per unit. Answer: B

Type: T Topic: 5 E: 515-516 MI: 271-272 134. Refer to the above data. If the prices of labor and capital are $9 and $15 respectively, the firm will hire: A) 5 units of labor and 3 of capital. C) 4 units of labor and 4 of capital. B) 5 units of labor and 2 of capital. D) 3 units of labor and 4 of capital. Answer: A

Type: T Topic: 5 E: 516 MI: 272 135. Refer to the above data. The firm's total output will be: A) 38 units. B) 60 units. C) 64 units. D) 27 units. Answer: C

Type: T Topic: 5 E: 515-516 MI: 271-272 136. Refer to the above data. The firm's total revenue will be: A) $114. B) $180. C) $129. D) $192. Answer: D

Type: T Topic: 5 E: 515-516 MI: 271-272 137. Refer to the above data. If labor and capital are the only inputs, the firm's total costs will be: A) $106. B) $126. C) $47. D) $90. Answer: D

Type: T Topic: 5 E: 516 MI: 272 138. Refer to the above data. Assuming labor and capital are the only inputs, the firm's economic profits will be: A) $102. B) $82. C) $67. D) $28. Answer: A

McConnell/Brue: Economics, 16/e

Page 867

Chapter 27: The Demand for Resources

Type: A Topic: 5 E: 515 MI: 271 139. The profit-maximizing and the least-cost combination of inputs are: A) the result of unrelated decisions. B) always identical. C) such that the minimization of costs always results in profit maximization. D) such that the maximization of profits always entails the least-cost combination of inputs. Answer: D

Use the following to answer questions 140-142: Answer the next question(s) on the basis of the following information: Suppose a firm hires both labor (L) and capital (C) under purely competitive conditions. The price of labor is PL and that of capital is PC. The marginal product of labor is MPL and that of capital is MPC. The firm sells its product competitively at a price of PX.

Type: A Topic: 5 E: 514 MI: 270 140. Refer to the above information. Which of the following must pertain if the firm is to minimize the cost of producing any output? A) MPC = MPL = PX. C) MPC/PC = MPL/PL. B) MPC = PC and MPL = PL. D) MPC/PX = MPL/PX. Answer: C

Type: A Topic: 5 E: 514 MI: 270 141. Refer to the above information. If MPC/PC > MPL/PL, the firm: A) may be maximizing profits, but it is not minimizing costs. B) may be minimizing costs, but it is not maximizing profits. C) is neither minimizing costs nor maximizing profits. D) is minimizing costs and maximizing profits. Answer: C

Type: A Topic: 5 E: 514 MI: 270 142. Refer to the above information. In competitive labor markets, the marginal cost of an additional unit of labor: A) is equal to PL × MPL. B) is equal to MPL/PL. C) is equal to PL. D) cannot be determined from the information given. Answer: C

McConnell/Brue: Economics, 16/e

Page 868

Chapter 27: The Demand for Resources

Use the following to answer questions 143-145:

Type: G Topic: 5 E: 514 MI: 270 143. Refer to the above diagram. If a firm produces output Q1 at a unit cost of c, then the: A) firm is operating in a purely competitive industry. B) firm is maximizing profits. C) marginal product per dollar's worth of each resource employed is not the same. D) firm is fulfilling the least-cost rule in employing resources. Answer: C

Type: G Topic: 5 E: 514 MI: 270 144. Refer to the above diagram. If a firm produces output Q1 at a unit cost of b, then the: A) firm is not fulfilling the least-cost rule in employing resources. B) firm may or may not be maximizing profits. C) marginal product per dollar's worth of each resource employed is not the same. D) firm has achieved minimum efficient scale. Answer: B

Type: G Topic: 5 E: 514 MI: 270 145. Refer to the above diagram. The production of Q1 units of output at an average cost of a: A) is not possible, given present technology and resource prices. B) can be achieved if the firm would hire the optimal mix of resources. C) would entail X-inefficiency. D) can be realized if the last dollar spent on each input were equal to its marginal product. Answer: A

Marginal productivity theory of income distribution

Type: D Topic: 6 E: 516 MI: 272 146. The marginal productivity theory of income distribution suggests that : A) government should subsidize the most productive workers through a system of transfer payments. B) each individual should receive income based on his contribution to total output. C) resource owners should receive income based on the idea of "from each according to his ability, to each according to his wants." D) resource owners should receive income based upon their needs. Answer: B

McConnell/Brue: Economics, 16/e

Page 869

Chapter 27: The Demand for Resources

Type: A Topic: 6 E: 516 MI: 272 147. "Income receivers should be paid in accordance with the value of output each produces." This statement is consistent with the: A) monopoly theory of income distribution. B) marginal productivity theory of income distribution. C) least-cost, but not profit-maximizing, combination of inputs. D) concept of compensating wage differences. Answer: B

Type: A Topic: 6 E: 516-517 MI: 272-273 148. The fact that monopoly and monopsony exist in resource markets means that: A) the marginal productivity theory of income distribution is valid. B) resource prices need not accurately measure contributions to output. C) the resulting income distribution is ethically correct. D) income shares do not exhaust the total output. Answer: B

Type: A Topic: 6 E: 517 MI: 273 149. The marginal productivity theory of income distribution has been criticized because: A) the resulting distribution of income is likely to be too equal to maintain production incentives. B) income from inherited property is inconsistent with the theory. C) purely competitive conditions characterize most resource markets. D) it fails to recognize that resource demand is derived from product demand. Answer: B

Consider This Questions

Type: A E: 509 MI: 265 Status: New 150. (Consider This) In the market for superstars: A) earnings reflect pricing power rather than marginal revenue product. B) small differences in talent get magnified into huge differences in pay. C) entry and exit rarely occur. D) product demand is typically highly elastic. Answer: B

Type: A E: 509 MI: 265 Status: New 151. (Consider This) According to the Consider This box "She's the One," the high pay of superstars reflects: A) elastic product demand. C) blocked occupational entry. B) high marginal revenue productivity. D) warped societal values. Answer: B

Last Word Questions

Type: A E: 517 MI: 273 152. (Last Word) "The Case of ATMs" best illustrates the: A) law of diminishing marginal utility. C) idea of derived demand. B) the substitutability of resources. D) principle of unintended side-effects. Answer: B

McConnell/Brue: Economics, 16/e

Page 870

Chapter 27: The Demand for Resources

Type: A E: 517 MI: 273 153. (Last Word) ATMs and human bank tellers: A) are substitute resources. B) are capital goods. C) have both declined in number because of bank mergers. D) are complementary resources. Answer: A

Type: A E: 517 MI: 273 154. (Last Word) The rapid spread of STMs has: A) resulted from changes in banking laws. B) increased the demand for bank tellers. Answer: C

C) reduced the demand for bank tellers. D) increased the hourly wage paid to bank tellers.

True/False Questions

Type: A E: 505 MI: 261 155. The marginal revenue product curve of a purely competitive seller declines solely because of the law of diminishing returns. Answer: True

Type: A E: 506 MI: 262 156. Producers should hire resources until the total output of each is equal. Answer: False

Type: A E: 506 MI: 262 157. It will be profitable for a firm to hire additional units of any resource up to the point at which its MRP is equal to its MRC. Answer: True

Type: A E: 513 MI: 269 158. The more elastic the demand for a product the less elastic will be the demand for the resources employed in producing it. Answer: False

Type: A E: 509-510 MI: 265-266 159. The demand for a resource depends on its productivity and the market value of the product it is producing. Answer: True

Type: A E: 510 MI: 266 160. If two resources are complementary, a decrease in the price of one will reduce the demand for the other. Answer: False

McConnell/Brue: Economics, 16/e

Page 871

Chapter 27: The Demand for Resources

Type: A E: 513 MI: 269 161. Other things equal, the less competitive the market in which a firm sells its product, the less elastic will be its resource demand curve. Answer: True

Type: A E: 510 MI: 266 162. If the substitution effect outweighs the output effect, an increase in the price of a substitute resource will increase the demand for labor. Answer: True

Type: A E: 505 MI: 261 Status: New 163. The demand for labor is a derived demand whereas the demand for capital is not. Answer: False

Type: F E: 506-507 MI: 262-263 Status: New 164. The MRP of labor curve is the labor demand curve. Answer: True

Type: D E: 506 MI: 262 Status: New 165. Marginal revenue product (MRP) is the change in total product (total output) associated with hiring an additional unit of labor. Answer: False

Type: A E: 506 MI: 262 Status: New 166. A firm should reduce its employment of a resource whose marginal resource cost exceeds its marginal revenue product. Answer: True

Type: F E: 512 MI: 268 Status: New 167. In percentage terms, many of the 10 most rapidly declining U.S. occupations include jobs for which capital is readily substitutable for labor. Answer: True

Type: D E: 513 MI: 269 Status: New 168. Elasticity of resource demand is measured by dividing "percentage change in resource price" by "percentage change in resource quantity." Answer: False

Type: A E: 510 MI: 266 Status: New 169. An increase in the price of capital will reduce the demand for labor if capital and labor are complementary resources. Answer: True

McConnell/Brue: Economics, 16/e

Page 872

Chapter 27: The Demand for Resources

Type: F E: 514 MI: 270 Status: New 170. The marginal productivity theory of income distribution holds that all resources are paid according to their marginal contribution to society's output. Answer: True

McConnell/Brue: Economics, 16/e

Page 873

CHAPTER 28

Wage Determination

Topic 1. 2. 3. 4. 5. 6. 7.

Question numbers 1-11 12-46 47-85 86-122 123-130 131-148 149-162 163-164 165-167 168-184

____________________________________________________________

_______________________________________

Real and nominal wage rates; trends Purely competitive labor markets Monopsony and imperfectly competitive labor markets Union models and licensure Minimum wage Wage differentials; human capital Pay and performance Consider This Last Word True-False

____________________________________________________________

_______________________________________

Multiple Choice Questions Real and nominal wage rates; trends

Type: F Topic: 1 E: 523 MI: 279 1. Real wages in the United States in the long run: A) show no discernible relationship to output per worker. B) have increased at about the same rate as increases in output per worker. C) have increased slower than increases in output per worker. D) have increased faster than increases in output per worker. Answer: B

Type: F Topic: 1 E: 523 MI: 279 2. The long-run trend of real wages: A) cannot be determined from available data on nominal wages and the price level. B) has been downward because the price level has risen faster than nominal wages. C) has been upward. D) has been downward because labor's share of the domestic income has fallen. Answer: C

Type: A Topic: 1 E: 521-522 MI: 277-278 3. If the nominal wages of carpenters rose by 5 percent in 2000 and the price level increased by 3 percent, then the real wages of carpenters: A) decreased by 2 percent. C) increased by 3 percent. B) increased by 2 percent. D) increased by 8 percent. Answer: B

Chapter 28: Wage Determination

Type: A Topic: 1 E: 523 MI: 279 4. Over the long run, real earnings per worker can increase only at about the same rate as the economy's rate of growth of: A) total output. B) stock of capital. C) output per worker. D) international trade. Answer: C

Type: A Topic: 1 E: 523 MI: 279 5. Increases in the productivity of labor result partly from: A) the law of diminishing returns. C) reductions in wage rates. B) improvements in technology. D) increases in the quantity of labor. Answer: B

Type: F Topic: 1 E: 522 MI: 278 6. Real wages in the United States are: A) the highest in the world. B) relatively high, but not as high as in some other industrially advanced nations. C) much higher than output per worker. D) higher than nominal wages. Answer: B

Type: F Topic: 1 E: 522 MI: 278 7. According to international comparisons, which nation had the highest real wages in U.S. dollar terms in 2001? A) the United States B) Germany C) Italy D) Canada Answer: B

Type: A Topic: 1 E: 521-522 MI: 277-278 8. The real wage will rise if the nominal wage: A) falls more rapidly than the general price level. B) increases at the same rate as labor productivity. C) increases more rapidly than the general price level. D) falls more rapidly than the general price level. Answer: C

Type: A Topic: 1 E: 521-522 MI: 277-278 9. Which of the following is correct? A) The nominal wage may fall, but the real wage can never decline. B) The real wage may fall, but the nominal wage can never decline. C) Both the nominal and the real wage must always rise. D) The nominal and the real wage may both fall. Answer: D

McConnell/Brue: Economics, 16/e

Page 876

Chapter 28: Wage Determination

Type: A Topic: 1 E: 522 MI: 278 10. The productivity and real wages of workers in the industrially advanced economies have risen historically partly because: A) workers have acquired less education and training over time. B) workers have been able to use larger quantities of capital equipment. C) over time the capital equipment used by workers has deteriorated in quality. D) the supply of labor has increased. Answer: B

Type: A Topic: 1 E: 521-522 MI: 277-278 11. If the nominal wage increases by less than the price level, the real wage: A) will increase. C) may either increase or decrease. B) will decrease. D) will diverge from labor productivity growth. Answer: B

Purely competitive labor markets

Type: D Topic: 2 E: 506 MI: 262 Status: New 12. Marginal revenue product (MRP) of labor refers to the: A) increase in total revenue resulting from the sale of an additional unit of output. B) amount by which a firm's total resource cost increases when it employs one more unit of labor. C) increase in total revenue resulting from the hire of one more unit of labor. D) price at which additional units of labor can be employed in a monopsonized labor market. Answer: C

Type: D Topic: 2 E: 506 MI: 262 13. Marginal resource cost refers to the: A) increase in total revenue resulting from the sale of the extra output of one more worker. B) price at which additional units of a resource can be hired in an imperfectly competitive resource market. C) increase in total cost resulting from the production of one more unit of output. D) amount by which a firm's total resource cost increases as the result of hiring one more unit of the resource. Answer: D

Type: A Topic: 2 E: 524 MI: 280 14. If a firm is hiring a certain type of labor under purely competitive conditions: A) its labor demand curve will be perfectly elastic at the market-determined wage rate. B) the labor supply curve will lie above the marginal labor cost curve. C) the labor supply and marginal labor (resource) cost curves will coincide and be upsloping. D) the labor supply and marginal labor (resource) cost curves will coincide and be perfectly elastic. Answer: D

McConnell/Brue: Economics, 16/e

Page 877

Chapter 28: Wage Determination

Type: A Topic: 2 E: 524 MI: 280 15. The labor supply curve for a particular occupation is upsloping because: A) higher wages will be needed to attract workers from other occupations. B) lower wages will be needed to increase employment. C) higher wages will enable some workers to afford more leisure. D) the labor demand curve is downsloping. Answer: A

Type: A Topic: 2 E: 524-525 MI: 280-281 16. The market supply curve for labor is upsloping because: A) of diminishing returns. B) of the opportunity cost of labor in housekeeping, leisure, or alternative employments. C) of declining MRC. D) each employer is a "wage taker." Answer: B

Type: A Topic: 2 E: 524-525 MI: 280-281 17. A firm operating in a purely competitive resource market has a resource supply curve that is: A) perfectly inelastic. B) perfectly elastic. C) highly inelastic. D) highly elastic. Answer: B

Type: A Topic: 2 E: 524 MI: 280 Status: New 18. A firm that is hiring labor in a purely competitive labor market and selling its product in a purely competitive product market will maximize its profit by hiring labor until: A) marginal revenue product is zero. B) marginal revenue product exceeds marginal resource (labor) cost by the greatest amount. C) marginal resource cost is zero. D) marginal revenue product equals marginal resource (labor) cost. Answer: D

Type: A Topic: 2 E: 524 MI: 280 Status: New 19. A profit-maximizing firm will: A) expand employment if marginal revenue product exceeds marginal resource cost. B) reduce employment if marginal revenue product exceeds marginal resource cost. C) expand employment if marginal revenue product equals marginal resource cost. D) reduce employment if marginal revenue product equals marginal resource cost. Answer: A

Type: A Topic: 2 E: 524 MI: 280 Status: New 20. A profit-maximizing firm will: A) expand employment if marginal revenue product equals marginal resource cost. B) reduce employment if marginal revenue product equals marginal resource cost. C) reduce employment if marginal revenue product is less than marginal resource cost. D) expand employment if marginal revenue product is less than marginal resource cost. Answer: C

McConnell/Brue: Economics, 16/e

Page 878

Chapter 28: Wage Determination

Type: A Topic: 2 E: 524-525 MI: 280-281 Status: New 21. A firm hiring labor in a perfectly competitive labor market faces a: A) downward sloping labor supply curve and upward sloping labor demand curve. B) upward sloping labor supply curve and downward sloping labor demand curve. C) upward sloping labor supply curve and horizontal labor demand curve. D) horizontal labor supply curve and downward sloping labor demand curve. Answer: D

Type: F Topic: 2 E: 524-526 MI: 280-282 Status: New 22. Which of the following describes a purely competitive labor market? A) MRP = Wage Rate. B) MRP > Wage Rate. C) Wage Rate > MRC. D) Wage Rate < MRC. Answer: A

Type: F Topic: 2 E: 524-526 MI: 280-282 Status: New 23. Which of the following describes a purely competitive labor market? A) MRP < Wage Rate. B) MRP > Wage Rate. C) Wage Rate = MRC. D) Wage Rate < MRC. Answer: C

Use the following to answer questions 24-28:
Units of labor 1 2 3 4 5 Wage rate $8 8 8 8 8 MRC (of labor) $8 8 8 8 8 MRP (of labor) $12 10 8 6 4

Type: T Topic: 2 E: 524 MI: 280 24. Refer to the above data. If there is neither a union nor a minimum wage, we can conclude that this firm: A) "purchases" labor in purely competitive labor market. B) is a monopsonist. C) faces a perfectly inelastic labor supply curve. D) has a perfectly elastic labor demand curve. Answer: A

Type: T Topic: 2 E: 524, 526 MI: 280, 282 25. Refer to the above data. In maximizing its profit, this firm will employ: A) 2 units of labor. B) 3 units of labor. C) 4 units of labor. D) 5 units of labor. Answer: B

Type: T Topic: 2 E: 524 MI: 280 26. Refer to the above data. At the profit maximizing level of employment, this firm's: A) MRP will exceed its MRC. C) MRP will be less than its MRC. B) MRP will equal its MRC. D) MRP will be zero. Answer: B

McConnell/Brue: Economics, 16/e

Page 879

Chapter 28: Wage Determination

Type: T Topic: 2 E: 524, 526 MI: 280, 282 27. Refer to the above data. At the profit maximizing level of employment, this firm's, total labor cost will be: A) $16. B) $30. C) $24. D) $32. Answer: C

Type: T Topic: 2 E: 524, 526 MI: 280, 282 Status: New 28. Refer to the above data. At the profit maximizing level of employment, this firm's, total revenue will be: A) $16. B) $32. C) $24. D) $30 Answer: D

Use the following to answer questions 29-32: Use the labor demand data on the left and the labor supply data on the right in answering the following question(s):
Employment 0 1 2 3 4 5 6 Marginal product 0 14 12 9 7 4 2 Product price $3 3 3 3 3 3 3 Employment 0 1 2 3 4 5 6 Wage rate $11 11 11 11 11 11 11

Type: T Topic: 2 E: 524, 526 MI: 280, 282 29. On the basis of the above information we: A) can say that the firm's labor supply curve is upsloping. B) cannot say whether the firm's product market is purely or imperfectly competitive. C) can say that the firm is selling its product in a purely competitive market. D) can say that the firm is selling its product in an imperfectly competitive market. Answer: C

Type: T Topic: 2 E: 525 MI: 281 30. Refer to the above data. The firm's labor supply curve is: A) such that it does not intersect the labor demand curve. B) upsloping. C) perfectly inelastic. D) perfectly elastic. Answer: D

Type: T Topic: 2 E: 524 MI: 280 31. Refer to the above data. The firm is hiring labor: A) at a wage rate that exceeds labor's MRP. B) under purely competitive conditions. Answer: B

C) in an imperfectly competitive market. D) as a monopsonist.

McConnell/Brue: Economics, 16/e

Page 880

Chapter 28: Wage Determination

Type: T Topic: 2 E: 524, 526 MI: 280, 282 32. Refer to the above data. The firm will maximize profits (or minimize losses) by employing: A) 5 workers. B) 4 workers. C) 3 workers. D) 2 workers. Answer: B

Use the following to answer questions 33-37:

Type: G Topic: 2 E: 525 MI: 281 33. Refer to the above diagrams. The firm: A) is a monopsonist in the hire of labor. B) must be selling its product in an imperfectly competitive market. C) is a "wage taker." D) must pay a higher marginal resource cost for each successive worker. Answer: C

Type: G Topic: 2 E: 525 MI: 281 34. Refer to the above diagrams. The firm: A) has a principal-agent problem. B) has a constant marginal resource cost of $5. C) has a marginal resource cost that exceeds the wage rate for each worker. D) will fail to maximize profits if it hires 5 workers. Answer: B

Type: G Topic: 2 E: 525 MI: 281 35. Refer to the above diagrams. The firm's total wage cost: A) is 0abc. B) is 0wbc. C) is wab. D) cannot be determined. Answer: B

Type: G Topic: 2 E: 525 MI: 281 36. Refer to the above diagrams. The firm's total revenue: A) is 0abc. B) is 0wbc. C) is wab. D) cannot be determined. Answer: A

Type: G Topic: 2 E: 525 MI: 281 37. Refer to the above diagrams. The amount available to pay to nonlabor resources: A) is 0abc. B) is 0wbc. C) is wab. D) cannot be determined. Answer: C

McConnell/Brue: Economics, 16/e

Page 881

Chapter 28: Wage Determination

Type: A Topic: 2 E: 525 MI: 281 38. The individual firm in a purely competitive labor market faces: A) a perfectly elastic labor supply curve and a downsloping labor demand curve. B) a perfectly elastic labor demand curve and an upsloping labor supply curve. C) labor demand and labor supply curves both of which are perfectly elastic. D) a downsloping labor demand curve and an upsloping labor supply curve. Answer: A

Use the following to answer questions 39-46:

Quantity of labor 1 2 3 4 5

Total product 4 8 11 13 14

Total revenue $16 32 44 52 56

Type: T Topic: 2 E: 525-526 MI: 281-282 39. Refer to the above data. This firm's product price is: A) $2. B) $3. C) $4. D) $16. Answer: C

Type: T Topic: 2 E: 524 MI: 280 40. Refer to the above data. The marginal revenue product of the second worker is: A) $16. B) $32. C) $8. D) $4. Answer: A

Type: T Topic: 2 E: 524 MI: 280 41. Refer to the above data. The marginal revenue product of the fourth worker is: A) $8. B) $52. C) $2. D) $4. Answer: A

Type: T Topic: 2 E: 525 MI: 281 42. Refer to the above data. We can conclude from the information given that this firm is a: A) pure monopolist. B) discriminating monopolist. C) monopolistic competitor. D) pure competitor. Answer: D

Type: T Topic: 2 E: 524-525 MI: 280-281 43. Refer to the above data. If the market wage rate is $8, this firm will employ: A) 2 workers. B) 3 workers. C) 4 workers. D) 5 workers. Answer: C

Type: T Topic: 2 E: 524-525 MI: 280-281 44. Refer to the above data. If the market wage rate is $8 and the firm hires its profit-maximizing number of workers, the firm's total wage bill (payment) will be: A) $16. B) $24. C) $32. D) $48. Answer: C

McConnell/Brue: Economics, 16/e

Page 882

Chapter 28: Wage Determination

Type: T Topic: 2 E: 524-525 MI: 280-281 45. Refer to the above data. If the market wage rate is $8 and the firm hires its profit-maximizing number of workers, the firm's total revenue will exceed its total wage payment by: A) $20. B) $16. C) $12. D) $8. Answer: A

Type: T Topic: 2 E: 524-525 MI: 280-281 46. Refer to the above data. If this firm can hire as few or many workers as it wants at $8, it is: A) hiring labor in a monopsony labor market. B) hiring labor in a purely competitive labor market. C) selling its product in a monopolized product market. D) selling its product in a purely competitive product market. Answer: B

Monopsony and imperfectly competitive labor markets

Use the following to answer questions 47-49: 1) W < MRP; W < MRC 2) W = MRP; W < MRC 3) W = MRP; W = MRC 4) W > MRP; W > MRC

Type: F Topic: 3 E: 527 MI: 283 Status: New 47. Refer to the above list. The outcome in a purely competitive labor market is shown by: A) 1. B) 2. C) 3. D) 4. Answer: C

Type: F Topic: 3 E: 527 MI: 283 Status: New 48. Refer to the above list. The outcome in a monopsony labor market is shown by: A) 1. B) 2. C) 3. D) 4. Answer: A

Type: F Topic: 3 E: 527 MI: 283 Status: New 49. The labor supply curve facing a purely competitive employer is __________ whereas the labor supply curve facing a monopsonist is ___________. A) upward sloping; horizontal C) vertical; upward sloping B) downward sloping; vertical D) horizontal; upward sloping Answer: D

McConnell/Brue: Economics, 16/e

Page 883

Chapter 28: Wage Determination

Use the following to answer questions 50-54: Use the resource demand data shown on the left and the resource supply data on the right in answering the following question(s):
Employment 0 1 2 3 4 5 6 Total product 0 15 28 39 48 55 60 Product price $2.20 2.00 1.80 1.60 1.40 1.20 1.00 Employment 0 1 2 3 4 5 6 Wage rate -$1.00 2.00 3.00 4.00 5.00 6.00

Type: T Topic: 3 E: 527 MI: 283 50. Refer to the above data. How many workers will this firm choose to employ? A) 6 B) 5 C) 4 D) 3 Answer: D

Type: T Topic: 3 E: 527 MI: 283 51. Refer to the above data. How many units of output will the firm produce? A) 60 B) 55 C) 48 D) 39 Answer: D

Type: T Topic: 3 E: 527 MI: 283 52. Refer to the above data. What will be the equilibrium wage rate? A) $6 B) $5 C) $4 D) $3 Answer: D

Type: T Topic: 3 E: 527 MI: 283 53. Refer to the above data. What will be the selling price of the product? A) $1.40 B) $1.60 C) $1.80 D) $2.00 Answer: B

Type: T Topic: 3 E: 527 MI: 283 54. Refer to the above data. We can conclude that: A) both the product and resource markets are imperfectly competitive. B) the resource market is imperfectly competitive but the product market is purely competitive. C) both the resource and product markets are purely competitive. D) the resource market is purely competitive but the product market is imperfectly competitive. Answer: A

Type: D Topic: 3 E: 526 MI: 282 55. The economic term for a sole employer in a nonunion community is: A) monopsonist. B) monopolist. C) bilateral competitor. D) bilateral monopolist. Answer: A

McConnell/Brue: Economics, 16/e

Page 884

Chapter 28: Wage Determination

Type: A Topic: 3 E: 527 MI: 283 56. In a monopsonistic labor market the employer will maximize profits by employing workers up to that point at which: A) the difference between the wage rate and marginal resource (labor) cost is at a maximum. B) marginal revenue product equals marginal resource (labor) cost. C) the wage rate equals marginal revenue product. D) the wage rate equals marginal resource (labor) cost. Answer: B

Type: A Topic: 3 E: 527 MI: 283 57. A firm can hire six workers at a wage rate of $8 per hour but must pay $9 per hour to all of its employees to attract a seventh worker. The marginal wage cost of the seventh worker is: A) $9. B) $10. C) $15. D) $21. Answer: C

Type: A Topic: 3 E: 527 MI: 283 58. Suppose the MRP of a firm's twelfth worker is $22 and the worker's marginal wage cost is $16. We can say with certainty that the firm: A) is hiring labor in a competitive labor market at a wage rate of $16. B) is hiring labor in a monopsonistic labor market. C) will find it profitable to hire fewer workers. D) will find it profitable to hire more workers. Answer: D

Type: A Topic: 3 E: 526 MI: 282 59. In monopsony: A) each firm employs a small portion of the total supply of labor. B) the work force is highly mobile. C) the wage rate paid by the employer varies directly with the number of workers employed. D) the employer is a "wage taker." Answer: C

Type: A Topic: 3 E: 528 MI: 284 60. Which of the following is most likely to be an example of monopsony? A) the market for fast-food workers in a large summer resort town B) the market for card dealers in Las Vegas. C) the market for major league baseball umpires. D) the market for retail sales clerks in a major city. Answer: C

Type: A Topic: 3 E: 526-527 MI: 282-283 61. If a firm faces an upsloping labor supply curve (and there is no union or minimum wage), its: A) MRC curve is also upsloping. C) MRP curve is perfectly inelastic. B) MRC curve is perfectly elastic. D) MRP curve is also uploping. Answer: A

McConnell/Brue: Economics, 16/e

Page 885

Chapter 28: Wage Determination

Type: A Topic: 3 E: 527 MI: 283 62. A monopsonist's wage cost in hiring an additional worker is the: A) worker's wage rate. B) worker's wage rate plus the wage increases paid to all workers already employed. C) worker's wage rate adjusted for the lower price that must be charged for the extra output. D) marginal wage cost less the wage rate. Answer: B

Type: A Topic: 3 E: 526 MI: 282 63. A large hospital in a relatively small city finds that, if its demand for nurses increases, the wages of nurses will rise. We can say that the hospital: A) is a monopsonist. B) faces a perfectly elastic supply of nurses. C) is functioning in a perfectly competitive labor market. D) will confront a surplus of nurses. Answer: A

Type: A Topic: 3 E: 527 MI: 283 64. A monopsonist: A) boosts the wage rate above the competitive level to attract more workers. B) reduces the number of workers it employs so that it can pay each worker a lower wage rate. C) is a "wage taker." D) pays a wage rate equal to MRP. Answer: B

Type: A Topic: 3 E: 527 MI: 283 65. A monopsonistic employer: A) has a perfectly elastic labor supply curve. B) is necessarily a monopolist in the product market. C) confronts a marginal resource (labor) cost that is greater than the wage rate. D) confronts a marginal resource (labor) cost that is less than the wage rate. Answer: C

Type: A Topic: 3 E: 527 MI: 283 66. Other things equal, the monopsonistic employer will pay a: A) lower wage rate and hire fewer workers than will a purely competitive employer. B) higher wage rate but hire fewer workers than will a purely competitive employer. C) lower wage rate but hire a larger number of workers than will a purely competitive employer. D) higher wage rate and hire a larger number of workers than will a purely competitive employer. Answer: A

Type: A Topic: 3 E: 526 MI: 282 67. Assume the Ajax Mining Company hires 80 percent of the nonunion labor force of Mother Lode, New Mexico. Also, suppose that this labor force is highly immobile. Economists would describe this employer as a: A) monopolist. B) oligopolist. C) monopsonist. D) monopolistic competitor. Answer: C

McConnell/Brue: Economics, 16/e

Page 886

Chapter 28: Wage Determination

Type: A Topic: 3 E: 527 MI: 283 68. A monopsonistic employer in an unorganized (nonunion) labor market will: A) pay a wage rate less than labor's MRP. B) pay the same wage rate but hire fewer workers than if the market was purely competitive. C) hire the number of workers indicated by the intersection of the MRC and the labor supply curves. D) pay a wage rate in excess of labor's MRP. Answer: A

Type: A Topic: 3 E: 527 MI: 283 69. As compared to a purely competitive labor market, in a nonunionized monopsonistic labor market wages: A) and employment will both be lower. C) will be lower, but employment will be higher. B) will be higher, but employment will be lower. D) and employment will both be higher. Answer: A

Type: A Topic: 3 E: 527 MI: 283 70. A monopsonist pays a wage rate that is: A) less than the MRP of labor. B) equal to the firm's marginal resource (labor) cost. C) equal to the MRP of labor. D) greater than the MRP of labor. Answer: A

Type: A Topic: 3 E: 528 MI: 284 71. "Player drafts" of professional athletes: A) increase the competitiveness of the labor market for professional athletes. B) reduce the profitability of professional sports franchises. C) promote monopsony in the hire of professional athletes. D) increase salaries of professional athletes. Answer: C

Type: A Topic: 3 E: 527 MI: 283 72. Which of the following is not correct? A) Other things equal, a monopsonist will pay a lower wage rate than will a firm hiring labor competitively. B) A monopsonistic employer will pay workers a wage rate equal to their MRP. C) A purely competitive seller will pay workers a wage rate equal to their MRP. D) An imperfectly competitive seller will pay workers a wage rate equal to their MRP. Answer: B

Type: A Topic: 3 E: 527 MI: 283 73. A monopsonistic employer's marginal resource (labor) cost curve: A) is always more elastic than the labor supply curve. B) coincides with the labor supply curve. C) lies below the labor supply curve because the higher wage paid to an additional worker must also be paid to all other employed workers. D) lies above the labor supply curve because the higher wage paid to an additional worker must also be paid to all other employed workers. Answer: D

McConnell/Brue: Economics, 16/e

Page 887

Chapter 28: Wage Determination

Type: A Topic: 3 E: 527 MI: 283 74. The critical feature of a monopsonistic labor market is that the employer: A) has a perfectly elastic demand curve for labor. B) can hire any number of workers it chooses at the going wage rate. C) faces an upsloping labor supply curve. D) faces a perfectly inelastic labor supply curve. Answer: C

Type: A Topic: 3 E: 527 MI: 283 75. If a firm is a monopsonist in the hire of both labor and capital, it will obtain the profit-maximizing quantities of labor and capital when: A) MRPL/PL = MRPC/PC = 1. C) the MRP of labor equals the MRP of capital. B) MRPL/MRCL = MRPC/MRCC = 1. D) the MRC of labor equals the MRC of capital. Answer: B

Type: A Topic: 3 E: 527 MI: 283 76. If a firm is hiring variable resources D and F in imperfectly competitive input markets, it will maximize profits by employing D and F in such quantifies that: A) MRPD / MRCD = MRPF / MRCF = 1. C) MRPD / PD = MRPF / PF = 1. B) MRPD / MRCD = MRPF / MRCF . D) MRPD / PD = MRPF / PF . Answer: A

Type: A Topic: 3 E: 527 MI: 283 77. If an employer is a monopsonist: A) its MRC curve will lie below its labor demand curve. B) its labor supply and MRC curves will coincide and be perfectly elastic. C) it must also be a monopolist in the product market. D) its labor supply curve will be upsloping and the MRC curve will lie above it. Answer: D

Use the following to answer questions 78-79: Answer the next question(s) on the basis of the following supply information for a single firm in a particular labor market:

Wage rate $ 5 10 15 20 25

Quantity supplied 1 2 3 4 5

Type: T Topic: 3 E: 526 MI: 282 78. Refer to the above information. This labor supply curve demonstrates that: A) the firm is selling its output under imperfectly competitive conditions. B) the firm is selling its output under purely competitive conditions. C) higher wage rates must be paid to successive workers to overcome their higher opportunity costs. D) the firm is hiring labor under purely competitive conditions. Answer: C

McConnell/Brue: Economics, 16/e

Page 888

Chapter 28: Wage Determination

Type: T Topic: 3 E: 526 MI: 282 79. Refer to the above information. The marginal resource (labor) cost of the third worker is: A) $15. B) $25. C) $35. D) $45. Answer: B

Type: A Topic: 3 E: 527 MI: 283 80. Empirical studies suggest that, other things equal, the smaller the number of hospitals in a city, the lower are nurses' wages. This is evidence that: A) the labor markets of nurses are purely competitive. B) hospitals may possess some degree of monopsony power. C) the minimum wage does not apply to nurses. D) labor unions have been ineffective in increasing the wages of nurses. Answer: B

Use the following to answer questions 81-85:

Type: G Topic: 3 E: 527 MI: 283 81. If the above diagram was relevant to an individual firm, we could conclude that the firm is: A) a pure competitor in the hire of labor. B) a monopsonist in the hire of labor. C) selling its product in an imperfectly competitive market. D) selling its product in a purely competitive market. Answer: B

Type: G Topic: 3 E: 527 MI: 283 82. Refer to the above diagram. The MRC curve lies above the labor supply curve because: A) any number of workers can be hired at the going equilibrium wage rate. B) the firm must lower product price to increase its sales. C) the higher wage needed to attract additional workers must also be paid to the workers already employed. D) there is an inverse relationship between wage rate and the amount of labor employed. Answer: C

McConnell/Brue: Economics, 16/e

Page 889

Chapter 28: Wage Determination

Type: G Topic: 3 E: 527 MI: 283 83. Refer to the above diagram. Assuming no union or relevant minimum wage, the firm represented will hire: A) Q2 workers and pay a W4 wage rate. C) Q3 workers and pay a W2 wage rate. B) Q2 workers and pay a W1 wage rate. D) Q4 workers and pay a W1 wage rate. Answer: B

Type: G Topic: 3 E: 527 MI: 283 84. Refer to the above diagram. An industrial (inclusive) union could increase employment in this labor market: A) by negotiating any wage rate between W1 and W4. B) by negotiating a wage rate greater than W4. C) only if it accepted a wage rate below W1. D) only if it could shift the labor demand curve rightward. Answer: A

Type: G Topic: 3 E: 527 MI: 283 85. Refer to the above diagram. An industrial union could maximize employment by negotiating a wage rate of: A) W4. B) W3. C) W2. D) W1. Answer: C

Union models and licensure

Type: A Topic: 4 E: 528 MI: 284 86. A union may increase the demand for the services of its constituents by all of the tactics below except: A) successfully increasing labor productivity. B) lobbying for increases in public expenditures on the product it is producing. C) successfully advertising the product it is producing to private consumers. D) increasing the price of products that are complements for the one it is producing. Answer: D

Type: A Topic: 4 E: 529-530 MI: 285-286 87. The United Mine Workers is a good illustration of: A) how unions have increased wages but reduced job opportunities by shifting the supply-of-labor curve to the left. B) how unions have raised wages and increased job opportunities by increasing the demand for labor. C) inclusive unionism. D) exclusive unionism. Answer: C

Type: A Topic: 4 E: 529-530 MI: 285-286 88. Which of the following unions best represents the exclusive unionism model? A) the mine workers B) the teamsters C) the carpenters D) the steelworkers Answer: C

McConnell/Brue: Economics, 16/e

Page 890

Chapter 28: Wage Determination

Type: A Topic: 4 E: 530 MI: 286 89. Inclusive unionism is practiced mostly by: A) professional and semiprofessional employees. B) small unions comprised of skilled workers, such as the bricklayers. C) industrial unions. D) craft unions. Answer: C

Use the following to answer questions 90-93:

Type: G Topic: 4 E: 527 MI: 283 90. Refer to the above diagram. If this labor market is purely competitive, the wage rate and level of employment respectively will be: A) D and E. B) C and E. C) B and G. D) B and F. Answer: C

Type: G Topic: 4 E: 527 MI: 283 91. Refer to the above diagram. If this labor market is monopsonistic, the wage rate and level of employment respectively will be: A) D and E. B) C and F. C) B and F. D) A and F. Answer: D

Type: G Topic: 4 E: 530 MI: 286 92. Refer to the above diagram. Assume that an inclusive union is formed to bargain with the monopsonistic employer of the previous question. To what level can this union increase the wage rate without causing the number of jobs to decline below that which the monopsonist would otherwise have provided? A) D minus A B) D C) C D) B Answer: C

Type: G Topic: 4 E: 530 MI: 286 93. Refer to the above diagram. If an inclusive union seeks to maximize the number of jobs available for its members, what wage rate will it seek to impose on the monopsonist? A) D minus A B) F C) C D) B Answer: D

McConnell/Brue: Economics, 16/e

Page 891

Chapter 28: Wage Determination

Type: A Topic: 4 E: 529-530 MI: 285-286 94. A craft union attempts to increase wage rates by: A) equating the MRP and the MRC curves. B) shifting the labor supply curve to the left. Answer: B

C) shifting the labor supply curve to the right. D) shifting the MRP curve to the right.

Type: A Topic: 4 E: 530 MI: 286 95. Occupational licensing has much the same effect as: A) inclusive unionism. B) exclusive unionism. C) bilateral monopoly. Answer: B

D) monopsony.

Type: A Topic: 4 E: 530 MI: 286 96. Occupational licensing: A) functions essentially the same as inclusive unionism. B) attracts large numbers of workers and therefore depresses wages. C) often restricts occupational entry and raises the incomes of licensees. D) has been declared illegal in the majority of states. Answer: C

Type: A Topic: 4 E: 530 MI: 286 97. Occupational licensing can best be understood in terms of: A) the inclusive unionism model. C) the bilateral monopoly model. B) the exclusive unionism model. D) the monopsony model. Answer: B

Type: A Topic: 4 E: 529-530 MI: 285-286 98. If an exclusive union is successful in restricting the supply of labor, the: A) wage rate will rise. B) the quantity of labor demanded will rise: C) the number of job opportunities in the firm or industry will increase. D) the demand for labor curve will shift leftward. Answer: A

Type: A Topic: 4 E: 530 MI: 286 99. If an industrial union is formed to bargain with a monopsonistic employer, then in this labor market: A) the resulting wage rate will necessarily be above the competitive level. B) employment may either increase or decrease. C) employment will increase. D) employment will decrease. Answer: B

Type: A Topic: 4 E: 529-530 MI: 285-286 100. The electricians union is a good example of: A) exclusive unionism. B) countervailing power. C) how unions can simultaneously increase wage rates and employment by increasing the demand for labor. D) inclusive unionism. Answer: A

McConnell/Brue: Economics, 16/e

Page 892

Chapter 28: Wage Determination

Type: A Topic: 4 E: 529-530 MI: 285-286 101. Exclusive unionism attempts to increase wage rates by: A) decreasing the supply of labor. B) setting a minimum or standard wage above the competitive level. C) increasing the supply of labor. D) increasing the demand for labor through productivity increases. Answer: A

Type: A Topic: 4 E: 528-530 MI: 284-286 102. Labor unions may attempt to raise wage rates by: A) increasing the supply of labor. B) forcing employers, under the threat of a strike, to pay above-equilibrium wage rates. C) decreasing the demand for labor. D) increasing the price of complementary resources. Answer: B

Type: A Topic: 4 E: 528 MI: 284 103. Construction workers frequently sponsor political lobbying in support of greater public spending on highways and public buildings. One reason they do this is to: A) restrict the supply of construction workers. B) increase the elasticity of demand for construction workers. C) increase the demand for construction workers. D) increase the price of substitute inputs. Answer: C

Type: A Topic: 4 E: 529 MI: 285 104. Unions often oppose increases in the prices of complementary inputs (for example, truckdrivers may oppose increases in taxes on diesel fuel). They do this because increases in the prices of complementary inputs might: A) increase the supply of competing labor through the output effect. B) increase the supply of competing labor through the substitution effect. C) decrease the demand for union labor through the output effect. D) decrease the demand for union labor through the substitution effect. Answer: C

Type: A Topic: 4 E: 529-530 MI: 285-286 105. Craft unions: A) attempt to organize workers at all skill levels in a firm or industry. B) have been declared illegal by Federal legislation. C) only organize workers who have a particular skill. D) attempt to increase the supply of their particular type of labor. Answer: C

Type: A Topic: 4 E: 529 MI: 285 106. Labor unions are restrained in their wage demands because: A) legislation limits annual increases in nominal wages to 6 percent. B) the labor demand curve is downsloping. C) marginal wage cost curves lie above labor supply curves in most labor markets. D) most unions deal with monopsonists who have superior bargaining power. Answer: B

McConnell/Brue: Economics, 16/e

Page 893

Chapter 28: Wage Determination

Type: G Topic: 4 E: 529 MI: 285 107.

A shift in union labor demand from D1 to D2 in the above diagram might be the result of: A) a refusal by union members to buy the product they are producing. B) an increase in tariffs on products competing with those produced by relevant union workers. C) increases in the prices of complementary inputs. D) a strike (workstoppage) by the union. Answer: B

Type: A Topic: 4 E: 529 MI: 285 108. A union might increase the demand for the labor services of its members by: A) decreasing the demand for the product it is producing. B) enhancing the productivity of its members. C) decreasing the prices of substitute inputs. D) increasing the prices of complementary inputs. Answer: B

Type: F Topic: 4 E: 531 MI: 287 109. Authoritative estimates suggest that currently union workers on the average: A) achieve no wage advantage over nonunion workers in the same occupation. B) realize a 5 percent wage advantage over nonunion workers in the same occupation. C) realize a 20-30 percent wage advantage over nonunion workers in the same occupation. D) realize a 15 percent wage advantage over nonunion workers in the same occupation. Answer: D

Type: A Topic: 4 E: 531-532 MI: 287-288 110. In a labor market characterized by bilateral monopoly the wage rate will: A) be logically indeterminate. B) be established at the level desired by the union. C) be established at the level desired by the employer. D) always be established at the competitive level. Answer: A

Type: D Topic: 4 E: 531 MI: 287 111. If a single large employer bargains with an inclusive union, the resulting labor market model can best be described as: A) a cartel. B) countervailing power. C) a bilateral monopoly. D) an internal labor market. Answer: C

McConnell/Brue: Economics, 16/e

Page 894

Chapter 28: Wage Determination

Type: D Topic: 4 E: 531 MI: 287 112. Bilateral monopoly occurs where: A) a monopsonistic employer bargains with an inclusive union. B) a monopsonistic employer bargains with an exclusive union. C) a craft union bargains with a purely competitive employer. D) an industrial union bargains with a purely competitive employer. Answer: A

Use the following to answer questions 113-117:

Type: G Topic: 4 E: 531 MI: 287 113. Refer to the above labor market diagrams. A monopsonistic labor market is represented by Figure: A) 5. B) 4. C) 3. D) 2. Answer: C

Type: G Topic: 4 E: 529 MI: 285 114. Refer to the above labor market diagrams. The tactics of exclusive unionism are portrayed in Figure: A) 4. B) 3. C) 2. D) 1. Answer: D

Type: G Topic: 4 E: 530 MI: 286 115. Refer to the above labor market diagrams. The tactics of inclusive unionism are shown in Figure(s): A) 5 only. B) 3 only. C) 4 and 5. D) 1 and 2. Answer: C

McConnell/Brue: Economics, 16/e

Page 895

Chapter 28: Wage Determination

Type: G Topic: 4 E: 531 MI: 287 116. Refer to the above labor market diagrams. The case of bilateral monopoly is represented by Figure: A) 5. B) 4. C) 2. D) 1. Answer: A

Type: G Topic: 4 E: 530 MI: 286 117. Refer to the above labor market diagrams. The economic impact of occupational licensing can best be demonstrated through Figure: A) 4. B) 3. C) 2. D) 1. Answer: D

Use the following to answer questions 118-122:

Type: G Topic: 4 E: 527 MI: 283 118. Refer to the above labor market diagram where D is the labor demand curve, S is the labor supply curve, and MRC is the marginal resource (labor) cost curve. If this were a purely competitive labor market, the equilibrium wage rate and level of employment would be: A) $5 and 3 respectively. C) $7 and 5 respectively. B) $6 and 4 respectively. D) $8 and 3 respectively. Answer: C

Type: G Topic: 4 E: 527 MI: 283 119. Refer to the above labor market diagram where D is the labor demand curve, S is the labor supply curve, and MRC is the marginal resource (labor) cost curve. If this were a monopsonistic labor market, the equilibrium wage rate and level of employment would be: A) $5 and 3 respectively. C) $7 and 5 respectively. B) $6 and 4 respectively. D) $8 and 3 respectively. Answer: A

McConnell/Brue: Economics, 16/e

Page 896

Chapter 28: Wage Determination

Type: G Topic: 4 E: 531 MI: 287 120. Refer to the above labor market diagram where D is the labor demand curve, S is the labor supply curve, and MRC is the marginal resource (labor) cost curve. If an inclusive union was formed and was able to get the monopsonist to agree to a $7 wage rate, then the monopsonist would: A) reduce employment from 5 to 3 workers. C) increase employment from 3 to 5 workers. B) reduce employment from 5 to 2 workers. D) not alter its level of employment. Answer: C

Type: G Topic: 4 E: 531 MI: 287 121. Refer to the above labor market diagram where D is the labor demand curve, S is the labor supply curve, and MRC is the marginal resource (labor) cost curve. If an inclusive union was able to get the monopsonist to pay a $6 wage rate, then: A) the supply curve would be perfectly elastic for the first four workers, but the MRC curve would be unaffected. B) the supply curve would be perfectly elastic for all workers and the MRC curve would coincide with it. C) the supply curve would be perfectly elastic for the first four workers and the MRC would be $6 for the first four workers. D) eight workers would be hired. Answer: C

Type: G Topic: 4 E: 531 MI: 287 122. Refer to the above labor market diagram where D is the labor demand curve, S is the labor supply curve, and MRC is the marginal resource (labor) cost curve. An inclusive union could increase the level of employment above that which the monopsonist would provide if it could get the monopsonist to agree to any wage rate: A) below $7. B) between $5 and $8. C) above $5. D) above $8. Answer: B

Minimum wage

Type: A Topic: 5 E: 532 MI: 288 123. Minimum-wage legislation is less likely to have adverse effects on employment when the: A) affected labor market is monopsonistic. C) derived demand for labor is shifting to the left. B) economy has high unemployment. D) affected labor market is perfectly competitive. Answer: A

Type: A Topic: 5 E: 532 MI: 288 124. Critics of minimum-wage legislation argue that it: A) keeps inefficient producers in business. B) reduces employment. Answer: B

C) undermines incentives to work. D) is deflationary.

Type: A Topic: 5 E: 532 MI: 288 125. Many economists are critical of the minimum wage because they believe that it: A) hurts the efforts of labor unions. B) reduces the number of available job opportunities. C) conflicts with policies designed to equalize the distribution of income. D) causes labor shortages in affected markets. Answer: B

McConnell/Brue: Economics, 16/e

Page 897

Chapter 28: Wage Determination

Type: A Topic: 5 E: 532 MI: 288 126. Unions might support a higher minimum wage because: A) their constitutions obligate them to do so. B) they feel a higher minimum wage will lower labor's tax payments for welfare programs. C) a higher minimum wage makes less-skilled workers less substitutable for union workers. D) the minimum wage is better targeted than are alternative income-maintenance programs. Answer: C

Type: A Topic: 5 E: 532 MI: 288 127. Critics of the minimum wage argue that as an antipoverty device it is "poorly targeted." By this they mean that: A) the minimum wage only applies to a small percentage of the labor force. B) many who benefit from the minimum wage are not poor. C) the government has been unable to enforce the minimum wage. D) the average level of wages in the economy is considerably higher than the minimum wage. Answer: B

Type: A Topic: 5 E: 532-533 MI: 288-289 128. If the minimum wage is set too high, in some labor markets we can expect to see: A) a shortage of labor. C) a surplus of labor. B) an increase in on-the-job training. D) a decline in wage costs. Answer: C

Type: A Topic: 5 E: 533 MI: 289 129. Increases in the Federal minimum wage during the 1990s: A) reduced the demand for workers who were earning more than the minimum wage. B) demonstrated that the demand for teenage labor is highly elastic. C) produced smaller decreases in teenage employment than did previous minimum wage hikes. D) helped reduce poverty substantially in the United States. Answer: C

Type: A Topic: 5 E: 532 MI: 288 130. According to some supporters of the minimum wage, it has very small or even nonexistent negative employment effects because: A) the demand for minimum wage labor is highly elastic. B) it reduces turnover among minimum wage workers, prompts employers to use them more efficiently, and thus raises their average productivity. C) it encourages teenagers to stay in school. D) employers substitute lower fringe benefits for higher pay, keeping their compensation costs the same. Answer: B

McConnell/Brue: Economics, 16/e

Page 898

Chapter 28: Wage Determination

Wage differentials; human capital

Type: A Topic: 6 E: 533 MI: 289 131. Wage differentials can arise from: A) both the demand-side and supply-side of labor markets. B) the demand-side of labor markets only. C) the supply-side of labor markets only. D) neither the demand-side or supply-side of labor markets. Answer: A

Type: A Topic: 6 E: 533 MI: 289 132. If all workers are homogeneous, all jobs are equally attractive to workers, and labor markets are perfectly competitive: A) compensating differences would cause wage differentials. B) noncompeting groups of workers would result in wage differentials. C) all workers would receive the same wage rate. D) worker mobility would occur such that wage differentials would widen. Answer: C

Type: A Topic: 6 E: 533 MI: 289 133. Wage differentials may result from all the following except: A) differences in the nonmonetary aspects of various occupations. B) differences in the education and skills of workers. C) geographic and sociological immobilities of workers. D) the tendency of qualified workers to move from lower pay jobs to higher pay jobs. Answer: D

Type: A Topic: 6 E: 533 MI: 289 134. Suppose all workers are identical, but working for Ajax is more pleasant than working for Acme. In all other nonwage aspects the two firms offer the same job characteristics. We would expect: A) wage rates at Ajax to be higher than at Acme. B) wage rates at Ajax to be lower than at Acme. C) wage rates at Ajax and Acme to be the same. D) workers at Ajax would have to be monitored more closely than at Acme. Answer: B

Type: A Topic: 6 E: 534-535 MI: 290-291 135. Noncompeting groups of workers are the result of: A) differences in the age-earnings profiles of workers. B) differences in the "job tastes" of workers. C) differences in the innate and acquired abilities of workers. D) geographic immobilities. Answer: C

McConnell/Brue: Economics, 16/e

Page 899

Chapter 28: Wage Determination

Type: A Topic: 6 E: 535 MI: 291 136. Compensating differences in wages: A) compensate workers for differences in their human capital. B) are wage differences that compensate for differences in the desirability of jobs. C) describe the tendency for the wages of all occupations to adjust to the median level. D) do not exist if jobs have different nonmonetary characteristics. Answer: B

Type: A Topic: 6 E: 535 MI: 291 137. Compensating differences in wages pay workers for: A) differences in worker training and skills. B) differences in the nonmonetary characteristics of jobs. C) geographic immobilities. D) discrimination in hiring and firing. Answer: B

Type: A Topic: 6 E: 535 MI: 291 138. Which of the following factors is not relevant in explaining the persistence of wage differentials? A) labor immobilities C) free public education B) compensating differences D) noncompeting groups Answer: C

Type: D Topic: 6 E: 535 MI: 291 139. The idea of compensating differences is used: A) by inclusive unions as an argument in bargaining for wage rate increases. B) to justify the application of minimum wages to low-wage labor markets. C) to explain the divergence between wage rates and marginal resource cost. D) to explain wage rate differences based on differing nonmonetary aspects of jobs. Answer: D

Type: A Topic: 6 E: 535 MI: 291 140. The concept of investment in human capital indicates that: A) union workers are better educated and more productive than nonunion workers. B) expenditures on education can be explained in essentially the same way as expenditures on machinery and equipment. C) worker productivity correlates negatively with annual earnings. D) the level of education is unrelated to the level of one's income. Answer: B

Type: A Topic: 6 E: 535 MI: 291 141. Data on education and earnings reveal: A) negative age-earnings profiles for male workers. B) no relationship between the two. C) a positive relationship between the two. D) a negative relationship between the two. Answer: C

McConnell/Brue: Economics, 16/e

Page 900

Chapter 28: Wage Determination

Type: F Topic: 6 E: 535 MI: 291 142. According to age-earnings data, A) lower educated workers have similar earnings at age 65 as higher educated workers. B) investments in education result in higher earnings. C) high earnings are due to motivation and innate ability, rather than education. D) there is no clear relationship between education and worker productivity. Answer: B

Type: F Topic: 6 E: 535 MI: 291 143. The earnings of highly educated workers: A) rise more slowly than those of less-educated workers. B) rise more rapidly than those of less-educated workers. C) rise at about the same rate as those of less-educated workers. D) stagnate earlier than do those of less-educated workers. Answer: B

Type: A Topic: 6 E: 535 MI: 291 144. Economists regard expenditures on education as investments because: A) they are subject to tax deductions at the same rate as are expenditures on machinery and equipment. B) education is economically beneficial at the same time it is being acquired. C) such expenditures are current costs that are intended to enhance future earnings. D) they differ from expenditures on health and worker mobility. Answer: C

Type: A Topic: 6 E: 535 MI: 291 145. Which of the following involves the creation of human capital? A) the XYZ Corporation upgrades the machinery on its assembly line B) Jones receives apprenticeship training as a carpenter C) Smith buys 30 shares of common stock D) a retired person decides to reenter the labor force Answer: B

Type: D Topic: 6 E: 535 MI: 291 146. Human capital is best defined as: A) the productive skills and knowledge that workers acquire from education and training. B) the substitution of labor for machinery in the production process. C) any piece of machinery that must be combined with labor to be productive. D) the exchange of money for real assets. Answer: A

Type: A Topic: 6 E: 536-537 MI: 292-293 147. Which of the following is a market imperfection that might explain persistent wage differentials within an occupation? A) movement of labor from lower-wage to higher-wage jobs B) readily available information about job opportunities and pay C) principal-agent problems D) discrimination Answer: D

McConnell/Brue: Economics, 16/e

Page 901

Chapter 28: Wage Determination

Type: A Topic: 6 E: 536 MI: 292 148. Which of the following is a market imperfection that might explain persistent wage differentials within an occupation? A) geographical immobility of workers B) readily available information about job opportunities and pay C) principal-agent problems D) compensating wage differentials Answer: A

Pay and performance

Type: A Topic: 7 E: 537 MI: 293 149. The principal-agent problem arises in labor markets because: A) a firm may realize excessively large profits. B) workers may provide less-than-expected work effort. C) compensating wage differences do not pay for differences in the nonmonetary aspects of jobs. D) human capital investments vary among workers. Answer: B

Type: A Topic: 7 E: 537 MI: 293 150. The principal-agent problem arises primarily because: A) principals and agents share a common interest, leading to free-rider problems. B) principals and agents are in an adversarial role, sharing no common interests. C) principals pursue some of their own objectives that may conflict with the objectives of the agents. D) agents pursue some of their own objectives that may conflict with the objectives of the principals. Answer: D

Type: A Topic: 7 E: 537 MI: 293 151. Which one of the following best exemplifies the principal-agent problem in the employer-employee relationship? A) A worker takes 20 minute coffee-breaks although the employer allots only 15 minutes for this purpose. B) A worker is on the job 50 hours per week although only 40 hours are required for promotion. C) A worker opts for early retirement in response to the firm's incentive plan. D) A worker's productivity is independent of the wage paid. Answer: A

Type: D Topic: 7 E: 538 MI: 294 152. In the context of labor markets, shirking refers to: A) the nonmonetary disadvantages of certain jobs. B) the neglecting or evading of work. C) the elimination of monitoring costs. D) any scheme where pay is directly related to worker output. Answer: B

McConnell/Brue: Economics, 16/e

Page 902

Chapter 28: Wage Determination

Type: A Topic: 7 E: 538 MI: 294 153. Traveling sales representative Harold Hill only calls on clients four days a week rather than the five days expected by his employer. This is an example of: A) equalizing differences. C) shirking. B) a nonmonetary job disadvantage. D) the free-rider problem. Answer: C

Type: D Topic: 7 E: 538 MI: 294 154. The idea of efficiency wages is that: A) the wages of each type of labor must be proportionate to their marginal products. B) the wages of each type of labor must be equal to their marginal products. C) firms might get greater work effort by paying above-equilibrium wage rates. D) workers are more diligent when paid below-equilibrium wages. Answer: C

Type: A Topic: 7 E: 538 MI: 294 155. Paying an above-equilibrium wage rate might reduce unit labor costs by: A) permitting the firm to attract lower-quality labor. B) increasing the cost to workers of being fired for shirking. C) increasing voluntary worker turnover. D) increasing the supply of labor. Answer: B

Type: A Topic: 7 E: 538 MI: 294 156. One implication of efficiency wages is that: A) labor turnover is reduced as wages are reduced. B) the market-clearing wage always exceeds the efficiency wage. C) worker productivity falls as wage rates rise beyond the equilibrium wage. D) if the efficiency wage exceeds the market-clearing wage, permanent unemployment may result. Answer: D

Type: D Topic: 7 E: 537 MI: 293 157. Compensation paid in proportion to the number of units of personal output best describes: A) royalties. B) profit-sharing. C) bonuses. D) piece rates. Answer: D

Type: A Topic: 7 E: 538 MI: 294 158. A firm might choose to pay its employees a wage higher than that which would clear the market because: A) the higher wage raises the opportunity cost of shirking. B) the higher wage may shift the labor demand curve to the left. C) the firm will have higher turnover, allowing "new blood" to invigorate older workers who have a greater tendency to shirk. D) this policy reduces the proportion of experienced to inexperienced workers, resulting in a lower overall wage bill. Answer: A

McConnell/Brue: Economics, 16/e

Page 903

Chapter 28: Wage Determination

Type: A Topic: 7 E: 537 MI: 293 159. Author Joe Writer receives $2 per book sold. This payment is a(n): A) piece rate. B) royalty. C) bonus. D) example of profit-sharing. Answer: B

Type: A Topic: 7 E: 538 MI: 294 160. For the firm, the major goal of profit sharing plans is to: A) force workers to incur some of the business risk. B) overcome the monopsony problem of having to pay higher wages to attract additional workers. C) overcome the principal-agent problem by better aligning the workers' interests with those of the firm. D) reduce total compensation payments. Answer: C

Type: A Topic: 7 E: 538 MI: 294 161. Stock options as a form of payment are designed to: A) elude the equal-pay-for-equal work provisions of the Federal antidiscrimination law. B) boost the overall earnings of minimum wage workers. C) offset monopsony. D) address the principal-agent problem. Answer: D

Type: A Topic: 7 E: 537 MI: 293 162. One of the potential negative side-effects of pay in the form of sales commissions is: A) a greater incentive for sales people to engage in unethical or fraudulent sales practices that may eventually cause legal problems for the firm. B) increased volatility of sales revenue for the firm. C) the potential that pay levels may get so high that they will increase a firm's marginal wage cost more than its marginal revenue product. D) an increased likelihood of shirking by workers. Answer: A

Consider This Questions

Type: F E: 536 MI: 292 Status: New 163. (Consider This) The main idea highlighted in the story about artist Pablo Picasso is: A) derived demand. B) human capital. C) opportunity cost. D) occupational licensure. Answer: B

Type: A E: 536 MI: 292 Status: New 164. (Consider This) The story about artist Pablo Picasso illustrates the point that: A) the demand for labor is a derived demand. B) geographical immobility impedes an efficient allocation of labor resources. C) principal-agent problems can occur in unusual settings. D) present skills reflect past accumulations of human capital. Answer: D

McConnell/Brue: Economics, 16/e

Page 904

Chapter 28: Wage Determination

Last Word Questions

Type: F E: 539 MI: 295 Status: New 165. (Last Word) Chief executive officers (CEOs) of large American corporations: A) have higher average salaries than CEOs of similar-size firms in other industrial nations. B) have similar salaries, on average, to CEOs of same-size firms in other industrial nations. C) are, on average, older than CEOs of same-size firms in other industrial nations. D) are, on average, younger than CEOs of same-size firms in other industrial nations. Answer: A

Type: F E: 539 MI: 295 Status: New 166. (Last Word) In 2002, the five highest paid chief executive officers (CEOs) in the United States each received total pay exceeding: A) $200 million. B) $150 million. C) $100 million. D) $50 million. Answer: D

Type: F E: 539 MI: 295 Status: New 167. (Last Word) The dispute over the pay of chief executive officers (CEOs) of U.S. corporations hinges on whether or not such pay: A) should be granted for past performance or for current performance. B) is determined in a competitive labor market or in a monopsonistic labor market. C) is justified on productivity grounds or mainly reflects an overestimation of CEO importance by corporate boards of directors. D) should contain performance incentives such as stock options, stock shares, or bonuses. Answer: C

True/False Questions

Type: A E: 524 MI: 280 168. Marginal resource (labor) cost will exceed the wage rate when there is imperfect competition in the hire of labor. Answer: True

Type: A E: 524 MI: 280 169. The rising general level of real wages in the United States has occurred because growing population has increased the supply of labor relative to the demand for it. Answer: False

Type: A E: 532 MI: 288 170. Advocates of the minimum wage argue that its effects should be analyzed within the context of a dynamic and imperfectly competitive labor market. Answer: True

Type: A E: 526 MI: 282 171. Marginal resource (labor) cost will always exceed the wage rate when the employer is selling its product in an imperfectly competitive market. Answer: False

McConnell/Brue: Economics, 16/e

Page 905

Chapter 28: Wage Determination

Type: A E: 526 MI: 282 172. A monopsonistic employer may sell its product in a competitive market. Answer: True

Type: A E: 530 MI: 286 173. Industrial unions are more likely to increase wage rates by restricting the supply of labor than are craft unions. Answer: False

Type: A E: 525 MI: 281 174. The labor supply curve of a purely competitive firm is perfectly inelastic. Answer: False

Type: A E: 527 MI: 283 175. The monopsonist in a nonunionized labor market pays a wage rate below the MRP of labor. Answer: True

Type: A E: 530 MI: 286 176. Inclusive unions restrict the number of jobs directly by shifting the labor supply curve to the left; exclusive unions restrict the number of jobs by imposing above-equilibrium wage rates on the employer. Answer: False

Type: A E: 532 MI: 288 177. Critics of the minimum wage contend that higher minimums cause employers to move up their labor demand curves, reducing employment of low-wage workers. Answer: True

Type: A E: 537 MI: 293 178. Seniority pay is the same as piece rate pay. Answer: False

Type: A E: 537 MI: 293 179. The principal-agent problem in labor markets arises because of the possibility of shirking by workers. Answer: True

Type: A E: 538 MI: 294 180. Efficiency wages are determined at below-equilibrium levels. Answer: False

Type: D E: 535 MI: 291 181. Human capital investment refers to spending on education and worker training. Answer: True

McConnell/Brue: Economics, 16/e

Page 906

Chapter 28: Wage Determination

Type: A E: 534-535 MI: 290-291 182. Noncompeting groups of workers are the result of geographic immobilities. Answer: False

Type: D E: 538 MI: 294 183. Shirking refers to the behavior of workers who provide less-than-expected effort on the job. Answer: True

Type: A E: 537 MI: 293 184. Piece-rates may not be appropriate pay in some situations because they might reduce product quality. Answer: True

McConnell/Brue: Economics, 16/e

Page 907

CHAPTER 29

Rent, Interest, and Profit

Topic 1. 2. 3. 4. 5. 6. 7. 8. 9. Economic rent Land taxation Interest rate concept Loanable funds theory of interest Structure and impact of interest rates Nominal and real interest rates Usury laws Profit Income shares Consider This Last Word True-False

Question numbers 1-21 22-27 28-33 34-47 48-56 57-64 65-69 70-79 80-86 87-88 89-92 93-103

____________________________________________________________

_______________________________________

____________________________________________________________

_______________________________________

Multiple Choice Questions Economic rent

Type: D Topic: 1 E: 542 MI: 298 1. Economic or pure rent is: A) a payment made for the use of housing, factory buildings, or capital goods. B) a payment for resources used in the production of "free goods." C) a payment for the use of those resources whose supply is perfectly elastic. D) the price paid for the use of land and other nonreproducible resources. Answer: D

Type: A Topic: 1 E: 543 MI: 299 2. To say that land rent performs no incentive function means that: A) higher rental payments will not bring forth a larger quantity of land. B) rent is not a cost to specific firms but it is a cost from the standpoint of the economy as a whole. C) rent does not allocate land in terms of productive efficiency. D) rent tends to allocate land into the most productive uses. Answer: A

Chapter 29: Rent, Interest, and Profit

Type: D Topic: 1 E: 542 MI: 298 3. Which of the following statements is correct? A) Economic profits can properly be regarded as the salaries received by the hired managers of corporations. B) Economic rent is a price paid for productive resources whose supply is perfectly inelastic. C) Economic profits would be nonexistent in a dynamic, purely competitive economy. D) Economic or pure profit is the minimum return which entrepreneurs must receive to continue in a particular line of production. Answer: B

Type: D Topic: 1 E: 543 MI: 299 4. Economic rent refers to the price paid for land and other natural resources that: A) are fixed in total supply. C) vary inversely with their market prices. B) vary directly with their market prices. D) are available in nearly unlimited quantities. Answer: A

Use the following to answer questions 5-6:

Type: G Topic: 1 E: 543 MI: 299 5. Refer to the above diagram. Land: A) will cease to be used in production if demand falls below D4. B) would be a free resource if demand is D4 or less. C) would be an economic (scarce) resource in the case of all four demand curves. D) would be a free resource in the case of all four demand curves. Answer: B

Type: G Topic: 1 E: 543 MI: 299 6. Refer to the above diagram. If demand is D2, a tax of A per acre will: A) encourage the substitution of land for other productive resources. B) lower the cost of land to individual farmers. C) reduce the demand for land to, say, D3 or D4. D) not affect the quantity of land available to society. Answer: D

McConnell/Brue: Economics, 16/e

Page 910

Chapter 29: Rent, Interest, and Profit

Type: G Topic: 1 E: 543 MI: 299 7. Landowners will not receive any rent so long as: A) there is any tax on land. B) the supply and demand curves for land intersect. C) the supply curve of land is perfectly inelastic. D) the supply curve lies entirely to the right of the demand curve. Answer: D

Type: A Topic: 1 E: 543 MI: 299 8. The incentive function of prices: A) indicates that price increases bring forth more of a resource. B) is the idea that competitive markets will always clear. C) applies to all resources. D) only applies to land. Answer: A

Type: A Topic: 1 E: 543 MI: 299 9. The demand for farmland will increase if: A) the demand for food decreases. B) technological advances make land more productive. C) the price of farm labor increases and the output effect exceeds the substitution effect. D) the supply of farmland increases. Answer: B

Type: A Topic: 1 E: 543 MI: 299 10. The supply of land is: A) almost perfectly inelastic. B) negatively sloped. Answer: A

C) relatively elastic.

D) perfectly elastic.

Type: D Topic: 1 E: 543 MI: 299 11. Economic rent is: A) nonexistent in a static, purely competitive economy. B) the price paid for a resource that has a perfectly inelastic supply. C) the price paid for a resource that has a perfectly elastic supply. D) equal to the pure rate of interest if all markets are competitive. Answer: B

Type: A Topic: 1 E: 543 MI: 299 12. Which of the following is correct? A) Although land has no production cost from society's viewpoint, rental payments are costs to individual producers. B) Land rent is not a cost to either society or to individual producers. C) Although land rent is a cost from society's viewpoint, it is not a cost to individual producers. D) Land rent is a cost to both society and individual producers. Answer: A

McConnell/Brue: Economics, 16/e

Page 911

Chapter 29: Rent, Interest, and Profit

Type: A Topic: 1 E: 543 MI: 299 13. The total supply of land is: A) upsloping. B) perfectly elastic. Answer: C

C) perfectly inelastic. D) greater in the short run than in the long run.

Type: A Topic: 1 E: 543-545 MI: 299-301 14. The rent paid for the pasture land used to graze cattle would increase if: A) the productivity of the land increased. C) oil deposits were discovered on the land. B) people decided to consume more beef. D) any of the above occurred. Answer: D

Use the following to answer questions 15-16:

Type: G Topic: 1 E: 543 MI: 299 15. Refer to the above diagrams. Assume that only wheat can be grown on the three grades of land shown in Figures a, b, and c. Also assume that identical amounts of labor, capital, and other needed inputs are used in farming each grade of land. On the basis of these three figures we: A) can say that the land in Figure a is most productive. B) can say that the land in Figure b is most productive. C) can say that the land in Figure c is most productive. D) cannot compare the productivity of the three grades of land. Answer: C

Type: G Topic: 1 E: 543 MI: 299 16. Refer to the above diagrams. Assume that only wheat can be grown on the three grades of land shown in Figures a, b, and c. Also assume that identical amounts of labor, capital, and other needed inputs are used in farming each grade of land. On the basis of these three figures we can say that: A) the land shown in both Figures (a) and (b) is a "free good." B) the land shown in Figure (a) only is a "free good." C) the land shown in all three figures is a "free good." D) land is not a "free good" in any of the three figures. Answer: B

McConnell/Brue: Economics, 16/e

Page 912

Chapter 29: Rent, Interest, and Profit

Use the following to answer questions 17-21:
(1) Land Rent $400 300 200 100 0 (2) Quantity Supplied 60 60 60 60 60 (3) Quantity Demanded 40 50 60 70 80 (4) Quantity Demanded 30 40 50 60 70 (5) Quantity Demanded 10 20 30 40 50

Type: T Topic: 1 E: 543 MI: 299 Status: New 17. Refer to the above table, in which the values for columns (2) through (5) are in acres. If the relevant columns are (1), (2), and (3), land rent will be: A) $100 per acre. B) $200 per acre. C) $300 per acre. D) $400 per acre. Answer: B

Type: T Topic: 1 E: 543 MI: 299 Status: New 18. Refer to the above table, in which the values for columns (2) through (5) are in acres. If the relevant columns are (1), (2), and (4), land rent will be: A) $100 per acre. B) $200 per acre. C) $300 per acre. D) $400 per acre. Answer: A

Type: T Topic: 1 E: 543 MI: 299 Status: New 19. Refer to the above table, in which the values for columns (2) through (5) are in acres. If the relevant columns are (1), (2), and (5), land rent will be: A) $0 per acre. B) $100 per acre. C) $200 per acre. D) $300 per acre. Answer: A

Type: T Topic: 1 E: 543 MI: 299 Status: New 20. Refer to the above table, in which the values for columns (2) through (5) are in acres. Positive land rent will occur if the relevant columns are: A) (1), (2), and (3) only. C) (1), (2), and (4) and (1), (2), and (5). B) (1), (2), and (3) and (1), (2), and (5). D) (1), (2), and (3) and (1), (2), and (4). Answer: D

Type: T Topic: 1 E: 543 MI: 299 Status: New 21. Refer to the above table, in which the values for columns (2) through (5) are in acres. Zero land rent will occur if the relevant columns are: A) (1), (2), and (3) only. C) (1), (2), and (5) only. B) (1), (2), and (4) and (1), (2), and (5). D) (1), (2), and (3) and (1), (2), and (4). Answer: C

McConnell/Brue: Economics, 16/e

Page 913

Chapter 29: Rent, Interest, and Profit

Land taxation

Type: F Topic: 2 E: 544 MI: 300 22. The economist who advocated a single tax on land was: A) Adam Smith. B) John Maynard Keynes. C) Henry George. Answer: C

D) Milton Friedman.

Type: A Topic: 2 E: 544 MI: 300 23. A unique characteristic of taxes on economic rents is that such taxes: A) stimulate aggregate production. C) are paid by consumers. B) do not lead to a reallocation of the resource. D) are always regressive. Answer: B

Type: A Topic: 2 E: 544 MI: 300 24. Henry George's single tax movement was based on the argument that: A) the tax structure should consist solely of a highly progressive tax on nonwage incomes. B) interest is unearned income and should be taxed away by government. C) in less developed countries the supply of and demand for land will be such that land will be a free good and therefore capable of bearing sizable taxes. D) a high tax on land rent is justified because land rent performs no incentive function. Answer: D

Type: F Topic: 2 E: 544 MI: 300 25. In his Progress and Poverty, Henry George argued that: A) poverty is associated with the personal characteristics of individuals and therefore cannot be remedied by government antipoverty programs. B) economic rent could be heavily taxed without impairing the supply of land or therefore the production capacity of the economy. C) rents should not be taxed because rental income is the basic source of saving, which ultimately permits a high level of investment and economic growth. D) taxes on rents are undesirable because they have a severe disincentive effect on landlords. Answer: B

Type: F Topic: 2 E: 544 MI: 300 26. Henry George advocated a single tax on: A) real capital. B) entrepreneurial profits. Answer: C

C) land.

D) labor income.

Type: A Topic: 2 E: 544 MI: 300 27. Some economists advocate taxes on land because such taxes: A) do not affect the supply of land. B) increase the supply of land. C) improve the allocation of land by shifting it from low-productivity to high-productivity uses. D) have a positive incentive function. Answer: A

McConnell/Brue: Economics, 16/e

Page 914

Chapter 29: Rent, Interest, and Profit

Interest rate concept

Type: A Topic: 3 E: 545 MI: 301 Status: New 28. Interest is the: A) price paid for the use of money. B) opportunity cost of time. Answer: A

C) expectation of a future return on investment. D) reward for consuming rather than saving.

Type: A Topic: 3 E: 545 MI: 301 Status: New 29. Suppose a person pays $80 of annual interest on a loan that has a 5 percent annual interest rate. The loan amount is: A) $400. B) $1600. C) $160. D) $85. Answer: B

Type: A Topic: 3 E: 545 MI: 301 Status: New 30. Suppose a loan customer is considering two alternative $22,000 loans. Loan 1 requires payment of $1,100 of interest each year and Loan 2 has a 6 percent annual interest rate. Other things equal, the loan customer will: A) be indifferent between the two loans because they both have the same annual percentage rate. B) reject both loans because they each carry too high an interest rate. C) choose Loan 1 because it has a lower annual interest rate than Loan 2. D) choose Loan 2 because it has a lower annual interest rate than Loan 1. Answer: C

Type: A Topic: 3 E: 545 MI: 301 31. Suppose that interest payments are $140 per year on a $1000 loan and $1188 per on a $8485 loan. The interest rates on the two loans are: A) 14 percent and 20 percent, respectively. C) 18.8 percent on both loans. B) 14 percent on both loans. D) 1.4 percent and 11.8 percent, respectively. Answer: B

Type: A Topic: 3 E: 545 MI: 301 32. Which of the following is correct? A) Money is a resource, but real capital is not. B) Real capital is a resource, but money is not. Answer: B

C) Neither money nor real capital is a resource. D) Both money and real capital are resources.

Type: A Topic: 3 E: 546 MI: 302 33. The equilibrium interest rate equates: A) nominal and real interest rates. B) the quantities demanded and supplied of loanable funds. C) consumption and saving. D) taxes and government spending. Answer: B

McConnell/Brue: Economics, 16/e

Page 915

Chapter 29: Rent, Interest, and Profit

Loanable funds theory of interest

Type: A Topic: 4 E: 546 MI: 302 34. The supply curve of loanable funds is upsloping because: A) businesses find more investments to be profitable at low interest rates than at high interest rates. B) households are willing to save more at high interest rates than they are at low interest rates. C) government budget deficits vary inversely with the equilibrium interest rate. D) banks lend more at low interest rates than they do at high interest rates. Answer: B

Type: A Topic: 4 E: 546 MI: 302 35. The fact that people prefer present consumption to future consumption results in: A) a downsloping demand for loanable funds curve. B) an upsloping supply of loanable funds curve. C) a downsloping supply of loanable funds curve. D) an upsloping demand for loanable funds curve. Answer: B

Type: A Topic: 4 E: 546 MI: 302 36. Which of the following is not a source of loanable funds? A) the saving of households C) commercial bank lending B) business saving D) government budget deficits Answer: D

Type: A Topic: 4 E: 546 MI: 302 37. Which of the following is not a component of the demand for loanable funds? A) household purchases of housing and durable consumer goods B) business purchases of capital goods C) government financing of the public debt D) household saving Answer: D

Type: A Topic: 4 E: 546 MI: 302 38. The demand for loanable funds is downsloping: A) because businesses find that more investments are profitable at low interest rates than at high interest rates. B) because households are willing to save more at high interest rates than at low interest rates. C) only when the nominal interest rate exceeds the real interest rate. D) because the amount of profitable business investment varies directly with the interest rate. Answer: A

Type: A Topic: 4 E: 547 MI: 303 39. Other things equal, an increase in the productivity of capital goods will: A) increase the demand for loanable funds and decrease the equilibrium interest rate. B) increase the demand for loanable funds and increase the equilibrium interest rate. C) increase the supply of loanable funds and decrease the equilibrium interest rate. D) increase the supply of loanable funds and increase the equilibrium interest rate. Answer: B

McConnell/Brue: Economics, 16/e

Page 916

Chapter 29: Rent, Interest, and Profit

Type: A Topic: 4 E: 547 MI: 303 40. If Congress were to pass a law exempting interest on saving from taxation, the: A) supply of loanable funds would decrease and the equilibrium interest rate rise. B) supply of loanable funds would increase and the equilibrium interest rate fall. C) demand for loanable funds would increase and the equilibrium interest rate rise. D) equilibrium interest rate would be unaffected. Answer: B

Type: A Topic: 4 E: 547 MI: 303 41. A doubling of Social Security benefits is likely to: A) decrease the demand for loanable funds and increase the equilibrium interest rate. B) increase the supply of loanable funds and decrease the equilibrium interest rate. C) decrease the supply of loanable funds and increase the equilibrium interest rate. D) increase the real interest rate but not the nominal interest rate. Answer: C

Type: A Topic: 4 E: 547 MI: 303 42. In the market for loanable funds: A) an increase in bank lending will increase the interest rate. B) a decrease in saving will reduce the interest rate. C) an increase in borrowing for investment will increase the interest rate. D) a decrease in government borrowing will increase the interest rate. Answer: C

Use the following to answer questions 43-47:

S1 S0 G

Interest Rate

F
E D1

D0

0

A

B

C

Quantity

Type: G Topic: 4 E: 546 MI: 302 43. Refer to the above diagram. The demand for loanable funds curve D1 slopes downward because: A) business will borrow more funds at lower interest rates than at higher ones. B) at lower interest rates households will make more funds available for lending. C) at lower interest rates, households will consume more and save less. D) business will borrow more funds at higher interest rates than at lower ones. Answer: A

McConnell/Brue: Economics, 16/e

Page 917

Chapter 29: Rent, Interest, and Profit

Type: G Topic: 4 E: 546 MI: 302 44. Refer to the above diagram. The supply of loanable funds curve S1 slopes upward because: A) business will borrow more funds at lower interest rates than at higher ones. B) at higher interest rates households will make more funds available for lending. C) at lower interest rates, households will consume less and save more. D) business will borrow more funds at higher interest rates than at lower ones. Answer: B

Type: G Topic: 4 E: 546 MI: 302 45. Refer to the above diagram. If the supply of loanable funds is S1 and the demand for loanable funds is D1, the equilibrium interest rate and quantity of funds borrowed will be: A) G and A. B) F and A. C) F and C. D) E and A. Answer: B

Type: G Topic: 4 E: 546 MI: 302 46. Refer to the above diagram. If the supply of loanable funds is S0 and the demand for loanable funds is D0, the equilibrium interest rate and quantity of funds borrowed will be: A) G and A. B) F and A. C) F and C. D) E and A. Answer: C

Type: G Topic: 4 E: 547 MI: 303 47. Refer to the above diagram. Suppose that the demand for loanable funds is D0 and the supply of loanable funds initially is S0. If the supply of loanable funds declines to S1, the equilibrium interest rate will: A) decline from G to F. B) increase from E to F. C) decline from F to E. D) increase from F to G. Answer: D

Structure and impact of interest rates

Type: A Topic: 5 E: 548 MI: 304 48. Other things equal, interest rates are: A) higher on large loans than on small loans. B) higher on loans with tax-exempt interest payments. C) lower on less risky loans than on riskier loans. D) lower on short-term loans than on long-term loans. Answer: C

Type: A Topic: 5 E: 548 MI: 304 49. Which of the following generalizations is false? Other things equal: A) interest rates are higher if lenders are imperfectly, rather than purely, competitive. B) the interest rate is less on small loans than on larger loans. C) long-term loans normally command higher interest rates than short-term loans. D) the greater the risk on a loan, the greater the interest rate. Answer: B

McConnell/Brue: Economics, 16/e

Page 918

Chapter 29: Rent, Interest, and Profit

Type: D Topic: 5 E: 548-549 MI: 304-305 50. The pure rate of interest is approximated by the: A) rate which savings and loan associations charge on mortgage loans. B) rate charged consumers by credit card companies. C) rate paid on long-term government bonds. D) announced rate at which commercial banks make business loans. Answer: C

Type: D Topic: 5 E: 548-549 MI: 304-305 51. The pure rate of interest refers to the: A) nominal rate of interest adjusted for inflation. B) nominal rate of interest. C) interest rate paid on virtually riskless long-term bonds. D) rate which large banks charge their corporate borrowers. Answer: C

Type: C Topic: 5 E: 549 MI: 305 52. A lower equilibrium interest rate: A) increases saving, reduces total spending, and increases total output. B) decreases saving, increases total spending, and decreases total output. C) increases investment, increases total spending, and increases total output. D) decreases investment, decreases total spending, and increases total output. Answer: C

Type: A Topic: 5 E: 549 MI: 305 53. The equilibrium interest rate: A) allocates the available supply of loanable funds to investment projects that have high enough rates of return to justify the borrowing. B) rises when the supply of loanable funds increases. C) is the price paid for the use of any resource. D) effects the size of total output but not the composition of that output. Answer: A

Type: A Topic: 5 E: 549 MI: 305 54. The equilibrium interest rate: A) affects both the size of total output and its composition. B) falls when the demand for loanable funds increases. C) determines the composition of R&D spending but not its total amount. D) increases when the expected rate of return on R&D spending falls. Answer: A

Type: C Topic: 5 E: 549 MI: 305 55. Other things equal, an increase in the equilibrium interest rate will: A) increase R&D spending. B) rise when the supply of loanable funds increases. C) decrease purchases of capital goods and reduce R&D spending. D) increase bank lending. Answer: C

McConnell/Brue: Economics, 16/e

Page 919

Chapter 29: Rent, Interest, and Profit

Type: A Topic: 5 E: 549 MI: 305 56. Changes in the equilibrium interest rate will: A) affect both the size of the domestic output and the allocation of capital goods among industries. B) affect the size of the domestic output, but not the allocation of capital goods among industries. C) affect the allocation of capital goods among industries, but not the size of the domestic output. D) have no perceptible effect on either the size of the domestic output or the allocation of capital goods among industries. Answer: A

Nominal and real interest rates

Type: D Topic: 6 E: 549 MI: 305 57. The real rate of interest is: A) the interest rate charged on long-term government bonds. B) the interest rate associated with a riskless loan. C) the interest rate that large commercial banks charge their best customers. D) the interest rate after adjustment has been made for inflation. Answer: D

Type: A Topic: 6 E: 549-550 MI: 305-306 58. The real interest rate can be estimated by: A) subtracting the pure interest rate from the nominal interest rate. B) dividing the nominal interest rate by the consumer price index. C) subtracting the nominal interest rate from the rate of inflation. D) subtracting the rate of inflation from the nominal interest rate. Answer: D

Type: A Topic: 6 E: 549 MI: 305 59. In year 1 the price level is constant and the nominal rate of interest is 6 percent. But in year 2 the inflation rate is 3 percent. If the real rate of interest is to remain at the same level in year 2 as it was in year 1, then in year 2 the nominal interest rate must: A) rise by 9 percentage points. C) fall by 3 percentage points. B) rise by 3 percentage points. D) rise by 6 percentage points. Answer: B

Type: A Topic: 6 E: 548-549 MI: 304-305 60. The pure interest rate is: A) the nominal rate plus the rate of inflation. B) not used in making investment decisions. C) is the nominal rate of interest less the rate of return on an investment. D) the rate paid on long term, relatively risk-free bonds. Answer: D

Type: A Topic: 6 E: 549 MI: 305 61. The XYZ Corporation can make a real (inflation-adjusted) return on an investment of 9 percent. The nominal rate of interest is 13 percent and the rate of inflation is 7 percent. We can conclude that the: A) investment will be profitable. C) real rate of interest is 4 percent. B) investment will be unprofitable. D) real rate of interest is 2 percent. Answer: A

McConnell/Brue: Economics, 16/e

Page 920

Chapter 29: Rent, Interest, and Profit

Type: A Topic: 6 E: 549-550 MI: 305-306 62. Which of the following is incorrect? A) The nominal interest rate is the rate of interest expressed in terms of current dollars. B) The real interest rate is the rate of interest expressed in terms of dollars of constant or inflation-adjusted value. C) The nominal interest rate is the real interest rate less the rate of inflation. D) During periods of inflation the nominal interest rate will exceed the real interest rate. Answer: C

Type: A Topic: 6 E: 549 MI: 305 63. Suppose that in some year nominal interest rates are less than the rate of inflation. This means that: A) money demand exceeds money supply. B) real interest rates are negative. C) real interest rates are positive and unusually high. D) real interest rates exceed nominal interest rates. Answer: B

Type: A Topic: 6 E: 549 MI: 305 64. In making an investment decision a business firm is most interested in the: A) nominal interest rate. B) real interest rate. C) nominal interest rate minus the real interest rate. D) the future supply of loanable funds. Answer: B

Usury laws

Type: A Topic: 7 E: 550 MI: 306 65. Effective usury laws cause: A) a surplus in the market for loanable funds. B) the quantity of loanable funds demanded to be brought into balance with the quantity supplied. C) the quantity of loanable funds demanded to exceed the quantity supplied. D) the quantity of loanable funds supplied to exceed the quantity demanded. Answer: C

Type: A Topic: 7 E: 550 MI: 306 66. Usury laws: A) allocate funds from low-productivity to high-productivity investments. B) establish a legal ceiling on interest rates. C) make more funds available to low-income borrowers. D) create a surplus of loanable funds. Answer: B

McConnell/Brue: Economics, 16/e

Page 921

Chapter 29: Rent, Interest, and Profit

Type: A Topic: 7 E: 550 MI: 306 67. Effective usury laws cause: A) a surplus of money in money markets. B) the quantity of money demanded to be brought into balance with the quantity supplied. C) the quantity of money demanded to exceed the quantity supplied. D) the quantity of money supplied to exceed the quantity demanded. Answer: C

Type: A Topic: 7 E: 550 MI: 306 68. Effective usury laws: A) subsidize lenders. B) penalize those who borrow at the below-market interest rate. C) improve efficiency in investing. D) keep some low-income people from obtaining credit and loans. Answer: D

Type: A Topic: 7 E: 550 MI: 306 69. A major purpose of usury laws is to make more funds available to low-income borrowers. Economic analysis suggests that usury laws: A) are effective in achieving this goal. B) allocate available funds to high-income borrowers. C) have no impact on the allocation of funds between high-income and low-income people. D) help low-income people only when the legal interest rate is above the market rate. Answer: B

Profit

Type: A Topic: 8 E: 551-552 MI: 307-308 70. Economic profit is most closely associated with: A) the process of saving and investing. B) monopoly, innovation, and uninsurable risks. Answer: B

C) long-run competitive equilibrium. D) a static economy.

Type: A Topic: 8 E: 551 MI: 307 71. In long-run equilibrium there will be no economic profit in a purely competitive static economy because: A) barriers to entry will prevent profit from arising. B) there will be no uncertainty, no innovations, and no monopoly. C) there will be no need for professional managers and therefore no profit rewards will be needed. D) the marginal revenue product of capital will be zero. Answer: B

Type: A Topic: 8 E: 551 MI: 307 72. In a purely competitive static economy: A) the demand for loanable funds would disappear. B) uncertainty would increase, causing profit to rise. C) economic profit would be zero. D) economic profit would be maximized. Answer: C

McConnell/Brue: Economics, 16/e

Page 922

Chapter 29: Rent, Interest, and Profit

Type: A Topic: 8 E: 551 MI: 307 73. Which of the following represents an uninsurable risk to a business firm? A) the possibility that its warehouse will burn down B) the possibility that several of its workers will be injured at work C) the possibility that an adverse change in consumer tastes will decrease the demand for the firm's product D) the possibility that a tornado will damage the plant and stop production for a month Answer: C

Type: G Topic: 8 E: 424 MI: 180 74.

On the basis of the above diagram we can say that the firm is earning: A) a normal profit. B) an economic profit. C) neither a normal nor an economic profit. D) total revenue insufficient to cover its total costs. Answer: A

Type: A Topic: 8 E: 551 MI: 307 75. Economic profit might result from: A) easy entry into industries B) dynamic change and uncertainty. Answer: B

C) x-inefficiency. D) a decline in entrepreneurship

Type: D Topic: 8 E: 551 MI: 307 76. A normal profit is: A) the average profitability of a firm over one complete business cycle. B) calculated by subtracting explicit costs from total revenue. C) the return required to retain entrepreneurial talent in some particular line of production. D) the amount by which total revenue exceeds total costs. Answer: C

Type: A Topic: 8 E: 551 MI: 307 77. The entrepreneur: A) makes routine day-to-day business decisions. B) is a colorful figure from the past who is rarely relevant in today's complex economy. C) introduces innovations in the form of new products or new production processes. D) receives income mainly as wages. Answer: C

McConnell/Brue: Economics, 16/e

Page 923

Chapter 29: Rent, Interest, and Profit

Type: D Topic: 8 E: 551 MI: 307 78. Pure or economic profit is: A) the amount by which total revenue exceeds total costs. B) determined by subtracting explicit costs from total revenue. C) the return required to retain entrepreneurial talent in some particular line of production. D) the return to any resource the supply of which is perfectly inelastic. Answer: A

Type: A Topic: 8 E: 552 MI: 308 79. Economic profit affects: A) the allocation of resources, but not the level of resource use. B) the level of resource use, but not the allocation of resources. C) the allocation of resources and the level of resource use. D) neither the allocation of resources nor the level of resource use. Answer: C

Income shares

Type: F Topic: 9 E: 554 MI: 310 80. The largest single share of the national income consists of: A) wages and salaries. B) interest. C) rents. D) corporate profits. Answer: A

Type: F Topic: 9 E: 554 MI: 310 81. Currently capitalist income, that is, corporate profits, interest, and rent, accounts for about what percentage of the national income? A) 10 percent B) 20 percent C) 50 percent D) 80 percent Answer: B

Type: F Topic: 9 E: 554 MI: 310 82. If labor's share of the national income is narrowly defined as wages and salaries, we can say that labor's relative share of national income: A) declined between 1900 and 1940, but has increased since 1940. B) has remained constant since 1900. C) has increased since 1900. D) has fallen since 1900. Answer: C

Type: F Topic: 9 E: 554 MI: 310 83. If labor's share of the national income is broadly defined as the sum of wages and salaries and proprietors' income, we can say that labor's relative share has: A) remained approximately constant since 1900. B) increased dramatically at the expense of capitalist income. C) declined by about one-third since 1900. D) decreased because of the decline of unionism. Answer: A

McConnell/Brue: Economics, 16/e

Page 924

Chapter 29: Rent, Interest, and Profit

Type: F Topic: 9 E: 554 MI: 310 84. Capitalist income (corporate profits, interest, and rent) has: A) declined sharply since 1900 because of the growing strength of labor unions. B) remained approximately constant in this century. C) increased significantly because of rising rents. D) fallen in this century because of the declining importance of corporations. Answer: B

Type: F Topic: 9 E: 554 MI: 310 85. Defined narrowly as wages and salaries, labor's share of the national income is about: A) 70 percent. B) 50 percent. C) 40 percent. D) 90 percent. Answer: A

Type: F Topic: 9 E: 554 MI: 310 86. Interest, rent, and corporate profits combined account for about what percentage of the national income? A) 80 percent B) 55 percent C) 35 percent D) 20 percent Answer: D

Consider This Questions

Type: A E: 547 MI: 303 Status: New 87. (Consider This) The story about economist Irving Fisher's conversation with his masseuse illustrates that: A) other things equal, interest rates are higher on smaller loans than on larger loans. B) interest is a payment required for someone to give up the present use of their money. C) other things equal, longer term loans have lower interest rates than shorter-term loans. D) real interest rates differ from nominal interest rates. Answer: B

Type: A E: 547 MI: 303 Status: New 88. (Consider This) The story about economist Irving Fisher's conversation with his masseuse illustrates that interest payments arise because of: A) the possibility of inflation. C) imperfect information about the future. B) the reality of credit risk. D) the time-value of money. Answer: D

Last Word Questions

Type: A E: 553 MI: 309 89. (Last Word) Suppose you borrow $500 and agree to pay this $500 plus $75 of interest at the end of a year. The interest rate is: A) 10 percent. B) 15 percent. C) 12.5 percent. D) 7.5 percent. Answer: B

McConnell/Brue: Economics, 16/e

Page 925

Chapter 29: Rent, Interest, and Profit

Type: A E: 553 MI: 309 90. (Last Word) Suppose you borrow $500 for a year and the lender discounts $75 of interest at the time the loan is made. The interest rate on this loan is about: A) 12.5 percent. B) 14 percent. C) 18 percent. D) 10 percent. Answer: C

Type: A E: 553 MI: 309 91. (Last Word) Suppose you deposit $5,000 in a bank which pays 10 percent interest compounded twice a year. The actual annual interest rate you receive is: A) 10 percent. B) 11 percent. C) 10.25 percent. D) 12 percent. Answer: C

Type: A E: 553 MI: 309 92. (Last Word) Suppose you borrow $10,000 for one year and must pay $1,000 in interest at the end of the year. If you are required to repay the loan principle in 12 equal monthly installments, the effective interest rate is: A) 20 percent. B) 10 percent. C) 15 percent. D) 5 percent. Answer: A

True/False Questions

Type: A E: 543 MI: 299 93. Demand is the active and supply the passive determinant of land rent. Answer: True

Type: A E: 544-545 MI: 300-301 94. Different rents on land reflect differences in the marginal revenue productivity of land. Answer: True

Type: A E: 543 MI: 299 Status: New 95. The free-land era of U.S. history reflected a situation in which the quantity of land available at a zero price exceeded the quantity of land demanded. Answer: True

Type: A E: 543 MI: 299 96. Rent performs an incentive function, but no rationing function. Answer: False

Type: A E: 545 MI: 301 97. The interest rate is the price paid for the use of money. Answer: True

Type: A E: 546 MI: 302 Status: New 98. Unlike most demand curves, the demand curve for loanable funds is upsloping. Answer: False

McConnell/Brue: Economics, 16/e

Page 926

Chapter 29: Rent, Interest, and Profit

Type: A E: 548 MI: 304 Status: New 99. Other things equal, the shorter the loan period and the larger the loan size, the higher is the interest rate charged by the lender. Answer: False

Type: A E: 546 MI: 302 100. The supply of loanable funds is perfectly elastic. Answer: False

Type: D E: 551 MI: 307 101. Economic profits are the salaries received by the hired managers of business corporations. Answer: False

Type: A E: 551 MI: 307 102. The basic function of profits and losses is to allocate society's scarce resources to their highest valued uses. Answer: True

Type: F E: 554 MI: 310 103. Broadly defined, labor's share of national income has been remarkably stable since 1900. Answer: True

McConnell/Brue: Economics, 16/e

Page 927

CHAPTER 30

Government and Market Failure
Topic 1. 2. 3. 4. 5. 6. 7. 8. Public goods Cost-benefit analysis Externalities and government intervention Coase theorem Market for externality rights Optimal pollution abatement Recycling Asymmetric information Consider This Last Word True-False Question numbers 1-20 21-27 28-38 39-43 44-54 55-64 65-72 73-85 86-88 89-91 92-107

____________________________________________________________

_______________________________________

____________________________________________________________

_______________________________________

Multiple Choice Questions Public goods

Type: D Topic: 1 E: 558 MI: 314 Status: New 1. The two main characteristics of a public good are: A) production at constant marginal cost and rising demand. B) nonexcludability and production at rising marginal cost. C) nonrivalry and nonexcludability. D) nonrivalry and large spillover costs. Answer: C

Type: D Topic: 1 E: 558 MI: 314 Status: New 2. Nonrivalry and nonexcludability are the main characteristics of: A) consumption goods. B) capital goods. C) private goods. D) public goods. Answer: D

Type: D Topic: 1 E: 558 MI: 314 Status: New 3. Unlike a private good, a public good: A) has no opportunity costs. B) has benefits available to all, including nonpayers. C) produces no spillover benefits or spillover costs. D) is characterized by rivalry and excludability. Answer: B

Type: A Topic: 1 E: 558 MI: 314 Status: New 4. Which of the following is an example of a public good? A) a weather warning system B) a television set C) a sofa D) a bottle of soda Answer: A

Chapter 30: Government and Market Failure

Type: A Topic: 1 E: 558 MI: 314 Status: New 5. An example of a public good is: A) broadband Internet access. B) homeland defense. C) a baseball. D) a lawn mower. Answer: B

Type: A Topic: 1 E: 558 MI: 314 Status: New 6. A public good: A) can be profitably produced by private firms. B) is characterized by rivalry and excludability. C) produces no spillover costs or spillover benefits. D) is available to all and cannot be denied to anyone. Answer: D

Type: D Topic: 1 E: 558 MI: 314 Status: New 7. The market system does not produce public goods because: A) there is no need or demand for such goods. B) private firms cannot stop consumers who are unwilling to pay for such goods from benefiting from them. C) public enterprises can produce such goods at lower cost than can private enterprises. D) their production seriously distorts the distribution of income. Answer: B

Type: D Topic: 1 E: 558 MI: 314 8. Public goods are those for which there: A) is no free-rider problem. B) are no externalities. C) is nonrivalry and nonexcludability. D) is neither an adverse selection nor moral hazard problem. Answer: C

Type: A Topic: 1 E: 559 MI: 315 9. Because of the free-rider problem: A) the market demand for a public good is overstated. B) the market demand for a public good is nonexistent or understated. C) government has increasingly yielded to the private sector in producing public goods. D) public goods often create moral hazard and adverse selection problems. Answer: B

Type: A Topic: 1 E: 560-561 MI: 316-317 10. At the optimal quantity of a public good: A) marginal benefit exceeds marginal cost by the greatest amount. B) total benefit equals total cost. C) marginal benefit equals marginal cost. D) marginal benefit is zero. Answer: C

McConnell/Brue: Economics, 16/e

Page 930

Chapter 30: Government and Market Failure

Use the following to answer questions 11-14: Answer the next question(s) on the basis of the following information for a public good. Pa and Pb are the prices that individuals A and B are willing to pay for the last unit of a public good, rather than do without it. These people are the only two members of society.
Q 1 2 3 4 5 Pa $3 2 1 0 0 Pb $5 4 3 2 1

Type: T Topic: 1 E: 559 MI: 315 11. Refer to the above data. The collective willingness of this society to pay for the 2nd unit of this public good is: A) $2. B) $4. C) $6. D) $8. Answer: C

Type: T Topic: 1 E: 560 MI: 316 12. Refer to the above data. If the marginal cost of this good at the optimal quantity is $4, the optimal quantity must be: A) 1 unit. B) 2 units. C) 3 units. D) 4 units. Answer: C

Type: T Topic: 1 E: 559 MI: 315 13. Refer to the above data. Suppose government has already produced 4 units of this public good. The amount individual B is willing voluntarily to pay for the 4th unit is: A) $14. B) $5. C) $2. D) $0. Answer: D

Type: T Topic: 1 E: 559 MI: 315 14. Refer to the above data. If this good were a private good instead of a public one, the total quantity demanded at a $3 market price would be: A) 2 units. B) 3 units. C) 6 units. D) 4 units. Answer: D

Type: A Topic: 1 E: 559-560 MI: 315-316 15. A demand curve for a public good is determined by: A) summing vertically the individual demand curves for the public good. B) summing horizontally the individual demand curves for the public good. C) combining the amounts of the public good that the individual members of society demand at each price. D) multiplying the per-unit cost of the public good by the quantity made available. Answer: A

McConnell/Brue: Economics, 16/e

Page 931

Chapter 30: Government and Market Failure

Type: A Topic: 1 E: 559 MI: 315 16. Suppose that Mick and Cher are the only two members of society and are willing to pay $10 and $8, respectively, for the 3d unit of a public good. Also, assume that the marginal cost of the 3d unit is $17. We can conclude that: A) the 3d unit should not be produced. C) zero units should be produced. B) the 3d unit should be produced. D) 4 units should be produced. Answer: B

Type: A Topic: 1 E: 559 MI: 315 17. Ed, Mike, and Scott are the only three people in a community and Ed is willing to pay $20 for the 5th unit of a public good; Mike, $15, and Scott, $25. Government should produce the 5th unit of the public good if the marginal cost is less than: A) $25. B) $15. C) $60. D) $300. Answer: C

Type: A Topic: 1 E: 559 MI: 315 18. For which one of the following goods would we need to sum individual demand curves vertically to obtain the total demand curve? A) frozen yogurt B) bubble gum C) microwave popcorn D) courts of law Answer: D

Use the following to answer questions 19-20:

Type: G Topic: 1 E: 559-560 MI: 315-316 19. Refer to the above diagrams in which figures (a) and (b) show demand curves reflecting the prices Alvin and Elmer are willing to pay for a public good, rather than do without it. The collective willingness to pay for the 1st unit of this public good is: A) $18. B) $14. C) $10. D) $6. Answer: A

McConnell/Brue: Economics, 16/e

Page 932

Chapter 30: Government and Market Failure

Type: G Topic: 1 E: 559-560 MI: 315-316 20. Refer to the above diagrams in which figures (a) and (b) show demand curves reflecting the prices Alvin and Elmer are willing to pay for a public good, rather than do without it. If the marginal cost of the optimal quantity of this public good is $10, the optimal quantity must be: A) 1 unit. B) 2 units. C) 3 units. D) 4 units. Answer: C

Cost-benefit analysis

Type: A Topic: 2 E: 560 MI: 316 21. Cost-beneifit analysis attempts to: A) compare the real worth, rather than the market values, of various goods and services. B) compare the relative desirability of alternative distributions of income. C) determine whether it is better to cut government expenditures or reduce taxes. D) compare the benefits and costs associated with any economic project or activity. Answer: D

Use the following to answer questions 22-24: The following data are for a series of increasingly extensive flood control projects:

Plan A--levees Plan B--small reservoir Plan C--medium reservoir Plan D--large reservoir

Total cost per year $10,000 24,000 44,000 72,000

Total benefit per year $16,000 36,000 52,000 64,000

Type: T Topic: 2 E: 561 MI: 317 22. Refer to the above data. For Plan D marginal costs and marginal benefits are: A) $72,000 and $64,000 respectively. C) $24,000 and $18,000 respectively. B) $28,000 and $12,000 respectively. D) $16,000 and $28,000 respectively. Answer: B

Type: T Topic: 2 E: 561 MI: 317 23. Refer to the above data. On the basis of cost-benefit analysis government should undertake: A) Plan D. B) Plan C. C) Plan B. D) Plan A. Answer: C

Type: T Topic: 2 E: 561 MI: 317 24. Plan C above entails: A) marginal benefits in excess of marginal costs. B) fewer spillovers than either Plan A or Plan B. C) an overallocation of resources to flood control. D) an underallocation of resources to flood control. Answer: C

McConnell/Brue: Economics, 16/e

Page 933

Chapter 30: Government and Market Failure

Use the following to answer questions 25-26: Answer the next question(s) on the basis of the following information for four highway programs of increasing scope. All figures are in millions of dollars.

Program A B C D

Total cost Total benefit $ 2 $ 9 6 16 21 12 23 20

Type: T Topic: 2 E: 561 MI: 317 25. The above data indicate that: A) there is no highway program that is economically justifiable on the basis of cost-benefit analysis. B) the marginal cost and marginal benefit of Program A are $2 and $9 respectively. C) the marginal cost and marginal benefit of Program C are $12 and $21 respectively. D) the marginal cost and marginal benefit of Program A cannot be determined. Answer: B

Type: T Topic: 2 E: 561 MI: 317 26. On the basis of the above data we can say that: A) Program D is the most efficient on economic grounds. B) Program C is the most efficient on economic grounds. C) Program B is the most efficient on economic grounds. D) Program A is the most efficient on economic grounds. Answer: C

Type: A Topic: 2 E: 562-563 MI: 318-319 27. An important problem in evaluating public projects through the use of cost-benefit analysis is that: A) real costs cannot be stated in monetary terms. B) one must decide whether to compare total costs and total benefits or marginal costs and marginal benefits. C) spillover costs and benefits associated with such projects may be difficult to measure. D) the funding of such projects is inherently inflationary. Answer: C

Externalities and government intervention

Type: A Topic: 3 E: 562-563 MI: 318-319 28. A positive externality or spillover benefit occurs when: A) product differentiation increases the variety of products available to consumers. B) the benefits associated with a product exceed those accruing to people who consume it. C) a firm produces at the P = MC output. D) economic profits are zero in the long run. Answer: B

McConnell/Brue: Economics, 16/e

Page 934

Chapter 30: Government and Market Failure

Type: A Topic: 3 E: 562 MI: 318 29. A negative externality or spillover cost occurs when: A) firms fail to achieve allocative efficiency. B) firms fail to achieve productive efficiency. C) price exceeds marginal cost. D) the total cost of producing a good exceeds the costs borne by the producer. Answer: D

Use the following to answer questions 30-31:

Type: G Topic: 3 E: 562 MI: 318 30. Refer to the above diagram in which S is the market supply curve and St is a supply curve comprising all costs of production, including external costs. Assume that the number of people affected by these external costs is large. Without government interference, this market will result in: A) an optimal allocation of society's resources. B) an underallocation of resources to this product. C) an overallocation of resources to this product. D) a higher price than is consistent with an optimal allocation of resources. Answer: C

Type: G Topic: 3 E: 562 MI: 318 31. Refer to the above diagram in which S is the market supply curve and S t is a supply curve comprising all costs of production, including external costs. Assume that the number of people affected by these external costs is large. If the government wishes to establish an optimal allocation of resources in this market, it should: A) not intervene because the market outcome is optimal. B) subsidize consumers so that the market demand curve shifts leftward. C) subsidize producers so that the market supply curve shifts leftward (upward). D) tax producers so that the market supply curve shifts leftward (upward). Answer: D

McConnell/Brue: Economics, 16/e

Page 935

Chapter 30: Government and Market Failure

Use the following to answer questions 32-34:

Type: G Topic: 3 E: 562-563 MI: 318-319 32. Refer to the above diagrams for two separate product markets. Assume that society's optimal level of output in each market is Q0 and that government purposely shifts the market supply curve from S to S1 in diagram (a) and from S to S2 in diagram (b). We can conclude that the government is correcting for: A) spillover costs in diagram (a) and spillover benefits in diagram (b). B) spillover benefits in diagram (a) and spillover costs in diagram (a). C) spillover costs in both diagrams. D) spillover benefits in both diagrams. Answer: A

Type: G Topic: 3 E: 562-563 MI: 318-319 33. Refer to the above diagrams for two separate product markets. Assume that society's optimal level of output in each market is Q0 and that government purposely shifts the market supply curve from S to S1 in diagram (a) and from S to S2 in diagram (b). The shift of the supply curve from S to S1 in diagram (a) might be caused by a per unit: A) subsidy paid to the producers of this product. C) subsidy paid to the buyers of this product. B) tax on the producers of this product. D) tax on the buyers of this product. Answer: B

Type: G Topic: 3 E: 562-563 MI: 318-319 34. Refer to the above diagrams for two separate product markets. Assume that society's optimal level of output in each market is Q0 and that government purposely shifts the market supply curve from S to S1 in diagram (a) and from S to S2 in diagram (b). The shift of the supply curve from S to S2 in diagram (b) might be caused by a per unit: A) subsidy paid to the producers of this product. C) subsidy paid to the buyers of this product. B) tax on the producers of this product. D) tax on the buyers of this product. Answer: A

McConnell/Brue: Economics, 16/e

Page 936

Chapter 30: Government and Market Failure

Use the following to answer questions 35-36:

Type: G Topic: 3 E: 562-563 MI: 318-319 35. Refer to the above competitive market diagram for product Z. Assume that the current market demand and supply curves for Z are D2 and S2. If there are substantial spillover benefits associated with the production of Z, then: A) efficient resource allocation occurs at output G and price B because the market mechanism does not measure all benefits. B) an output smaller than G would improve resource allocation. C) government should levy a per unit excise tax on Z to shift the demand curve toward D1. D) an output greater than G would result in a more efficient allocation of resources. Answer: D

Type: G Topic: 3 E: 562-563 MI: 318-319 36. Refer to the above competitive market diagram for product Z. Assume that the current market demand and supply curves for Z are D2 and S2. If there are substantial spillover costs associated with the production of Z, then: A) a price lower than B and an output greater than G would improve resource allocation. B) government should levy a per unit excise tax on Z to shift the demand curve to the right. C) government should levy a per unit excise tax on Z to shift the supply curve toward S 1. D) government should subsidize the production of Z to lower equilibrium price and increase equilibrium output. Answer: C

McConnell/Brue: Economics, 16/e

Page 937

Chapter 30: Government and Market Failure

Use the following to answer questions 37-38:

Type: G Topic: 3 E: 562-563 MI: 318-319 37. Refer to the above diagram of the market for product X. Curve S t embodies all costs (including spillovers) and Dt embodies all benefits (including spillovers) associated with the production and consumption of X. Assuming the equilibrium output is Q1, we can conclude that the existence of spillover: A) costs has resulted in an overallocation of resources to X. B) benefits has resulted in an overallocation of resources to X. C) costs has resulted in an underallocation of resources to X. D) benefits has resulted in an underallocation of resources to X. Answer: D

Type: G Topic: 3 E: 562 MI: 318 38. Refer to the above diagram of the market for product X. Curve S t embodies all costs (including spillovers) and Dt embodies all benefits (including spillovers) associated with the production and consumption of X. Assuming the equilibrium output is Q2, we can conclude that the existence of spillover: A) costs has resulted in an overallocation of resources to X. B) benefits has resulted in an overallocation of resources to X. C) costs has resulted in an underallocation of resources to X. D) benefits has resulted in an underallocation of resources to X. Answer: A

Coase theorem

Type: D Topic: 4 E: 563 MI: 319 39. The Coase theorem states that: A) government should levy excise taxes on firms that generate spillover or external costs. B) taxes should be levied such that they change private behavior as little as possible. C) bargaining between private parties will remedy externality problems where property rights are clearly defined, the number of people involved are few, and bargaining costs are small. D) trading of votes to secure favorable voting outcomes may increase efficiency. Answer: C

McConnell/Brue: Economics, 16/e

Page 938

Chapter 30: Government and Market Failure

Type: A Topic: 4 E: 563 MI: 319 40. The Coase theorem: A) applies only to circumstances in which externalities are extensive and bargaining costs are high. B) holds that the median voter will decide the outcome of elections. C) states that in some circumstances majority voting can yield inconsistent results. D) suggests that in some circumstances government intervention is not needed to resolve externality problems. Answer: D

Type: D Topic: 4 E: 563 MI: 319 41. The proposition that under some circumstances externalities can get resolved through private negotiation is known as: A) the paradox of externalities. B) the median-voter theory. C) the Coase theorem. D) logrolling. Answer: C

Type: D Topic: 4 E: 563 MI: 319 42. Suppose that a large tree on Shawn's property is blocking Sam's view of the lake below. Shawn accepts Sam's offer to pay Shawn $100 for the right to cut down the tree. This situation describes: A) the Coase theorem. C) logrolling. B) the diamond-water paradox. D) a market for externality rights. Answer: A

Type: A Topic: 4 E: 563-564 MI: 319-320 43. Clearly defined property rights and liability rules reduce negative spillovers by: A) threatening the perpetrators with lawsuits. B) shifting the perpetrators' market supply curve rightward. C) shifting the perpetrators' market demand curve leftward. D) creating a market for externality rights. Answer: A

Market for externality rights

Type: D Topic: 5 E: 566 MI: 322 44. The tendency for society to overuse and therefore abuse common resources is called the: A) law of conservation of matter and energy. C) tragedy of the commons. B) law of diminishing returns. D) the Coase theorem. Answer: C

Type: A Topic: 5 E: 563 MI: 319 45. Depletion of fish stocks through overfishing is a good example of the: A) Coase theorem. B) law of increasing returns. C) tragedy of the commons. D) the Coase theorem. Answer: D

McConnell/Brue: Economics, 16/e

Page 939

Chapter 30: Government and Market Failure

Type: D Topic: 5 E: 566 MI: 322 46. The tragedy of the commons is the idea that: A) society has a tendency to overuse and thus abuse common resources. B) total spillover costs in society far outweigh total spillover benefits. C) matter can be transformed to other matter or into energy but can never vanish. D) crime rates typically are higher in public places than where property is privately owned. Answer: A

Type: A Topic: 5 E: 566-567 MI: 322-323 47. In a market for pollution rights an increase in demand would: A) raise the price of pollution rights, but leave the quantity unchanged. B) stimulate the economic incentive to pollute. C) increase the actual amount of pollution. D) induce an increase in the supply of pollution rights. Answer: A

Type: A Topic: 5 E: 567 MI: 323 48. The creation of markets for pollution rights would provide: A) neither an incentive not to pollute nor revenue for environmental improvement. B) funds for environmental improvement, but would not provide an incentive to refrain from polluting. C) an incentive not to pollute, but would not provide funds for environmental improvement. D) both an incentive not to pollute and revenue which could be devoted to environmental improvement. Answer: D

Type: A Topic: 5 E: 567 MI: 323 49. The creation of a market for pollution rights would: A) reduce air and water pollution to zero. B) stimulate the search for pollution-reducing technologies. C) induce an increase in the supply of pollution rights. D) be in conflict with the concept of user charges. Answer: B

McConnell/Brue: Economics, 16/e

Page 940

Chapter 30: Government and Market Failure

Use the following to answer questions 50-52:

Type: G Topic: 5 E: 566-567 MI: 322-323 50. Refer to the above diagram of a market for pollution rights. Which of the following would best explain the P1 to P2 increase in price of pollution rights? A) implementation of improved technology for reducing pollution B) an expansion of the number of firms C) a subsidy of P1P2 to polluters D) a shift of the supply curve of pollution rights from some point to the left of S to S Answer: B

Type: G Topic: 5 E: 566-567 MI: 322-323 51. Refer to the above diagram of a market for pollution rights. The increase in the price of pollution rights from P1 to P2 will: A) reduce the quantity of pollution rights. B) increase the quantity of pollution rights. C) increase the incentive for environmental groups to buy pollution rights. D) increase the opportunity cost of polluting. Answer: D

Type: G Topic: 5 E: 566-567 MI: 322-323 52. Refer to the above diagram of a market for pollution rights. Without this market for pollution rights, the quantity (tons) of pollution would be: A) Q3, if demand is D2. B) Q1, if demand is D1. C) Q2, if demand is D2. D) Q1, if demand is D2. Answer: A

Type: F Topic: 5 E: 567 MI: 323 53. The right to buy and sell emission rights for sulfur-dioxide pollution was granted in the: A) Clean Air Act of 1990. C) Market for Externalities Act of 1970. B) Superfund Law of 1980. D) Acid-Rain Act of 1995. Answer: A

McConnell/Brue: Economics, 16/e

Page 941

Chapter 30: Government and Market Failure

Type: F Topic: 5 E: 567 MI: 323 54. The Clean Air Act of 1990: A) places taxes on utilities emitting sulfur dioxide, the major cause of acid rain. B) sets aside billions of dollars to promote recycling. C) allows utilities to buy and sell sulfur dioxide emission credits provided by government. D) places taxes on producers who emit toxic chemicals. Answer: C

Optimal pollution abatement

Type: A Topic: 6 E: 568-569 MI: 324-325 55. The socially optimal amount of pollution abatement occurs where society's marginal: A) benefit of abatement exceeds its marginal cost of abatement by the greatest amount. B) benefit of abatement equals its marginal cost of abatement. C) benefit of abatement is zero. D) cost of abatement is at its maximum. Answer: B

Type: A Topic: 6 E: 568-569 MI: 324-325 56. The marginal benefit to society of reducing pollution declines with increases in pollution abatement because of the law of: A) increasing costs. C) diminishing marginal utility. B) diminishing returns. D) conservation of matter and energy. Answer: C

Type: A Topic: 6 E: 568-569 MI: 324-325 57. The marginal cost to society of reducing pollution rises with increases in pollution abatement because of the law of: A) diminishing marginal utility. C) demand. B) conservation of matter and energy. D) diminishing returns. Answer: D

McConnell/Brue: Economics, 16/e

Page 942

Chapter 30: Government and Market Failure

Use the following to answer questions 58-62:

Type: G Topic: 6 E: 568-569 MI: 324-325 58. Refer to the above diagram. From society's perspective, if MB1 and MC2 are relevant: A) Q2 represents too little pollution abatement. B) Q1 represents too much pollution abatement. C) Q2 represents an optimal amount of pollution abatement. D) Q4 represents too little pollution abatement. Answer: A

Type: G Topic: 6 E: 568-569 MI: 324-325 59. Refer to the above diagram. From society's perspective, if MB2 and MC1 are relevant: A) Q4 represents too little pollution abatement. B) Q1 represents too little pollution abatement. C) Q3 represents an optimal amount of pollution abatement. D) Q2 represents too little polution abatement. Answer: B

Type: G Topic: 6 E: 568-569 MI: 324-325 60. Refer to the above diagram. With MB1 and MC1, society's optimal amount of pollution abatement is: A) Q1. B) Q2. C) Q3. D) Q4. Answer: A

Type: G Topic: 6 E: 568-569 MI: 324-325 61. Refer to the above diagram. Which one of the following might shift the marginal cost curve from MC1 to MC2? A) major new studies strongly linking cancer to pollution B) improved technology for reducing pollution C) a change in consumer tastes from services to manufactured goods D) an increase in the price of recycled goods Answer: B

McConnell/Brue: Economics, 16/e

Page 943

Chapter 30: Government and Market Failure

Type: G Topic: 6 E: 568-569 MI: 324-325 62. Refer to the above diagram. Which one of the following might shift the marginal benefit curve from MB1 to MB2? A) major new studies strongly linking cancer to pollution B) improved technology for reducing pollution C) a change in consumer tastes from manufacturing goods to services D) a decrease in the price of recycled goods Answer: A

Type: G Topic: 6 E: 568 MI: 324 63. The MC curves in the above diagram slope upward because of the law of: A) demand. C) diminishing marginal utility. B) conservation of matter and energy. D) diminishing returns. Answer: D

Type: G Topic: 6 E: 568 MI: 324 64. The MB curves in the above diagram slope downward because of the law of: A) conservation of matter and energy. C) diminishing marginal utility. B) diminishing returns. D) increasing cost. Answer: C

Recycling

Type: D Topic: 7 E: 569 MI: 325 65. The law of conservation of matter and energy indicates that: A) the elimination of political fragmentation is the only hope for environmental improvement. B) society as a whole, not particularly polluters, should bear the cost of cleansing the environment. C) the only feasible solution to the pollution problem is to create markets for pollution rights. D) the weight of inputs which go into the production process will result in approximately an equivalent amount of wastes. Answer: D

Type: D Topic: 7 E: 569 MI: 325 66. The idea that matter can be transformed into other matter or energy, but cannot vanish: A) is known as the law of sustained yield. B) means that society is better off incinerating garbage than putting it into dumps. C) is known as the law of conservation of matter and energy. D) underlies the idea of diminishing returns. Answer: C

Type: A Topic: 7 E: 570 MI: 326 67. The market demand curve for recyclable items such as aluminum, glass, and paper can best be thought of as a: A) product demand curve. B) resource demand curve. C) vertical line. D) horizontal line. Answer: B

McConnell/Brue: Economics, 16/e

Page 944

Chapter 30: Government and Market Failure

Type: A Topic: 7 E: 569-570 MI: 325-326 68. An increase in the demand for soft drinks will: A) decrease the supply of recyclable aluminum. B) decrease the demand for recyclable aluminum. C) increase the demand for recyclable aluminum. D) decrease the demand for recyclable newspapers. Answer: C

Type: A Topic: 7 E: 569-570 MI: 325-326 69. Which of the following would decrease the equilibrium price of recyclable items while increasing the equilibrium amount of recycling? A) a tax on recycled production inputs B) providing curbside pickup of recyclable items at a lower rate than pickup of normal garbage C) an increase in the demand for recycled inputs D) a pronounced national shift away from manufactured goods toward services. Answer: B

Type: A Topic: 7 E: 570 MI: 326 70. Assuming that new plastic and recyclable plastic are highly substitutable, a tax increase on original resources used to make plastic containers will: A) increase the demand for recyclable plastic. B) decrease the demand for recyclable plastic. C) decrease the supply of recyclable plastic. D) reduce the equilibrium price of recyclable plastic. Answer: A

Type: A Topic: 7 E: 570 MI: 326 71. Additional recycling will reduce economic efficiency if: A) all costs considered, including the externalities associated with alternative disposal, the marginal benefit of recycling exceeds the marginal cost. B) all costs considered, including externalities associated with alternative disposal, the marginal cost of recycling exceeds the marginal benefit. C) the spillover costs associated with garbage dumps and incinerators are sufficiently large. D) the spillover costs associated with garbage dumps are sufficiently small. Answer: B

Type: A Topic: 7 E: 570 MI: 326 72. If both the demand for, and the supply of, recyclable items increase, the equilibrium: A) price of recyclable items will necessarily fall. B) price of recyclable items will necessarily rise. C) quantity of recyclable items will necessarily increase. D) quantity of recyclable items will necessarily decline. Answer: C

McConnell/Brue: Economics, 16/e

Page 945

Chapter 30: Government and Market Failure

Asymmetric information

Type: A Topic: 8 E: 572 MI: 328 73. Where there is asymmetric information between buyers and sellers. A) product shortages will occur at the equilibrium price. B) product surpluses will occur at the equilibrium price. C) markets can produce inefficient outcomes. D) markets will fail due to the "free-rider problem." Answer: C

Type: A Topic: 8 E: 572 MI: 328 74. Buyers will opt out of markets in which: A) there are significant negative externalities. B) standardized products are being produced. C) there is inadequate information about sellers and their products. D) there are only foreign sellers. Answer: C

Type: A Topic: 8 E: 573 MI: 329 75. Sellers will opt out of markets in which: A) there are significant negative externalities. B) standardized products exist. C) there are only foreign buyers. D) information about buyers is inadequate, and some buyers can impose high costs on the sellers. Answer: D

Type: A Topic: 8 E: 573-574 MI: 329-330 76. As it applies to insurance, the moral hazard problem is the tendency for: A) those most likely to collect on insurance to buy it. B) those who buy insurance to take less precaution in avoiding the insured risk. C) sellers to price discriminate. D) sellers to restrict output and charge high prices. Answer: B

Type: A Topic: 8 E: 574 MI: 330 77. As it applies to insurance, the adverse selection problem is the tendency for: A) those most likely to collect on insurance to buy it. B) those who buy insurance to take less precaution in avoiding the insured risk. C) sellers to price discriminate. D) sellers to restrict output and charge high prices. Answer: A

Type: A Topic: 8 E: 574 MI: 330 78. Suppose a firm offers its workers a cafeteria plan in which it allows workers to allocate a set amount of fringe benefit money toward specific insurance. Mary, who has five kids needing braces, selects the family dental coverage. This is an example of the: A) free-rider problem. C) adverse selection problem. B) principle-agent problem. D) moral hazard problem. Answer: C

McConnell/Brue: Economics, 16/e

Page 946

Chapter 30: Government and Market Failure

Type: A Topic: 8 E: 574 MI: 330 79. Because the Federal government typically provides disaster relief to farmers, many farmers do not buy crop insurance even through it is Federally subsidized. This illustrates: A) the adverse selection problem. C) a failure of the market for externalities. B) the moral hazard problem. D) the existence of spillover benefits. Answer: B

Type: A Topic: 8 E: 573-574 MI: 329-330 80. Upon buying a car having airbags, Joe begins to drive recklessly. This is an example of the: A) principle-agent problem. C) moral hazard problem. B) adverse selection problem. D) free-rider problem. Answer: C

Type: A Topic: 8 E: 574-575 MI: 330-331 81. Firms are not likely to provide sufficient workplace safety if: A) workers are unaware of workplace hazards. B) they have some degree of monopoly power. C) they are pure competitors and therefore price takers. D) they are profit-maximizers. Answer: A

Type: A Topic: 8 E: 574-575 MI: 330-331 82. Government can promote workplace safety by: A) breaking up concentrations of monopoly power. B) reducing monopsony power. C) promoting the use of recycled inputs. D) providing workers information about workplace hazards. Answer: D

Type: A Topic: 8 E: 572 MI: 328 Status: New 83. Professional buyers of antiques often have more information about the value of antique objects than do the sellers. This illustrates: A) the Coase theorem. C) the free-rider problem. B) the moral hazard problem. D) asymmetric information. Answer: D

Type: A Topic: 8 E: 572 MI: 328 Status: New 84. Upon learning that his auto transmission is about to fail, Ray Roma sells his car to an unsuspecting buyer. This circumstance illustrates: A) asymmetric information. C) the moral hazard problem. B) the Coase theorem. D) the principal-agent problem. Answer: A

Type: A Topic: 8 E: 573-574 MI: 329-330 Status: New 85. In a television advertisement for AFLAC supplemental health insurance, an ice skater says to his skating partner, "Do you want to try a triple jump?" She responds, "Why not, I have AFLAC." This response illustrates the: A) principal-agent problem. B) Coase theorem. C) moral hazard problem. D) free-rider problem. Answer: C

McConnell/Brue: Economics, 16/e

Page 947

Chapter 30: Government and Market Failure

Consider This Questions

Type: A E: 572 MI: 328 Status: New 86. (Consider This) In the market for used cars: A) "lemon laws" reduce the demand for, and the price of, used cars. B) owners of defective cars have less incentive than owners of high-quality used-cars to sell their cars because buyers refuse to pay high prices for "lemons." C) late-model used-cars look like they are brand new and therefore buyers tend to ignore the possibility that a particular car has defects. D) new cars lose much of their value immediately when buyers drive them off the dealers' lots because potential used-car buyers cannot easily distinguish the defective "lemons" from the high-quality cars. Answer: D

Type: A E: 572 MI: 328 Status: New 87. (Consider This) The "lemon" problem associated with the market for used cars: A) results from asymmetric information and the moral hazard problem. B) results from asymmetric information and adverse selection. C) is an example of the Coase theorem at work. D) in an example of a spillover benefit. Answer: B

Type: A E: 572 MI: 328 Status: New 88. (Consider This) Owners of defective used cars have more information about the condition of their vehicles than potential buyers of those used cars. This is an example of: A) the moral hazard problem. C) a positive externality. B) a spillover cost. D) asymmetric information. Answer: D

Last Word Questions

Type: A E: 576 MI: 332 89. (Last Word) According to economists Ayres and Levitt, owners of car that have traditional automobile alarms: A) may impose costs on owners of unalarmed cars whose cars become greater targets for thieves. B) create spillover benefits for owners of unalarmed cars. C) cause widespread negative externalities in the form of the noise pollution that results from false alarms. D) create a moral hazard problem because they tend to park their cars in riskier locations than they would otherwise. Answer: A

McConnell/Brue: Economics, 16/e

Page 948

Chapter 30: Government and Market Failure

Type: A E: 576 MI: 332 90. (Last Word) According to economists Ayres and Levitt, Lojack and other car retrieval systems that rely on hidden radio transmitters create a significant: A) negative externality because they cause numerous false alarms. B) negative externality in the form of radio signals that are disruptive to cell phone communications. C) positive externality because they help police find and break-up "chop-shops." D) moral hazard problem because people who own them tend to park their cars in riskier locations than they would otherwise. Answer: B

Type: A E: 576 MI: 332 91. (Last Word) According to economists Aryes and Levitt, the total benefits of Lojack and other car retrieval system are divided: A) 90 percent to the owners and 10 percent to other car owners in the community. B) 50 percent to the owners and 50 percent to other car owners in the community. C) 0 percent to the owners and 100 percent to other car owners in the community. D) 10 percent to the owners and 90 percent to other car owners in the community. Answer: D

True/False Questions

Type: A E: 560-561 MI: 316-317 92. That government that has the smallest budget is the most efficient in the economic sense. Answer: False

Type: A E: 559-560 MI: 315-316 93. A demand curve for a public good is determined by summing horizontally the individual demand curves for the public good. Answer: False

Type: A E: 560 MI: 316 94. The optimal quantity of a public good occurs where the marginal benefit of the citizen who has the highest preference for the good just equals the good's marginal cost. Answer: False

Type: F E: 560 MI: 316 95. Cost-benefit analysis is frequently difficult to apply because it is difficult to quantify the full benefits of a public good or service. Answer: True

Type: D E: 563 MI: 319 96. The principle that private negotiation can resolve potential externalities without resort to government intervention is known as the Coase theorem. Answer: True

McConnell/Brue: Economics, 16/e

Page 949

Chapter 30: Government and Market Failure

Type: F E: 566 MI: 322 97. Present law forbids all trades or sales of pollution rights. Answer: False

Type: A E: 573 MI: 329 98. Pollution is a good example of the moral hazard problem. Answer: False

Type: A E: 566 MI: 322 99. Pollution has its roots in the law of conservation of matter and energy. Answer: True

Type: A E: 568 MI: 324 100. Society's optimal amount of pollution abatement is where society's marginal benefit of abatement is zero. Answer: False

Type: A E: 568 MI: 324 101. An improvement in the technology of pollution control is likely to increase society's optimal amount of pollution abatement. Answer: True

Type: A E: 568-569 MI: 324-325 102. Society's marginal cost of pollution abatement curve slopes upward because of the law of diminishing marginal utility. Answer: False

Type: F E: 567 MI: 323 103. The Clean Air Act of 1990 established a program for trading emission rights. Answer: True

Type: A E: 569-570 MI: 325-326 104. Other things equal, an increase in the demand for recycled inputs will reduce the amount of recycling which takes place. Answer: False

Type: D E: 574 MI: 330 105. The adverse selection problem is the tendency for workers to shirk when they are not being monitored. Answer: False

Type: D E: 573 MI: 329 106. The moral hazard problem is the tendency of some parties to a contract to alter their behavior as a result of the contract in ways which are costly to the other party. Answer: True

McConnell/Brue: Economics, 16/e

Page 950

Chapter 30: Government and Market Failure

Type: A E: 568 MI: 324 107. The optimal (economically-efficient) level of air pollution is zero emissions. Answer: False

McConnell/Brue: Economics, 16/e

Page 951

CHAPTER 31

Public Choice Theory and Taxation

Topic 1. 2. 3. 4. 5. 6. 7. 8. 9. Public choice economics Inefficient voting outcomes Paradox of voting Median-voter model Public sector failure Benefits received; ability-to-pay Progressive, proportional, and regressive taxes Tax incidence and efficiency loss Tax system progressivity Last Word True-False

Question numbers 1-6 7-14 15-25 26-34 35-50 51-59 60-73 74-112 113-117 118-120 121-136

____________________________________________________________

_______________________________________

____________________________________________________________

_______________________________________

Multiple Choice Questions Public choice economics

Type: D Topic: 1 E: 580 MI: 336 1. The field of economics that analyzes government decision making, politics, and elections is called: A) public finance. B) public choice theory. C) collective economics. D) political economy. Answer: B

Type: A Topic: 1 E: 580 MI: 336 2. Public choice economists: A) analyze the incidence of taxes. B) are also known as Keynesian economists. C) use the tools of economics to analyze decision making, politics, and elections in the public sector. D) are economists employed by Federal, state, and local governments. Answer: C

Type: A Topic: 1 E: 584 MI: 340 3. Which one of the following topics would be of most interest to a public choice economist? A) the theory of comparative advantage C) inflation and unemployment B) the law of increasing opportunity cost D) rent-seeking behavior Answer: D

Chapter 31: Public Choice Theory and Taxation

Type: A Topic: 1 E: 580 MI: 336 Status: New 4. An economic analysis of the relationship between proposed legislation affecting major employers in each state and the voting patterns of Senators and representatives in Congress on that legislation would fit within the subcategory of economics called: A) the economics of fiscal policy. C) behavioral economics. B) public choice theory. D) monetarism. Answer: B

Type: A Topic: 2 E: 580 MI: 336 Status: New 5. "Government failure" is a prominent topic in: A) public choice theory. B) Keynesian economics. C) socialist theory. D) behavioral economics. Answer: A

Type: D Topic: 1 E: 580 MI: 336 Status: New 6. Public choice theory focuses on the economics of: A) fiscal and monetary policy. B) the behavior of business firms. C) antitrust and regulatory policy. D) government decision making, politics, and elections. Answer: D

Inefficient voting outcomes

Type: A Topic: 2 E: 581 MI: 337 7. Because majority voting fails to incorporate the strength of the preferences of individual voters, it: A) creates negative externalities. B) produces economically inefficient outcomes under some circumstances. C) leads to market failure. D) leads to politics dominated by special interest groups. Answer: B

Type: A Topic: 2 E: 581 MI: 337 8. Suppose that Kate and Kyle perceive $500 of marginal benefit from a proposed new recreation center, whereas Keith perceives $100. If the tax levied on each for the center is $400, a majority vote will: A) defeat this project and resources will be underallocated to it. B) pass this project and resources will be efficiently allocated. C) pass the this project and resources will be underallocated to it. D) pass this project and resources will be overallocated to it. Answer: D

Type: A Topic: 2 E: 581 MI: 337 9. Suppose that Sam and Susan perceive $200 of marginal benefit from a proposed new park, whereas Keith perceives $800. If the tax levied on each for the park is $300, a majority vote will: A) defeat this project and resources will be underallocated to it. B) pass this project and resources will be efficiently allocated. C) pass the this project and resources will be underallocated to it. D) pass this project and resources will be overallocated to it. Answer: A

McConnell/Brue: Economics, 16/e

Page 954

Chapter 31: Public Choice Theory and Taxation

Use the following to answer question 10: Answer the next question on the basis of this table showing the marginal benefit that a particular public project will provide to each of the three members of a community. No vote trading is allowed.
Voter Xavier Yoho Zest Marginal benefit $ 500 200 2000

Type: T Topic: 2 E: 581 MI: 337 10. If the tax cost of this proposed project is $600 per person, a majority vote will: A) defeat this project and resources will be underallocated to it. B) defeat this project and resources will be allocated efficiently. C) pass this project and resources will be overallocated to it. D) defeat this project and resources will be overallocated to it. Answer: A

Use the following to answer question 11: Answer the next question on the basis of this table showing the marginal benefit a particular public project will provide to each of the three members of a community. No vote trading is allowed. Voter Xavier Yoho Zest Marginal benefit $800 800 100

Type: T Topic: 2 E: 581 MI: 337 11. If the tax cost of this proposed project is $600 per person, a majority vote will: A) defeat this project and resources will be underallocated to it. B) pass this project and resources will be allocated efficiently. C) pass this project and resources will be overallocated to it. D) pass this project and resources will be underallocated to it. Answer: C

Type: A Topic: 2 E: 582 MI: 338 Status: New 12. The political technique called logrolling: A) always increases economic efficiency. B) always decreases economic efficiency. C) involves trading votes to secure favorable outcomes that otherwise would be rejected. D) is an example of the paradox of voting. Answer: C

Type: D Topic: 2 E: 582 MI: 338 Status: New 13. The trading of votes by elected officials to secure favorable outcomes is called: A) splitting the difference. B) social engineering. C) logrolling. D) grandstanding. Answer: C

McConnell/Brue: Economics, 16/e

Page 955

Chapter 31: Public Choice Theory and Taxation

Type: A Topic: 2 E: 582 MI: 338 Status: New 14. Senator A agrees to vote for Senator K's state project in exchange for Senator K voting for Senator A's state project. This is an example of: A) log rolling. C) the principal-agent problem. B) the paradox of voting. D) the median voter model. Answer: A

Paradox of voting

Type: D Topic: 3 E: 582-583 MI: 338-339 15. A situation in which society may not be able to rank its preferences consistently through majority voting refers to: A) logrolling. B) the paradox of voting. C) the median-voter model. D) the special-interest effect. Answer: B

Type: D Topic: 3 E: 582-583 MI: 338-339 16. The principle that under some circumstances majority voting fails to make consistent choices reflecting the community's underlying preference is best demonstrated by the: A) paradox of voting. B) concept of logrolling. C) median-voter model. D) Coase theorem. Answer: A

Type: D Topic: 3 E: 582-583 MI: 338-339 17. According to the paradox of voting: A) public goods that cost more than the total benefits they confer may get produced under majority voting. B) trading of votes may either add to or subtract from economic efficiency. C) the median voter decides what public goods all voters should have. D) majority voting fails under some circumstances to make consistent choices that reflect the community's underlying preferences. Answer: D

Type: A Topic: 3 E: 582-583 MI: 338-339 18. Suppose that in a series of paired-choice votes a new park is preferred to new recreation center and a new recreation center is preferred to street widening. Also suppose that street widening is preferred to a new park. This set of votes is an example of the: A) principal-agent problem. C) median-voter model. B) benefits-received principle. D) paradox of voting. Answer: D

McConnell/Brue: Economics, 16/e

Page 956

Chapter 31: Public Choice Theory and Taxation

Use the following to answer questions 19-21:

Public good Lighthouse Highway Submarine

Larry third choice second choice first choice

Curley first choice third choice second choice

Moe second choice first choice third choice

Type: A Topic: 3 E: 582-583 MI: 338-339 19. Refer to the above table. In a choice between the highway and the lighthouse: A) a majority of voters favor the lighthouse. C) the voters are indifferent as between the two. B) a majority of voters favor the highway. D) no voter decision is possible. Answer: B

Type: A Topic: 3 E: 582-583 MI: 338-339 20. The inconsistency illustrated by the above table is that, while a majority of voters prefer the: A) highway to the lighthouse and the submarine to the highway, they also prefer the lighthouse to the submarine. B) lighthouse to the highway and the lighthouse to the submarine, they also prefer the submarine to the highway. C) highway to the lighthouse and the submarine to the lighthouse, they also prefer the submarine to the highway. D) lighthouse to the submarine and the highway to the submarine, they also prefer the highway to the lighthouse. Answer: A

Type: A Topic: 3 E: 582-583 MI: 338-339 21. The information contained in the above table illustrates: A) political logrolling. C) the paradox of voting. B) the median-voter model. D) the Coase theorem. Answer: C

Use the following to answer questions 22-25:
Public good Public safety Parks Education Adam second choice first choice third choice Bob third choice second choice first choice Carl first choice third choice second choice

Type: A Topic: 3 E: 582-583 MI: 338-339 Status: New 22. Refer to the above table. In a choice between public safety and parks: A) the outcome would depend on which item was listed first on the ballot. B) a majority of voters would favor public safety. C) no voter decision is possible. D) a majority of voters would favor parks. Answer: D

McConnell/Brue: Economics, 16/e

Page 957

Chapter 31: Public Choice Theory and Taxation

Type: A Topic: 3 E: 582-583 MI: 338-339 Status: New 23. Refer to the above table. In a choice between parks and education: A) no voter decision is possible. B) a majority of voters would favor education. C) the outcome would depend on which item was listed first on the ballot. D) a majority of voters would favor parks. Answer: B

Type: A Topic: 3 E: 582-583 MI: 338-339 Status: New 24. Refer to the above table. In a choice between education and public safety: A) a majority of voters would favor education. B) no voter decision is possible. C) a majority of voters would favor public safety. D) the outcome would depend on which item was listed first on the ballot. Answer: C

Type: A Topic: 3 E: 582-583 MI: 338-339 Status: New 25. The results of the three sets of paired-choice majority voting in the answers to Questions 22-24 illustrate the: A) paradox of voting. C) principal-agent problem. B) inefficiency of logrolling. D) the benefits of majority rule. Answer: A

Median-voter model

Type: A Topic: 4 E: 583 MI: 339 26. The median-voter model implies that: A) political voting will be as economically efficient as "dollar voting" in competitive markets. B) all voters have about the same preferences for various public goods and services. C) many people will be dissatisfied with the size of government in the economy. D) with majority voting there can never be a consistent ordering of public good preferences. Answer: C

Type: A Topic: 4 E: 583 MI: 339 27. The median-voter model implies that a political office seeker will: A) adopt more extreme views when seeking his or her party's nomination than when running against the other party's opponent. B) adopt less extreme views when seeking his or her party's nomination than when running against the other party's opponent. C) favor extensive government spending because demand curves for public goods are added vertically rather than horizontally. D) favor the private resolution of externality problems rather than governmental intervention. Answer: A

McConnell/Brue: Economics, 16/e

Page 958

Chapter 31: Public Choice Theory and Taxation

Use the following to answer questions 28-30: Answer the next question(s) on the basis of the following table that shows the total costs and total benefits facing a city of five different potential baseball stadiums of increasing size. All figures are in millions of dollars.
Stadium A B C D E Total cost $ 80 100 130 165 220 Total benefit $140 200 250 290 300

Type: T Topic: 4 E: 583-584 MI: 339-340 28. Refer to the above table. Suppose a five-person city council must decide via majority voting which of these stadiums to build. Also suppose that each of the stadium sizes has the support of one council member. According to the median voter model, the council will ultimately vote in favor of stadium: A) A. B) B. C) C. D) D. Answer: C

Type: T Topic: 4 E: 583-584 MI: 339-340 29. Refer to the above table. The marginal cost and marginal benefit of stadium B (relative to A) are: A) $20 million and $50 million, respectively. C) $30 million and $50 million, respectively. B) $100 million and $200 million, respectively. D) $20 million and $60 million, respectively. Answer: D

Type: T Topic: 4 E: 583-584 MI: 339-340 30. Refer to the above table. Based on cost-benefit analysis, the city should: A) not build any of these stadiums. B) build stadium E. C) build stadium C. Answer: D

D) build stadium D.

Type: A Topic: 4 E: 583 MI: 339 31. Suppose three roommates cannot agree on the size of a pizza to order. Domino argues for a medium pizza, Godfather contends a large pizza will be needed, and Little Caesar wants a super-large pizza. Assuming no paradox of voting, majority voting will result in a decision to order: A) a medium pizza. B) a large pizza. C) a super-large pizza. D) two medium pizzas. Answer: B

Type: A Topic: 4 E: 583 MI: 339 32. Some people argue that the three main television networks all have similar programming. If true, this observation might best be explained by the: A) paradox of voting. C) law of diminishing marginal utility. B) median-voter model. D) ability-to-pay principle. Answer: B

McConnell/Brue: Economics, 16/e

Page 959

Chapter 31: Public Choice Theory and Taxation

Type: A Topic: 4 E: 583 MI: 339 33. Suppose that friends Jennifer, Stephanie, and Megan cannot agree on how expensive a bouquet of flowers to send to a person that allowed them to use their beach house for the weekend. Jennifer want to buy a moderately priced bouquet, Stephanie wants to buy an expensive bouquet, and Megan wants to buy a very expensive bouquet. Assuming no paradox of voting, majority voting will result in decision to buy: A) an inexpensive bouquet. C) a moderately priced bouquet. B) a very expensive bouquet. D) an expensive bouquet. Answer: D

Type: T Topic: 4 E: 582-583 MI: 338-339 Status: New 34. Suppose a college economics department decides to use a single economics text for all sections of principles of economics. Also assume that the three individual members of the textbook selection committee have the following preferences.

Professor Adams Bennett Clay

Ideal Textbook C/F M/B O/S

Analytical level of the textbook very difficult moderately difficult relatively easy

Assuming all other textbook qualities except analytical level are the same, paired-choice majority voting will result in the committee: A) being deadlocked and unable to decide on a book. B) selecting the C/F book. C) selecting the M/B book. D) selecting the O/S book. Answer: C

Public sector failure

Type: D Topic: 5 E: 584 MI: 340 35. "Vote for my special local project and I will vote for yours." This political technique: A) illustrates the paradox of voting. C) illustrates the median voter model. B) often accompanies pork-barrel politics. D) undermines the benefits-received principle. Answer: B

Type: D Topic: 5 E: 584 MI: 340 36. Factors that impede the attainment of economic efficiency in the public sector are called: A) market failures. B) externalities. C) government failures. D) voting irregularities. Answer: C

Type: F Topic: 5 E: 585 MI: 341 37. According to public choice theorists, the private sector is more efficient than the public sector mainly because: A) the private sector has a clear test of performance: profit and loss. B) wages, salaries, and fringe benefits are higher in the private sector. C) worker turnover is higher in the public sector. D) of extensive negative externalities in the public sector. Answer: A

McConnell/Brue: Economics, 16/e

Page 960

Chapter 31: Public Choice Theory and Taxation

Type: A Topic: 5 E: 583-584 MI: 339-340 38. The idea of government failure includes all of the following except: A) limited and bundled choices. B) bureaucratic inefficiency. C) pressure by special-interest groups. D) extensive positive externalities from public and quasi-public goods. Answer: D

Type: A Topic: 5 E: 584 MI: 340 39. Suppose American winemakers convince the Federal government to issue a directive to serve only domestically produced wine at government functions. This would be an example of: A) nonselectivity. B) the Coase theorem. C) logrolling. D) rent-seeking behavior. Answer: D

Type: A Topic: 5 E: 584 MI: 340 40. Public choice theorists contend that: A) government can efficiently correct instances of market system failure. B) the existence of cost-benefit analysis has brought about the efficient use of resources in the public sector. C) public bureaucracies are inherently more efficient than private enterprises. D) public bureaucracies are inherently less efficient than private enterprises. Answer: D

Type: D Topic: 5 E: 584 MI: 340 41. The pursuit through government of a transfer of wealth at someone else's expense refers to: A) logrolling. B) rent-seeking behavior. C) the paradox of voting. D) the median-voter model. Answer: B

Type: D Topic: 5 E: 584 MI: 340 42. Economists call the pursuit of a transfer of wealth through government at someone else's expense: A) the paradox of voting. C) rent-seeking behavior. B) the Coase theorem. D) the benefits-received principle. Answer: C

Type: A Topic: 5 E: 585 MI: 341 43. Public choice theorists hold that politicians will: A) favor programs entailing immediate and clear-cut costs and vaguely defined or deferred benefits. B) follow policies leading to an optimal allocation of resources between public and private sectors. C) favor programs entailing immediate and clear-cut benefits and vaguely defined or deferred costs. D) objectively weigh the costs and benefits of various government programs and vote accordingly. Answer: C

Type: A Topic: 5 E: 585-586 MI: 341-342 44. Public choice theorists contend public bureaucracies are inefficient primarily because: A) the value of public goods is more easily measured than is the value of private goods. B) of the absence of competitive market pressures. C) public sector workers are more security-conscious than are private sector workers. D) relatively low pay in government attracts workers of lesser quality. Answer: B

McConnell/Brue: Economics, 16/e

Page 961

Chapter 31: Public Choice Theory and Taxation

Type: D Topic: 5 E: 585 MI: 341 45. A special-interest issue is one whose passage yields: A) large private benefits compared to external benefits. B) large external benefits compared to private benefits. C) small economic losses to a small number of people and large economic losses to a large number of people. D) large economic gains to a small number of people and small economic losses to a large number of people. Answer: D

Type: A Topic: 5 E: 585 MI: 341 46. Public choice theorists point out that the political process: A) differs from the marketplace in that voters and congressional representatives often face limited and bundled choices. B) is less prone to failure than is the marketplace. C) is a much fairer way to allocate society's scarce resources than is the impersonal marketplace, which is dominated by high-income consumers. D) involves logrolling, which is always inefficient. Answer: A

Type: A Topic: 5 E: 585 MI: 341 47. When congressional representatives vote on an appropriations bill, they must vote yea or nay, taking the bad with the good. This statement best reflects the: A) paradox of voting. C) benefits-received principle. B) special-interest effect. D) idea of limited and bundled choice. Answer: D

Type: D Topic: 2 E: 584 MI: 340 48. "Pork-barrel" legislation that contains funding for hundreds of separate special products scattered throughout numerous states often reflects: A) the paradox of voting. B) logrolling. C) the benefits-received principle. D) the Coase theorem. Answer: B

Type: A Topic: 5 E: 584-585 MI: 340-341 Status: New 49. Suppose lawyers seek legislation to limit the use of computer software that enables people to use their personal computers to self-prepare their own wills, trusts, and other legal documents. This is an example of: A) logrolling. C) rent-seeking behavior. B) the principal-agent problem. D) limited and bundled choices. Answer: C

Type: A Topic: 5 E: 585 MI: 341 Status: New 50. In a sporting good store, you can buy the equipment you want and forgo the rest. But in an election you "buy" the entire range of the candidate's positions, including some you may not agree with. This difference: A) reflects limited and bundled choices in the public sector. B) describes the paradox of voting. C) describes the principal-agent problem in the public sector. D) creates bureaucratic inefficiency in the public sector. Answer: A

McConnell/Brue: Economics, 16/e

Page 962

Chapter 31: Public Choice Theory and Taxation

Benefits received; ability-to-pay

Type: D Topic: 6 E: 587 MI: 343 51. The ability-to-pay principle of taxation: A) has been declared unconstitutional because it deprives individuals of property without due process of law. B) suggests that people should pay taxes in proportion to the benefits they derive from public goods and services. C) suggests that taxes should vary directly with people's income and wealth. D) suggests that taxes should vary inversely with people's income and wealth. Answer: C

Type: A Topic: 6 E: 587 MI: 343 52. Which of the following taxes reflects the benefits-received principle of taxation? A) cigarette taxes B) gasoline taxes C) sales taxes D) Federal personal income taxes Answer: B

Type: A Topic: 6 E: 587 MI: 343 53. The benefits-received principle of taxation is most evident in: A) inheritance taxes. C) the personal income tax. B) excise taxes on gasoline. D) the corporate income tax. Answer: B

Type: A Topic: 6 E: 587 MI: 343 54. Which of the following best reflects the ability-to-pay philosophy of taxation? A) a tax on residential property C) an excise tax on gasoline B) a progressive income tax D) an excise tax on coffee Answer: B

Type: A Topic: 6 E: 587 MI: 343 55. Entry fees at national parks and monuments are an example of: A) the ability-to-pay principle of taxation. C) government bureaucracy and inefficiency. B) the benefits-received principle of taxation. D) the principle of limited and bundled choice. Answer: B

Type: A Topic: 6 E: 587 MI: 343 56. The rationale for ability-to-pay taxation and the contention that those with large incomes should pay more taxes both absolutely and relatively is that: A) high-income receivers are generally in a better position to shift taxes than are low-income receivers. B) the transfer system is regressive and it is therefore essential to have an offsetting progressive tax structure. C) rational consumers spend their first dollars of income on the most urgently desired goods and successive dollars on less essential goods. D) taxes should be paid for financing public goods in direct proportion to the satisfaction an individual derives from those goods. Answer: C

McConnell/Brue: Economics, 16/e

Page 963

Chapter 31: Public Choice Theory and Taxation

Type: A Topic: 6 E: 587 MI: 343 57. Which of the following statements is most consistent with the benefits-received principle of taxation? A) A childless couple should not be required to pay taxes for the support of public schools. B) Prosperous corporations should pay substantial taxes even if they use few government goods and services. C) The best tax is the income tax. D) People with high incomes should pay more taxes than people with low incomes. Answer: A

Type: A Topic: 6 E: 587 MI: 343 Status: New 58. The Federal gasoline tax is assessed on a per-gallon basis and the proceeds are used for highway maintenance and improvements. This tax is consistent with the: A) ability-to-pay principal of taxation. C) single-tax theory of taxation. B) benefits-received principal of taxation. D) pay-as-you go theory of taxation. Answer: B

Type: A Topic: 6 E: 588 MI: 344 Status: New 59. The Federal income tax is consistent with the __________ principle of taxation, whereas an excise tax on sporting event tickets is consistent with the ___________ principle of taxation. A) benefits-received; ability-to-pay C) ability-to-pay; benefits received B) benefits-received; pay-as-you go D) ability-to-pay; pay-as-you-go. Answer: C

Progressive, proportional, and regressive taxes

Type: A Topic: 7 E: 588 MI: 344 60. Using income as the tax base, which of the following is a regressive tax? A) the Federal inheritance tax C) the corporate income tax B) a 7 percent general sales tax D) the personal income tax Answer: B

Use the following to answer questions 61-63: Answer the next question(s) on the basis of the following five schedules, all of which represent income tax schedules for an economy. All figures are in billions of dollars.
I Tax $30 50 60 70 Base (income) $100 200 300 400 Tax $10 20 30 40 II Base (income) $100 200 300 400 III Tax $ 5 15 30 50 Base (income) $100 200 300 400 IV Tax $ 30 60 90 120 Base (income) $100 200 300 400 V Base Tax (income) $ 10 $100 30 200 60 300 100 400

Type: T Topic: 7 E: 588 MI: 344 61. Which of the above schedules represent(s) a progressive tax? A) V only B) III and V C) II and III D) III only Answer: B

McConnell/Brue: Economics, 16/e

Page 964

Chapter 31: Public Choice Theory and Taxation

Type: T Topic: 7 E: 588 MI: 344 62. Which of the above schedules represent(s) a regressive tax? A) V only B) III and V C) IV only D) I only Answer: D

Type: T Topic: 7 E: 588 MI: 344 63. Which of the above schedules represent(s) a proportional tax? A) II and IV B) III and V C) IV only D) I only Answer: A

Type: A Topic: 7 E: 588 MI: 344 64. The sales tax is a regressive tax because the: A) percentage of income paid as taxes falls as income rises. B) administrative costs associated with the collection of the tax are relatively high. C) percentage of income paid as taxes is constant as income rises. D) tax tends to reduce the total volume of consumption expenditures. Answer: A

Type: D Topic: 7 E: 588 MI: 344 65. A tax that takes a larger proportion of income from low-income groups than from high-income groups is a: A) stabilizing tax. B) regressive tax. C) progressive tax. D) proportional tax. Answer: B

Type: A Topic: 7 E: 588 MI: 344 66. If each taxpayer paid the same lump-sum amount regardless of income level, the tax system would be: A) disproportionate. B) progressive. C) proportional. D) regressive. Answer: D

Type: A Topic: 7 E: 588 MI: 344 67. The general sales tax is regressive because: A) the incidence of the tax is on sellers rather than buyers. B) tax rates are constant and incomes are variable. C) lower income families save a smaller percentage of their incomes than do higher income families. D) tax rates decrease as consumer spending increases. Answer: C

McConnell/Brue: Economics, 16/e

Page 965

Chapter 31: Public Choice Theory and Taxation

Use the following to answer questions 68-70:

Type: G Topic: 7 E: 588 MI: 344 68. Which of the lines in the above diagram represent(s) a progressive tax? A) both A and B. B) D only. C) C only. D) B only. Answer: A

Type: G Topic: 7 E: 588 MI: 344 69. Which of the lines in the above diagram represent(s) a proportional tax? A) both A and B. B) D only. C) C only. D) B only. Answer: C

Type: G Topic: 7 E: 588 MI: 344 70. Which of the lines in the above diagram represent(s) a regressive tax? A) both A and B. B) D only. C) C only. D) B only. Answer: B

Type: A Topic: 7 E: 588 MI: 344 71. Assume you pay a tax of $4000 on a taxable income of $24,000. If your taxable income were $30,000, your tax payment would be $5000. This suggests that the tax is: A) progressive. B) proportional. C) regressive. D) discriminatory. Answer: B

Type: A Topic: 7 E: 588 MI: 344 Status: New 72. Assume that you pay $5000 of tax on a taxable income of $20,000. If your taxable income were $40,000, your tax payment would be $12,000. This suggests the tax is: A) progressive. B) proportional. C) regressive. D) discriminatory. Answer: A

Type: A Topic: 7 E: 588 MI: 344 Status: New 73. Assume that you pay $10,000 of tax on a taxable income of $50,000. If your taxable income were $150,000, your tax payment would be $25,000. This suggests the tax is: A) progressive. B) proportional. C) regressive. D) discriminatory. Answer: C

McConnell/Brue: Economics, 16/e

Page 966

Chapter 31: Public Choice Theory and Taxation

Tax incidence and efficiency loss

Type: D Topic: 8 E: 589 MI: 345 74. The incidence of a tax pertains to: A) the degree to which it alters the distribution of income. B) how easy it is to evade the tax. C) who actually bears the burden of a tax. D) the progressiveness or regressiveness of tax rates. Answer: C

Use the following to answer questions 75-76: Answer the next question(s) on the basis of the following demand and supply data for a competitive market:

Quantity demanded 1,500 2,000 2,500 3,000 3,500

Price $10 9 8 7 6

Quantity supplied 3,500 3,000 2,500 2,000 1,500

Type: T Topic: 8 E: 589 MI: 345 75. Refer to the above data. If government levies a per unit excise tax of $1 on suppliers of this product, equilibrium price and quantity will be: A) $9 and 3,000. B) $7.50 and 2,250. C) $8.50 and 2,250. D) $7 and 3,000. Answer: C

Type: T Topic: 8 E: 589 MI: 345 76. Refer to the above data. If government has instead provided a per unit subsidy of $2 to suppliers of this product, equilibrium price and quantity would be: A) $9 and 3,000. B) $7.50 and 2,250. C) $8.50 and 2,750. D) $7 and 3,000. Answer: D

Type: A Topic: 8 E: 589 MI: 345 77. Assume the Environmental Protection Agency imposes an excise tax on polluting firms. In which of the following situations would we expect the additional costs to be borne most heavily by consumers? A) demand is highly elastic and supply is highly inelastic B) demand and supply are both highly elastic C) demand and supply are both highly inelastic D) demand is highly inelastic and supply is highly elastic Answer: D

McConnell/Brue: Economics, 16/e

Page 967

Chapter 31: Public Choice Theory and Taxation

Type: A Topic: 8 E: 589 MI: 345 78. If the demand for a product is perfectly elastic and supply is upsloping, a $1 excise tax per unit on suppliers will: A) not raise price at all. C) raise price by more than $1. B) lower price by $1. D) raise price by $1. Answer: A

Type: A Topic: 8 E: 589 MI: 345 79. If the supply of a product is perfectly elastic and demand is downsloping, an excise tax of $2 per unit will increase price by: A) more than $2. C) $2 and increase equilibrium output. B) less than $2. D) $2 and reduce equilibrium output. Answer: D

Type: C Topic: 8 E: 590 MI: 346 80. Suppose that government imposes a specific excise tax on product X of $2 per unit and that the price elasticity of demand for X is unitary (coefficient = 1). If the incidence of the tax is such that the producers of X pay $1.75 of the tax and the consumers pay $.25, we can conclude that the: A) supply of X is highly inelastic. C) demand for X is highly inelastic. B) supply of X is highly elastic. D) demand for X is highly elastic. Answer: A

Type: C Topic: 8 E: 589 MI: 345 81. Suppose that government imposes a specific excise tax on product X of $2 per unit and that the price elasticity of demand for X is unitary (coefficient = 1). If the incidence of the tax is such that consumers pay $1.80 of the tax and the producers pay $.20, we can conclude that the: A) supply of X is highly inelastic. C) demand for X is highly inelastic. B) supply of X is highly elastic. D) demand for X is highly elastic. Answer: B

Type: C Topic: 8 E: 589 MI: 345 82. Suppose that government imposes a specific excise tax on product X of $2 per unit and that the price elasticity of supply of X is unitary (coefficient = 1). If the incidence of the tax is such that the producers of X pay $1.90 of the tax and the consumers pay $.10, we can conclude that the: A) supply of X is highly inelastic. C) demand for X is highly inelastic. B) supply of X is highly elastic. D) demand for X is highly elastic. Answer: D

Type: C Topic: 8 E: 590 MI: 346 83. Suppose that government imposes a specific excise tax on product X of $2 per unit and that the price elasticity of supply of X is unitary (coefficient = 1). If the incidence of the tax is such that the consumers of X pay $1.85 of the tax and the consumers pay $.15, we can conclude that the: A) supply of X is highly inelastic. C) demand for X is highly inelastic. B) supply of X is highly elastic. D) demand for X is highly elastic. Answer: C

McConnell/Brue: Economics, 16/e

Page 968

Chapter 31: Public Choice Theory and Taxation

Use the following to answer questions 84-86:

Type: G Topic: 8 E: 589-590 MI: 345-346 84. In which of the above market situations will the largest portion of an excise tax of a specified amount per unit of output be borne by producers? A) 4 B) 3 C) 1 D) 2 Answer: B

Type: G Topic: 8 E: 589-590 MI: 345-346 85. In which of the above market situations will the largest portion of an excise tax of a specified amount per unit of output be borne by buyers? A) 4 B) 3 C) 1 D) 2 Answer: C

Type: G Topic: 8 E: 590 MI: 346 86. In which of the above market situations will the efficiency loss of an excise tax be the greatest? A) 4 B) 3 C) 1 D) 2 Answer: A

Type: A Topic: 8 E: 589-590 MI: 345-346 87. Assume the demand for automobile tires is highly inelastic and that the supply is highly elastic. The burden of a $2 excise tax on each tire will be: A) borne by resource suppliers who provide the inputs for manufacturing tires. B) shared about equally by buyers and sellers of tires. C) borne primarily by buyers of tires. D) borne primarily by sellers of tires. Answer: C

McConnell/Brue: Economics, 16/e

Page 969

Chapter 31: Public Choice Theory and Taxation

Type: A Topic: 8 E: 589-590 MI: 345-346 88. Assume the supply curve for product X is perfectly elastic and that government imposes a $2 per unit excise tax. We can conclude that the resulting: A) increase in output will be greater the less elastic the demand curve. B) decrease in output will be greater the less elastic the demand curve. C) decrease in output will be greater the more elastic the demand curve. D) increase in output will be greater the more elastic the demand curve. Answer: C

Type: A Topic: 8 E: 589-590 MI: 345-346 89. If the demand for a product is perfectly inelastic and the supply curve is upsloping, a $1 excise tax per unit of output will: A) raise price by less than $1. C) raise price by $1. B) raise price by more than $1. D) lower price by $1. Answer: C

Use the following to answer questions 90-92:

Type: G Topic: 8 E: 590 MI: 346 90. Refer to the above figure in which S is the before-tax supply curve and St is the supply curve after an excise tax is imposed. The total tax collection from this excise tax will be area: A) ABCE + ECF. B) ABCE. C) ECF. D) 0BCG. Answer: B

Type: G Topic: 8 E: 590 MI: 346 91. Refer to the above figure in which S is the before-tax supply curve and St is the supply curve after an excise tax is imposed. The efficiency loss of this tax will be area: A) ABCE. B) ABCE + ECF. C) 0AEG. D) ECF. Answer: D

Type: G Topic: 8 E: 590 MI: 346 92. Refer to the above figure in which S is the before-tax supply curve and St is the supply curve after an excise tax is imposed. The burden of this tax is borne: A) equally by consumers and producers. C) most heavily by producers. B) most heavily by consumers. D) only by consumers. Answer: B

McConnell/Brue: Economics, 16/e

Page 970

Chapter 31: Public Choice Theory and Taxation

Type: A Topic: 8 E: 591 MI: 347 93. The more inelastic is demand and supply the: A) larger is the efficiency loss of an excise tax. B) smaller is the efficiency loss of an excise tax. C) higher the proportion of an excise tax paid by consumers. D) smaller the proportion of an excise tax paid by producers. Answer: B

Type: A Topic: 8 E: 589 MI: 345 94. If the demand for a product is perfectly inelastic, the incidence of an excise tax will be: A) entirely on the buyer. B) mostly on the buyer. C) entirely on the seller. D) mostly on the seller. Answer: A

Type: A Topic: 8 E: 589-590 MI: 345-346 95. Which of the following generalizations is correct? A) the more elastic the supply of a product, the larger the portion of an excise tax that is borne by buyers B) the more elastic the demand for a product, the larger the portion of an excise tax that is borne by buyers C) the more inelastic the supply of a product, the larger the portion of an excise tax that is borne by buyers D) the more inelastic the demand for a product, the larger the portion of an excise tax that is borne by sellers Answer: A

Type: D Topic: 8 E: 590 MI: 346 96. The efficiency loss of a tax is the idea that: A) in addition to taking income from the citizenry, taxes also increase the rate of inflation. B) taxes cause a decline in output for which marginal benefit exceeds marginal cost. C) taxes diminish incentives to work. D) government spends dollars less efficiently than do households and businesses. Answer: B

Type: A Topic: 8 E: 590-591 MI: 346-347 97. The greater the elasticity of supply of and demand for a good the: A) smaller will be the efficiency loss of an excise tax on the good. B) more likely the good will be a public good rather than a private good. C) larger will be the efficiency loss of an excise tax on the good. D) larger will be the proportion of an excise tax on the good that will be shifted forward to consumers. Answer: C

Type: D Topic: 8 E: 590-591 MI: 346-347 98. The efficiency loss of a tax is: A) the net value of sacrificed output caused by the tax. B) that portion of the tax paid by producers minus the portion paid by consumers. C) that portion of the tax paid by consumers minus the portion paid by producers. D) the total tax revenue minus the output loss caused by the tax. Answer: A

McConnell/Brue: Economics, 16/e

Page 971

Chapter 31: Public Choice Theory and Taxation

Use the following to answer questions 99-102:

Type: G Topic: 8 E: 591 MI: 347 99. Refer to the above diagram in which S is the before-tax supply curve and St is the supply curve after an excise tax is imposed. The total tax payment to government is shown by area(s): A) A only. B) A + B + C + E + F . C) A + B + C. D) E + F. Answer: C

Type: G Topic: 8 E: 591 MI: 347 100. Refer to the above diagram in which S is the before-tax supply curve and St is the supply curve after an excise tax is imposed. The total amount of the tax paid by consumers is shown by area(s): A) A + B + F. B) A + B. C) A + B + C. D) E + F. Answer: B

Type: G Topic: 8 E: 591 MI: 347 101. Refer to the above diagram in which S is the before-tax supply curve and St is the supply curve after an excise tax is imposed. The total amount of the tax paid by producers is shown by area(s): A) A + B + F. B) C only. C) A + B + C. D) E + F. Answer: B

Type: G Topic: 8 E: 591 MI: 347 102. Refer to the above diagram in which S is the before-tax supply curve and St is the supply curve after an excise tax is imposed. The efficiency loss of the tax is shown by area(s): A) A + B + C + E + F . B) A + B + C. C) A + B + F. D) E + F. Answer: D

Use the following to answer questions 103-107: (Advanced analysis) Answer the next question(s) on the basis of the following information: The equations for the demand and supply curves for a particular product are P = 10 - .4Q and P = 2 + .4Q, where P is price and Q is quantity expressed in units of 100. After an excise tax is imposed on the product the supply equation is P = 3 + .4 Q.

Type: E Topic: 8 E: 592 MI: 348 103. Refer to the above information. The excise tax on each unit of the product is: A) $1. B) $2. C) $3. D) $4. Answer: A

McConnell/Brue: Economics, 16/e

Page 972

Chapter 31: Public Choice Theory and Taxation

Type: E Topic: 8 E: 592 MI: 348 104. Refer to the above information. The equilibrium quantity before the excise tax is imposed is: A) 800 units. B) 1000 units. C) 1200 units. D) 1400 units. Answer: B

Type: E Topic: 8 E: 592 MI: 348 105. Refer to the above information. The equilibrium quantity after the excise tax is imposed is: A) 750 units. B) 850 units. C) 875 units. D) 950 units. Answer: C

Type: E Topic: 8 E: 592 MI: 348 106. Refer to the above information. Government's revenue from this tax is: A) $750. B) $1500. C) $875. D) $800. Answer: C

Type: E Topic: 8 E: 591 MI: 347 107. Refer to the above information. The efficiency loss of this tax is: A) $125.00. B) $62.50. C) $87.50. D) $1.00. Answer: B

Type: A Topic: 8 E: 591 MI: 347 108. The incidence of an excise tax is: A) such that the tax is progressive in its effect. B) distributed among the consumer and the seller in a manner determined by the elasticities of demand and supply. C) upon the consumer of the product. D) upon the seller of the product. Answer: B

Type: D Topic: 8 E: 592 MI: 348 109. Sales and excise taxes are levied on retailers, but retailers add these taxes to the prices of their products. This illustrates the: A) equal sacrifice theory of taxation. C) ability-to-pay principle of taxation. B) shifting of taxes to consumers. D) benefits-received principle of taxation. Answer: B

Type: A Topic: 8 E: 592 MI: 348 110. The property tax may be regressive even though wealthy people own much more taxable property than do poor people. This possibility arises because: A) marginal and average tax rates on property tend to converge. B) wealthy people can evade property taxes while poor people cannot. C) property taxes on rental property and business property are shifted. D) statutory property tax rates decline as the value of property rises. Answer: C

McConnell/Brue: Economics, 16/e

Page 973

Chapter 31: Public Choice Theory and Taxation

Type: A Topic: 8 E: 592 MI: 348 111. Which of the following taxes is most likely to be shifted? A) a general sales tax C) a progressive Federal income tax B) a flat-rate state income tax D) a property tax on an owner-occupied residence Answer: A

Type: A Topic: 8 E: 592 MI: 348 112. Which of the following taxes is least likely to be shifted? A) a state excise tax on the sellers of football tickets B) a personal income tax C) a general sales tax on retailers who sell foodstuffs and clothing D) a Federal excise tax on the producers of whiskey Answer: B

Tax system progressivity

Type: F Topic: 9 E: 593 MI: 349 113. Which of the following is correct? A) The Federal tax system is regressive, while the state and local tax system is progressive. B) Both the Federal tax system and the state and local tax system are regressive. C) The Federal tax system is progressive, while the state and local tax system is regressive. D) Both the Federal tax system and the state and local tax systems are progressive. Answer: C

Type: F Topic: 9 E: 594 MI: 350 114. Overall, the U.S. tax system (combined Federal, state, and local) is: A) highly progressive. B) slightly progressive. C) slightly regressive. Answer: B

D) highly regressive.

Type: F Topic: 9 E: 593 MI: 349 115. The top 5 percent of all taxpayers in the United States pay about: A) 11 percent of the total personal income taxes. C) 56 percent of the total personal income taxes. B) 25 percent of the total personal income taxes. D) 81 percent of the total personal income taxes. Answer: C

Type: F Topic: 9 E: 594 MI: 350 116. The U.S. tax-transfer system (as distinct from the tax system alone) is: A) progressive. B) proportional. C) bimodal. D) regressive. Answer: A

Type: F Topic: 9 E: 593 MI: 349 117. In 1999 the 20 percent of families with the lowest incomes paid an average Federal tax rate (on all Federal taxes) of about ____ percent, whereas the 20 percent of families with the highest incomes paid an average tax rate of about ____ percent. A) 5; 50. B) 10; 30. C) 8; 60. D) 5; 30. Answer: D

McConnell/Brue: Economics, 16/e

Page 974

Chapter 31: Public Choice Theory and Taxation

Last Word Questions

Type: A E: 594 MI: 350 Status: New 118. (Last Word) In 2001 Congress appropriated $400,000 for a parking lot in Talkeena, Alaska (population, 300). This is an example of: A) the paradox of voting. C) the benefits-received principle. B) pork barrel politics. D) the fallacy of limited decisions. Answer: B

Type: A E: 594 MI: 350 119. (Last Word) As part of a radiation cleanup project, the EPA spent an average of $651,700 to custom-build 10 new houses for families. The old houses were worth an average of $147,000 each. This best demonstrates the outcome of: A) the benefits-received principle. C) bureaucracy inefficiency. B) logrolling. D) the problem of limited and bundled choices. Answer: C

Type: A E: 594 MI: 350 Status: New 120. (Last Word) A section of the 2003 Appropriations bill contained a provision to qualify catfish farmers for livestock compensation payments for disaster relief. This is an example of the: A) special-interest effect. C) paradox of voting. B) benefits-received principle. D) principal-agent problem. Answer: A

True/False Questions

Type: A E: 582 MI: 338 121. Logrolling can either increase or diminish economic efficiency. Answer: True

Type: D E: 582 MI: 338 122. The paradox of voting is that under majority voting rules the median-voter decides the election outcome. Answer: False

Type: D E: 584 MI: 340 123. The pursuit through government of a transfer of wealth at society's expense is called "rent seeking." Answer: True

Type: A E: 587 MI: 343 124. The benefits-received principle of taxation is used to support corporate and personal income taxes. Answer: False

McConnell/Brue: Economics, 16/e

Page 975

Chapter 31: Public Choice Theory and Taxation

Type: A E: 587 MI: 343 125. The benefits-received principle of taxation supports the case for highly progressive taxation. Answer: False

Type: A E: 588 MI: 344 126. Sales taxes are proportional in relation to income because the same tax rate applies regardless of the size of a purchase. Answer: False

Type: A E: 588 MI: 344 127. Sales taxes on consumer goods are regressive because poor people consume a larger proportion of their incomes than do rich people. Answer: True

Type: A E: 588 MI: 344 128. If you pay a $1000 tax on $10,000 of taxable income and a $3000 tax on a taxable income of $16,000, the tax is progressive. Answer: True

Type: A E: 588 MI: 344 129. A highly progressive tax takes relatively more from the rich than it does from the poor. Answer: True

Type: A E: 590 MI: 346 130. Given supply, the more inelastic the demand for a product, the larger the portion of an excise tax that is shifted forward to consumers. Answer: True

Type: A E: 590 MI: 346 131. Given demand, the more inelastic the supply of a product, the larger the portion of an excise tax that is borne by producers. Answer: True

Type: D E: 591 MI: 347 132. The efficiency loss of a tax is the tax revenue collected by government minus the value of the public goods financed through the tax. Answer: False

Type: A E: 591 MI: 347 133. The greater the elasticity of demand and supply, the greater is the efficiency loss of a tax. Answer: True

McConnell/Brue: Economics, 16/e

Page 976

Chapter 31: Public Choice Theory and Taxation

Type: F E: 592 MI: 348 134. Economists agree that corporations always shift the corporate income tax to consumers by raising product prices. Answer: False

Type: F E: 593 MI: 349 135. Although state and local taxes are highly progressive, Federal taxation is predominantly regressive. Answer: False

Type: A E: 582 MI: 338 136. Majority voting assures that government will provide a public good if it yields total benefits in excess of total costs. Answer: False

McConnell/Brue: Economics, 16/e

Page 977

CHAPTER 32

Antitrust Policy and Regulation

Topic 1. 2. 3. 4. 5. 6. Antitrust laws and enforcement Antitrust cases Types of mergers Antitrust interpretations and issues Natural monopoly and industrial regulation Social regulation Consider This Last Word True-False

Question numbers 1-34 35-48 49-57 58-78 79-97 98-105 106-107 108-112 113-126

____________________________________________________________

_______________________________________

____________________________________________________________

_______________________________________

Multiple Choice Questions Antitrust laws and enforcement

Type: F Topic: 1 E: 599 MI: 355 1. All of the following can file antitrust charges under the Sherman Act except: A) the U.S. Justice Department. C) injured private parties. B) state attorney generals. D) the Federal Energy Regulatory Commission. Answer: D

Type: A Topic: 1 E: 599 MI: 355 2. Movie producers A, B, and C secretly meet and agree to release their summer blockbuster films in sequence, rather than at the same time. The U.S. Justice Department learns of the agreement and files an antitrust suit. The Federal government would most likely file charges under the: A) Sherman Act, Section 1. C) Clayton Act B) Sherman Act, Section 2. D) Federal Trade Commission Act. Answer: A

Type: F Topic: 1 E: 599 MI: 355 Status: New 3. Which of the following is least likely to violate the Sherman Act or the Clayton Act? A) Competitive firms A, B, and C meet and agree to charge a common price. B) Competitive firms D and E, each with 35 percent market shares, merge into a single firm. C) Competitive firms F and G independently charge lower prices to frequent customers than to occasional customers. D) Large dominant firm H forces buyers to purchase its product X in order to buy its popular product Y. Answer: C

Chapter 32: Antitrust Policy and Regulation

Type: F Topic: 1 E: 599 MI: 355 4. The Sherman Act was designed to: A) exempt commercial banks from the antitrust laws. B) make interlocking directorates legal. C) prohibit misleading and antisocial advertising. D) make monopoly and acts that restrain trade illegal. Answer: D

Type: F Topic: 1 E: 599 MI: 355 5. Which one of the following acts declared "Every contract, combination ... or conspiracy, in restraint of trade or commerce among the several states ... to be illegal"? A) the Wheeler-Lea Act C) the Sherman Act B) the Federal Trade Commission Act D) the Interstate Commerce Act Answer: C

Type: F Topic: 1 E: 599 MI: 355 6. The Clayton Act of 1914: A) outlawed price discrimination, tying contracts, intercorporate stockholding, and interlocking directorates that lessen competition. B) prohibited unfair or deceptive acts or practices in commerce that tend to reduce competition. C) outlawed vertical and conglomerate mergers. D) prohibited one firm from acquiring the assets of another when the effect was to limit competition. Answer: A

Type: F Topic: 1 E: 600 MI: 356 7. A function of the Federal Trade Commission is to: A) investigate instances of faulty and misleading advertising. B) establish railway rates for interstate railroads. C) ban or recall unsafe consumer products. D) prevent insider trading in securities markets. Answer: A

Type: F Topic: 1 E: 600 MI: 356 8. The function of investigating instances of fraudulent advertising has been assigned to the: A) Joint Economic Committee of Congress. C) Bureau of Standards. B) Federal Trade Commission. D) Department of Justice. Answer: B

Type: A Topic: 1 E: 600 MI: 356 9. Which one of the following is not prohibited by the original Clayton Act? A) the purchase of the stocks of rival firms that lessens competition B) the purchase of the assets of rival firms that lessens competition C) an exclusive dealer or tying agreements that lessen competition D) price discrimination that lessens competition Answer: B

McConnell/Brue: Economics, 16/e

Page 980

Chapter 32: Antitrust Policy and Regulation

Type: A Topic: 1 E: 599 MI: 355 10. Which of the following is directly illegal under the Sherman Act? A) price discrimination. B) tying contracts. C) price fixing. D) interlocking directorates. Answer: C

Type: F Topic: 1 E: 600 MI: 356 11. Tying contracts are illegal under the: A) Wagner Act of 1935. B) Clayton Act of 1914. Answer: B

C) FTC Act of 1914. D) Celler-Kefauver Act of 1950.

Type: A Topic: 1 E: 599 MI: 355 12. Suppose Slow Ketchup requires that, as a condition of purchase, all restaurants using its product must buy and make available its new sales product. This arrangement is an example of: A) price fixing. B) an interlocking directive. C) a tying contract. D) price discrimination. Answer: C

Type: D Topic: 1 E: 599 MI: 355 13. Required payments to parties injured by firms found to have violated the Sherman Act are known as: A) antitrust fines. B) treble damages. C) economic reparations. D) loss carry-forwards. Answer: B

Type: F Topic: 1 E: 600 MI: 356 14. The government was successful in gaining an antitrust conviction in the: A) U.S. Steel case. B) IBM case. C) Alcoa case. D) DuPont cellophane case. Answer: C

Type: F Topic: 1 E: 599 MI: 355 15. Which of the following prohibited mergers by stock acquisition if the effect was to lessen competition? A) Celler-Kefauver Act of 1950 C) Clayton Act of 1914 B) Wheeler-Lea Act of 1938 D) Sherman Act of 1890 Answer: C

Type: F Topic: 1 E: 600 MI: 356 16. Which of the following gave the Federal Trade Commission responsibility to protect the public against false and misleading advertising? A) Celler-Kefauver Act of 1950 C) Clayton Act of 1914 B) Wheeler-Lea Act of 1938 D) Sherman Act of 1890 Answer: B

Type: F Topic: 1 E: 600 MI: 356 17. Which of the following amended the Clayton Act's prohibition against mergers that substantially lessen competition? A) Celler-Kefauver Act of 1950 C) Clayton Act of 1914 B) Wheeler-Lea Act of 1938 D) Sherman Act of 1890 Answer: A

McConnell/Brue: Economics, 16/e

Page 981

Chapter 32: Antitrust Policy and Regulation

Type: F Topic: 1 E: 599 MI: 355 18. Which of the following made monopoly and restraints of trade criminal offenses against the Federal government? A) Celler-Kefauver Act of 1950 C) Clayton Act of 1914 B) Wheeler-Lea Act of 1938 D) Sherman Act of 1890 Answer: D

Type: D Topic: 1 E: 599 MI: 355 19. Tying agreements: A) establish common boards of directors for previously competing firms. B) obligate a purchaser of product X to also buy product Y from the same seller. C) allow manufacturers to specify the retail prices of their products. D) prohibit firms from selling their products outside of specified geographic areas. Answer: B

Type: F Topic: 1 E: 600 MI: 356 20. False advertising and the misrepresentation of products were made illegal by the: A) Federal Trade Commission Act. B) Wheeler-Lea Act. C) Sherman Act. D) Clayton Act. Answer: B

Type: A Topic: 1 E: 599 MI: 355 21. Responsibility for enforcing the antitrust laws rests: A) with the Interstate Commerce Commission. B) with both the Department of Justice and the Federal Trade Commission. C) solely with the Federal Trade Commission. D) solely with the Department of Justice. Answer: B

Type: F Topic: 1 E: 600 MI: 356 22. The Celler-Kefauver Act of 1950: A) outlawed price fixing. B) amended the Sherman Act. Answer: C

C) amended the Clayton Act. D) created the Civil Aeronautics Board (CAB).

Type: F Topic: 1 E: 600 MI: 356 23. The Celler-Kefauver Act of 1950: A) modified patent legislation by reducing the number of years over which a patent is applicable. B) prohibited any firm from acquiring the real assets of another firm where the effect was to lessen competition. C) declared all conglomerate mergers to be illegal. D) prohibited any firm from buying the stock of another firm where the effect was to lessen competition. Answer: B

Type: A Topic: 1 E: 599 MI: 355 24. Tying contracts which lessen competition are specifically illegal under provisions of the: A) Sherman Act. B) Federal Trade Commission Act. C) Celler-Kefauver Act. D) Clayton Act. Answer: D

McConnell/Brue: Economics, 16/e

Page 982

Chapter 32: Antitrust Policy and Regulation

Type: A Topic: 1 E: 599 MI: 355 25. Suppose the courts declare that XYZ Corporation violated the antitrust laws and as a result the ABC Corporation lost $100 million of profits. XYZ Corporation will have to pay ABC Corporation a monetary award of: A) $100 million. B) $33.3 million. C) $150 million. D) $300 million. Answer: D

Type: A Topic: 1 E: 603 MI: 359 26. Price fixing: A) is prohibited by Section 7 of the Clayton Act. B) is a per se violation of the antitrust laws. C) may be either legal or illegal depending on whether or not it produces above-normal profits. D) is illegal under terms of the Federal Trade Commission Act. Answer: B

Type: D Topic: 1 E: 599 MI: 355 27. Tying agreements: A) are legal in the United States, even though they may reduce competition. B) occur when the purchase of one product obligates the buyer to purchase other products from the same seller. C) refer to any long-term contract between a buyer and a seller. D) were outlawed by the Celler-Kefauver Act. Answer: B

Type: A Topic: 1 E: 599 MI: 355 28. The antitrust laws are based on the: A) creative destruction view of competition. B) idea that competition leads to greater economic efficiency than does monopoly. C) view that nonprice competition should be strictly regulated by government. D) view that all negative spillovers should be eliminated by government action. Answer: B

Type: F Topic: 1 E: 599 MI: 355 29. The Sherman Act: A) was declared unconstitutional in 1895. B) provided for government regulation of the railroads. C) declared monopoly and restraints of trade to be illegal. D) exempted the railroad and communications industries from the antitrust laws. Answer: C

Type: A Topic: 1 E: 598 MI: 354 30. The main purpose of the antitrust laws is: A) to encourage firms to produce where P > MC. B) the elimination of both negative and positive externalities. C) to prevent the monopolization of industries. D) to regulate natural monopolies. Answer: C

McConnell/Brue: Economics, 16/e

Page 983

Chapter 32: Antitrust Policy and Regulation

Type: F Topic: 1 E: 599 MI: 355 31. The antitrust laws are enforced by the: A) Federal Bureau of Investigation. B) Antimonopoly Court of Appeals. C) Federal Justice Department and the Federal Trade Commission. D) Department of Commerce. Answer: C

Type: F Topic: 1 E: 599-600 MI: 355-356 32. Which of the following is not an antitrust law? A) the Sherman Act B) the Clayton Act C) the Celler-Kefauver Act Answer: D

D) the Wagner Act

Type: F Topic: 1 E: 599 MI: 355 33. Interlocking directorates are: A) legal if the two firms have small market shares. B) illegal under provisions of the Federal Trade Commission Act of 1914. C) illegal under provisions of the Celler-Kefauver Act of 1950. D) illegal under provisions of the Clayton Act of 1914. Answer: D

Type: F Topic: 1 E: 599 MI: 355 34. The Federal Trade Commission: A) is empowered to file antitrust suits. B) prohibits interlocking directorates in interstate industries. C) regulates airline fares. D) regulates such transportation industries as railroads and trucking. Answer: A

Antitrust cases

Type: F Topic: 2 E: 600 MI: 356 Status: New 35. In which of the following cases did the final court decision result in a breakup of the firm into competing businesses? A) Microsoft case B) Standard Oil case C) U.S. Steel case D) DuPont cellophane case Answer: B

Type: F Topic: 2 E: 601 MI: 357 Status: New 36. In which of the following cases was the firm found not guilty of violating the Sherman Act? A) Standard Oil case B) Microsoft case C) Alcoa case D) DuPont cellophane case Answer: D

McConnell/Brue: Economics, 16/e

Page 984

Chapter 32: Antitrust Policy and Regulation

Type: F Topic: 2 E: 602 MI: 358 37. In the Microsoft antitrust case, the Federal government said in essence that: A) the mere presence of monopoly violated the Sherman Act, irrespective of Microsoft's behavior. B) Microsoft was a "bad monopoly." C) Microsoft was generally a "good monopoly," but that its tying contracts involving Internet Explorer violated the Clayton Act. D) the case was similar to the U.S. Steel case of 1920. Answer: B

Type: F Topic: 2 E: 602 MI: 358 38. Which of the following antitrust cases is most recent? A) the U.S. Steel case B) the Alcoa case C) the AT&T case D) the Microsoft case Answer: D

Type: F Topic: 2 E: 602 MI: 358 39. In which of the following sets of antitrust cases did the government gain convictions? A) the U.S. Steel case and the Microsoft case (subject to appeal) B) the Alcoa case and the Microsoft case (subject to appeal) C) the DuPont cellophane case and the AT&T case D) the U.S. Steel case and the Alcoa case Answer: B

Type: F Topic: 2 E: 601 MI: 357 40. In which of the following pairs of antitrust cases did the firms prevail against the antitrust charges leveled against them? A) the Alcoa case and the Microsoft case (subject to appeal) B) the U.S. Steel case and the Alcoa case C) the DuPont cellophane case and the U.S. Steel Case D) the U.S. Steel case and the Microsoft Case (subject to appeal) Answer: C

Type: D Topic: 2 E: 600 MI: 356 41. The rule of reason indicated that: A) if less than four firms account for three-fourths of an industry's sales, the industry is in violation of the Sherman Act. B) social regulation should not be enforced unreasonably so that costs exceed benefits. C) the mere possession of monopoly power is a violation of the antitrust laws. D) only contracts and combinations that unreasonably restrain trade violate the antitrust laws. Answer: D

Type: F Topic: 2 E: 600 MI: 356 42. In the Alcoa case of 1945 the courts held that: A) the mere possession of monopoly power is a violation of the antitrust laws. B) only contracts and combinations that unreasonably restrain trade are in violation of the Sherman Act. C) retail and wholesale firms are exempt from antitrust legislation. D) firms which sell more than one-half of their output overseas are exempt from antitrust. Answer: A

McConnell/Brue: Economics, 16/e

Page 985

Chapter 32: Antitrust Policy and Regulation

Type: F Topic: 2 E: 601 MI: 357 43. The basic issue in the DuPont cellophane case was: A) whether trade crossed state lines. B) defining the relevant market. Answer: B

C) structure versus behavior. D) the rule of reason.

Type: A Topic: 2 E: 600 MI: 356 44. Suppose a court rules that the ABC Corporation is in violation of the antitrust laws because it produces 70 percent of the output of its industry. This decision is consistent with the: A) U.S. Steel case. C) behavioralist approach to antitrust. B) Alcoa case. D) legal cartel theory of regulation. Answer: B

Type: F Topic: 2 E: 600 MI: 356 45. The Alcoa case: A) supported the structuralist approach to antitrust. B) struck down the treble damages provision of the antitrust laws. C) called for Federal regulation of any industry with a four-firm concentration ratio in excess of 50 percent. D) outlawed all conglomerate mergers. Answer: A

Type: F Topic: 2 E: 600 MI: 356 46. In the U.S. Steel case of 1920 the courts held that: A) the structure of an industry is more important than its behavior in determining violations of the antitrust laws. B) any firm which faces substantial import competition is exempt from the antitrust laws. C) although U.S. Steel possessed monopoly power, it had not violated the Sherman Act because it had not unreasonably used that power. D) the fact that U.S. Steel possessed monopoly power was a violation of the Sherman Act. Answer: C

Type: F Topic: 2 E: 602 MI: 358 47. Restructuring of a major industry resulted from the: A) U.S. Steel case. B) AT&T case. C) IBM case. Answer: B

D) DuPont cellophane case.

Type: F Topic: 2 E: 600 MI: 356 48. In the U.S. Steel case, the court ruled that: A) even though a firm's behavior might be legal, the mere possession of monopoly power was in violation of the Sherman Act. B) only monopolies that unreasonably restrain trade are subject to antitrust action under the Sherman Act. C) when made by dominant firms, tying contracts are illegal, per se. D) the company violated the Clayton Act and therefore should be dissolved into several competing firms. Answer: B

McConnell/Brue: Economics, 16/e

Page 986

Chapter 32: Antitrust Policy and Regulation

Types of mergers

Type: A Topic: 3 E: 603 MI: 359 Status: New 49. Which of the following is most likely to increase the 4-firm concentration ratio of a particular industry? A) a horizontal merger between two of the industry's largest firms. B) a vertical merger between one of an industry's largest firms and one of the many input suppliers in the resource market. C) a conglomerate merger involving one of the industry's major firms. D) an agreement by all the industry firms to divide up the market among them. Answer: A

Type: A Topic: 3 E: 603 MI: 359 Status: New 50. The U.S. Justice Department would most likely block a merger between: A) Walt Disney and Coca Cola. C) Wal-Mart and Subway Sandwiches B) Sony and Home Depot. D) Boeing and Airbus. Answer: D

Type: C Topic: 3 E: 603 MI: 359 51. Which of the following findings would be the most likely to lead the U.S. Justice Department to block a corporate merger under terms of the Clayton Act? A) a buyer-seller relationship between the two firms B) a high pre-merger Herfindahl index in the industry and a large boost in the index because of the merger C) a low pre- and post-merger concentration ratio in the industry D) evidence that one of the firms is highly unprofitable. Answer: B

Type: A Topic: 3 E: 603 MI: 359 52. A merger between an automobile manufacturer and a maker of automobile tires is an example of a: A) conglomerate merger. B) horizontal merger. C) vertical merger. D) tying contract. Answer: C

Type: A Topic: 3 E: 603 MI: 359 53. Which one of the following is most likely to increase the Herfindahl index of a particular industry? A) a conglomerate merger B) a vertical merger C) a price fixing arrangement among all the industry firms D) a horizontal merger Answer: D

Type: A Topic: 3 E: 603 MI: 359 54. A merger between a maker of household detergents and a fast food chain would be an example of: A) a horizontal merger. C) a conglomerate merger. B) an interlocking directorate. D) a tying contract. Answer: C

McConnell/Brue: Economics, 16/e

Page 987

Chapter 32: Antitrust Policy and Regulation

Type: D Topic: 3 E: 603 MI: 359 55. A conglomerate merger: A) can extend the line of products sold, extend the territories in which products are sold, or combine totally unrelated products. B) is defined as a merger involving two firms that previously had a buyer-seller relationship. C) is defined as a merger involving two firms producing the same or similar products and selling them in the same geographical market. D) is illegal, per se. Answer: A

Type: D Topic: 3 E: 603 MI: 359 56. A vertical merger involves a combining of one or more firms: A) as the result of one firm purchasing the assets of the other. B) that are operating in entirely different industries. C) operating at different stages of the production process in a particular industry. D) operating at the same stage of the production process. Answer: C

Type: D Topic: 3 E: 603 MI: 359 57. A merger of several firms operating in different industries--for example, a trucking company, a fast-food chain, and a brokerage house--is called: A) an integrated merger. B) a conglomerate merger. C) a vertical merger. D) a horizontal merger. Answer: B

Antitrust interpretations and issues

Type: D Topic: 4 E: 601 MI: 357 58. Structuralists take the position that: A) the rule of reason is appropriate and desirable in interpreting the Sherman Act. B) only unreasonable anticompetitive acts should be regarded as violations of the antitrust laws. C) industries should be judged on the basis of their technological progress and their price-output behavior. D) an industry that has a monopolistic structure will behave monopolistically. Answer: D

Type: D Topic: 4 E: 601 MI: 357 59. Behavioralists believe that: A) if four or fewer firms control more than half of the market for a product, then the Sherman Act is being violated. B) industries should be judged on the basis of their price-output behavior and their technological progressiveness. C) there is no evidence that any monopolistic industries has abused its market power. D) all concentrations of economic power are socially undesirable. Answer: B

McConnell/Brue: Economics, 16/e

Page 988

Chapter 32: Antitrust Policy and Regulation

Type: A Topic: 4 E: 601 MI: 357 60. A firm charged with monopolizing a market is less likely to be convicted if: A) the court accepts a broad definition of the market. B) the court accepts a narrow definition of the market. C) it has gained its monopoly through abusive means. D) it sells its product to other firms, rather than directly to consumers. Answer: A

Type: A Topic: 4 E: 603 MI: 359 61. Price fixing is considered to be a per se violation of the antitrust laws because: A) a guilty verdict requires proof of injury to consumers. B) a guilty verdict requires proof of injury to other competitors. C) the rule of reason is applicable. D) a guilty verdict requires proof that the activity was attempted, not that it unreasonably restrained trade. Answer: D

62.

Type: A Topic: 4 E: 603 MI: 359 Conspiracies to fix prices are: A) illegal under the Clayton Act. B) illegal under the Celler-Kefauver Act. C) per se violations of the antitrust laws. D) more tolerated by government today than two or three decades ago. Answer: C

Type: F Topic: 4 E: 603 MI: 359 63. Which of the following is correct? A) Vertical mergers are more likely to be acceptable under antitrust laws than are horizontal mergers. B) A vertical merger entails the merging of two or more competing firms. C) Horizontal mergers are more likely to be acceptable under antitrust laws than are vertical mergers. D) Conglomerate mergers occur when two or more firms at various stages in a good's production are combined. Answer: A

Type: F Topic: 4 E: 599 MI: 355 64. Which one of the following is not correct? A) In antitrust cases defendants attempt to define the relevant market broadly. B) The courts have varied over time in their interpretations of the antitrust statutes. C) Antitrust suits can only be originated by the Federal Trade Commission. D) In antitrust cases the prosecution attempts to define the relevant market narrowly. Answer: C

Type: A Topic: 4 E: 603 MI: 359 65. Antitrust authorities are least likely to take action against: A) conglomerate mergers. B) horizontal mergers. C) interlocking directorates. Answer: A

D) price fixing.

McConnell/Brue: Economics, 16/e

Page 989

Chapter 32: Antitrust Policy and Regulation

Type: A Topic: 4 E: 601 MI: 357 66. Patent laws are controversial because: A) they are in legal conflict with the provisions of the Federal Trade Commission Act. B) they are at the core of the problem of whether monopoly is based on industry structure or behavior. C) while patents encourage innovation, they are also a source of monopoly power. D) any firm whose monopoly power is based on patents is automatically exempt from the antitrust laws. Answer: C

Use the following to answer questions 67-71: Answer the next question(s) on the basis of the following table showing market shares of firms in hypothetical industries. Assume these are distinct industries with no buyer-seller relationships or competition among them.
Industry Alpha Beta Cappa Delta 1 30 80 25 20 Market share of firms in industry 2 3 4 5 6 30 20 20 --10 5 3 1 1 25 25 25 --20 20 20 10 10

Type: T Topic: 4 E: 603 MI: 359 67. Refer to the above table. The Herfindahl index for Cappa is: A) 2500. B) 100. C) 100,000. D) 5000. Answer: A

Type: T Topic: 4 E: 603 MI: 359 68. Refer to the above table. The industry with the greatest market power as measured by the Herfindahl index is: A) Alpha. B) Beta. C) Cappa. D) Delta. Answer: B

Type: T Topic: 4 E: 603 MI: 359 69. Refer to the above table. A merger between Firm 2 and Firm 3 in Alpha would be a: A) vertical merger. B) horizontal merger. C) diagonal merger. D) conglomerate merger. Answer: B

Type: T Topic: 4 E: 603 MI: 359 70. Refer to the above table. A merger between Firm 1 in Alpha and Firm 2 in Delta would be a: A) vertical merger. B) horizontal merger. C) conglomerate merger. D) diagonal merger. Answer: C

Type: T Topic: 4 E: 603 MI: 359 71. Refer to the above table. The government would likely challenge a merger between: A) Firm 1 in Alpha and Firm 6 in Delta. C) Firms 1 and 2 in Cappa. B) Firms 3 and 4 in Beta. D) Firm 4 in Alpha and Firm 3 in Cappa. Answer: C

McConnell/Brue: Economics, 16/e

Page 990

Chapter 32: Antitrust Policy and Regulation

Type: A Topic: 4 E: 603 MI: 359 Status: New 72. Suppose the firms in a five-firm industry have market shares of 30, 30, 20, 10, and 10 percent, respectively. The Herfindahl index for the industry is: A) 1900. B) 2400. C) 90. D) 10,000. Answer: B

Type: A Topic: 4 E: 603 MI: 359 Status: New 73. Suppose the firms in a five-firm industry have market shares of 60, 20, 10, 5, and 5 percent, respectively. The Herfindahl index for the industry is: A) 3600. B) 9025. C) 95. D) 4150. Answer: D

Type: A Topic: 4 E: 603 MI: 359 Status: New 74. Suppose that two firms in an industry with a Herfindahl index of 5000 announce a merger. The U.S. Justice Department concludes the merger will boost the index to 5500. The antitrust authorities will most likely: A) ignore this merger because of the relatively small increase in the Herfindahl index. B) allow the merger but watch the new firm carefully for future violations of the antitrust laws. C) allow the merger if foreign entry to the industry is possible. D) prevent the merger, contending that it violates the Clayton Act. Answer: D

Type: A Topic: 4 E: 603 MI: 359 Status: New 75. Suppose that two firms in an industry that has a Herfindahl index of 1000 announce a merger. The U.S. Justice Department concludes the merger will boost the index to 1050. The antitrust authorities will most likely: A) ignore this merger because of the relatively small size of, and increase in, the Herfindahl index. B) prevent the merger, contending that it violates the Clayton Act. C) allow the merger if foreign entry to the industry is possible. D) allow the merger but watch the new firm carefully for future violations of the antitrust laws. Answer: A

Type: D Topic: 4 E: 601 MI: 357 Status: New 76. The view that the antitrust laws should be enforced relatively leniently because of the tendency for monopoly power to erode over time is known as the: A) structuralist view of antitrust. C) laissez-faire perspective on antitrust. B) behavioralist view of antitrust. D) active antitrust perspective. Answer: C

Type: D Topic: 4 E: 601 MI: 357 Status: New 77. The view that the antitrust laws need to be strongly enforced to prevent illegal business behaviors, monopolization of markets, and allocative inefficiency is known as the: A) structuralist view of antitrust. C) laissez-faire perspective on antitrust. B) behavioralist view of antitrust. D) active antitrust perspective. Answer: D

McConnell/Brue: Economics, 16/e

Page 991

Chapter 32: Antitrust Policy and Regulation

Type: D Topic: 4 E: 601 MI: 357 Status: New 78. Economists who adhere to the laissez-faire antitrust perspective: A) view competition as a long-run dynamic process in which firms battle for dominance of markets but rarely can sustain such dominance once it is achieved. B) believe the antitrust laws are as important today as they were when they were passed in the early 1900s. C) say that an industry's structure, which is based on economies of scale, usually predicts the behavior of the industry firms. D) contend that large, dominant firms should be broken into smaller competitive firms and then government should stand back and let competition prevail. Answer: A

Natural monopoly and industrial regulation

Type: D Topic: 5 E: 604 MI: 360 79. A firm is likely to be a natural monopoly: A) when the demand for its product or service is inelastic. B) if it is producing an inferior good. C) if economies of scale are experienced over the full range of output. D) because government grants it an exclusive franchise. Answer: C

Type: A Topic: 5 E: 605 MI: 361 80. Which of the following is characteristic of a regulated natural monopoly? A) extensive economies of scale B) the wasteful duplication of capital facilities in the event of competition C) the provision of an essential service D) all of the above Answer: D

Type: A Topic: 5 E: 606 MI: 362 81. Suppose the transportation industry has been regulated for many years. Government now proposes to deregulate the industry, only to find that firms in the industry oppose this action. This is consistent with the: A) public interest theory of regulation. C) legal cartel theory of regulation. B) theory of natural monopolies. D) Alcoa and U.S. Steel court decisions. Answer: C

Type: A Topic: 5 E: 605 MI: 361 82. Using antitrust law to split up an unregulated natural monopoly into several competing firms: A) would reduce product price. B) would increase product price. C) might either increase product price or reduce product price. D) will reduce average total cost. Answer: C

McConnell/Brue: Economics, 16/e

Page 992

Chapter 32: Antitrust Policy and Regulation

Type: A Topic: 5 E: 605 MI: 361 83. A major criticism of industrial regulation is that: A) it has been applied to virtually all major U.S. corporations in the post-Second World War period. B) marginal cost pricing has created an underallocation of resources. C) by allowing a fair return price, it gives natural monopolists little incentive to contain costs. D) regulatory commissions have frequently caused natural monopolies to go bankrupt. Answer: C

Type: A Topic: 5 E: 606 MI: 362 84. Critics of the regulation of natural monopolies contend that: A) regulation increases the incentive of firms to lower costs. B) regulated firms may use creative accounting to reduce costs, prices, and profits. C) when rates of return are based on the value of real capital, an uneconomic substitution of labor for capital may occur. D) the industry may "capture" or control the regulatory commission. Answer: D

Type: D Topic: 5 E: 604-605 MI: 360-361 85. A market in which the entire demand for a good or service can be satisfied at the least cost by a single firm is a: A) horizontal market. B) natural monopoly. C) contestable market. D) perfect market. Answer: B

Type: D Topic: 5 E: 606 MI: 362 86. The legal cartel theory of regulation argues that: A) regulation encourages firms to inflate their production costs. B) firms in certain industries want to be regulated rather than face the rigors of competition. C) social regulation has been carried beyond the point at which marginal benefits and marginal costs are equal. D) the government is the logical agency to protect consumers from natural monopolies. Answer: B

Type: A Topic: 5 E: 606 MI: 362 87. The legal cartel theory best describes the structure and economic outcomes of the: A) automobile industry. C) airline industry before deregulation. B) deregulated airline industry. D) local telephone communications industry. Answer: C

Type: D Topic: 5 E: 605 MI: 361 88. The public interest theory of industrial regulation contends that: A) while industrial regulation is sound in theory, bureaucrats allow monopolists to obtain excessive profits. B) regulated monopolies are tantamount to legal cartels. C) the objective of regulation is to protect the public from the market power inherent in natural monopolies. D) firms in some industries want to be regulated. Answer: C

McConnell/Brue: Economics, 16/e

Page 993

Chapter 32: Antitrust Policy and Regulation

Type: F Topic: 5 E: 607 MI: 363 89. Overall, deregulation of industries formerly subjected to industrial regulation: A) has been a clear failure. B) is contributing an estimated $3 billion annually to society's well-being. C) is contributing an estimated $50 billion annually to society's well-being. D) has produced sizable efficiency gains in the communications industry, but not in the transportation industry (railways, trucking, airlines). Answer: C

Type: F Topic: 5 E: 607 MI: 363 90. The largest efficiency gains from deregulation have occurred in the: A) natural gas and cable television industries. C) communications and stockbrokering industries. B) cable television and railroad industries. D) airlines, trucking, and railroad industries. Answer: D

Type: A Topic: 5 E: 607-608 MI: 363-364 91. Which one of the following is concerned with industrial regulation, as distinct from social regulation? A) Occupational Safety and Health Administration C) Federal Communications Commission B) Consumer Products Safety Commission D) Environmental Protection Agency Answer: C

Type: F Topic: 5 E: 608 MI: 364 92. All of the following are regulatory commissions dealing with industrial regulation (as distinct from social regulation) except the: A) Food and Drug Administration. C) Federal Communications Commission. B) Federal Energy Regulatory Commission. D) 50 state public utility commissions. Answer: A

Type: F Topic: 5 E: 607 MI: 363 93. The latest industry to begin the deregulation process is the: A) trucking industry. B) airlines industry. C) electricity industry. D) railroad industry. Answer: C

Type: F Topic: 5 E: 605 MI: 361 94. Where there is natural monopoly, government is most likely to implement: A) social regulation. C) industrial regulation. B) antitrust policy. D) an externality containment policy. Answer: C

Type: F Topic: 5 E: 605 MI: 361 95. The main purpose of industrial regulation is to: A) lower price to marginal cost. B) lower price to average total cost such that the firm earns a fair return. C) break monopolies into competing firms. D) reduce X-inefficiency. Answer: B

McConnell/Brue: Economics, 16/e

Page 994

Chapter 32: Antitrust Policy and Regulation

Type: F Topic: 5 E: 605 MI: 361 96. Critics of industrial regulation say that such regulation: A) contributes to X-inefficiency. B) benefits small firms at the expense of large firms. C) creates insurmountable principal-agent problems. D) suffers from the free-rider problem. Answer: A

Type: F Topic: 5 E: 605 MI: 361 97. Critics of industrial regulation say that such regulation: A) benefits small firms at the expense of large firms. B) perpetuates monopoly long after new technology has eroded natural monopoly. C) creates insurmountable principal-agent problems. D) has resulted mainly from the paradox of voting. Answer: B

Social regulation

Type: F Topic: 6 E: 608 MI: 364 98. Social, as distinct from industrial, regulation is the major focus of the: A) Federal Trade Commission C) Federal Communications Commission. B) Federal Energy Regulatory Commission. D) Consumer Products Safety Commission. Answer: D

Type: A Topic: 6 E: 608 MI: 364 99. Defenders of social regulation point out that: A) social regulation is a better alternative than unregulated natural monopoly. B) critics who stress the high administrative and compliance costs of social regulation underestimate the social benefits that the regulations produce. C) the number of regulatory agencies has declined over the past two decades. D) social regulations reduce product prices. Answer: B

Type: A Topic: 6 E: 608 MI: 364 100. Social regulation differs from industrial regulation in that: A) social regulation applies to virtually all industries, while industrial regulation applies to a restricted number. B) industrial regulation is involved in the details of the production process, while social regulation is not. C) social regulation has expanded less rapidly in recent years than has industrial regulation. D) industrial regulation regulates products whereas social regulation regulates prices. Answer: A

Type: A Topic: 6 E: 608 MI: 364 101. Critics of social regulation argue that it: A) causes deflation. B) violates the due process clause of the U.S. constitution. C) is a relatively greater burden for small firms than for large firms. D) improves allocative efficiency. Answer: C

McConnell/Brue: Economics, 16/e

Page 995

Chapter 32: Antitrust Policy and Regulation

Type: A Topic: 6 E: 608 MI: 364 102. Which one of the following is concerned with social regulation? A) Equal Employment Opportunity Commission C) Sherman Commission B) Federal Communications Commission D) Federal Energy Regulatory Commission Answer: A

Type: A Topic: 6 E: 608 MI: 364 103. The optimal amount of social regulation occurs where the marginal benefit of such regulation: A) equals the marginal cost. B) exceeds the marginal cost by the greatest amount. C) is zero. D) is at its maximum. Answer: A

Type: F Topic: 6 E: 608-609 MI: 364-365 104. All of the following are regulatory commissions dealing with social regulation (as distinct from industrial regulation) except the: A) Food and Drug Administration. C) Equal Employment Opportunities Commission. B) Federal Energy Regulatory Commission. D) Consumer Products Safety Commission. Answer: B

Type: A Topic: 6 E: 608-609 MI: 364-365 105. Recently, some Congressional representative have called for extensive ergonomics regulations to reduce strains and injuries from repetitive activities by workers. Such regulation, if passed, would be a good example of: A) industrial regulation. C) the free-rider problem. B) the principal-agent problem. D) social regulation. Answer: D

Consider This Questions

Type: F E: 604 MI: 360 Status: New 106. (Consider This) The Consider This box "Of Catfish and Sneakers (and Other Things in Common)" lists examples of recent antitrust cases involving: A) monopolization. B) tying contracts. C) price fixing. D) horizontal mergers. Answer: C

Type: F E: 604 MI: 360 Status: New 107. (Consider This) According to the Consider This box on catfish and sneakers, which of the following firms were recently convicted of price fixing? A) Dell and Gateway (personal computer makers) C) Heinz and Del Monte (food product firms) B) Boeing and Airbus (aircraft manufacturers) D) Sotheby's and Christy's (art auction houses) Answer: D

McConnell/Brue: Economics, 16/e

Page 996

Chapter 32: Antitrust Policy and Regulation

Last Word Questions

Type: F E: 610 MI: 366 108. (Last Word) In 2001 Microsoft was found guilty of violating: A) Section 7 of the Clayton Act. C) the Federal Trade Commission Act. B) Sections 1 and 2 of the Sherman Act. D) Section 20 of the Wagner Act. Answer: B

Type: F E: 610 MI: 366 109. (Last Word) In 2001 Microsoft was found guilty of: A) using anticompetitive means to maintain and broaden its monopoly in Intel-compatible operating systems for personal computers. B) monopolizing the market for word processing software. C) conspiring with Netscape and Sun to monopolize the market for Internet browsers. D) deliberately pricing Windows 95 and 98 below marginal cost to monopolize the market for operating systems for personal computers. Answer: A

Type: F E: 610 MI: 366 110. (Last Word) In 2001 a U.S. court of appeals tossed out an earlier U.S. district court order that Microsoft: A) license Windows for sale by competitors. B) be split into two competing firms. C) divest itself of its Word and PowerPoint software programs. D) end its proposed acquisition of Sun Microsystems. Answer: B

Type: F E: 611 MI: 367 Status: New 111. (Last Word) The final settlement of the United States v. Microsoft case: A) broke up Microsoft into two competing firms. B) forced Microsoft to provide uniform royalty and licensing arrangements for Windows with all manufacturers of personal computers. C) forced Microsoft to sell off its major applications programs such as Word and PowerPoint. D) found Microsoft not guilty of violating the Sherman Act. Answer: B

Type: F E: 611 MI: 367 Status: New 112. (Last Word) The final settlement of the United States v. Microsoft case: A) requires that Microsoft provide technical information to competing companies so they can develop software programs that work as well with Windows as Microsoft's own products. B) broke up Microsoft into two competing firms. C) found Microsoft not guilty of violating the Sherman Act. D) forced Microsoft to sell off its major applications programs such as Word and PowerPoint. Answer: A

McConnell/Brue: Economics, 16/e

Page 997

Chapter 32: Antitrust Policy and Regulation

True/False Questions

Type: F E: 599 MI: 355 113. Price fixing is illegal under Section 1 of the Sherman Act. Answer: True

Type: F E: 599 MI: 355 114. Monopolization is illegal under Section 1 of the Sherman Act. Answer: False

Type: F E: 599 MI: 355 115. The U.S. Justice Department, the Federal Trade Commission, state attorneys general, and injured private parties can independently file charges against firms under the Sherman Act. Answer: True

Type: F E: 599 MI: 355 116. Anticompetitive mergers are illegal under provisions of the Clayton Act (as amended). Answer: True

Type: A E: 608-609 MI: 364-365 117. Proposed ergonomics regulations are an example of industrial regulation (rather than social regulation). Answer: False

Type: F E: 600 MI: 356 118. The Wheeler-Lea Act prohibits corporations from purchasing the stock of competing firms. Answer: False

Type: D E: 599 MI: 355 119. Tying agreements are contracts by which retailers agree to charge the prices that manufacturers set on branded goods. Answer: False

Type: F E: 599 MI: 355 120. The Celler-Kefauver Act outlawed interlocking directorates. Answer: False

Type: A E: 603 MI: 359 121. Horizontal mergers may be anticompetitive because nonmerged firms are foreclosed from selling their products to the buying firm. Answer: False

McConnell/Brue: Economics, 16/e

Page 998

Chapter 32: Antitrust Policy and Regulation

Type: A E: 605-606 MI: 361-362 122. The regulation of natural monopolies has been criticized because it creates a tendency for regulated firms to use too much labor and too little capital in the production process. Answer: False

Type: F E: 600 MI: 356 123. The Celler-Kefauver Act made vertical mergers legal, provided each firm does not have more than 30 percent of its relevant market. Answer: False

Type: D E: 605 MI: 361 124. The legal cartel theory indicates that, in any industry where market demand and the long-run average total cost curve intersect close to the latter's minimum, government regulation is mandatory and desirable. Answer: False

Type: F E: 608 MI: 364 125. The Consumer Product Safety Commission engages in social regulation, rather than industrial regulation. Answer: True

Type: A E: 605 MI: 361 126. The Americans with Disabilities Act of 1990 is an example of industrial regulation. Answer: False

McConnell/Brue: Economics, 16/e

Page 999

CHAPTER 33

Agriculture: Economics and Policy

Topic 1. 2. 3. 4. 5. 6. 7. 8. Demand for farm products Short-run farm problem Long-run farm problem Parity concept Effects of price supports Political aspects of farm policy Global aspects of farm policy Recent reform Last Word True-False

Question numbers 1-7 8-21 22-38 39-45 46-62 63-68 69-73 74-83 84-86 87-100

____________________________________________________________

_______________________________________

____________________________________________________________

_______________________________________

Multiple Choice Questions Demand for farm products

Type: F Topic: 1 E: 614 MI: 370 1. The demand for agricultural products is: A) relatively elastic with respect to price. B) relatively inelastic with respect to price. C) relatively elastic with respect to income. D) downward sloping to the individual farmer, but perfectly elastic to farmers as a group. Answer: B

Type: F Topic: 1 E: 614 MI: 370 2. The demand for agricultural products: A) has a price elasticity coefficient of about .20 to .25. B) is elastic with respect to income but inelastic with respect to price. C) has been decreasing about 8 percent per year. D) has been rising more rapidly than the national income. Answer: A

Chapter 33: Agriculture: Economics and Policy

Type: A Topic: 1 E: 615 MI: 371 3. Which of the following statements best describes the demand for agricultural commodities? A) It takes a small decline in price to induce a large increase in the amount of agricultural products demanded. B) The marginal utility of additional units of agricultural output diminishes very rapidly. C) Small increases in income cause demand to increase by a proportionately larger amount. D) When price declines, the resulting substitution effect is very large. Answer: B

Type: F Topic: 1 E: 614 MI: 370 4. The demand for most agricultural products: A) is such that as price falls total revenue will increase. B) is price inelastic. C) declines as population increases. D) is increasing faster in the United States than the supply of these products. Answer: B

Type: F Topic: 1 E: 614 MI: 370 5. The demand for most agricultural products is: A) elastic with respect to price, but inelastic with respect to income. B) inelastic with respect to price, but elastic with respect to income. C) elastic with respect to both price and income. D) inelastic with respect to both price and income. Answer: D

Type: A Topic: 1 E: 614 MI: 370 Status: New 6. The price elasticity coefficient of the demand for agricultural products is .2 to .25. This means that the demand for agricultural products is: A) price elastic. B) income elastic. C) income inelastic. D) price inelastic. Answer: D

Type: A Topic: 1 E: 614-615 MI: 370-371 Status: New 7. The demand for agricultural products rises less rapidly than income. This means that the demand for agricultural products is: A) income inelastic. B) income elastic. C) price inelastic. D) price elastic. Answer: A Short-run farm problem Type: C Topic: 2 E: 615-616 MI: 371-372 8. Which of the following best describes the short-run (as distinct from long-run) farm problem? A) New technology has increased the productivity of farmers and therefore resulted in declining farm prices and low farm incomes. B) The highly inelastic nature of agricultural demand, together with fluctuations in exports of farm goods, has caused small year-to-year fluctuations in farm output to result in highly unstable farm incomes. C) The supply of farm products has increased relative to the demand for them, and because demand is inelastic, prices of farm output and farm income have therefore declined. D) The demand for farm products has increased relative to their supply, but the elastic nature of agricultural demand has caused these shifts to result in declining farm incomes. Answer: B

McConnell/Brue: Economics, 16/e

Page 1002

Chapter 33: Agriculture: Economics and Policy

Type: F Topic: 2 E: 616 MI: 372 9. Since 1950, U.S. farm exports have: A) increased more rapidly than U.S. farm output. B) declined as a percentage of U.S. farm output. Answer: A

C) average about 10 percent of U.S. farm output. D) averaged about 50 percent of U.S. farm output.

Type: C Topic: 2 E: 617 MI: 373 10. Which of the following would, other things equal, reduce the demand for U.S. farm products? A) poorer crops abroad C) improved trade relations with China and Russia B) strong economic growth abroad D) appreciation of the U.S. dollar Answer: D

Type: A Topic: 2 E: 617 MI: 373 11. Which of the following would, other things equal, increase the demand for U.S. farm products? A) bumper crops abroad B) depreciation of the U.S. dollar C) deteriorating trade relations with China and Russia D) increases in foreign tariffs on imported farm products Answer: B

Use the following to answer questions 12-14:

Type: G Topic: 2 E: 616 MI: 372 12. Refer to the above diagram. If farmers produce a normal crop of Qn, their gross income: A) will be 0PpPNQn. B) will be 0PnNQn. C) will be Pp × Qn. D) cannot be determined. Answer: B

Type: G Topic: 2 E: 616 MI: 372 13. Refer to the above diagram. If farmers produce a bumper crop of Qb, their gross income: A) will be 0PpPBQb. B) will be 0PbBQb. C) will be Pb × Qn. D) cannot be determined. Answer: B

McConnell/Brue: Economics, 16/e

Page 1003

Chapter 33: Agriculture: Economics and Policy

Type: G Topic: 2 E: 616 MI: 372 14. Refer to the above diagram. If output changes from a poor crop, Qp, to a bumper crop, Qb: A) farm incomes will increase. C) price and quantity will both increase. B) farm incomes will decrease. D) farm incomes may either rise or fail. Answer: B

Type: A Topic: 2 E: 616 MI: 372 15. Small changes in the demand for agricultural products cause large changes in farm prices and incomes because: A) demand is relatively elastic with respect to price. B) demand is relatively inelastic with respect to price. C) agricultural supply is stable in the long run. D) the supply of agricultural products is perfectly elastic. Answer: B

Type: A Topic: 2 E: 615 MI: 371 16. If the demand for an agricultural product is inelastic, a bumper crop will: A) raise price and decrease total revenues. C) lower price and decrease total revenues. B) raise price and increase total revenues. D) lower price and increase total revenues. Answer: C

Type: A Topic: 2 E: 615 MI: 371 17. Which of the following statements is correct? A) The price support program hastened the exodus of resources from agriculture. B) The main beneficiaries of government price support assistance were the very low-income farmers. C) If the demand for agricultural products is inelastic, a relatively small decrease in supply will increase gross farm incomes. D) There is a misallocation of resources in rural America because of absolute and relative increases in the size of the farm population. Answer: C

McConnell/Brue: Economics, 16/e

Page 1004

Chapter 33: Agriculture: Economics and Policy

Use the following to answer questions 18-19:

Type: G Topic: 2 E: 615 MI: 371 18. Which diagram above best represents the short-run farm problem? A) A B) B C) C D) D Answer: B

Type: G Topic: 2 E: 615 MI: 371 19. Which diagram above best represents the long-run farm problem? A) A B) B C) C D) D Answer: A

Type: C Topic: 2 E: 615 MI: 371 Status: New 20. A bumper crop of farm products causes: A) only a slight decline in the price of farm products because the demand for farm products is income inelastic. B) a large decline in the price of farm products because the demand for farm products is price inelastic. C) only a slight decline in the price of farm products because the demand for farm products is income elastic. D) a large decline in the price of farm products because the demand for farm products is price elastic. Answer: B

McConnell/Brue: Economics, 16/e

Page 1005

Chapter 33: Agriculture: Economics and Policy

Type: C Topic: 2 E: 615 MI: 371 Status: New 21. An extraordinarily small crop of farm products due to drought causes: A) a large increase in the price of farm products because the demand for farm products is price inelastic. B) only a slight increase in the price of farm products because the demand for farm products is income elastic. C) only a slight increase in the price of farm products because the demand for farm products is income inelastic. D) a large increase in the price of farm products because the demand for farm products is price elastic. Answer: A

Long-run farm problem

Type: A Topic: 3 E: 617-618 MI: 373-374 22. Which of the following best describes the long-run farm problem? A) Lagging technology has decreased the productivity of farmers and therefore resulted in low farm prices and incomes. B) The highly inelastic nature of agricultural demand has caused small year-to-year fluctuations in farm output to result in highly unstable farm incomes. C) The supply of farm products has increased relative to the demand for them, and, because demand is inelastic, farm prices and incomes have therefore declined. D) The demand for farm products has increased relative to their supply, but the highly elastic nature of agricultural demand has caused these shifts to result in declining farm incomes. Answer: C

Type: F Topic: 3 E: 618 MI: 374 23. Each farm worker currently produces enough food and fiber to support about: A) 106 persons. B) 60 persons. C) 24 persons. D) 10 persons. Answer: A

Type: A Topic: 3 E: 618 MI: 374 Status: New 24. Over time, technological change has: A) reduced both the price elasticity and income elasticity of the demand for farm products. B) reduced the minimum efficient scale of production in agriculture and increased the prices of farm products. C) increased both price elasticity and income elasticity of the demand for farm products. D) increased the minimum efficient scale of production in agriculture and reduced the prices of farm products. Answer: D

Type: A Topic: 3 E: 618-619 MI: 374-375 Status: New 25. Which of the following statements about U.S. agriculture is true as it relates to the past several decades? A) The demand for farm products has declined, the supply of farm products has increased, and the price of farm products has declined. B) The demand for farm products has become both more income elastic and more price elastic. C) Minimum efficient scale has increased, the price of farm products has declined, and the number of farms has declined. D) The price of farm products have increased, minimum efficient scale has declined, and the supply of farm products has been stagnant. Answer: C

McConnell/Brue: Economics, 16/e

Page 1006

Chapter 33: Agriculture: Economics and Policy

Type: G Topic: 3 E: 617 MI: 373 26.

Which of the above diagrams best describes the long-run path of real (inflation-adjusted) farm prices? A) A B) B C) C D) D Answer: B

Type: A Topic: 3 E: 618-619 MI: 374-375 27. Productivity increases in agriculture have: A) increased the demand for foodstuffs. B) been negligible since the Second World War. C) increased the incomes of farm households above the national average. D) lowered farm prices. Answer: D

Type: F Topic: 3 E: 617 MI: 373 28. Relative to 1950, the inflation-adjusted prices of: A) cattle and hogs have increased in the United States, but the prices of corn and wheat have declined. B) cattle and hogs have decreased in the United States, but the price of corn and wheat have increased. C) cattle, hogs, corn, and wheat have decreased in the United States. D) cattle, hogs, corn, and wheat have increased in the United States. Answer: C

Type: F Topic: 3 E: 619 MI: 375 29. One consequence of the long-run farm problem has been a: A) rapid increase in the price of farm output. B) massive exit of workers from agriculture to other sectors of the economy. C) smaller average farm size. D) reduction in U.S. exports of farm products. Answer: B

McConnell/Brue: Economics, 16/e

Page 1007

Chapter 33: Agriculture: Economics and Policy

Use the following to answer questions 30-31:

Type: G Topic: 3 E: 619 MI: 375 30. Refer to the above diagram. Which of the following supply and demand shifts portray the long-run farm problem? A) S to S' and D to D' B) S to S' and D' to D C) S' to S and D' to D D) S' to S and D to D' Answer: B

Type: G Topic: 3 E: 619 MI: 375 31. The long-run farm problem as portrayed in the above diagram would involve price and quantity changes from: A) P2 to P3 and Q1 to Q4. C) P2 to P1 and Q1 to Q2. B) P1 to P4 and Q1 to Q4. D) P4 to P1 and Q4 to Q1. Answer: A

Type: A Topic: 3 E: 616 MI: 372 32. The growing importance of export demand for American agriculture has: A) reduced the international value of the dollar. B) had no significant effect on the stability of the demand for farm products. C) destabilized the total demand for farm products. D) stabilized the total demand for farm products. Answer: C

Type: F Topic: 3 E: 618 MI: 374 33. Which of the following is not characteristic of U.S. agriculture? A) Productivity has been increasing more slowly in agriculture than in the rest of the economy. B) The demand for agricultural commodities increases less than proportionate to increases in income. C) Resources in agriculture are relatively immobile. D) Demand is inelastic with respect to price. Answer: A

McConnell/Brue: Economics, 16/e

Page 1008

Chapter 33: Agriculture: Economics and Policy

Type: A Topic: 3 E: 617 MI: 373 34. Which of the following is correct? A) The rapid expansion of foreign incomes will reduce U.S. agricultural exports. B) A decrease in the international value of the dollar will reduce U.S. agricultural exports. C) An increase in the international value of the dollar will reduce U.S. agricultural exports. D) Changes in the international value of the dollar have no effect on U.S. agricultural exports. Answer: C

Type: F Topic: 3 E: 619 MI: 375 35. Currently farm employment is about: A) 42 percent of total employment. B) 20 percent of total employment. Answer: C

C) 2 percent of total employment. D) 8 percent of total employment.

Type: F Topic: 3 E: 619 MI: 375 36. Measured in terms of farm employment and the number of farms, agriculture has been: A) a declining industry. B) an expanding industry. C) a stable industry. D) a volitle industry. Answer: A

Type: F Topic: 3 E: 619 MI: 375 37. Over the past several decades, farm employment has: A) grown absolutely, but declined as a percentage of total employment. B) declined both absolutely and as a percentage of total employment. C) increased both absolutely and as a percentage of total employment. D) declined absolutely, but increased as a percentage of total employment. Answer: B

Type: F Topic: 3 E: 619 MI: 375 38. Which country has the smallest percentage of its labor force employed in agriculture? A) China B) India C) Brazil D) U.S. Answer: D

Parity concept

Type: F Topic: 4 E: 620 MI: 376 39. Historically, many aspects of U.S. farm policies had their origins in the: A) Agricultural Extension Service Act. C) Homestead Act. B) Agricultural Adjustment Act. D) Land Grant Act. Answer: B

McConnell/Brue: Economics, 16/e

Page 1009

Chapter 33: Agriculture: Economics and Policy

Type: D Topic: 4 E: 620-621 MI: 376-377 40. Which of the following statements best describes the parity concept? A) The prices of farm commodities should vary inversely with changes in the prices-paid index for farmers. B) Because of productivity increases, farmers are entitled to the same real income for a smaller volume of output. C) The money incomes of farmers should always be the same, regardless of increases or decreases in the prices of the products they buy. D) The production of a given real output entitles the producer to the same real income over time. Answer: D

Type: A Topic: 4 E: 621 MI: 377 41. If the prices paid by farmers increase and the prices received by farmers decrease, then the parity ratio: A) will necessarily decrease. C) may either increase or decrease. B) will necessarily increase. D) will be unaffected. Answer: A

Type: A Topic: 4 E: 621 MI: 377 42. If in a certain year the indices of prices received and paid by farmers were 115 and 142 respectively, the parity ratio would be: A) 123. B) 81. C) 69. D) 27.. Answer: B

Type: D Topic: 4 E: 621 MI: 377 43. The parity ratio: A) compares worker productivity in the farm and nonfarm sectors. B) is the ratio of per capita farm income to per capita nonfarm income. C) is the ratio of prices received by farmers to prices paid by farmers. D) is the ratio of prices paid by farmers to prices received by farmers. Answer: C

Type: F Topic: 4 E: 621 MI: 377 44. Which of the following statements is correct? A) The parity ratio has not exceeded 100 in this century. B) An increase in prices paid by farmers relative to prices received by farmers will increase the parity ratio. C) The parity ratio has generally been greater than 100 in the past four decades. D) The parity ratio has generally declined over the past five decades. Answer: D

Type: A Topic: 4 E: 621 MI: 377 45. If the prices received by farmers increased and the prices paid by farmers also increased, the parity ratio: A) will necessarily be unchanged. C) will necessarily increase. B) may either increase or decrease. D) will necessarily decline. Answer: B

McConnell/Brue: Economics, 16/e

Page 1010

Chapter 33: Agriculture: Economics and Policy

Effects of price supports

Use the following to answer questions 46-50:

Type: G Topic: 5 E: 622 MI: 378 46. Refer to the above diagram for the corn market. As the result of a supported corn price of B: A) a surplus of LG will result. C) a surplus of HG will result. B) a surplus of LH will result. D) a shortage of LG will result. Answer: A

Type: G Topic: 5 E: 622 MI: 378 47. Refer to the above diagram for the corn market. As a consequence of a price support of B, consumers will: A) increase their purchases of the product. B) pay a lower price, M rather than A, for the product. C) pay a higher price, B rather than A, for the product. D) pay a higher price, A rather than M , for the product. Answer: C

Type: G Topic: 5 E: 622 MI: 378 48. Refer to the above diagram for the corn market. A price support of B will cause a transfer from taxpayers to farmers of: A) 0BKL. B) MAFG. C) 0MFG. D) LKCG. Answer: D

Type: G Topic: 5 E: 622 MI: 378 49. Refer to the above diagram for the corn market. What effect will a price support of B have on the gross income of farmers? A) Gross income will be unchanged, although profits will rise. B) Gross income will increase from 0MFG to 0BKL. C) Gross income will increase from 0AJH to 0BCG. D) Gross income will decrease from 0BCG to 0BKL. Answer: C

Type: G Topic: 5 E: 622 MI: 378 50. Refer to the above diagram for the corn market. Assuming no externalities, a price support of B causes: A) economies of scale. C) an underallocation of resources to this product. B) production to be less profitable to farmers. D) an overallocation of resources to this product. Answer: D

McConnell/Brue: Economics, 16/e

Page 1011

Chapter 33: Agriculture: Economics and Policy

Type: A Topic: 5 E: 622 MI: 378 51. Which of the following statements is correct? A) Price supports may induce either an underallocation or an overallocation of resources to farm products. B) Supported prices have no effect on the allocation of resources to farm products. C) Supported prices induce an underallocation of resources to farm products. D) Price supports induce an overallocation of resources to farm products. Answer: D

Type: A Topic: 5 E: 623 MI: 379 52. The misallocation of resources associated with price supports: A) affects both domestic and foreign economies. C) affects only the domestic economy. B) affects only foreign economies. D) is fully offset by reductions in food prices. Answer: A

Type: A Topic: 5 E: 623 MI: 379 53. Acreage allotment programs were designed to: A) reduce the supply of agricultural products. B) make the demand for farm products more price elastic. C) bolster the demand for agricultural commodities. D) accelerate the movement of human resources out of farming. Answer: A

Type: A Topic: 5 E: 622 MI: 378 54. Which of the following is not an effect of an above-equilibrium price support on a farm product? A) the quantity of the product supplied will exceed the quantity demanded B) the consumer will pay a lower price and consume more of the product C) the gross incomes of farmers will rise D) the consumer will pay a higher price and consume less of the product Answer: B

Type: A Topic: 5 E: 623 MI: 379 55. Domestic price supports on, say, sugar: A) generate trade barriers to reduce imports of foreign sugar. B) increase depress world sugar prices. C) keep the domestic price of sugar artificially low. D) increase the earnings of foreign sugar producers. Answer: A

Type: A Topic: 5 E: 623 MI: 379 56. Price supports: A) decrease pollution because participants must practice organic farming. B) neither increase nor decrease environmental pollution. C) increase pollution because supports encourage the cultivation of marginal land and the use of more pesticides and fertilizers. D) discourage rent-seeking activity. Answer: C

McConnell/Brue: Economics, 16/e

Page 1012

Chapter 33: Agriculture: Economics and Policy

Type: A Topic: 5 E: 622 MI: 378 57. Because government price supports cause surplus production, government policies have been designed to: A) increase demand and decrease supply of farm products. B) decrease demand and increase supply of farm products. C) increase demand and increase supply of farm products. D) decrease demand and decrease supply of farm products. Answer: A

Type: A Topic: 5 E: 624 MI: 380 58. The food-stamp program is designed to: A) increase the supply of farm products. B) decrease the supply of farm products. Answer: C

C) increase the demand for farm products. D) decrease the demand for farm products.

Type: A Topic: 5 E: 624 MI: 380 59. Price supports in agriculture have been criticized because they: A) have hastened the exodus of labor from agriculture. B) subsidize consumers at the expense of farmers. C) help large farmers more than small farmers. D) create product shortages. Answer: C

Type: A Topic: 5 E: 624 MI: 380 60. When critics of U.S. farm policy say that it treats symptoms rather than causes, they mean that the: A) policy attempts to bolster low farm income, while the real problem is an overallocation of resources to agriculture. B) policy deals with the overallocation of resources to agriculture, while the basic farm problem is low incomes. C) policy attempts to bolster low farm incomes, while the real problem is an underallocation of resources to agriculture. D) restriction of output in the short run may reduce productive capacity in agriculture in the long run. Answer: A

Type: A Topic: 5 E: 624 MI: 380 61. Critics of U.S. farm policy: A) argue that most of the farm subsidies go to high-income farmers. B) argue that various components of agricultural policy are contradictory and seek conflicting goals. C) argue that the cost of farm policies increased sharply in the past two decades. D) make all of the above arguments. Answer: D

Type: A Topic: 5 E: 624 MI: 380 62. The Food for Peace program is designed to: A) achieve 100 percent price parity for all farm products. B) discover new uses for farm products through research and development. C) take agricultural land out of the production of feed grains. D) facilitate the distribution of U.S. farm products in the developing countries. Answer: D

McConnell/Brue: Economics, 16/e

Page 1013

Chapter 33: Agriculture: Economics and Policy

Political aspects of farm policy

Type: A Topic: 6 E: 625 MI: 381 63. Import quotas on sugar may cost consumers $2 billion per year. But this quota goes unchallenged because the $10 average annual cost per person is so small that probably not one voter in 200 knows the quota exists. This statement describes: A) the voting paradox. C) the median-voter model. B) the special-interest effect. D) free-rider problem. Answer: B

Type: A Topic: 6 E: 625 MI: 381 64. Farm groups spend considerable amounts of money to maintain and enlarge political support for farm subsidies. This illustrates: A) coalitions. B) rent-seeking activity. C) the special-interest effect. D) the voting paradox. Answer: B

Type: A Topic: 6 E: 625 MI: 381 65. The persistence of large subsidies to farmers can be explained in terms of: A) public choice theory. C) the paradox of voting. B) the inelasticity of demand for farm products. D) cost-benefit analysis. Answer: A

Type: A Topic: 6 E: 625 MI: 381 66. As applied to agriculture, the special-interest effect suggests that: A) urban legislators support farm legislation in exchange for rural legislators supporting urban-oriented legislation. B) a relatively small number of farmers receive large benefits at the expense of a much larger group of taxpayers who individually suffer small losses. C) politicians support farm subsidies because most of the associated costs are hidden rather than explicit. D) the size of farm subsidies should vary directly with a farmer's earned income. Answer: B

Type: A Topic: 6 E: 625 MI: 381 67. If farm state legislators support the food-stamp program to aid the urban poor and urban state legislators in turn support farm subsidies, this is an example of: A) the special-interest effect. C) the paradox of voting. B) political logrolling. D) cost-benefit analysis. Answer: B

Type: A Topic: 6 E: 620 MI: 376 68. Supporters of farm subsidies have: A) argued that farmers are comparatively poor and therefore should receive public help. B) contended that the family farm is an American "way of life" and should be protected. C) argued that farmers are at a disadvantage because they buy inputs in monopolized markets and sell their output in competitive markets. D) made all of the above arguments. Answer: D

McConnell/Brue: Economics, 16/e

Page 1014

Chapter 33: Agriculture: Economics and Policy

Global aspects of farm policy

Type: F Topic: 7 E: 626 MI: 382 69. Farm programs such as those of the United States and the European Union: A) are consistent with free world trade in agricultural products. B) cause a misallocation of agricultural resources internationally. C) cause the United States and the European Union to underproduce, while other nations overproduce, farm products. D) raise productivity in developing nations. Answer: B

Type: F Topic: 7 E: 626 MI: 382 70. Farm programs such as those of the United States and the European Union: A) encourage the United States and the European Union to use tariffs and quotas to restrict agricultural imports. B) cause United States and European Union farmers to produce less than domestic consumers want to purchase. C) increase world market prices for agricultural products. D) raise farm output in developing nations. Answer: A

Type: F Topic: 7 E: 626 MI: 382 71. Farm programs such as those of the United States and the European Union cause a misallocation of international agricultural resources primarily because: A) they distort domestic and world agricultural prices. B) they inhibit technological progress in agriculture. C) production and trade occur on the basis of comparative advantage. D) price supports are set below market-clearing levels. Answer: A

Type: F Topic: 7 E: 626 MI: 382 72. An international agreement reached in 1994 by the world's trading nations provides for: A) an increase in export subsidies on agricultural products. B) an increase in agricultural price supports. C) a reduction of tariffs on agricultural products. D) an international research organization designed to discover nonfood uses of agricultural output. Answer: C

Type: A Topic: 7 E: 626 MI: 382 73. In 1994 the world's trading nations agreed to: A) increase export subsidies to reduce world surpluses of farm products. B) reduce agricultural subsidies and tariffs on agricultural imports. C) eliminate all price supports for farm products. D) create an international farm price support system to replace the price support systems of individual countries. Answer: B

McConnell/Brue: Economics, 16/e

Page 1015

Chapter 33: Agriculture: Economics and Policy

Recent reform

Type: F Topic: 8 E: 626 MI: 382 Status: New 74. The purpose of the Freedom to Farm Act of 1996 was to: A) immediately end U.S. farm subsidies. B) end 60 years of U.S. price supports for American grain crops. C) eliminate U.S. tariffs and quotas on imported farm goods. D) stabilize short-run crop prices. Answer: B

Type: F Topic: 8 E: 626 MI: 382 Status: New 75. The U.S. price support program, which guaranteed prices for currently grown crops: A) ended with passage of the Freedom to Farm Act of 1996. B) began with the Grain Planting Act of 1914. C) remains the core of U.S. farm policy. D) was restored in full and expanded by the Farm Act of 2002. Answer: A

Type: F Topic: 8 E: 626-627 MI: 382-383 Status: New 76. The Farm Act of 2002: A) ended 60 years of U.S. price supports for American grain crops. B) restored the U.S. price support system (for currently grown crops) that was ended in the Freedom to Farm Act of 1996. C) ended the "freedom to plant" approach of the Freedom to Farm Act of 1996 and restored acreage allotments. D) maintained the "freedom to plant" and "direct-payment" features of the Freedom to Farm Act of 1996 but replaced "emergency aid" with an automatic system of aid based on low prices of farm products. Answer: D

Type: A Topic: 8 E: 626-627 MI: 382-383 Status: New 77. Irving Tiller exclusively grew wheat in each of the past three years. Under the Farm Act of 2002, Tiller: A) must grow wheat in the current year in order to receive direct payments from the Federal government. B) can grow whatever crop he wants to in the current year but will only receive direct payments from the Federal government if the price of wheat falls below a targeted price. C) can grow whatever crop he wants in the current year and will still receive direct payments from the Federal government based on his previously grown wheat crops. D) must grow something other than wheat to qualify for direct payments from the Federal government. Answer: C

Type: F Topic: 8 E: 627 MI: 383 Status: New 78. Which of the following is a feature of the Farm Act of 2002? A) direct payments (direct subsidies) based on crops currently grown B) countercyclical payments C) farm buyouts by government D) acreage allotments (restrictions on planting) Answer: B

McConnell/Brue: Economics, 16/e

Page 1016

Chapter 33: Agriculture: Economics and Policy

Type: F Topic: 8 E: 627 MI: 383 Status: New 79. Which of the following is not a feature of the Farm Act of 2002? A) direct payments (direct subsidies) based on crops previously grown B) countercyclical payments C) marketing loans D) acreage allotments (restrictions on the number of acres that can be planted) Answer: D

Type: D Topic: 8 E: 627 MI: 383 Status: New 80. Under the Farm Act of 2002, subsidies to farmers based on gaps between actual prices of crops and targeted prices are called: A) direct payments B) price supports C) countercyclical payments D) price gap set-asides. Answer: C

Type: A Topic: 8 E: 627 MI: 383 Status: New 81. John Deer exclusively grew corn in each of the past three years. Under the Farm Act of 2002, Deer: A) must grow corn in the current year in order to receive direct payments from the Federal government. B) can grow whatever crop he wants to in the current year but will only receive direct payments from the Federal government if the price of corn falls below a targeted price. C) will receive countercyclical payments if the price of corn falls below a targeted price, even though he does not grow corn in the current year. D) must grow something other than corn to qualify for direct payments from the Federal government. Answer: C

Type: A Topic: 8 E: 627 MI: 383 Status: New 82. Ward Planter exclusively grew soybeans in each of the past three years and currently participates in the marketing loan program of the Farm Act of 2002. If the "crop price" of soybeans at harvest is less than the pre-harvest "loan price", Planter can: A) sell his crop in the market and receive the difference between the crop price and loan price as a direct payment from the Federal government. B) take a "crop credit" based on the difference between the crop price and the loan price and use the credit to reduce Federal income taxes owed. C) receive an "emergency loan" that can be paid back over the following five years. D) forfeit the harvest to the lender and be free of the loan, thus receiving a subsidy because the proceeds from the loan exceed the revenues from the sale of the crop in the market. Answer: D

Type: A Topic: 8 E: 627 MI: 383 Status: New 83. The Farm Act of 2002 does not address the fundamental problem of agricultural subsidies, which is that those subsidies: A) reduce work incentive for farmers, thus reducing agricultural production. B) slow the exodus of resources from agriculture, increase production of agricultural products, and reduce crop prices and market incomes. C) increase the exodus of resources from agriculture and reduce investments by farmers in land and equipment. D) are based on past agricultural production and not on current agricultural production. Answer: B

McConnell/Brue: Economics, 16/e

Page 1017

Chapter 33: Agriculture: Economics and Policy

Last Word Questions

Type: A E: 628 MI: 384 84. (Last Word) The U.S. sugar program: A) decreases the domestic price of sugar. B) requires import quotas or tariffs on foreign sugar. C) increases the export earnings of other sugar-producing countries. D) aids developing countries that produce sugar. Answer: B

Type: A E: 628 MI: 384 85. (Last Word) The U.S. sugar program has: A) improved the world allocation of agricultural resources. B) caused the world price of sugar to rise above its domestic price. C) increased domestic sugar production. D) lowered the incomes of U.S. sugar growers. Answer: C

Type: F E: 628 MI: 384 86. (Last Word) The U.S. sugar price support program has: A) cost U.S. consumers much more than it has benefited U.S. sugar producers. B) cost U.S. consumers much less than it has benefited U.S. sugar producers. C) reduced the price of sugar to U.S. consumers. D) increased sugar imports as a percentage of U.S. sugar consumption. Answer: A

True/False Questions

Type: F E: 627 MI: 383 87. The Federal government has not paid subsidies to farmers since passage of the Freedom to Farm Act in 1996. Answer: False

Type: F E: 616 MI: 372 88. U.S. exports of farm products have generally declined as a percentage of U.S. farm output over the past half-century. Answer: False

Type: A E: 618 MI: 374 89. Increases in incomes usually result in more than proportionate increases in the demand for agricultural products in a growing economy. Answer: False

Type: F E: 619 MI: 375 90. Agriculture is overcrowded because of absolute and relative increases in the size of farm employment. Answer: False

McConnell/Brue: Economics, 16/e

Page 1018

Chapter 33: Agriculture: Economics and Policy

Type: F E: 619 MI: 375 91. About 20 percent of the labor force is in agriculture. Answer: False

Type: A E: 620 MI: 376 92. Farmers typically sell their products in highly competitive markets and buy in imperfectly competitive markets. Answer: True

Type: A E: 614-615 MI: 370-371 93. If the demand for agricultural products is inelastic, a relatively small increase in supply will cause farm prices and incomes to decline. Answer: True

Type: F E: 624 MI: 380 94. The principal beneficiaries of government agricultural aid have been the very low-income farmers. Answer: False

Type: A E: 624 MI: 380 95. The use of price support programs in agriculture has hastened the exodus of resources from agriculture. Answer: False

Type: A E: 624 MI: 380 96. The fact that most of a farmer's costs are variable costs has hastened the movement of resources from agriculture to industry. Answer: False

Type: A E: 621 MI: 377 97. The concept of parity has provided a rationale for government price supports for farm products. Answer: True

Type: A E: 621 MI: 377 98. If prices received by farmers decline and prices paid by farmers increase, the parity ratio will decline. Answer: True

Type: A E: 623 MI: 379 99. Environmentalists generally support price supports because these subsidies motivate additional farm production. Answer: False

Type: F E: 626-627 MI: 382-383 Status: New 100. The Farm Act of 2002 provides three types of agricultural subsidies: direct payments, countercyclical payments, and marketing loans. Answer: True

McConnell/Brue: Economics, 16/e

Page 1019

CHAPTER 34

Income Inequality and Poverty

Topic 1. 2. 3. 4. 5. 6. 7. 8. 9. Income inequality; Lorenz Curve Income mobility; government redistribution Causes of income inequality Trends in income inequality Arguments for and against inequality Equality versus efficiency tradeoff Poverty Income maintenance system Welfare reform and TANF Consider This Last Word True-False

Question numbers 1-27 28-42 43-54 55-62 63-68 69-72 73-85 86-115 116-125 126-127 128-130 131-145

____________________________________________________________

_______________________________________

____________________________________________________________

_______________________________________

Multiple Choice Questions Income inequality; Lorenz Curve

Type: F Topic: 1 E: 631 MI: 387 1. In 2001 the average income for families in the United States was about: A) $67,000. B) $22,000. C) $9,600. D) $82,000. Answer: A

Type: F Topic: 1 E: 632 MI: 388 2. In the quintile distribution of income, the term "quintile" represents: A) 5 percent of the income receivers. C) 20 percent of the income receivers. B) 10 percent of the income receivers. D) 25 percent of the income receivers. Answer: C

Type: F Topic: 1 E: 632 MI: 388 3. In 2001 a family with an annual income of $100,000 would find itself in the: A) second lowest quintile of the family income distribution. B) third quintile of the family income distribution. C) fourth quintile of the family income distribution. D) fifth (highest) quintile of the family income distribution. Answer: D

Chapter 34: Income Inequality and Poverty

Type: F Topic: 1 E: 632 MI: 388 4. In 2001 a family with an annual income of $18,000 would find itself in the: A) highest quintile of the family income distribution. B) lowest quintile of the family income distribution. C) second lowest quintile of the family income distribution. D) second highest quintile of the family income distribution. Answer: B

Type: F Topic: 1 E: 632 MI: 388 5. The lowest quintile of families in the income distribution receives about: A) 2 percent of the total income. C) 8 percent of the total income. B) 4 percent of the total income. D) 10 percent of the total income. Answer: B

Type: A Topic: 1 E: 632 MI: 388 6. If the top quintile (fifth) of all families receives 48 percent of total income and the lowest quintile receives 4 percent of total income, we can conclude that: A) the resulting Lorenz curve will be a 45-degree straight line. B) the top quintile receives twelve times as much income as the lowest quintile. C) the middle quintile must receive 60 percent of all income. D) government must be subsidizing the top quintile. Answer: B

Type: A Topic: 1 E: 632 MI: 388 7. The highest quintile of families in the income distribution: A) receives about 55 percent of the total income. C) is comprised of 5 percent of all families. B) is comprised of 20 percent of all families. D) receives about 35 percent of the total income. Answer: B

Use the following to answer questions 8-9: Answer the next question(s) on the basis of the following Census data.
Quintile Lowest Second Third Fourth Highest (A) Income 10% 20 40 20 10 (B) Income 15% 14 19 22 30 (C) Income 4% 10 15 23 48 (D) Income 22% 17 13 40 8

Type: F Topic: 1 E: 632 MI: 388 8. Which of the above distributions is accurate for the United States? A) A B) B C) C D) D Answer: C

McConnell/Brue: Economics, 16/e

Page 1022

Chapter 34: Income Inequality and Poverty

Type: F Topic: 1 E: 632 MI: 388 9. In 2001 the highest quintile of families in the U.S. income distribution received about: A) 35 percent of the income. C) 60 percent of the income. B) 52 percent of the income. D) 48 percent of the income. Answer: D

Type: F Topic: 1 E: 632 MI: 388 10. Census data indicate that currently the top fifth of all families receive about ________ times as much income as the bottom fifth. A) eight B) twelve C) four D) three Answer: B

Type: F Topic: 1 E: 633 MI: 389 11. The share of income going to the highest 20 percent of income receiver is: A) nearly 75 percent in the United States. B) lower in the United States than in some South American countries such as Mexico, Guatemala, and Brazil. C) lower today in the United States than in 1969. D) remarkably similar over a wide range of nations, including those which are rich and those which are poor. Answer: B

Type: A Topic: 1 E: 632 MI: 388 12. The Lorenz curve: A) plots graphically the poverty rate over time. B) is located closer to the diagonal today than it was immediately after the Second World War. C) plots graphically the distribution of income. D) is located farther from the diagonal when income is defined to include the value of noncash transfers. Answer: C

Type: D Topic: 1 E: 633 MI: 389 13. The Lorenz curve shows the: A) progressiveness of the Federal tax system. B) degree of inequality in the distribution of income. C) degree of discrimination against blacks and other minorities. D) percentage of the population that is below the poverty line. Answer: B

Type: D Topic: 1 E: 633 MI: 389 14. The Lorenz curve portrays: A) the functional distribution of income. B) the ratio of labor to capitalist income. Answer: C

C) the personal distribution of income. D) income equality.

McConnell/Brue: Economics, 16/e

Page 1023

Chapter 34: Income Inequality and Poverty

Type: D Topic: 1 E: 633 MI: 389 Status: New 15. The Gini ratio: A) measures the relative extent of poverty in a nation. B) compares the income of persons, households, or families at the 90th percentile of the income distribution to the income at the 10th percentile. C) is a numerical measure of the overall dispersion of income in a nation. D) is found by dividing the entire area below and to the right of the diagonal in the Lorenz diagram by the area between the diagonal and Lorenz curve. Answer: C

Type: F Topic: 1 E: 633 MI: 389 Status: New 16. The Gini ratio of income inequality ranges between: A) 0 and 10,000. B) 1 and 10. C) -1 and +1 D) 0 and 1. Answer: D

Use the following to answer questions 17-20:

Percent of income

(a)

(b)

(c)

(d)

(e)

Percent of families
Type: G Topic: 1 E: 632 MI: 388 17. Refer to the above diagram where curves (a) through (d) are for five different countries. Income is equally distributed in: A) none of the countries shown. B) country b. C) countries a and e. D) country a. Answer: D

Type: G Topic: 1 E: 633 MI: 389 Status: New 18. Refer to the above diagram where curves (a) through (e) are for five different countries. The Gini ratio is lowest in country: A) A. B) B. C) C. D) E. Answer: A

McConnell/Brue: Economics, 16/e

Page 1024

Chapter 34: Income Inequality and Poverty

Type: G Topic: 1 E: 633 MI: 389 19. Refer to the above diagram where curves (a) through (e) are for five different countries. Of the countries listed below, income is most unequally distributed in country: A) a. B) b. C) c. D) d. Answer: D

Type: G Topic: 1 E: 633 MI: 389 Status: New 20. Refer to the above diagram where curves (a) through (e) are for five different countries. The Gini ratio is: A) zero in country (e) and 1 in country (a). C) zero in country (a) and 1 in country (e), B) greater in country (b) than in country (c). D) less than 0.5 in country (d). Answer: C

Use the following to answer questions 21-23: Answer the next question(s) on the basis of the following information. Suppose population A, consisting of Al, Bob, Curt, Doris, and Ellie, receive annual incomes of $5000, $2500, $1250, $750, and $500, respectively.

Type: A Topic: 1 E: 632 MI: 388 21. Refer to the above information. What percentage of total income is received by the richest quintile? A) 50 B) 5 C) 25 D) 20 Answer: A

Type: A Topic: 1 E: 632 MI: 388 22. Refer to the above information. What percentage of total income is received by the lowest 60 percent of the income receivers in population A? A) 60 B) 50 C) 25 D) 18 Answer: C

Type: A Topic: 1 E: 632 MI: 388 23. Refer to the above information. Population B, consisting of Fred, George, Holly, Irma, and Joan, receive incomes of $4000, $3000, $1250, $950, and $800, respectively. We: A) can say that the income of population B is less equally distributed than that of population A. B) can say that the income of population B is more equally distributed than that of population A. C) cannot make a meaningful comparison of the income distributions of populations A and B. D) can say that the poorest quintile of population B receives 12 percent of total income. Answer: B

Type: A Topic: 1 E: 633 MI: 389 Status: New 24. The greater the area between the Lorenz curve and the diagonal in the Lorenz Curve diagram, the: A) smaller is the Gini ratio and the greater is the degree of income inequality. B) larger is the Gini ratio and the greater is the degree of income inequality. C) smaller is the Gini ratio and the greater is the percentage of the population in poverty. D) larger is the Gini ratio and the greater is the percentage of the population in poverty. Answer: B

McConnell/Brue: Economics, 16/e

Page 1025

Chapter 34: Income Inequality and Poverty

Type: A Topic: 1 E: 633 MI: 389 Status: New 25. Which of the following Gini ratios indicates the highest degree of income inequality? A) 0.78 B) 0.65 C) 0.29 D) 0.42 Answer: A

Type: A Topic: 1 E: 633 MI: 389 Status: New 26. Which of the following Gini ratios indicates the least degree of income inequality? A) 0.71 B) 0.55 C) 0.31 D) 0.45 Answer: C

Type: A Topic: 1 E: 633 MI: 389 Status: New 27. Which of the following countries has the highest Gini ratio? A) France B) United States C) Japan D) Mexico Answer: D

Income mobility; government redistribution

Type: D Topic: 2 E: 633 MI: 389 28. The movement of individuals and families from one income quintile to another over time is called: A) income averaging. B) wealth turnover. C) income mobility. D) the ratchet effect. Answer: C

Type: D Topic: 2 E: 633 MI: 389 29. Income mobility: A) contributes to greater wealth inequality in the United States. B) is less in the United States than in most developing nations. C) is the movement of individuals and families from one income quintile to another over time. D) makes lifetime income inequality among income receivers in the United States greater than income inequality in any single year. Answer: C

Type: A Topic: 2 E: 633 MI: 389 30. Income mobility: A) makes lifetime income inequality among income receivers in the United States less than income income inequality in any single year. B) is less in the United States than in most developing nations. C) is the movement of wage earners from one job to another. D) reduces the total percentage of families in the lowest quintile of the income distribution over time. Answer: A

Type: A Topic: 2 E: 633 MI: 389 31. The lifetime distribution of income is more equal than the annual distribution. This statement is: A) true, because the rich have gotten richer and the poor have gotten poorer. B) true, because there is considerable income mobility over time. C) false, because people tend to stay in the same income quintile over extended periods of time. D) false, because the rich have gotten richer and the poor have gotten poorer. Answer: B

McConnell/Brue: Economics, 16/e

Page 1026

Chapter 34: Income Inequality and Poverty

Type: A Topic: 2 E: 633 MI: 389 32. People's incomes are relatively low when they are young, reach a peak in middle age, and then decline. This fact helps explain: A) the wide variations of Gini ratios among nations. B) the equality-efficiency tradeoff. C) why the lifetime distribution of income is more equal than the distribution in any given year. D) why the lifetime distribution of income is less equal than the distribution in any given year. Answer: C

Type: A Topic: 2 E: 634 MI: 390 33. Some economists have criticized standard government figures on income inequality, arguing that these data: A) exclude families whose earnings arise in the public sector. B) conceal the strong trend toward greater equality in family sizes. C) conceal the growth of poverty due to a growing number of discouraged workers. D) overstate the degree of income inequality by failing to include noncash transfers as income. Answer: D

Type: A Topic: 2 E: 634 MI: 390 34. In the United States,: A) taxes decrease, but transfers increase, income inequality. B) taxes increase, but transfers reduce, income inequality. C) both taxes and transfers decrease income inequality. D) both taxes and transfers increase income inequality. Answer: C

Type: F Topic: 2 E: 634 MI: 390 35. Which of the following statements concerning the relative distribution of income is correct? A) The relative distribution of before-tax incomes has become decidedly more equal since the Second World War. B) Taxes increase, but transfer payments decrease, the degree of income inequality. C) Taxes and transfer payments both decrease the degree of income inequality. D) Taxes and transfer payments both increase the degree of income inequality. Answer: C

Type: A Topic: 2 E: 634 MI: 390 36. Housing subsidies for low-income families: A) represent a cash transfer. C) represent a noncash transfer. B) are a part of the U.S. social insurance programs. D) conflict with the leaky-bucket analogy. Answer: C

Type: F Topic: 2 E: 634 MI: 390 37. Standard Census data on the distribution of income: A) take all taxes and transfer payments into account. B) are before taxes in that they do not account for personal income and payroll taxes. C) include noncash transfers. D) exclude cash transfers. Answer: B

McConnell/Brue: Economics, 16/e

Page 1027

Chapter 34: Income Inequality and Poverty

Type: A Topic: 2 E: 634 MI: 390 38. A person arguing that income inequality is actually less than that described by the standard Census data might say that the: A) Census data fail to include cash transfers as income. B) Census data fail to include the value of noncash transfers as income. C) lifetime distribution of income is less equal than the annual distribution. D) household distribution of income is much more equally distributed than the family distribution. Answer: B

Type: A Topic: 2 E: 634 MI: 390 39. Which of the following is a noncash transfer? A) a payment to a divorced mother under AFDC B) food stamps Answer: B

C) a social security payment to a retiree D) unemployment compensation benefits

Use the following to answer questions 40-42:

Type: G Topic: 2 E: 634 MI: 390 40. Refer to the above diagram. If line b represents the pretax and transfer distribution of income in the United States, we would expect the posttax and transfer distribution to be: A) line a. B) line b, because taxes and transfers have no effect on income distribution. C) line c. D) line d. Answer: A

Type: G Topic: 2 E: 634 MI: 390 41. Refer to the above diagram. If line c represents the distribution of income in 1969 in the United States, we would expect the distribution of income for 2003 to be: A) line a. B) line b. C) line c. D) line d. Answer: D

McConnell/Brue: Economics, 16/e

Page 1028

Chapter 34: Income Inequality and Poverty

Type: A Topic: 2 E: 634 MI: 390 Status: New 42. Refer to the above diagram. If line c represents the distribution of income before taxes and transfers and line b represents the distribution after taxes and transfers, then taxes and transfers have: A) added to income inequality. C) increased poverty. B) decreased the Gini ratio. D) reduced real GDP per person. Answer: B

Causes of income inequality

Type: A Topic: 3 E: 635-636 MI: 391-392 43. Each of the following contributes to income inequality except: A) differences in ability and training C) differences in wealth ownership B) differences in job tastes D) government transfers Answer: D

Type: A Topic: 3 E: 635 MI: 391 44. If discrimination based on gender and race was eliminated, we would expect the: A) personal distribution of income to become less equal. B) personal distribution of income to become more equal. C) personal distribution of income to be unaffected. D) functional distribution of income to change in favor of profits and interest. Answer: B

Type: A Topic: 3 E: 635 MI: 391 45. Differences in the amounts and quality of education and training: A) combine with differences in mental, physical, and aesthetic talents to produce income inequality. B) explain none of the income inequality in the United States. C) explain nearly all the income inequality in the United States. D) have lessened in the United States in the past decade, sharply reducing income inequality. Answer: A

Type: A Topic: 3 E: 635 MI: 391 46. One cause of income inequality in the United States is: A) the free public school system. B) differences in preferences for market work relative to nonmarket activities as well as differences in preferences for types of work. C) antidiscrimination laws. D) the social security system. Answer: B

Type: A Topic: 3 E: 635 MI: 391 47. Which of the following would likely reduce income inequality? A) a reduction in the number of high school dropouts. B) a reduction in social security benefits. C) greater inequality in the distribution of wealth. D) more stringent requirements to obtain occupational licenses. Answer: A

McConnell/Brue: Economics, 16/e

Page 1029

Chapter 34: Income Inequality and Poverty

Type: A Topic: 3 E: 635 MI: 391 48. Income: A) is a stock concept. B) consists of accumulated assets. Answer: C

C) is a flow concept. D) consists of wages only.

Type: D Topic: 3 E: 635 MI: 391 49. Wealth: A) is a flow concept. B) refers to accumulated financial assets only. Answer: D

C) refers to accumulated real assets only. D) refers to accumulated financial and real assets.

Type: A Topic: 3 E: 635 MI: 391 50. Wealth represents: A) a stock of real and financial assets. B) a flow of income. Answer: A

C) financial assets only. D) real assets only.

Type: A Topic: 3 E: 635 MI: 391 51. Which of the following is correct? A) Income is a stock and wealth is a flow concept. B) Both income and wealth are flow concepts. Answer: D

C) Both income and wealth are stock concepts. D) Income is a flow and wealth is a stock concept.

Type: F Topic: 3 E: 635 MI: 391 52. Wealth in the United States is: A) distributed in a way that reduces the degree of income inequality. B) more unequally distributed than is income. C) less unequally distributed than is income. D) distributed in a way that has no effect on income inequality. Answer: B

Type: F Topic: 3 E: 635 MI: 391 53. The distribution of wealth in the United States is such that it: A) is randomly distributed among income classes. B) has no perceptible impact on the distribution of income. C) reduces income inequality. D) contributes to income inequality. Answer: D

Type: A Topic: 3 E: 635 MI: 391 54. Which is correct? A) Wealth is less equally distributed than income and therefore increases income inequality. B) Wealth is more equally distributed than income and therefore increases income inequality. C) Wealth is less equally distributed than income and therefore decreases income inequality. D) Wealth is more equally distributed than income and therefore decreases income inequality. Answer: A

McConnell/Brue: Economics, 16/e

Page 1030

Chapter 34: Income Inequality and Poverty

Trends in income inequality

Type: F Topic: 4 E: 636 MI: 392 55. The distribution of U.S. income by quintiles: A) reveals a marked increase in the relative importance of the middle class between 1947 and 1999. B) shows considerably more inequality in the distribution of income immediately after the Second World War than presently. C) shows a movement toward greater inequality since 1969. D) shows a sharp movement toward greater equality since the Second World War. Answer: C

Type: F Topic: 4 E: 636 MI: 392 56. Since 1969 the distribution of income has: A) remained virtually constant from year to year. B) become more equal. C) varied considerably from year to year but in general has been stable. D) become less equal. Answer: D

Type: A Topic: 4 E: 636-637 MI: 392-393 57. Economists generally agree that the most significant cause of the recent increase in income inequality in the United States is: A) the failure of the legal minimum wage to keep pace with inflation. B) increased international competition. C) increased immigration. D) increased demand for, and hence pay to, highly skilled workers. Answer: D

Type: A Topic: 4 E: 636-637 MI: 392-393 58. Which of the following helps explain growing income inequality in the United States in recent years? A) reduced immigration B) increased demand for highly skilled workers C) the falling labor force participation rate of women D) increased power of unions Answer: B

Type: A Topic: 4 E: 637 MI: 393 59. The term "growing income inequality" implies that the: A) poor are getting poorer in both a relative and an absolute sense. B) Lorenz curve is shifting toward the diagonal. C) diagonal is shifting toward the Lorenz curve. D) top quintile of income receivers is getting relatively more of the total income than before and the bottom quintile is getting relatively less. Answer: D

McConnell/Brue: Economics, 16/e

Page 1031

Chapter 34: Income Inequality and Poverty

Type: A Topic: 4 E: 637 MI: 393 60. Which of the following does not help explain growing income inequality since 1969? A) the entry of unusually large numbers of less experienced workers into the labor force B) increased import competition C) declining returns to education as college graduates have trouble finding jobs D) increased immigration Answer: C

Type: F Topic: 4 E: 637 MI: 393 61. In the past two decades the difference between the earnings of college graduates and high school graduates has: A) fallen, thereby increasing income inequality. B) increased, thereby increasing income inequality. C) remained constant. D) increased, thereby decreasing income inequality. Answer: B

Type: F Topic: 4 E: 637 MI: 393 62. The entrance of large numbers of "baby boomers" into the labor force in the 1970s and 1980s: A) caused substantial reductions in permanent unemployment. B) lessened income inequality. C) increased income inequality. D) had no impact on income inequality. Answer: C

Arguments for and against inequality

Type: A Topic: 5 E: 638-639 MI: 394-395 63. The basic argument for income inequality is that: A) the very rich establish consumption patterns that are desirable for the rest of society to emulate. B) the rich buy luxury goods that soon become affordable to everyone else because of economies of scale. C) income inequality is essential to maintain incentives to produce. D) inequality undermines incentives and tends to reduce the size of the national income. Answer: C

Type: A Topic: 5 E: 638 MI: 394 64. The basic economic argument for greater income equality is that: A) an equal distribution of income is the logical outcome of any tax-transfer program. B) because citizens enjoy political equality, they are also entitled to economic equality. C) a more equal distribution of income will tend to maximize incentives to work, invest, and assume risk. D) a more equal distribution of a given amount of income will increase the total utility of consumers. Answer: D

McConnell/Brue: Economics, 16/e

Page 1032

Chapter 34: Income Inequality and Poverty

Use the following to answer questions 65-68:

Type: G Topic: 5 E: 639 MI: 395 65. Refer to the above diagrams that show identical marginal utility from income curves for Singer and Catalano. The marginal utility from income curves are drawn on the assumption that: A) Singer buys more inferior goods than does Catalano. B) Singer and Catalano have identical capacities to enjoy income. C) Catalano has a greater capacity to enjoy income than does Singer. D) Singer has a greater capacity to enjoy income than does Catalano. Answer: B

Type: G Topic: 5 E: 639 MI: 395 66. Refer to the above diagrams that show identical marginal utility from income curves for Singer and Catalano. If a given income of $20,000 is initially distributed so that Singer receives $15,000 and Catalano $5,000, the marginal utility: A) of the last dollar of income will be greater for Catalano than for Singer. B) derived from the last dollar will not be comparable as between the two income receivers. C) of the last dollar of income will be the same for both Singer and Catalano. D) of the last dollar of income will be greater for Singer than for Catalano. Answer: A

Type: G Topic: 5 E: 639 MI: 395 67. Refer to the above diagrams that show identical marginal utility from income curves for Singer and Catalano. If a given income of $20,000 is initially distributed so that Singer receives $15,000 and Catalano $5,000: A) no judgment can be made as to the effect of a redistribution of income on total utility. B) this initial distribution of income is maximizing the combined total utility of the two consumers. C) the combined total utility of the two consumers can be increased by redistributing income from Catalano to Singer. D) the combined total utility of the two consumers can be increased by redistributing income from Singer to Catalano. Answer: D

McConnell/Brue: Economics, 16/e

Page 1033

Chapter 34: Income Inequality and Poverty

Type: G Topic: 5 E: 639 MI: 395 68. Refer to the above diagrams that show identical marginal utility from income curves for Singer and Catalano. If this initial distribution of $15,000 to Singer and $5,000 to Catalano is altered in favor of greater equality, it may be argued that: A) the combined total utility of the two consumers will decline because Catalano has a greater capacity to derive utility from income than does Singer. B) incentives to produce will be weakened and total income will decrease. C) incentives to produce will be enhanced and total income will increase. D) the combined total utility of the two consumers will decline because Singer has a greater capacity to derive utility from income than does Catalano. Answer: B

Equality versus efficiency tradeoff

Type: D Topic: 6 E: 639-640 MI: 395-396 69. The equality-efficiency tradeoff refers to: A) the conflict between risk averters and risk takers. B) the willingness of Congress to abandon existing welfare programs in favor of a comprehensive plan to increase education and training for low-income persons. C) possible conflicts between the goals of economic efficiency and greater income equality. D) the difference between the goals of income equality and equality of economic opportunity. Answer: C

Type: D Topic: 6 E: 639-640 MI: 395-396 70. The equality-efficiency tradeoff suggests that: A) welfare programs stimulate incentives to work. B) inefficiencies result when income is transferred from rich to poor. C) noncash transfers are superior to cash transfers. D) economic growth is the best means of reducing poverty. Answer: B

Type: D Topic: 6 E: 639-640 MI: 395-396 71. The equality-efficiency tradeoff best describes the: A) tradeoff between a lower benefit-reduction rate and a higher break-even level of income maintenance plans. B) failure to include noncash benefits in calculating the poverty rate. C) inequities in Temporary Assistance for Needy Families benefits among the various states. D) the choice between "more equlity and less output" and "less equality and more output." Answer: D

Type: A Topic: 6 E: 639-640 MI: 395-396 72. Gomez argues that we need to increase the nation's output. Chang contends that our top priority should be a more equal distribution of income and output. It can be correctly stated that these two goals are: A) essentially unrelated. B) complementary because the realization of one will promote fulfillment of the other. C) at least partially competing because the redistribution of income might impair incentives to work and produce. D) complementary because a more equal distribution of income always promotes economic growth. Answer: C

McConnell/Brue: Economics, 16/e

Page 1034

Chapter 34: Income Inequality and Poverty

Poverty

Type: F Topic: 7 E: 640 MI: 396 73. According to the standard government definitions, the percentage of the population now considered to be living in poverty is about: A) 25 percent. B) 20 percent. C) 5 percent. D) 12 percent. Answer: D

Type: F Topic: 7 E: 640 MI: 396 Status: New 74. In 2001 the official poverty line for a family of four in the United States was about: A) $18,000. B) $24,000. C) $12,000. D) $28,000. Answer: A

Type: F Topic: 7 E: 640-641 MI: 396-397 75. Which of the following demographic groups does not bear a disproportionate burden of poverty? A) families headed by women B) the elderly C) Hispanics D) blacks Answer: B

Type: F Topic: 7 E: 641 MI: 397 76. The poverty rate in the United States was: A) 5 percent in 2001. B) higher in 2001 than in 1960. Answer: C

C) lower in 2001 than in 1993. D) 15 percent in 2001.

Type: F Topic: 7 E: 641 MI: 397 77. Which of the following groups has the highest poverty rate? A) age 65 or over B) foreign-born (non citizens) C) Hispanics D) female-headed families Answer: D

Type: F Topic: 7 E: 641 MI: 397 78. Which of the following groups has the lowest poverty rate? A) married-couple families B) whites C) foreign-born (not citizens) D) blacks Answer: A

Type: F Topic: 7 E: 641 MI: 397 79. The poverty rate for blacks is: A) about the same as that for Hispanics. B) below that for whites. Answer: A

C) considerably lower than that for Hispanics. D) below that for persons 65 years of age or older.

Type: F Topic: 7 E: 641 MI: 397 80. Which of the following would we expect to have the highest poverty rate? A) white households headed by males C) elderly white households B) white households headed by females D) black households headed by females Answer: D

McConnell/Brue: Economics, 16/e

Page 1035

Chapter 34: Income Inequality and Poverty

Type: C Topic: 7 E: 631, 641 MI: 388, 397 81. An increase in the poverty rate: A) will necessarily shift the Lorenz curve toward the diagonal. B) will necessarily shift the Lorenz curve away from the diagonal. C) may leave the Lorenz curve unchanged. D) normally accompanies strong growth of the economy. Answer: C

Type: F Topic: 7 E: 641 MI: 397 Status: New 82. The U.S. poverty rate for: A) Hispanics and blacks are quite similar. B) blacks is much lower than the rate for Hispanics. C) whites is greater than the overall poverty rate. D) persons aged 65 or older is higher than for children under 18. Answer: A

Type: F Topic: 7 E: 641 MI: 397 Status: New 83. The U.S. poverty rate for: A) Hispanics is much higher than that for blacks. B) children under 18 years of age is higher than for the overall population. C) blacks has increased since 1993. D) foreign-born individuals who are not citizens is below the poverty rate for the general population. Answer: B

Type: F Topic: 7 E: 642 MI: 398 Status: New 84. Which of the following statements about U.S. poverty is true? A) Foreign-born individuals who are not citizens have lower poverty rates than does the general population. B) The poverty in rural areas is highly visible to the general population. C) An estimated one-half of the people in poverty remain there for less than 2 years before climbing out of poverty. D) Poverty necessarily increases when income inequality rises. Answer: C

Type: A Topic: 8 E: 643 MI: 399 85. A major difference between social insurance and welfare is that social insurance: A) exclusively involves older Americans whereas welfare is confined mainly to mothers with young children. B) forces recipients to demonstrate need while welfare does not. C) is normally financed by earmarked payroll taxes while welfare is financed out of general tax revenues. D) provides cash transfers while welfare does not. Answer: C

McConnell/Brue: Economics, 16/e

Page 1036

Chapter 34: Income Inequality and Poverty

Income maintenance system

Type: A Topic: 8 E: 642 MI: 398 86. The U.S. income maintenance program consists of two kinds of programs. They are: A) the minimum wage law and Social Security. B) antidiscrimination law and education and training programs. C) social insurance and public assistance or welfare. D) progressive income taxes and transfer payments. Answer: C

Type: F Topic: 7 E: 641-642 MI: 397-398 87. The poor are said to be invisible because: A) many are located in isolated rural areas. B) many are located in central cities. C) poor people do not have a strong political voice. D) of all of the above reasons. Answer: D

Type: A Topic: 8 E: 642 MI: 398 88. An example of a social insurance program is ____________, whereas an example of public assistance program is _______. A) Temporary Assistance for Needy Families; unemployment compensation B) Supplemental Security Income; unemployment compensation C) Social Security; food stamps D) Medicaid; Medicare Answer: C

Type: F Topic: 8 E: 642 MI: 398 89. Which of the following is a social insurance program? A) TANF B) Medicaid C) Supplemental Security Income Answer: D

D) unemployment compensation

Type: D Topic: 8 E: 642 MI: 398 90. Which of the following is a public assistance or welfare program as opposed to a social insurance program? A) Supplemental Security Income (SSI) C) Medicare B) unemployment compensation D) Social Security Answer: A

Type: A Topic: 8 E: 643 MI: 399 91. A major difference between social insurance and public assistance is that the: A) former is regarded as an earned right while the latter is public charity. B) latter is regarded as an earned right while the former is public charity. C) latter is financed through earmarked payroll taxes and the former is financed by general tax revenues. D) former are state and local programs while the latter are Federal programs. Answer: A

McConnell/Brue: Economics, 16/e

Page 1037

Chapter 34: Income Inequality and Poverty

Type: F Topic: 8 E: 643 MI: 399 92. OASDHI (Social Security) is financed: A) by state income tax revenues. B) by payroll taxes on employees and employers. Answer: B

C) by Federal excise taxes. D) out of general tax revenues.

Type: F Topic: 8 E: 643 MI: 399 93. The primary purpose of Social Security is to: A) subsidize familes in need. B) offset the progressiveness of the personal income tax. C) provide income for workers who are temporarily unemployed. D) provide income for retired and disabled workers. Answer: D

Type: F Topic: 8 E: 643 MI: 399 94. For which of the following income-maintenance programs is aggregate spending the greatest? A) Medicare B) Social Security C) unemployment compensation D) TANF Answer: B

Type: D Topic: 8 E: 643 MI: 399 95. Medicaid: A) helps finance medical expenses for those participating in the TANF and Supplemental Security Income programs. B) has been abandoned in favor of privately provided medical insurance. C) is a program of medical insurance for the aged and retired. D) is a compulsory national health insurance program which covers all adult Americans. Answer: A

Type: D Topic: 8 E: 643 MI: 399 96. Which of the following terms describes a system of subsidized health insurance available to the aged? A) Medicare B) Supplemental Security Income (SSI) C) Medicaid D) TANF Answer: A

Type: D Topic: 8 E: 643 MI: 399 97. Which of the following provides public assistance to those who are blind and otherwise disabled? A) Medicare B) Supplemental Security Income (SSI) C) Medicaid D) TANF Answer: B

Type: D Topic: 8 E: 643 MI: 399 98. Which of the following provides retirement income to qualified retirees? A) Medicare B) Supplemental Security Income (SSI) C) OASDHI D) TANF Answer: C

Type: D Topic: 8 E: 643 MI: 399 99. Which of the following helps finance the medical expenses of individuals receiving TANF or SSI? A) Medicare B) Supplemental Security Income (SSI) C) Medicaid D) OASDHI Answer: C

McConnell/Brue: Economics, 16/e

Page 1038

Chapter 34: Income Inequality and Poverty

Type: A Topic: 8 E: 643 MI: 399 100. The Earned Income Tax Credit: A) increases the personal income tax liability of low-income working families. B) provides a cash payment to low-income working families if their tax credit exceeds their tax liability. C) is designed to make labor force employment less attractive. D) was eliminated as part of welfare reform in 1996. Answer: B

Type: A Topic: 8 E: 643 MI: 399 101. All but which one of the following are cash transfer programs? A) AFDC B) Supplemental Security Income (SSI) C) low-rent public housing Answer: C

D) OASDHI

Type: C Topic: 8 E: 643 MI: 399 102. Which one of the following would increase income inequality as measured by official Census data and the quintile distribution? A) a doubling of social security retirement benefits C) the elimination of the TANF program B) the elimination of the food stamp program D) reduced divorce rates Answer: C

Type: C Topic: 8 E: 643 MI: 399 103. Which of the following would move the Lorenz curve closer to the diagonal? A) a 20 percent boost in Social Security retirement benefits B) an increase in payroll taxes C) a reduction in the progressiveness of the Federal personal income tax D) the elimination of the TANF program Answer: A

Type: A Topic: 8 E: 644 MI: 400 104. In a public assistance plan the minimum annual income is: A) the sum of the income subsidy and earned income. B) earned income less the income subsidy. C) the amount of the income subsidy when earned income is zero. D) the income level at which the income subsidy becomes zero. Answer: C

Type: D Topic: 8 E: 644 MI: 400 105. The break-even income in a public assistance plan is: A) the amount of the income subsidy when earned income is zero. B) the sum of the income subsidy and earned income. C) the income level at which the income subsidy becomes zero. D) earned income less the income subsidy. Answer: C

Type: D Topic: 8 E: 644 MI: 400 106. In a public assistance plan, a family's income subsidy will be reduced by some percentage of each dollar of earned income. This percentage is the: A) transformation rate. B) benefit-reduction rate. C) rate of substitution. D) marginal utility rate. Answer: B

McConnell/Brue: Economics, 16/e

Page 1039

Chapter 34: Income Inequality and Poverty

Use the following to answer questions 107-112:
Plan A Earned Income Total income subsidy income $ 0 $1,000 $1,000 1,000 750 1,750 2,000 500 2,500 3,000 250 3,250 4,000 0 4,000 Plan B Earned Income Total income subsidy income $ 0 $2,000 $2,000 1,000 1,500 2,500 2,000 1,000 3,000 3,000 500 3,500 4,000 0 4,000 Plan C Earned Income Total income subsidy income $ 0 $1,500 $1,500 1,500 1,000 2,500 3,000 500 3,500 4,500 0 4,500

Type: T Topic: 8 E: 644 MI: 400 107. Which of the above plans entails the highest minimum annual income? A) Plan A B) Plan B C) Plan C D) cannot be determined from the data given Answer: B

Type: T Topic: 8 E: 644 MI: 400 108. Which of the above plans entails the highest benefit-reduction rate? A) Plan A B) Plan B C) Plan C D) cannot be determined from the data given Answer: B

Type: T Topic: 8 E: 644 MI: 400 109. The break-even income for Plan C above is: A) $4,000. B) $1,500. C) $4,500. D) $8,000. Answer: C

Type: T Topic: 8 E: 644 MI: 400 110. Refer to the above data. With respect to the objective of preserving incentives to work: A) Plan A is the most effective. C) Plan C is the most effective. B) Plan B is the most effective. D) Plans B and C are equally effective. Answer: A

Type: T Topic: 8 E: 645 MI: 401 111. Refer to the above data. With respect to the objective of alleviating poverty: A) Plan A is the most effective. C) Plan C is the most effective. B) Plan B is the most effective. D) Plans A and B are equally effective. Answer: B

Type: T Topic: 8 E: 644 MI: 400 112. For Plan A above the minimum annual income is: A) $3,000 and the benefit-reduction rate is 25 percent. B) $1,000 and the benefit-reduction rate is 25 percent. C) $4,000 and the benefit-reduction rate is 50 percent. D) $1,000 and the benefit-reduction rate is 50 percent. Answer: B

McConnell/Brue: Economics, 16/e

Page 1040

Chapter 34: Income Inequality and Poverty

Type: A Topic: 8 E: 644 MI: 400 113. If the minimum annual income is $5,000 in a public assistance plan and the benefit-reduction rate is 40 percent, the break-even level of income will be: A) $10,000. B) $2,500. C) $20,000. D) $12,500. Answer: D

Type: A Topic: 8 E: 644 MI: 400 114. If a public assistance plan has a benefit-reduction rate of 50 percent and the break-even income is $12,000, the plan's minimum annual income must be: A) $6,000. B) $24,000. C) $10,500. D) $8,000. Answer: A

Type: A Topic: 8 E: 644-645 MI: 400-401 115. A basic problem associated with public assistance plans is that: A) such plans necessarily are more costly than other income maintenance plans. B) it is extremely difficult to determine which families are eligible for benefits. C) there are conflicts between the goals of eliminating poverty, maintaining work incentives, and restraining program costs. D) the required bureaucracy is necessarily greater than it is for other income maintenance plans. Answer: C

Welfare reform and TANF

Type: A Topic: 9 E: 645 MI: 401 116. Which of the following was major criticism of the welfare program before it was reformed in 1996? A) It created dependency and reduced motivation to work. B) It paid benefits to high-income retirees as well as low-income retirees. C) It penalized recipients who had additional children. D) Benefits could only be collected for 26 weeks. Answer: A

Type: A Topic: 9 E: 645 MI: 401 117. Which of the following was a major criticism of the welfare program before it was reformed in 1996? A) Benefits could only be collected for 26 weeks. B) It paid benefits to high-income retirees as well as low-income retirees. C) It penalized recipients who had additional children. D) It allegedly created a culture of poverty and unhealthy dependency on government. Answer: D

Type: A Topic: 9 E: 645 MI: 401 118. Welfare reform enacted in 1996: A) required welfare recipients to work after receiving assistance for 2 years. B) replaced food stamps with cash grants. C) provided life-time free health insurance benefits for those permanently leaving welfare for work. D) greatly increased the minimum income level of the former AFDC program (now TANF). Answer: A

McConnell/Brue: Economics, 16/e

Page 1041

Chapter 34: Income Inequality and Poverty

Type: D Topic: 9 E: 645 MI: 401 119. The TANF program: A) is referred to as a negative income tax plan. B) have been found to be unconstitutional by the Supreme Court. C) limited total lifetime welfare benefits to 5 years and required people to work after receiving benefits for 2 years. D) perpetuate the so-called culture of poverty. Answer: C

Type: A Topic: 9 E: 645 MI: 401 120. The saying "Give a person a fish and he shall eat today; teach a person to fish and he will eat forever" is most consistent with: A) the food stamp program. C) cash transfer programs. B) workfare plans that pay for job training. D) social insurance plans. Answer: B

Type: F Topic: 9 E: 645 MI: 401 121. The TANF program is designed to: A) increase the minimum wage. B) shift people from welfare to employment. C) provide medical insurance to poverty level workers. D) provide income to the blind and disabled. Answer: B

Type: A Topic: 9 E: 645 MI: 401 122. The Personal Responsibility (welfare reform) Act of 1996: A) ended the AFDC program, replacing it with public employment program. B) set a life-time limit of 5 years on receiving welfare benefits and required able-bodied adults to work after receiving assistance for 2 years. C) cut back on cash assistance but expanded the food stamp program. D) extended welfare payments to families with unemployed fathers present. Answer: B

Type: A Topic: 9 E: 645 MI: 401 123. Supporters of the Personal Responsibility (welfare reform) Act of 1996: A) cite large recent declines in the welfare rolls as evidence of the law's effectiveness. B) say that its elimination of the food stamp program will encourage people to work. C) argue that its work provisions should also be incorported into the Supplemental Security Income program. D) say that, although it defers the future insolvency of the TANF program, it does not eliminate it. Answer: A

Type: F Topic: 9 E: 645 MI: 401 Status: New 124. Since its inception in 1996, the Temporary Assistance for Needy Families (TANF) program has: A) increased, rather than reduced, the number of people on welfare. B) reduced the number of people on welfare by more than one-half. C) aided the poor by automatically increasing welfare payments when inflation occurs. D) greatly increased the unemployment rate. Answer: B

McConnell/Brue: Economics, 16/e

Page 1042

Chapter 34: Income Inequality and Poverty

Type: F Topic: 9 E: 645 MI: 401 Status: New 125. The recession of 2001: A) fully undid the decline in welfare recipients experienced in the prior 4 years under welfare reform. B) led Congress to "index" TANF payments to inflation; that is, automatically increase benefits to compensate for higher prices. C) slowed the decline in welfare recipients over the prior 4 years but did not reverse the overall downward trend. D) led Congress to replace the TANF program with the AFDC (Aid for Families with Dependent Children) program. Answer: C

Consider This Questions

Type: A E: 640 MI: 396 Status: New 126. (Consider This) In "Slicing the Pizza" the shrinkage of the pizza refers to the: A) fact that poor people are not helped by the income maintenance system. B) efficiency losses--the losses of output and income-associated with the tax--transfer system. C) tendency of rich people in developing nations to hoard their wealth overseas. D) failure of the Internal Revenue Service to collect all the tax revenues owed by taxpayers. Answer: B

Type: A E: 640 MI: 396 Status: New 127. (Consider This) The main focus of the vignette "Slicing the Pizza" is the: A) equality-efficiency tradeoff. B) principal-agent problem. C) impact of market power on economic efficiency. D) the highly unequal distribution of U.S. wealth. Answer: A

Last Word Questions

Type: F E: 646 MI: 402 128. (Last Word) Between 1989 and 2001 in the United States: A) average U.S. household wealth increased and median household declined. B) average U.S. household wealth declined and median household wealth increased. C) both average and median U.S. household wealth declined. D) both average and median U.S. household wealth increased. Answer: D

Type: F E: 646 MI: 402 129. (Last Word) In 2001 the wealthiest 1 percent of U.S. households held ____ percent of U.S. household wealth. A) 33 percent B) 66 percent C) 82 percent. D) 22 percent. Answer: A

McConnell/Brue: Economics, 16/e

Page 1043

Chapter 34: Income Inequality and Poverty

Type: F E: 646 MI: 402 130. (Last Word) Between 1989 and 2001: A) the distribution of U.S. household wealth became more unequal. B) the distribution of U.S. household wealth become less unequal. C) median U.S. household wealth, adjusted for inflation, surpassed average household wealth. D) median U.S. household wealth, adjusted for inflation, declined. Answer: A

True/False Questions

Type: F E: 643 MI: 399 131. Because of TANF and a strong economy, the U.S. welfare rolls fell by one-half between 1996 and 2000. Answer: True

Type: A E: 632 MI: 388 132. Government transfer programs result in a U.S. Lorenz Curve that is closer to the diagonal line than would be the case without the programs. Answer: True

Type: D E: 632 MI: 388 133. A curve showing the quintile distribution of a nation's income is called the Phillips Curve. Answer: False

Type: D E: 643 MI: 399 134. TANF stands for Temporary Assistance for Needy Families. Answer: True

Type: F E: 645 MI: 401 135. The Personal Responsibility Act of 1996 required all employees to buy insurance against industrial accidents and diseases. Answer: False

Type: F E: 645 MI: 401 136. In the United States the poor have gotten poorer since 1969 and the rich have gotten richer. Answer: False

Type: A E: 634 MI: 390 137. The top 20 percent of U. S. income earners receive nearly 80 percent of total U. S. income. Answer: False

Type: F E: 643 MI: 399 138. Currently over 90 percent of all workers in the United States are covered by OASDHI. Answer: True

McConnell/Brue: Economics, 16/e

Page 1044

Chapter 34: Income Inequality and Poverty

Type: F E: 642, 643 MI: 398, 399 139. OASDHI is financed by payroll taxes levied on both employers and employees. Answer: True

Type: F E: 642, 643 MI: 398, 399 140. Unemployment compensation is financed by taxes levied on employers. Answer: True

Type: A E: 632 MI: 388 141. The closer the Lorenz curve is to the diagonal, the greater is the degree of income inequality. Answer: False

Type: F E: 641 MI: 397 142. The U.S. poverty rate for the elderly (65 and over) is higher than for the general population. Answer: False

Type: F E: 634 MI: 390 143. The standard data on income distribution do not include the value of noncash transfers as income. Answer: True

Type: F E: 641 MI: 397 144. The U.S. poverty rate was considerably lower in 2001 than in 1960. Answer: True

Type: F E: 641 MI: 397 145. The U.S. poverty rate for blacks is substantially higher than that for Hispanics. Answer: False

McConnell/Brue: Economics, 16/e

Page 1045

CHAPTER 35

Labor Market Institutions and Issues: Unionism, Discrimination, Immigration

Topic 1. American unionism 2. Decline of union membership 3. Collective bargaining 4. Economic effects of unions 5. Types of discrimination 6. Costs of discrimination 7. Taste for discrimination model 8. Statistical discrimination 9. Crowding model 10. Antidiscrimination policies and controversies 11. Immigration Last Word True-False

Question numbers 1-21 22-29 30-47 48-68 69-76 77-81 82-96 97-104 105-114 115-119 120-140 141-143 144-160

____________________________________________________________

_______________________________________

____________________________________________________________

_______________________________________

Multiple Choice Questions American unionism

Type: F Topic: 1 E: 650 MI: 406 1. Presently, there are about _____ million union members in the United States. A) 5 B) 11 C) 16 D) 32 Answer: C

Type: F Topic: 1 E: 650 MI: 406 2. Presently, about ____ percent of employed wage and salary workers belong to unions. A) 20 B) 6 C) 10 D) 13 Answer: D

Type: F Topic: 1 E: 650 MI: 406 Status: New 3. In 2002 the number of union members in the United States and the rate of U.S. unionization were: A) 10 million and 7 percent. C) 25 million and 18 percent. B) 16 million and 13 percent. D) 32 million and 25 percent. Answer: B

Chapter 35: Labor Market Institutions and Issues: Unionism, Discrimination, Immigration

Type: F Topic: 1 E: 650-652 MI: 406-408 Status: New 4. In 2002 the U.S. unionization rate was: A) 5 percent, down by nearly one-half from the mid-1950s. B) 18 percent, up by about one-fourth from the mid-1950s. C) 13 percent, down by nearly one-half from the mid-1950s. D) 21 percent, down by 4 percentage points from the mid-1950s. Answer: C

Type: F Topic: 1 E: 650 MI: 406 5. The majority of union members in the United States belong to unions that are: A) affiliated with the United Workers of America (UWA). B) affiliated with the AFL-CIO. C) independent unions that are not affiliated with the AFL-CIO. D) growing rapidly in membership. Answer: B

Type: A Topic: 1 E: 650 MI: 406 6. Independent unions: A) have greater combined membership than the AFL-CIO. B) are not affiliated with the AFL-CIO. C) include the United Autoworkers and Carpenters Union. D) have about the same combined membership as the AFL-CIO. Answer: B

Type: F Topic: 1 E: 650 MI: 406 7. AFL-CIO stands for: A) American Federation of Labor-Congress of Industrial Organizations. B) American Federation of Labor-Committee for Industrial Opportunity. C) Assembly of Factory Laborers-Congress of Industrial Organizations. D) Assembly of Factory Laborers-Committee for Industrial Opportunity. Answer: A

Type: D Topic: 1 E: 650 MI: 406 8. The philosophy that unions should focus on achieving practical economic objectives such as higher pay, shorter hours, and improved working conditions, rather than schemes to reorganize or overthrow capitalism, is called: A) craft unionism. B) industrial unionism. C) business unionism. D) worker unionism. Answer: C

Type: A Topic: 1 E: 651 MI: 407 9. Suppose in some economy there are 100 million workers, that 10 million of those workers work in retail trade, and 1 million of the retail workers belong to unions. Total union membership in this economy is 40 million. The rate of unionization in retail trade is: A) 1 percent. B) 10 percent. C) 40 percent. D) 100 percent. Answer: B

McConnell/Brue: Economics, 16/e

Page 1048

Chapter 35: Labor Market Institutions and Issues: Unionism, Discrimination, Immigration

Type: A Topic: 1 E: 651 MI: 407 10. Suppose in some economy there are 100 million workers, that 8 million of those workers work in retail trade, and 2 million of the retail workers belong to unions. Total union membership in this economy is 30 million. The rate of unionization in the economy is: A) 30 percent and the rate of unionization in retail trade in 20 percent. B) 8 percent and the rate of unionization in retail trade is 2 percent. C) 30 percent and rate of unionization in retail trade is 25 percent. D) 20 percent and rate of unionization in retail trade is 25 percent. Answer: C

Type: A Topic: 1 E: 651 MI: 407 11. Suppose in some economy there are 100 million workers, that 2 million of those workers work in mining, and 1 million of the miners belong to unions. The rate of unionization in mining is: A) 50 percent. B) 2 percent. C) 1 percent. D) 20 percent. Answer: A

Type: A Topic: 1 E: 651 MI: 407 12. In the United States, the rate of unionization is: A) higher in mining than in government. B) lower in transportation than agriculture. C) higher in transportation than in retail trade. D) lower in government than in finance, insurance, and real estate. Answer: C

Type: A Topic: 1 E: 651 MI: 407 13. In which of the following U.S. industries is the rate of unionization the lowest? A) transportation B) mining C) manufacturing D) services Answer: D

Type: A Topic: 1 E: 651 MI: 407 14. In which of the following U.S. industries is the rate of unionization the highest? A) construction B) services C) retail trade D) agriculture Answer: A

Type: A Topic: 1 E: 651 MI: 407 15. In the United States, the rate of unionization is: A) higher for sales workers than for transportation workers. B) lower for managers than for machine operators. C) lower for craft workers than for sales workers. D) higher for technicians than for protective service workers. Answer: B

Type: A Topic: 1 E: 651 MI: 407 16. In which of the following U.S. occupations is the rate of unionization the highest? A) protective services B) sales workers C) managers D) clerical workers Answer: A

McConnell/Brue: Economics, 16/e

Page 1049

Chapter 35: Labor Market Institutions and Issues: Unionism, Discrimination, Immigration

Type: A Topic: 1 E: 651 MI: 407 17. In which of the following U.S. occupations is the rate of unionization the lowest? A) transportation workers B) machine operators C) craft workers D) sales workers Answer: D

Type: A Topic: 1 E: 651 MI: 407 18. In the United States: A) whites have higher unionization rates than blacks. B) men have higher unionization rates than women. C) managers have higher unionization rates than transportation workers. D) workers in mining have higher unionization rates than workers in government. Answer: B

Type: A Topic: 1 E: 651 MI: 407 19. In the United States: A) blacks have higher unionization rates than whites. B) women have higher unionization rates than men. C) managers have higher unionization rates than transportation workers. D) workers in mining have higher unionization rates than workers in government. Answer: A

Type: A Topic: 1 E: 651 MI: 407 20. Who is the most likely to be a union member in the United States? A) A male retail trade worker. C) A black manager. B) A female transportation worker. D) A white agricultural worker. Answer: B

Type: F Topic: 1 E: 651 MI: 407 21. Compared to other industrially advanced nations, the rate of unionization in the United States is: A) very high. B) slightly above the average. C) very low. D) slightly below the average. Answer: C

Decline of union membership

Type: F Topic: 2 E: 652 MI: 408 22. Since 1980, labor union membership in the United States has been: A) increasing absolutely and as a percentage of the labor force. B) increasing absolutely but declining as a percentage of the labor force. C) decreasing absolutely and as a percentage of the labor force. D) decreasing absolutely but increasing as a percentage of the labor force. Answer: C

McConnell/Brue: Economics, 16/e

Page 1050

Chapter 35: Labor Market Institutions and Issues: Unionism, Discrimination, Immigration

Type: A Topic: 2 E: 652 MI: 408 23. U.S. union membership has declined substantially in recent years. Which of the following is not usually cited as a causal factor? A) Most currently nonunion workers are in manufacturing industries. B) Import competition has reduced or restrained the growth of employment in some highly unionized industries. C) Some industries and firms have migrated to the South. D) An unusually large proportion of the increase in employment has been concentrated among young workers and women. Answer: A

Type: A Topic: 2 E: 652 MI: 408 24. The relative decline of union membership in the United States is the result of changes in the composition of domestic output, shifts in the location of industry, and changes in the demographic makeup of the labor force. This statement describes the: A) labor-demand hypothesis. C) managerial-opposition hypothesis. B) shock effect. D) structural-change hypothesis. Answer: D

Type: D Topic: 2 E: 652 MI: 408 25. The structural-change hypothesis suggests that: A) the structure of industry has changed so as to encourage union growth. B) in the past two decades management has more aggressively opposed unionization. C) labor market discrimination has decreased because of the changing composition of the labor force. D) changes in the structure of industry have contributed to the decline of union membership. Answer: D

Type: D Topic: 2 E: 652 MI: 408 26. The managerial-opposition hypothesis suggests that: A) employers are less opposed to unions now than they were two or three decades ago. B) employers are more reluctant to hire minorities and women than they were two or three decades ago. C) employers are increasingly resistant to unions because they impose higher wage costs on firms. D) unions increase worker turnover and thereby increase productivity. Answer: C

Type: A Topic: 2 E: 652 MI: 408 27. The industry mix of domestic output has changed from manufacturing to services, resulting in a decline in union membership. This statement is most consistent with the: A) structural-change hypothesis. C) managerial-opposition hypothesis. B) substitution hypothesis. D) union-growth hypothesis. Answer: A

Type: A Topic: 2 E: 652 MI: 408 28. Which of the following might be expected to increase union membership? A) increased labor force participation by women and young people B) decreased imports of manufactured goods C) increased substitution of capital for labor in the production process D) continued rapid growth of computer-related indusries Answer: B

McConnell/Brue: Economics, 16/e

Page 1051

Chapter 35: Labor Market Institutions and Issues: Unionism, Discrimination, Immigration

Type: A Topic: 2 E: 652 MI: 408 29. Unionized firms pay higher wages and are therefore at a cost disadvantage compared to nonunion firms. Employers therefore want to resist unionization. This perspective best describes: A) the structural change hypothesis. C) the managerial-opposition hypothesis. B) the voice mechanism. D) the substitution hypothesis. Answer: C

Collective bargaining

30.

Type: F Topic: 3 E: 652-653 MI: 408-409 Collective bargaining agreements usually cover: A) wages and hours. B) union status. C) seniority and job opportunities. Answer: D

D) all of the above.

Type: A Topic: 3 E: 653 MI: 409 31. Suppose that under its collective bargaining agreement the ABC Corporation can hire nonunion workers, but such workers must join the union within 30 days. This agreement embodies: A) an open shop. B) a closed shop. C) a union shop. D) an agency shop. Answer: C

Type: A Topic: 3 E: 653 MI: 409 32. The XYZ Corporation's collective bargaining agreement indicates that it may hire either union or nonunion workers and that the latter are under no obligation to join the union. This agreement embodies: A) an open shop. B) a closed shop. C) a union shop. D) an agency shop. Answer: A

Type: D Topic: 3 E: 653 MI: 409 33. State right-to-work laws: A) permit employers to hire nonunion workers only if union labor is unavailable. B) prohibit employers from discriminating against minority groups in hiring workers. C) prohibit unions from discriminating against minority groups in recruiting members. D) make compulsory union membership (for example, a union shop) illegal. Answer: D

Type: D Topic: 3 E: 653 MI: 409 34. The ABC Company can hire either union or nonunion workers. But any nonunion workers who are hired must join the union in thirty days or forfeit their jobs. The ABC Company has a(n): A) preferential hiring agreement. B) union shop. C) closed shop. D) open shop. Answer: B

Type: A Topic: 3 E: 652 MI: 408 35. The Overnight Construction Company has just signed a collective bargaining contract in which it agrees that all workers it hires must be union members in good standing at the time they are hired. This provision reflects: A) preferential hiring. B) a maintenance-of-membership shop. C) a union shop. D) a closed shop. Answer: D

McConnell/Brue: Economics, 16/e

Page 1052

Chapter 35: Labor Market Institutions and Issues: Unionism, Discrimination, Immigration

Type: D Topic: 3 E: 653 MI: 409 36. State right-to-work laws: A) have been enacted by over one-half of the states in the nation. B) make yellow dog contracts illegal. C) allow for union shops while prohibiting closed shops. D) make union and agency shops illegal. Answer: D

Type: F Topic: 3 E: 653 MI: 409 Status: New 37. Twenty-two states currently (2003) have: A) prohibitions against unionization by state employees. B) compulsory union membership in the public sector. C) laws outlawing lockouts in labor disputes. D) state-right-to work laws. Answer: D

Type: A Topic: 3 E: 653 MI: 409 38. Under an agency shop agreement, firms can hire: A) only union workers. B) nonunion workers but these workers must join the union within a specified period, say, 30 days. C) nonunion workers but these workers must either join the union within a specified period, pay union dues, or donate an equivalent amount to charity. D) only nonunion workers. Answer: C

Type: A Topic: 3 E: 653 MI: 409 39. Suppose Steve Stone takes a job with Zemo Manufacturing, whose labor contract with its unions has an agency shop clause (30 days). Steve: A) must join the union within 30 days. B) can decide against joining the union and place the union dues in a retirement account. C) can decide against joining the union but then will not receive any union-negotiated wage increases. D) can decide against joining the union but nevertheless will have to pay union dues or donate an equivalent amount to an approved charity. Answer: D

Type: A Topic: 3 E: 653 MI: 409 40. Unions prefer: A) open shops to agency shops. B) agency shops to closed shops. Answer: D

C) agency shops to union shops. D) union shops to agency shops.

Type: D Topic: 3 E: 653 MI: 409 41. A COLA is a clause in a collective bargaining agreement that: A) specifies that one or more soft drink machines be available in each plant. B) requires nonunion workers nevertheless to pay union dues. C) automatically adjusts vacation time upward along with seniority. D) automatically adjusts pay upward when inflation occurs. Answer: D

McConnell/Brue: Economics, 16/e

Page 1053

Chapter 35: Labor Market Institutions and Issues: Unionism, Discrimination, Immigration

Type: A Topic: 3 E: 653 MI: 409 42. Which of the following is least likely to be found in a collective bargaining agreement (work agreement)? A) a union status clause B) a clause specifying where new plants, if any, will be located C) a cost-of-living adjustment clause D) grievance procedures Answer: B

Type: A Topic: 3 E: 653 MI: 409 43. In a labor dispute in which the existing contract has expired, a: A) firm can legally lock up unruly workers. B) firm can legally lock out union workers. C) union can legally restrict customer physical access to the firm. D) union can legally restrict physical access by management to the firm. Answer: B

Type: F Topic: 3 E: 653 MI: 409 Status: New 44. Which of the following bargaining tactics is illegal? A) a lockout B) a strike C) refusal to bargain in good faith D) hiring strikebreakers Answer: C

Type: A Topic: 3 E: 653 MI: 409 Status: New 45. Private-sector strikes and lockouts typically end because: A) the Federal government intervenes with a "back-to-work" order. B) the parties eventually tire of bickering. C) they are costly in terms of lost profits and lost wage income. D) the parties rely heavily on outside arbitration. Answer: C

Type: F Topic: 3 E: 654 MI: 410 46. Collective bargaining occurs under a framework of rules established in the: A) Sherman Act. C) National Labor Relations Act B) Clayton Act. D) Employer-Employee Dispute Act. Answer: C

Type: F Topic: 3 E: 654 MI: 410 47. A major function of the National Labor Relations Board is to: A) monitor potentially inflationary increases in wages. B) provide compulsory arbitration so that strikes and lockouts do not occur. C) set annual wage increases on an industry-by-industry basis. D) investigate and rule on charges of unfair labor practices by either mangement or unions. Answer: D

McConnell/Brue: Economics, 16/e

Page 1054

Chapter 35: Labor Market Institutions and Issues: Unionism, Discrimination, Immigration

Economic effects of unions

Type: F Topic: 4 E: 654 MI: 410 48. The union wage advantage: A) has consistently been in the 25-30 percent range since the Second World War (1945). B) is estimated to be about 15 percent. C) has steadily increased along with the relative decline in unionism occurring since 1955. D) reached a record-high level in 2003. Answer: B

Type: A Topic: 4 E: 655 MI: 411 49. Which of the following is correct? A) The Federal government can delay any strike for eighty days. B) About 3 percent of total work-time is lost in the United States because of strikes. C) Work-time lost may overstate the cost of a strike if the work stoppage disrupts production in related industries. D) Work-time lost may overstate the cost of a strike if nonstruck firms increase their production. Answer: D

Type: A Topic: 4 E: 656-657 MI: 412-413 50. Those who feel that unions positively affect productivity and efficiency argue that unions are: A) "voice mechanisms" which reduce labor turnover. B) "exit mechanisms" which reduce labor turnover. C) "voice mechanisms" which accelerate labor turnover. D) "exit mechanisms" which accelerate labor turnover. Answer: A

Type: D Topic: 4 E: 656 MI: 412 51. The shock effect refers to the: A) fact that the union wage advantage causes unemployment in nonunion labor markets. B) positive effect that a union wage increase may have on managerial efficiency. C) impact of a strike on nonstruck firms. D) misallocation of labor that results from the union wage advantage. Answer: B

Type: A Topic: 4 E: 656 MI: 412 52. By reducing labor turnover, unions may increase productivity because a lower turnover rate: A) results in a less-experienced work force. B) increases the incentive for firms to provide training to their workers. C) allows firms to employ a greater number of younger, more energetic workers. D) increases the incentive for firms to substitute labor for capital in the production process. Answer: B

McConnell/Brue: Economics, 16/e

Page 1055

Chapter 35: Labor Market Institutions and Issues: Unionism, Discrimination, Immigration

Type: F Topic: 4 E: 655 MI: 411 53. With respect to the effect of unions on productivity, empirical results show: A) productivity is usually higher in specific unionized industries, but economywide productivity is lower. B) unionization reduces productivity. C) unionization improves productivity. D) mixed results; unionization appears to increase productivity in some industries while reducing it in others. Answer: D

Type: A Topic: 4 E: 655 MI: 411 54. Some economists claim that unions reduce economic efficiency by: A) providing a voice mechanism for workers. B) insisting that promotions be based on ability rather than seniority. C) imposing restrictions on the kinds of jobs workers may perform. D) increasing worker turnover. Answer: C

Type: A Topic: 4 E: 656-657 MI: 412-413 55. Unions may increase productivity by: A) providing an exit mechanism for workers. B) increasing worker turnover so that younger workers are more likely to be employed. C) reducing the amount of capital used per worker. D) providing a voice mechanism for workers. Answer: D

Type: D Topic: 4 E: 654-655 MI: 410-411 56. Featherbedding refers to: A) a situation in which a union forces an employer to hire union workers in preference to nonunion workers. B) the requirement that unneeded workers be retained on a job. C) the refusal by one union to handle or transport goods produced by workers in another union. D) disputes among two or more unions as to which will perform certain jobs. Answer: B

Type: A Topic: 4 E: 657 MI: 413 57. Which of the following does not tend to improve the productivity or efficiency of workers? A) the shock effect C) labor market discrimination B) the voice mechanism D) job security through seniority Answer: C

Type: F Topic: 4 E: 656 MI: 412 58. Available research suggests that the union wage advantage diminishes the national output by: A) 10 percent. B) 15 percent. C) 5 percent. D) less than 1 percent. Answer: D

Type: A Topic: 4 E: 656 MI: 412 59. Suppose a worker feels unhappy with his job and consequently quits. This illustrates: A) the voice mechanism. B) featherbedding. C) the exit mechanism. D) blacklisting. Answer: C

McConnell/Brue: Economics, 16/e

Page 1056

Chapter 35: Labor Market Institutions and Issues: Unionism, Discrimination, Immigration

Type: A Topic: 4 E: 656-657 MI: 412-413 60. The voice mechanism is important because it may: A) reduce worker turnover and increase productivity. B) increase worker mobility. C) increase the number of strikes. D) conflict with the Taft-Hartley Act. Answer: A

Use the following to answer questions 61-64:

Assumptions: These two graphs show two sectors of the labor market for a particular kind of labor. Relevant product markets are competitive. The two labor demand curves are identical and initially the quantities of labor employed in the two sectors are L1 and L'1 and the wage rate in each sector is Wn .

Type: G Topic: 4 E: 656 MI: 412 61. Refer to the above diagram and assumptions. If a union is formed in sector 1 and the union increases the wage rate from Wn to Wu, then employment will: A) decrease, but we cannot determine by how much. B) decrease by 0L2 in sector 1. C) decrease by L1L2 in sector 1. D) increase by L1L2 in sector 1. Answer: C

Type: G Topic: 4 E: 656 MI: 412 62. Refer to the above diagrams and assumptions. The change in employment in sector 1 to which a Wn to Wu union wage increase gives rise will cause output in that sector to: A) decrease by A + B. B) decrease by B + C + D. C) decrease by C + D. D) increase by A + B. Answer: B

Type: G Topic: 4 E: 656 MI: 412 63. Refer to the above diagrams and assumptions. If all the workers who lose their jobs in the union sector because of a Wn to Wu union wage increase are reemployed in nonunion sector 2, output in that sector will: A) decrease by F + G. B) increase by F + G. C) increase by E + F. D) increase by E. Answer: B

McConnell/Brue: Economics, 16/e

Page 1057

Chapter 35: Labor Market Institutions and Issues: Unionism, Discrimination, Immigration

Type: G Topic: 4 E: 656 MI: 412 64. Refer to the above diagrams and assumptions. This analysis suggests that a union wage advantage of Wu minus Ws causes a net efficiency: A) loss equal to E - A. B) loss equal to C. C) gain equal to C. D) gain equal to B. Answer: B

Type: F Topic: 4 E: 656 MI: 412 65. Empirical studies suggest that the efficiency loss associated with the misallocation of labor caused by the union wage advantage is: A) $6 billion per year. B) about 2 percent of domestic output. C) less than one-half of one percent of domestic output. D) about 4 percent of domestic output. Answer: C

Type: A Topic: 4 E: 655 MI: 411 66. Critics of unions argue that unions diminish efficiency and productivity by: A) engaging in featherbedding. C) causing a misallocation of labor. B) precipitating strikes. D) doing all of the above. Answer: D

Type: A Topic: 4 E: 654 MI: 410 67. Which of the following statements is correct? A) There is clear evidence that unions have increased the average level of real wage rates for all workers. B) When fringe benefits are taken into account, the overall compensation of union and nonunion workers is equal. C) Approximately 10 percent of all work time is lost annually because of strikes. D) Featherbedding means that unions may require employers to hire unneeded workers. Answer: D

Type: A Topic: 4 E: 656 MI: 412 68. Which one of the following research findings is most consistent with the hypothesis that unions increase productivity? A) Other things equal, firm profits are lower where unions are present. B) Union workers receive, on average, higher fringe benefits relative to wages than nonunion workers. C) The average amount of work time lost annually to strikes is surprisingly small. D) Labor turnover is less in unionized firms than in nonunionized firms. Answer: D

Types of discrimination

Type: F Topic: 5 E: 657 MI: 413 69. About ____ percent of the pay differences between men and women and between whites and blacks is attributable to nondiscriminatory factors. A) 10 B) 20. C) 50. D) 80. Answer: C

McConnell/Brue: Economics, 16/e

Page 1058

Chapter 35: Labor Market Institutions and Issues: Unionism, Discrimination, Immigration

Type: F Topic: 5 E: 657 MI: 413 70. About _____ percent of the pay differences between men and women and between whites and blacks cannot be accounted for by nondiscriminatory factors and this is thought to reflect discrimination. A) 20. B) 50. C) 70. D) 80. Answer: B

Type: A Topic: 5 E: 657 MI: 413 71. Black worker Jackson earns $30,000 per year; white worker Johnson earns $40,000 per year. On the basis of this information we can conclude that: A) it is not clear whether discrimination is present in this situation. B) there are no discriminatory forces at work in the labor market. C) Jackson is being discriminated against, but the dollar amount is unclear. D) Jackson is being discriminated against in the amount of $10,000 per year. Answer: A

Type: A Topic: 5 E: 658 MI: 414 72. Suppose a company promotes Ronald Robinson rather than more-qualified Samantha Shubert. This action primarily illustrates: A) wage discrimination. C) employment discrimination. B) human-capital discrimination. D) occupational discrimination. Answer: C

Type: A Topic: 5 E: 658 MI: 414 73. In 2001 the percentage of black men who had completed college was 16 percent compared to 30 percent of white men. This primarily suggests the possibility of: A) wage discrimination. C) employment discrimination. B) human-capital discrimination. D) occupational discrimination. Answer: B

Type: A Topic: 5 E: 658 MI: 414 74. In a certain year about 31 percent of the white labor force held professional-technical-managerial positions compared to 15 percent of the Hispanic labor force. This suggests: A) wage discrimination. C) reverse discrimination. B) statistical discrimination. D) occupational discrimination Answer: D

Type: D Topic: 5 E: 657-658 MI: 413-414 75. Discrimination that results in the payment of a lower wage to a woman relative to an equally productive man on the same job is called: A) wage discrimination. C) reverse discrimination. B) human-capital discrimination. D) occupational discrimination. Answer: A

Type: D Topic: 5 E: 658 MI: 414 76. Discrimination in the form of access barriers to productivity-increasing opportunities such as education and training is called: A) wage discrimination. C) employment discrimination. B) human-capital discrimination. D) occupational discrimination. Answer: B

McConnell/Brue: Economics, 16/e

Page 1059

Chapter 35: Labor Market Institutions and Issues: Unionism, Discrimination, Immigration

Costs of discrimination

Type: A Topic: 6 E: 659 MI: 415 77. Labor market discrimination creates a: A) redistribution of a larger domestic output. B) larger domestic output but no redistribution. Answer: D

C) smaller domestic output but no redistribution. D) redistribution of a smaller domestic output.

Type: A Topic: 6 E: 659 MI: 415 78. Economic discrimination: A) affects the distribution of domestic output and income, but not its total size. B) is shown as some point outside of an economy's production possibilities curve. C) places the economy at some point inside of its production possibilities curve. D) affects the total size of domestic output and income, but not its distribution. Answer: C

Type: A Topic: 6 E: 659 MI: 415 79. Economic discrimination puts the economy inside its production possibilities curve because discrimination: A) redistributes income from low-paid to high-paid persons. B) promotes present consumption rather than production of capital goods. C) arbitrarily blocks women and certain minorities from higher-productivity, higher wage jobs and thus keeps the economy from producing its maximum output. D) often causes inflation, which reduces the nation's real output. Answer: C

Use the following to answer questions 80-81:

Type: G Topic: 6 E: 659 MI: 415 80. In the above diagram, economic discrimination is best represented by point: A) A B) C rather than D or E. C) E rather than D or C. D) F Answer: A

McConnell/Brue: Economics, 16/e

Page 1060

Chapter 35: Labor Market Institutions and Issues: Unionism, Discrimination, Immigration

Type: G Topic: 6 E: 659 MI: 415 81. In the above diagram, the elimination of discrimination is best represented by: A) a move from C to E. B) an inward shift of the production possibilities curve. C) a move from A to D. D) a move from E to C. Answer: C

Taste for discrimination model

Type: A Topic: 7 E: 659 MI: 415 82. A particular employer's discrimination coefficient d: A) will decrease if the employer becomes more prejudiced against blacks. B) must equal the actual ratio of black to white wage rates. C) measures the amount an employer is willing to pay to hire a white rather than a black worker. D) varies inversely with the actual black-white wage ratio. Answer: C

Type: A Topic: 7 E: 659 MI: 415 83. Which of the following employers is the most prejudiced? Employer: A) A whose d is $0. B) B whose d is $2. C) C whose d is $4. D) D whose d is $6. Answer: D

Type: A Topic: 7 E: 659 MI: 415 84. In the taste-for-discrimination model: A) white employers behave as if employing black workers adds to costs. B) individual workers are judged by the characteristics of the groups to which they belong. C) prejudiced white employers will never hire black workers. D) women and minorities are confined to a limited number of occupations. Answer: A

Type: A Topic: 7 E: 659 MI: 415 85. Assume that all workers are equally productive but the wage rate for men is $12 compared to $9 for women. An employer who employs only male workers must have a discrimination coefficient of: A) more than $3. B) at most .3. C) less than $3. D) 1.33 or less Answer: A

Type: A Topic: 7 E: 659 MI: 415 86. An employer whose discrimination coefficient is $4 will: A) refuse to hire blacks at any wage rate. B) hire only blacks if the actual black-white wage differential is $3 an hour. C) hire only whites if the actual black-white wage differential is $3 an hour. D) hire only whites if the black-white wage differential is $4. Answer: C

McConnell/Brue: Economics, 16/e

Page 1061

Chapter 35: Labor Market Institutions and Issues: Unionism, Discrimination, Immigration

Type: A Topic: 7 E: 660 MI: 416 87. A employer whose discrimination coefficient is zero will: A) refuse to hire blacks at any wage rate. B) hire only whites if black and white wages are the same. C) randomly hire black and white workers if the actual black-white wage differential is also zero. D) refuse to hire whites at any wage rate. Answer: C

Type: A Topic: 7 E: 659 MI: 415 88. An employer whose discrimination coefficient is $2 will: A) randomly hire black and white workers if the actual black-white wage differential is more than $2. B) randomly hire black and white workers if the actual black-white wage differential is less than $2. C) hire only blacks if the actual black-white wage differential is less than $2. D) hire only whites if the actual black-white wage differential is less than $2. Answer: D

Type: A Topic: 7 E: 659 MI: 415 89. Suppose that wages for black and white workers of equal productivity are $12 and $13 an hour, respectively. If a particular firm hires only whites, its discrimination coefficient must be: A) greater than $1. B) less than $1. C) zero. D) infinity. Answer: A

Type: C Topic: 7 E: 660 MI: 416 90. An increase in the collective discrimination coefficients of employers will: A) reduce the black wage rate, increase black employment, and lower the actual black-white wage ratio. B) reduce the black wage rate, decrease black employment, and lower the actual black-white wage ratio. C) increase the black wage rate, increase black employment, and increase the actual black-white wage ratio. D) increase the black wage rate, reduce black employment, and increase the actual black-white wage ratio. Answer: B

Type: C Topic: 7 E: 660 MI: 416 91. A reduction in the collective discrimination coefficients of employers will. A) reduce the black wage rate, increase black employment, and lower the actual black-white wage ratio. B) reduce the black wage rate, decrease black employment, and lower the actual black-white wage ratio. C) increase the black wage rate, increase black employment, and increase the actual black-white wage ratio. D) increase the black wage rate, reduce black employment, and increase the actual black-white wage ratio. Answer: C

McConnell/Brue: Economics, 16/e

Page 1062

Chapter 35: Labor Market Institutions and Issues: Unionism, Discrimination, Immigration

Use the following to answer questions 92-93:

Type: G Topic: 7 E: 660 MI: 416 92. Refer to the above diagram. Which of the following would be consistent with an increase in racial prejudice and thus an increase in the discrimination coefficients of employers, taken as a group. A) a shift in the labor demand curve from D2 to D3. B) a shift in the labor demand curve from D2 to D1. C) an increase in black employment from Q2 to Q3. D) an increase in the black wage rage from W1 to W2. Answer: B

Type: G Topic: 7 E: 660 MI: 416 93. Refer to the above diagram. Which of the following would be consistent with a decrease in racial prejudice and thus in the discrimination coefficients of employers, taken as a group? A) a shift in the labor demand curve from D3 to D2. B) an increase in the black wage rate from W1 to W2. C) a reduction in black employment from Q3 to Q2. D) a decrease in the black wage rate from W2 to W1. Answer: B

Type: A Topic: 7 E: 659 MI: 415 94. In the taste-for-discrimination model: A) a decline in discrimination will reduce the actual black-white wage ratio. B) an increase in collective discrimination coefficients of employers will reduce the demand for black workers, decrease the black wage, and increase black employment. C) firms that discriminate will have lower costs than firms that do not discriminate. D) competitive forces will tend to reduce discrimination in the very long run. Answer: D

McConnell/Brue: Economics, 16/e

Page 1063

Chapter 35: Labor Market Institutions and Issues: Unionism, Discrimination, Immigration

Type: A Topic: 7 E: 659-660 MI: 415-416 95. An implication of the taste-for-discrimination model is that: A) discrimination can lower a firm's production costs. B) discrimination will move a firm along its declining average total cost curve. C) other things equal, nondiscriminating firms will have lower production costs than discriminating firms. D) other things equal, discriminating firms will have lower production costs than nondiscriminating firms. Answer: C

Type: A Topic: 7 E: 659 MI: 415 96. An employer is prejudiced, prefers to hire white rather than black workers, and is willing to pay higher wages to obtain white workers. This illustrates: A) reverse discrimination. C) the taste-for-discrimination model. B) the crowding model. D) statistical discrimination. Answer: C

Statistical discrimination

Type: D Topic: 8 E: 661 MI: 417 97. Statistical discrimination refers to: A) the crowding of women or minorities into low-paying occupations. B) significant differences in average levels of earnings by gender, race, and ethnicity, after accounting for nondiscriminatory factors. C) making individual hiring decisions on the basis of the characteristics of the group to which a person belongs, rather than on his or her personal characteristics and productivity. D) the 50-percent unexplained residual in studies that try to account for wage differences by gender, race, and ethnic origin. Answer: C

Type: A Topic: 8 E: 661 MI: 417 98. Statistical discrimination: A) is malicious. B) is profitable, but not malicious. Answer: B

C) cannot persist because of competition. D) explains all occupational discrimination.

Type: A Topic: 8 E: 661 MI: 417 99. Insurance companies require male drivers under age 25 to pay higher insurance rates than female drivers under age 25. Craig Raymond, however, is a safer driver than the average female driver under age 25. Craig's higher insurance rate reflects: A) monopoly power. C) the insurance firm's taste for discrimination. B) statistical discrimination. D) human-capital discrimination. Answer: B

McConnell/Brue: Economics, 16/e

Page 1064

Chapter 35: Labor Market Institutions and Issues: Unionism, Discrimination, Immigration

Type: A Topic: 8 E: 661 MI: 417 100. Statistical discrimination: A) can persist in the long run if differences in average characteristics among groups continue. B) will tend to diminish in the long run, because nondiscriminating firms will drive out discriminating firms. C) requires that employers have discrimination coefficients greater than zero. D) is also known as occupational segregation. Answer: A

Type: A Topic: 8 E: 661 MI: 417 101. Which of the following is an example of statistical discrimination? A) An employer hires only white workers even though there are otherwise identical black workers available at lower pay. B) Women students in college business schools are overrepresented in human resource management courses and underrepresented in finance courses. C) A young woman who plans to work for only five to seven years after graduating college decides that getting an advanced degree "just won't pay off." D) A firm hires a man rather than a woman for a specific job because, on average, women have higher rates of absenteeism than do men. Answer: D

Type: A Topic: 8 E: 661 MI: 417 102. Suppose Gigantic Health Cooperative doesn't hire Mandy Morrison solely because she is a graduate of a particular foreign medical school, which is not known for the high quality of its graduates. Yet, Mandy is actually more qualified than the average graduate of American medical schools. This is an example of: A) monopsony exploitation. C) statistical discrimination. B) human-capital discrimination. D) irrational behavior. Answer: C

Type: A Topic: 8 E: 661 MI: 417 103. Suppose Copernicus Corporation hires Damon Danielson rather than Elmer Elliason, simply because Damon is a graduate of an Ivy League school while Elmer is a graduate of a small public university. Elmer is actually more qualified than Damon. This is an example of: A) statistical discrimination. C) monopsony exploitation. B) irrational behavior. D) human-capital discrimination. Answer: A

Type: A Topic: 8 E: 661 MI: 417 104. A particular woman is denied on-the-job training because women on average are more likely to drop out of the workforce than men. This illustrates: A) wage discrimination. C) the taste-for-discrimination model. B) the crowding model. D) statistical discrimination. Answer: D

McConnell/Brue: Economics, 16/e

Page 1065

Chapter 35: Labor Market Institutions and Issues: Unionism, Discrimination, Immigration

Crowding model

Type: A Topic: 9 E: 661 MI: 417 105. The crowding model of discrimination suggests that: A) women and selected minorities are systematically excluded from high-paying occupations and crowded into low-paying occupations, decreasing their wages and reducing domestic output. B) employers having high discrimination coefficients will be crowded out by nondiscriminating employers in the long run. C) firms will base hiring decisions on group averages, rather than on individual characteristics and productivity. D) occupational segregation is largely the result of freely made rational choices of women and minorities. Answer: A

Type: A Topic: 9 E: 661-662 MI: 417-418 106. The crowding of women and minorities into a restricted number of occupations: A) has no impact on the size of the domestic output or its distribution in the long run. B) will increase the size of the domestic output and make its distribution more equal. C) will decrease the size of the domestic output and make its distribution less equal. D) will increase the size of the domestic output, but make its distribution less equal. Answer: C

Type: A Topic: 9 E: 661 MI: 417 107. A minority person is prohibited from entering certain high paying occupations. This illustrates: A) human-capital discrimination C) the taste-for-discrimination model. B) the crowding model. D) statistical discrimination. Answer: B

Type: A Topic: 9 E: 661 MI: 417 108. The crowding model is primarily concerned with explaining: A) occupational discrimination. B) human-capital discrimination. C) the size of the discrimination coefficient. D) why affirmative action is reverse discrimination. Answer: A

McConnell/Brue: Economics, 16/e

Page 1066

Chapter 35: Labor Market Institutions and Issues: Unionism, Discrimination, Immigration

Use the following to answer questions 109-113:

Assumptions: (1) the labor force is comprised of 9 million men and 9 million women workers; (2) the economy has 3 occupations, X, Y, and Z, each having identical demand curves for labor; (3) men and women workers are homogeneous with respect to their labor-market capabilities; (4) women are discriminated against by being excluded from occupations X and Y and are confined to Z; and (5) aside from discrimination, the economy is competitive.

Type: G Topic: 9 E: 662 MI: 418 109. Refer to the above diagram and list of assumptions. Under these circumstances 9 million women will be employed in occupation Z: A) 5 million men in X, and 4 million men in Y. B) 3 million men in XD, and 6 million men in Y. C) 6 million men in X, and 3 million men in Y. D) and 4.5 million men each in occupations X and Y. Answer: D

Type: G Topic: 9 E: 662 MI: 418 110. Refer to the above diagram and list of assumptions. The allocation of employment determined in the previous question will result in which of the following wage structures? A) Men and women will both be paid $5. C) Men will be paid $6 and women $3. B) Men will be paid $5 and women $4. D) Women and men will both be paid $6. Answer: C

Type: G Topic: 9 E: 662 MI: 418 111. Refer to the above diagram and list of assumptions. If discrimination is ended: A) men will leave occupations X and Y and enter occupation Z. B) 4 million women will leave occupation Z, with 2 million entering occupation X and 2 million entering occupation Y. C) 3 million women will leave occupation Z, with 1.5 million entering occupation X and 1.5 million entering occupation Y. D) 3 million women will leave occupation Z, all of whom will enter industry X. Answer: C

McConnell/Brue: Economics, 16/e

Page 1067

Chapter 35: Labor Market Institutions and Issues: Unionism, Discrimination, Immigration

Type: G Topic: 9 E: 662 MI: 418 112. Refer to the above diagram and list of assumptions. If discrimination is ended, women: A) and men will both receive a wage rate of $4. B) and men will both receive a wage rate of $5. C) and men will both receive a wage rate of $6. D) will receive a wage rate of $4 and men will receive a wage rate of $5. Answer: B

Type: G Topic: 9 E: 662 MI: 418 113. Refer to the above diagram and list of assumptions. The elimination of gender discrimination: A) may either increase or reduce real domestic output, depending on what happens to the level of wages.. B) will increase real domestic output. C) will have no effect on real domestic output. D) will reduce real domestic output. Answer: C

Type: A Topic: 9 E: 662 MI: 418 114. As applied to gender discrimination, the crowding model of occupational segregation: A) helps explain why women earn more than men. B) predicts that men's wages would fall and women's wages would rise if occupational segregtion was eliminated. C) predicts that domestic output would decline if occupational segregation was ended. D) predicts that competition will eventually totally end discrimination. Answer: B

Antidiscrimination policies and controversies

Type: A Topic: 10 E: 663 MI: 419 115. A strong, growing economy accompanied by a brisk expansion of labor demand reduces discriminatory outcomes by: A) making it more costly for employers to exercise their tastes for discrimination. B) boosting the black-white and female-male pay gaps, thus increasing the incentive of less-biased firms to hire blacks and women. C) increasing saving and investment. D) increasing the labor supply of women and minorities. Answer: A

Type: A Topic: 10 E: 663 MI: 419 116. Wage and employment discrimination were expressly outlawed in the: A) Comparable Worth Act of 1975 and the Affirmative Action Act of 1976. B) Labor Market Discrimination Act of 1965 and Civil Rights Act of 1970. C) Equal Pay Act of 1963 and Civil Rights Act of 1964. D) Civil Rights Act of 1984 and Labor Market Discrimination Act of 1985. Answer: C

McConnell/Brue: Economics, 16/e

Page 1068

Chapter 35: Labor Market Institutions and Issues: Unionism, Discrimination, Immigration

Type: F Topic: 10 E: 663 MI: 419 117. Affirmative action plans were mandated under: A) the Equal Pay Act of 1963. B) the Civil Rights Act of 1964. Answer: D

C) the Comparable Worth Act of 1985. D) a series of Executive Orders in the 1960s.

Type: A Topic: 10 E: 663-664 MI: 419-420 118. Which of the following is an argument in support of affirmative action plans? A) The female-male wage ratio has increased significantly in the past two decades. B) Women now have lower unemployment rates than men. C) Something more than equal treatment is necessary for women and selected minorities to overcome centuries of discrimination and occupational segregation. D) Preferential treatment is itself a form of discrimination; showing preference for A is to discriminate against B. Answer: C

Type: A Topic: 10 E: 664 MI: 420 119. Which of the following is an argument in opposition to affirmative action plans? A) Preferential treatment is itself a form of discrimination; showing preference for A is to discriminate against B. B) Market wages provide a better indication of female and minority job worth than job evaluations by administrators. C) Affirmative action plans are required to overcome subtle discrimination that otherwise would occur in hiring. D) Discrimination in labor markets is on the rise. Answer: A

Immigration

Type: F Topic: 11 E: 665 MI: 421 120. About ________ of recent annual population growth in the United States is the result of immigration. A) one-tenth B) one-fifth C) one-third D) one-half. Answer: C

Type: F Topic: 11 E: 665 MI: 421 121. In 2001 the greatest number of immigrants arriving in the United States came from: A) India and Brazil. C) China and Vietnam. B) the Dominican Republic and Cuba. D) Mexico and India. Answer: D

Type: F Topic: 11 E: 665 MI: 421 122. The U.S. Census Bureau estimates that the net inflow of illegal immigrants is now about: A) 5,000 annually. B) 100,000 annually. C) 250,000 annually. D) 400,000 annually. Answer: B

McConnell/Brue: Economics, 16/e

Page 1069

Chapter 35: Labor Market Institutions and Issues: Unionism, Discrimination, Immigration

Use the following to answer questions 123-129:

Assumptions: (1) The demand for labor in Alphania and Betania are as shown by DA and DB, respectively; (2) Alphania's native labor force is F and that of Betania is g; and (3) full employment exists in both countries.

Type: G Topic: 11 E: 666 MI: 422 123. Refer to the above diagram and assumptions. If migration is costless and unimpeded: A) migration will cause the average level of wages to fall in Alphania. B) no migration will occur. C) fg workers will move from Betania to Alphania. D) GF workers will move from Alphania to Betania. Answer: D

Type: G Topic: 11 E: 666 MI: 422 124. Refer to the above diagram and assumptions. If migration is costless and unimpeded, the absolute wage bill will necessarily: A) increase in Alphania if its labor demand curve is elastic. B) increase in Betania if its labor demand curve is elastic. C) decrease in Betania. D) increase in Betania. Answer: B

Type: G Topic: 11 E: 666 MI: 422 125. Refer to the above diagram and assumptions. If migration is costless and unimpeded, business income will: A) decrease in Betania, but increase in Alphania. C) decrease in both Alphania and Betania. B) increase in Betania, but decrease in Alphania. D) increase in both Alphania and Betania. Answer: B

Type: G Topic: 11 E: 666 MI: 422 126. Refer to the above diagram and assumptions. If migration is costless and unimpeded, the average level of wages will: A) decrease in Betania, but remain unchanged in Alphania. B) increase in Alphania, but remain unchanged in Betania. C) increase in Alphania and decrease in Betania. D) increase in Betania and decrease in Alphania. Answer: C

McConnell/Brue: Economics, 16/e

Page 1070

Chapter 35: Labor Market Institutions and Issues: Unionism, Discrimination, Immigration

Type: G Topic: 11 E: 666 MI: 422 127. Refer to the above diagram and assumptions. After migration has ceased: A) world output will have increased by mljh - MLJH. B) Betania's output will have increased and Alphania's output will have decreased, but world output will not have changed. C) world output will have increased by gjhf - GJHF. D) world output will have decreased by gjhf - GJHF. Answer: C

Type: G Topic: 11 E: 666 MI: 422 128. Refer to the above diagram and assumptions. We would expect a flow of remittances from migrants to: A) decrease in national income loss in Betania. B) increase the national income gain in Betania. C) decrease the national income loss in Alphania. D) decrease the national income gains in both countries. E) increase the national income gains in both countries. Answer: C

Type: G Topic: 11 E: 666 MI: 422 129. Refer to the above diagram and assumptions. If unemployment, rather than full employment, had initially existed in Alphania: A) then Betania's loss of output would have been greater. B) then Betania's gain of output would have been less. C) its loss of output would have been less. D) its loss of output would have been greater. Answer: C

Use the following to answer questions 130-135:
Zinnia Q 1 2 3 4 W $20 18 16 14 VTP $20 38 54 68 Q 1 2 3 4 Marigold W $16 14 12 10 VTP $16 30 42 52

Symbols: Q = number of workers demanded; W = wage rate; and VTP = value of the cumulative total product of the particular number of workers. Assumptions: The current wage in Zinnia is $20 and the current wage in Marigold is $12.

Type: T Topic: 11 E: 666 MI: 422 Status: New 130. Refer to the above data, symbols, and assumptions. Both nations have: A) identical labor demand curves. C) downsloping labor demand curves. B) perfectly elastic labor demand curves. D) upsloping labor demand curves. Answer: C

McConnell/Brue: Economics, 16/e

Page 1071

Chapter 35: Labor Market Institutions and Issues: Unionism, Discrimination, Immigration

Type: T Topic: 11 E: 666 MI: 422 Status: New 131. Refer to the above data, symbols, and assumptions. The combined number of workers in the two nations is: A) 2. B) 3. C) 5. D) 4. Answer: D

Type: T Topic: 11 E: 666 MI: 422 Status: New 132. Refer to the above data, symbols, and assumptions. If migration is costless and unimpeded: A) no migration will occur. B) migration will cause the wage in Marigold to fall. C) 2 workers will move from Marigold to Zinnia. D) 4 workers will move from Marigold to Zinnia. Answer: C

Type: T Topic: 11 E: 666 MI: 422 Status: New 133. Refer to the above data, symbols, and assumptions. If migration is costless and unimpeded, the wage in both countries will equalize at: A) $16. B) $18. C) $20. D) $14. Answer: A

Type: T Topic: 11 E: 666 MI: 422 Status: New 134. Refer to the above data, symbols, and assumptions. If migration is costless and unimpeded, the combined value of total product in the two countries will: A) decline from $62 to $36. C) increase from $36 to $62. B) decline from $120 to $70. D) increase from $62 to $70. Answer: D

Type: T Topic: 11 E: 666 MI: 422 Status: New 135. Refer to the above data, symbols, and assumptions. Migration of workers will: A) increase the combined value of total product but reduce the wage in Zinnia. B) increased the combined value of total product but reduce the wage in Marigold. C) reduce the combined value of total product but increase the wage in Marigold. D) reduce the combined value of total product but increase the wage in Zinnia. Answer: A

Type: A Topic: 11 E: 666-667 MI: 422-423 136. Other things equal, the voluntary relocation of employable migrants from low-paying nations to highpaying nations will: A) increase business or capitalist incomes in the low-paying nations. B) reduce real output in the world. C) increase business or capitalist incomes in the high-paying nations. D) reduce wage rates in the low-paying nations. Answer: C

Type: A Topic: 11 E: 667 MI: 423 137. The voluntary relocation of employable migrants from low-paying nations to high-paying nations reduces: A) wage rate disparities among nations. C) labor productivity in the world. B) business or capitalist income in the world. D) total wage income in the world. Answer: A

McConnell/Brue: Economics, 16/e

Page 1072

Chapter 35: Labor Market Institutions and Issues: Unionism, Discrimination, Immigration

Type: A Topic: 11 E: 667 MI: 423 138. If the elasticity of demand for labor in the United States is unitary, immigration into the United States can be expected to: A) increase the average U.S. wage rate. B) decrease the total amount of wage earnings that U.S. workers receive. C) increase the total amount of wage earnings that U.S. workers receive. D) leave the total amount of wage earnings that U.S. workers receive unchanged. Answer: D

Type: A Topic: 11 E: 667 MI: 423 139. Voluntary migration of skilled craftworkers from low-paying to high-paying nations is most likely to be opposed by: A) business groups in the high-paying nations. B) craft workers who stay in the low-paying nations. C) industrial unions in the high-paying nations. D) craft unions in the high-paying nations. Answer: D

Type: A Topic: 11 E: 667 MI: 423 140. The marginal cost-marginal benefit approach to migration suggests that: A) the optimal amount of immigration to the United States is probably zero. B) everyone who wishes to come to the United States should be allowed to do so. C) a particular level of immigration may be too low or too high. D) efforts to reduce the inflow of illegal immigrants should be abandoned. Answer: C

Last Word Questions

Type: A E: 668 MI: 424 141. (Last Word) "Orchestrating Impartiality" deals with: A) the affirmative action controversy. B) successful efforts to reduce discrimination in hiring. C) establishing "color blind" immigration policies. D) the difficulties government has in establishing the comparable worth of different jobs. Answer: B

Type: F E: 668 MI: 424 142. (Last Word) Today, about: A) 75 percent of the membership of top symphony orchestras in the United States are women, compared to 25 percent in 1970. B) 10 percent of the membership of top symphony orchestras in the United States are women, compared to 20 percent in 1970. C) 25 percent of the membership of top symphony orchestras in the United States are women, compared to 5 percent in 1970. D) 50 percent of the membership of top symphony orchestras in the United States are women, the same percentage as in 1970. Answer: C

McConnell/Brue: Economics, 16/e

Page 1073

Chapter 35: Labor Market Institutions and Issues: Unionism, Discrimination, Immigration

Type: F E: 668 MI: 424 143. (Last Word) Blind auditioning (through the use of screens separating the musician and the judges) resulted in: A) no change in the gender composition of major symphony orchestras. B) a decrease in the number of women selected for major symphony orchestras. C) a sizable increase in the number of blacks and other minorities selected for major symphony orchestras. D) a sizable increase in the number of women selected for major symphony orchestras. Answer: D

True/False Questions

144.

Type: F E: 650 MI: 406 Since the mid-1950s, union membership has declined as a percentage of employed wage and salary workers. Answer: True

Type: D E: 652 MI: 408 145. The structural-change hypothesis holds that changes in the composition of industry and of the labor force have caused unionism to decline. Answer: True

Type: F E: 651 MI: 407 146. The rate of unionization is substantially higher for protective service workers than for sales workers. Answer: True

Type: A E: 652-653 MI: 408-409 147. Unions prefer agency shops to open shops. Answer: True

Type: F E: 654 MI: 410 148. There is no evidence that unions are able to increase wage rates above those that the market would otherwise provide. Answer: False

Type: F E: 656 MI: 412 149. Union workers have higher rates of job turnover than do nonunion workers. Answer: False

Type: F E: 657 MI: 413 150. Labor market discrimination increases the size of the nation's GDP. Answer: False

McConnell/Brue: Economics, 16/e

Page 1074

Chapter 35: Labor Market Institutions and Issues: Unionism, Discrimination, Immigration

Type: A E: 659 MI: 415 151. An employer (biased against blacks) whose discrimination coefficient is $5 will hire only whites if the actual black-white wage gap is $7. Answer: False

Type: C E: 659 MI: 415 152. A reduction in the collective discrimination coefficients of employers will increase the wage rate of those discriminated against, but reduce their employment. Answer: False

Type: D E: 659 MI: 415 153. Statistical discrimination is also known as reverse discrimination. Answer: False

Type: A E: 660 MI: 416 154. The crowding model of occupational segregation predicts that domestic output will increase if occupational segregation is ended. Answer: True

Type: E E: 663 MI: 419 155. The Civil Rights Act of 1964 mandated that large businesses implement affirmative action plans. Answer: False

Type: A E: 664 MI: 420 156. Critics of affirmative action plans claim that such plans are themselves discriminatory. Answer: True

Type: A E: 665 MI: 421 157. If all nations prohibited the international migration of labor, we would expect world output to decline. Answer: True

Type: C E: 665 MI: 421 158. If the demand for labor in a country receiving immigrants is inelastic, the immigration will increase the total wages paid in that country. Answer: False

Type: A E: 666 MI: 422 159. Business income will decrease in the nation from which workers emigrate. Answer: True

160.

Type: A E: 667 MI: 423 If unemployed workers leave a nation, the standard of living (national income per capita) will rise in that nation. Answer: True

McConnell/Brue: Economics, 16/e

Page 1075

CHAPTER 36

The Economics of Health Care

Topic 1. 2. 3. 4. Health care industry: costs and access Health care market: demand and supply Role of health insurance Health care reform Consider This Last Word True-False

Question numbers 1-24 25-54 55-69 70-85 86-87 88-91 92-113

____________________________________________________________

_______________________________________

____________________________________________________________

_______________________________________

Multiple Choice Questions Health care industry: costs and access

Type: F Topic: 1 E: 672 MI: 428 1. Approximately how many workers are employed in the U.S. health care industry? A) 200,000 B) 1 million C) 4 million D) 9 million Answer: D

Type: A Topic: 1 E: 672-673 MI: 428-429 2. The twin problems of the U.S. health care industry are: A) rapidly rising costs and unequal access to health care. B) declining quality of health care and the duplication of specialized equipment at hospitals. C) declining per capita spending on health care and the moral hazard problem. D) the decline in the number of family physicians and the failure to vaccinate children. Answer: A

Type: A Topic: 1 E: 673 MI: 429 3. The major purpose of Medicare is to: A) provide health care services to the aged. B) provide health care services to those receiving public assistance. C) contain rising health care costs. D) make a basic health care package available to all Americans. Answer: A

Chapter 36: The Economics of Health Care

Type: F Topic: 1 E: 674 MI: 430 4. The major objective of Medicaid is to: A) provide health care services to the aged. B) provide health care services to those receiving public assistance. C) contain rising health care costs. D) make a basic health care package available to all Americans. Answer: B

Type: F Topic: 1 E: 674 MI: 430 5. For most nations, per capita spending on health care tends to: A) remain constant as per capita income increases. C) rise proportionately with per capita income. B) rise much faster than per capita income. D) fall as per capita income increases. Answer: C

Type: F Topic: 1 E: 674 MI: 430 6. As a percentage of GDP, health care spending in the United States has: A) decreased substantially since 1960. C) increased substantially since 1960. B) increased slightly since 1960. D) remained relatively constant since 1960. Answer: C

Type: F Topic: 1 E: 674 MI: 430 7. Aggregate U.S. health care spending in 2001 was about: A) 6 percent of domestic output. C) 14 percent of domestic output. B) 2 percent of domestic output. D) 20 percent of domestic output. Answer: C

Type: F Topic: 1 E: 674 MI: 430 8. The United States devotes about ______ percent of its domestic output to health care. A) 2 B) 5 C) 10 D) 14 Answer: D

Type: F Topic: 1 E: 674 MI: 430 9. Between 1960 and 2001, U.S. health care spending as percentage of domestic output: A) more than doubled. C) declined by one-half. B) more than quadrupled. D) remained relatively constant. Answer: A

Type: F Topic: 1 E: 674 MI: 430 10. As a percentage of GDP, U.S. health care spending is: A) higher than that for Germany and Japan, but lower than that of the United Kingdom and Sweden. B) higher than for any other major industrial country. C) lower than that for Canada. D) nearly identical to that of the other major industrial nations. Answer: B

McConnell/Brue: Economics, 16/e

Page 1078

Chapter 36: The Economics of Health Care

11.

Type: F Topic: 1 E: 674 MI: 430 Status: New U.S. health care spending as a percentage of GDP is projected to rise from 14.1 percent in 2001 to: A) 16.4 percent in 2008. B) 15.3 percent in 2008. C) 14.5 percent in 2008. D) 18.0 percent in 2008. Answer: A

Type: F Topic: 1 E: 674 MI: 430 12. In the past several decades U.S. health care expenditures have: A) risen absolutely, but declined as a percentage of GDP. B) declined absolutely, but risen as a percentage of GDP. C) risen absolutely and as a percentage of GDP. D) declined absolutely and as a percentage of GDP. Answer: C

Type: F Topic: 1 E: 673 MI: 429 13. Roughly what portion of U.S. total health spending is paid for by private and public insurance? A) one-tenth B) one-fourth C) four-fifths D) one-half Answer: C

Type: F Topic: 1 E: 674 MI: 430 14. In recent decades, total health care spending in the United States has: A) increased, but health care spending as a percentage of GDP has decreased. B) increased, and so has health care spending as a percentage of GDP. C) decreased, and so has health care spending as a percentage of GDP. D) decreased, but health care spending as a percentage of GDP has increased. Answer: B

Type: A Topic: 1 E: 675 MI: 431 15. Rapidly rising U.S. health care costs have: A) reduced the ability of many Americans to afford health care. B) slowed the growth of real wages. C) caused Federal and state governments to restrict spending on nonhealth budget items. D) done all of the above. Answer: D

Type: A Topic: 1 E: 675 MI: 431 16. Rapidly rising U.S. health care costs have: A) slowed the growth of real wages. B) diminished labor mobility. C) caused some employers to use more part-time and temporary workers. D) done all of the above. Answer: D

Type: A Topic: 1 E: 675 MI: 431 17. Which of the following statements is correct? A) Limited access to the health care system is a major cause of rising health care costs. B) Rising health care costs are a major cause of limited access to the health care system. C) Rising health care costs have forced employers to raise real wages above labor productivity. D) The tax subsidy which government provides for health care causes health care to be underconsumed. Answer: B

McConnell/Brue: Economics, 16/e

Page 1079

Chapter 36: The Economics of Health Care

Type: A Topic: 1 E: 675-676 MI: 431-432 18. The perceived central economic problem associated with the U.S. health care system is: A) too many frivolous malpractice law suits. B) an overabundance of scanning machines. C) an overallocation of resources to the system. D) that workers lose their insurance when they lose their jobs. Answer: C

Type: A Topic: 1 E: 676 MI: 432 19. The fundamental problem associated with the U.S. health care system is that: A) the financing of health care through insurance has resulted in the underallocation of resources to the health care industry. B) frivolous malpractice suits have increased malpractice insurance premiums for doctors. C) at the margin the value of health care services may be less than the value of alternative goods and services. D) there are too many general practitioners and not enough specialists. Answer: C

Type: D Topic: 1 E: 676 MI: 432 20. When economists say that health care services are overconsumed, they mean that: A) rich people buy too much health care and poor people buy too little. B) some resources now used in the health care industry could produce alternative goods and services that society values more highly. C) health care is being purchased in amounts such that marginal benefits exceed marginal costs. D) the price of health care is below equilibrium so that quantity demanded exceeds quantity supplied. Answer: B

Type: F Topic: 1 E: 676 MI: 432 21. The number of Americans currently without health insurance is approximately: A) 5 million or about 2 percent of the population. B) 10 million or about 4 percent of the population. C) 41 million or about 15 percent of the population. D) 65 million or about 27 percent of the population. Answer: C

Type: F Topic: 1 E: 676 MI: 432 22. Given the availability of the Medicaid program, why are so many poor people uninsured? A) Because only a fixed percentage of the population can participate in Medicaid at any time. B) Because many poor people earn enough that they do not qualify for Medicaid. C) Because nonincome requirements screen many poor people from the program. D) Because only native-born Americans are eligible for the program. Answer: B

Type: F Topic: 1 E: 676 MI: 432 23. Which of the following persons is most likely to be insured for health care? A) A minimum-wage teenager working for a fast-food restaurant. B) A skilled worker employed by a large multinational corporation. C) An unemployed retail clerk. D) A part-time groundskeeper for a small manufacturing plant. Answer: B

McConnell/Brue: Economics, 16/e

Page 1080

Chapter 36: The Economics of Health Care

Type: A Topic: 1 E: 676 MI: 432 24. The unemployed are disproportionately represented among the uninsured because: A) one must be working to qualify for Medicaid. B) most workers obtain health insurance through their employers. C) most are young and in excellent health, so they choose not to purchase health insurance. D) a large percentage of the unemployed are heads of single-parent families. Answer: B

Health care market: demand and supply

Type: A Topic: 2 E: 677 MI: 433 25. The health care market is characterized by: A) important ethical-equity considerations. B) asymmetric information between providers and consumers. C) third-party payment of health care costs. D) all of the above. Answer: D

Type: A Topic: 2 E: 677 MI: 433 26. The health care market is characterized by: A) spillover costs. B) spillover benefits. Answer: B

C) perfect knowledge by both buyers and sellers. D) a perfectly inelastic demand.

Type: A Topic: 2 E: 677 MI: 433 27. In the health care market: A) demand has increased relative to supply. B) supply has increased relative to demand. C) neither demand nor supply has changed significantly in the past two decades. D) the concepts of demand and supply are irrelevant. Answer: A

Type: A Topic: 2 E: 677 MI: 433 28. The price elasticity of demand for health care is such that an increase in the price of health care will: A) decrease total health care expenditures. C) shift the demand for health care rightward. B) increase total health care expenditures. D) shift the demand for health care leftward. Answer: B

Type: A Topic: 2 E: 677 MI: 433 29. The price elasticity of demand for health care is: A) perfectly inelastic. B) relatively inelastic. C) relatively elastic. Answer: B

D) perfectly elastic.

McConnell/Brue: Economics, 16/e

Page 1081

Chapter 36: The Economics of Health Care

Type: A Topic: 2 E: 677 MI: 433 30. The demand for health care in industrially advanced economies is: A) highly elastic with respect to both price and income. B) highly inelastic with respect to both price and income. C) highly elastic with respect to income, but highly inelastic with respect to price. D) about unit elasticity with respect to income and relatively inelastic with respect to price. Answer: D

Type: A Topic: 2 E: 677 MI: 433 31. In industrially advanced countries estimates suggest the income elasticity of demand for health care is about: A) +3. B) +6. C) -2. D) +1. Answer: D

Type: A Topic: 2 E: 677 MI: 433 32. Health care: A) is an inferior good. B) is a normal good. Answer: B

C) is highly elastic with respect to price. D) has a price elasticity of 1.

Type: A Topic: 2 E: 677 MI: 433 33. In industrially advanced countries the price elasticity of demand for health care is about: A) 2.0. B) 0.2. C) 4.5. D) 1.0. Answer: B

Type: A Topic: 2 E: 677 MI: 433 Status: New 34. Other things equal, a 10 percent increase in the price of health care in the United States will reduce the quantity of health care demanded by about: A) 1 percent. B) 2 percent. C) 5 percent. D) 10 percent. Answer: B

Type: A Topic: 2 E: 677 MI: 433 Status: New 35. Other things equal, a 10 percent increase in income in the United States will increase the quantity of health care demanded by: A) 2 percent. B) 6-8 percent. C) 10-15 percent. D) 25 percent. Answer: C

Type: A Topic: 2 E: 677 MI: 433 Status: New 36. The demand for health care in the United States is price: A) elastic and, according to some studies, income inelastic. B) elastic and, according to some studies, income elastic. C) inelastic and, according to some studies, income inelastic. D) inelastic and, according to some studies, income elastic. Answer: D

McConnell/Brue: Economics, 16/e

Page 1082

Chapter 36: The Economics of Health Care

Type: A Topic: 2 E: 677-678 MI: 433-434 37. Which of the following is a demand-increasing factor in the health care market? A) rising incomes B) the aging of the population C) asymmetric information Answer: D

D) all of the above

Type: D Topic: 2 E: 678 MI: 434 38. Defensive medicine refers to the idea that: A) it is more cost-efficient to prevent illnesses than to cure them. B) physicians may require unnecessary testing as a means of protecting themselves against malpractice suits. C) doctors know much more about diagnosing and treating illnesses than do health care consumers. D) physicians do not advertise their services or fees. Answer: B

Type: A Topic: 2 E: 678 MI: 434 39. Defensive medicine: A) solves the moral hazard problem. B) increases the demand for health care. Answer: B

C) solves the principal-agent problem. D) is the same as preventive medicine.

Type: D Topic: 2 E: 677 MI: 433 40. The problem of asymmetric information is that: A) neither health care buyers nor providers are well-informed. B) health care providers are well-informed, but buyers are not. C) the outcomes of many complex medical procedures cannot be predicted. D) insurance companies are well-informed but policy purchasers are not. Answer: B

Type: A Topic: 2 E: 677 MI: 433 41. A major implication of asymmetric information is that: A) health care suppliers may reduce the supply of health care. B) health care suppliers may increase the demand for health care. C) collusion between health care suppliers and purchasers may accelerate the rise in costs. D) resources may be underallocated to the health care industry. Answer: B

Type: A Topic: 2 E: 681 MI: 437 42. If labor productivity in the health care industry rises very slowly relative to wages and salaries in the industry, this would tend to: A) increase the demand for health care. C) increase the supply of health care. B) decrease the demand for health care. D) decrease the supply of health care. Answer: D

McConnell/Brue: Economics, 16/e

Page 1083

Chapter 36: The Economics of Health Care

Type: A Topic: 2 E: 682 MI: 438 43. According to most experts, which of the following factors is most important in causing health care costs to rise? A) the aging of the population B) rising incomes C) malpractice suits D) fee-for-service health insurance and cost-increasing technology. Answer: D

Type: F Topic: 2 E: 682 MI: 438 44. Which of the following is correct? A) The number of physicians per 100,000 of population has been decreasing. B) The average income of physicians is the highest of all major professions. C) As compared to other major professions, physicians earn the highest rate of return on their educational investments. D) The "target income hypothesis" suggests that an increase in the supply of physicians would reduce patient fees. Answer: B

Type: A Topic: 2 E: 682 MI: 438 45. Economists argue that in treating patients: A) physicians should use any test or procedures that might help the patient. B) health care should not be rationed by physicians because it is an entitlement or right. C) both benefits and costs should be taken into account by physicians. D) physicians should use any test or procedure whose marginal benefit is positive. Answer: C

Use the following to answer questions 46-47:

Type: G Topic: 2 E: 680 MI: 436 46. Refer to the above diagram. Economists would argue that health care should be provided to patients in: A) some amount less than Q1. B) amount Q1. C) some amount between Q1 and Q2. D) amount Q2. Answer: B

McConnell/Brue: Economics, 16/e

Page 1084

Chapter 36: The Economics of Health Care

Type: G Topic: 2 E: 680 MI: 436 47. Refer to the above diagram. Physicians likely would argue that health care should be provided to patients in: A) some amount less than Q1. B) amount Q1. C) some amount between Q1 and Q2. D) amount Q2. Answer: D

Type: A Topic: 2 E: 681 MI: 437 48. Which of the following is a supply factor in the health care market? A) defensive medicine B) the aging of the population C) slow productivity growth in the health care industry D) asymmetric information Answer: C

Type: A Topic: 2 E: 682 MI: 438 Status: New 49. Health care costs have greatly increased in recent years in the United States. This fact alone does not establish an overallocation of resources to health care because: A) the benefits of health care have also greatly increased in recent years. B) rising medical care prices have inflated health care costs. C) the marginal cost of health care exceeds the average total cost of health care. D) negative externalities sometime result from additional health care spending. Answer: A

Use the following to answer questions 50-54:

P

S1 S2 S3

D3 D1 0 Q
Type: G Topic: 2 E: 677 MI: 433 Status: New 50. Refer to the above diagram that shows the market for U.S. health care. Other things equal, which of the following would shift the demand curve for medical care from D1 to D2? A) higher copayments and deductibles in insurance policies. B) an aging population. C) cutbacks in the government's Medicaid program. D) restrictions by firms on the eligibility of part-time workers for medical benefits. Answer: B

McConnell/Brue: Economics, 16/e

Page 1085

Chapter 36: The Economics of Health Care

Type: G Topic: 2 E: 681 MI: 437 Status: New 51. Refer to the above figure that shows the market for U.S. health care. Other things equal, which of the following would shift the supply curve for medical care from S1 to S2? A) greater reliance on skilled nurses rather than physicians. B) higher insurance copayments and deductibles. C) higher physician fees. D) higher medical malpractice insurance premiums. Answer: A

Type: G Topic: 2 E: 677 MI: 433 Status: New 52. Refer to the above figure that shows the market for U.S. health care. The problem of rapidly rising medical care costs is best illustrated by the shifts of the demand and supply curves from D1 to: A) D2 and S1 to S2. B) D2 and S1 to S3. C) D3 and S1 to S2. D) D3 and S1 to S3. Answer: C

Type: G Topic: 2 E: 677 MI: 433 Status: New 53. Refer to the above figure that shows the market for U.S. health care. Other things equal, which of the following would shift the demand curve for medical care from D2 to D1? A) an aging population. B) loss of medical insurance benefits by part-time workers and early retirees. C) expansion of the Medicaid program. D) higher medical malpractice premiums. Answer: B

Type: G Topic: 2 E: 681 MI: 437 Status: New 54. Refer to the above figure that shows the market for U.S. health care. Other things equal, which of the following would shift the supply curve for medical care from S2 to S1? A) an aging population. B) cost-increasing medical technology. C) expansion of the Medicare program to include prescription drugs. D) lower medical malpractice insurance premiums. Answer: B

Role of health insurance

Type: A Topic: 3 E: 679 MI: 435 55. If an individual is less inclined to avoid accidents or illness because she has health insurance, this is an example of: A) the free-rider problem. C) the adverse selection problem. B) the moral hazard problem. D) the Coase theorem. Answer: B

Type: A Topic: 3 E: 679 MI: 435 56. If the existence of health insurance increases one's incentive to use the health care system more intensively, this is an illustration of: A) the adverse selection problem. C) the benefits-received principle. B) the moral hazard problem. D) the Coase theorem. Answer: B

McConnell/Brue: Economics, 16/e

Page 1086

Chapter 36: The Economics of Health Care

Type: A Topic: 3 E: 679 MI: 435 57. Sam decides to join the Gigantic State University's rugby team when he learns that his health insurance will pay for any subsequent injury. This illustrates: A) the diagnosis-related-group system. C) the moral hazard problem. B) a "pay or play" system. D) the Coase theorem. Answer: C

Type: A Topic: 3 E: 679 MI: 435 58. The availability of health insurance tends to: A) decrease the demand for health care and cause an underallocation of resources to the health care industry. B) increase the quantity of health care demanded and cause an underallocation of resources to the health care industry. C) increase the quantity of health care demanded and cause an overallocation of resources to the health care industry. D) decrease the quantity of health care demanded and cause an overallocation of resources to the health care industry. Answer: C

Type: A Topic: 3 E: 675 MI: 431 59. It is generally agreed that in the long run the cost of private health insurance provided by employers is: A) at the expense of business profits. B) at the expense of real wages. C) paid by taxpayers through government. D) included as taxable income for income tax purposes. Answer: B

Type: A Topic: 3 E: 677 MI: 433 60. Other things equal, increased emphasis on providing the best possible medical care for all, regardless of income, will: A) increase the demand for health care. B) increase the supply of health care. C) have no effect. D) decrease the demand for health care. Answer: A

McConnell/Brue: Economics, 16/e

Page 1087

Chapter 36: The Economics of Health Care

Use the following to answer questions 61-64:

Type: G Topic: 3 E: 680 MI: 436 61. Refer to the above demand and supply diagram that relates to the health care market. Without health insurance the equilibrium price and quantity of health care would be: A) P1 and Q1. B) P1 and Q2. C) P2 and Q2. D) P2 and Q1. Answer: A

Type: G Topic: 3 E: 680 MI: 436 62. Refer to the above demand and supply diagram that relates to the health care market. If insurance pays onehalf of all health care costs, the immediate price to the consumer and the quantity of health care consumed would be: A) P1 and Q1. B) P1 and Q2. C) P2 and Q2. D) P2 and Q1. Answer: C

Type: A Topic: 3 E: 680 MI: 436 63. Refer to the above demand and supply diagram that relates to the health care market. Health care insurance causes: A) resources to be underallocated to the health care industry. B) resources to be overallocated to the health care industry. C) health care to be underconsumed. D) the price of health care to the insuree to be higher than the market price. Answer: B

Type: G Topic: 3 E: 680 MI: 436 64. Refer to the above demand and supply diagram that relates to the health care market. The efficiency loss caused by the availability of health insurance is shown by area: A) P1abP2. B) abc. C) adc. D) Q1acQ2. Answer: C

McConnell/Brue: Economics, 16/e

Page 1088

Chapter 36: The Economics of Health Care

Use the following to answer questions 65-67:
Quantity Supplied 12,000 10,000 7,000 4,000 1,000 Price $5,000 4,000 3,000 2,000 1,000 Quantity Demanded 2,000 4,000 7,000 11,000 16,000

Type: T Topic: 3 E: 680 MI: 436 65. Refer to the above supply and demand data for a certain elective surgical procedure. Without health insurance, the equilibrium price and quantity would be: A) $5,000 and 2,000. B) $4,000 and 4,000. C) $3,000 and 7,000. D) $2,000 and 11,000. Answer: C

Type: T Topic: 3 E: 680 MI: 436 66. Refer to the above supply and demand data for a certain elective surgical procedure. If insurance pays twothirds of the expense, the immediate price to the consumer and quantity of health care consumed would be: A) $4,000 and 10,000. B) $3,000 and 7,000. C) $2,000 and 11,000. D) $1,000 and 16,000. Answer: D

Type: T Topic: 3 E: 680 MI: 436 67. Refer to the above supply and demand data for a certain elective surgical procedure. If insurance pays 100 percent after a $1000 deductible is satisfied, the quantity of health care consumed will be: A) $4,000 and 4,000. B) $3,000 and 7,000. C) $2,000 and 11,000. D) $1,000 and 16,000. Answer: C

Type: A Topic: 3 E: 679 MI: 435 68. Federal tax policy: A) treats employer health insurance premiums as taxable income. B) subsidizes health insurance and thereby increases the demand for health care. C) subsidizes health insurance and thereby decreases the demand for health care. D) corrects the overallocation of resources to the health care industry which would otherwise exist. Answer: B

Type: A Topic: 3 E: 679 MI: 435 69. A tax subsidy is involved in employer-financed health insurance because: A) all working adults are covered by Medicare. B) all working adults are covered by Medicaid. C) employer payments for health insurance are not subject to income or payroll taxes. D) corporations that provide health insurance pay lower corporate income tax. Answer: C

McConnell/Brue: Economics, 16/e

Page 1089

Chapter 36: The Economics of Health Care

Health care reform

Type: A Topic: 4 E: 683 MI: 439 70. The "play or pay" proposal for health care reform is designed to: A) increase access to health care by inducing more employers to provide health insurance. B) make health care vouchers freely available to all people living in poverty. C) provide health care to all Americans through the Federal government as the single insurer. D) contain health care costs through government determination of doctors' fees and hospital charges. Answer: A

Type: A Topic: 4 E: 683 MI: 439 71. Under a system of national health insurance: A) the Federal government would provide a basic health care package to all citizens at no direct cost. B) health care would be financed out of tax revenues. C) government could negotiate to reduce the fees charged by doctors and hospitals. D) all of the above might be done. Answer: D

Type: A Topic: 4 E: 683 MI: 439 72. One of the problems involved in government attempting to set fixed fees for doctors is that: A) a surplus of health care services will result. B) doctors might react by increasing the quantity of health care provided. C) such fees are usually set above the equilibrium level. D) doctors' fees have not risen as rapidly as the general price level. Answer: B

Type: A Topic: 4 E: 684 MI: 440 73. One of the criticisms of national health insurance is that the shift from insurance premiums to taxes in financing health care will: A) redistribute income toward greater equality. C) redistribute income toward greater inequality. B) necessarily increase the public debt. D) necessarily reduce the growth of real wages. Answer: A

Type: A Topic: 4 E: 684 MI: 440 74. Insurance companies use deductibles and copayments to: A) increase access to health care. B) reduce health care costs by alleviating overuse of the health care system. C) prevent small companies from self-insuring their workers. D) keep government out of the health care insurance industry. Answer: B

Type: A Topic: 4 E: 684 MI: 440 75. Suppose you go to a doctor but your health insurance plan does not reimburse you because you have not yet paid enough out-of-pocket for the year to qualify for insurance benefits. This is an example of: A) coinsurance. B) a deductible. C) monopsony power. D) a deferred benefit plan. Answer: B

McConnell/Brue: Economics, 16/e

Page 1090

Chapter 36: The Economics of Health Care

Type: A Topic: 4 E: 684 MI: 440 76. Suppose you go to a doctor but your health insurance plan reimburses you for only 80 percent of the bill. This is an example of: A) coinsurance. B) a deductible. C) monopsony power. D) a deferred benefit plan. Answer: A

Type: A Topic: 4 E: 684 MI: 440 77. The two main types of managed care organizations are: A) U.S. veterans hospitals and university health clinics. B) health maintenance organizations (HMOs) and private nursing homes. C) health maintenance organizations (HMOs) and preferred provider organizations (PPOs). D) preferred provider organizations (PPOs) and nonprofit hospitals. Answer: B

Type: A Topic: 4 E: 684 MI: 440 78. Health maintenance organizations (HMOs): A) are based on the traditional fee-for-service system of paying physicians. B) charge a fixed amount per member, hire many of their own physicians, and provide health services only to members. C) are also known as preferred provider organizations. D) are illegal in several states. Answer: B

Type: A Topic: 4 E: 684 MI: 440 79. Preferred provider organizations (PPOs): A) charge a fixed amount per member, hire many of their own physicians, and provide health services only to members. B) require that their members give up the right to file medical malpractice suits. C) are illegal in several states. D) require physicians and hospitals to provide discounted prices for their services as a condition for being included in the insurance plan. Answer: D

Type: A Topic: 4 E: 684 MI: 440 80. The main purpose of HMOs and PPOs is to: A) reduce health care costs for employers and their employees. B) reduce medical malpractice suits. C) enable groups of physicians to increase their fees. D) direct patients to specialists rather than to more expensive primary-care physicians. Answer: A

Type: A Topic: 4 E: 684 MI: 440 81. One of the main difference between PPOs and HMOs is that: A) HMO physicians charge on a traditional fee-for-service basis, while PPO physicians do not. B) HMOs are usually for-profit organizations, whereas PPOs are not. C) PPOs employ their own doctors, whereas HMOs do not. D) PPO physicians charge on a traditional fee-for service basis, while HMOs do not. Answer: D

McConnell/Brue: Economics, 16/e

Page 1091

Chapter 36: The Economics of Health Care

Type: A Topic: 4 E: 684 MI: 440 82. All of the following are designed to reduce health care expenses for consumers except: A) HMOs. B) PPOs. C) the DRG payment system. D) the fee-for-service system. Answer: D

Type: A Topic: 4 E: 685 MI: 441 83. Which of the following is a recent Congressional proposal relating to health care? A) establishment of health maintenance organizations to reduce health care costs. B) establishment of deductibles and copayments in health insurance policies. C) provision of prescription drug coverage under Medicare. D) establishment of fixed Medicare payment to hospitals based on one of several hundred diagnostic categories. Answer: C

Type: A Topic: 4 E: 685 MI: 441 84. Which of the following is a recent Congressional proposal relating to health care? A) establishment of health maintenance organizations to reduce health care costs B) establishment of deductibles and copayments in health insurance policies C) establishment of a patients bill of rights to protect members from abuses by HMOs D) establishment of fixed Medicare payment to hospitals based on one of several hundred diagnostic categories Answer: C

Type: F Topic: 4 E: 685 MI: 441 Status: New 85. "Patients' bill of rights" legislation has been stalled in recent years by disputes over: A) which groups of patients should be excluded. B) how the legislation should be financed. C) limits on malpractice awards for "pain and suffering." D) whether or not prescription drug coverage should be included. Answer: C

Consider This Questions

Type: F E: 682 MI: 438 Status: New 86. (Consider This) The hand-held ultrasound devices that are slowly replacing stethoscopes for physical exams are: A) considerably less expensive than stethoscopes on a per patient basis. B) considerably more expensive than stethoscopes on a per patient basis. C) an example of a cost-saving medical technology. D) an example of the benefits of government tax subsidies for medical research. Answer: B

Type: A E: 682 MI: 438 Status: New 87. (Consider This) The main focus of the story about hand-held ultrasound devices is: A) cost-saving medical technology. B) cost reductions from spreading an overhead expense over many patients. C) longer-lasting equipment than the equipment being replaced. D) benefit-producing, but cost-increasing, medical technology. Answer: D

McConnell/Brue: Economics, 16/e

Page 1092

Chapter 36: The Economics of Health Care

Last Word Questions

Type: A E: 686 MI: 442 88. (Last Word) A market for human organs (rather than the current volunteer-donor system) would be expected to: A) reduce the price of organs. C) reduce the supply of organs. B) create a surplus of organs. D) eliminate the shortage of organs. Answer: D

Type: A E: 686 MI: 442 89. (Last Word) A market-based system of buying and selling human organs for transplant would: A) reduce total health care spending. B) create a surplus of organs. C) increase the quantity of organs available for transplant. D) reduce the price of organs. Answer: C

Type: A E: 686 MI: 442 90. (Last Word) A market for human organs (rather than the current volunteer-donor system) would be expected to: A) reduce the quantity of organs supplied. B) increase total health care spending. C) decrease the demand for physicians who do transplants. D) create a surplus of organs. Answer: B

Type: A E: 686 MI: 442 91. (Last Word) A major objection to creating a legal market for human organs is that such a market would: A) create a substantial surplus of unused organs. B) increase the present shortage of organs. C) commercialize human body parts and thus diminish the special nature of human life. D) produce substantial spillover benefits. Answer: C

True/False Questions

Type: F E: 674 MI: 430 92. As a percentage of GDP, U.S. health care spending is higher than any other major industrial country. Answer: True

Type: F E: 674 MI: 430 93. The health care industry currently absorbs about 14 percent of U.S. gross domestic product. Answer: True

McConnell/Brue: Economics, 16/e

Page 1093

Chapter 36: The Economics of Health Care

Type: F E: 674 MI: 430 94. Health care costs have risen absolutely but remained constant as a percentage of gross domestic product. Answer: False

Type: A E: 675 MI: 431 95. Rising health care costs have prompted workers to change jobs with greater frequency. Answer: False

Type: A E: 675-676 MI: 431-432 96. There is a consensus among economists that the amount of resources allocated to the health care industry is optimal. Answer: False

Type: A E: 675 MI: 431 97. Some employers have reacted to rising health care costs by hiring more part-time and temporary workers. Answer: True

Type: F E: 674 MI: 430 98. About three-fourths of all health care costs are paid out-of-pocket by patients. Answer: False

Type: F E: 673 MI: 429 99. In 2001, some 41 million Americans did not have health insurance. Answer: True

Type: A E: 677 MI: 433 100. The demand for health care is highly elastic with respect to both price and income. Answer: False

Type: A E: 678 MI: 434 101. Because of health insurance, resources are underallocated to the health care industry. Answer: False

Type: A E: 677 MI: 433 102. Health care is an inferior good. Answer: False

Type: A E: 679 MI: 435 103. Third-party (insurance) payment of health care costs causes health care to be overconsumed. Answer: True

Type: A E: 681 MI: 437 104. Technological progress in the health care industry has typically reduced costs and increased supply. Answer: False

McConnell/Brue: Economics, 16/e

Page 1094

Chapter 36: The Economics of Health Care

Type: A E: 677-678 MI: 433-434 105. Changes in the average age of the population have decreased the demand for health care. Answer: False

Type: A E: 677 MI: 433 106. Asymmetric information in the health care market has increased the supply of health care. Answer: False

Type: A E: 679 MI: 435 107. A moral hazard problem arises in the health care market because health insurance causes people to overconsume health care. Answer: True

Type: A E: 684 MI: 440 108. Insurance companies use deductibles and copayments to control increases in the amount of health care demanded. Answer: True

Type: A E: 683 MI: 439 109. The primary purpose of the "play or pay" reform proposal is to reduce aggregate health care costs. Answer: False

Type: A E: 684 MI: 440 110. It is generally agreed that the most promising reform of the health care system is for government to fix doctors' fees below current levels. Answer: False

Type: A E: 679 MI: 435 111. Because employer payments for health insurance are not subject to income or payroll taxes, government in effect provides a subsidy to health care. Answer: True

Type: A E: 684 MI: 440 112. Preferred provider organizations (PPOs) are a type of managed-care organization. Answer: True

Type: F E: 684 MI: 440 113. HMO stands for holistic medical organization. Answer: False

McConnell/Brue: Economics, 16/e

Page 1095

CHAPTER 37

International Trade

Topic 1. 2. 3. 4. 5. 6. 7. Trade facts Comparative advantage Supply and demand analysis of exports and imports Types of trade barriers Analysis of tariffs and quotas Arguments for protectionism World Trade Organization Consider This Last Word True-False

Question numbers 1-9 10-55 56-79 80-87 88-118 119-125 126-128 129-130 131-132 133-151

____________________________________________________________

_______________________________________

____________________________________________________________

_______________________________________

Multiple Choice Questions Trade facts

Type: F Topic: 1 E: 690 MI: 446 MA: 356 1. United States exports of goods and services are about: A) 20 percent of U.S. GDP. C) 28 percent of U.S. GDP. B) 4 percent of U.S. GDP. D) 11 percent of U.S. GDP. Answer: D

Type: F Topic: 1 E: 690 MI: 446 MA: 356 2. Since 1975 U.S. exports and imports have: A) declined as a percentage of U.S. GDP. B) more than doubled as a percentage of U.S. GDP. C) more than tripled as a percentage of U.S. GDP. D) remained virtually constant as a percentage of U.S. GDP. Answer: B

Type: F Topic: 1 E: 690 MI: 446 MA: 356 3. In recent years the United States has: A) exported more services abroad than it has imported. B) had a small goods trade surplus with Japan. C) had a large goods trade surplus with the rest of the world. D) fallen to third behind Japan and Germany in the list of leading export nations (absolute volume basis). Answer: A

Chapter 37: International Trade

Type: F Topic: 1 E: 691 MI: 447 MA: 357 Status: New 4. As a percentage of GDP, U.S. exports are: A) greater than U.S. imports. B) about 20 percent. C) considerably lower than in several other industrially advanced nations. D) higher than in Canada but lower than Mexico. Answer: C

Type: F Topic: 1 E: 691 MI: 447 MA: 357 Status: New 5. Which of the following statements is false? A) In recent years the United States has had large annual trade deficits in goods and services. B) The United States imports some of the same categories of goods as it exports. C) The United States is the world's leading exporter in terms of absolute dollar volume. D) As a percentage of GDP, U.S. exports are the highest among the industrially advance nations. Answer: D

Type: F Topic: 1 E: 690 MI: 446 MA: 356 6. Which country has the largest share of total world exports? A) Japan B) Germany C) United States D) China Answer: C

Type: F Topic: 1 E: 690 MI: 446 MA: 356 7. In terms of absolute volume, world trade is dominated by: A) Japan, Germany, and China. B) the United States, England, and Canada. C) Germany, England, and France. D) the United States, Japan and the western European nations. Answer: D

Type: F Topic: 1 E: 690 MI: 446 MA: 356 Status: New 8. In 2002 the United States: A) imported more services than it exported. B) imported more goods than it exported. C) traded mainly with developing nations such as Mexico and China. D) had a small trade surplus in goods and services. Answer: B

Type: F Topic: 1 E: 691 MI: 447 MA: 357 Status: New 9. Which of the following countries recently joined the ranks of the world's leading trading nations by total dollar volume? A) China. B) Chile. C) Ireland. D) Belgium. Answer: A

Comparative advantage Type: A Topic: 2 E: 691 MI: 447 MA: 357 10. Which of the following is an example of a land-intensive commodity? A) chemicals B) autos C) watches D) wool Answer: D

McConnell/Brue: Economics, 16/e

Page 1098

Chapter 37: International Trade

Type: A Topic: 2 E: 691 MI: 447 MA: 357 11. Which of the following is an example of a labor-intensive commodity? A) cameras B) beer C) aspirin tablets D) gasoline Answer: A

Type: A Topic: 2 E: 691 MI: 447 MA: 357 12. Which of the following is an example of a capital-intensive commodity? A) clothing B) wool C) sunflower seeds D) chemicals Answer: D

Type: A Topic: 2 E: 691 MI: 447 MA: 357 13. Differences in production efficiencies among nations in producing a particular good result from: A) different endowments of fertile soil. C) different levels of technological knowledge. B) different amounts of skilled labor. D) all of the above. Answer: D

Type: A Topic: 2 E: 692 MI: 448 MA: 358 14. Countries engaged in international trade specialize in production based on: A) relative levels of GDP. C) relative exchange rates. B) comparative advantage. D) relative inflation rates. Answer: B

Type: A Topic: 2 E: 692 MI: 448 MA: 358 Status: New 15. Examples of exports based on actual or perceived quality superiority, and not simply on cost-advantages, include: A) wheat from the United States and natural gas from Russia. B) bottled water from France and chocolates from Belgium. C) pine furniture from Mexico and textiles from China. D) coffee from Brazil and wool from Australia. Answer: B

Use the following to answer questions 16-18: Answer the next question(s) on the basis of the following production possibilities data for Gamma and Sigma. All data are in tons. Gamma production possibilities:
Tea Pots A 120 0 B 90 30 C 60 60 D 30 90 E 0 120

Sigma production possibilities:
Tea Pots A 40 0 B 30 30 C 20 60 D 10 90 E 0 120

McConnell/Brue: Economics, 16/e

Page 1099

Chapter 37: International Trade

Type: T Topic: 2 E: 693 MI: 449 MA: 359 16. On the basis of the above information: A) Gamma should export both tea and pots to Sigma. B) Sigma should export tea to Gamma and Gamma should export pots to Sigma. C) Gamma should export tea to Sigma and Sigma should export pots to Gamma. D) Gamma should export tea to Sigma, but it will not be profitable for the two nations to exchange pots. Answer: C

Type: T Topic: 2 E: 693 MI: 449 MA: 359 17. Refer to the above data. What are the limits of the terms of trade between Gamma and Sigma? A) 1 tea = 2 pots to 1 tea = 6 pots C) 1 tea = 2 pots to 1 tea = 3.5 pots B) 1 tea = 3 pots to 1 tea = 6 pots D) 1 tea = 1 pot to 1 tea = 3 pots Answer: D

Type: T Topic: 2 E: 693 MI: 449 MA: 359 18. Refer to the above data. Assume that before specialization and trade Gamma and Sigma both chose production possibility "C." Now if each specializes according to comparative advantage, the gains from specialization and trade will be: A) 40 tons of pots. B) 20 tons of tea and 20 tons of pots. C) 20 tons of tea. D) 40 tons of tea. Answer: D

Type: A Topic: 2 E: 692-693 MI: 448-449 MA: 358-359 19. If country A can produce both goods X and Y more efficiently, that is, with smaller absolute amounts of resources, than can country B: A) mutually advantageous specialization and trade between A and B may still be possible. B) we can conclude that A is an industrially advanced economy and B is a developing economy. C) it will necessarily be advantageous for B to import both X and Y from A. D) then there is no possible basis for mutually advantageous specialization and trade between A and B. Answer: A

Type: D Topic: 2 E: 694 MI: 450 MA: 360 20. The terms of trade reflect the: A) rate at which gold exchanges internationally for any domestic currency. B) ratio at which nations will exchange two goods. C) fact that the gains from trade will be equally divided. D) cost conditions embodied in a single country's production possibilities curve. Answer: B

Type: A Topic: 2 E: 694 MI: 450 MA: 360 21. Assume that by devoting all of its resources to the production of X, nation Alpha can produce 40 units of X. By devoting all of its resources to Y, Alpha can produce 60Y. Comparable figures for nation Beta are 60X and 40Y. We can conclude that: A) the terms of trade will be 3X equals 1Y. B) Alpha should specialize in Y and Beta in X. C) Alpha should specialize in X and Beta in Y. D) there is no basis for mutually beneficial specialization and trade. Answer: B

McConnell/Brue: Economics, 16/e

Page 1100

Chapter 37: International Trade

Use the following to answer questions 22-26: Answer the next question(s) on the basis of the following production possibilities tables for two countries, Latalia and Trombonia:
Latalia's production possibilities A B C D E 4 3 2 1 0 0 5 10 15 20 Trombonia's production possibilities A B C D E 8 6 4 2 0 0 6 12 18 24

Pork (tons) Beans (tons)

Pork (tons) Beans (tons)

Type: T Topic: 2 E: 693-694 MI: 449-450 MA: 359-360 22. The above data indicate that production in: A) both Latalia and Trombonia is subject to constant opportunity costs. B) Trombonia is subject to decreasing costs, but production in Latalia occurs under increasing opportunity costs. C) Latalia is subject to increasing costs, but production in Trombonia occurs under constant opportunity costs. D) both Latalia and Trombonia are subject to the law of increasing opportunity costs. Answer: A

Type: T Topic: 2 E: 693-694 MI: 449-450 MA: 359-360 23. Refer to the above tables. In Latalia the domestic real cost of 1 ton of pork: A) is 3 tons of beans. C) is 5 tons of beans. B) diminishes with the level of pork production. D) is 1/5 of a ton of beans. Answer: C

Type: T Topic: 2 E: 693-694 MI: 449-450 MA: 359-360 24. Refer to the above tables. If these two nations specialize on the basis of comparative advantage: A) Trombonia will produce beans and Latalia will produce pork. B) Trombonia will produce both beans and pork. C) Latalia will produce both beans and pork and Trombonia will produce neither. D) Latalia will produce beans and Trombonia will produce pork. Answer: D

Type: T Topic: 2 E: 693-694 MI: 449-450 MA: 359-360 25. Refer to the above tables. Assume that before specialization and trade, Latalia produced combination C and Trombonia produced combination B. If these two nations now specialize completely based on comparative advantage, the total gains from specialization and trade will be: A) 4 tons of beans. C) 4 tons of pork. B) 1 ton of pork and 2 tons of beans. D) 2 tons of pork and 4 tons of beans. Answer: A

McConnell/Brue: Economics, 16/e

Page 1101

Chapter 37: International Trade

Type: T Topic: 2 E: 693-694 MI: 449-450 MA: 359-360 26. Refer to the above tables. Which of the following would be feasible terms for trade between Latalia and Trombonia? A) 1 ton of beans for 1 ton of pork C) 6 tons of beans for 1 ton of pork B) 2 tons of beans for 1 ton of pork D) 4 tons of beans for 1 ton of pork Answer: D

Type: A Topic: 2 E: 693 MI: 449 MA: 359 27. In the theory of comparative advantage, a good should be produced in that nation where: A) the production possibilities line lies further to the right than the trading possibilities line. B) its cost is least in terms of alternative goods that might otherwise be produced. C) its absolute cost in terms of real resources used is least. D) its absolute money cost of production is least. Answer: B

Type: A Topic: 2 E: 692 MI: 448 MA: 358 28. If two nations have straight-line production possibilities curves: A) then their trading possibilities curves must lie inside the production possibilities curves. B) there will be no basis for mutually advantageous trade. C) there will be a basis for mutually advantageous trade whether the slopes are equal or not. D) there will be a basis for mutually advantageous trade provided the slopes differ. Answer: D

Use the following to answer questions 29-33:

McConnell/Brue: Economics, 16/e

Page 1102

Chapter 37: International Trade

Type: G Topic: 2 E: 694-695 MI: 450-451 MA: 360-361 29. Refer to the above diagrams. The solid lines are production possibilities curves; the dashed lines are trading possibilities curves. The data contained in the production possibilities curves are based on the assumption of: A) imperfect shiftability of resources as between beer and pizza production. B) constant costs. C) decreasing costs. D) increasing costs. Answer: B

Type: G Topic: 2 E: 694-695 MI: 450-451 MA: 360-361 30. Refer to the above diagrams. The solid lines are production possibilities curves; the dashed lines are trading possibilities curves. The opportunity cost of producing a: A) pizza is 2 beers in both countries. C) pizza in East Lothian is 1 beer. B) beer is 1/2 a pizza in both countries. D) beer in West Lothian is 1/2 a pizza. Answer: D

Type: G Topic: 2 E: 694-695 MI: 450-451 MA: 360-361 31. Refer to the above diagrams. The solid lines are production possibilities curves; the dashed lines are trading possibilities curves. The data suggest that: A) West Lothian should specialize in, and export, beer. B) both countries will be better off if they do not engage in specialization and trade involving these two products. C) West Lothian should specialize in, and export, pizza. D) East Lothian should specialize in, and export, beer. Answer: A

Type: G Topic: 2 E: 694-695 MI: 450-451 MA: 360-361 32. Refer to the above diagrams. The solid lines are production possibilities curves; the dashed lines are trading possibilities curves. The trading possibilities curves imply that: A) both countries have a trade surplus that will result in economic growth. B) the domestic production possibilities curves entail unemployment and/or the domestic misallocation of resources. C) world resources will be allocated more efficiently if the two nations specialize and trade based on comparative advantage. D) both nations will be worse off as a result of international specialization and trade. Answer: C

Type: G Topic: 2 E: 694-695 MI: 450-451 MA: 360-361 33. Refer to the above diagrams. The solid lines are production possibilities curves; the dashed lines are trading possibilities curves. The trading possibilities curves suggest that the terms of trade are: A) 1.5 beers for 1 pizza. B) 1 beer for 2 pizzas. C) 2 beers for 1 pizza. D) 1 beer for 1.5 pizzas. Answer: D

McConnell/Brue: Economics, 16/e

Page 1103

Chapter 37: International Trade

Type: A Topic: 2 E: 694 MI: 450 MA: 360 34. The fact that international specialization and trade based on comparative advantage can increase world output is demonstrated by the reality that: A) the production possibilities curve of any two nations are identical. B) a nation's production possibilities and trading possibilities lines coincide. C) a nation's trading possibilities line lies to the right of its production possibilities line. D) a nation's production possibilities line lies to the right of its trading possibilities line. Answer: C

Type: A Topic: 2 E: 694-695 MI: 450-451 MA: 360-361 35. Free trade based on comparative advantage is economically beneficial because: A) it promotes an efficient allocation of world resources. B) it increases competition. C) it provides consumers with a wider range of products. D) of all of the above reasons. Answer: D

Use the following to answer questions 36-38: Answer the next question(s) on the basis of the following information about the cost ratios for two products--fish (F) and chicken (C)--in Singsong and Harmony. Assume that production occurs under conditions of constant costs and these are the only two nations in the world. Singsong: 1F = 2C Harmony: 1F = 4C

Type: T Topic: 2 E: 693-694 MI: 449-450 MA: 359-360 36. Refer to the above information. In Singsong the domestic real cost of each chicken: A) is 1/2 a fish. C) increases with the level of fish caught. B) is 2 fish. D) decreases with the level of fish caught. Answer: A

Type: T Topic: 2 E: 693 MI: 449 MA: 359 37. Refer to the above information. If these two nations specialize based on comparative advantage: A) Singsong will both produce chicken and catch fish. B) Harmony will both produce chicken and catch fish. C) Harmony will produce chicken and Singsong will catch fish. D) Singsong will produce chicken and Harmony will catch fish. Answer: C

Type: T Topic: 2 E: 693-694 MI: 449-450 MA: 359-360 38. Refer to the above information. Which one of the following would not be feasible terms for trade between Singsong and Harmony? A) 1 fish for 21/2 chicken C) 1 chicken for 1/5 of a fish B) 1 fish for 3 chicken D) 1 chicken for 1/3 of a fish Answer: C

McConnell/Brue: Economics, 16/e

Page 1104

Chapter 37: International Trade

Type: T Topic: 2 E: 694 MI: 450 MA: 360 39. The gain from international trade is: A) increased employment in the domestic export sector. B) more goods than would be attainable through domestic production alone. C) tariff revenue. D) increased employment in the domestic import sector. Answer: B

Use the following to answer questions 40-41:

Type: G Topic: 2 E: 692-693 MI: 448-449 MA: 358-359 40. The production possibilities curves above suggest that: A) West Mudville should specialize in, and export, baseball bats. B) West Mudville should specialize in, and export, both baseballs and baseball bats. C) East Mudville should specialize in, and export, baseball bats. D) workers will try to immigrate from West Mudville to East Mudville. Answer: A

Type: G Topic: 2 E: 692-693 MI: 448-449 MA: 358-359 41. Assuming labor forces of equal size, the production possibilities curves above suggest that workers in West Mudville will have: A) lower wages than workers in East Mudville before trade but equal wages after trade. B) higher wages than workers in East Mudville both before and after trade. C) lower wages than workers in East Mudville both before and after trade. D) higher wages than workers in East Mudville before trade but lower wages after trade. Answer: B

Type: A Topic: 2 E: 692-693 MI: 448-449 MA: 358-359 42. If a nation has a comparative advantage in the production of X, this means the nation: A) cannot benefit by producing and trading this product. B) must give up less of other goods than other nations in producing a unit of X. C) has a production possibilities curve identical to those of other nations. D) is not subject to increasing opportunity costs. Answer: B

McConnell/Brue: Economics, 16/e

Page 1105

Chapter 37: International Trade

Use the following to answer questions 43-45:
B
B

0

Stanville

A

0

A Terryville

Type: G Topic: 2 E: 693-694 MI: 449-450 MA: 359-360 43. Refer to the above graphs. Stanville has a comparative advantage in producing: A) product A. B) product B. C) both product A and B. D) neither product A nor B. Answer: B

Type: G Topic: 2 E: 693-694 MI: 449-450 MA: 359-360 44. Refer to the above graphs. Terryville has a comparative advantage in producing: A) product A. B) product B. C) both product A and B. D) neither product A nor B. Answer: A

Type: G Topic: 2 E: 693-694 MI: 449-450 MA: 359-360 45. Refer to the above graphs. These production possibilities curves: A) demonstrate that there can be gains from specialization and trade between the two nations. B) reflect the law of increasing opportunity costs. C) reflect the law of diminishing marginal utility. D) imply that specialization will be incomplete. Answer: A Use the following to answer questions 46-50: Answer the next question(s) on the basis of the following production possibilities data for two countries, Alpha and Beta, which have populations of equal size.

Fish (tons) Chips (tons)

Alpha's production possibilities: A B C D E 80 60 40 20 0 0 5 10 15 20 Beta's production possibilities: A B B C E 240 180 120 60 0 0 10 20 30 40

Fish (tons) Chips (tons)

Type: T Topic: 2 E: 693-694 MI: 449-450 MA: 359-360 46. The above data show that: A) Beta has a comparative advantage in producing chips. B) Alpha has a comparative advantage in catching fish. C) Alpha is subject to constant costs and Beta is subject to increasing costs. D) Beta is more efficient than Alpha both in catching fish and in producing chips. Answer: D

McConnell/Brue: Economics, 16/e

Page 1106

Chapter 37: International Trade

Type: T Topic: 2 E: 693 MI: 449 MA: 359 47. Refer to the above data. The domestic opportunity cost of: A) producing a ton of chips in Alpha is 1/5 of a ton of fish. B) producing a ton of chips in Beta is 6 tons of fish. C) catching a ton of fish in Alpha is 5 tons of chips. D) catching a ton of fish in Beta is 6 tons of chips. Answer: B

Type: T Topic: 2 E: 693 MI: 449 MA: 359 48. Refer to the above data. Beta: A) should specialize in catching fish and trade with Alpha for chips. B) should specialize in producing chips and trade with Alpha for fish. C) will not realize gains from specialization and trade. D) will export both fish and chips to Alpha. Answer: A

Type: T Topic: 2 E: 693-694 MI: 449-450 MA: 359-360 49. Refer to the above data. Suppose that before specialization and trade Alpha chose production alternative C and Beta chose production alternative B. After specialization and trade the gains will be: A) 20 tons of fish. C) 20 tons of fish and 20 tons of chips. B) 20 tons of chips. D) 240 tons of fish and 20 tons of chips. Answer: A

Type: T Topic: 2 E: 693-694 MI: 449-450 MA: 359-360 50. Refer to the above data. Assume the production possibilities in Beta double at alternatives A through E while remaining as shown in the table for Alpha. As a result Beta should: A) continue to specialize in producing chips. B) continue to specialize in fishing. C) no longer specialize and trade. D) specialize both in fishing and in producing chips and sell the surplus to Alpha. Answer: B

McConnell/Brue: Economics, 16/e

Page 1107

Chapter 37: International Trade

Use the following to answer questions 51-52:

Type: G Topic: 2 E: 694-695 MI: 450-451 MA: 360-361 51. Refer to the above diagram in which line AB is the United States production possibility curve and AC is its trading possibilities curve. We can conclude that the United States: A) has chosen to specialize in the production of cheese. B) has chosen to specialize in the production of beef. C) has decided to trade beef for cheese. D) is relatively more efficient than its trading partners in producing both cheese and beef. Answer: A

Type: G Topic: 2 E: 694-695 MI: 450-451 MA: 360-361 52. Refer to the above diagram in which line AB is the United States production possibility curve and AC is its trading possibilities curve. The international exchange ratio between beef and cheese (terms of trade): A) is the absolute value of slope of line AB. B) is the absolute value of slope of line AC. C) could lie anywhere between the absolute value of the slopes of lines AB and AC. D) cannot be determined on the basis of this information. Answer: B

Type: A Topic: 2 E: 696 MI: 452 MA: 362 53. The impact of increasing, as opposed to constant, costs is to: A) intensify and prolong the comparative advantages that any nation may have initially. B) expand the limits of the terms of trade. C) cause the bases for further specialization to disappear as nations specialize according to comparative advantage. D) cause nations to realize economies of scale in those products in which they specialize. Answer: C

McConnell/Brue: Economics, 16/e

Page 1108

Chapter 37: International Trade

Type: A Topic: 2 E: 696 MI: 452 MA: 362 54. In the real world, specialization is rarely complete because: A) nations normally experience increasing opportunity costs in producing more of the product in which they are specializing. B) production possibilities curves are straight lines rather than curves bowed outward as viewed from the origin. C) one nation's imports are necessarily another nation's exports. D) international law prohibits monopolies. Answer: A

Type: A Topic: 2 E: 696 MI: 452 MA: 362 55. The law of increasing opportunity costs: A) applies to land-intensive commodities, but not to labor-intensive or capital-intensive commodities. B) results in straight-line production possibilities curves rather than curves that are bowed outward from the origin. C) refutes the principle of comparative advantage. D) may limit the extent to which a nation specializes in producing a particular product. Answer: D

Supply and demand analysis of exports and imports Type: A Topic: 3 E: 697 MI: 453 MA: 363 56. Suppose the domestic price (no-international-trade price) of copper is $1.20 a pound in the United States while the world price is $1.00 a pound. Assuming no transportation costs, the United States will: A) have a domestic surplus of copper. C) import copper. B) export copper. D) neither export nor import copper. Answer: C

Type: A Topic: 3 E: 697-698 MI: 453-454 MA: 363-364 57. Suppose the domestic price (no-international-trade price) of wheat is $3.50 a bushel in the United States while the world price is $4.00 a bushel. Assuming no transportation costs, the United States will: A) have a domestic shortage of wheat. C) import wheat. B) export wheat. D) neither export nor import wheat. Answer: B

Type: A Topic: 3 E: 698 MI: 454 MA: 364 58. A nation's import demand curve for a specific product: A) is upsloping. B) shows the amount of the product it will import at prices below its domestic price. C) lies above its export supply curve for the product. D) depends on domestic demand for the product, but not on domestic supply. Answer: B

Type: A Topic: 3 E: 697-698 MI: 453-454 MA: 363-364 59. A nation's export supply curve for a specific product: A) is upsloping. B) shows the amount of the product it will export at prices below its domestic price. C) lies below its import demand curve for the product. D) depends on domestic supply of the product, but not on domestic demand. Answer: A

McConnell/Brue: Economics, 16/e

Page 1109

Chapter 37: International Trade

Type: A Topic: 3 E: 697-698 MI: 453-454 MA: 363-364 60. A nation will neither export nor import a specific product when its: A) domestic price (no-international-trade price) equals the world price. B) export supply curve lies above its import demand curve. C) export supply curve is upsloping. D) import demand curve is downsloping. Answer: A

Type: A Topic: 3 E: 698 MI: 454 MA: 364 61. Export supply curves are __________________; import demand curves are ___________________. A) horizontal; vertical C) downsloping; upsloping B) vertical; horizontal D) upsloping; downsloping Answer: D

Use the following to answer questions 62-64:

Type: G Topic: 3 E: 698 MI: 454 MA: 364 62. Refer to the above diagram showing the domestic demand and supply curves for a specific standardized product in a particular nation. If the world price for this product is $1.60, this nation will experience a domestic: A) shortage of 160 units, which it will meet with 160 units of imports. B) shortage of 160 units, which will increase the domestic price to $1.60. C) surplus of 160 units, which it will export. D) surplus of 160 units, which will reduce the world price to $1.00. Answer: C

Type: G Topic: 3 E: 698 MI: 454 MA: 364 63. Refer to the above diagram showing the domestic demand and supply curves for a specific standardized product in a particular nation. If the world price for this product is $.50, this nation will experience a domestic: A) shortage of 160 units, which it will meet with 160 units of imports. B) shortage of 160 units, which will increase the domestic price to $1.60. C) surplus of 160 units which it will export. D) surplus of 160 units, which will reduce the world price to $1.00. Answer: A

McConnell/Brue: Economics, 16/e

Page 1110

Chapter 37: International Trade

Type: G Topic: 3 E: 698 MI: 454 MA: 364 64. Refer to the above diagram showing the domestic demand and supply curves for a specific standardized product in a particular nation. If the world price of this product is $1, this nation will: A) export all of the product. B) import all of the product. C) import some of the product and produce some of the product domestically. D) neither export nor import the product. Answer: D

Type: A Topic: 3 E: 700 MI: 456 MA: 366 65. In a two-nation model, the equilibrium world price will occur where: A) one nation's export supply curve intersects the other nation's import demand curve. B) exports are exactly twice the level of imports. C) both nations' export supply curves are horizontal. D) both nations' import demand curves are vertical. Answer: A

Use the following to answer questions 66-71:

Type: G Topic: 3 E: 699-700 MI: 455-456 MA: 365-366 66. Refer to the above diagram pertaining to two nations and a specific product. Lines FA and GB are: A) domestic supply curves for two countries. C) import demand curves for two countries. B) domestic demand curves for two countries. D) export supply curves for two countries. Answer: D

Type: G Topic: 3 E: 699-700 MI: 455-456 MA: 365-366 67. Refer to the above diagram pertaining to two nations and a specific product. Lines FC and GD are: A) domestic supply curves for two countries. C) import demand curves for two countries. B) domestic demand curves for two countries. D) export supply curves for two countries. Answer: C

McConnell/Brue: Economics, 16/e

Page 1111

Chapter 37: International Trade

Type: G Topic: 3 E: 699-700 MI: 455-456 MA: 365-366 68. Refer to the above diagram pertaining to two nations and a specific product. Point G is the: A) domestic price for the nation represented by lines FA and FC. B) world equilibrium price. C) domestic price for the nation represented by lines GB and GD. D) price above the world equilibrium price. Answer: C

Type: G Topic: 3 E: 699-700 MI: 455-456 MA: 365-366 69. Refer to the above diagram pertaining to two nations and a specific product. In equilibrium, the nation represented by lines FA and FC will: A) export H to the country represented by lines GB and GD. B) import H from the country represented by lines GB and GD. C) pay price F for its imports. D) receive price G for its exports. Answer: B

Type: G Topic: 3 E: 699-700 MI: 455-456 MA: 365-366 70. Refer to the above diagram pertaining to two nations and a specific product. The equilibrium world price occurs at: A) F B) I C) G D) J Answer: B

Type: G Topic: 3 E: 699-700 MI: 455-456 MA: 365-366 71. Refer to the above diagram pertaining to two nations and a specific product. The equilibrium level of exports and imports occurs at: A) H, where GB and FC intersect. B) J, where the vertical distance between A and B just equals the vertical distance between C and D . C) world price level F. D) world price level G. Answer: A

Use the following to answer questions 72-79: Answer the next question(s) on the basis of the following data for the hypothetical nations of Alpha and Beta. Qs is domestic quantity supplied and Qd is domestic quantity demanded.

Domestic market for steel, Alpha Qs P Qd 60 $5 10 40 4 20 30 3 30 20 2 40 10 1 50

Domestic market for steel, Beta Qs P Qd 80 $5 20 70 4 30 60 3 40 50 2 50 40 1 60

McConnell/Brue: Economics, 16/e

Page 1112

Chapter 37: International Trade

Type: T Topic: 3 E: 700 MI: 456 MA: 366 72. Refer to the above data. The equilibrium prices of steel in Alpha and Beta are: A) $5 and $4, respectively. C) $3 and $2, respectively. B) $2 and $4, respectively. D) $1 and $2, respectively. Answer: C

Type: T Topic: 3 E: 700 MI: 456 MA: 366 73. Refer to the above data. At a world price of $5: A) Alpha will want to import 50 units of steel. B) Beta will want to import 60 units of steel. Answer: C

C) Alpha will want to export 50 units of steel. D) neither country will want to export steel.

Type: T Topic: 3 E: 700 MI: 456 MA: 366 74. Refer to the above data. At a world price of $2: A) Alpha will want to import 20 units of steel. B) Beta will want to export 20 units of steel. Answer: A

C) Alpha will want to export 20 units of steel. D) neither country will want to import steel.

Type: T Topic: 3 E: 700 MI: 456 MA: 366 75. Refer to the above data. Alpha's export supply is represented by: A)

P $5 4 3
B)

Q 40 20 0 Q 50 20 0 Q 60 30 0 Q 40 30 15

P $5 4 3
C)

P $5 4 3
D)

P $5 4 3
Answer: B

Type: T Topic: 3 E: 700 MI: 456 MA: 366 76. Refer to the above data. The equilibrium world price must be lower than $4 because at $4: A) both nations want to import steel. C) Beta wants to export more than Alpha. B) both nations want to export steel. D) Alpha wants to import more than Beta. Answer: B

McConnell/Brue: Economics, 16/e

Page 1113

Chapter 37: International Trade

Type: T Topic: 3 E: 700 MI: 456 MA: 366 77. Refer to the above data. The equilibrium world price must be higher than $1 because at $1: A) Beta wants to import more than Alpha. C) both nations want to export steel. B) Alpha wants to export more than Beta. D) both nations want to import steel. Answer: D

Type: T Topic: 3 E: 700 MI: 456 MA: 366 78. Refer to the above data. The equilibrium world price of steel must be between: A) $5 and $4. B) $4 and $3. C) $3 and $2. D) $2 and $1. Answer: C

Type: T Topic: 3 E: 700 MI: 456 MA: 366 79. Refer to the above data. At the equilibrium world price: A) both nations will export steel. B) both nations will import steel. C) Alpha will export steel and Beta will import steel. D) Beta will export steel and Alpha will import steel. Answer: D

Types of trade barriers

Type: A Topic: 4 E: 700 MI: 456 MA: 366 80. Tariffs: A) may be imposed either to raise revenue (revenue tariffs) or to shield domestic producers from foreign competition (protective tariffs). B) are also called import quotas. C) are excise taxes on goods exported abroad. D) are per unit subsidies designed to promote exports. Answer: A

Type: D Topic: 4 E: 700 MI: 456 MA: 366 81. An excise tax on an imported good that is not produced domestically is called a: A) protective tariff. B) import quota. C) revenue tariff. D) voluntary export restriction. Answer: C

Type: D Topic: 4 E: 700 MI: 456 MA: 366 82. An excise tax on an imported good that is also produced by domestic firms is called a: A) protective tariff. B) import quota. C) revenue tariff. D) voluntary export restriction. Answer: C

Type: A Topic: 4 E: 700 MI: 456 MA: 366 83. Country A limits other nation's exports to Country A to 1,000 tons of coal annually. This is an example of a(n): A) protective tariff. B) export subsidy. C) import quota. D) voluntary export restriction. Answer: C

McConnell/Brue: Economics, 16/e

Page 1114

Chapter 37: International Trade

Type: A Topic: 4 E: 700-701 MI: 456-457 MA: 366-367 84. Which is an example of a nontariff barrier (NTB)? A) an export subsidy B) an excise tax on the physical volume of imported goods C) box-by-box inspection requirements for imported fruit D) an excise tax on the dollar value of imported goods Answer: C

Type: D Topic: 4 E: 700 MI: 456 MA: 366 85. A tariff can best be described as: A) an excise tax on an imported good. B) a government payment to domestic producers to enable them to sell competitively in world markets. C) an excise tax on an exported good. D) a law that sets a limit on the amount of a good that can be imported. Answer: A

Type: A Topic: 4 E: 701 MI: 457 MA: 367 Status: New 86. In the past, Canada has agreed to set an upper limit on the total amount of softwood lumber exported to the United States. This is an example of a(n): A) import quota. B) export subsidy. C) voluntary export restriction. D) protective tariff. Answer: C

Type: A Topic: 4 E: 700 MI: 456 MA: 366 Status: New 87. Suppose the United States sets a limit on the number of tons of sugar that can be imported each year. This is an example of a(n): A) protective tariff. B) revenue tariff. C) voluntary export restriction. D) import quota. Answer: D

Analysis of tariffs and quotas

Use the following to answer questions 88-94:

Type: G Topic: 5 E: 701 MI: 457 MA: 367 88. Refer to the above diagram, where Sd and Dd are the domestic supply and demand for a product and Pc is the world price of that product. If this economy was entirely closed to international trade, equilibrium price and quantity would be: A) Pa and z. B) Pa and x. C) Pc and z. D) Pc and v. Answer: B

McConnell/Brue: Economics, 16/e

Page 1115

Chapter 37: International Trade

Type: G Topic: 5 E: 701 MI: 457 MA: 367 89. Refer to the above diagram, where Sd and Dd are the domestic supply and demand for a product and Pc is the world price of that product. If the economy is opened to free trade, the price and quantity sold of this product would be: A) Pc and v. B) Pa and z. C) Pt and y. D) Pc and z. Answer: D

Type: G Topic: 5 E: 701 MI: 457 MA: 367 90. Refer to the above diagram, where Sd and Dd are the domestic supply and demand for a product and Pc is the world price of that product. With free trade, that is, assuming no tariff, the outputs produced by domestic and foreign producers respectively would be: A) v and vz. B) w and wy. C) w and wz. D) vx and xz. Answer: A

Type: G Topic: 5 E: 701 MI: 457 MA: 367 91. Refer to the above diagram, where Sd and Dd are the domestic supply and demand for a product and Pc is the world price of that product. With a per unit tariff in the amount PcPt, price and total quantity sold will be: A) Pt and x. B) Pc and z. C) Pt and y. D) Pa and x. Answer: C

Type: G Topic: 5 E: 701 MI: 457 MA: 367 92. Refer to the above diagram, where Sd and Dd are the domestic supply and demand for a product and Pc is the world price of that product. With a PcPt per unit tariff, the quantities sold by foreign and domestic producers respectively will be: A) xz and x. B) xv and xz. C) x and xz. D) wy and w. Answer: D

Type: G Topic: 5 E: 701 MI: 457 MA: 367 93. Refer to the above diagram, where Sd and Dd are the domestic supply and demand for a product and Pc is the world price of that product. With a PcPt per unit tariff, per unit revenue received by domestic and foreign producers respectively will be: A) Pc and Pa. B) Pa and Pc. C) Pa and Pt. D) Pt and Pc. Answer: D

Type: G Topic: 5 E: 701 MI: 457 MA: 367 94. Refer to the above diagram, where S d and Dd are the domestic supply and demand for a product and Pc is the world price of that product. With a per unit tariff of PcPt, the total amount of tariff revenue collected on this product will be: A) PaPt times wy. B) PcPa times x. C) PcPt times wy. D) PcPt times z. Answer: C

McConnell/Brue: Economics, 16/e

Page 1116

Chapter 37: International Trade

Type: A Topic: 5 E: 701 MI: 457 MA: 367 95. Suppose the United States eliminates high tariffs on German bicycles. As a result, we would expect: A) the price of German bicycles to increase in the United States. B) employment to decrease in the German bicycle industry. C) employment to decrease in the U.S. bicycle industry. D) profits to rise in the U.S. bicycle industry. Answer: C

Type: A Topic: 5 E: 701 MI: 457 MA: 367 96. In effect tariffs on imports are: A) special taxes on domestic producers. B) subsidies to domestic consumers. Answer: D

C) subsidies to foreign producers. D) subsidies for domestic producers.

Use the following to answer questions 97-103: Answer the next question(s) on the basis of the following domestic supply and demand schedules for a product. Suppose that the world price of the product is $1.

Quantity supplied (domestic) 12 10 7 4 1

Price $5 4 3 2 1

Quantity demanded (domestic) 2 4 7 11 16

Type: T Topic: 5 E: 701 MI: 457 MA: 367 97. Refer to the above data. If this nation were entirely closed to international trade, equilibrium price and quantity would be: A) $5 and 2 units. B) $1 and 1 unit. C) $4 and 4 units. D) $3 and 7 units. Answer: D

Type: T Topic: 5 E: 701 MI: 457 MA: 367 98. Refer to the above data. If the economy is opened to free trade and the world price of $1 prevailed, the price and quantity sold of this product would be: A) $1 and 1 unit. B) $1 and 16 units. C) $3 and 7 units. D) $2 and 11 units. Answer: B

Type: T Topic: 5 E: 701 MI: 457 MA: 367 99. Refer to the above data. With free trade, that is, assuming no tariff, the outputs produced by domestic and foreign producers respectively would be: A) 1 unit and 15 units. B) 4 units and 7 units. C) 7 units and 0 units. D) 4 units and 6 units. Answer: A

McConnell/Brue: Economics, 16/e

Page 1117

Chapter 37: International Trade

Type: T Topic: 5 E: 701 MI: 457 MA: 367 100. Refer to the above data. With a $1 dollar per unit tariff, price and total quantity sold will be: A) $3 and 7 units. B) $5 and 2 units. C) $3 and 7 units. D) $2 and 11 units. Answer: D

Type: T Topic: 5 E: 701 MI: 457 MA: 367 101. Refer to the above data. With a $1 per unit tariff, the quantities sold by foreign and domestic producers respectively will be: A) 1 unit and 15 units. B) 7 units and 4 units. C) 11 units and 4 units. D) indeterminate. Answer: B

Type: T Topic: 5 E: 701 MI: 457 MA: 367 102. Refer to the above data. With a $1 per unit tariff, prices (revenue per unit) received by domestic and foreign producers respectively will be: A) $2 and $1. B) $1 and $2. C) $2 and $2. D) $3 and $2. Answer: A

Type: T Topic: 5 E: 701 MI: 457 MA: 367 103. Refer to the above data. The total amount of revenue collected from a $1 per unit tariff on this product will be: A) $22. B) $8. C) $7. D) $14. Answer: C

Type: A Topic: 5 E: 701 MI: 457 MA: 367 104. A protective tariff will: A) increase the sales of foreign exporters. B) increase the price and sales of domestic producers. C) increase the welfare of domestic consumers. D) create an efficiency gain in the domestic economy. Answer: B

Type: A Topic: 5 E: 702 MI: 458 MA: 368 105. Other things equal, a tariff is: A) superior to an import quota for Americans because a tariff increases the profits of foreign producers. B) inferior to an import quota for Americans because a tariff increases the profits of domestic producers. C) superior to an import quota for Americans because a tariff generates revenue for the United States Treasury. D) inferior to an import quota for Americans because a tariff generates revenue for the United States Treasury. Answer: C

Type: A Topic: 5 E: 702 MI: 458 MA: 368 106. In comparing a tariff and an import quota we find that: A) the tariff and quota both generate the same amount of revenue for the United States Treasury. B) the tariff generates revenue for the United States Treasury but the quota does not. C) the quota generates revenue for the United States Treasury but the tariff does not. D) neither the tariff nor the quota generates revenue for the United States Treasury. Answer: B

McConnell/Brue: Economics, 16/e

Page 1118

Chapter 37: International Trade

Type: A Topic: 5 E: 703 MI: 459 MA: 369 107. Other things equal, economists would prefer: A) free trade to tariffs and tariffs to import quotas. B) free trade to import quotas and import quotas to tariffs. C) import quotas to tariffs and tariffs to voluntary export restrictions. D) import quotas to free trade and free trade to tariffs. Answer: A

Use the following to answer questions 108-113:

Type: G Topic: 5 E: 701 MI: 457 MA: 367 108. Refer to the above diagram, where Sd and Dd are the domestic supply and demand for a product and Pc is the world price of that product. Sd + Q is the product supply curve after an import quota is imposed. The size of the import quota: A) is vz. B) is vy. C) is wy. D) cannot be determined. Answer: C

Type: G Topic: 5 E: 701 MI: 457 MA: 367 109. Refer to the above diagram, where Sd and Dd are the domestic supply and demand for a product and Pc is the world price of that product. Sd + Q is the product supply curve after an import quota is imposed. The effect of the import quota on domestic price and domestic consumption is: A) the same as that of a tariff of PcPt. B) the same as that of a tariff of PtPa. C) the same as that of a tariff of PcPa. D) to raise price by more and reduce consumption less than a tariff of PcPt. Answer: A

Type: G Topic: 5 E: 701 MI: 457 MA: 367 110. Refer to the above diagram, where Sd and Dd are the domestic supply and demand for a product and Pc is the world price of that product. Sd + Q is the product supply curve after an import quota is imposed. Assuming there is no tariff, the import quota: A) will increase United States Treasury revenue by areas G + H . B) will increase United States Treasury revenue by areas E + F + G + H + J . C) may either increase or decrease the total revenues of foreign producers, depending on the elasticity of domestic demand. D) will increase the revenues of foreign firms by area E. Answer: C

McConnell/Brue: Economics, 16/e

Page 1119

Chapter 37: International Trade

Type: G Topic: 5 E: 701 MI: 457 MA: 367 111. Refer to the above diagram, where Sd and Dd are the domestic supply and demand for a product and Pc is the world price of that product. Sd + Q is the product supply curve after an import quota is imposed. A tariff of Pc Pt or an import quota of wy will: A) have the same effect on the volume of imports. B) have the same effect on domestic price. C) have the same effect on the revenues of domestic producers. D) do all of the above. Answer: D

Type: G Topic: 5 E: 701 MI: 457 MA: 367 112. Refer to the above diagram, where Sd and Dd are the domestic supply and demand for a product and Pc is the world price of that product. Sd + Q is the product supply curve after an import quota is imposed. A tariff of Pc Pt will: A) lower domestic price and increase domestic consumption. B) increase the revenues of domestic producers by areas E + F + K . C) increase the revenues of domestic producers by areas G + H . D) increase the revenues of domestic producers by areas E + F + G + H + J . Answer: B

Type: G Topic: 5 E: 701 MI: 457 MA: 367 113. Refer to the above diagram, where Sd and Dd are the domestic supply and demand for a product and Pc is the world price of that product. Sd + Q is the product supply curve after an import quota is imposed. A quota of wy will: A) lower domestic price and increase domestic consumption. B) increase the revenues of domestic producers by areas E + F + K . C) increase the revenues of domestic producers by areas G + H . D) increase the revenues of domestic producers by areas E + F + G + H + J . Answer: B

Type: A Topic: 5 E: 706 MI: 462 MA: 372 114. Which of the following statements is false? A) Studies show that developing nations that have relied on import restrictions to protect domestic industries have had higher growth rates than similar nations pursuing more open economic policies. B) The United States Constitution forbids individual states from levying tariffs. C) The high tariffs of the Smoot-Hawley Act of 1930 and the retaliation they caused worsened the Great Depression. D) The European Union has enhanced prosperity in Western Europe. Answer: A

Type: A Topic: 5 E: 706 MI: 462 MA: 372 115. As it relates to income distribution, the domestic overcharge resulting from tariffs and quotas is: A) neutral. B) regressive. C) progressive. D) elastic. Answer: B

McConnell/Brue: Economics, 16/e

Page 1120

Chapter 37: International Trade

Type: A Topic: 5 E: 706 MI: 462 MA: 372 116. Studies show that: A) it is impossible to estimate the benefits of trade barriers. B) costs and benefits of trade barriers are about equal. C) benefits of trade barriers exceed their costs in developing nations. D) costs of trade barriers exceed their benefits, creating an efficiency loss for society. Answer: D

Type: A Topic: 5 E: 706 MI: 462 MA: 372 117. Research studies indicate that: A) U.S. producers gain more from tariffs than U.S. consumers lose. B) the costs of trade restrictions are proportionately higher for high-income groups than for low-income groups. C) the revenue from tariffs equals the total cost that tariffs impose on consumers. D) U.S. consumers lose more from tariffs than U.S. producers gain. Answer: D

Type: A Topic: 5 E: 706 MI: 462 MA: 372 118. A high tariff on imported good X might reduce domestic employment in industry Y if: A) X is an input used domestically in producing Y. C) X is an inferior good. B) X and Y are substitute goods. D) Y is an inferior good. Answer: A

Arguments for protectionism

Type: D Topic: 6 E: 703 MI: 459 MA: 369 119. The increased-domestic-employment argument for tariff protection holds that: A) domestic inflation is a desirable policy goal because it stimulates exports. B) domestic deflation is a desirable policy goal because it stimulates imports. C) an increase in tariffs will reduce net exports and stimulate domestic employment. D) an increase in tariffs will increase net exports and stimulate domestic employment. Answer: D

Type: A Topic: 6 E: 704 MI: 460 MA: 370 120. Which of the following arguments comes closest to constituting a legitimate economic exception to the case for free trade? A) the increase-domestic-employment argument C) the diversification-for-stability argument B) the cheap-foreign-labor argument D) the infant-industry argument Answer: D

Type: A Topic: 6 E: 704 MI: 460 MA: 389 121. The infant industry argument for tariffs is criticized: A) because it is difficult to determine which industries merit protection. B) because direct subsidies are probably a better means of stimulating such industries. C) because the tariffs may remain after the industry reaches maturity. D) for all of the above reasons. Answer: D

McConnell/Brue: Economics, 16/e

Page 1121

Chapter 37: International Trade

Type: A Topic: 6 E: 705 MI: 461 MA: 371 122. A major difficulty with the argument that trade barriers are necessary because foreign workers are paid low wages is that: A) labor costs and product prices are not related. B) there is no discernible relationship between wage rates and labor productivity. C) wage rates and labor productivity are directly related. D) wage rates and labor productivity are inversely related. Answer: C

Type: D Topic: 6 E: 705 MI: 461 MA: 371 123. As it relates to international trade, dumping: A) is a form of price discrimination illegal under U.S. antitrust laws. B) is the practice of selling goods in a foreign market at less than cost. C) constitutes a general case for permanent tariffs. D) is defined as selling more goods than allowed by an import quota. Answer: B

Type: A Topic: 6 E: 705 MI: 461 MA: 371 124. Dumping of goods abroad: A) constitutes a general case for permanent tariffs. B) may be part of a firm's price discrimination strategy. C) may be part of a nation's strategy to rectify its trade deficit. D) drives up prices of the dumped goods. Answer: B

Type: A Topic: 6 E: 704 MI: 460 MA: 370 125. As used in strategic trade policy, tariffs are a variation of the: A) military self-sufficiency argument for tariffs. B) increased-domestic-employment argument for tariffs. C) diversification-for-stability argument for tariffs. D) infant-industry argument for tariffs. Answer: D

World Trade Organization

Type: F Topic: 7 E: 707-708 MI: 463-464 MA: 373-374 126. The World Trade Organization: A) is also known as the International Monetary Fund (IMF). B) is also known as NAFTA. C) was established to resolve disputes arising under world trade rules. D) enhances world trade by providing interest rate subsidies to foreign borrowers who buy exports on credit. Answer: C

Type: F Topic: 7 E: 708 MI: 464 MA: 374 127. The number of countries belonging to the World Trade Organization (WTO) currently is about. A) 145. B) 100. C) 80. D) 22. Answer: A

McConnell/Brue: Economics, 16/e

Page 1122

Chapter 37: International Trade

Type: F Topic: 7 E: 707-708 MI: 463-464 MA: 373-374 128. The organization created to oversee the provisions of multilateral trade agreements, resolve disputes under the international trade rules, and meet periodically to consider further trade liberalization is called the: A) International Monetary Fund (IMF). C) Common Market Organization (CMO). B) World Trade Organization (WTO). D) International Trade Commission (ITC). Answer: B

Consider This Questions

Type: F E: 704 MI: 460 MA: 370 Status: New 129. (Consider This) The textbook authors apply the metaphor "shooting yourself in the foot" to: A) trade wars and trade boycotts. C) import quotas. B) fixed exchange rates. D) voluntary export restraints. Answer: A

Type: A E: 704 MI: 460 MA: 370 Status: New 130. (Consider This) According to the textbook, trade wars and trade boycotts are much like: A) "killing two birds with the same stone." B) "closing the barn door after the horses have left." C) "shooting yourself in the foot." D) "making a mountain out of a molehill." Answer: C

Last Word Questions

Type: A E: 706 MI: 462 MA: 372 131. (Last Word) Critics of the World Trade Organization (WTO) say that liberalized world trade does all of the following except: A) produce environmental degradation. B) allow producers to circumvent labor protections such as workplace safety, child labor restrictions, and collective bargaining rights. C) helps developing nations escape from poverty. D) promote the interests of multinational corporations. Answer: C

Type: A E: 706 MI: 462 MA: 372 132. (Last Word) Which of the following groups is most likely to support the World Trade Organization (WTO)? A) anarchists. B) extreme environmentalists. C) labor union leaders in industrially advanced countries. D) economists. Answer: D

McConnell/Brue: Economics, 16/e

Page 1123

Chapter 37: International Trade

True/False Questions

Type: F E: 692 MI: 448 MA: 358 133. It is impossible for a nation to have a comparative advantage in producing everything. Answer: True

Type: F E: 697 MI: 453 MA: 363 134. A side benefit of international trade is that it links national interests and increases the opportunity costs of war. Answer: True

Type: A E: 693 MI: 449 MA: 359 135. The nation that has a comparative advantage in a particular product will be the only world exporter of that product. Answer: False

Type: A E: 693 MI: 449 MA: 359 136. International trade based on the principle of comparative advantage creates a more efficient allocation of world economic resources. Answer: True

Type: A E: 696 MI: 452 MA: 362 137. The law of increasing opportunity costs limits international specialization. Answer: True

Type: F E: 690 MI: 446 MA: 356 138. The percentage of the United States' domestic output that is derived from international trade is higher than that for any other industrially advanced nation. Answer: False

Use the following to answer questions 139-143: Answer the next question(s) on the basis of the following information. Assume that by devoting all its resources to the production of X, nation Alpha can produce 40 units of X. By devoting all its resources to Y, Alpha can produce 60Y. Comparable figures for nation Beta are 60X and 40Y.

Type: T E: 693-694 MI: 449-450 MA: 359-360 139. Refer to the above information. Alpha should specialize in Y and Beta in X. Answer: True

Type: T E: 694 MI: 450 MA: 360 140. Refer to the above information. The terms of trade will be at or within the 1X = 11/2Y to 1X= 2/3Y range. Answer: True

McConnell/Brue: Economics, 16/e

Page 1124

Chapter 37: International Trade

Type: T E: 694 MI: 450 MA: 360 141. Refer to the above information. Alpha would prefer terms of trade at, or close to, 1X = 2/3Y. Answer: True

Type: T E: 694 MI: 450 MA: 360 142. Refer to the above information. Beta would prefer terms of trade at, or close to, 1X = 1 1/2Y. Answer: True

Type: T E: 694 MI: 450 MA: 360 143. Refer to the above information. If Alpha had produced 20X and 30Y and Beta had produced 30X and 20Y before specialization and trade, then we can say that the gains from specialization and trade are 10X and 10Y. Answer: True

Type: F E: 697-698 MI: 453-454 MA: 363-364 Status: New 144. A nation's export supply curve is downsloping and its import demand curve is upsloping. Answer: False

Type: F E: 698 MI: 454 MA: 364 Status: New 145. The equilibrium world price of a product equates the quantities of exports supplied and imports demanded. Answer: True

Type: A E: 703 MI: 459 MA: 369 Status: New 146. Economists prefer free trade to tariffs and prefer tariffs to import quotas. Answer: True

Type: A E: 702 MI: 458 MA: 368 Status: New 147. Tariffs create larger gains to domestic producers than losses to domestic consumers. Answer: False

Type: F E: 706 MI: 462 MA: 372 148. During the Great Depression most nations lowered tariffs and abolished import quotas to encourage the flow of trade. Answer: False

Type: A E: 706 MI: 462 MA: 372 149. Barriers to free trade impair efficiency in the international allocation of resources. Answer: True

Type: F E: 707-708 MI: 463-464 MA: 373-374 150. The World Trade Organization is comprised of 25 European nations and dedicated to abolishing trade barriers and integrating their economies. Answer: False

McConnell/Brue: Economics, 16/e

Page 1125

Chapter 37: International Trade

Type: F E: 707 MI: 463 MA: 373 151. The World Trade Organization (WTO) is an international organization designed to provide short-term advances of foreign monies to those nations faced with trade deficits. Answer: False

McConnell/Brue: Economics, 16/e

Page 1126

CHAPTER 38

Exchange Rates, the Balance of Payments, and Trade Deficits

Topic 1. 2. 3. 4. 5. 6. 7. 8. Financing international trade Balance of payments Exchange rates Floating exchange rates; fixed exchange rates Gold standard Bretton Woods system Managed float U.S. trade deficits Consider This Last Word True-False

Question numbers 1-8 9-64 65-78 79-113 114-120 121-123 124-132 133-140 141-142 143-145 146-163

____________________________________________________________

_______________________________________

____________________________________________________________

_______________________________________

Multiple Choice Questions Financing international trade

Type: A Topic: 1 E: 712 MI: 468 MA: 378 1. U.S. export transactions create: A) a U.S. demand for foreign monies and the satisfaction of this demand decreases the supplies of dollars held by foreign banks. B) a U.S. demand for foreign monies and the satisfaction of this demand increases the supplies of dollars held by foreign banks. C) a foreign demand for dollars and the satisfaction of this demand decreases the supplies of foreign monies held by U.S. banks. D) a foreign demand for dollars and the satisfaction of this demand increases the supplies of foreign monies held by U.S. banks. Answer: D

Type: A Topic: 1 E: 712 MI: 468 MA: 378 2. U.S. import transactions create: A) a foreign demand for dollars and the satisfaction of this demand decreases the supplies of foreign monies held by U.S. banks. B) a foreign demand for dollars and the satisfaction of this demand increases the supplies of foreign monies held by U.S. banks. C) a U.S. demand for foreign monies and the satisfaction of this demand decreases the supplies of foreign monies held by U.S. banks. D) a U.S. demand for foreign monies and the satisfaction of this demand increases the supplies of dollars held by foreign banks. Answer: C

Chapter 38: Exchange Rates, the Balance of Payments, and Trade Deficits

Type: A Topic: 1 E: 711-712 MI: 467-468 MA: 377-378 3. If a U.S. importer can purchase 10,000 pounds for $20,000, the rate of exchange is: A) $1 = 2 pounds in the United States. C) $1 = 2 pounds in Great Britain. B) $2 = 1 pound in the United States. D) $.5 = 1 pound in Great Britain. Answer: B

Type: A Topic: 1 E: 712 MI: 468 MA: 378 4. Which of the following creates a supply of Canadian dollars in foreign exchange markets? A) a Frenchman redeems a bond issued by a Canadian manufacturer B) a Canadian exporter buys insurance from a U.S. firm C) an American student takes a summer trip to Canada D) a U.S. importer buys 500 cases of Canadian maple syrup Answer: B

Type: A Topic: 1 E: 712 MI: 468 MA: 378 5. Other things equal, the financing of a U.S. export transaction: A) reduces U.S. interest rates. B) decreases the supplies of foreign currency held by United States banks. C) decreases GDP in the United States. D) increases the supplies of foreign currency held by U.S. banks. Answer: D

Type: A Topic: 1 E: 712 MI: 468 MA: 378 6. Other things equal, the financing of a U.S. import transaction: A) increases the supplies of foreign currency held by United States banks. B) increases U.S. interest rates. C) decreases the supplies of foreign currency held by U.S. banks. D) increases GDP in the United States. Answer: C

Type: A Topic: 1 E: 712 MI: 468 MA: 378 Status: New 7. Travel by U.S. citizens within Europe creates a: A) demand for euros and a supply of dollars. B) both a supply of dollars and a demand for dollars. C) demand for dollars and a supply of euros. D) supply of euros and a demand for dollars. Answer: A

Type: A Topic: 1 E: 712 MI: 468 MA: 378 Status: New 8. Purchases of land in the United States by people from Canada create a: A) supply of U.S. dollars and a demand for Canadian dollars. B) both a supply of U.S. dollars and a demand for U.S. dollars. C) demand for Canadian dollars and a supply of U.S. dollars D) demand for U.S. dollars and a supply of Canadian dollars. Answer: D

McConnell/Brue: Economics, 16/e

Page 1128

Chapter 38: Exchange Rates, the Balance of Payments, and Trade Deficits

Balance of payments

Type: A Topic: 2 E: 712 MI: 468 MA: 378 9. Which of the following would call for inpayments to the United States? A) gold flows into the United States C) U.S. sends foreign aid to developing countries B) U.S. firms sell insurance to Brazilian shippers D) U.S. imports German automobiles Answer: B

Type: A Topic: 2 E: 713 MI: 469 MA: 379 10. Which of the following would call for outpayments from the United States? A) U.S. exports computer software B) U.S. purchases assets abroad C) foreigners purchase assets in the United States D) foreign tourists spend money in the United States. Answer: B

Type: A Topic: 2 E: 712-713 MI: 468-469 MA: 378-379 11. The current account in a nation's balance of payments includes: A) its goods exports and imports, and its services exports and imports. B) changes in its official reserves. C) purchases of foreign assets, and foreign purchases of assets. D) all of the above. Answer: A

Type: A Topic: 2 E: 714 MI: 470 MA: 380 12. A nation's capital account: A) contains inpayment items, but not outpayment items. B) includes service exports and service imports. C) includes both inpayments and outpayments. D) includes net investment income and net transfers. Answer: C

Type: F Topic: 2 E: 714 MI: 470 MA: 380 13. In 2001, the capital account in the U.S. balance of payments was in: A) deficit, and larger than the current account surplus. B) surplus, and larger than the current account surplus. C) balance, with no deficit or surplus. D) surplus, and smaller than the current account deficit. Answer: D

Type: D Topic: 2 E: 714 MI: 470 MA: 380 14. The capital account balance is a nation's: A) net investment income minus its net transfers. B) exports of goods and services minus its imports of goods and services. C) sale of real and financial assets to people living abroad minus its purchases of real and financial assets from foreigners. D) domestic investment spending minus domestic saving. Answer: C

McConnell/Brue: Economics, 16/e

Page 1129

Chapter 38: Exchange Rates, the Balance of Payments, and Trade Deficits

Type: A Topic: 2 E: 714 MI: 470 MA: 380 15. A nation's official reserves account: A) compensates for differences in the current and capital accounts. B) is always positive. C) is always zero. D) is always negative. Answer: A

Type: A Topic: 2 E: 714 MI: 470 MA: 380 16. If a nation has a current account surplus and its official reserves account balance is zero, it must have a: A) surplus in its capital account. C) balance of payments surplus. B) balance of payments deficit. D) deficit in its capital account Answer: D

Type: A Topic: 2 E: 714 MI: 470 MA: 380 17. If a nation has a current account deficit and its official reserves account balance is zero, it must have a: A) surplus in its capital account. C) balance of payments surplus. B) balance of payments deficit. D) deficit in its capital account. Answer: A

Type: A Topic: 2 E: 715 MI: 471 MA: 381 18. In the U.S. balance of payments, foreign purchases of assets in the United States are a: A) foreign currency outflow. C) current account item. B) foreign currency inflow. D) debit, or outpayment Answer: B

Type: A Topic: 2 E: 715 MI: 471 MA: 381 19. In the U.S. balance of payments, U.S. purchases of assets abroad are a: A) U.S. dollar outflow. B) U.S. dollar inflow. C) current account item. D) debit, or outpayment Answer: A

Type: A Topic: 2 E: 715 MI: 471 MA: 381 20. Which of the following combinations is plausible, as it relates to a nation's balance of payments? A) current account = $ + 40 billion; capital account = $ + 20 billion; official reserves account = $ - 50 billion. B) current account - $ + 50 billion; capital account = $ - 20 billion; official reserves account = $ + 30 billion. C) current account = $ + 10 billion; capital account = $ + 40 billion; official reserves account = $ + 50 billion. D) current account = $ + 30 billion; capital account = $ - 20 billion; official reserves account = $ - 10 billion. Answer: D

McConnell/Brue: Economics, 16/e

Page 1130

Chapter 38: Exchange Rates, the Balance of Payments, and Trade Deficits

Type: A Topic: 2 E: 715 MI: 471 MA: 381 21. Which of the following combinations is plausible, as it relates to a nation's balance of payments? A) current account = $ + 40 billion; capital account = $ + 20 billion; official reserves account = $ - 50 billion. B) current account = $ - 50 billion; capital account = $ + 20 billion; official reserves account = $ + 30 billion. C) current account = $ + 10 billion; capital account = $ + 40 billion; official reserves account = $ + 50 billion. D) current account = $ + 30 billion; capital account = $ - 20 billion; official reserves account = $ - billion. Answer: B

Type: A Topic: 2 E: 715 MI: 471 MA: 381 22. There must always be a balance of a nation's: A) goods exports and gold imports. B) total international payments. Answer: B

C) imports and exports of goods and services. D) net transfers and net investment income.

Type: C Topic: 2 E: 715 MI: 471 MA: 381 23. Which of the following would contribute to a United States balance of payments surplus? A) the United States makes a unilateral tariff reduction on imported goods B) General Motors pays a dividend to a Swiss stockholder C) the United States cuts back on U.S. military personnel stationed in Germany D) Russian vodka becomes increasingly popular in the United States Answer: C

Type: C Topic: 2 E: 715 MI: 471 MA: 381 24. Which of the following would contribute to a United States balance of payments deficit? A) Kawasaki builds a motorcycle manufacturing plant in Kansas City B) United States tourists travel in large numbers to Europe C) a wealthy Mexican citizen builds a mansion in Beverly Hills D) Zaire pays interest on its debt to the United States Answer: B

Type: A Topic: 2 E: 715 MI: 471 MA: 381 25. Evidence of a chronic balance of payments deficit is: A) a decline in amount of the nation's currency held by other nations. B) an excess of exports over imports. C) diminishing reserves of foreign currencies. D) an increase in the international value of the nation's currency. Answer: C

McConnell/Brue: Economics, 16/e

Page 1131

Chapter 38: Exchange Rates, the Balance of Payments, and Trade Deficits

Use the following to answer questions 26-33: The following table contains hypothetical data for the 2003 U.S. balance of payments. Answer the next question(s) on the basis of this information. All figures are in billions of dollars.
1) 2) 3) 4) 5) 6) 7) 8) 9) U.S. goods exports U.S. goods imports U.S. service exports U.S. service imports Net investment income Net transfers Foreign purchases of assets in the United States U.S. purchases of foreign assets abroad Official reserves +$100 -80 +40 -90 +20 -15 +30 -10 +5

Type: T Topic: 2 E: 713 MI: 469 MA: 379 26. Refer to the above data. The United States has a balance of goods: A) deficit of $10 billion. C) deficit of $30 billion. B) surplus of $30 billion. D) surplus of $20 billion. Answer: D

Type: T Topic: 2 E: 713 MI: 469 MA: 379 27. Refer to the above data. The U.S. balance on goods and services is a: A) $10 billion deficit. B) $20 billion deficit. C) $30 billion surplus. Answer: D

D) $30 billion deficit.

Type: T Topic: 2 E: 713-714 MI: 469-470 MA: 379-380 28. Refer to the above data. The U.S. balance on current account is a: A) $40 billion surplus. B) $25 billion deficit. C) $25 billion surplus. Answer: B

D) $30 billion deficit.

Type: T Topic: 2 E: 713 MI: 469 MA: 379 29. Item (6) above indicates that: A) the United States used $15 billion of its international monetary reserves to balance its international payments. B) the United States provided $15 billion of foreign aid to developing nations. C) Americans provided a net amount of $15 billion in remittances to the rest of the world. D) Americans received a net amount of $15 billion in remittances from the rest of the world. Answer: C

Type: T Topic: 2 E: 713 MI: 469 MA: 379 30. Item (5) above indicates: A) that the United States' current account was in surplus. B) the size of the net inflow of foreign investment to the United States that occurred in 2001. C) the net amount Americans received as interest and dividends on existing U.S. investments abroad. D) the net amount Americans paid as interest and dividends on existing foreign investments in the United States. Answer: C

McConnell/Brue: Economics, 16/e

Page 1132

Chapter 38: Exchange Rates, the Balance of Payments, and Trade Deficits

Type: T Topic: 2 E: 713-714 MI: 469-470 MA: 379-380 31. Refer to the above data. The United States' balance on capital account is a: A) $20 billion surplus. B) $15 billion surplus. C) $30 billion deficit. D) $20 billion deficit. Answer: A

Type: T Topic: 2 E: 712 MI: 468 MA: 378 32. Refer to the above data. The United States has a balance of payments: A) surplus of $15. B) deficit of $10. C) surplus of $5. D) deficit of $5. Answer: D

Type: T Topic: 2 E: 713-714 MI: 469-470 MA: 379-380 33. Item (9) above indicates that: A) exchange rates are freely floating. B) the United States is adding to its stock of foreign currencies. C) the United States is drawing down its stock of foreign currencies. D) the United States has a balance of payments surplus. Answer: C

Type: A Topic: 2 E: 713-714 MI: 469-470 MA: 379-380 34. If a nation's balance on current account is a negative $200 billion, while its balance on capital account is a positive $175 billion, we can conclude with certainty that this nation has a: A) goods trade deficit. C) reduction in its stock of foreign currency. B) goods trade surplus. D) balance of payments surplus. Answer: C

Type: A Topic: 2 E: 712-713 MI: 468-469 MA: 378-379 35. If a nation's goods exports are $55 billion, while its goods imports are $50 billion, we can conclude with certainty that this nation has a: A) balance of trade (goods) surplus. C) positive balance on current account. B) balance of payments surplus. D) positive balance on goods and services. Answer: A

Type: A Topic: 2 E: 727 MI: 483 MA: 393 36. It may be misleading to label a trade deficit as unfavorable or adverse because: A) the multiplier does not apply to a trade deficit. B) a trade deficit increases a nation's aggregate output and employment. C) a nation's consumers benefit from a trade deficit during the period it occurs. D) a trade deficit precludes inflation. Answer: C

Type: A Topic: 2 E: 713 MI: 469 MA: 379 37. Which of the following is not included in the current account of a nation's balance of payments? A) its goods exports C) its net investment income B) its goods imports D) its purchases of real assets abroad Answer: D

McConnell/Brue: Economics, 16/e

Page 1133

Chapter 38: Exchange Rates, the Balance of Payments, and Trade Deficits

Type: A Topic: 2 E: 715 MI: 471 MA: 381 38. A deficit on the current account: A) normally causes a surplus on the capital account. B) normally causes a deficit on the capital account. C) has no relationship to the capital account. D) means that a nation is making international transfers. Answer: A

Use the following to answer questions 39-43: Answer the next question(s) on the basis of the following 2004 balance of payments data (+ and -) for the hypothetical nation of Zabella. All figures are in billions of dollars.
Current Account 1) Goods exports 2) Goods imports 3) Exports of services 4) Imports of services 5) Net investment income 6) Net transfers Capital Account 7) Foreign purchases of assets in the United States 8) U.S. purchases of assets abroad Official Reserves Account 9) Official reserves +$80 -70 +20 -25 +5 -5 +13 -23 +5

Type: T Topic: 2 E: 713 MI: 469 MA: 379 39. Refer to the above data. Zabella has a balance of trade (goods): A) deficit of $10 billion. C) surplus of $10 billion. B) surplus of $5 billion. D) deficit of $5 billion. Answer: C

Type: T Topic: 2 E: 713 MI: 469 MA: 379 40. Refer to the above data. Zabella's balance on goods and services shows a: A) $5 billion deficit. B) $5 billion surplus. C) $10 billion surplus. D) $15 billion deficit. Answer: B

Type: T Topic: 2 E: 714 MI: 470 MA: 380 41. Refer to the above data. Zabella's balance on capital account shows a: A) deficit of $10 billion. C) deficit of $28 billion. B) surplus of $5 billion. D) surplus of $13 billion. Answer: A

Type: T Topic: 2 E: 715 MI: 471 MA: 381 42. Refer to the above data. Zabella has a balance of payments: A) deficit of $5 billion. C) deficit of $10 billion. B) surplus of $10 billion. D) surplus of $5 billion. Answer: A

McConnell/Brue: Economics, 16/e

Page 1134

Chapter 38: Exchange Rates, the Balance of Payments, and Trade Deficits

Type: T Topic: 2 E: 714 MI: 470 MA: 380 43. Refer to the above data. The official reserves account indicates that Zabella: A) added $5 billion to its stock of foreign currencies. B) imported more goods and services than it exported. C) "exported" $5 billion of its stock of foreign currencies. D) experienced a balance of payments surplus in 2001. Answer: C

Use the following to answer questions 44-50: The plus items below are "export-type" entries and the minus items are "import-type" entries in the balance of payments for the hypothetical country of Zippo. 1) 2) 3) 4) 5) 6) 7) 8) 9) Goods exports Official reserves Net transfers Imports of services Net investment income U.S. purchases of assets abroad Goods imports Foreign purchases of assets in the United States Export of services +$200 0 0 -100 0 -50 -250 +150 +50

Type: T Topic: 2 E: 712-713 MI: 468-469 MA: 378-379 44. Refer to the above information. The current account items for Zippo are: A) 1, 2, 3, and 4. B) 1, 3, 4, 5, 7, and 9. C) 6 and 8. D) 1, 2, 4, 7, and 9. Answer: B

Type: T Topic: 2 E: 713-714 MI: 469-470 MA: 379-380 45. Refer to the above information. The capital account items for Zippo are: A) 1, 2, 3, and 4. B) 1, 3, 4, 5, 7, and 9. C) 6 and 8. D) 1, 2, 4, 7, and 9. Answer: C

Type: T Topic: 2 E: 714 MI: 470 MA: 380 46. Refer to the above information. Zippo has a: A) current account surplus. B) capital account deficit. Answer: C

C) capital account surplus. D) surplus on goods and services.

Type: T Topic: 2 E: 713-714 MI: 469-470 MA: 379-380 47. Refer to the above information. Zippo has a: A) current account deficit. C) balance of payments deficit. B) capital account deficit. D) trade surplus on goods and services. Answer: A

McConnell/Brue: Economics, 16/e

Page 1135

Chapter 38: Exchange Rates, the Balance of Payments, and Trade Deficits

Type: T Topic: 2 E: 715 MI: 471 MA: 381 48. Refer to the above information. Zippo has: A) a current account surplus. B) a capital account deficit. C) a trade surplus on goods and services. D) neither a balance of payments deficit nor a surplus. Answer: D

Type: T Topic: 2 E: 715 MI: 471 MA: 381 49. Refer to the above information. Zippo has a: A) current account surplus. B) capital account deficit. C) trade deficit on goods and services. D) balance of services surplus. Answer: C

Type: T Topic: 2 E: 715 MI: 471 MA: 381 50. Refer to the above information. On the basis of its balance of payments position, and other things equal, we can expect the international value of Zippo's currency to: A) increase. B) decrease. C) remain constant. D) gyrate up and down. Answer: C

Type: A Topic: 2 E: 715 MI: 471 MA: 381 51. In the balance of payments of the United States, a reduction of U.S. holdings of official reserves of foreign currencies is recorded as a: A) current account entry. B) negative entry. C) net transfer. D) positive entry. Answer: D

Type: A Topic: 2 E: 715 MI: 471 MA: 381 52. In the balance of payments of the United States, U.S. goods imports are recorded as a: A) positive entry. B) capital account entry. C) current account entry. D) official reserves entry. Answer: C

Type: A Topic: 2 E: 714 MI: 470 MA: 380 53. In the balance of payments of the United States, inflows of foreign currencies to the United States are recorded as: A) a positive entry. B) a current account entry. C) official reserves. D) net investment income. Answer: A

Type: A Topic: 2 E: 713-714 MI: 469-470 MA: 379-380 54. Which one of the following will not directly affect the U.S. balance on current account? A) an increase in U.S. goods imports C) an increase in U.S. purchases of assets abroad B) a decrease in U.S. net investment income D) an increase in U.S. imports of services Answer: C

McConnell/Brue: Economics, 16/e

Page 1136

Chapter 38: Exchange Rates, the Balance of Payments, and Trade Deficits

Type: A Topic: 2 E: 713 MI: 469 MA: 379 55. Which one of the following, other things equal, will directly alter the United States balance of trade? A) an increase in official reserves C) an increase in net transfers B) a decrease in U.S. goods exports D) a decrease in U.S. purchases of assets abroad. Answer: B

Type: A Topic: 2 E: 713 MI: 469 MA: 379 56. Which item below will affect the U.S. balance on goods and services, but not affect its balance of trade in goods? A) an increase in U.S. goods exports C) an increase in official reserves B) a decrease in U.S. exports of services D) an increase in net transfers Answer: B

Type: A Topic: 2 E: 713 MI: 469 MA: 379 57. In a nation's balance of payments, which one of the following items is always recorded as a positive entry? A) goods imports C) U.S. purchases of assets abroad B) changes in foreign currency reserves D) exports of services Answer: D

Type: F Topic: 2 E: 715 MI: 471 MA: 381 Status: New 58. In 2002 the United States held official reserves of foreign currency of about: A) $435 billion. B) $210 billion. C) $150 billion. D) $80 billion. Answer: D

Type: A Topic: 2 E: 714 MI: 470 MA: 380 Status: New 59. Suppose the balance on the capital account is + $200 billion and the official reserve account is + $2 billion. The size of the current account is: A) + $200 billion. B) - $202 billion. C) - $198 billion. D) + $2 billion. Answer: B

Type: A Topic: 2 E: 714 MI: 470 MA: 380 Status: New 60. Suppose the balance on the capital account is - $300 billion and the official reserve account is + $5 billion. The size of the current account is: A) + $295 billion. B) - $295 billion. C) + $305 billion. D) + $5 billion. Answer: A

Type: A Topic: 2 E: 714 MI: 470 MA: 380 Status: New 61. Suppose the balance on the current account is + $100 billion and the official reserve account is - $1 billion. The size of the current account is: A) + $101 billion. B) - $100 billion. C) - $99 billion. D) + $101 billion. Answer: C

Type: A Topic: 2 E: 714 MI: 470 MA: 380 Status: New 62. Suppose the balance on the current account is + $50 billion and the official reserve account is + $1 billion. The size of the current account is: A) - $51 billion. B) - $50 billion. C) - $49 billion. D) + $51 billion. Answer: A

McConnell/Brue: Economics, 16/e

Page 1137

Chapter 38: Exchange Rates, the Balance of Payments, and Trade Deficits

Type: A Topic: 2 E: 715 MI: 471 MA: 381 Status: New 63. In the U.S. balance of payments account for a certain year, a positive number in the official reserve account means a: A) buildup of the nation's total stock of official reserves. B) drawdown of the nation's total stock of official reserves. C) buildup of total foreign debt. D) reduction of total foreign debt. Answer: B

Type: A Topic: 2 E: 715 MI: 471 MA: 381 Status: New 64. In the U.S. balance of payments account for a certain year, a negative number in the official reserve account means a: A) reduction of foreign debt. C) buildup of foreign debt. B) drawdown of the nation's official reserves. D) buildup of the nation's official reserves. Answer: D

Exchange rates

Type: D Topic: 3 E: 711 MI: 467 MA: 377 65. A market in which the money of one nation is exchanged for the money of another nation is a: A) resource market. B) bond market. C) stock market. D) foreign exchange market. Answer: D

Type: A Topic: 3 E: 715 MI: 471 MA: 381 66. If the dollar price of yen rises, then: A) the yen price of dollars also rises. B) the dollar depreciates relative to the yen. Answer: B

C) the yen depreciates relative to the dollar. D) the dollar will buy fewer U.S. goods.

Type: A Topic: 3 E: 716 MI: 472 MA: 382 67. If the exchange rate between the U.S. dollar and the Japanese yen is $1 = 200 yen, then the dollar price of yen is: A) $.005. B) $.05. C) $.50. D) $5. Answer: A

Type: A Topic: 3 E: 716 MI: 472 MA: 382 68. The following are hypothetical exchange rates: $1 = 140 yen; 1 Swiss franc = $.10. We can conclude that: A) 1 yen = 280 Swiss francs. C) 1 Swiss franc = 28 yen. B) 1 yen = 14 Swiss francs. D) 1 Swiss franc = 14 yen. Answer: D

Type: A Topic: 3 E: 716 MI: 472 MA: 382 69. The following are hypothetical exchange rates: 2 euros = 1 pound; $1 = 2 pounds. We can conclude that: A) $1 = 4 euros. B) $1 = .5 euros. C) 1 euros = $.50. D) 1 euro = $2. Answer: A

McConnell/Brue: Economics, 16/e

Page 1138

Chapter 38: Exchange Rates, the Balance of Payments, and Trade Deficits

Type: A Topic: 3 E: 716 MI: 472 MA: 382 70. If the rate of exchange for a pound is $4, the rate of exchange for the dollar is: A) 1/4 pound. B) 4 pounds. C) $.25. D) $1.00. Answer: A

Type: A Topic: 3 E: 716 MI: 472 MA: 382 71. In considering yen and dollars, when the dollar rate of exchange for the yen rises: A) the yen rate of exchange for the dollar will fall. B) the yen rate of exchange for the dollar will also rise. C) the yen rate of exchange for the dollar may either fall or rise. D) U.S. net exports to Japan will fall. Answer: A

Type: A Topic: 3 E: 715 MI: 471 MA: 381 72. In considering euros and dollars, the rates of exchange for the euro and the dollar: A) are directly related. B) are inversely related. C) are unrelated. D) move in the same direction. Answer: B

Type: A Topic: 3 E: 716 MI: 472 MA: 382 73. If the equilibrium exchange rate changes so that fewer dollars are needed to buy a South Korean won, then: A) Americans will buy fewer Korean goods and services. B) the won has appreciated in value. C) fewer U.S. goods and services will be demanded by the South Koreans. D) the dollar has depreciated in value. Answer: C

Type: A Topic: 3 E: 716 MI: 472 MA: 382 74. If the exchange rate changes so that more Mexican pesos are required to buy a dollar, then: A) the peso has appreciated in value. B) Americans will buy more Mexican goods and services. C) more U.S. goods and services will be demanded by the Mexicans. D) the dollar has depreciated in value. Answer: B

Type: A Topic: 3 E: 716 MI: 472 MA: 382 75. Depreciation of the dollar will: A) decrease the prices of both U.S. imports and exports. B) increase the prices of both U.S. imports and exports. C) decrease the prices of U.S. imports, but increase the prices to foreigners of U.S. exports. D) increase the prices of U.S. imports, but decrease the prices to foreigners of U.S. exports. Answer: D

Type: A Topic: 3 E: 716 MI: 472 MA: 382 76. Appreciation of the Canadian dollar will: A) intensify an existing disequilibrium in Canada's balance of payments. B) make Canada's exports less expensive and its imports more expensive. C) make Canada's exports more expensive and its imports less expensive. D) make Canada's exports and imports both more expensive. Answer: C

McConnell/Brue: Economics, 16/e

Page 1139

Chapter 38: Exchange Rates, the Balance of Payments, and Trade Deficits

Type: A Topic: 3 E: 716 MI: 472 MA: 382 77. If the dollar depreciates relative to the Russian ruble, the ruble: A) will be less expensive to Americans. B) may either appreciate or depreciate relative to the dollar. C) will appreciate relative to the dollar. D) will depreciate relative to the dollar. Answer: C

Type: A Topic: 3 E: 715 MI: 471 MA: 381 78. The U.S. demand for British pounds is: A) downsloping because a higher dollar price of pounds means British goods are cheaper to Americans. B) downsloping because a lower dollar price of pounds means British goods are more expensive to Americans. C) upsloping because a lower dollar price of pounds means British goods are cheaper to Americans. D) downsloping because a lower dollar price of pounds means British goods are cheaper to Americans. Answer: D

Floating exchange rates; fixed exchange rates

Type: A Topic: 4 E: 715 MI: 471 MA: 381 79. The U.S. supply of Japanese yen is: A) downsloping because a lower dollar price of yen means U.S. goods are cheaper to the Japanese. B) upsloping because a higher dollar price of yen means U.S. goods are cheaper to the Japanese. C) upsloping because a lower dollar price of yen means U.S. goods are cheaper to the Japanese. D) downsloping because a higher dollar price of yen means U.S. goods are cheaper to the Japanese. Answer: B

Type: A Topic: 4 E: 715 MI: 471 MA: 381 80. The U.S. demand for euros is: A) downsloping because, at lower dollar prices for euros, Americans will want to buy more European goods and services. B) downsloping because, at higher dollar prices for euros, Americans will want to buy more European goods and services. C) downsloping because the dollar price of euros and the euro price of dollars are directly related. D) upsloping because a higher dollar price of euros makes European goods and services more attractive to Americans. Answer: A

Type: A Topic: 4 E: 714 MI: 470 MA: 380 81. Which of the following will generate a demand for country X's currency in the foreign exchange market? A) travel by citizens of country X in other countries B) the desire of foreigners to buy stocks and bonds of firms in country X C) the imports of country X D) charitable contributions by country X's citizens to citizens of developing nations Answer: B

McConnell/Brue: Economics, 16/e

Page 1140

Chapter 38: Exchange Rates, the Balance of Payments, and Trade Deficits

Use the following to answer questions 82-87: The following diagram is a flexible exchange market for foreign currency:

S
Dollar price of 1 euro

$.80

D 0

Q1 Quantity of euros

Type: G Topic: 4 E: 716 MI: 472 MA: 382 82. Refer to the above diagram. At the equilibrium exchange rate: A) $1 will buy 1.25 euros. B) 1 euro will buy $.80. C) 1.25 euro will buy $1. D) $1 will buy 8 euros. Answer: C

Type: G Topic: 4 E: 716 MI: 472 MA: 382 83. Refer to the above diagram. At the price $.80 for 1 euro: A) the quantity of euros demanded equals the quantity supplied. B) the dollar-euro exchange rate is unstable. C) the dollar price of 1 euro equals the euro price of 1 dollar. D) there will be a surplus of euros in the foreign exchange market. Answer: A

Type: G Topic: 4 E: 716 MI: 472 MA: 382 84. Refer to the above diagram. Other things equal, a rightward shift of the demand curve would: A) depreciate the dollar. C) reduce the equilibrium quantity of euros. B) appreciate the dollar. D) depreciate the euro. Answer: A

Type: G Topic: 4 E: 716 MI: 472 MA: 382 85. Refer to the above diagram. Other things equal, a leftward shift of the demand curve would: A) depreciate the dollar. C) reduce the equilibrium quantity of euros. B) appreciate the euro. D) cause a surplus of euros. Answer: C

Type: G Topic: 4 E: 716 MI: 472 MA: 382 86. Refer to the above diagram. Other things equal, a leftward shift of the supply curve would: A) appreciate the euro. C) increase the equilibrium quantity of euros. B) cause a shortage of euros. D) appreciate the dollar. Answer: A

McConnell/Brue: Economics, 16/e

Page 1141

Chapter 38: Exchange Rates, the Balance of Payments, and Trade Deficits

Type: G Topic: 4 E: 716 MI: 472 MA: 382 87. Refer to the above diagram. Other things equal, a rightward shift of the supply curve would: A) appreciate the euro. C) decrease the equilibrium quantity of euros. B) cause a surplus of euros. D) appreciate the dollar. Answer: D

Use the following to answer questions 88-91:

Type: G Topic: 4 E: 719 MI: 475 MA: 385 88. Refer to the above diagram. The initial demand for and supply of pesos are shown by D1 and S1. The exchange rate will be: A) M dollars for one peso. C) A dollars for one peso. B) 1/B pesos for one dollar. D) C dollars for one peso. Answer: B

Type: G Topic: 4 E: 719 MI: 475 MA: 385 89. Refer to the above diagram. The initial demand for and supply of pesos are shown by D1 and S1. Suppose the United States reduces its imports of Mexican goods, shifting its demand for pesos from D1 to D2. If the United States was operating under a system of exchange controls, the U.S. government would: A) find that, at the controlled exchange rate, pesos would be in surplus. B) be faced with deteriorating terms of trade. C) be faced with the problem of rationing BG pesos to U.S. importers who want BF pesos. D) be faced with the problem of rationing BF pesos to U.S. importers who want BG pesos. Answer: A

Type: G Topic: 4 E: 722 MI: 478 MA: 388 90. Refer to the above diagram. The initial demand for and supply of pesos are shown by D1 and S1. If the decline in U.S. imports from Mexico described in the previous question occurred and the United States and Mexico were both on the international gold standard: A) gold would flow from Mexico to the United States. B) the exchange rate would rise from B dollars equals 1 peso to C dollars equals 1 peso. C) gold would flow from the United States to Mexico. D) the exchange rate would fall from B dollars equals 1 peso to A dollars equals 1 peso. Answer: A

McConnell/Brue: Economics, 16/e

Page 1142

Chapter 38: Exchange Rates, the Balance of Payments, and Trade Deficits

Type: G Topic: 4 E: 719 MI: 475 MA: 385 91. Refer to the above diagram. The initial demand for and supply of pesos are shown by D1 and S 1. If the decline in U.S. imports from Mexico described in the two previous questions occurred under a system of freely floating exchange rates: A) gold would flow from Mexico to the United States. B) the peso price of dollars would rise from 1/B pesos equals $1 to 1/A pesos equals $1. C) a problem of rationing a shortage of pesos would arise in the United States. D) the dollar price of pesos would increase to C dollars equals 1 peso. Answer: B

Type: A Topic: 4 E: 719 MI: 475 MA: 385 92. Under a system of freely flexible (floating) exchange rates a U.S. trade deficit with Mexico will tend to cause: A) the United States government to ration pesos to U.S. importers. B) a flow of gold from the United States to Mexico. C) an increase in the peso price of dollars. D) an increase in the dollar price of pesos. Answer: D

Type: C Topic: 4 E: 717-718 MI: 473-474 MA: 383-384 93. Which of the following have substantially equivalent effects on a nation's volume of exports and imports? A) exchange rate appreciation and a decrease in the domestic supply of money B) exchange rate appreciation and domestic deflation C) exchange rate depreciation and domestic deflation D) exchange rate depreciation and domestic inflation Answer: C

Type: A Topic: 4 E: 719 MI: 475 MA: 385 94. If in a system of fixed exchange rates the dollar price of euros is above the market equilibrium level: A) gold will flow from the United States to Europe. B) there will be a surplus of euros. C) the United States government will have to ration euros to U.S. importers. D) there will be a shortage of euros. Answer: B

Use the following to answer questions 95-97: Answer the next question(s) on the basis of the following table which indicates the dollar price of libras, the currency used in the hypothetical nation of Libra. Assume that a system of freely floating exchange rates is in place.

(1) Quantity of libras demanded (billions) 100 200 300 400

(2) Dollar price of libras $5 4 3 2

(3) Quantity of libras supplied (billions) 325 200 100 75

McConnell/Brue: Economics, 16/e

Page 1143

Chapter 38: Exchange Rates, the Balance of Payments, and Trade Deficits

Type: T Topic: 4 E: 716 MI: 472 MA: 382 95. Refer to the above table. The equilibrium dollar price of libras is: A) $5. B) $4. C) $3. D) $2. Answer: C

Type: T Topic: 4 E: 716 MI: 472 MA: 382 96. Refer to the above table. The exchange rate is: A) 4 libras for one dollar. B) .33 libras for one dollar. Answer: B

C) .40 libras for one dollar. D) 3 libras for one dollar.

Type: T Topic: 4 E: 716 MI: 472 MA: 382 97. Refer to the above table. Suppose that Libra decided to import more U.S. products. We would expect the quantity of libras: A) demanded at each dollar price to rise and the dollar to depreciate relative to the libra. B) demanded at each dollar price to fall and the dollar to appreciate relative to the libra. C) supplied at each dollar price to rise and the dollar to appreciate relative to the libra. D) supplied at each dollar price to fall and the dollar to depreciate relative to the libra. Answer: C

Use the following to answer questions 98-99: Answer the next question(s) on the basis of the following information: In 1985 the exchange rate between the U.S. dollar and the Japanese yen was $1 = 262 yen; in 2003, the rate was $1 = 110 yen.

Type: A Topic: 4 E: 716 MI: 472 MA: 382 98. Refer to the above information. Between 1985 and 2003 the: A) dollar appreciated in value relative to the yen. C) dollar price of yen fell. B) yen appreciated in value relative to the dollar. D) yen price of dollars rose. Answer: B

Type: A Topic: 4 E: 717 MI: 473 MA: 383 99. Refer to the above information. Which one of the following might be a plausible explanation for the change in the dollar-yen exchange rate cited in the previous question? A) Japan exported much more to the United States during this period than it imported from the United States. B) Japan greatly increased its purchases of military equipment from the United States during this period. C) Japan's economy grew far faster than the U.S. economy during this period. D) Japan's government devalued the yen during this period. Answer: A

Type: A Topic: 4 E: 720 MI: 476 MA: 386 100. Under a system of freely floating exchange rates, an increase in the international value of a nation's currency will: A) cause an international surplus of its currency. B) contribute to disequilibrium in its balance of payments. C) cause gold to flow into that country. D) cause its imports to rise. Answer: D

McConnell/Brue: Economics, 16/e

Page 1144

Chapter 38: Exchange Rates, the Balance of Payments, and Trade Deficits

Type: A Topic: 4 E: 717 MI: 473 MA: 383 101. According to the purchasing power parity theory of exchange rates: A) a dollar, when converted to other currencies at the prevailing floating exchange rate, has the same purchasing power in various countries. B) in equilibrium, national currencies have equal value in terms of gold. C) the higher a nation's price level in terms of its own currency, the greater is the amount of foreign exchange it can obtain for a unit of its currency. D) nominal currency values will tend to equalize (become 1 = 1) in the long run. Answer: A

Type: D Topic: 4 E: 717 MI: 473 MA: 383 102. The idea that freely floating exchange rates equate the purchasing power of national currencies is called: A) the equation of exchange. C) Say's Law. B) the balance of payments. D) the purchasing power parity theory. Answer: D

Type: A Topic: 4 E: 717 MI: 473 MA: 383 103. Assume that Japan and South Korea have floating exchange rates. Other things equal, if economic growth is more rapid in Japan than in South Korea: A) gold bullion will flow out of Japan. C) the South Korean won will depreciate. B) the Japanese yen will depreciate. D) the Japanese yen will appreciate. Answer: B

Type: A Topic: 4 E: 717 MI: 473 MA: 383 104. Assume that Brazil and Mexico have floating exchange rates. Other things unchanged, if the price level is stable in Mexico but Brazil experiences rapid inflation: A) gold bullion will flow into Brazil. C) the Mexican peso will depreciate. B) the Brazilian real will depreciate. D) the Brazilian real will appreciate. Answer: B

Type: A Topic: 4 E: 718 MI: 474 MA: 384 105. Assume that Switzerland and Britain have floating exchange rates. Other things unchanged, if a tight money policy raises interest rates in Britain as compared to Switzerland: A) gold bullion will flow into Switzerland. C) the pound will depreciate. B) the Swiss franc will depreciate. D) the Swiss franc will appreciate. Answer: B

McConnell/Brue: Economics, 16/e

Page 1145

Chapter 38: Exchange Rates, the Balance of Payments, and Trade Deficits

Use the following to answer questions 106-109:

Type: G Topic: 4 E: 718 MI: 474 MA: 384 106. Refer to the above diagram where D and S are the United States' demand for and supply of Swiss francs. At the equilibrium exchange rate, E, the United States' balance of payments is in equilibrium. A shift of the demand curve to D' might be the result of: A) a relative decline in interest rates in Switzerland. B) a reduction in the United States' relative price level. C) a recession in the United States which slows its rate of growth. D) a relative decline in interest rates in the United States. Answer: D

Type: G Topic: 4 E: 720-721 MI: 476-477 MA: 386-387 107. Refer to the above diagram where D and S are the United States' demand for and supply of Swiss francs. At the equilibrium exchange rate, E, the United States' balance of payments is in equilibrium. Given a change in demand from D to D' the United States could maintain the dollar price of Swiss francs by: A) shifting the S curve to the right through the use of domestic expansionary policies. B) instituting exchange controls to ration Ed Swiss francs to U.S. importers who want Ec francs. C) using international monetary reserves to cover the Ec shortage of Swiss francs. D) using international monetary reserves to cover the cd shortage of Swiss francs. Answer: D

Type: G Topic: 4 E: 719 MI: 475 MA: 385 108. Refer to the above diagram where D and S are the United States' demand for and supply of Swiss francs. At the equilibrium exchange rate, E, the United States' balance of payments is in equilibrium. Under a system of flexible exchange rates, the shift in demand from D to D' will: A) ultimately reduce U.S. exports and raise U.S. imports. B) cause the dollar to appreciate. C) cause the Swiss franc to depreciate. D) cause the dollar to depreciate. Answer: D

McConnell/Brue: Economics, 16/e

Page 1146

Chapter 38: Exchange Rates, the Balance of Payments, and Trade Deficits

Type: G Topic: 4 E: 720 MI: 476 MA: 386 109. Refer to the above diagram where D and S are the United States' demand for and supply of Swiss francs. At the equilibrium exchange rate, E, the United States' balance of payments is in equilibrium. Under a system of fixed exchange rates, the shift in demand from D to D' will cause: A) the United States to increase its stocks of international monetary reserves. B) a Swiss balance of payments deficit. C) a U.S. balance of payments deficit. D) a U.S. balance of payments surplus. Answer: C

Type: A Topic: 4 E: 720-721 MI: 476-477 MA: 386-387 110. Under a system of fixed exchange rates, a nation that has chronic balance of payments deficits may: A) initiate protectionist trade policies. B) run short of international monetary reserves. C) be forced to invoke contractionary monetary and fiscal policies. D) do all of the above. Answer: D

Type: A Topic: 4 E: 715-716 MI: 471-472 MA: 381-382 111. If the United States has full employment and the dollar dramatically depreciates in value, we can expect (other things equal): A) both U.S. imports and U.S. exports to rise. B) both U.S. imports and U.S. exports to fall. C) U.S. exports to fall and U.S. imports to increase. D) inflation to occur. Answer: D

Type: A Topic: 4 E: 718 MI: 474 MA: 384 112. Suppose interest rates fall sharply in the United States but are unchanged in Great Britain. Other things equal, under a system of freely floating exchange rates we can expect the demand for pounds in the United States to: A) decrease, the supply of pounds to increase, and the dollar to appreciate relative to the pound. B) increase, the supply of pounds to increase, and the dollar may either appreciate or depreciate relative to the pound. C) increase, the supply of pounds to decrease, and the dollar to depreciate relative to the pound. D) decrease, the supply of pounds to increase, and the dollar to depreciate relative to the pound. Answer: C

Type: A Topic: 4 E: 718 MI: 474 MA: 384 113. Assume that, under a system of floating exchange rates, Mexicans decide to increase their investments in the United States. As a result: A) the peso and the dollar will both depreciate. B) the peso and the dollar will both appreciate. C) the peso will depreciate and the dollar will appreciate. D) the peso will appreciate and the dollar will depreciate. Answer: C

McConnell/Brue: Economics, 16/e

Page 1147

Chapter 38: Exchange Rates, the Balance of Payments, and Trade Deficits

Gold standard

Type: A Topic: 5 E: 722 MI: 478 MA: 388 114. Under the gold standard: A) nations can protect their domestic price and employment levels from changes in the volume and direction of world trade. B) exchange rates are virtually fixed. C) differences in exports and imports will be precisely balanced by capital account flows, excluding gold. D) exchange rates fluctuate freely in response to changes in the supply of, and demand for, foreign currencies. Answer: B

Type: A Topic: 5 E: 722 MI: 478 MA: 388 115. Which of the following is not a condition of the international gold standard? A) a nation must be willing to accept very wide fluctuations in its exchange rate B) a nation must allow gold to be freely exported and imported C) a nation must be willing to convert gold into paper money and vice versa at a stipulated rate D) a nation must define its monetary unit in terms of a certain quantity of gold Answer: A

Type: A Topic: 5 E: 722 MI: 478 MA: 388 116. Under the international gold standard: A) a nation's exchange rate is virtually fixed. B) domestic output and the price level will fall in those nations receiving international gold flows. C) a nation's balance of payments surplus will be corrected by an outflow of gold. D) a nation's balance of payments deficit will be corrected by an inflow of gold. Answer: A

Type: A Topic: 5 E: 723 MI: 479 MA: 389 117. Under the international gold standard: A) a nation sacrifices an independent monetary policy. B) gold flows between nations would always promote macroeconomic stability. C) exchange rates would fluctuate with changes in demand and supply. D) balance of payments imbalances would be magnified. Answer: A

Type: A Topic: 5 E: 722 MI: 478 MA: 388 118. Under the international gold standard: A) exchange rates would fluctuate inversely with the domestic interest rates of the participating countries. B) each nation must agree to depreciate its currency in direct proportion to the growth of its real GDP. C) gold would flow into a nation experiencing a balance of payments surplus. D) exchange rates would fluctuate directly with the domestic price levels of the various trading countries. Answer: C

McConnell/Brue: Economics, 16/e

Page 1148

Chapter 38: Exchange Rates, the Balance of Payments, and Trade Deficits

Type: A Topic: 5 E: 722 MI: 478 MA: 388 119. Under the gold standard a balance of payments disequilibrium would be corrected automatically by: A) the depreciation of that country's currency. B) an increase in the gold content of that nation's monetary unit. C) the appreciation of that country's currency. D) an outflow or inflow of gold. Answer: D

Type: A Topic: 5 E: 722-723 MI: 478-479 MA: 388-389 120. Under the international gold standard a flow of gold from country A into country B would be halted by: A) a rise in the price of B's currency measured in terms of A's currency. B) government export controls on gold. C) rising prices and incomes in B and falling prices and incomes in A. D) rising prices and incomes in A and falling prices and incomes in B. Answer: C

Bretton Woods system

Type: F Topic: 6 E: 723 MI: 479 MA: 389 121. The basis for the Bretton Woods international monetary system was: A) a completely fixed system of exchange rates. C) the gold standard. B) an adjustable peg system of exchange rates. D) a freely flexible system of exchange rates. Answer: B

Type: F Topic: 6 E: 723-724 MI: 479-480 MA: 389-390 122. The Bretton Woods system of exchange rates relied on: A) freely floating exchange rates. B) fixed exchange rates with no mechanism for changing them. C) fixed or pegged exchange rates, with occasional orderly adjustments to the rates. D) the United States to set and periodically review worldwide exchange rates. Answer: C

Type: F Topic: 6 E: 723-724 MI: 479-480 MA: 389-390 123. The Bretton Woods system of exchange rates: A) is also known as the gold standard and met its demise in the 1930s. B) relied heavily on floating exchange rates determined in the market for foreign exchange. C) was abandoned in the 1930s. D) was a system of fixed or pegged exchange rates, which occasionally could be adjusted. Answer: D

McConnell/Brue: Economics, 16/e

Page 1149

Chapter 38: Exchange Rates, the Balance of Payments, and Trade Deficits

Managed float

Type: D Topic: 7 E: 724-725 MI: 480-481 MA: 390-391 124. In saying that the present system of floating exchange rates is managed we mean that: A) countries that allow their exchange rate to move freely will lose their borrowing privileges with the IMF. B) the value of any IMF member's currency can only vary 2 percent from its par value. C) IMF officials determine exchange rates on a day-to-day basis. D) the central banks of various countries sometimes buy and sell foreign exchange to alter undesirable trends in exchange rates. Answer: D

Type: F Topic: 7 E: 724 MI: 480 MA: 390 125. The exchange rate system currently used by the industrially advanced nations is: A) the gold standard. B) the Bretton Woods system. C) the managed float. D) a fixed rate system. Answer: C

Type: A Topic: 7 E: 724-727 MI: 480-483 MA: 390-393 126. Under the managed floating system of exchange rates: A) all exchange rates vary with changes in the free-market prices of gold. B) industrialized nations meet once each year to negotiate readjustments in their exchange rates. C) exchange rates are essentially flexible, but governments intervene to offset disorderly fluctuations in rates. D) exchange rates are adjusted at the discretion of the IMF. Answer: C

Type: A Topic: 7 E: 720-721 MI: 476-477 MA: 386-387 127. A government may be able to reduce the international value of its currency by: A) selling its currency in the foreign exchange market. B) buying its currency in the foreign exchange market. C) selling foreign currencies in the foreign exchange market. D) increasing its domestic interest rates. Answer: A

Type: F Topic: 7 E: 724-725 MI: 480-481 MA: 390-391 128. The current system of exchange rates can best be described as: A) freely fluctuating exchange rates. C) rigidly fixed exchange rates. B) managed floating exchange rates. D) a crawling peg system. Answer: B

Type: F Topic: 7 E: 722-725 MI: 478-481 MA: 388-391 129. Which of the following lists of exchange rates is arranged in proper historical order? A) Bretton Woods system, gold standard, managed float B) gold standard, managed float, Bretton Woods system C) managed float, Bretton Woods system, gold standard D) gold standard, Bretton Woods system, managed float Answer: D

McConnell/Brue: Economics, 16/e

Page 1150

Chapter 38: Exchange Rates, the Balance of Payments, and Trade Deficits

Type: F Topic: 7 E: 725 MI: 481 MA: 391 130. Which one of the following is not one of the so-called G8 nations? A) Japan B) Canada C) United States D) Mexico Answer: D

Type: F Topic: 7 E: 725 MI: 481 MA: 391 131. The Group of Eight (G8) nations which periodically have jointly intervened to influence the value of the dollar include: A) Canada, U.S., France, Britain, Mexico, Germany, and Brazil. B) Canada, U.S., France, Japan, Italy, Germany, and Great Britain. C) Canada, U.S., Mexico, Brazil, Argentina, Uruguay, and Chile. D) Italy, France, Britain, Germany, Netherlands, Norway, and Sweden. Answer: B

Type: A Topic: 7 E: 725 MI: 481 MA: 391 Status: New 132. Suppose the G8 nations decide that the dollar is too strong (high in value) relative to the yen. These nations might: A) use official reserves of yen to buy dollars. B) use official reserves of dollars to buy yen. C) encourage Japan to print more yen. D) encourage the United States to increase interest rates. Answer: B

U.S. trade deficits

Type: F Topic: 8 E: 726 MI: 482 MA: 392 133. In recent years, the United States has had large: A) current account surpluses. B) capital account deficits. Answer: C

C) balance of trade deficits. D) balance of payments surpluses.

Type: F Topic: 8 E: 726 MI: 482 MA: 392 134. In recent years, the United States has had large: A) current account surpluses. B) current account deficits. Answer: B

C) balance of trade surpluses. D) balance of payments surpluses.

Type: A Topic: 8 E: 726 MI: 482 MA: 392 135. Relatively rapid U.S. growth between 1996 and 2000 contributed to large U.S. trade deficits by: A) increasing U.S. national income, which decreased U.S. exports. B) reducing real interest rates in the United States. C) increasing U.S. tax revenues and reducing the Federal budget deficit. D) increasing U.S. national income, which increased U.S. imports. Answer: D

McConnell/Brue: Economics, 16/e

Page 1151

Chapter 38: Exchange Rates, the Balance of Payments, and Trade Deficits

Type: A Topic: 8 E: 727 MI: 483 MA: 393 136. Two of the implications of large U.S. trade deficits for the United States are: A) decreased current consumption and decreased indebtedness to foreigners. B) reduced budget deficits and decreased indebtedness to foreigners. C) reduced current consumption and higher saving. D) increased current consumption and increased indebtedness to foreigners. Answer: D

Type: A Topic: 8 E: 727 MI: 483 MA: 393 137. Largely because of large current account deficits, the United States: A) is the leading exporting nation in the world. B) has the world's largest external debt. C) has the world's highest saving rate. D) is experiencing an increase in its net inflow of investment income. Answer: B

Type: A Topic: 8 E: 727 MI: 483 MA: 393 138. One of the consequences of the U.S. trade deficit is that: A) domestic inflation has resulted. B) the accumulation of American dollars in foreign hands has enabled foreign firms to build factories in America. C) the distribution of income in the United States has become less unequal. D) the system of flexible exchange rates has been abandoned in favor of a new gold standard. Answer: B

Type: A Topic: 8 E: 726 MI: 482 MA: 392 Status: New 139. In terms of individual nations, the largest U.S. trade deficit is with: A) Japan. B) Mexico. C) China. D) Canada. Answer: C

Type: F Topic: 8 E: 727 MI: 483 MA: 393 Status: New 140. The world's largest debtor nation in terms of debt owed to foreign citizens and governments is: A) Russia. B) Argentina. C) Japan. D) the United States. Answer: D

Consider This Questions

Type: A E: 717 MI: 473 MA: 383 Status: New 141. (Consider This) The main focus of the Big Mac index is relative: A) inflation rates among countries. C) real wage rates among countries. B) purchasing power of the world's currencies. D) income changes among countries. Answer: B

McConnell/Brue: Economics, 16/e

Page 1152

Chapter 38: Exchange Rates, the Balance of Payments, and Trade Deficits

Type: A E: 717 MI: 473 MA: 383 Status: New 142. (Consider This) Suppose that the dollar-pound exchange rate is $1 = £2. According to the Big Mac index, the pound is overvalued if the price of a Big Mac sandwich is $2 in the United States and: A) £4 in the United Kingdom. C) £3 in the United Kingdom. B) £5 in the United Kingdom. D) £2 in the United Kingdom. Answer: B

Last Word Questions

Type: D E: 728 MI: 484 MA: 394 143. (Last Word) People who buy foreign currency for the sole goal of selling it at a profit are called: A) numismatics. B) currency hedgers. C) currency manipulators. D) currency speculators. Answer: D

Type: A E: 728 MI: 484 MA: 394 144. (Last Word) Currency speculators aid international trade by: A) absorbing exchange rate risk that others do not want to bear. B) increasing the volatility of exchange rates. C) making the demand for imports less elastic. D) promoting barter. Answer: A

Type: A E: 728 MI: 484 MA: 394 145. (Last Word) Firms engaged in international trade can reduce exchange-rate risk by: A) paying for foreign goods only when they are delivered. B) buying on credit. C) hedging in the futures market. D) dealing only with highly reputable firms. Answer: C

True/False Questions

Type: A E: 722 MI: 478 MA: 388 146. Under the international gold standard, exchange rates fluctuate without restraint to correct any international disequilibrium by affecting the relative attractiveness of domestic and foreign goods. Answer: False

Type: A E: 722 MI: 478 MA: 388 147. If the United States and France are both on the international gold standard and U.S. exports to France exceed United States imports from France, gold will flow from the United States to France. Answer: False

Type: A E: 712 MI: 468 MA: 378 148. U.S. exports increase and U.S. imports decrease the supplies of foreign monies owned by U.S. banks. Answer: True

McConnell/Brue: Economics, 16/e

Page 1153

Chapter 38: Exchange Rates, the Balance of Payments, and Trade Deficits

Type: A E: 715-716 MI: 471-472 MA: 381-382 149. Under freely flexible (floating) exchange rates, if the dollar price of pounds rises, the pound price of dollars will fall. Answer: True

Type: A E: 715-716 MI: 471-472 MA: 381-382 150. If the price of British pounds, measured in terms of U.S. dollars, is rising, then the price of U.S. dollars, measured in terms of British pounds, is also rising. Answer: False

Type: A E: 719 MI: 475 MA: 385 151. Under freely flexible (floating) exchange rates a U.S. trade deficit with Japan will eventually cause the dollar price of yen to rise. Answer: True

Type: D E: 713 MI: 469 MA: 379 152. A nation that imports more goods and services than it exports is necessarily realizing an international balance of payments deficit. Answer: False

Type: A E: 719 MI: 475 MA: 385 153. If the dollar depreciates, U.S. exports will eventually rise and U.S. imports will eventually fall. Answer: True

Type: A E: 720 MI: 476 MA: 386 154. A system of fixed exchange rates is more likely to result in exchange controls than is a system of flexible (floating) exchange rates. Answer: True

Use the following to answer questions 155-161: Answer the next question(s) on the basis of the following 2004 balance of payments statement for Transylvania. All figures are in billions of dollars.

Goods exports Goods imports Service exports Service imports Net investment income Net transfers Foreign purchases of assets Purchases of foreign assets Official reserves

+$15 -17 +5 -2 -5 +4 +5 -11 +1

Type: T E: 713 MI: 469 MA: 379 155. Refer to the above data. In 2004 Transylvania imported more products than it exported. Answer: True

McConnell/Brue: Economics, 16/e

Page 1154

Chapter 38: Exchange Rates, the Balance of Payments, and Trade Deficits

Type: T E: 713 MI: 469 MA: 379 156. Refer to the above data. Transylvania had a $2 billion balance of trade (goods) surplus in 2004. Answer: False

Type: T E: 713-714 MI: 469-470 MA: 379-380 157. Refer to the above data. In 2004 Transylvania realized a $1 billion surplus on goods and services. Answer: True

Type: T E: 713 MI: 469 MA: 379 158. Refer to the above data. In 2004 Transylvania was a net recipient of transfers from the rest of the world. Answer: True

Type: T E: 713 MI: 469 MA: 379 159. Refer to the above data. Foreigners made a smaller volume of asset purchases in Transylvania in 2004 than Transylvanians made asset purchases abroad. Answer: True

Type: T E: 715 MI: 471 MA: 381 160. Refer to the above data. Transylvania realized a balance of payments deficit in 2004. Answer: True

Type: T E: 715-716 MI: 471-472 MA: 381-382 161. Refer to the above data. If Transylvania was on a system of freely floating exchange rates, its balance of payments position would cause the international value of its currency to depreciate. Answer: True

Type: F E: 726-727 MI: 482-483 MA: 392-393 162. The United States has had significant trade and current account surpluses in recent years. Answer: False

Type: A E: 726-727 MI: 482-483 MA: 392-393 163. A current account deficit will reduce U.S. foreign indebtedness. Answer: False

McConnell/Brue: Economics, 16/e

Page 1155

BONUS WEB CHAPTER 39W

The Economics of Developing Countries
(Found at the book's website, www.mcconnell16.com)

Topic 1. 2. 3. 4. 5. DVC characteristics; DVC-IAC comparisons Development obstacles; vicious circle Government's role Aid, trade, and foreign investment IAC and DVC policies for DVC development Last Word True-False

Question numbers 1-30 31-70 71-75 76-91 92-97 98-99 100-113

____________________________________________________________

_______________________________________

____________________________________________________________

_______________________________________

Multiple Choice Questions DVC characteristics; DVC-IAC comparisons

Type: F Topic: 1 E: Web MI: Web MA: Web 1. Approximately what percentage of the world's income is received by the richest one-fifth of the world's population? A) 20 percent B) 30 percent C) 60 percent D) 80 percent Answer: D

Type: F Topic: 1 E: Web MI: Web MA: Web 2. Approximately what percent of the world's income is received by the poorest one-fifth of the world's population? A) 2 B) 5 C) 7 D) 10. Answer: A

Web Chapter 39W: The Economics of Developing Countries

Use the following to answer questions 3-8:
Country A B C D E Per Capita Income, 2001 $ 621 14,894 31,555 3,498 2,005

Type: T Topic: 1 E: Web MI: Web MA: Web Status: New 3. Which of the above nations are low-income developing countries (DVCs), according to the World Bank? A) country A only B) countries A, D, and E C) countries A and E D) countries A, B, D, and E Answer: A

Type: T Topic: 1 E: Web MI: Web MA: Web Status: New 4. Which of the above nations are middle-income developing countries (DVCs), according to the World Bank? A) country E only B) countries A, D, and E C) countries D and E D) countries D only Answer: C

Type: T Topic: 1 E: Web MI: Web MA: Web Status: New 5. Which of the above nation are high-income countries (IACs), according to the World Bank? A) country C only B) countries B, C, and D C) countries B, C, D, and E D) countries B and C Answer: D

Type: T Topic: 1 E: Web MI: Web MA: Web Status: New 6. Refer to the above table. If per capita income increases by 10 percent over five years in each of the nations shown, the per capita income gap between country C and country A: A) will remain the same B) will rise from $30,934 to $34,027. C) may rise or fall depending on the rate of population growth. D) will rise from $31,555 to $34,710. Answer: B

Type: T Topic: 1 E: Web MI: Web MA: Web Status: New 7. Refer to the above table. Which of the following might reduce the per capita income gap between countries A and E? A) faster population growth in country A than in country E. B) greater investment relative to GDP in country E than in country A. C) more rapid improvement in literacy and education in country A than in E. D) increased capital flight from country A relative to that from country E. Answer: C

Type: T Topic: 1 E: Web MI: Web MA: Web Status: New 8. Which of the following is not inversely related to per capita income? A) mortality rates for children under 5 years of age C) per capita energy consumption B) adult illiteracy rates D) population growth rates. Answer: C

McConnell/Brue: Economics, 16/e

Page 1158

Web Chapter 39W: The Economics of Developing Countries

Type: F Topic: 1 E: Web MI: Web MA: Web 9. Which of these sets of nations are advanced industrial nations? A) Brazil, Poland, South Africa C) Canada, Switzerland, and France B) China, India, and Russia D) United States, South Korea, and Mexico Answer: C

Type: F Topic: 1 E: Web MI: Web MA: Web 10. Which of these sets of nations are low-income developing nations? A) Brazil, Australia, South Africa C) Canada, Switzerland, and France B) Uganda, India, and Nicaragua D) Germany, South Korea, and Mexico Answer: B

Type: F Topic: 1 E: Web MI: Web MA: Web 11. The United States has about ___ percent of the world's population and produces about ___ percent of the world output. A) 20, 30 B) 3, 45 C) 10, 30 D) 5, 30 Answer: D

Type: F Topic: 1 E: Web MI: Web MA: Web 12. Examples of industrially advanced nations (IACs) are: A) the United States, Canada, and Mexico. C) Japan, South Korea, and China. B) Pakistan, India, and China. D) Germany, Italy, and France. Answer: D

Type: F Topic: 1 E: Web MI: Web MA: Web 13. To be classified as a low-income developing country, annual per capita output in 2001 needed to be: A) $2020 or less B) $745 or less. C) $4015 or less. D) $201 or less. Answer: B

Type: F Topic: 1 E: Web MI: Web MA: Web 14. Examples of low-income developing countries are: A) Switzerland, New Zealand, and Australia. B) Germany, Austria, and Italy. Answer: C

C) Sudan, Bangladesh, and Ethiopia. D) Mexico, South Korea, and Brazil.

Type: F Topic: 1 E: Web MI: Web MA: Web 15. The very poorest low-income DVCs typically have relatively: A) low rates of economic growth and relatively high rates of population growth. B) high rates of economic growth and relatively low rates of population growth. C) low rates of both population growth and economic growth. D) high rates of both population growth and economic growth. Answer: A

McConnell/Brue: Economics, 16/e

Page 1159

Web Chapter 39W: The Economics of Developing Countries

Type: A Topic: 1 E: Web MI: Web MA: Web 16. If two nations have different per capita income levels and their rates of economic growth are identical, then the absolute per capita income differential: A) will remain constant. B) may either widen or diminish. C) will diminish. D) will widen. Answer: D

Type: F Topic: 1 E: Web MI: Web MA: Web 17. Most of the world's population lives in: A) North America. B) the DVCs. C) Western Europe. Answer: B

D) the IACs.

Type: F Topic: 1 E: Web MI: Web MA: Web 18. Which of the following does not correlate positively with economic growth? A) output per capita B) life expectancy C) the percentage of the population engaged in agriculture D) the literacy rate Answer: C

Type: F Topic: 1 E: Web MI: Web MA: Web 19. The exports of the DVCs consist largely of: A) high-technology goods. B) raw materials and farm products. Answer: B

C) manufactured goods. D) services and financial capital.

Type: F Topic: 1 E: Web MI: Web MA: Web 20. The DVCs are: A) located primarily in Northern Europe. B) located primarily in Western Europe. C) located primarily in Africa, Asia, and Latin America. D) more-or-less evenly distributed over the various continents. Answer: C

Type: F Topic: 1 E: Web MI: Web MA: Web 21. The absolute income gap between the IACs and the DVCs has: A) remained constant over time. B) increased over time. C) decreased over time. D) increased in nominal terms, but decreased in real terms. Answer: B

Type: A Topic: 1 E: Web MI: Web MA: Web 22. If nation X has a larger real per capita output than nation Y, then equal rates of growth of per capita output will: A) leave the absolute income gap unchanged. B) decrease the absolute income gap. C) increase the absolute income gap. D) cause the nominal per capita outputs of both nations to increase faster than their real per capita outputs. Answer: C

McConnell/Brue: Economics, 16/e

Page 1160

Web Chapter 39W: The Economics of Developing Countries

Type: A Topic: 1 E: Web MI: Web MA: Web 23. Assume a DVC has a real per capita output of $1000 as compared to $20,000 for an IAC. If both nations realize a 4 percent growth of their real per capita outputs, the absolute real per capita output gap will: A) remain unchanged at $19,000. C) decrease by $1,000. B) increase by $760. D) increase by $19,760. Answer: B

Type: A Topic: 1 E: Web MI: Web MA: Web 24. If the real GDP of a DVC increases from $600 billion to $630 billion and its population increases from 200 million to 216 million, its real per capita GDP will have: A) increased by about $83. C) remained unchanged. B) decreased by about $83. D) decreased by about $19. Answer: B

Type: A Topic: 1 E: Web MI: Web MA: Web 25. In recent decades: A) all countries classified as DVCs have had little or no economic growth. B) some nations classified as DVCs have grown rapidly while others have grown very slowly or not at all. C) all countries classified as DVCs have experienced rapid economic growth and rising living standards. D) all countries classified as low-income DVCs have had declining per capita GDPs. Answer: B

Type: A Topic: 1 E: Web MI: Web MA: Web 26. If population is expanding at the same rate as real output: A) real per capita output will increase. C) real per capita output will remain unchanged. B) real per capita output will decrease. D) living standards will increase. Answer: C

Type: A Topic: 1 E: Web MI: Web MA: Web 27. If the real output of a DVC increases from $200 billion to $260 billion and its population increases from 100 to 120 million, its real per capita output will have: A) remained unchanged. C) increased by about $55. B) increased by about $167. D) decreased by about $20. Answer: B

Type: A Topic: 1 E: Web MI: Web MA: Web 28. If a nation's real output is growing by 5 percent per year, its real output will double in approximately: A) 22 years. B) 20 years. C) 14 years. D) 8 years. Answer: C

Type: A Topic: 1 E: Web MI: Web MA: Web 29. Suppose that Alpha's real output rose from $400 billion in year 1 to $428 billion in year 2. Its growth rate for this period was: A) 14 percent. B) 12 percent. C) 9 percent. D) 7 percent. Answer: D

McConnell/Brue: Economics, 16/e

Page 1161

Web Chapter 39W: The Economics of Developing Countries

Type: F Topic: 1 E: Web MI: Web MA: Web 30. Over the next fifteen years, ___ out of 10 people added to the world's population will be born in developing countries. A) 6 B) 7 C) 8 D) 9 Answer: D

Development obstacles; vicious circle

Type: A Topic: 2 E: Web MI: Web MA: Web Status: New 31. Which attitude or custom is the most conducive to long-term economic growth? A) focus on group contentment rather than individual achievement B) the belief there is little or no correlation between an individual's economic actions and her or his economic fortunes. C) direct connections between individual efforts (including educational efforts) and economic rewards D) use of the majority of resources for religious structures and ceremonies Answer: C

Type: A Topic: 2 E: Web MI: Web MA: Web Status: New 32. Which of the following is typically not a problem for low-income DVCs? A) capital flight B) "brain drains" C) high saving rates D) poor infrastructure Answer: C

Type: A Topic: 2 E: Web MI: Web MA: Web 33. Increases in the total real output of many DVCs do not increase the nation's standard of living because: A) diminishing returns may be encountered in increasing total output. B) population increases may dissipate the increase in real output. C) disguised unemployment in agriculture will persist. D) surplus farm labor may move from rural areas to industrial areas, causing unemployment. Answer: B

Type: A Topic: 2 E: Web MI: Web MA: Web 34. At the present time the largest percentage of the national incomes of the low-income DVCs is used for: A) imports of the finished products of foreign industries. B) food. C) infrastructure. D) industrial development. Answer: B

Type: F Topic: 2 E: Web MI: Web MA: Web 35. The populations of the developing nations are growing: A) at about 5 percent per year. B) at about the same rate as those of the industrially advanced nations. C) slower than those of the industrially advanced nations. D) faster than those of the industrially advanced nations. Answer: D

McConnell/Brue: Economics, 16/e

Page 1162

Web Chapter 39W: The Economics of Developing Countries

Type: D Topic: 2 E: Web MI: Web MA: Web 36. Underemployment occurs: A) when workers do not have jobs. B) when farm workers become more productive. C) when workers are working fewer hours than they desire or when they are working less productively than they are capable. D) in IACs, but not in the DVCs. Answer: C

Type: A Topic: 2 E: Web MI: Web MA: Web 37. Which of the following characterizes the labor sector of most DVCs? A) educational levels are low B) a large proportion of the labor force is in agriculture C) labor productivity is low D) all of the above Answer: D

Type: A Topic: 2 E: Web MI: Web MA: Web 38. Rapid population growth can be an obstacle to economic development because: A) it can translate a relatively large increase in real output into a small increase in real output per capita. B) more investment will be required to simply maintain the quantity of capital goods per person. C) it may lead to the overutilization and therefore ecological degradation of farmland. D) of all of the above reasons. Answer: D

Type: A Topic: 2 E: Web MI: Web MA: Web 39. Population growth remains high in most DVCs because: A) religious and sociocultural considerations favor large families. B) children may provide economic security for aging parents. C) children provide agricultural labor in rural areas. D) of all of the above reasons. Answer: D

Type: D Topic: 2 E: Web MI: Web MA: Web 40. The demographic transition view alleges that: A) the DVCs must first accept the use of birth control techniques to increase their standards of living. B) population growth will only decline if mortality rates exceed birth rates. C) if incomes first rise, population growth will then decline. D) population growth has no bearing on a nation's per capita income. Answer: C

Type: D Topic: 2 E: Web MI: Web MA: Web 41. For DVC per capita incomes to rise, birth rates must first be reduced. This statement describes the: A) human capital view of population growth. B) traditional view of population growth. C) capricious universe view. D) demographic transition view of population growth. Answer: B

McConnell/Brue: Economics, 16/e

Page 1163

Web Chapter 39W: The Economics of Developing Countries

Type: D Topic: 2 E: Web MI: Web MA: Web 42. Per capita incomes must first grow for birth rates to decline. This statement describes the: A) human capital view of population growth. B) traditional view of population growth. C) capricious universe view. D) demographic transition view of population growth. Answer: D

Type: A Topic: 2 E: Web MI: Web MA: Web 43. The demographic transition concept suggests that: A) effective birth control is the primary prerequisite of DVC income growth. B) income growth must first occur before DVC birth rates will decline. C) children are economic assets in the IACs, but economic liabilities in the DVCs. D) the IACs will have higher birth rates than the DVCs. Answer: B

Type: A Topic: 2 E: Web MI: Web MA: Web 44. The demographic transition view of population growth argues that, on average (and as perceived by parents) the marginal: A) benefits of extra children are larger in IACs than in DVCs. B) costs of extra children are lower in IACs than in DVCs. C) costs of extra children are larger in IACs than in DVCs. D) benefits of extra children are the same in DVCs and IACs. Answer: C

Type: A Topic: 2 E: Web MI: Web MA: Web 45. In the DVCs underemployment frequently takes the form of: A) factory workers who are working longer hours than they would prefer. B) workers who are employed inefficiently in small industry when they could be highly productive in agriculture. C) farmers whose productivity is very low. D) craftsworkers and artisans who are replaced by simple machinery and equipment. Answer: C

Type: A Topic: 2 E: Web MI: Web MA: Web 46. Which of the following is (are) characteristic of DVCs? A) a large percentage of the labor force is in agriculture B) high levels of saving and investment C) high labor productivity D) high levels of training Answer: A

Type: D Topic: 2 E: Web MI: Web MA: Web 47. The "brain drain" problem in the DVCs refers to the fact that the best-educated workers: A) are concentrated in the public, rather than the private, sector. B) are concentrated in the private, rather than the public, sector. C) are concentrated in urban, rather than rural, areas. D) have emigrated from the DVCs to the IACs. Answer: D

McConnell/Brue: Economics, 16/e

Page 1164

Web Chapter 39W: The Economics of Developing Countries

Type: F Topic: 2 E: Web MI: Web MA: Web 48. Capital flight from the DVCs may be is caused by: A) a fear of government privitization efforts. B) slow domestic inflation. Answer: D

C) low rates of domestic taxation. D) risks of severe fluctuations in exchange rates.

Type: F Topic: 2 E: Web MI: Web MA: Web 49. When economists refer to capital flight, they are speaking of an: A) outflow of financial capital or simply money from a certain country. B) outflow of real capital from a certain country. C) outflow of financial capital or simply money from a certain country. D) outflow of real capital from a certain country. Answer: A

Type: F Topic: 2 E: Web MI: Web MA: Web 50. Capital flight is a problem to DVCs because it: A) causes the value of an DVC's currency to appreciate. B) reduces the volume of DVC investment. C) reduces the flow of foreign aid from the IACs. D) causes inflation in the DVCs. Answer: B

Type: D Topic: 2 E: Web MI: Web MA: Web 51. Capital flight refers to: A) the tendency of large corporations of IACs to build new plants in the DVCs because labor is cheaper. B) DVC citizens accumulating or investing their savings in the IACs. C) the high international mobility of speculative funds caused by variations in exchange rates. D) the tendency of DVCs to overinvest in commercial aircraft. Answer: B

Type: A Topic: 2 E: Web MI: Web MA: Web 52. Most of the DVCs find it difficult to accumulate capital goods because: A) the terms of trade prohibit the inflow of private capital from the advanced nations. B) it is very difficult to restrict consumption and thus to free resources for capital goods production. C) domestic monetary policies designed to achieve price stability result in low interest rates, thereby discouraging investment. D) investment is interest inelastic in DVCs. Answer: B

Type: A Topic: 2 E: Web MI: Web MA: Web 53. All of the following contribute to low investment spending in DVCs except: A) low rates of saving. B) inadequacy of public capital goods (infrastructure). C) political instability. D) expensive labor. Answer: D

McConnell/Brue: Economics, 16/e

Page 1165

Web Chapter 39W: The Economics of Developing Countries

Type: A Topic: 2 E: Web MI: Web MA: Web 54. Which of the following is not part of a nation's infrastructure? A) communications facilities C) the electrical power system B) roads, highways, and bridges D) industrial plants and equipment Answer: D

Type: A Topic: 2 E: Web MI: Web MA: Web 55. Infrastructure is best illustrated by: A) farm equipment. B) school buildings and highways. C) machinery and equipment for the production of consumer goods. D) government tax revenues. Answer: B

Type: D Topic: 2 E: Web MI: Web MA: Web 56. Investment in kind refers to the possibility that: A) DVCs will invest for the purpose of becoming less dependent on world markets. B) a DVC will overinvest in industries in which it has a comparative advantage, disrupting its development program. C) newly established manufacturing firms may expand by reinvesting their profits. D) surplus labor in, say, agriculture can be diverted to the production of simple capital goods such as earthen dams. Answer: D

Type: A Topic: 2 E: Web MI: Web MA: Web 57. Suppose that surplus labor in a Pakistani village is used to build a medical clinic and dig several wells. This is an illustration of: A) foreign aid. C) nonfinancial or in-kind investment. B) capital-saving investment. D) technological advance. Answer: C

Type: A Topic: 2 E: Web MI: Web MA: Web 58. Technological change: A) always entails increased investment. B) never entails increased investment. C) may or may not entail increased investment. D) is of little relevance to economic growth in the DVCs. Answer: C

Type: A Topic: 2 E: Web MI: Web MA: Web 59. If a technological advance expands output and requires a smaller investment in capital goods, this technological advance: A) is capital-using. C) is capital-saving. B) must involve nonfinancial investment. D) must pertain to the infrastructure. Answer: C

McConnell/Brue: Economics, 16/e

Page 1166

Web Chapter 39W: The Economics of Developing Countries

Type: A Topic: 2 E: Web MI: Web MA: Web 60. The ability of the DVCs to use the technologies of the IACs is somewhat limited because: A) the IACs have patents on most of their technologies. B) the IACs and the DVCs have much different resource endowments. C) the technologies of the IACs rely heavily on unskilled labor. D) IAC technologies are labor-intensive, while DVC technologies are capital-intensive. Answer: B

Type: A Topic: 2 E: Web MI: Web MA: Web 61. It is sometimes difficult to transfer the advanced technologies of the industrialized nations to the DVCs because: A) the technologies of the IACs are frequently based on relatively expensive labor and relatively cheap capital. B) the resource endowments of the IACs and the DVCs are highly similar. C) international patent laws prohibit such transfers. D) the technologies of the IACs are frequently based on relatively cheap labor and relatively expensive capital. Answer: A

Type: A Topic: 2 E: Web MI: Web MA: Web 62. The low per capita outputs of the DVCs are explained by: A) insufficient saving and investment. C) slow population growth. B) over investment in human capital. D) excessively rapid technological advance. Answer: A

Type: D Topic: 2 E: Web MI: Web MA: Web 63. An economy's infrastructure refers to its: A) land and natural resources. B) financial and banking institutions. C) public capital goods, such as roads, schools, and power facilities. D) surplus supplies of unskilled labor. Answer: C

Type: D Topic: 2 E: Web MI: Web MA: Web 64. A nation's infrastructure refers to: A) its ability to realize economies of scale. B) its stock of technological knowledge. C) public capital goods such as highways and utilities. D) the productivity of its labor force. Answer: C

Type: D Topic: 2 E: Web MI: Web MA: Web 65. The "brain drain" problem in the DVCs refers to the fact the best-educated workers: A) are reluctant to become entrepreneurs. B) are concentrated in rural areas where their skills are underutilized. C) often emigrate to industrialized countries. D) are reluctant to work in the public sector. Answer: C

McConnell/Brue: Economics, 16/e

Page 1167

Web Chapter 39W: The Economics of Developing Countries

Type: A Topic: 2 E: Web MI: Web MA: Web 66. Which of the following describes the vicious circle of poverty? A) Government spending for public goods is inflationary and this undermines incentives to save and invest. B) Higher incomes increase consumption at the expense of capital accumulation, which causes income to fall. C) Low per capita incomes cause low levels of saving and investment, which mean low productivity and therefore low incomes. D) A growing national income increases the demand for money, which increases the interest rate and reduces investment. Answer: C

Type: D Topic: 2 E: Web MI: Web MA: Web 67. Many countries remain poor because they currently are poor. This statement summarizes the: A) infrastructure problem. C) vicious circle of poverty. B) capricious universe view. D) problem of capital flight. Answer: C

Type: D Topic: 2 E: Web MI: Web MA: Web 68. The capricious universe view indicates that: A) in the long run "acts of God" have contained population growth and thus contributed to economic development. B) DVCs will obtain biological and chemical weapons and force the redistribution of world wealth. C) there is little or no correlation between one's efforts and the rewards he or she receives. D) international and civil wars have been the primary impediment to growth. Answer: C

Type: D Topic: 2 E: Web MI: Web MA: Web 69. Economic growth may hinge on whether individuals and institutions within a nation want growth badly enough to change their traditional ways of doing things. This statement refers to: A) in-kind investment. C) the will to develop. B) the capricious universe view. D) human capital investment. Answer: C

Type: D Topic: 2 E: Web MI: Web MA: Web 70. The idea that a person's productive efforts and his or her economic rewards are unrelated: A) is the neocolonialism view of economic development. B) describes the vicious circle of poverty. C) is the surplus labor theory of economic development. D) is the capricious universe view. Answer: D

McConnell/Brue: Economics, 16/e

Page 1168

Web Chapter 39W: The Economics of Developing Countries

Government's role

Type: A Topic: 3 E: Web MI: Web MA: Web 71. The government of a DVC may force the economy to save by deliberately causing inflation. This policy is undesirable because inflation may: A) distort investment away from productive facilities and toward luxury housing and precious metals. B) reduce voluntary saving because the value of money is depreciating. C) contribute to a balance of trade deficit. D) entail all of the above problems. Answer: D

Type: A Topic: 3 E: Web MI: Web MA: Web 72. Which of the following statements is consistent with evidence? A) It is widely accepted that central planning will accelerate a DVC's growth rate. B) It is generally agreed that a laissez-faire policy for government will maximize economic growth. C) The effectiveness of government policies in enhancing growth has varied from country to country. D) Government intervention in DVC economies has universally hampered economic growth. Answer: C

Type: F Topic: 3 E: Web MI: Web MA: Web 73. Which of these sets of countries have the greatest degree of internal corruption, according to Transparency International? A) Bangladesh, Nigeria, and Paraguay C) United States, France, and Italy. B) Finland, Denmark, and New Zealand D) Russia, China, and Sweden. Answer: A

Type: A Topic: 3 E: Web MI: Web MA: Web Status: New 74. A highly active role by government may be needed to promote economic growth in low-income DVCs because of: A) a lack of infrastructure. C) a lack of local entrepreneurship. B) lawlessness in the country. D) all the above reasons. Answer: D

Type: A Topic: 3 E: Web MI: Web MA: Web 75. The government of a DVC may purposely cause inflation because: A) a rapid rate of inflation attracts private foreign capital into a DVC. B) there is conclusive evidence that inflation discourages people from having large families. C) this will strengthen the nation's position in international markets. D) inflation works like taxation in that it may release resources from consumption so that they can be invested. Answer: D

McConnell/Brue: Economics, 16/e

Page 1169

Web Chapter 39W: The Economics of Developing Countries

Aid, trade, and foreign investment

Type: C Topic: 4 E: Web MI: Web MA: Web 76. Successful foreign aid programs: A) enhance a DVC's resources and therefore shift its production possibilities curve to the left. B) enhance a DVC's resources and therefore shift its production possibilities curve to the right. C) move the DVC from a high investment-low consumption position to a low investment-high consumption position on its stable production possibilities curve. D) cause a DVC's exchange rate to depreciate. Answer: B

Type: F Topic: 4 E: Web MI: Web MA: Web Status: New 77. The international agency that lends money to DVCs for economic development projects is the: A) World Bank. C) World Trade Organization (WTO). B) International Monetary Fund (IMF). D) World Credit Union Answer: A

Type: F Topic: 4 E: Web MI: Web MA: Web 78. The World Bank: A) is also known as the International Monetary Fund (IMF). B) lends money to developing nations for basic infrastructure projects such as dams, irrigation, health and sanitation, communications, and transportation. C) is an affiliate of the World Trade Organization (WTO). D) provides subsidies to private firms so they can improve their wages and working conditions. Answer: B

Type: F Topic: 4 E: Web MI: Web MA: Web 79. The World Bank: A) provides military assistance to those nations interested in improving national defense. B) makes and guarantees loans for basic development projects such as the construction of dams, roads, and schools. C) provides gold for DVCs that want to go on the gold standard. D) provides short-term loans to DVCs that are incurring balance of payments deficits. Answer: B

Type: F Topic: 4 E: Web MI: Web MA: Web 80. The primary function of the International Finance Corporation (IFC) is to: A) provide U.S. surplus food to low-income countries. B) provide short-term loans to poor countries to finance international trade deficits. C) make infrastructure loans, that is, loans for highways, schools, communication facilities, and so forth. D) make loans to private enterprises in the DVCs. Answer: D

Type: F Topic: 4 E: Web MI: Web MA: Web 81. In recent years the industrially advanced nations as a group have provided foreign aid amounting to about what percentage of their aggregate outputs? A) .25 percent B) .7 percent C) 1 percent D) 2 percent Answer: A

McConnell/Brue: Economics, 16/e

Page 1170

Web Chapter 39W: The Economics of Developing Countries

Type: F Topic: 4 E: Web MI: Web MA: Web 82. Government-provided foreign aid to developing countries has: A) averaged about 1 percent of the IAC's GDP each year. B) increased sharply since 1995. C) dwarfed the size of foreign direct investment to the DVC in recent years. D) declined substantially since 1995. Answer: D

Type: F Topic: 4 E: Web MI: Web MA: Web 83. In the 1990s: A) direct private investment to the DVCs increased and government-provided foreign aid decreased. B) both direct private investment and government-provided foreign aid to the DVCs increased. C) both direct private investment and government-provided foreign aid to the DVCs increased. D) direct private investment to the DVCs decreased and government-provided foreign aid increased. Answer: A

Type: F Topic: 4 E: Web MI: Web MA: Web Status: New 84. Development assistance as a percentage of GDP is greatest for which of the following industrialized nations? A) U.S. B) Netherlands C) Canada D) Japan Answer: B

Type: F Topic: 4 E: Web MI: Web MA: Web 85. In recent years U.S. foreign aid has been: A) less than 1 percent of its output. B) about 2 percent of its output. Answer: A

C) about 3 percent of its output. D) about 5 percent of its output.

Type: A Topic: 4 E: Web MI: Web MA: Web 86. Foreign aid to the DVCs has been criticized: A) because it may generate economic dependence on the IACs. B) because it encourages the centralization of government power over the economy. C) because government corruption in the DVCs causes aid to be misused. D) for all of the above reasons. Answer: D

Type: A Topic: 4 E: Web MI: Web MA: Web 87. Private capital flows to the DVCs: A) increased substantially in the 1990s. B) have changed in composition in recent years from direct foreign investment toward bank loans. C) now must first be approved by the IMF. D) have dwindled to near zero in the last few years. Answer: A

McConnell/Brue: Economics, 16/e

Page 1171

Web Chapter 39W: The Economics of Developing Countries

Type: F Topic: 4 E: Web MI: Web MA: Web 88. Which of the following is correct? A) Most IAC private capital flows in the 1990s were to African nations. B) IAC capital flows in the 1990s shifted from private enterprises in the DVCs to their governments. C) IAC capital flows in the 1990s were largely to DVCs that instituted macro and micro reforms. D) Generally, in the 1990s DVC nations restructured their economies toward greater governmental direction and involvement. Answer: C

Type: F Topic: 4 E: Web MI: Web MA: Web 89. Most of the private capital flows to DVCs in the 1990s were in the form of: A) interest-free government loans. C) direct foreign investment. B) educational and training assistance. D) bank loans. Answer: C

Type: A Topic: 4 E: Web MI: Web MA: Web 90. An example of direct foreign investment is: A) a U.S. bank granting a loan to a Guatemalan firm. B) General Motors building an auto production facility in China. C) a U.S. government foreign aid grant to Bangladesh. D) The purchase of debt issued by the Panamanian government. Answer: B

Type: A Topic: 4 E: Web MI: Web MA: Web 91. Which of the following is a true statement? A) The DVC debt problem has been totally solved. B) Direct foreign investment to DVCs has dwindled to near zero in recent years. C) Within the last decade, the IMF has had to coordinate financial rescue efforts in Mexico and several countries in Southeast Asia to help them meet their external financial obligations. D) Foreign aid from the IACs to the DVCs has greatly expanded in the past several years. Answer: C

IAC and DVC policies for DVC development

Type: A Topic: 5 E: Web MI: Web MA: Web 92. Which of the following is not a DVC policy that might increase DVC economic growth? A) encouraging direct foreign investment B) opening economics to world trade C) establishing independent central banks D) encouraging emigration of highly skilled workers Answer: D

Type: A Topic: 5 E: Web MI: Web MA: Web 93. Which of the following is not a DVC policy that might increase DVC economic growth? A) privatizing state industries B) controlling population growth C) discouraging direct foreign investment from abroad D) building human capital Answer: C

McConnell/Brue: Economics, 16/e

Page 1172

Web Chapter 39W: The Economics of Developing Countries

Type: A Topic: 5 E: Web MI: Web MA: Web 94. State industries are notoriously poor 'incubators' for the development of profit-focused, entrepreneurial persons who may leave the firm to set up their own businesses. This statement is an argument in favor of: A) free immigration and emigration. C) central planning. B) privatizing state industries. D) free trade. Answer: B

Type: A Topic: 5 E: Web MI: Web MA: Web 95. Economic growth in the DVCs might increase if IACs: A) reduce their tariffs and import quotas. B) encourage more immigration of high-skilled DVC workers. C) outlaw direct private investment abroad by IAC corporations. D) discourage capital flight to the DVCs. Answer: A

Type: A Topic: 5 E: Web MI: Web MA: Web Status: New 96. Large agricultural subsidies for food and fiber in IACs hurt the economies of the DVCs by: A) causing higher prices for imported food products. B) lowering saving rates in the DVCs. C) encouraging "brain drains" from the DVCs. D) reducing world agricultural prices and thus export income of the DVCs. Answer: D

Type: F Topic: 5 E: Web MI: Web MA: Web 97. In recent years, the governments of the IACs have: A) forgiven a portion of the debt owed by some low-income DVCs. B) substantially increased foreign aid to the DVCs. C) substantially reduced their contributions to the World Bank. D) discouraged skilled DVC workers from emigrating to the IACs. Answer: A

Last Word Questions

Type: A E: Web MI: Web MA: Web 98. (Last Word) Famines in sub-Saharan Africa: A) are solely the result of unalterable weather conditions. B) have become less common in recent decades. C) are the result of drought, civil strife, overpopulation, and inappropriate public policies. D) are solely the result of government policies that overprice agricultural products. Answer: C

Type: A E: Web MI: Web MA: Web 99. (Last Word) In many of the sub-Saharan African nations: A) population is growing more rapidly than food production. B) population growth is causing deforestation and diminished land quality. C) governments have favored investment in industry and military spending rather than investment for agriculture. D) all of the above have been true. Answer: D

McConnell/Brue: Economics, 16/e

Page 1173

Web Chapter 39W: The Economics of Developing Countries

True/False Questions

Type: A E: Web MI: Web MA: Web 100. The most important growth obstacle common to all DVCs is the lack of desire to increase their standards of living. Answer: False

Type: A E: Web MI: Web MA: Web 101. Saving is low in many DVCs primarily because income is very equally distributed. Answer: False

Type: F E: Web MI: Web MA: Web 102. The differences in the per capita incomes of the IACs and the DVCs has diminished sharply since the Second World War because of U.S. aid programs. Answer: False

Type: F E: Web MI: Web MA: Web 103. The vast majority of the labor forces of the low-income DVCs are engaged in agriculture. Answer: True

Type: F E: Web MI: Web MA: Web 104. Most of the DVCs of the world are located in Western Europe. Answer: False

Type: F E: Web MI: Web MA: Web 105. DVCs tend to have permanent shortages of farm labor. Answer: False

Type: A E: Web MI: Web MA: Web 106. If the real outputs per capita of a rich nation and a poor nation grow at the same percentage rate, the absolute income gap between the two nations will shrink. Answer: False

Type: D E: Web MI: Web MA: Web 107. Capital flight refers to the fact that many LDCs must use their export earnings to pay interest on their outstanding external debts. Answer: False

Type: A E: Web MI: Web MA: Web 108. Because families can afford to have more children, population growth is greater in the IACs than in the LDCs. Answer: False

McConnell/Brue: Economics, 16/e

Page 1174

Web Chapter 39W: The Economics of Developing Countries

Type: D E: Web MI: Web MA: Web 109. The capricious universe view is the idea that the IACs are exploiting the LDCs. Answer: False

Type: A E: Web MI: Web MA: Web 110. An advantage of direct foreign investment (compared to foreign loans) is that management skill and technological knowledge often accompany such capital flows. Answer: True

Type: A E: Web MI: Web MA: Web 111. DVCs might increase their rates of economic growth through privatization of state industries, encouraging direct foreign investment, controlling population growth, and opening economies to international trade. Answer: True

Type: A E: Web MI: Web MA: Web 112. Reduction of tariff barriers against DVC imports would benefit both the DVCs and the IACs. Answer: True

Type: F E: Web MI: Web MA: Web 113. Most nations of the world are now IACs, not middle- and low-income DVCs. Answer: False

McConnell/Brue: Economics, 16/e

Page 1175

BONUS WEB CHAPTER 40W

Transition Economies: Russia and China
(Found at the book's website, www.mcconnell16.com)

Topic 1. Comparisons: China, Russia, and the United States (Global Perspective) 2. Marxist ideology 3. State ownership and central planning 4. Problems with central planning 5. Collapse of the Soviet economy and the transition to a market system in Russia 6. Market reforms in China Last Word True-False Multiple Choice Questions

Question numbers

____________________________________________________________

_______________________________________

1-4 5-10 11-21 22-28 29-59 60-82 83-84 85-100

____________________________________________________________

_______________________________________

Comparisons: China, Russia, and the United States (Global Perspective)

Type: A Topic: 1 E: Web MI: Web MA: Web 1. In terms of geographical size: A) the United States is larger than Russia. B) China and Russia are of about equal size. Answer: D

C) China is larger than Russia. D) Russia is larger than the United States.

Type: A Topic: 1 E: Web MI: Web MA: Web 2. China has a: A) larger percentage of its labor force in agriculture than does Russia. B) smaller percentage of its labor force in agriculture than does the United States. C) smaller population that Russia. D) larger geographical area than Russia. Answer: A

Type: A Topic: 1 E: Web MI: Web MA: Web 3. Russia has a: A) larger percentage of its labor force in agriculture than does China. B) larger population than the United States. C) longer male life expectancy than the United States. D) larger output per capita than China. Answer: D

Web Chapter 40W: Transition Economies: Russia and China

Type: A Topic: 1 E: Web MI: Web MA: Web 4. Russia has a: A) larger output per capita than the United States. B) smaller population than the United States. C) larger population than China D) smaller geographical size than the United States. Answer: B

Marxist ideology

Type: D Topic: 2 E: Web MI: Web MA: Web 5. According to Marxism, surplus value is: A) the difference between the value of a worker's output and the amount capitalists pay that worker. B) that portion of a nation's national income that is neither consumption nor investment. C) equal to the economy's gross saving. D) the difference between wholesale and retail prices in a capitalist system. Answer: A

Type: D Topic: 2 E: Web MI: Web MA: Web 6. In Marxian ideology surplus value is basically: A) that portion of national output that is invested rather than consumed. B) that portion of a product's economic value that is taxed away by the turnover tax. C) capitalist profits resulting from the exploitation of labor. D) the excess of product price over unit production costs. Answer: C

Type: D Topic: 2 E: Web MI: Web MA: Web 7. The labor theory of value indicates that: A) labor cannot be exploited in a communistic society. B) labor's productivity depends solely on the capital-labor ratio. C) labor is only productive when combined with capital and natural resources. D) the economic value of any good is determined by the amount of labor time required for its production. Answer: D

Type: D Topic: 2 E: Web MI: Web MA: Web 8. The labor theory of value suggests that: A) all economic value derives from the productive efforts of the proletariat or working class. B) consumer prices are market determined, while resource prices are government determined. C) labor is more productive when it is not combined with capital. D) workers are more efficient when they own the enterprises in which they work. Answer: A

Type: D Topic: 2 E: Web MI: Web MA: Web 9. The belief that the economic value of a commodity is determined solely by the amount of labor time required for its production refers to the: A) theory of surplus value. C) marginal utility theory of value. B) marginal productivity theory of value. D) labor theory of value. Answer: D

McConnell/Brue: Economics, 16/e

Page 1178

Web Chapter 40W: Transition Economies: Russia and China

Type: A Topic: 2 E: Web MI: Web MA: Web 10. Before reforms, the Soviet Union based its economy on: A) Marxist-Maoist principles while China based its economy on Maoist-Leninist principles. B) Marxist-Leninist principles while China based its economy on Marxist-Maoist principles. C) Smith-Friedman principles while China based its economy on Malthusian-Keynesian principles. D) Marxist-Leninist principles while China based its economy on Smith-Friedman principles. Answer: B

State ownership and central planning

Type: A Topic: 3 E: Web MI: Web MA: Web 11. Which of the following best describes the institutional characteristics of the Soviet Union and prereform China? A) private ownership of the means of production combined with the allocation of resources through free market prices B) public ownership of the means of production combined with central economic planning C) public ownership of the means of production combined with the allocation of resources through free market prices D) private ownership of the means of production combined with central economic planning Answer: B

Type: A Topic: 3 E: Web MI: Web MA: Web 12. Historically, Soviet economic planning stressed: A) the rapid expansion of trade with Western Europe and the United States. B) rapid industrial growth and military strength. C) the full development of an efficient agricultural sector. D) consumer welfare as reflected in durable goods production. Answer: B

Type: A Topic: 3 E: Web MI: Web MA: Web 13. Which of the following was not a characteristic or primary goal of Soviet and Chinese central planning? A) military strength C) consumer abundance B) economic self-sufficiency D) the development of heavy industry Answer: C

Type: A Topic: 3 E: Web MI: Web MA: Web 14. In the Soviet Union and prereform China: A) both consumer and capital goods outputs were determined by central planning. B) both consumer and capital goods outputs were determined by the market. C) capital goods output was determined by planning, but consumer goods output was determined by the market. D) consumer goods output was determined by planning, but capital goods output was determined by the market. Answer: A

McConnell/Brue: Economics, 16/e

Page 1179

Web Chapter 40W: Transition Economies: Russia and China

Type: A Topic: 3 E: Web MI: Web MA: Web 15. Soviet and prereform Chinese industrial enterprises historically were: A) self-financing corporations. B) government-owned monopolies. C) employee-owned state businesses. D) privately owned, but governmentally managed firms. Answer: B

Type: F Topic: 3 E: Web MI: Web MA: Web 16. The major success indicator under communist central planning was: A) enterprise profitability. B) quantity of output. C) quality of output. Answer: B

D) net exports.

Type: A Topic: 3 E: Web MI: Web MA: Web 17. Soviet growth in the 1950s and 1960s was achieved primarily by: A) increasing the productive efficiency of inputs. B) using more inputs. C) encouraging foreign private investment. D) shifting resources from the military sector to the civilian sector. Answer: B

Type: A Topic: 3 E: Web MI: Web MA: Web 18. In the Soviet Union and prereform China most of the: A) capital stock was owned by private corporations. B) capital stock was owned by government. C) land was owned by individuals. D) labor was employed by private corporations. Answer: B

Type: A Topic: 3 E: Web MI: Web MA: Web 19. Central planners in the Soviet Union and prereform China stressed national self-sufficiency because: A) both countries had all the resources needed to produce their outputs at lowest average total cost. B) their populations preferred the high-quality of domestic-made goods to the low-quality of imported goods. C) government leaders viewed capitalist countries as a political, economic, and military threat. D) they were geographically isolated from other countries. Answer: C

Type: A Topic: 3 E: Web MI: Web MA: Web 20. In the early history of communist Soviet Union and China, most of the economic growth was the result of: A) increased productivity of the labor force. C) increased employment of inputs. B) increased allocative efficiency. D) improved technology. Answer: C

McConnell/Brue: Economics, 16/e

Page 1180

Web Chapter 40W: Transition Economies: Russia and China

Type: A Topic: 3 E: Web MI: Web MA: Web 21. A major economic implication of the emphasis on economic self-sufficiency in the centrally planned economies of the Soviet Union and pre-reform China was that: A) their price levels were lower than they would have been otherwise. B) unemployment was greater than it would have been otherwise. C) potential income gains based on free trade were sacrificed. D) technological advance was greater than it would have been otherwise. Answer: C

Problems with central planning

Type: A Topic: 4 E: Web MI: Web MA: Web 22. The coordination problem in the centrally planned economies refers to the idea that: A) planners had to direct required inputs to each enterprise. B) the price level and the level of employment were inversely related. C) the immediate effect of more investment was less consumption. D) exports had to be equal to imports for a central plan to work. Answer: A

Type: A Topic: 4 E: Web MI: Web MA: Web 23. "Under central planning, some group has to decide how to get the necessary inputs produced in the right amounts and delivered to the right places at the right time. This is a nearly impossible task without markets and profits." This quotation best identifies the: A) incentive problem under central planning. B) coordination problem under central planning. C) self-sufficiency dilemma under communism. D) resource overcommitment problem under communism. Answer: B

Type: A Topic: 4 E: Web MI: Web MA: Web 24. "Because the outputs of many industries are the inputs to other industries, the failure of any single industry to fulfill the output quantities specified in the central plan caused a chain-reaction of adverse repercussions on production." This quotation best identifies the: A) incentive problem under central planning. B) self-sufficiency dilemma under communism. C) resource overcommitment problem under communism. D) coordination problem under central planning. Answer: D

Type: A Topic: 4 E: Web MI: Web MA: Web 25. The incentive problem under communist central planning refers to the idea that: A) planners had to direct required inputs to each enterprise. B) workers, managers, and entrepreneurs could not personally gain by responding to shortages or surpluses or by introducing new and improved products. C) the immediate effect of more investment was less consumption. D) exports had to be equal to imports for a central plan to work. Answer: B

McConnell/Brue: Economics, 16/e

Page 1181

Web Chapter 40W: Transition Economies: Russia and China

Type: A Topic: 4 E: Web MI: Web MA: Web 26. Suppose that an individual sees a tremendous opportunity to produce and sell a new product, but dismisses the idea because there is no way to exploit this opportunity for personal gain. This situation best identifies the: A) coordination problem under communist central planning. B) self-sufficiency dilemma under communism. C) asymmetric information problem under communism. D) incentive problem under communist central planning. Answer: D

Type: A Topic: 4 E: Web MI: Web MA: Web 27. Shortages and unmet demand provide opportunities for individuals and firms to profit under capitalism, but they present no such opportunities under central planning. This reality represents central planning's: A) incentive problem. B) coordination problem. C) paradox of value. D) X-efficiency problem. Answer: A

Type: A Topic: 4 E: Web MI: Web MA: Web 28. Innovation lagged in the centrally planned economies because: A) there was too much domestic business competition. B) there was too much competition from foreign firms. C) enterprises resisted innovation in fear that their production targets would be raised. D) exports had to equal imports for the plan to work. Answer: C

Collapse of the Soviet economy and the transition to a market system in Russia

Type: A Topic: 5 E: Web MI: Web MA: Web 29. The fact that the major indicator of enterprise success in the Soviet Union and prereform China was the quantity of output implied that: A) product quality was neglected. C) product-mix met consumer needs. B) production costs were minimized. D) technological advance was too rapid. Answer: A

Type: A Topic: 5 E: Web MI: Web MA: Web 30. Workers in the Soviet Union and prereform China were not motivated to work hard because: A) the Communist Party set strict limits on production rates. B) higher money incomes did not translate into more consumer goods. C) the income tax was confiscatory. D) the prices of consumer goods were market determined while wages were government determined. Answer: B

Type: A Topic: 5 E: Web MI: Web MA: Web 31. Enterprise managers and workers in the Soviet Union often resisted innovations in production methods because: A) production targets were often increased when innovation occurred. B) there was a chronic shortage of computers. C) workers could not be reallocated geographically. D) innovations ordinarily increased dependence on world markets. Answer: A

McConnell/Brue: Economics, 16/e

Page 1182

Web Chapter 40W: Transition Economies: Russia and China

Type: A Topic: 5 E: Web MI: Web MA: Web 32. The amount of resources allocated to the military in the Soviet Union was: A) the same both absolutely and relative to domestic output as in the United States. B) larger relative to the domestic output than in the United States. C) smaller relative to the domestic output than in the United States. D) smaller both absolutely and relative to the domestic output than in the United States. Answer: B

Type: A Topic: 5 E: Web MI: Web MA: Web 33. The failure of Soviet central planning was reflected in: A) a declining growth rate. B) poor quality goods. C) the failure to provide promised consumer goods. D) all of the above. Answer: D

Type: A Topic: 5 E: Web MI: Web MA: Web 34. Government price fixing for basic consumer goods in the Soviet Union established prices: A) at the equilibrium level. B) below the equilibrium level. C) above the equilibrium level. D) which were closely related to the relative scarcity of each good. Answer: B

Type: A Topic: 5 E: Web MI: Web MA: Web 35. If products were in short or surplus supply in the Soviet Union: A) price and profit signals eliminated those shortages and surpluses. B) price and profit signals intensified those shortages and surpluses. C) producers would not react because no price or profit signals occurred. D) the planners would immediately adjust produciton to achieve equilibrium. Answer: C

Type: A Topic: 5 E: Web MI: Web MA: Web 36. Soviet government pricing of consumer goods was typically at: A) equilibrium prices so that markets cleared. B) below equilibrium prices, which caused shortages. C) above equilibrium prices, which caused surpluses. D) above equilibrium prices, which caused shortages. Answer: B

McConnell/Brue: Economics, 16/e

Page 1183

Web Chapter 40W: Transition Economies: Russia and China

Use the following to answer questions 37-39:

Type: G Topic: 5 E: Web MI: Web MA: Web 37. Refer to the above diagram where Pf is the government-determined price. At this price there will be: A) a shortage of 0Q1. C) neither a shortage nor a surplus. B) a surplus of 0Q1. D) a shortage of Q1Q2. Answer: D

Type: G Topic: 5 E: Web MI: Web MA: Web 38. Refer to the above diagram where Pf is the government-determined price. This price is: A) above the equilibrium price. C) equal to the equilibrium price. B) below the equilibrium price. D) a cause of inflation. Answer: B

Type: A Topic: 5 E: Web MI: Web MA: Web 39. Refer to the above diagram where Pf is the government-determined price. If there is no privatization and this diagram is representative of all Soviet consumer goods markets, then the decontrol of prices will: A) cause inflation. C) eliminate the product shortage. B) cause the market to clear. D) do all of the above. Answer: D

Type: D Topic: 5 E: Web MI: Web MA: Web 40. As it relates to reforms in Russia privatization refers to: A) Soviet consumer cooperatives. C) the sale of public property to private buyers. B) the decontrol of prices. D) the demonopolization of Soviet industry. Answer: C

Type: D Topic: 5 E: Web MI: Web MA: Web 41. Convertibility of the ruble means that: A) the ruble is exchangeable for other foreign currencies. B) the government might issue low-denomination rubles for high-denomination rubles to control inflation. C) international trade with Russia must occur in the form of barter. D) the Russian monetary authorities have fixed the value of the ruble to the value of the U.S. dollar. Answer: A

McConnell/Brue: Economics, 16/e

Page 1184

Web Chapter 40W: Transition Economies: Russia and China

Type: A Topic: 5 E: Web MI: Web MA: Web 42. Opening the Russian economy to world trade: A) has increased competition in the Russian economy. B) has resulted in income gains because of trade based on comparative advantage. C) required that the ruble be made convertible. D) has involved all of the above. Answer: D

Type: A Topic: 5 E: Web MI: Web MA: Web 43. An important implication of the ruble overhang in Russia was that: A) its existence prevented ruble convertibility. B) it caused inflation when prices were decontrolled. C) the saving rate was too low in Russia to permit rapid economic growth. D) it was a source of funds for financing Russian exports. Answer: B

Type: D Topic: 5 E: Web MI: Web MA: Web 44. The ruble overhang of the early 1990s consisted of: A) the excess reserves of the Russian banking system. B) large accumulations of rubles as forced or involuntary saving. C) the low interest rates that inhibited Russian investment. D) an excess of rubles supplied relative to rubles demanded in foreign exchange markets. Answer: B

Type: A Topic: 5 E: Web MI: Web MA: Web 45. The Russian transition from a planned to a market economy has resulted in: A) the privatization of public property. C) the removal of price controls. B) enhanced business competition. D) all of the above. Answer: D

Type: A Topic: 5 E: Web MI: Web MA: Web 46. The initial transition from government-determined to market-determined prices in Russia caused: A) inflation. B) deflation. C) price level stability. D) a surge of exports from Russia. Answer: A

Type: A Topic: 5 E: Web MI: Web MA: Web 47. Inflation in Russia during its transition to capitalism was caused by: A) the decontrol of prices. B) spending of the ruble overhang. C) government budget deficits financed by money creation. D) all of the above. Answer: D

McConnell/Brue: Economics, 16/e

Page 1185

Web Chapter 40W: Transition Economies: Russia and China

Type: F Topic: 5 E: Web MI: Web MA: Web 48. Which of the following is correct? A) All prices remain governmentally determined in Russia. B) About 90 percent of all small companies in Russia are now privatized. C) About 33 percent of prices in Russia have been decontrolled. D) Virtually all firms in Russia are still state-owned enterprises. Answer: B

Type: A Topic: 5 E: Web MI: Web MA: Web 49. A major problem in Russia's early economic transition to capitalism was: A) massive unemployment. C) declines in real output. B) a very slow rate of privatization. D) the failure to decontrol prices. Answer: C

Type: F Topic: 5 E: Web MI: Web MA: Web Status: New 50. Between 1992 and 1996, real GDP in Russia: A) declined by magnitudes equivalent to the Great Depression (1930s) in the United States. B) increased by an average of 6 percent a year. C) declined in nominal terms but increased in real terms due to deflation. D) rose more rapidly than the rate of inflation. Answer: A

Type: F Topic: 5 E: Web MI: Web MA: Web Status: New 51. From 1992 through 1996, Russia had: A) declining income inequality. C) rising GDP in each year. B) rising annual rates of inflation. D) falling GDP in each year. Answer: D

Type: F Topic: 5 E: Web MI: Web MA: Web Status: New 52. In 1998 Russia changed its currency by: A) replacing the ruble with the Russia wren. B) replacing the ruble with the European euro. C) eliminating all coins. D) removed three zeros, such that a 1,000-ruble bill became a 1-ruble bill. Answer: D

Type: F Topic: 5 E: Web MI: Web MA: Web Status: New 53. The current (2003) dollar-ruble exchange rate is about: A) $1 = 6000 rubles. B) $1 = 6 rubles. C) $1 = 30 rubles. D) $1 = 80 rubles. Answer: C

Type: F Topic: 5 E: Web MI: Web MA: Web Status: New 54. By removing three zeros from its currency, in 1998 the Russian government immediately: A) ended rapid inflation. C) altered the nominal dollar-ruble exchange rate. B) reduced income inequality. D) reduced the purchasing power of each ruble. Answer: C

McConnell/Brue: Economics, 16/e

Page 1186

Web Chapter 40W: Transition Economies: Russia and China

Type: F Topic: 5 E: Web MI: Web MA: Web Status: New 55. The current (2003) annual rate of inflation in Russia is about: A) zero percent. B) 20 percent. C) 2-3 percent. D) 100 percent. Answer: B

Type: F Topic: 5 E: Web MI: Web MA: Web Status: New 56. Between 1999 and 2002, real GDP in Russia increased by an average annual rate of: A) 6 percent. B) 4 percent. C) 2 percent. D) zero percent. Answer: A

Type: F Topic: 5 E: Web MI: Web MA: Web Status: New 57. Over recent years in Russia: A) large budget deficits have given way to large budget surpluses. B) the distribution of income has become considerably more equal. C) political and economic corruption has largely ended. D) inflation has given way to fears of deflation. Answer: A

Type: F Topic: 5 E: Web MI: Web MA: Web Status: New 58. The recent swing from large budget deficits to large budget surpluses in Russia has resulted from: A) rising rates of inflation. B) rising international oil prices and increased real GDP. C) large tax cuts. D) higher tariffs on imported goods. Answer: B

Type: F Topic: 5 E: Web MI: Web MA: Web Status: New 59. Which of the following statements in true? A) Russia continues to suffer from annual declines is real GDP. B) At least 30,000 scientists have left Russia to work in other nations. C) Military spending in Russia is higher today as a percentage of GDP than it was during the Soviet era. D) Russia continues to suffer from huge annual budget deficits. Answer: B

Market reforms in China

Type: F Topic: 6 E: Web MI: Web MA: Web 60. Market reform in China: A) has been approved and directed by the Communist Party. B) has involved so-called radical "shock therapy." C) occurred first in manufacturing and then in agriculture. D) was led by Mao Zedong, the successor to Deng Xiaoping. Answer: A

McConnell/Brue: Economics, 16/e

Page 1187

Web Chapter 40W: Transition Economies: Russia and China

Type: F Topic: 6 E: Web MI: Web MA: Web 61. Market reform in China: A) has involved so-called radical "shock therapy." B) occurred first in agriculture and then in manufacturing. C) began after the Russian reforms of 1992. D) began in 1978 with a selling off of all state-owned enterprises. Answer: B

Type: A Topic: 6 E: Web MI: Web MA: Web 62. The key elements of the 1978-1984 reform in China were: A) privatization of state-owned enterprises and the demise of the Communist Party. B) leasing of land to individual farmers and price reform in agriculture. C) elimination of urban collectives and freeing of industrial prices. D) corporatization of the state-owned enterprises and the establishment of special enterprise zones. Answer: B

Type: D Topic: 6 E: Web MI: Web MA: Web 63. Nonstate manufacturing firms in rural China are known as: A) township and village enterprises. C) urban collectives. B) limited liability companies. D) joint stock companies. Answer: A

Type: D Topic: 6 E: Web MI: Web MA: Web 64. Urban collectives in China are: A) nonstate enterprises owned jointly by managers and their workforces. B) state-owned and operated enterprises. C) also known as township and village enterprises. D) mainly owned by foreign financial investors. Answer: A

Type: A Topic: 6 E: Web MI: Web MA: Web 65. Township and village enterprises and urban collectives in China are: A) directed through the central plan. C) types of foreign-owned enterprises. B) types of state-owned enterprises. D) types of private (nonstate) enterprises. Answer: D

Type: A Topic: 6 E: Web MI: Web MA: Web 66. The early years of market reform in China were characterized by a: A) rapid decline in agricultural production. B) rapid decline in industrial production. C) two-track price system in agriculture D) "corporatization" of the state-owned enterprises. Answer: C

McConnell/Brue: Economics, 16/e

Page 1188

Web Chapter 40W: Transition Economies: Russia and China

Type: A Topic: 6 E: Web MI: Web MA: Web 67. The Chinese reform that allowed portions of output to be sold on the market at market prices: A) was allowed only in special enterprise zones. B) reduced agricultural output but increased industrial output. C) occurred first in manufacturing and then in agriculture. D) was known as a two-track price system. Answer: D

Type: A Topic: 6 E: Web MI: Web MA: Web 68. The special economic zones in China: A) resulted in "near capitalism" in these regions. B) were established mainly in rural inland regions. C) were first established by Jiang Zemin after Deng Xiaoping's death in 1996. D) were largely based on Marxian principles. Answer: A

Type: A Topic: 6 E: Web MI: Web MA: Web 69. In recent years China market reforms have turned toward: A) increasing the numbers of more township and village enterprises. B) establishing more urban collectives. C) transforming state-owned enterprises into shareholder-owned corporations. D) reestablishing central planning. Answer: C

Type: A Topic: 6 E: Web MI: Web MA: Web 70. Under recent reforms, the 1000 largest state-owned enterprises in China will eventually: A) be converted into shareholder-owned corporations. B) no longer be allowed to operate in the special enterprise zones. C) broken into 10,000 competing firms. D) be allowed to engage in international trade. Answer: A

Type: A Topic: 6 E: Web MI: Web MA: Web 71. Since 1991, China's annual growth of real GDP as been: A) negative. B) about the same as that of Russia. C) substantially greater than that of the United States. D) about the same as that of the United States. Answer: C

Type: A Topic: 6 E: Web MI: Web MA: Web 72. China's high rate of economic growth over the past decade has been accompanied by: A) triple digit rates of inflation. C) significant declines in direct foreign investment. B) rapid increases in exports. D) substantial decline in income inequality. Answer: B

McConnell/Brue: Economics, 16/e

Page 1189

Web Chapter 40W: Transition Economies: Russia and China

Type: A Topic: 6 E: Web MI: Web MA: Web 73. China's per capita purchasing power is: A) about $12,000 per year. B) greater than that of Russia. C) substantially greater than its per capita GDP (based on exchange rates). D) about $840 annually, whereas its per capita GDP (based on exchange rates) is about one-half that amount. Answer: C

Type: F Topic: 6 E: Web MI: Web MA: Web 74. China's per capita GDP (based on exchange rates) is ____, whereas its per capita purchasing power is ____. A) $4000; $580. B) $890; $4,260. C) $6,000; $1,000. D) $1,520; $6,080. Answer: B

Type: F Topic: 6 E: Web MI: Web MA: Web 75. Since the beginning of its market reforms in 1978, China's annual economic growth rate has averaged about: A) 3 percent. B) 5 percent. C) 9 percent. D) 12 percent. Answer: C

Type: A Topic: 6 E: Web MI: Web MA: Web 76. China's: A) exports have declined substantially since the beginning of its market reforms. B) real GDP has declined slightly since the beginning of its market reforms. C) economic success has not been evenly achieved throughout the country. D) inflation rate was less than 5 percent each year between 1991 and 2002. Answer: C

Type: A Topic: 6 E: Web MI: Web MA: Web 77. Which of the following is not a current economic problem facing China? A) slow growth of real GDP and real GDP per capita B) incomplete property rights which may be slowing productivity growth in agriculture C) the potential for defaults by SOEs on large bank loans from Chinese banks D) geographically uneven economic development Answer: A

Type: F Topic: 6 E: Web MI: Web MA: Web Status: New 78. Which of the following statements is false? A) China is a member of the United Nations. B) China has dissolved its Communist Party. C) China has experienced a rapid growth of exports in recent years. D) China is a member of the World Trade Organization. Answer: B

Type: A Topic: 6 E: Web MI: Web MA: Web 79. China's economic reforms have: A) been gradual compared to Russia's. B) put democratization before economic reform. Answer: A

C) privatized all state-owned enterprises. D) slowed considerably in recent years.

McConnell/Brue: Economics, 16/e

Page 1190

Web Chapter 40W: Transition Economies: Russia and China

Type: A Topic: 6 E: Web MI: Web MA: Web 80. China's special economic zones are designed to: A) reduce China's international trade surplus. B) attract foreign companies along with their advanced technologies and business expertise. C) raise productivity in Chinese agriculture. D) subsidize China's state-owned enterprises. Answer: B

Type: A Topic: 6 E: Web MI: Web MA: Web 81. A possible problem in sustaining future Chinese economic growth is: A) the ineffectiveness of the special economic zones. B) low rates of saving. C) rising sentiments for a return to central planning. D) a conflict between greater economic freedom and a totalitarian government. Answer: D

Type: A Topic: 6 E: Web MI: Web MA: Web 82. In recent years, China's economy has experienced: A) very low rates of inflation. B) recession. Answer: A

C) slight declines in real GDP per capita. D) declining exports.

Last Word Questions

Type: A E: Web MI: Web MA: Web 83. (Last Word) The main point demonstrated by the article "Police Smash Down Smirnov Doors" is that: A) alcoholism remains a major problem in Russia. B) property ownership in Russia is not yet clearly defined. C) many Russians are desperate to receive the back pay owed them by their employers. D) state-owned enterprises still dominate in Russia. Answer: B

Type: F E: Web MI: Web MA: Web Status: New 84. (Last Word) The news report about "Smirnov" vodka is relevant to the economic concepts of: A) property rights and trademarks. C) alcoholism and corruption. B) public goods and spillover benefits. D) elasticity of demand and pricing policies. Answer: A

True/False Questions

Type: A E: Web MI: Web MA: Web 85. The labor theory of value is the idea that labor gains most of its value through the use of capital. Answer: False

Type: A E: Web MI: Web MA: Web 86. Surplus value is the Marxian idea that all economic value derives from the productive effort of workers. Answer: False

McConnell/Brue: Economics, 16/e

Page 1191

Web Chapter 40W: Transition Economies: Russia and China

Type: A E: Web MI: Web MA: Web 87. Central planning in the Soviet Union and prereform China emphasized the expansion of the production of consumer goods to raise the domestic standard of living. Answer: False

Type: A E: Web MI: Web MA: Web 88. The Soviet Union and prereform China typically set consumer prices at below-equilibrium levels that resulted in product shortages. Answer: True

Type: A E: Web MI: Web MA: Web 89. Central planning is plagued with a coordination problem and an incentive problem. Answer: True

Type: F E: Web MI: Web MA: Web 90. Most prices in Russia have been decontrolled. Answer: True

Type: F E: Web MI: Web MA: Web 91. Most firms in Russia are now privately owned and operated. Answer: True

Type: F E: Web MI: Web MA: Web 92. The Russian transition to capitalism produced almost immediate increases in economic growth. Answer: False

Type: F E: Web MI: Web MA: Web 93. Market reform in China began in agriculture. Answer: True

Type: A E: Web MI: Web MA: Web 94. China's special economic zones resulted in "near-capitalism" in these regions and their success undermined support for central planning elsewhere in China. Answer: True

Type: F E: Web MI: Web MA: Web 95. China's growth rate has average 3 percent annually since reforms began in 1978. Answer: False

Type: F E: Web MI: Web MA: Web 96. China's economic development has been highly uneven geographically. Answer: True

McConnell/Brue: Economics, 16/e

Page 1192

Web Chapter 40W: Transition Economies: Russia and China

Type: F E: Web MI: Web MA: Web Status: New 97. Unemployment is a major problem in the interior regions of China. Answer: True

Type: F E: Web MI: Web MA: Web Status: New 98. China has greatly expanded its exports in recent years. Answer: True

Type: F E: Web MI: Web MA: Web Status: New 99. Russia is member of the World Trade Organization (WTO) but China is not a member. Answer: False

Type: F E: Web MI: Web MA: Web Status: New 100. China's standard of living is much higher when measured in terms of equivalent dollars of purchasing power than when measured on the basis of dollar exchange rates. Answer: True

McConnell/Brue: Economics, 16/e

Page 1193

You May Also Find These Documents Helpful

Related Topics